Sie sind auf Seite 1von 619

Self Assessment & Review

Pharmacology
4th
Edition

Hira Bhalla
KGMU Lcknow

Yogesh Gulati
KGMU Lcknow

Deepak Mishra
BRDMC Gorakhpur

the arora medical book publishers pvt. ltd.


A Group of Jaypee Brothers Medical Publishers (P) Ltd.
Published by
the arora medical book publishers pvt. ltd.
A Group of Jaypee Brothers Medical Publishers (P) Ltd
Corporate Office
4838/24 Ansari Road, Daryaganj, New Delhi - 110002, India, Phone: +91-11-43574357
Registered Office
B-3 EMCA House, 23/23B Ansari Road, Daryaganj, New Delhi - 110 002, India
Phones: +91-11-23272143, +91-11-23272703, +91-11-23282021, +91-11-23245672
Rel: +91-11-32558559, Fax: +91-11-23276490, +91-11-23245683
e-mail: jaypee@jaypeebrothers.com, Website: www.jaypeebrothers.com
Branches
q 2/B, Akruti Society, Jodhpur Gam Road Satellite
Ahmedabad 380 015, Phones: +91-79-26926233, Rel: +91-79-32988717
Fax: +91-79-26927094, e-mail: ahmedabad@jaypeebrothers.com
q 202 Batavia Chambers, 8 Kumara Krupa Road, Kumara Park East
Bengaluru 560 001, Phones: +91-80-22285971, +91-80-22382956
91-80-22372664, Rel: +91-80-32714073, Fax: +91-80-22281761
e-mail: bangalore@jaypeebrothers.com
q 282 IIIrd Floor, Khaleel Shirazi Estate, Fountain Plaza, Pantheon Road
Chennai 600 008, Phones: +91-44-28193265, +91-44-28194897
Rel: +91-44-32972089, Fax: +91-44-28193231, e-mail: chennai@jaypeebrothers.com
q 4-2-1067/1-3, 1st Floor, Balaji Building, Ramkote Cross Road,
Hyderabad 500 095, Phones: +91-40-66610020, +91-40-24758498
Rel:+91-40-32940929Fax:+91-40-24758499, e-mail: hyderabad@jaypeebrothers.com
q No. 41/3098, B & B1, Kuruvi Building, St. Vincent Road
Kochi 682 018, Kerala, Phones: +91-484-4036109, +91-484-2395739
+91-484-2395740, e-mail: kochi@jaypeebrothers.com
q 1-A Indian Mirror Street, Wellington Square
Kolkata 700 013, Phones: +91-33-22651926, +91-33-22276404
+91-33-22276415, Rel: +91-33-32901926, Fax: +91-33-22656075
e-mail: kolkata@jaypeebrothers.com
q Lekhraj Market III, B-2, Sector-4, Faizabad Road, Indira Nagar
Lucknow 226 016, Phones: +91-522-3040553, +91-522-3040554
e-mail: lucknow@jaypeebrothers.com
q 106 Amit Industrial Estate, 61 Dr SS Rao Road, Near MGM Hospital, Parel
Mumbai 400 012, Phones: +91-22-24124863, +91-22-24104532,
Rel: +91-22-32926896, Fax: +91-22-24160828, e-mail: mumbai@jaypeebrothers.com
q “KAMALPUSHPA” 38, Reshimbag, Opp. Mohota Science College, Umred Road
Nagpur 440 009 (MS), Phone: Rel: +91-712-3245220, Fax: +91-712-2704275
e-mail: nagpur@jaypeebrothers.com
USA Office
1745, Pheasant Run Drive, Maryland Heights (Missouri), MO 63043, USA, Ph: 001-636-6279734
e-mail: jaypee@jaypeebrothers.com, anjulav@jaypeebrothers.com
Self Assessment & Review Pharmacology
© 2008, Jaypee Brothers Medical Publishers
All rights reserved. No part of this publication should be reproduced, stored in a retrieval system, or transmitted in any form or by any
means: electronic, mechanical, photocopying, recording, or otherwise, without the prior written permission of the authors and the
publisher.

This book has been published in good faith that the material provided by authors is original. Every effort is made to ensure accuracy of
material, but the publisher, printer and author will not be held responsible for any inadvertent error(s). In case of any dispute, all legal
matters are to be settled under Delhi jurisdiction only.

Fourth Edition: 2008


ISBN 978-81-8448-487-8
Typeset at JPBMP typesetting unit
Printed at
ii
Dedicated
to our respected
Parents
and above all
God
for showing us the right path

iii
Special thanks to...

Sarvesh Singh
MBBS, LLRM Medical College, Meerut
MD, Pharmacology, KGMU, Lucknow

Chandresh Soni
DMRD, LHMC, New Delhi

Mini Bhalla
BDS, CDC, Ludhiana

Arvind Kumar
MBBS, BRD Medical College, Gorakhpur
iv MD (Paediatrics, Std.) KGMU, Lucknow
Preface

“Success begins with the fellow’s will, its all in the state of mind,
Life’s battles don’t always go to the stronger or faster man,
but sooner or later the man who wins is the man who thinks he can.”

Preparing for PGMEs could never be an easy job, be it 20 years back or the ones you are preparing for. More so, for the
subjects that are continuously evolving and with the new information being incorporated in the books with each
coming edition, you are bound to have new questions being popped up in every examination.

Pharmacology is one such subject. It is evolving at such a fast pace, that even those who are engaged twenty four-
seven with it are unable to memorize each and every new information.

The present book aims at making it easy for PG aspirants to keep hold of the new knowledge, at the same time
focusing equally on the basics and yet providing with the comprehensive view of whole question bank, so as to enable
the reader to come up with correct answers, most of the times.

In this book, we have used a lot of flow-charts, illustrations and comparison tables to present the facts which are
needed to be learned, in an easy to assimilate and easy to remember way.

While no stones have been left unturned to make this book error-free, we sincerely apologize for any mistake that
may have escaped our notice (specially the grammatical ones) and request our readers for feedback, so as to enable us
to improve the text further.

We are grateful to our colleagues, readers and students all over the country for their valuable suggestions and
constructive criticisms. By your support and enthusiasm, the book in its very first edition has gained popularity
amongst the best seller books of the country.

We are looking forward to you valuable feedback and suggestions at – yogesh1706@yahoo.com,


dmishra12@rediffmail.com and hira_bhalla@yahoo.com

“The harder you work, the luckier you get”


All the best

Yogesh Gulati Hira Bhalla Deepak Mishra


(MD, KGMU, Lucknow) (MD, KGMU, Lucknow) (BRDMC, Gorakhpur)

October, 2008

v
Acknowledgement
To the ALMIGHTY CREATOR, for the aspirations and inspirations.
We will like to thank all teachers of Department of Pharmacology & Therapeutics, KGMU; Subharti medical college
and teachers of BRDMC, Gorakhpur for their blessings and support.
We would like to thank Dr. M. M. Singh (CDRI, Lucknow); Dr. V K Srivastava (Prof HOD, Subharti medical college,
Meerut); Dr Surabhi Gupta (Associate. Prof. Subharti medical college, Meerut); ); Dr. Prof. M.K. Singh (HOD, RIO, IGIMS,
Patna); Dr. B.P. Sinha (Asst. Prof.); Dr. R. K. Rai (Asst. Prof.); Dr. Sarbjeet Kaur (Deptt. of Community Medicine, IHBAS,
Delhi); Dr. Ashok Maharana (M.D., Pharmacology) for their blessings and guidance and Dr. Rakesh Kumar, Dr. Asif,
Dr. Anita, Dr. Neha of RIO, IGIMS, Patna for their unconditional support and has been a source of inspiration for me
to hard work.
Each one in my family has a special place in my life; I like to thank them all from the deepest corner of my heart.
Parents Mr. S. C. Gulati & Mrs. Manju Gulati
Grandfather Late Mr. K. L. Gulati
Grandmother Mrs. S. V. Gulati
Uncle & Aunty Late Dr. (Prof.) M. S. Agnihotri & Mrs. Raj Agnihotri
Maternal Grandfather Mr. S. L. Sahni
Wife Mrs. Richa Gulati
Daughter Baby Ritisha Gulati
Brother Mr. Sumit Gulati
Brother’s wife Mrs. Jyoti Gulati
Father in law M r. R. K. Khare
Mother In law Mrs. Manju Khare
Brother in law Mr. Saurabh Khare
Brother in law’s wife Mrs. Tanya Khare
– Yogesh Gulati
I would like to say that what I am is because of your love, sacrifice, encouragement and support.
Parents Dr. O. P. Mishra & Mrs. G. Mishra
Grandfather Mr. L. B. Mishra, Mr. R. R. Mishra
Grandmother Late Mrs. K. Mishra, Late Mrs. H. Mishra
Uncle Mr. V. P. Mishra, Mr. J. P. Mishra
Brother Dr. Himanshu Mishra
– Deepak Mishra
My expressions and emotions fail to find words when I think of my family members for their unconditional support
and has been a source of inspiration for me to hard work.
Parents Late Mr. P. N. Bhalla & Mrs. Bimla Bhalla
Brother & Bhabhi Mr. Sudhir Bhalla & Mrs. Kanchan Bhalla
Mr. Tarun Bhalla & Mrs. Anju Bhalla
Sister & Jijaji Dr. Mrs. Renu Kapoor & Dr. Uday Kapoor
Nephew Shorya, Shrestha, Shrey
Neice Palki, Samiara Kapoor
Wife Dr. Mini Bhalla
Father in law M r. Surinder Kumar Gakhar
Mother In law Mrs. Sushma Gakhar
Brother in law Mr. Raghuvir Gakhar
– Hira Bhalla
vi
We would like to thanks Mr. Karan Malik & Mrs. Meghna Malik for their blessings and guidance and Mr. S C Gulati
(Ex Executive Director, HAL) for their unconditional support and has been a source of inspiration for me to hard work.
We will always remain grateful to you all for your moral support and encouragement –

Dr. Ajay Shukla (MBBS, BRDMC, Gorakhpur) Dr. Faisal Ameer (MS, MCH Plastic Surgery, LLRM, Meerut)
Dr. Ajeet Sachan (MBBS, DOMS) Dr. Garima Bhasin (MBBS, DGO, LLRM, Meerut)
Dr. Alka Saini (MBBS) Dr. Heramb Agnihotri (MLNMC, Allahabad)
Dr. Amit Agarwal (MD, GSVM) Dr. Hiralal Chopra (MS, SMS MC, Jaipur)
Dr. Amit Bhardwaj (MD, KGMU) Dr. Indu (MBBS, Meerut)
Dr. Amit Kumar Ojha (MBBS, BRDMC, Gorakhpur) Dr. Jayant Agarwal (MS, KGMU, Lucknow)
Dr. Amit Singhvi (MS, GMC, Kota) Dr. Juhi Kalra (MD, KGMU, Lucknow)
Dr. Anant Prakash Pandey (MBBS, BRDMC, Gorakh.) Dr. K. N. Dwivedy (MD, BRDMC, Gorakhpur)
Dr. Anjana Katiyar (BDS) Dr. Kamal Kaushik (MD, Std., KGMU, Lucknow)
Dr. Ankush Singh (MD, BJMC, Ahmedabad) Dr. Kamendra Singh (MS, KGMU, Lucknow)
Dr. Anuj Mishra (MS, Surgery, BRDMC, Gorakhpur) Dr. Kauser Sayedda (MD, KGMU, Lucknow)
Dr. Anurag Khare (MLNMC, Allahabad) Dr. Kishnan G Barupal (MD, Medicine, SNMC, Jodhpur)
Dr. Arpit Gupta (Dr SNMC, Jodhpur) Dr. Kranti Kishore (MBBS, BRDMC, Gorakhpur)
Dr. Ashok Singh (MBBS, BRDMC, Gorakhpur) Dr. Kunwar Amarendra Singh (MBBS, BRDMC, Gorak.)
Dr. Ashutosh Garg (MD, Anaesthesia) Dr. Madhukar Pandey (MD, IMS, BHU)
Dr. B.K. Singh (MS, MLNMC, Allahabad) Dr. Mahendra Narayan Singh (MD, IMS, BHU)
Dr. Bhavesh Joshi (MBBS, BRDMC, Gorakhpur) Dr. Mamta Soni (MD, GSVM, Kanpur)
Dr. C.N. Dwivedi (MBBS, BRDMC, Gorakhpur) Dr. Manoj Sharma (Dr. SNMC, Jodhpur)
Dr. Chandan Kishore (MBBS, NMCH, Patna) Dr. Manoj Singh (MBBS, GSVM, Kanpur)
Dr. Charu Gautam (MD Pharmacology) Dr. Milan Sachan (MBBS, BRDMC, Gorakhpur)
Dr. D. N. Singh (MS, Surgery, KGMU) Dr. Moti Lal Khatri (Dr. SNMC, Jodhpur)
Dr. Deepa Goel (BDS, Std., Lucknow) Dr. Monica gupta (MBBS, SMC Meerut)
Dr. Deepanshu Garg (MD, Medicine, New York) Dr. Narendra Kumar (MD, Std., KGMU, Lucknow)
Dr. Devendra (MD, KGMU, Lucknow) Dr. Neeraj Goenka (MD, NHLMC, Ahmedabad)
Dr. Dharmendra Gangwar (MBBS, BRDMC, Gorakh.) Dr. Neetu Verma (BAMS)
Dr. Dimple Shah (MD, BJMC, Ahmedabad) Dr. N. N. Khanna (Cardiologist, Apollo Hospital, Delhi)
Mr. Dinesh Kumar (RIO, IGIMS, Patna) Dr. Neetu Singh (MBBS, DGO, Gorakhpur)
Dr. Dinesh Singh (MBBS, BRDMC, Gorakhpur) Dr. Neha Bhargava (BDS, S. P. Medical College, Lucknow)

vii
Dr. Nidhi Garg (BDS) Dr. Sachin Bansal (DCH, KGMU)
Dr. Nishant Parashar (Dr SNMC, Jodhpur) Mr. Sachin Goel
Dr. Omkar Nath Rai (MD, GSVM, Kanpur) Dr. Sagan Jain (MBBS, BRDMC, Gorakhpur)
Dr. Pinakin Patel (MD, BJMC, Ahmedabad) Dr. Sakshi Puri (Delhi)
Dr. Pooja Ghambir (DLO, New Delhi) Dr. Sameer Gupta (MBBS, BRDMC, Gorakhpur)
Dr. Prabhat Kumar (MD, Medicine, KGMU) Dr. Sandeep Sharma (DA, Madurai)
Dr. Pragya Garg (MBBS, DGO) Dr. Sanjay Mishra (MBBS, BRDMC, Gorakhpur)
Dr. Pramila Kumari (BHMS, Lucknow) Dr. Sanjeev Singh (MBBS, BRDMC, Gorakhpur)
Dr. Pratibha Singh (MBBS, LLRMMC, Meerut) Dr. Shailendra Verma (MD, KGMU)
Dr. Mrs. Preeti Singh (Distt. Commandant) Dr. Shishir Gupta (MDRD, Indore)
Dr. Prerana (MBBS, KGMU, Lucknow) Dr. Shivendu Ojha (MBBS, BRDMC, Gorakhpur)
Dr. R.S. Prashad (MS, IMS, BHU) Dr. Shyam Singh (MBBS, BRDMC, Gorakhpur)
Dr. Rajendra Kumar (MBBS, KGMU, Lucknow) Dr. Shyam Vijay Singh (MBBS, BRDMC, Gorakhpur)
Dr. Rajesh Chaturvedi (MD, FMT, BHU) Dr. Sonakshi Sharma (MBBS, Ghazibad)
Dr. Rakesh Banda (MD, SNSMC) Dr. S. N. Khanna (Cardio, Surgeon, Escorts, New Delhi)
Dr. Rakesh Dubey (MBBS, BRDMC, Gorakhpur) Dr. Sonali Chaturvedi (MD, KGMU, Lucknow)
Mr. Ratnesh Singh (SDM) Dr. Sudeep Verma (MBBS, MLNMC, Allahabad)
Dr. Reeti Singhal (MD, KGMU, Lucknow) Dr. Sudheer Sharma (MD, Anaesthesia)
Dr. Rishikesh Shweale (MD, Std., Pharma, KGMU, Lucknow) Dr. Sunil Mishra (Dept. of Pathology, BHU, Varanasi)
Dr. Ritesh Sanguri (MD, KGMU, Lucknow) Dr. Vikas Soni (MD, GSVM, Kanpur)
Dr. Ruchita Sharma (MBBS, SMC Meerut) Dr. Vikram Singh (MD, IMS, BHU)
Dr. S. P. Chaudary (MBBS, LLRM, Meerut)

Our sincere thanks to Mr. J P Vij (CMD) and Shri Bhupesh Arora (GM Publishing) Jaypee Brothers Medical Publshers,
Rajan, Kusum, Ravinder and production department.

viii
New Drugs on Horizon
Monoclonal Antibody Based Drugs
Name MOA Indication Toxicity
Rituximab Anti CD20 • B-cell lymphoma – Infusion related (fever,
• CLL urticaria, urticaria, hypotension)
Alemtuzumab Anti CD52 • B cell CLL – Late onset neutropenia
• T cell lymphoma – T cell depletion with infection
– Pancytopenia
Daclizumab Anti CD25 • T -cell mycosis – Fever, arthralgia
fungoides – Asthenia, Hypotension
Gemtuzumab Anti CD33 • AML, Myelodysplasias – Infusion related
– Bone marrow suppression
– Hepatic toxicity
Trastuzumab Anti HER-2/neu • Breast cancer – Cardiomyopathy
(ErbB–2) – Infusion related
Cetuximab Anti EFGR* • Non small cell lung – Skin rash (in 75% patients)
(ErbB–1) cancer
• Colorectal cancer – Infusion
• Breast cancer
• Pancreatic cancer
Bevacizumab Anti VEGF** • Colorectal cancer – Hypertension
• RCC – Pulmonary haemorrhage
– GIT perforation
– Proteinuria
– Congestive heart failure
Omalizumab Anti IgE antibody • Bronchial asthma – Injection site reactions (stinging
(In chronic asthma as bruising, induration) a rescue
against Fc portion of IgE) medication & in status
Not given in acute asthma asthmatics)
Muromonab Anti CD3 or • Allograft rejection
OKT 3 Kidney, liver & heart
Infliximab Anti TNFα*** • Rheumatoid arthritis – Infusion related
• Crohn’s disease – Upper respiratory tract
infections
– Urinary infections
Adalimumab Anti TNFα • Rheumatoid arthritis
Etanercept (not Anti TNFα • Rheumatoid arthritis
a monoclonal Ab)

ix
New Drugs on Horizon
Efalizumab Anti LFA-1*** • Organ transplantation
• Psoriasis
Linalidomide Immunomodulator • MDS Anaemia
thalidomide
analogue
Anagrelide HCL • Essential thrombocytopenia

Abatacept Selective T cell • Moderate to severe active


rheumatoid arthritis who have
had an inadequate response to
more than one DMARDs
Costimulation
Modulator

* EGFR – Epidural growth factor receptor


** VEGF – Vascular endothelial growth factor
*** TNFá – Tumour necrosis factor alpha
**** LFA-1– Lymphocytes function associated antigen

Cardiovascular Drugs
Name MOA Indication Toxicity
Nesiritide Recombinant from Refractory CHF Hypertension
(given only i.v.) of BNP (Brain (congestive heart failure
natriuretic peptide) with arrythmia risk)

– Natriuretic
– Diuretic
– Vasodilator
Omapatrilat Vasopeptidase inhibitor Congestive heart failure Hypotension

↑ ↑BNP,↑ ↑ANP
↑ ↑ ATII
Ibutilide / Selective K+ channel Atrial fibrillation Torsades de pointes
Dofetilide blocker (inward rectifying
K+ channel blocker)
Isradipine Calcium channel blocker AV block
Mibefradil HCL Calcium channel blocker Hypertension & Angina

x
New Drugs on Horizon
Hypolipidemic Drugs
Name MOA Indication Toxicity

Ezetimibe – ↓↓ Cholesterol Primary hypercholesterolemia None reported


absorption
– Compensatory increase
in cholesterol synthesis in
so, always combined with
a statin (synergistic effect)
– ↓↓ LDL-C by 18%

So, torcetrapib increase HDL
by inhibiting its degradation in
liver because TG loaded HDL2
is destroyed by liver
CETP Inhibitors – Increased HDL-C Atherosclerosis
[Cholesteryl ester
transfer protein]
Pramlintide – Antidiabetic, Diabetes mellitus
Amylinomimetic
Exenatide – Antidiabetic, Type II DM
incertin mimetic
Insulin detemir – Antidiabetic
(rDNA) long acting insulin OD/ BD/ Type I or Type II DM
Drospirenine/
Estradiol – Hormones, Progestin & vasomotor symptoms @ with
Estrogen Menopause

xi
New Drugs on Horizon
Antiviral Drugs
Name MOA Indications Toxicity
Integrase Inhibitor
Raltegravir Inhibit integrase HIV
enzyme
Enfuvirtide Fusion inhibitor HIV positive adults (only Injection site reactions
in treatment experienced)
Emtricitabine – Nucleoside reverse HIV infection Non - specific
transcriptase inhibitor
– Same as lamivudine but
10 times more potent
Oseltamivir / Neuraminidase inhibitor Influenza Viruses Cough, Bronchospasm,
Zanamivir (Aggregation spread) : Nausea, Vomiting
Inhibition
Imiquimod Genital warts

HIV regimen: NNRTI based = Efavirenz + Lamivudine / Emtricitabine + Zidovudine / Tenofovir


PI based = Lopinavir / Ritonavir + Lamivudine / Emtricitabine + Zidovudine

Anti Emetic Drugs


Name MOA Indications Toxicity
Dronabinol – Stimulates CB1 – Cancer chemotherapy
cannabinoid receptors induced vomiting
around vomiting – AIDS with anorexia
centre (to stimulate appetite)
– Also stimulates appetite
Aprepitant – Antagonists of NK1 subtyp – Cancer chemotherapy
of tachykinin receptor induced vomiting
(Delayed phase)
Clidinium – Anticholenergic, – ↓ Motility of gut
Dexloxiglumide – Cholicystokinene – Gastroparesis
receptor antagonist

xii
New Drugs on Horizon
Anti Hepatitis Drugs
Name MOA Indications Toxicity
Entecavir – Nucteoside analogue – Chronic hepatitis B

Adeforvir – Inhibits HBV DNA – Chronic HBV infectioni Nephrotoxicity


polymerase – Lamivudine resistant
HBV infection
Lamivudine – Inhibits HBV DNA – Chronic HBV infection
polymerase
– Inhibits HIV reverse
transcriptase
Ribavirin – Inhibits viral replication – Chronic HCV infection Rash, Wheezing
(given with interferon a) – Conjunctival irritation
Interferon a – Protein synthesis – Chronic HBV infection – Acute influenza like
inhibitor – Chronic HCV infection syndrome
– HIV associated Kaposi’s
sarcoma
– Condylomata accuminata

Tyrosine Kinase Inhibitors


Name MOA Indications Toxicity
Imatinib mesylate Inhibitor of : • CML – Nausea, Vomiting
• ABL • CMML, HES* – Edema
• PDGFR • GIST** – Muscle cramps
Geftinib Inhibits EGFR type I • Non small cell lung – Diarrhoea
cancer – Acne
(ErbBI or HERI) – Pruritis
Erlotinib Inhibits EGFR type I • Non small cell lung – Diarrhoea
cancer – Interstitial lung disease
Oprelvekin Recombinant form of • Non myeloid malignancies – Fatigue, headache and
Interleukin-11 • Severe thrombocytopenia dizziness
Denileukin • Cutaneous T-cell Lymphoma – Hypersensitivity
Sorafenib Antica multikinase • RCC
Inhibitor

* HES = Hypereosinophilic Syndrome


** GIST = Gastrointestinal Stromal Tumour

xiii
New Drugs on Horizon
Miscellaneous Drugs

 Glycycline, Tetracycline group used in SSSI – Tigecycline


 Ophthalmic corticosteroid – Fluocinolone, acetomine
 Ophthalmic NSAIDs – Bromfenac sodium
 Selective estrogen receptor modulators (SERM) – Tamoxifene, Raloxifene, Ormeloxifene
 Selective estrogen receptor downregulators (SERD) – Fulvestrant
 Aromatase inhibitors (Al) – Formestane, Exemestane,
Aminoglutethimide, Anastrazole, Letrozole
 Glucagon like peptide I analogue – Exenatide (Exendin - 4)
(approved for twice daily injection for type 2 diabetes – Liraglutide (NN221 I)
mellitus in combination with other drugs)
 Newer Doxorubicin analogues – Valrubicin
(approved for intravescical therapy of BCG - resistant – Epirubicin
urinary bladder carcinoma in situ)
 Anti Condylomaacuminata (topical therapy) – Imiquimod
 Cessation of Nicotine Addiction – Bupropion
– Rimonabant (CB receptor antagonist)
 Oxytocin receptor antagonist – Atosiban
(used for suppressing preterm labour)
 Recombinant urate oxidase (used in Gout) – Rasburicase
 Becaplermin (rh - PDGF) – Topically used in non healing diabetic ulcer
 Capsofungin – Antifungal: Invasive aspergillosis
resistant to AMB or viriconazole,
Esophagal candidiasis
 Eflornithine – Ornithine decarboxylase (irreversible
inhibitor)
– West Africal Trypanosomiasis
 Arsenic trioxide – Leukemia
 Homoharringtonine (Alkaloid) – Block Peptide bond (CML)
 Protirelin – Tri peptide (TRH).

xiv
Contents

Chapter 1 General Pharmacology --------------------------------------------------------------------------- 1 - 66


Chapter 2 Pharmacology of ANS, PNS and Autacoids ------------------------------------------------ 67 - 150
Chapter 3 Pharmacology of Central Nervous System ------------------------------------------------- 151 - 238
Chapter 4 Pharmacology of Cardiovascular and Renal System ------------------------------------ 239 - 314
Chapter 5 Pharmacology of Respiratory, Endocrinal and Gastrointestinal System ---------- 315 - 390
Chapter 6 Antimicrobial and Antineoplastic Drugs ---------------------------------------------------- 391 - 548
Chapter 7 Miscellaneous --------------------------------------------------------------------------------------- 549 - 568
Questions from Bihar, Jharkhand, Karnataka, Kolkata MP, UP
Entrance Examinations (2000-2006) --------------------------------------------------------------- 569 - 603

xv
PHARMACODYNAMICS : Pharmacodynamics refers to the action of the drug at the cellular level. The binding of a drug to its
receptor or binding site, the relationship of dose and therapeutic level to the physiologic response, and the relationship of
drug action and efficacy to dosage interval.

TARGETS FOR DRUG ACTION (Most drug targets are proteins)


Receptors : Sensing elements of chemical communication that coordinates the overall function of the body. The majority of
drug receptors are regulatory proteins. The classification includes:
a. Extracellular receptors, which work through a second messenger E.g. Muscarinic and Nicotinic receptors
b. Intracellular receptors, which alter gene expression by altering the configuration of structural proteins regulating DNA
transcription.

Other targets include:


Ion channels Enzymes Carriers

Cl GABA Acetylcholinesterase Choline Carrier - Hemicholinium
Na+ - Antiarrhythmics Tyrosine kinase Noradrenaline uptake 1 - TCA, Cocaine
K+ channels MAO, DHFR, Na+ / K+/Cl– co transporter - Loop diuretics
Ca++ H+ / K+ ATPase, Serotonin Re-uptake - SSRIs
Na+ / K+ ATPase etc.
COX, Thymidine kinase (Acyclovir),
Reverse transcriptase

Molecules such as enzymes, transport proteins or structural proteins (e.g., histone, tubulin) may also be classified as drug
targets:
• Structural proteins (Tubulin, Vinca alkaloids, and Colchicine)
• DNA : Alkylating agents
• Immunophylins : Cyclosporine, FK-506 (Sirolimus)
• Therapeutic antibodies e.g. Abciximab / Adalimumab
• Cell wall constituents - Antibiotics - Penicillins / Cephalosporins / Amphotericin-B

Other non-proteins: • Digitalis - Digibind (antibody)


• Metals - Chelating agents
• HCl - Oral antacids

TRANSDUCER MECHANISMS
SIGNAL TRANSDUCTION : The processes which take place between Drug-receptors interactions and the effect at cellular
level.
1 Self Assessment & Review Phar macology
Pharmacology
Depending upon the molecular structure and nature of the linkage, 4 different receptor super families are identified:
1. Ligand Gated Ion-channel receptor (Inotropic - located in the neurons)
– Receptors are directly linked to an ion-channel without the intervention of second messenger
– Drug - Receptor interaction →Depolarization / Hyperpolarization ? of cell Effect (milliseconds)

E.g. : INHIBITORY EXCITATORY


- Nicotinic cholinergic - Kainate
- GABAA - NMDA
- Glycine - Aspartate
- Glutamate - 5-HT3

MOA : Once the ligand binds to the receptor, there is opening of ion-channel, thus causing inflow / outflow of ions
(depending upon the nature of channel).
• Important for moment-to-moment transfer of information across the synapses.

2. G - Protein coupled Receptor (they interact with GTP & GDP; also called as metabotropic or transmembrane spanning
receptors)
Examples of GPCR: Muscarinic, Adrenergic, Dopaminergic, 5-HT (except 5-HT3 ), Opioids, Odorant etc.
• Most abundant type of receptors
• All are single polypeptide chains having GTPase activity
Three major effectors pathways:
1. Adenylyl cyclase cAMP pathway
2. Channel regulation Ca++, Na+, K+
3. Phospholipase C: IP3- DAG pathway
• Activated G : Protein then changes the concentration of second messenger, cAMP, Calcium, PIP.
Several kinds of G - proteins are described.

Gs- Gi -
↑ Adenylyl cyclase →↑ cAMP ↓ adenylyl cyclase - ↓ cAMP
Examples:
β adrenergic amines α - adrenergic
Glucagon Muscarinic (M2)
Histamine Opioids
Gt - Gq-
↑ cGMP Phosphodiesterase →↓cGMP ↑ PLC ↑ IP3, DAG, Ca++
Examples:
photons (on Retinal Rods and Cones) 5 HT IC, M3 Acetylcholine

3. Enzyme linked Receptor (cytokine receptors)


• Membrane bound receptors one extra-cellular and the other intracellular terminal
• The Intracellular terminal binds to a kinase and activates it when binds to a ligand
• Kinases are mainly involved in cell growth and differentiation
• Two important effector pathways include:
– Ras / Raf / MAP kinase pathway - involved in cell growth, division and differentiation
– JAK / STAT pathway activated by cytokines involved in control and synthesis of many inflammatory mediators
Eg: Growth Hormone, erythropoietin, IFNs, Insulin, ANP, cytokines, EGF

4. Nuclear Receptors
Receptor mediated DNA transcription
Eg: Glucocorticoids, mineralocorticoids, sex - steroids, Vit. A, retinoic acid, Vit. D, glilazones, fibrates etc.

4
General Phar macology
Pharmacology 1
– Receptors are intranuclear (except steroids which are cytoplasmic)
– The ligand enters the cell to bind to receptor to form complex
– Receptor ligand complex cause gene transcription causing protein synthesis
E.g. : – Glucocorticoids → Lipocortin
– Mineralocorticoids - water channel proteins and other transport proteins
– Retinoic acid → regulates protein synthesis during fetal life

DRUG - RECEPTOR INTERACTIONS


Agonist - Affinity + Intrinsic activity [e.g. Adrenaline on beta adrenergic receptors]
Antagonist - Affinity + No Intrinsic activity [Propranolol on Beta receptors]
Partial agonist - Affinity + Intrinsic activity < 100%; In the presence of full agonist, PA behaves as an antagonist
[naloxone, buprenorphine on μ receptors], A partial agonist (e.g. pindolol, pentazocine) is that which
has the ability to stimulate the receptor, but has less potency at the receptor than the endogenous
ligand.
The endogenous ligand is always in the body and competes with the drug for receptor binding sites. When centrations of the
endogenous ligand are low, the partial agonist acts as an agonist, because it binds and stimulates unbound receptors,
albeit with decreased potency. However, when endogenous ligand concentrations are high, the drug competes for binding
sites on the receptors, and because of the drug’s decreased activity at the receptor, effectively blocks the binding sites for the
more potent endogenous ligand. In this case, the drug is effectively acting as an antagonist.
Inverse Agonist - Affinity + Intrinsic activity in the negative direction to that of the agonist
The Beta carboline on Benzodiazepine receptors
Competitive Antagonism
• Reversibility
• Surmountability
• Parallel shift of DRC to right
E.g. : propranolol, atropine, pheniramine
A drug receptor antagonist binds to an endogenous receptor and does not elicit a response, no intrinsic activity; the antagonistic
drug prevents the binding of the endogenous agonist, resulting in decreased activation of the receptor.
A competitive antagonist binds and dissociates from the receptor, in accordance with the binding affinity of the drug for the
receptor.
Once a drug molecule binds to the receptor, produces activation, and dissociates, the opportunity exists for a second drug
molecule to occupy the binding site on the receptor. If this molecule is a receptor agonist, continued stimulation occurs. If,
however, the molecules is an antagonist, or is decreased in potency with regards to the endogenous agonist, stimulation
decreases and pharmacologic effect is impaired.

A noncompetitive antagonist has a strong affinity for the receptor binding site. This type of drug may bind covalently to the
site, resulting in a relatively permanent association. Thus, because the drug does not dissociate, it blocks the receptor
permanently. (Receptors are fairly short-lived, and are endocytosed and recycled frequently, so the term “permanent” is
relative.)

Irreversible Antagonism

Some antagonists bind to the receptor in an irreversible fashion by forming covalent bond therapeutically. Once an
irreversible antagonist occupies the receptor, the duration of action of such an antagonist is relatively independent
of its own rate of elimination and more dependent on the rate of turnover of receptor molecules e.g. phenoxybenzamine
and irreversible alpha adrenergic receptor antagonist.

Antagonist can function noncompetitively in a different way that is by binding to a site on the receptor protein separate
from the agonist binding site and thereby preventing receptor activation without blocking agonist binding. Although
these drugs act noncompetitively, their actions are reversible if they do not bind covalently.

Some drugs often called allostearic modulators’, bind to a separate site on the receptor protein and alter receptor function
without inactivating the receptors for e.g. benzodiazepine bind noncompetitively to ion channels activated by the
neurotransmitters GABA, enhancing the net activating effect of GABA on channel conductance.

5
1 Self Assessment & Review Phar macology
Pharmacology
Other types of Antagonism
• Physical Antagonism : Universal antidote
• Chemical Antagonism : NaHCO3 + HCI, Heavy metal + chelating agent
• Physiological Antagonism : Adrenaline in Anaphylaxis
• Pharmaceutical Interaction : Occurs outside the body to drug with other agent
Eg: IV phenytoin glucose bolus (Precipitation)
• Pharmacokinetic antagonism : Enzyme inducers decreasing activation of substrate

Tolerance
When a drug is given over time, the effects may decrease accordingly. This may be the consequence of, for example,
desensitization of receptors or depletion of neurotransmitter stores.
Requirement of higher quantity of drug to produce the desired pharmacological effect.
– Pharmacokinetic Tolerance : Eg. Alcohol, phenobarbitone, carbamazepine inducing their own metabolism
– Pharmacodynamic Tolerance : Drug induced changes in receptors density (down regulation)
Impairment in receptor coupling to signal transduction pathway
Eg. Tolerance to Dopamine agonists, Benzodiazepines

Tachyphylaxis : If a drug is administered and produces a response that diminishes with subsequent doses, the effect is
termed tachyphylaxis. This phenomenon is a rapid development of tolerance with subsequent dosage.
Eg. Indirect acting sympathomimetics like ephedrine, amphetamine, tyramine, nicotine.

Factors that result in variation in drug responsiveness


• idiosyncratic drug response
• tachyphylaxis
• hypersensitivity response
• alteration in concentration of drug that reaches the receptor (pharmacokinetic effect)
• variation in endogenous receptor ligand concentration
• alteration in the number or function of receptors

PHARMACOKINETICS (PK)
PK concerns with dose and concentration while PD concerns with concentration and subsequent effect.

Components of PK (ADME)
• Absorption : into the systemic circulation
• Distribution : from plasma to other organs
• Metabolism : chemical alteration of the drug
• Elimination : from the body
Before absorption one-step takes place (All except IV) i.e. Liberation i.e. liberation of the active drug from the formulation
a. Liberation
• First step that determines the onset intensity & duration of drug action.
• Liberation takes place in all routes except i.v. route and oral use of True solutions.
• Liberation is controlled by the characteristics of the drug product.
• Disintegration and dissolution are essential components of liberation. Disintegration is the smallest detectable
particle. Disintegration time and dissolution practically determine onset/intensity and duration of drug action.
b. Absorption : Unless the drug is present in the solution form, it won’t be absorbed (except i.v.) therefore dissolution
becomes the first and rate-limiting step in absorption. After dissolution the drug diffuses to the site of absorption. Except
i.v., from all other routes the drug has to pass biological membranes. Among various routes of absorption, passive
diffusion accounts for the maximum, which depends on the pH and pKa and the lipid solubility of the unionized from. The
pH of a physiologic compartment in relation to the pKa of a particular drug and the classification of the drug as a weak
base determine the amount of drug that will be absorbed in that compartment. Because drugs can only pass through cell
membranes in nonionized (neutral) form, optimizing the pH of the compartment of the pKa of the drug will result in more
drug particles existing in nonionized form (as calculated using the Henderson-Hasselbalch equation). This will result in
a greater absorption of drug in that compartment.

6
General Phar macology
Pharmacology 1
E.g. Ingestion of an antacid results in an increase in the pH of the gastric milieu. Because the pKa aspirin (a weak acid)
is 3.5, and therefore exist mainly in nonionized form in the gastric milieu, an increase in gastric pH would shift the
equilibrium to the right, resulting in an increase in the ionized form and decreased absorption of the drug.
To change urinary pH:
Acidify : NH4CL, Vitamin C, cranberry juice
Alkalinize: NaHCO3, acetazolamide
Partition coefficient measure the lipophilic characteristic of drug. The more lipophic a drug is, the higher is its partition
coefficient.
First Pass metabolism can occur in g.i.t mucosa / blood vessel / liver (maximum in liver).
During absorption from oral and rectal route, the same amount of drug may be destroyed before reaching the central
compartment. In the central compartment, the same amount of drug is bound to proteins & the same amount remains free.
Only the free fraction of the drug is therapeutically active.
There is a fixed ratio between the free and bound fraction. As the free fraction of drug is absorbed, drug from the bound form
gets released only to act therapeutically.

Modes of absorption
The major transport mechanism are as under :
• Passive Diffusion (Non-ionic or simple diffusion)
• Carrier Mediated Transport : – Facilitated diffusion / Symport
– Active transport
• Pinocytosis / Phagocytosis and Filtration.

BIOAVAILABILITY
This is the fraction of administered drug that reaches the systemic circulation in the unchanged from. In general, food retards
absorption of most orally administered drugs except: beta-blockers, griseofulvin, nitrofurantoin, ketoconazole, probucol.
Bioavailability has 2 components:
Rate of absorption, Tmax , the time to achieve maximum concentration, Cmax is the maximum concentration achieved
Extent of absorption, AUC = reflects the extent of absorption
AUC oral
Relative bioavailability = ——————— X 100
AUC IV

Factors affecting bioavailability


. First Pass metabolism
- CCBs / B.B./ Nitrates / Lignocaine / L - Dopa
. Solubility of the drug.
- for a drug to be absorbed, it should be largely hydrophobic, yet have some aqueous solubility.
. Chemical instability: drugs such as penicillin, insulin, peptide hormones are unstable in stomach pH or degraded by
intestinal enzymes
. Nature of Formulation: Particle size / salt form presence of excipients affects bioavailability

Steady-State concentration : when a drug is administered at regular intervals, there is a time when the rate of administration
is equal to the rate of elimination. The drug concentration varies in a very narrow range. When a fixed dose is administered
at each half life interval it takes 5-6 half- lives.
When a drug is administered by continuous administration, it takes one half-life to saturate 50% steady - steady
concentration, 2 half lives 75% and 3 half lives 87.5% and so on.
Bioequivalence : When the bioavailability of 2 pharmaceutical equivalent drug products of same drug is similar, they are said
to be bioequivalent.
Therapeutic Equivalence : When 2 different formulations have similar safety & efficacy. They may not be bioequivalent.

7
1 Self Assessment & Review Phar macology
Pharmacology
DRUG DISTRIBUTION
It depends on :
– Blood flow
– Permeability
– Degree of binding to plasma or tissue proteins
– Relative hydrophobicity of the drug
Volume of distribution : It is a hypothetical volume of fluid that would accommodate all the drug in the body, if the concentration
throughout was the same as in plasma.
A VD = 5L means the drug is confined to plasma volume; this the minimum volume of distribution.
E.g. Heparin / Aminoglycoside antibiotics / Mannitol / Hippurate / Urea etc.
If VD is 10-20L : Extra cellular fluid.
40L = Total body water - e.g. alcohol.
If more than that, it means the drugs is sequestered elsewhere: e.g. digoxin /chloroquine / vit. B12

Clinical Significance of VD
In case of drug overdose, dialysis can be life saving if the drug has low VD.
Larger the VD, larger is the T½

BINDING of drugs :
Albumin - Acidic drugs bind to albumin
- Has multiple sites for it
- Competitive displacement e.g. Tolbutamide/sulfa drugs
The basic groups in albumin are Arginine, Histidine, Lysine →which bind to Acidic drugs
Acidic groups are Aspartic acid / Glutamic acid / Tyrosine. Bind to basic drugs
(Binding of basic drugs to albumin has less significance)
Very lipophilic drugs bind to erythrocyte membranes than to plasma proteins.
• Displacement interactions are important in case of Narrow therapeutic index drugs.
• Only the free fraction of the drug is pharmacologically active
• Hypoalbuminemia can cause decreased protein binding. Since the free fraction of the drug is pharmacologically active
it has little significance (Unless plasma albumin falls below 2.5 gm/dl)
• α1 - acidic glycoprotein acts as binder for BASIC DRUGS, whose levels increases in pregnancy, rheumatoid arthritis,
nephritis, surgery, trauma, chronic stress etc.
• Sex hormones bind to GLOBULINS : (Sex hormones binding Globulin - SHBG).

DRUG METABOLISM
AIMS of metabolism are to make the substance les reactive and/or more water soluble to be eliminated from the body easily.
Purpose: 1. Active drug - Active metabolite (Diazepam → desmethyl Diazepam → oxazepam)
2. Inactive drug - Active metabolite (Azathioprine - Mercaptopurine)
3. Active drug - Inactive metabolite (Most drugs)

Biotransformation
• Phase I Reactions
• Degradative reactions
• Microsomal reactions
• Non microsomal reactions : – Oxidation
– Reduction
– Hydrolysis
Microsomal reactions
• Associated primarily with smooth surface endoplasmic reticulum of liver.
• Principal enzymes involved are mixed function oxidases and cytochrome P450 (absorption peak is at 450 cm-1)
• Non specific in action, can be induced or activated.

8
General Phar macology
Pharmacology 1
• Only metabolize lipid soluble drugs.
E.g. - Can metabolize lipid soluble amphetamine, but cannot metabolize less lipid soluble natural amines like tyramine
or nor-epinephrine which are mainly metabolize by non-microsomal enzymes like Monoamine oxidase (MAO) and catechol-
o-methyl transferase.
Phase I : Reactions not involving CYP
– Amine oxidation (Histamine / catecholamines)
– Dehydrogenation - (Alcohol)
– Hydrolysis - (procainamide)
Major human CYP enzymes inducing metabolism: CYP1A2, 2C9, 2C19, 2D6, 2E1, and 3A4
CYP 3A4 is the most common drug metabolizing enzyme, which metabolizes about 50% of drugs used in man.
CYP enzymes are inducible and also inhibited by drugs.
• Common Enzyme Inducers : Rifampicin, Carbamazepine, Phenobarbitone, Nevirapine, Ethanol
• Common Enzyme Inhibitors : Macrolides, Ketoconazole, Metronidazole, Griesofulvin
Phase II reactions
• Synthetic reaction
• Mainly non microsomal reactions; Exception only one unique microsomal enzyme system e.g.:- the glucuronic acid
conjugation.
i. Being microsomal, it is soluble and intact its activity in soluble form.
ii. It can form glucuronide conjugates with a range of natural metabolites also e.g. -bilirubin.

Prosthetic groups arc added to the drug to make it more water soluble to facilitate its elimination.
E.g. : glucuronidation / sulfate / acetate / amino acid conjugation.
By far, glucuronidation is most common and most important of all conjugations.
Reversal: Not all drugs undergo phase I followed by phase II.
For INH, it is first acetylated (Phase II reaction) then hydrolysed to isonicotinic acid (phase I).

Non microsomal enzymes :


• As soluble cell fractions and can still retain their catalytic activity.
• Present in cytoplasm, mitochondria of hepatic cells and in plasma.
• E.g.: -MAO, esterases, amidases, transferases and conjugases.
• Non – inducible
• Show genetic variation (eg. Pseudocholinesterase and acetyl transferase etc.)

Kinetics of Metabolism
• 1st order (Rate of metabolism is directly proportional to substrate concentration).
• 0 order (Rate of metabolism is constant despite increased substrate concentration).
• Pseudo zero order – Rate of metabolism is depressed as substrate concentration increases.

FACTORS AFFECTING DRUG METABOLISM


• Genetic influence : Acetylation of INH, Procainamide / Hydralazine
Genetic factors play a role in determining dosage and dosage frequency. An example is acetylation-patients may be classified
into “fast” and slow acetylators, depending on the level of expression and activity of acetylases. It is considered when
prescribing a drug such as isoniazid. The same is true for persons expressing decreased levels or defective
pseudocholinesterases, which would affect the clearance of drugs such as succinylcholine.
• Age : Neonate : slow metabolism in neonates, 1st week peak. 1-5 years saturated to adult values.
– Elderly: Minor changes occur but liver / kidney function ↓↓
• Pregnancy : clearance of some drugs increased (metoprolol)
• Liver disease : ↓ ↓ ↓ drug metabolism
• Time of day : Ampicillin, steroids, digoxin, lithium, Indomethacin, AUC increases 23%, given at 9.00 pm than at 9.00 am
• Diet : Charcoal, brilled beef / alcohol / grape fruit juice inhibit microsomal enzyme
• Malnutrition : ↓ ↓ ↓

9
1 Self Assessment & Review Phar macology
Pharmacology
• Alcohol • Acute :↓ ↓
• Chronic : ↑ ↑
• Other drugs : drugs can interact to increases or decreased drugs clearances
• Cigarette smoking : ↑ metabolism of theophylline/caffeine and clozapine
• Disease states, Hyperthyroidism, Acromegaly, enhance drug metabolism
• CHF, Hypothyroidism chronic malnutrition decreases drug metabolism

ELIMINATION OF DRUGS
Clearance : Amount of unchanged drug cleared from the VD per unit time. It is rate of elimination / concentration.
So rate of elimination - CL X C
Clearance of a drug can occur in kidney, liver and lungs & other tissues, so Cl total = CLR + CLH + CLp + CL other.
Half-life : Time taken attains 50% of steady state. This is only valid when the drug follows 1st order kinetics
1st order kinetics : Rate of drug elimination is directly proportional to the plasma concentration i.e. if plasma concentration
is increased or decreased by 50%; the rate of elimination is increased or decreased by 50%.
0 order Kinetics : When the drug metabolism & elimination reach a saturation point so that further increase in plasma
concentration cannot lead to further increase in drug elimination i.e. a fixed quantity of drug is eliminated per unit time.
Pseudo zero order kinetics : Phenytoin / Ethanol / Aspirin
As the concentration increases, the increases in the elimination becomes less and less
In general : Lipid soluble drugs are predominantly excreted in the bile, while water soluble drugs are primarily excreted in
urine.

10
General Phar macology
Pharmacology 1
ALL INDIA 8. In unconjugated hyperbilirubinemia, the risk of
kernicterus increases with the use of : [AI 05]
a. Ceftriaxone
1. Which of the following reaction is not involved in
detoxification of drugs ? [AI 08] b. Phenobarbitone

a. Cytochrome oxidase c. Ampicillin

b. Cytochrome P450 d. Sulphonamide


c. Methylation 9. A highly ionized drug : [AI 05; 04]
d. Sulfate conjugation a. Is excreted mainly by the kidney
2. ED50 is a measure of : [AI 08] b. Can cross the placenta easily
a. Toxicity c. Is well absorbed from the intestine
b. Safety d. Accumulates in the lipids

c. Potency 10. In which of the following phases of clinical trial of


drug ethical clearance is not required?
d. Efficacy
a. Phase I [AI 04]
3. Therapeutic index for a drug is a measure of :
b. Phase II
a. Safety [AI 08]
c. Phase III
b. Potency
d. Phase IV
c. Efficacy
11. The extent to which ionisation of a drug takes place
d. Toxicity is dependent upon pKa of the drug and the pH of
4. Phase II drug trials are done to find : [AI 08] the solution in which the drug is dissolved. Which
a. Efficacy of the following statements is not correct :
b. Lethal dose a. pKa of a drug is the pH at which the drug is 50%
ionized [AI 03]
c. Maximal tolerated dose
b. Small changes of pH near the pKa of a weak
d. Safety index
acidic drug will not affect its degree of ionization
5. Forced Alkaline Diuresis of Alkanization of urine
c. Knowledge of pKa of a drug is useful in predict-
is used in Poisoning with : [AI 08]
ing its behaviour in various body fluids
a. Barbiturates
d. Phenobarbitone with a pKa of 7.2 is largely ion-
b. Amphetamina ized at acid pH and will be about 40% nonionised
c. Alcohol in plasma
d. Datura 12. Presence of food might be expected to interfere
6. G6PD enzyme deficiency does not cause hemoly- with drug absorption by slowing gastric empty-
sis in treatment with : [AI 08] ing. or by altering the degree of ionisation of the
drug in the stomach. Which of the following state-
a. Quinine
ments is not correct example : [AI 03]
b. Pyrimethamine
a. Absorption of digoxin is delayed by the presence
c. Chloroquine of food
d. Primaquine b. Concurrent food intake may severely reduce the
7. Which one of the following antiplatelet is a prodrug: rate of absorption of phenytion
a. Aspirin [AI 07] c. Presence of food enhances the absorption of
b. Dipyridamole hydrochlorthiazide
c. Ticlopidine d. Antimalarial drug halofantrine is more extensively
absorbed if taken with food
d. Abciximab

Answer 1. a. Cytochrome ... 2. c. Potency 3. a. Safety 4. a. Efficacy 5. a. Barbiturates


6. b. Pyrimethamine 7. c. Ticlopidine 8. d. Sulphonamide 9. a. Is excreted ... 10. d. Phase IV
11. b. Small changes ... 12. c. Presence of...

11
1 Self Assessment & Review Phar macology
Pharmacology
13. All of the following statements regarding 19. Which of the following is true : [AI 00]
bioavailability of a drug are true except : a. As the concentration of drug increases over the
a. It is the proportion (fraction) of unchanged drug therapeutic range, the bound form of the drug
that reaches the systemic circulation [AI 03] increases
b. Bioavailability of an orally administered drug can b. The bound form is not available for metabolism
be calculated by comparing the Area Under Curve but is available for excretion
(0-α. after oral and intravenous(iv) administra- c. Acidic drug binds to albumin; and basic drug
tion binds β globulin
c. Low oral bioavailability always and necessarily d. Binding sites are non-specific and one drug can
mean poor absorption displace the other
d. Bioavaialabilty can be determined from plasma 20. True about teratogenicity of a drug is all except :
concentration or urinary excretion data
a. It is genetically predetermined [AI 00]
14. Regarding efficacy and potency of a drug, all are
b. Environment influences it
true, except : [AI 02; PGI Dec. 04]
c. Related to the dose of the teratogenic drug
a. In a clinical setup, efficacy is more important than
potency d. Affects specially at a particular phase of devel-
opment of foetus
b. In the log dose response curve, the height of the
curve corresponds with efficacy 21. Which of the following is not an example of cyto-
chrome p450 dehydrogenase inducer :
c. ED50 of the drug corresponds to efficacy
a. Phenobarbitone [AI 98; PGI Dec. 06;
d. Drugs that produce a similar pharmacological
effect can have different levels of efficacy b. Rifampicin PGI June 05]
15. True statement regarding first order kinetics is : c. Phenytoin
a. Independent of plasma concentration d. Ketoconazole
b. A constant proportion of plasma concentration 22. Drug, which is contraindicated in pregnancy is:
is eliminated [AI 01; PGI Dec. 04] a. Tetracycline [AI 95; AIIMS Feb. 97]
c. T ½ increases with dose b. Erythromycin
d. Clearance decreases with dose c. Ampicillin
16. All are reasons for reducing drug dosage in eld- d. Chloroquine
erly except : [AI 01]
a. They are lean and their body mass is less AIIMS
b. Have decreasing renal function with age
c. Have increased baroceptor sensitivity 23. Which drug is not acetylated ? [AIIMS May 08]
d. Body water is decreased a. NH
17. True statement regarding inverse agonists is : b. Dapsone
a. Binds to receptor and causes intended action c . Hydralazine
b. Binds to receptor and causes opposite action d. Metoclopropamide
c. Bind to receptor and causes no action [AI 01] 24. Which is a prodrug ? [AIIMS May 08]
d. Bind to receptor and causes submaximal a. Enalapril c. Captopril
18. All are pharmacogenetic conditions, except : b. Clonidine d. Lisinopril
a. Adenosine deaminase deficiency [AI 00; 25. Loading dose depends on : [AIIMS May 08]
b. Malignant hyper-pyrexia AIIMS June 99; a. Volume of distribution
c. Coumarin insensitivity PGI Dec. 06] b. Clearance
d. G6PD deficiency c. Rate of administration
d. Half life

Answer 13. c. Low oral 14. c. ED of ... 15. b. A constant ... 16. c. Have ... 17. b. Binds ...
50
18. a. Adenosine ... 19. d. Binding ... 20. a. It is genetically ... 21. d. Ketocon ... 22. a. Tetracycline
23. d. Metoclopropa ... 24. a. Enalapril 25. a. Volume ...

12
General Phar macology
Pharmacology 1
26. Which is CPY P450 inhibitor ? [AIIMS May 08] c. Salmeterol
a. Ketoconazole d. Acetazolamide
b. Rifampicin 34. Drugs undergoing acetylation include except :
c. Phenytoin a. Dapsone [AlIMS Nov. 06]
d. INH b. Metoclopramide
27. Therapeutic monitoring is done for all of the fol- c. Procainamide
lowing except : [AIIMS Nov. 07] d. INH
a. Tacrolimus 35. Side effects of a drug arise due to the interactions
b. Metformin of the drug of molecules other than the target.
c. Cyclosporine These effects of a drug can be minimized by its
high : [AIIMS Nov. 05]
d. Phenytoin
a. Specificity
28. Good clinical practices (GCPs) are not a part of :
b. Affinity
a. Preclinical studies [AIIMS Nov. 07]
c. Solubility
b. Phae I studies
d. Hydrophobicity
c. Phase II studies
36. Which of the following property of drug will enable
d. Phase IV studies
it to be used in low concentrations :
29. The following are contraindicated in pregnancy
a. High affinity [AIIMS Nov. 05]
except : [AIIMS May 07]
b. High specificity
a. Sodium nitroprusside
c. Low specificity
b. Labetalol
d. High stability
c. Spironolactone
37. Which of the following drugs can be safely pre-
d. ACE inhibitors
scribed in pregnancy ? [AIIMS May 05]
30. The following drug is contraindicated in pregnancy:
a. Warfarin
a. ACE inhibitor [AIIMS May 07]
b. ACE inhibitors
b. Calcium channel blocker
c. Heparin
c. Beta blocker
d. β - blockers
d. Penicillin
38. Km of an enzyme is : [AIIMS May 03]
31. Maternal carbimazole intake causes all except :
a. Dissociation constant
a. Choanal atresia [AIIMS May 07]
b. The normal physiological substrate concentra-
b. Cleft lip and cleft palate tion
c. Fetal goitre c. The substrate concentration at half maximal ve-
d. Scalp defects locity
32. Loading dose depends on : [AlIMS Nov. 06] d. Numerically identical for all isozymes that cata-
a. Half life lyze a given reaction
b. Plasma volume 39. For drugs with first order kinetics the time required
c. Volume of distribution to achieve steady state levels can be predicted
from : [AIIMS May 03]
d. Rate of clearance
a. Volume of distribution
33. Which of the following is a prodrug ?
b. Half life
a. Clonidine [AlIMS Nov. 06]
c. Clearance
b. Enalapril
d. Loading dose

Answer 26. a. Ketoconazole 27. b. Metformin 28. a. Preclinical ... 29. b. Labetalol 30. a. ACE ...
31. b. Cleft ... 32. c. Volume ... 33. b. Enalapril 34. b. Metoclopra ... 35. a. Specificity
36. a. High ... 37. c. Heparin 38. c. The substrate ... 39. b. Half life

13
1 Self Assessment & Review Phar macology
Pharmacology
40. All of the following drugs are metabolised by acety- 46. Alcohol intake during pregnancy causes, all, ex-
lation except : [AIIMS May 03] cept : [AIIMS Nov. 99]
a. INH a. Brachycephaly
b. Sulfonamides b. Microcephaly
c. Ketoconazole c. Hyperkinetic movements
c. Hydralazine d. Congenital anomalies
41. The lymphocytopenia seen a few hours after ad- 47. New drug study and development can be done by:
ministration of a large dose of prednisone to a a. Pharmacogenetics [AIIMS Nov. 99]
patient with lymphocytic leukemia is due to :
b. Molecular modelling
a. Massive lymphocytic apoptosis [AIIMS Nov. 02]
c. Pharmacolibrary
b. Bone marrow depression
d. Neopharmacy
c. Activation of cytotoxic cells
48. Concentration of a drug in blood is 40 microgm/
d. Stimulation of natural killer cell activity ml. Dose of the drug is 200mg. Volume of distribu-
42. The substrate concentration used for determin- tion of the drug assuming minute elimination is :
ing the activity of an enzyme having Km=x μm will a. 5 litre [AIIMS June 99]
be:
b. 0.5 litre
a. 2xμm [AIIMS Nov. 02]
c. 2.5 litre
b. 4xμm
d. 3 litre
c. 8xμm
49. All of the followings can cause hemolytic anaemia
d. 10xμm except : [AIIMS June 99, Dec. 95; PGI Dec. 04]
43. Racemic mixture of two enantiomers with differ- a. Isoniazid
ent pharmacokinetic and pharmacodynamic prop-
b. Rifampicin
erties is seen in : [AIIMS May 02]
c. Co - trimoxazole
a. Dilantin
d. Propranolol
b. Digoxin
50. A drug X has affinity to bind with albumin and Y
c. Verapamil
has 150 times more affinity to bind with albumin
d. Octreotide than X. TRUE statement is : [AIIMS June 99]
44. A drug is more likely to cause toxicity in elderly a. Drug X will available more in tissues
patients due to all of the following reasons ex-
b. Drug Y will be more available in tissues
cept:
c. Free conc. of drug X in blood will be more
a. Decreased renal excretion of drugs
d. Toxicity of Y will be more
b. Decreased hepatic metabolism
51. Drug, not metabolised by liver is :
c. Increased receptor sensitivity [AIIMS May 02]
a. Penicillin G [AIIMS June 98]
d. Decreased volume of distribution
b. Phenytoin
45. Which of the following can be given with dose ad-
justment : [AIIMS June 00] c. Erythromycin
a. Levodopa + metoclopramide d. Cimetidine
b. Gentamicin + Furosemide 52. Which of the following is not an example of cyto-
chrome P450 dehydrogenase inducer :
c. Ferrous sulphate + Tetracycline
a. Phenobarbitone [AIIMS June 97]
d. Clonidine + Chlorpromazine
b. Rifampicin
c. Phenytoin
d. Ketoconazole

Answer 40. c. Ketoconazole 41. a. Massive ... 42. a. 2xμm 43. c. Verapamil 44. d. Decreased ...
45. c. Ferrous ... 46. a. Brachycephaly 47. b. Molecular ... 48. a. 5 litre 49. d. Propranolol
50. a. Drug X ... 51. a. Penicillin G 52. d. Ketoconazole

14
General Phar macology
Pharmacology 1
PGI a. Lipid solubility
b. Volume of distribution
53. Physiological antagonism found in : c. Clearance
a. Isoprenaline and Salbutamol [PGI Dec. 07] d. Drug concentration
b. Isoprenaline and Adrenalin 61. A 70 kg man was given a drug with dose of 100mg/
kg bd. wt t1/2 is 10 hours, plasma concn is 1.9 mg/
c. Isoprenaline and Propanolol ml clearance is : [PGI Dec. 06]
d. Adrenaline and Histamine a. 0-02 liter/hr
54. Drug dependence is characterized by : b. 20 liter/hr
a. Takes it daily [PGI Dec. 07] c. K is 0.0693
b. Tolerance d. K is 6.93
c. Withdrawal symptoms occurs e. 0.2 liter/hr
d. Uses despite knowing harmful effects 62. True about tachyphylaxis : [PGI Dec. 06]
55. Which is a pro-dug : [PGI Dec. 07] a. Direct sympathomimetic involved
a. Cyclophosphamide b. Mechanism clearly understood
b. Lisinopril c. Ephedrine tachyphylaxis reversed with norad-
c. Metochlorpramide renaline
d. Ranitidine d. Indirect sympathomimetics involved
e. Eptifibatide e. It is an anaphylaxis reaction
56. Narrow therapeutic index seen in : [PGI Dec. 07] 63. Drug transport mechanism include :
a. Metformin a. Active transport [PGI June 06]
b. Phenytoin b. Passive transport
c. Cyclosporin c. Lipid solubility
d. Digitalis d. Facilitated diffusion
57. In G6PD deficiency, drugs contraindicated are : e. Symport
a. Primaquine [PGI Dec. 07] 64. Causes for less bioavailability : [PGI June 06]
a. High first pass metabolism
b. Chloroquine
b. Increased absorption
c. Hydralazine
c. IV drug administration
d. Losartan
d. High solubility
58. Advantages of fixed drug combinations :
65. Drugs inducing CYP3A4 include : [PGI Dec. 06]
a. Decrease cost [PGI Dec. 07]
a. Fexofenadine
b. Increase efficacy
b. Phenytoin
c. Increase compliance
c. Carbamazepine
d. Decrease resistance
d. Rifampin
e. Dose titration can be done
e. Glucocorticoid
59. Drugs which cause fetal renal anomalies :
66. Which of the following statement is not true ?
a. Enalapril [PGI Dec. 06]
a. If drug is administered rectally it follow first order
b. Furosemide
b. If drug is administered IM it follow zero order
c. Angiotensin receptor blocker
c. If drug is administered IV it follow first order
d. Amlodipine
d. Bioavailability is irregular after oral administra-
e. Phenytoin tion [PGI Dec. 06]
60. On I.V. drug administration elimination of a drug e. In acute renal failure, renally excreted drug fol-
depend on : [PGI Dec. 06; June 06] low zero order kinetics
Answer 53. d. Adrenaline ... 54. a, b, c and d 55. a. Cyclophosph ... 56. b, c and d 57. a and b
58. a, b, c and d 59. a and c 60. a, b, c and d 61. e. 0.2 liter/hr 62. d. Indirect ...
63. a, b, c, d and e 64. a. High first ... 65. b, c, d and e 66. a, b and c

15
1 Self Assessment & Review Phar macology
Pharmacology
67. Pharmacogenetics is associated with : b. Ansacrine
a. Variability of enzyme action [PGI Dec. 06] c. Rifampicin
b. Environmental influence d. Tolcapone
c. Individual variability in oral absorption 74. First pass metabolism is seen in : [PGI Dec. 03]
d. Different MAO in different individual a. Lignocaine
e. Different dose response curve in different indi- b. Propranolol
vidual c. Salbutamol
68. Drug synergism is seen in all except : d. Dipyridamole
a. Flucytosine and amphotericin B for cryptococcal e. Erythromycin
b. Trimethoprim and sulphamethoxazole for UTI 75. Most important side effect of dapsone is hemo-
c. Penicillin and aminoglycoside [PGI June 05] lytic anaemia, the next adverse effect :
d. Chlortetracycline + penicillin a. G - 6 - PD deficiency [PGI June 03]
69. Drugs given by I.V. route : [PGI June 05] b. Infectious mononucleosis like syndrome
a. Heparin c. Agranulocytosis
b. Pantoprazole d. Lichenoid eruption
c. Ranitidine e. Skin pigmentation
c. Sumatriptan 76. Blood brain barrier crossed by : [PGI Dec. 02]
d. Neomycin a. Dopamine
70. True about efficacy and potency of drugs : b. Propranolol
a. Efficacy is clinically more important than potency c. Glycopyrrolate
b. Height of DRC corresponds to efficacy d. Physostigmine
c. The amount of drugs to produce certain re- e. Streptomycin
sponse is called efficacy [PGI Dec. 04] 77. Low therapeutic range is seen in : [PGI Dec. 02]
d. Drugs having similar pharmacological action a. Lithium
may have different efficacy
b. Erythromycin
71. True about routes of drug administration :
c. Phenytoin
a. 80% bioavailability by I.V. injection
d. Propranolol
b. I.M. administration needs sterile technique
e. Tricyclic antidepressant
c. I.D. injection produces local tissue necrosis and
78. In hepatic metabolism phase II reactions are :
irritation [PGI June 04]
a. Dealkylation [PGI Dec. 02]
d. Inhalation produces delayed systemic bioavail-
ability b. Sulfonation
72. Volume of distribution of drugs altered in : c. Methylation
a. Obesity [PGI June 04] d. Glucuronization
b. Athletes e. Deamination
c. Pregnancy 79. Drug safely given in pregnancy : [PGI Dec. 02]
d. Older age a. Antifolate
e. Neonate b. Quinine
73. Dose adjustment is needed if bilirubin is >1.5mg/ c. Chloroquine
dl in : [PGI June 04] d. Primaquine
a. Methotrexate e. Tetracycline

Answer 67. a and d 68. d. Chlortetra ... 69. a, b and c 70. a, b and d 71. b and c
72. a, c, d, and e 73. a, c and d 74. a, b and c 75. c and e 76. b and d
77. a and c 78. b, c and d 79. c. Chloroquine

16
General Phar macology
Pharmacology 1
80. High hepatic excretion ratio seen in : c. Toxicity
a. Propranolol [PGI June 02] d. Pharmacogenetics
b. Lidocaine 86. Characteristic feature of agonist is :
c. Ampicillin a. Has affinity only [PGI Dec. 99]
d. Imipramine b. Has affinity as well as intrinsic activity
e. Theophylline c. Has intrinsic activity only
81. Drugs which cause malformation in the foetal in- d. Neither has affinity nor activity
clude : [PGI Dec. 01] 87. True regarding dose-response curve is :
a. Heparin a. Cannot determine the potency of a drug
b. Warfarin b. Log dose response curve is sigmoid shaped
c. Valproic acid c. Cannot find response to antagonist [PGI June
d. Steroids d. A wide range of doses can not be plotted 99]
e. Phenytoin 88. Mechanism of action of GABA is on :
82. Which of the following drugs are secreted in breast a. G protein [PGI Dec. 99]
milk : [PGI June 01]
b. Tyrosine kinase
a. Antihistaminics
c. PIP / DAG
b. Antithyroid drugs
d. 5 - HT
c. Penicillin
89. The maximum effect of a drug is defined by its :
d. Diazepam
a. Therapeutic index [PGI Dec. 98]
e. Antiepileptics
b. Potency
83. Displacement of protein bound drug :
c. Efficacy
a. ↑ ↑ Drug plasma level [PGI June 00]
d. Adversity
b. ↑ ↑ Side effects
90. Alkalinity of urine is done in : [PGI June 97]
c. ↓ Free levels
a. Barbiturate poisoning
d. ↓ Effect
b. Lithium toxicity
84. Which is true about Max. distribution :
c. Alprazolam overdone
a. Highly lipophilic [PGI June 00]
d. Diazepam toxicity
b. Blood brain barrier
91. About acidic drug true is : [PGI June 97]
c. ↓ Excretion
a. Best absorbed in acidic medium
d. ↓ Receptor
b. Best absorbed in alkaline medium
85. At pharmacological doses unwanted unavoidable
c. Not absorbed in acidic medium
effects are called : [PGI Dec. 99]
a. Side effects d. Binds to alpha glycoprotein
b. Idiosyncratic reaction

Answer 80. a, b, d and e 81. b, c, d and e 82. a, b, c, and d 83. a. ↑ ↑ free ... 84. a. Highly ...
85. a. Side ... 86. b. Has affinity ... 87. b. Log dose ... 88. a. G-protein 89. c. Efficacy
90. a. Barbiturate ... 91. a. Best ...

17
1 Self Assessment & Review Phar macology
Pharmacology

ANSWERS, REFERENCES, EXPLANATIONS WITH INFORMATIVE ILLUSTR ATIONS

1. Ans. is a i.e. Cytochrome oxidase Ref. KDT 6/e, p 25 - 26

Cytochrome oxidase is the enzyme for final component of electron transport chain, but not for chemical
pathways of drug biotransformation.
Xenobiotics are metabolized by monooxygenases like CYP P-450.
Drug biotransformation is commonly grouped into two types, which is depicted below as Phase I (non-synthetic
reaction) and Phase II (synthetic reaction).

Biotransformation occurs as follows :

18
General Phar macology
Pharmacology 1
Mnemonics • All reactions starting with ‘De’ are phase - I reactions
– Dealkylations – Deaminations
– Desulphuration – Dechlorination
– Decyclization
• All the reactions starting with ‘G’ are phase - II reactions
– Glucoronide conjugation – Glycine conjugation
– Glutathione conjugation

2. Ans. is c i.e. Potency Ref. KDT 6/e, p 55

• ED50 means effective dose which can provide 50% of the maximal response.
• Smaller the ED50, more potent is the drug, which is shown in log dose response (LDR) curve below.

This graph shows that the drug A is more potent because its ED50 is comes at vary low dose as compared to
ED50 of drug B.
The potency of the drug tells us nothing about its efficacy and safety and therefore a relatively unimportant
characteristic for therapeutic purposes.

3. Ans. is a i.e. Safety Ref. Rang & Dale KDT 55, 5/e, p 89; KDT 6/e, p 55; Katzung 10/e, p 30 - 31

Therapeutic index of a drug in terms of ratio between the average minimum effective dose and the average
maximum tolerated dose in group of subjects:

Maximum non-toxic dose Median lethal dose LD 50

TI = = =
Minimum effective dose effective dose ED 50

LD is the dose that is lethal in 50% of the population and ED is the dose that is effective in 50% for a safe
50 50

drug, the TI should be at least more than one and hence or drug having larger value of LD but smaller value of
50

ED is considered to be more safe.


50

19
1 Self Assessment & Review Phar macology
Pharmacology
Limitation of TI:
• LD does not reflect toxicity in the therapeutic setting, where unwanted effects are common, but rarely
50

death.
• ED is often not definable for e.g. - the ED for aspirin used for a mild headache is much lower than for
50 50

aspirin as an antirheumatic drug.

4. Ans. is a i.e. Efficacy Ref. Goodman & Gilman 11/e, p 133 - 134; KDT 6/e, p 77

In Phase II drug is studied for first time in patients with target disease to determine its efficacy.
Clinical Testing has four phases
Phase I Non-blind or Open label

• Number of subjects 20 - 25 healthy volunteers.


• i. Determines - Pharmacokinetic differences in animals and humans
ii. Determines safe and tolerated dose
iii. Determines any predictable toxicity
iv. Determines Pharmacokinetics of the drugs in humans

Phase II Single or Double blind


• First time in patients with target disease
• Determines efficacy and safety
Early phase – 20 – 200 patients, single blind
Late phase – 50 – 300 patients, double blind.

Phase III Double blind


Are large scale randomized control trials in patients (250 - 1000 plus).
• Determines safety and efficacy
Designed to minimise error in information gathered in P - I and P - II.

Phase IV Post-licensing phase-field trials.


No fixed duration (it is the surveillance phase during post marketing clinical use)

5. Ans. is a i.e. Barbiturates Ref. KDT 6/e, p 13, KDT 6/e, p 392

Alkalisation of urine is done therapeutically for salicylate or barbiturate poisoning.


Since the pH of the urine is acidic, all acidic drugs (e.g. salicylates, barbiturates and sulfonamides) remain
predominantly noninonised and have more changes of their reabsorption than excretion.
On the contrary, if the pH of the urine is made alkaline by giving sodium bicarbonate or sodium citrate, the
ionization of acidic drugs would increase. As a result their reabsorption would be retarded while excretion
facilitated.
Drugs working effectively at alkaline pH (basic drug)
• Cotrimoxazole • Gentamicin
• Cephalosporins • Fluroquinolones
Drugs that work effectively better at Acidic pH (acidic drug)
• Nitrofurantoin • Tetracyclin
• Methicillin • Cloxacillin

20
General Phar macology
Pharmacology 1
6. Ans. is b i.e. Pyrimethamine Ref. Harrison 17/e, p 657

Drugs that carry risk of clinical hemolysis in persons with G6PD deficiency new list included in Harrison
17 edition
Groups Definite risk Possible risk Doubtful risk

Antimalarials Primaquine Chloroquine Quinine


Dapsone/chlorproguanil
Sulphonamidex/sulphones Sulphametoxazole Sulfasalazine Sulfisoxazole
Others Sulfadimidine Sulfadiazine
Dapsone
Antibacterial/antibiotics Cotrimoxazole Ciprofloxacin Chloramphenical
Nalidixic acid Norfloxacin p-Aminosalicylic acid
Nitrofurantoin
Niridazole
Antipyretic/analgesics Acentanilide Acetylsalicylicacid Acetylsalicylic acid <3 g/d
Phenazopyridine high dose (3 g/d) Acetaminophen
(Pyridium) Phenacetin
Other Naphthalene Vitamin K analogues Doxorubicin
Methylene blue Ascorbic acid >1g Probenecid
Rasburicase

7. Ans. is c i.e. Ticlopidine Ref. Goodman & Gilman 11/e, p 1482; KDT 6/e, p 23 - 24

“Ticlopidine is a prodrug that requires conversion to the active thiol metabolite by a hepatic cytochrome
P450 enzyme.” ... Goodman & Gilman
Clopidogrel is closely related to Ticlopidine but has more favorable toxicity profile-less thrombocytopenia and
leukopenia.
• Both the drugs are inhibitors of Purinergic receptors present on the platelets
Thienopyridine drugs Ticlopidine and Clopidogrel are prodrugs.

Prodrug
• Few drugs are inactive as such and need conversion in the body to one or more active metabolites such a
drug are called prodrugs.
• The prodrug may offer advantages over the active form in being :
– More stable
– Having better bioavailability
– Other desirable pharmacokinetic properties
– Less side effects and toxicity.
Prodrug Active form Prodrug Active form
Azathioprine Mercaptopurine Dipivefrine Epinephrine
Bacampicillin Ampicillin Enalapril Enalaprilat
Benyrolate Aspirin + Paracetamol Levodopa Dopamine
Cortisone Hydrocortisone Proguanil Proguanil triazine

21
1 Self Assessment & Review Phar macology
Pharmacology

Cyclophosphamide Aldophosphamide Sulindac Sulfide metabolite


Sulfasalazine 5 Aminosalicylic acid Zidovudine (All NRTI’s) Zidovudine triphosphate
Mercaptopurine Methylmercaptopurine Flourouracil Flourouridine mono PO4- - -
Prednisone Prednisolone

ALL ACE inhibitors are Prodrugs except : C – Captopril


L – Lisinopril

8. Ans. is d i.e. Sulphonamide Ref. KDT 6/e, p 684

Kernicterus in Neonates
Sulphonamide can precipitate kernicterus in the newborn, specially if premature, by displacement of bilirubin
from plasma protein binding sites and more permeable blood brain barrier (causing unconjugated
hyperbilirubinemia).

Common adverse effects of sulphonamide


• Nausea, vomiting and epigastric pain • Crystalluria - dose related
• Hypersensitivity reactions • Hepatitis - dose unrelated
• Hemolysis in individuals with G-6-PD deficiency • Kernicterus in the new born

Mnemonic : 5S - Sulfonamide major side effects


Sulfonamide side effects :
– Steven-Johnson syndrome
– Skin rash
– Solubility low (causes crystalluria)
– Serum albumin displaced (causes newborn kernicterus and potentiating of other serum
albumin binders like warfarin)

9. Ans. is a i.e. Is excreted mainly by the kidney Ref. KDT 6/e, p 16, 30

A highly ionized drug is not reabsorbed by the tubules and is easily excreted by the kidneys because it
is lipid insoluble.

Ionization increases renal clearance of drugs :


• Only free, unbound drug is filtered
• Only nonionized forms undergo active secretion and active or passive reabsorption.
• Ionized forms of drug are “trapped” in the filtrate.
Non Ionized Drugs Ionized Drugs

• Non ionized drugs are lipid soluble • Ionized drugs are lipid insoluble
• Rapidly pass across placenta • Poorly pass across placenta
• Rapidly absorbed from GIT • Poorly absorbed from GIT

22
General Phar macology
Pharmacology 1
10. Ans. is d i.e. Phase IV
Ref. Goodman & Gilman 11/e, p 133 - 134; KDT 6/e, p 77

Conducted by large number of clinicians at different centers.


• Looking at the flowchart below, we can easily conclude that phase 4 of clinical testing is a phase of post
marketing surveillance after reviewing the results of phase 1, 2 and 3 of clinical testing. This phase occurs
only after FDA safety review, NDA submission and NDA approval, so it does not require ethical clearance
anymore.

Clinical Testing has four phases


Phase I Non-blind or Open label
• Number of subjects 20 - 25 healthy volunteers.
• i. Determines - Pharmacokinetic differences in animals and humans
ii. Determines safe and tolerated dose
iii. Determines any predictable toxicity
iv. Determines Pharmacokinetics of the drugs in humans

23
1 Self Assessment & Review Phar macology
Pharmacology

Phase II Single or Double blind

• First time in patients with target disease


• Determines efficacy and safety
Early phase – 20 – 200 patients, single blind
Late phase – 50 – 300 patients, double blind.

Phase III Double blind


Are large scale randomized control trials in patients (250 - 1000 plus).
• Determines safety and efficacy
Designed to minimise error in information gathered in P - I and P - II.

Phase IV Post-licensing phase-field trials.


No fixed duration (it is the surveillance phase during post marketing clinical use)

11. Ans. is b i.e. Small changes of pH near the pKa of a weak acidic drug will not affect its degree of
ionization Ref. KDT 6/e, p 12 - 13

• Lets take each option separately :


Option ‘a’ Drugs are weak acids or weak bases and can exist in either nonionised (HA) or ionised (A)
forms in an equilibrium, depending on pH of the environment and their pKa (pKa is numerically
equal to the pH at which the molecule is 50% ionised and 50% nonionised.)
Option ‘b’ One scale change in pH causes a ten fold change in ionisation. With the change of pH, the rate
of change of ionisation is greatest at pH value near pKa.
Option ‘c’ Non ionized form of the drug cross biological membranes and enters different body fluids. pKa
of the drug therefore, is useful in predicting its behaviour in various body fluids.
Option ‘d’ Phenobarbitone is a weakly basic drug with a pKa of 7.2. It will, thus be largely ionised at acidic
pH.

An example of a case in which this is an issue is the use of:


• Weak basis local anesthetic (e.g., lidocaine prilocaine) when an infection or inflammation is present.
• Altered gastric pH. An acidic drug may be incompletely absorbed if gastric pH is increased. This may occur
through the use of antacids, H -receptor antagonists, or the use of proton pump inhibitors, or in conditions
2

where vomiting is present. This may result in systemic alkalosis, which further lowers drug potency.

12. Ans. is c i.e. Presence of food enhances the absorption of hydrochlorthiazide


Ref. KDT 6/e, p 16, 461, 748; Katzung 10/e, p 855

Food in general reduced absorption rate of drug.


• Presence of food in stomach delays absorption of digoxin as well as digitoxin.
• Oral absorption of halofantrine is variable and is enhanced with food but because of toxicity concerns, it
should however not be taken with meals.
• Food ingested has not been found to affect the absorption of hydrochlorthiazide.
• Absorption of phenytoin by oral route is slow, mainly because of its poor oral solubility. We could not find
any reference of the effect of food on phenytoin absorption.

24
General Phar macology
Pharmacology 1
Empty stomach favoured the absorption of drug.
For E.g. – Rifampin rate and extent of absorption reduced after meal.
– Absorption of tetracycline’s is also markedly reduced if taken with milk and milk products.

Exceptions
• Absorption of certain antifungal drugs (e.g. griseofulvia) is enhanced by administering the drug with fatty
diet.
• Vitamin C, iron in its ferrous form therefore, increase its bioavailability.
• Bioavailability of phenytoin improves after meals by its better dissolution due to food-induced bile secretion.

13. Ans. is c i.e. Low oral bioavailability always and necessarily means poor absorption
Ref. KDT 6/e, p 17

The absorbed drug may undergo first pass metabolism in the intestinal wall / liver or be excreted in bile. So,
drug can absorbed from intestine and before reaching the target organ it get metabolized in liver (MC) not means
that it is not or poorly absorbed.

Term bioavailability is used to indicate the proportion of unchanged drug that passes into the systematic
circulation after oral administration, taking into account both absorption and local metabolic
degradation.

Clearance can markedly affect the extent of availability because it determines the extraction ratio. (Option “d”).
• Bioavailability is determined by the area under the plasma concentration time curve. (Option “b”)
• Area under the curve of an orally administered drug when compared to the area under the curve for an IV
administered drug (100%) can certainly give an idea of its bioavailability.
• Importance of Bioavailability : Bioavailability greater concern with drugs that shows narrow margin of
safety (e.g, antiarrhythmics, antidiabetics etc.).
• Let’s look in to bioavailability of a drug administered by different routes :
IV / IA > TD > IM / SC > Rectal > Oral / Inhalational.
Route Bioavailability (%)

IV / IA 100
IM 75 to ≤ 100
SC 75 to ≤ 100
Oral (PO) 5 to < 100
Rectal (PR) 30 to < 100
Inhalation 5 to < 100
Transdermal (TD) 80 to < 100

14. Ans. is c i.e. ED50 of the drug corresponds to efficacy


Ref. Katzung 10/e, p 28; KDT = 54, 55

• ED means effective dose which can provide 50% of the maximal response.
50

• Smaller the ED more potent is the drug and hence, it tells about potency of drug.
50

25
1 Self Assessment & Review Phar macology
Pharmacology
Efficacy denotes the maximal response as reflected by the height of the Log dose response curve on its
ordinate i.e. Y axis (height of curve).
The curve for the drug with greater affinity (i.e., acting at the lower concentration), will lie close to the ordinate,
while the curve for the drug with lesser affinity will lie farther towards right (drug B in figure).
– Potency means the dose of a drug required to produce a standard effect.

• For analysing the options clearly, let’s go through the salient features of potency and efficacy in the following
table :
Potency Efficacy
• Potency refers to the amount of drug • Efficacy refers to the maximal response that can
needed to produce a certain response. be elicited by the drug. It refers to the maximum
height of the curve.
• Competitive antagonist acts by decreasing • Non competitive antagonist acts by decreasing
the potency of a drug, such that maximal the efficacy of a drug. Maximal response is
response (efficacy) can still be attained by suppressed and unsurmountable. Causes
increasing dose of agonist. Causes rightward flattening of DRC.
shift of DRC.
• Potency refers to the concentration (EC50) • The efficacy of a drug is obviously crucial for making
or dose (ED50) of a drug required to clinical decisions when a large response is needed.
produced 50% of that drugs maximal effect. It is a more decisive factor in the choice of a drug.
• Potency of a drug depends in part on the • It may be determined by the drug’s mode of
affinity of receptors for binding the drug and in interactions with receptors or by characteristics
part on the efficiency with which drug receptor of the receptor effector system involved.
interaction is coupled to response.

15. Ans. is b i.e. A constant proportion of plasma concentration is eliminated


Ref. KDT 6/e, p 31 - 32

Before coming to answer read following facts about First order and Zero order Elimination Rate.

26
General Phar macology
Pharmacology 1
First Order elimination Rate Zero order Elimination Rate

• Rate of elimination is directly proportional • Rate of elimination is independent of plasma


to plasma concentration or the amount concentration (or amount in the body)
present. The higher the concentration,
the more rapid the elimination. For eg.,

• A constant fraction of the drug is eliminated • A constant amount of the drug is eliminated
per unit time per unit time.

• Graphically, first order elimination follows an • Graphically, zero order elimination follows a
exponential decay versus time. straight-line decay versus time.

• T½ always remain constant irrespective of dose • Drugs with zero-order of elimination have
• Most of the drugs in pharmacology follow no fixed half life.
first order kinetics Eg. : Ethanol, Phenytoin, Salicylates, Tolbutamide,
Theophylline , Warfarin, Dicumerol

27
1 Self Assessment & Review Phar macology
Pharmacology
Mixed order of Kinetics (or mixed order kinetics or saturation kinetics, Pseudo zero order).
Michaelis-Menten Kinetics : Some important drugs, like phenytoin, digoxin, warfarin, dicumarol, tolbutamide
and aspirin (higher doses) obey mixed order elimination kinetics.

16. Ans. is c i.e. Have increased baroreceptor sensitivity Ref. KDT 62, Katzung 10/e, p 984

• Baroreceptor sensitivity does not increase with age. On the contrary it tends to decrease with age. This
leads to impaired blood pressure response to standing and volume depletion. Therefore dosage should be
increased in this situation.
• The most important of these is the decrease in Renal function.

Changes related to aging that affect pharmacokinetics of drugs

Variable Young Adults (20-30yrs) Older Adults (60-80 yrs)


Body water (%of body weight) 61 53
Lean body mass (% of body weight) 19 12
Body Fat (% of body weight) 26-33 (Female) 38-45
18-20 (Male) 36-38
Serum albumin (g/dl) 4.7 3.8
Kidney weight (% of young adult) (100) 80
Hepatic blood flow (% of young adult) (100) 55-60

17. Ans. is b i.e. Binds to receptor and causes opposite action


Ref. KDT 6/e, p 42

Inverse Agonist (Negative Antagonists)


These have full affinity towards the receptor, but their intrinsic activity ranges between ‘zero to minus one’. They
stabilise the receptor from undergoing the productive conformational change. Consequently, they will produce
an effect opposite to that of an agonist even in its absence.
For Eg., β-carbolines act as a inverse agoinsts at benzodiazepine receptor and produce the effects like, anxiety,
awakening and seizures, which are just the opposite of the effects of benzodiazepines (antianxiety, sedation,
anticonvulsant).

28
General Phar macology
Pharmacology 1
The Two State Receptor Model Theory

18. Ans. is a i.e. Adenosine deaminase deficiency Ref. KDT 6/e, p 63 - 64

Pharmacogenetic is the study of the genetic basis for variation in drug response. Pharmacogenetics
encompasses pharmacogenomic, which employs tools for surveying the entire genome to assess multigenic
determinants of drug response.

Examples :
• Atypical Pseudocholinesterase
– Prolonged succinylcholine Apnoea.
• G-6 PD deficiency
– Hemolysis with primaquine and other oxidizing drugs like Sulfonamides, Dapsone, Quinine, Chloroquine,
Nalidixic acid, Nitrofurantoin and menadione.
• Acetylators polymorphism
– Isoniazid neuropathy, Procainamide and hydralazine induced lupus in slow acetylators.
• Acute intermittent porphyria
– Precipitated by barbiturates due to genetic defect in repression of porphyrin synthesis.
• CYP2D6 abnormality causes poor metoprolol / debrisoquin metaboliser status.
• Malignant hyperthermia after halothane.
• Inability to hydroxylate phenytoin toxicity at usual doses.

29
1 Self Assessment & Review Phar macology
Pharmacology
• Resistance to coumarin anticoagulants due to an abnormal enzyme (that regenerates the reduced form of
Vit. K) which has low affinity for the coumarins.
• Precipitation of an attack of angle closure glaucoma by mydriatics in individuals with narrow iridocorneal
angle.

19. Ans. is d i.e. Binding sites are non-specific and one drug can displace the other
Ref. KDT 6/e, p 21; Goodman & Gilman 11/e, p 8

Considering each option separately :


Option ‘a’ : As concentration of drug increases over the therapeutic range, it is the free form of the drug that
increases (because most protein binding sites are already saturated).

Option ‘b’ : Protein binding of drug affects : Metabolism of drug and Elimination of drug.
Protein binding limits the glomerular filtration of drug but dose not affect the renal tubular
secretion. Protein bounded drugs excreat via renal by Tubular secretion.

Option ‘c’ : Plasma protein binding of drug


Albumin α−
−1 acid glycoprotein
• All the acidic drugs bind to • All the basic drugs bind to α−1
plasma protein Albumin acid glycoprotein
• Ex. : Valproic acid, Phenytion, Barbiturates • Ex. : Beta Blocker, Prazosin, Bupivacaine
Benzodiazipines, Sulphonamides, Lignocaine, Verapamil, Disopyramide
Tetracycline, Tolbutamide, Warfarin NSAIDs Imipramine, Methadone

Option ‘d’ : Binding sites are nonspecific and one drug can displace the other at therapeutic plasma
concentration. Only a tiny fraction is unbounded. Sulphonamides are an exception because
they occupy 50% of binding sites at therapeutic concentration and so can cause unexpected
effect by displacing other drugs. ... KDT 6/e, p 684

20. Ans. is a i.e. It is genetically predetermined


Ref. Katzung 10/e, p 975; KDT 6/e, p 84 - 85

Defining a teratogen :
• Results in a characteristic set of malformations, indicating selectivity for certain target organs.
• Exert its effects at a particular stage of foetal development i.e. during the limited period of organogenesis of
the target organs.

Drugs can affect the foetus at 3 stages :


• Fertilization and implantation - conception to 17 days, failure of pregnancy which often goes unnoticed.
• Orgamogenesis - 18 to 55 days of gestation, most vulnerable period, deformities are produced.
• Growth and development - 56 days onwards, development and functional abnormalities can occur, e.g.,
ACE inhibitors can cause hypoplasia of organs, specially lungs and kidneys; NSAIDs may induce premature
closure of ductus arteriosus.

30
General Phar macology
Pharmacology 1
21. Ans. is d i.e. Ketoconazole Ref. Katzung 10/e, p 61 - 62, ; KDT = 892 - 894

Drugs that enhance drug metabolism in humans.


Inducer Drug whose metabolism is enhanced
Benzo[a]pyrene Theophylline
Carbamazepine Carbamazepine, clonazepam, itraconazole
Glutethimide Antipyrine, glutethimide, warfarin
Griseofulvin Warfarin
Phenobarbital & Barbiturates, chloramphenicol, chlorpromazine, cortisol, coumarin anticoagulants,
other barbiturates digitoxin, doxorubicin, itraconazole, phenylbutazone, phenytoin, quinine, testosterone
Phenylbutazone Aminopyrine, cortisol, digitoxin
Phenytoin Cortisol, dexamethasone, digitoxin, itraconazole, theophylline
Rifampin Saquinavir Coumarin, digitoxin, glucocorticoids, itraconazole, oral contraceptive
Ritonavir Midazolam
St. John’s wort Alprazolam, cyclosporine, digoxin, indinavir, oral contraceptives, ritonavir, simvastatin,
tacrolimus, warfarin

Drugs that inhibit drug metabolism in humans.


Inhibitor Drug whose metabolism is inhibited

Ketoconazole Astemizole, cyclosporine, terfenadine


Allopurinol, chloramphenicol, Antipyrine, dicumarol, probenecid, tolbutamide
Isoniazid
Chlorpromazine Propranolol
Cimetidine Chlordiazepoxide, diazepam, warfarin
Dicumarol Phenytoin
Disulfiram Antipyrine, ethanol, phenytoin, warfarin
Grapefruit juice Alprazolam atorvastatin, cisapride, cyclosporine
Oral contraceptives Antipyrine
Phenylbutazone Phenytoin, tolbutamide
Ritonavir Amiodarone, cisapride, itraconazole, midazolam, triazolam
Saquinavir Cisapride, ergot derivatives, midazolam, triazolam
Secobarbital Secobarbital
Spironolactone Digoxin
Itraconazole Alfenatanil, astemizole, buspirone, cisapride,
cyclosporine, delavirdine, diazepam, digoxin, felodipine, loratidine,
phenytoin, quinidine, indinavir
Proteas inhibiters, sildenafil, statins, sirolimus, tacrolimus,
verapamil, warfarin

31
1 Self Assessment & Review Phar macology
Pharmacology
22. Ans. is a i.e. Tetracycline Ref. KDT 6/e, p 714

Tetracycline causes temporary separation of bone growth when given during late pregnancy. So it is
contraindicated in pregnancy.
• Chloroquine is antimalarial of choice in pregnancy.
• Erythromycin and Ampicillin : there is no teratogenicity seen with these drugs.

Mnemonic : TEtracycline is a TEratogen that causes staining of TEeth in the newborn.

23. Ans. is d i.e. Metoclopropamide Ref. KDT 6/e, p 25

Drugs metabolised by Acetylation : N-Acetyl Conjugations


All the drugs having amino or hydrazine residues e.g. :
• Hydralazine • Histamine
• INH (Isoniazid) • Procainamide
• PAS • Sulphonamides
• Dapsone

24. Ans. is a i.e. Enalapril Ref. Goodman & Gilman 11/e, p 1482; KDT 6/e, p 23

ALL ACE inhibitors are Prodrugs except : C – Captopril


L – Lisinopril
Prodrug - Few drugs are inactive as such and need conversion in the body to one or more active metabolites
such a drug are called prodrugs list given in answer 7 of all india 07

25. Ans. is a i.e. Volume of distribution Ref. KDT 6/e, p 34

Loading dose (or priming dose) is given to reduce the time needed to reach the steady state plasma
concentration. The loading dose can be calculated by the following formula :
Loading dose = Desired plasma conc. (mg/L) x aVd (L/kg body weight)
So loading directly proportional to aVd ie volume of distribution
Since five half lives are needed to reach the steady state plasma concentration, several days would be wasted
in obtaining the desired therapeutic effect. So, if there is a clinical emergency, like congestive heart failure with
atrial fibrillation or hyperpyrexia due to malaria, we use drugs in loading dose to reach steady state in single
short which is depicted in the figure below.

32
General Phar macology
Pharmacology 1
26. Ans. is a i.e. Ketoconazole Ref. Katzung 10/e, p 61 - 62

Already explained, refer answer no. 21

27. Ans. is b i.e. Metformin Ref. KDT 6/e, p 35

Therapeutic drug monitoring is defined as the science that combines the measurement of serum drug
concentrations with respect to clinical pharmacokinetics and pharmacodynamics.

Therapeutic drug monitoring (TDM) is particularly useful in the following situations :


• Drugs with low safety margin – digoxin, anticonvulsants, antiarrhythmics, theophylline, aminoglycoside
antibiotics, lithium, and tricyclic antidepressants.
• If individual variations are large – antidepressants, lithium.
• Potentially toxic drugs used in the presence of renal failure – aminoglycoside antibiotics, vancomycin.
• In case of poisoning.
• In case of failure of response without any apparent reason – antimicrobials.
• The check patient compliance – psychopharmacological agents.

Commonly monitored drugs


Aminoglycosides Antiarrhythmics Antiepileptic Antidepressants Antipsychotics Others

Amikacin Amiodarone Carbamazepine Amitriptyline Haloperidol Digoxin


Gentamicin Disopyramide Clonazepam Imipramine Lithium Salicylates
Netilmicin Flecainide Ethosuximide Nortriptyline Theophylline
Tobramycin Lidocaine Phenobarbitol Cylosporin
Mexiletine Phenytoin Vancomycin
Procainamide Valproic acid Tacrolimus
Quinidine
Sotalol

28. Ans. is a i.e. Preclinical studies Ref. KDT 6/e, p 76 - 77

Good clinical practice is a set of guidelines for biomedical studies which encompasses the design,
conduct, termination audit, analysis, reporting & documentation of the studies involving human subjects.
For preclinical studies subjects are animals & the that according to GLP Good Laboratory Practice during
which a wide range of non human studies e.g. toxicity testing pharmacokinetics analysis, formulation, etc are
performed.

29. Ans. is b i.e. Labetalol Ref. KDT 6/e, p 909 - 910

Choice of drugs for common problems during pregnancy.


Drug class Unsafe Safe

Antihypertensives ACE inhibitors (X), Angiotensin antagonists (X), Methyldopa, Hydralazine, Atenolol,
Thiazide diuretics, Furosemide, Propranolol, Metoprolol, Pindolol, Nifedipine,
Nitroprusside Prazosin, Clonidine labetalol

33
1 Self Assessment & Review Phar macology
Pharmacology

Antibacterials Cotrimoxazole, Fluoroquinolones (X), Tetracycline Penicillin G, Ampicillin, Amoxicillin-


(X), Doxycycline (X), Chloramphenicol (X), Clavulanate, Cloxacillin, Pipera-
Gentamicin, Streptomycin (X), Kanamycin (X), cillin, Cephalosporins, Erythro-
Tobramycin (X), Clarithromycin, Azithromycin, mycin
Clindamycin, Vancomycin, Nitrofurantoin
Ant itubercular Pyrazinamide, Ethambutol Streptomycin (X) Isoniazid, Rifampicin
Antiamoebic Tinidazole (X), Quiniodochlor Metronidazole
Antimalarial Quinine (X), Mefloquine Pyrimethamine + Chloroquine, Proguanil
sulfadoxine (X), Artemether, Artesunate,
Primaquine (X)
Antiretrovial Didanosine, Abacavir, Indinavir, Ritonavir, Zidovudine, Lamivudine, Nevira-
Efavirenz pine, Nelfinavir, Saquinavir
Antidiabetics Sulfonylureas (X), Metformin (X), Pioglitazone, Insulin (preferably human insulin)
Rosiglitazone, Repaglinide, Nateglindie,
Acarbose (X)
Corticosteroids Betamethasone, Dexamethasone (high dose Inhaled corticosteroids, Topical
& prolonged use corticosteroids, Prednisolone oral
(low dose)
Antithyroid drugs Carbimazole, Radioactive iodine (X), Iodide Propylthiouracil
Anticoagulants Warfarin (X), Acenocoumarol, Phenindione (X) Heparin (unfractionated)
Heparin (LMW)

Drugs marked (X) are contraindicated during pregnancy.

30. Ans. is a i.e. ACE inhibitor Ref. KDT 6/e, p 909 - 910; Katzung 10/e, p 975

Antihyperensive avoided in pregnancy are :


• Diuretics
• ACE inhibitors, causes renal damage
• Reserpine
• Nonselective β blockers
• Sodium nitroprusside

For more details, refer answer no. 29

31. Ans. is b i.e. Cleft lip and cleft palate Ref. Drugs in Pregnancy & Lactation, KDT 85, 6/e, p 888

Foetal aplasia cutis caused by carbimazole teratogenecity.


Congenital malformations secondary to exposure to carbimazole during gestation are :
• Scalp or patchy hair defect • Choanal atresia
• Esophageal atresia • Tracheo-esophageal fistula
• Minor facial anomalies • Hypoplastic or absent phalanges
• Psychomotor delay

34
General Phar macology
Pharmacology 1
Drugs and their congenital malformation when taken in pregnancy are given below :
Drugs Congenital malformation

• Penicillamine • Cutis lexa


• Warfarin • Contradi syndrome
• Chlorambucil • Genitourinary anomaly (Renal agenesis)
• Valproic acid • Neural tube defect
• Carbamezapine • Cleft lip and palate, Cardiac defect
• Costicosteroid • Congenital heart disease, cleft lip and palate
• Barbiturates • Respiratory depression
• Carbimazole • Foetal Aplasia Cutis
• Lithium • Ebstein anomaly, Foetal goiter.
• DES • Vaginal adenosis (Clear cell Ca)
• Thalidomide • Phocomelia
• Chloramphenicol • Grey baby syndrome
• Androgens • Virilization; Limb, oesophagus and cardiac defects
• Isotretinoin • Craniofacial defect, Heart anomaly, Abortion
• Indomethacin • Premature closure of DA
• Tamoxifen • Risk of Abortion and Foetal damage

32. Ans. is c i.e. Volume of distribution Ref. Goodman & Gilman 11/e, p 730; KDT 6/e, p 18 - 19

When the time to reach steady state is appreciable, as it is for drugs with long half-lives, it may be
desirable to administer a loading dose that promptly raises the concentration of drug in plasma to the
target concentration.

Loadign dose = Desired plasma concentration X Volume of distribution

Volume of drug distribution depends on :


• Lipid solubility • Ionisation at physiological pH (dependent on pka).
• Differences in regional blood flow. • Affinity for different tissues.
• Fat : lean body mass ratio. • Diseases like CHF, uremia, cirrhosis.
• Extent of plasma and tissue protein binding.

33. Ans. is b i.e. Enalapril Ref. KDT 6/e, p 23

All ACE inhibitor are prodrug except Captopril and Lasinopril.


For more details, refer answer no. 7

34. Ans. is b i.e. Metoclopramide Ref. KDT 6/e, p 25

Drugs metabolised by Acetylation ( N-Acetyl Conjugations ) are the drugs having amino or hydrazine
residues e.g. :
• Hydralazine • Histamine
• INH (Isoniazid) • Procainamide
• PAS • Sulphonamides
• Dapsone

35
1 Self Assessment & Review Phar macology
Pharmacology
35. Ans. is a i.e. Specificity Ref. Lawrence 9/e, p 93

Side effects of a drug arise due to the interactions of the drug molecules other than the target can be
minimized by making the drug more specific eg., as we used cardioselective β−blocker have less side
effect on respiratory system so side effects will be minimized by this process.
Lawrence States
“The pharmacologist who pruduces a new drug and the doctor who gives it to a patient share the desire
that it should possess a selective action, so that additional and unwanted adverse effects do not complicate
the management of the patient”
Affinity Will enable drug to be used in low concentrations
Solubilty & It will determine drugs capability to cross barriers in the
Hydrophobicity body and act on target organ

36. Ans. is a i.e. High affinity Ref. Rang & Dale 5/e, p 11, KDT 6/e, p 92

Affinity is the ability of the drug to bind with the receptor and is determined by the interaction of the
molecular structures of the receptor and the drug.

Affinity
• Affinity relates the drug concentration to the functional receptor occupancy.
• The receptor occupancy by the drug is needed to elicit a response, so drug with hyper affinity will occupy
larger number of receptor and produce more response at a given concentration than drug with lower affinity.
• The drug which has more affinity will occupy more receptor at a given concentration and the drug which has
low affinity will occupy lower number of receptor at a given concentration.

Efficacy (Intrinsic activity)


• It is the ability of the drug to activate (induce a conformational change in) the receptor consequent to
receptor occupation.
• The two properties of affinity and intrinsic activity (IA) are independently variable, i.e. drugs with the same
affinity can possess different degrees of intrinsic activity.

37. Ans. is c i.e. Heparin Ref. Katzung 9/e, p 995

• Heparin is a very large polar molecule, so unable to cross the placenta It is the drug of choice for the
management and prophylaxis of venous thromboembolism in pregnancy
Drugs SAFE & UNSAFE in pregnancy Already explained, refer answer no. 29
Placental drug transport depends upon
Lipid solubility :
• Unionized drug transportation > Ionized drug like thiopentone > Highly ionized drugs (SCh, d-tubocurarine).
• Exception - Salicylates are almost completely ionized at physiological pH, even then foetal level is high
because the small amount of salicylate which is not ionized is highly lipid soluble.

Molecular Size :
• Molecular size :
– 250-500 - Cross placenta easily, depending upon their lipid solubility and degree of ionization

36
General Phar macology
Pharmacology 1
– 500-1000 - Cross placenta more difficulty
– > 1000 - Cross very poorly

Membrane Transporters :

• P- glycoprotein transporter pumps back into the maternal circulation a variety of drugs, including cancer
drugs (vinblastine, doxorubicin) and other agent (digoxin). Inhibition of this transporter may cause drug
accumulation in the fetus.

Protein Binding, Placental and Foetal drug metabolism

38. Ans. is c i.e. The substrate concentration at half maximal velocity


Ref. Lippincot’s Biochem 2/e, p 53

• Km is Michaleis-Menten constant. It is the characteristic of an enzyme and a particular substrate


and it reflects the affinity of the enzyme for that substrate.
• Km is numerically equal to the substrate concentration at which the reaction velocity is equal to ½ Vmax.
• Numerically, small Km reflects a high affinity of the enzyme for substrate because a low concentration of the
substrate is needed to half saturate the enzyme.
• A numerically large Km reflects a low affinity of enzyme for substrate because a high concentration of
substrate is needed to half saturate the enzyme.
• The rate of the reaction is directly proportional to the enzyme concentration at all substrate concentration.
• First order reaction – when substrate concentration is much less than Km, the velocity of the reaction is
roughly proportional to the substrate concentration. The rate of the reaction is then said to be of first order.
• Zero order reaction – When substrate concentration is much greater than Km, the velocity is constant and
equal to Vmax. The rate of reaction is then independent of substrate concentration and said to be of the zero
order.

39. Ans. is b i.e. Half life Ref. Lippincott’s Pharmacology 2/e, p 18; KDT 6/e, p 34

For all drugs with first order kinetics, the time required to achieve steady-state levels can be predicted
from the half life. The steady state is reached in 3-5 half lives unless dose interval is very much longer
than t½.
As regarding other options affecting steady state level
Clearance
• Clearance affects the steady state plasma concentration (Css) of the drug. There is no effect on the time
required to achieve the steady level.
• Clearance is inversely proportional to the steady state concentration (Css).

Volume of distribution

• Volume of distribution is inversely proportional to Css. There is no relation to the time required to achieve the
level.
• If the volume of distribution is increased, the Css level falls.
Rate of drug infusion
• If rate of drug infusion is increased, the Css increases but the time required to achieve the steady state level
remains the same.

37
1 Self Assessment & Review Phar macology
Pharmacology
The following figure shows the rate of attainment of steady state concentration of drug in plasma and the rate of
wash-out of drug when the drug infusion is stopped.

40. Ans. is c i.e. Ketoconazole Ref. KDT 6/e, p 25

Drugs metabolised by Acetylation ( N-Acetyl Conjugations ) are the drugs having amino or hydrazine
residues e.g. :
• Hydralazine • Histamine
• INH (Isoniazid) • Procainamide
• PAS • Sulphonamides
• Dapsone

Synthetic / Phase - II Biotransformation reactions :


Glucoronide conjugation : Microsomal conjugation

• All compounds having hydroxyl or carboxylic acid groups :


– Chloramphenicol – Aspirin
– Phenacetin – Morphine
– Metronidazole – Endogenous substrates : - Bilirubin - Thyroxine

Non-Microsomal conjugation
• Glycine conjugation :
– Salicylates – Other drugs having carboxylic acid group
• Glutathione conjugation :
It inactivate highly reactive quinone or epoxide intermediates formed during metabolism of
certain drugs, e.g.
– Paracetamol

38
General Phar macology
Pharmacology 1
• Methylation :
Amines and phenols are methylated e.g.
– Histamine – Epinephrine – Nicotinic acid
Mnemonic : HEN - Meat
• Sulphate Conjugation :
Phenolic compounds and steroids are sulphated e.g.
– Chloramphenicol – Adrenal and Sex steroids

41. Ans. is a i.e. Massive lymphocytic apoptosis Ref. Goodman & Gilman 11/e, p 1599
Glucoconticoids Limited the recruitment of inflaumatory cells at the loca site KDT 6/e, p 278-79

Action of Glucocorticoids
Lymphoid malignancies Activation of programmed cell death (apoptosis) in certain lymphoid tissues
Normal tissues Redistribution of lymphocytes, eosinophils, monocytes and basophils away from
the periphery. It leads to decrease in these cell counts within 4-6 hours.
Increased release of neutrophils from marrow diminished rate of removal from the
circulation and increased demargination from vascular walls.

42. μm
Ans. is a i.e. 2xμ Ref. Lippincot’s Biochem 2/e, p 53, KDT 6/e, p 32-33

• Km is Michaleis Menten constant. It is numerically equal to the substrate concentration at which the reaction
velocity is equal to ½ Vmax.
• Now, in the question,
Km = xμm and
Km is the substrate concentration at which reaction velocity is ½ Vmax.
So, the maximum activity of the enzyme would be at a substrate concentration of 2xμm.

43. Ans. is c i.e. Verapamil Ref. Goodman & Gilman 11/e, p 949; KDT 6/e, p 529

• Goodman and Gillman describe Verapamil as :


– Verapamil is prescribed as a racemate
– “L-Verapamil is a more potent calcium channel blocker than d-verapamil.”
– However, with oral therapy, the l-enantiomer undergoes more extensive first pass hepatic metabolism.

• For this reason, a given concentration of verapamil prolongs the PR interval to a greater extent when the drug
is administered intravenously (where conc. of l- and d- enantiomers are equivalent) that when it is administered
orally.

44. Ans. is d i.e. Decreased volume of distribution Ref. KDT 6/e, p 62

There is increased volume of distribution for fat soluble drugs in elderly individuals. This results in
decreased elimination of the drug which leads to increased toxicity.

39
1 Self Assessment & Review Phar macology
Pharmacology

Other important changes seen in elderly which lead to increased toxicity of drugs

• Decreased renal excretion of drugs • Decreased lean body mass


• Decreased hepatic metabolism of drugs • Decreased total body water
• Increased receptor sensitivity in the target organs • Increase in body fat
• Decrease plasma albumin • Increase in α-1 acid glycoprotein
• Decrease GFR • Decrease Tubular secretion

45. Ans. is c i.e. Ferrous sulphate + Tetracycline Ref. KDT 6/e, p 712

• Tetracyclines have chelating property – forms insoluble and unabsorbable complexes with calcium and
other metals. Milk, iron preparations, nonsystemic antacids and sucralfate reduce their absorption.
Administration of these substances and tetracyclines should be staggered.
• Now looking at the other options :
About clonidine + chlorpromazine : Chlorpromazine abolishes the antihypertensive action of clonidine
probably by blocking the α (alpha) receptors on which it acts.
About Gentamicin + Furosemide : High ceiling diuretics (Furosemide) and Aminoglycoside antibiotics are
both ototoxic, produce additive toxicity should not be used together.
Levodopa and Metoclopramide : Metoclopramide reverse the therapeutic effect of Levodopa by blocking
DA receptors.

46. Ans. is a i.e. Brachycephaly Ref. Katzung 9/e, p 372; KDT 6/e, p 85

Alcohol intake during pregnancy cause Foetal Alcohol syndrome.


The abnormalities that have been characterized as Foetal Alcohol syndrome include :
• Intrauterine growth retardation • Microcephaly • Poor coordination
• Minor joint anomalies • Hyperkinetic movements
• Underdevelopment of midfacial region • Cardiac malformation (ASD, PDA)

“Alcohol-related neurodevelopmental disorder” may also occur and its incidence is about three times
more than foetal alcohol syndrome. it is characterised by behavioural as well as cognitive and motor deficits.
This ethanol then triggers apoptotic neurodegeneration in CNS. Alcohol is easily secreted through breast milk
which may delay motor development in the child

47. Ans. is b i.e. Molecular modelling


Ref. http://www.netsci.org/science/compchem/feature01.html; http://pubs.acs.org/subscribe/journals/mdd/v03/
i09/html/felton.html

• Molecular modelling allows new drug designing and development by the process of conceptualising for the
performance of required function based on essential characteristics (pharmacopores), including idealized
structural and physical properties.
• Molecular modelling / computational chemistry is the science or art of representing molecular structure
numerically and simulating their behaviour with the equation of quantum and classical physics.

40
General Phar macology
Pharmacology 1
• The development of molecular modelling programs and their application in pharmaceutic research has been
formalised as a field of study known as computer assisted drug design (CADD) or computer assisted
molecular design (CAMD).

48. Ans. is a i.e. 5 litre Ref. KDT 6/e, p 19

Dose administered IV
Volume of distribution (V) =
Plasma concentration

200 mg
V =
40 μg/ml
200 10-3 gm
= = 5 x 103 ml = 5000 ml
40 10 -6 gm

49. Ans. is d i.e. Propranolol Ref. Harrison 17/e, p 657

Drugs that carry risk of clinical hemolysis in persons with G6PD deficiency.
Groups Definite risk Possible risk Doubtful risk
Antimalarials Primaquine Chloroquine Quinine
Dapsone/chlorproguanil
Sulphonamidex/sulphones Sulphametoxazole Sulfasalazine Sulfisoxazole
Others Sulfadimidine Sulfadiazine
Dapsone
Antibacterial/antibiotics Cotrimoxazole Ciprofloxacin Chloramphenical
Nalidixic acid Norfloxacin p-Aminosalicylic acid
Nitrofurantoin
Niridazole
Antipyretic/analgesics Acentanilide Acetylsalicylicacid Acetylsalicylic acid <3 g/d
Phenazopyridine high dose (3 g/d) Acetaminophen
(Pyridium) Phenacetin
Other Naphthalene Vitamin K analogues Doxorubicin
Methylene blue Ascorbic acid >1g Probenecid
Rasburicase

50. Ans. is a i.e. Drug X will be available more in tissue Ref. KDT 6/e, p 20

In the question drug X has 150 times lower affinity to bind with albumin than drug Y and it is clearly
given in the book that highly plasma protein bound drugs are largely restricted to the vascular
compartment and tend to have lower volume of distribution.
• Option ‘c’ is confusing but the fact is that free concentration of the drug does not depend upon the plasma
protein binding.

41
1 Self Assessment & Review Phar macology
Pharmacology
Other important aspects of plasma protein binding :
• The bound fraction is not available for action. However, it is in equilibrium with the free drug in plasma and
dissociates when the concentration of the latter is reduced due to elimination. Plasma protein binding thus
implies to temporary storage of the drug.
• High degree of protein binding generally makes the drug long acting, because bound fraction is not available
for metabolism or excretion, unless it is actively extracted by liver or kidney tubules.
• Generally expressed plasma concentrations of the drug refer to bound as well as free drug.
• One drug can bind to may sites on the albumin molecule. Conversely, more than one drug can bind to the
same site. This can give rise to displacement interactions among drugs bound to the same site.
• In nephrotic syndrome, binding may be reduced and high concentrations of free drug may be attained, e.g.
phenytoin and furosemide.

51. Ans. is a i.e. Penicillin G Ref. KDT 6/e, p 696

• Penicillin G is metabolized mainly in kidney Pharmacokinetics of PnG is dominated by very rapid renal
excretion (10%), by glomerular filtration and rest by tubular secretion (90%).
• Rest of the drugs given in the question erythromycin cimetidine and phenytoin are metabolized in the liver.

52. Ans. is d i.e. Ketoconazole Ref. KDT 6/e, p 27 - 28

Ketoconazole is microsomal enzyme inhibitors (not inducer).

Microsomal enzyme Induction : • Anticonvulsant – Phenobarbitone


– Phenytoin
– Glucocorticoids
• Antitubercular – Rifampicin
– Isoniazid
• Steroid – Glucocorticoids
• Others – Chloral hydrate
– Phenylbutazone
– Griseofulvin
– DDT

53. Ans. is d i.e. Adrenaline and Histamine Ref. KDT 6/e, p 56

Physiological / functional antagonism


The two drugs act on different receptors or by different mechanisms, but have opposite overt effects on the
same physiological function, i.e. has pharmacological effects in opposite direction, e.g.
• Histamine and adrenaline on bronchial muscles and BP.
• Hydrochlorothiazide and triamterene on urinary K excretion.
+

• Glucagon and insulin on blood sugar level.

Other types of Antagonism


• Physical Antagonism : Universal antidote

42
General Phar macology
Pharmacology 1
• Chemical Antagonism : NaHCO + HCI, Heavy metal + chelating agent
3
• Pharmaceutical Interaction : Occurs outside the body to drug with other agent
Eg: IV phenytoin glucose bolus (Precipitation)
• Pharmacokinetic antagonism : Enzyme inducers decreasing activation of substrate

54. Ans. is a, b, c and d i.e. All are correct options Ref. KDT 6/e, p 83

Drug dependence is a state in which use of drugs for personal satisfaction is accorded a higher priority than
other basic needs, often in the face of known risks to health. Dependency includes both addiction and habituation.
• Reinforcement is the ability of the drug to produce effects that make the user wish to take it again or to
induce drug seeing behaviour. The frequency of use of drugs is usually daily and duration is inevitably greater
than 2-3 weeks. (Option “a”)
• Discontinuation of the drug results in a characteristic withdrawal (abstinence) syndrome. (Option “c”)

55. Ans. is a i.e. Cyclophosphamide Ref. Goodman & Gilman 11/e, p 1482; KDT 6/e, p 24

Prodrug
• Few drugs are inactive as such and need conversion in the body to one or more active metabolites such a
drug is called prodrugs.
• The prodrug may offer advantages over the active form in being :
– More stable
– Having better bioavailability
– Other desirable pharmacokinetic properties
– Less side effects and toxicity.
Prodrug Active form Prodrug Active form
Azathioprine Mercaptopurine Dipivefrine Epinephrine
Bacampicillin Ampicillin Enalapril Enalaprilat
Benyrolate Aspirin + Paracetamol Levodopa Dopamine
Cortisone Hydrocortisone Proguanil Proguanil triazine
Cyclophosphamide Aldophosphamide Sulindac Sulfide metabolite
Sulfasalazine 5 Aminosalicylic acid Zidovudine (All NRTI’s) Zidovudine triphosphate
---
Mercaptopurine Methylmercaptopurine Flourouracil Flourouridine mono PO
4
Prednisone Prednisolone
ALL ACE inhibitors are Prodrugs except : C – Captopril
L – Lisinopril

56. Ans. is b, c and d i.e. Phenytoin; Cyclosporin; and Digitalis Ref. Katzung 10/e, p 377; KDT 6/e, p 405

The therapeutic plasma level of phenytoin for most patient is between 10 and 20 μg/ml.

Therapeutic level of some important drugs characterized by their low therapeutic index.
• Digoxin – 0.8 - 2 ng/ml • Clonidine – 0.2 - 2.0 ng/ml
• Phenytoin – 10-20 μg/ml • Lithium – 0.6 - 1.2 mEq/L (for prophylaxis)
• Primidone – 10-40μg/ml – 0.8 - 1.2 mEq/L (for maintenance)

43
1 Self Assessment & Review Phar macology
Pharmacology

• Theophyline – 5 - 20 μg/ml • Quinidine – 2 - 6 μg/ml


• Valproic acid – 50 - 100 μg/ml

Mnemonic : Therapeutic dosage - toxicity values for most commonly monitored medications.
“The magic 2s”-
Digitalis (.5 - 1.5) toxicity = 2
Lithium (.6 - 1.2) toxicity = 2
Theophylline (10 - 20) toxicity = 20
Dilantin (10 - 20) toxicity = 20

57. Ans. is a and b i.e. Primaquine and Chloroquine Ref. Harrison 17/e, p 657

Drugs that carry risk of clinical hemolysis in persons with G6PD deficiency.
Groups Definite risk Possible risk Doubtful risk
Antimalarials Primaquine Chloroquine Quinine
Dapsone/chlorproguanil
Sulphonamidex/sulphones Sulphametoxazole Sulfasalazine Sulfisoxazole
Others Sulfadimidine Sulfadiazine
Dapsone
Antibacterial/antibiotics Cotrimoxazole Ciprofloxacin Chloramphenical
Nalidixic acid Norfloxacin p-Aminosalicylic acid
Nitrofurantoin
Niridazole
Antipyretic/analgesics Acentanilide Acetylsalicylicacid Acetylsalicylic acid <3 g/d
Phenazopyridine high dose (3 g/d) Acetaminophen
(Pyridium) Phenacetin
Other Naphthalene Vitamin K analogues Doxorubicin
Methylene blue Ascorbic acid >1g Probenecid
Rasburicase

58. Ans. is a, b, c and d i.e. Decrease cost; Increase efficacy; Increase compliance; and Decrease resistance
Ref. KDT 6/e, p 60

Advantage of fixed dose combination


• Convenience and better patient compliance. It is cost saving compared to both / all the components
administered separately.
• Certain drugs combination are synergistic e.g., sulfamethoxazole + trimethoprim; levodopa + carbidopa /
benserazide; combination oral contraceptives.
• The therapeutic effect of two components being same may add up while the side effect being different may
not add up e.g. amlodipine + atenolol.
• The side effect of one component may be counteracted by other, e.g., thaizide + K sparing diuretics.
+

• Combination drugs may decrease resistant to microbiological agents e.g. 1 + IV and tuberculosis.

44
General Phar macology
Pharmacology 1
Disadvantage of fixed dose combination
• The patient may not actually need all the drugs present in a combination. The patient is subjected to
additional side effect and expense.
• The dose of the most drugs needs to be adjusted as individualized; when a combined formulation is used,
thus can not be done without alteration of the dose of the components.
• The time course of action of the components may be different administering them at the same interval may
be inappropriate.
• Altered renal or hepatic function of the patient may differently affect the pharmacokinetics of the components.
• Adverse effect, when it occurs, can not easily ascribe to the particular drug causing it.
• Contraindication to one component (allergy / other conditions) contraindicates the whole preparation.
• Confusion of therapeutic aims and false sense of superiority of two drugs over is fostered, especially in case
of antimicrobials whose combinations should be avoided. Corticosteroids should never be combined with
any other meant for internal use.

59. Ans. is a and c i.e. Enalapril; and Angiotensin receptor blocker


Ref. Katzung 10/e, p 975

Enalapril and Angiotensin receptor blocker causes Renal damage when given during all course of
pregnancy.
Drugs safe and unsafe during pregnancy given in Ans. 29.

60. Ans. is a, b, c and d i.e. Lipid solubility; Volume of distribution; Clearance; and Drug concentration
Ref. KDT 6/e, p 31 - 32; Katzung 10/e, p 35

Clearance, the measure of the ability of the body to eliminate the drug.
Elimination of drug depends upon : – Bioavailability – Volume of distribution
– Clearance
• Drug injected intravenously are completely absorbed and rapidly distributed, as they reach the blood stream
directly without crossing any membrane.
• Absorption occurs by passive diffusion from the injection site to the plasma or lymph.
• Filtration through channels in the endothelial capillary membrane.

61. Ans. is e i.e. 0.2 liter/hr Ref. KDT 6/e, p 18, 30

Total dose of drug administered = 70 x 100 mg = 7000 mg


Plasma volume of distribution.
Total dose administred
So, Volume of distribution =
Plasma concentration
V
Again, t½ = 0.693 x where
CL
V = Volume of distribution
CL = Clearance
V 0.693 7000
So, clearance of the drug = 0.693 × 1/2 = = 255.3 ml = 0.2 L/hr
t 10 1.9

45
1 Self Assessment & Review Phar macology
Pharmacology
62. Ans. is d i.e. Indirect sympathomimetics involved
Ref. KDT 6/e, p 68, Goodman & Gilman’s 11/e, p 31, 162, 170

• Tachyphylaxis is acute development of tolerance after a rapid and repeated administration of a drug at
shorter intervals.
• Tachyphylaxis is seen with indirectly acting sympathomimetics like ephedrine, amphetamine and tyramine.
• These drugs act by releasing catecholamines (from the storage sites), the synthesis of which is unable to
match its release.

Reason : – There is gradual depletion of the agonist from the storage sites with no chances of its
replenishment because of the repeated administration of the drug at short intervals.
– Tachyphylaxis may also occur as a result of a change in the sensitivity of target cells
(pharmacodynamic reasons).

63. Ans. is a, b, c, d and e i.e. Active transport; Passive transport; Lipid solubility; Facilitated diffusion;
and Symport Ref. KDT 6/e, p 12

The major transport mechanism are as under :


• Passive Diffusion (Non-ionic or simple diffusion)
• Carrier Mediated Transport : – Facilitated diffusion / Symport
– Active transport
• Pinocytosis / Phagocytosis and Filtration.

Modes of absorption
a. Passive diffusion (Most common Mode)
• Passage though semi-permeable membrane
• Concentration dependent
• Does not involve a carrier
– It is not saturable
– Lipid soluble drugs readily penetrate the biological membranes
– Water-soluble drugs penetrate through the water channels
– The unionized moiety is lipid soluble. E.g. Weak organic bases / acids alcohol, urea, glycosides
– pKa of the drugs and pH of surrounding medium determines the degree of ionization and hence the
degree of absorption
b. Convective transport : Drug soluble dissolved in the aqueous medium move along with the solvent through
the pore. Since the diameter of the water filled pores is 7-10 A, only drugs having diameter <7A can pass
through. In general globular molecules having molecules wt of 150 can pass (or chain like e.g. organic &
inorganic electrolytes compounds having M.W < 400).
c. Active transport :
• Utilization of ATP and carrier molecule
• Drugs having higher affinity can displace the drug having lower affinity for it
• Drug can move against a concentration gradient
• The process shows saturation kinetics for the carrier
E.g. Monosaccharides / Amino Acids / cardiac glycosides / sex hormones

46
General Phar macology
Pharmacology 1
d. Facilitated transport :
• This process utilizes a carrier and no energy but does not move against a concentration gradient.
E.g. : Vit. B12 (+ intrinsic factor) ? absorption of B12

64. Ans. is a i.e. High first pass metabolism Ref. KDT 6/e, p 28 - 29; Katzung 10/e, p 40

• First pass (presystemic) metabolism refers to metabolism of a drug during its passage from the site of
absorption in to the systemic circulation.
• All orally administered drugs are exposed to drug metabolising enzymes in the intestinal wall and liver
(where they first reach through the portal vein.
For a drug administered orally the bioavailability is < 100% due to because, incomplete extent of absorption and
first pass metabolism.
Drugs may not be absorbed because of a reverse transporter associated with P-glycoprotein efflux pump.

65. Ans. is b, c, d and e i.e. Phenytoin; Carbamazepine; Rifampin; and Glucocorticoid


Ref. KDT 6/e, p 24

Cytochrome P-450-3A4 carry out biotransformation of largest number of drugs. It is found in liver, intestine and
kidney.

Inducers : • Carbamazepine • Glucocorticoids


• Rifampin • Macrolide antibiotics
• Phenytoin

For more details about List of Enzyme induces and Inhibitors, refer answer no .21

66. Ans. is a, b and c i.e. If drug is administered rectally it follow first order; If drug is administered IM it
follows zero order; and If drug is administered IV it follows first order
Ref. KDT 6/e, p 17 - 18, 28; Katzung 10/e, p 40

Bioavailability defined as the fraction of unchanged drug reaching the systemic circulation following
administration by any route.
For a drug administered orally bioavailability is < 100% due to incomplete extent of absorption and first pass
metabolism.
Drugs may not be absorbed because of a reverse transporter associated with P-glycoprotein.
Mode of administration Bioavailability
Oral Irregular (different brands of same drug
have different bioavailability)
Sublingual Relatively good
Rectal Relatively poor metabolized in liver
Inhalational Relatively good
Parenteral (IV, IM, S/C) -100%

Now come to pharmacokinetics


Zero order kinetics = Capacity limited = Saturable

47
1 Self Assessment & Review Phar macology
Pharmacology
• Rate of elimination remains constant irrespective of drug
• Clearance decrease with increase in dose. E.g., Ethanol, Methanol.
1 st
order kinetics = Linear kinetics
• Rate of elimination increase with increase in dose of drug
• Clearance is constant
Note : Few drugs, when given in high doses or in case of failure of excretion begin to show zero order
kinetics (pseudozero order kinetics) eg. Phenytoin, Tolbutamide, Theophylline, Warfarin, Clarithromycin,
Dicoumarol, Heparin, Salicylates.

67. Ans. is a and d i.e. Variability of enzyme action; and Different mechanism of action in different
individual
Ref. Katzung 10/e, p 58; KDT 6/e, KDT 6/e p 63

Cause of Variations in Action of Drugs :


• The variations are mainly because of different rates of drugs metabolism due to different enzyme levels
contributed by genetic factors.
• The dose of drug to produce the same affect may vary by 4 - 6 fold so dose response curve may be different.
• Example of genetically determined variations are :
– Slow and fast acetylators for isoniazid
– Atypical pseudocholinesterase resulting in prolongation of succinyl choline action.

68. Ans. is d i.e. Chlortetracycline + penicillin Ref. KDT 6/e, p 55, 685, 723

Drug Synergism : When the action of one drug is facilitated or increased by the other, they are said to be
synergistic. In a synergistic pair, both the drugs can have action in the same direction or
given alone one may be inactive but still enhance the action of the other when given
together.
Now, considering each option separately.
Option ‘a’ Flucytosine has supra-additive action with amphotericin-B (AMB) in the case of fungi sensitive
to both, e.g. Cryptococcoses, Coccidioidomycosis. (AMB increases the penetration of 5-FC
into the fungus).
Option ‘b’ Sulfamethoxazole was selected for combining with trimethoprim because both have nearly
the same t½ (∼ 10hr). Optimal synergy in case of most organisms is exhibited at a
concentration ratio of sulfamethoxazole 20 : trimethoprim 1, the minimum inhibitory
concentration (MIC) of each component may be reduced by 3-6 times.
Option ‘c’ Penicillin/ampicillin + streptomycin/gentamicin is combined. Penicillin disrupts the bacterial
cell wall, over come resistance.
Option ‘d’ This is the wrong answer. No synergism is reported for chlortetracycline + penicillin.

Other important antimicrobial combinations


• Carbenicillin/ticarcillin + gentamicin for pseudomonas infection, specially neutropenic patients.
• Ceftazidime + ciprofloxacin for pseudomonas infected orthopaedic prosthesis.
• Rifampicin + INH in tubercular infection.

48
General Phar macology
Pharmacology 1
69. Ans. is a, b and c i.e. Heparin; Pantoprazole; and Ranitidine
Ref. KDT 5/e, p 178, 562, 591, 593, 702

• Heparin is a large ionized molecule, therefore not absorbed orally. Injected I.V. it acts instantaneously but
after s/c injection, anti coagulant effect develops after 60min. It does not cross Blood brain barrier or placenta
It is the anticoagulant of choice during pregnancy.
• Pantoprazole is a newer H K ATPase inhibitor. It is the only proton pump inhibitor available for I/V
+ +

administration.
• Ranitidine is H blocker which can be given orally, I/M or I/V.
2

• Sumatriptan can be used by oral, nasal spray or subcutaneous route.


• Neomycin is highly toxic drug, can be used orally or topically.

70. Ans. is a, b and d i.e. Efficacy is clinically more important than potency; Height of DRC corres-ponds
to efficacy; and Drugs having similar pharmacological action may have different efficacy
Ref. Katzung 9/e, p 28

Potency Efficacy

• Potency refers to the amount of drug • Efficacy refers to the maximal response that can
needed to produce a certain response. be elicited by the drug. It refers to the maximum
height of the curve.
• Competitive antagonist acts by decreasing • Non competitive antagonist acts by decreasing
the potency of a drug, such that maximal the efficacy of a drug. Maximal response is
response (efficacy) can still be attained by suppressed and unsurmountable. Causes
increasing dose of agonist. Causes rightward flattening of DRC.
shift of DRC.
• Potency refers to the concentration (EC50) • The efficacy of a drug is obviously crucial for making
or dose (ED50) of a drug required to clinical decisions when a large response is needed.
produced 50% of that drugs maximal effect. It is a more decisive factor in the choice of a drug.
• Potency of a drug depends in part on the • It may be determined by the drug’s mode of
affinity of receptors for binding the drug and in interactions with receptors or by characteristics
part on the efficiency with which drug receptor of the receptor effector system involved.
interaction is coupled to response.

71. Ans. is b and c i.e. I.M. administration needs sterile technique; and I.D. injection produces local tissue
neurosis and irritation Ref. KDT 6/e, p 10

Let’s analyse each option one by one.


Option ‘a’ • Through i.v. route, the drug directly reaches the systemic circulation and effects are
produced immediately (great value in emergency).
• Bioavailability is 100%.
• The intima of veins is insensitive and drug gets diluted with blood, Therefore, even highly
irritant drugs can be injected I.V.
• Only aqueous solutions (not suspensions) can be injected I.V. and there are node pot
preparations for this route.

49
1 Self Assessment & Review Phar macology
Pharmacology

• Dose of the drug required is smallest.


Option ‘b’ & ‘c’ • Not only i.m., but all the parenteral routes such as subcutaneous (S.C.), intramuscular
(I.M.) intravenous (I.V.) and intradermal (I.D.) have the following disadvantages :
– Preparation and technique needs to be sterile.
– It is costlier.
– It is invasive, painful and can cause local tissue necrosis and irritation.
– Assistance of another person is mostly needed.
– There are chances of local tissue injury.
– It is in general, more risky.
Option ‘d’ • In inhalation, absorption takes place from the vast surface of alveoli - immediate
systemic bioavailability and very rapid action.
• Volatile liquids and gases are given by inhalation for systemic action e.g., amylnitrate.
• When administration is discontinued, the drug diffuses back and is rapidly eliminated
in expired air.

72. Ans. is a, c, d, and e i.e. Obesity; Pregnancy; Older age; and Neonate
Ref. KDT 6/e, p 19 - 20, 38; Katzung 10/e, p 40

Bioavailability is defined as the fraction of unchanged drug reaching the systemic circulation following
administration by any route.
• In obese patient, because of greater than normal adipose tissue content, V is increased Option ‘a’.
d

• In pregnancy, blood volume increased by 30-40%. Although total protein is increased, but plasma protein
concentration is decreased, thus altering V Option ‘c’.
d

• In elderly, V is more because of increased total body fat content and decreased plasma protein binding of
d

drugs Option ‘d’.


• In neonates, because of greater volume of e.c.f, it provides a larger V of highly ionized drugs Option ‘e’.
d

Volume of drug distribution depends on :


• Lipid solubility • Ionisation at physiological pH (dependent on pka).
• Differences in regional blood flow. • Affinity for different tissues.
• Fat : lean body mass ratio. • Diseases like CHF, uremia, cirrhosis.
• Extent of plasma and tissue protein binding.

There is increased volume of distribution for fat soluble drugs in elderly individuals. This results in
decreased elimination of the drug which leads to increased toxicity.

73. Ans. is a, c and d i.e. Methotrexate; Rifampicin; and Tolcapone Ref. Harrison 17/e, p 1951

Drugs which can cause diffuse hepatocellular damage have to be used cautiously in hepatic disease.
So, if biluribuin is more than 1.5 mg/dl, the dose adjustment of the following drugs is needed :
• Acetaminophen • Acebutolol • Allopurinol
• Amiodarone • Dapsone • Erythromycin estolate
• Halothane • Isoniazid • Methotrexate
• Minocycline • Nifedipine • Quinidine

50
General Phar macology
Pharmacology 1
• Rifampicin • Salicylates • Tolcapone
• Verapamil • Valproate • Zidovudine
• Flutamide • Nitrofurantoin • Troafloxacin
• Phenytoin • Carbamazine • Nefazodone
• Methyldopa • Oxyphenisatin • Troglitazone

74. Ans. is a, b and c i.e. Lignocaine; Propranolol; and Salbutamol Ref. KDT 6/e, p 28 - 29

• First pass (presystemic) metabolism refers to metabolism of a drug during its passage from the site of
absorption in to the systemic circulation.
• All orally administered drugs are exposed to drug metabolising enzymes in the intestinal wall and liver
(where they first reach through the portal vein).
Extent of first pass metabolism of important drugs :
Low Intermediate High
Not given orally Given orally
– Phenobarbitone – Aspirin – Isoprenaline – Propranolol
– Phenylbutazone – Quinidine – Lignocaine – Alprenolol
– Tolbutamide – Desipramine – Hydrocortisone – Verapamil
– Theophylline – Nortriptyline – Testosterone – Salbutamol
– Pindolol – Chlorpromazine – Nitroglycerine
– Pentazocine – Morphine
– Metoprolol – Pethidine
– Methyltestosterone
– Propoxyphene

75. Ans. is c and e i.e. Agranulocytosis; and Skin pigmentation Ref. KDT 6/e, p 752

• Dapsone is the simplest, cheapest and most active use antileprotic drug.
• It is generally well tolerated at doses 100mg/day or less.
Adverse effects of Dapsone
Most common • Hemolytic anaemia • Methemoglobinemia
Cutaneous • Allergic rashes • Fixed drug eruption
• Hypermelanosis • Phototoxicity
• Exfoliative dermatitis
Others • Agranulocytosis • Hepatitis
• Neuropathy • Gastrointestinal intolerance
• Headache • Rash
• Psychosis • Lepra reaction

76. Ans. is b and d i.e. Propranolol; and Physostigmine Ref. KDT 6/e, p 20, 101

• The blood brain barrier is formed by capillary endothelial cells in brain having tight junctions and lacking large
intercellular pores.
• Both blood brain barrier and blood CSF barrier are lipoidal and limit the entry of non-lipid soluble drugs.

51
1 Self Assessment & Review Phar macology
Pharmacology
• Lipid soluble drugs are able to cross the blood-brain barrier such as :
– Levodopa – Physostigmine
– Propranolol – Organophosphates

• Lipid insoluble drugs are not able to cross the blood-brain barrier such as :
– Streptomycin – Neostigmine – Hexamethonium
– Dopamine – Glycopyrrolate

77. Ans. is a and c i.e. Lithium; and Phenytoin Ref. KDT 6/e, p 35

Low therapeutic range is seen in following drugs :


• Lithium • Digoxin
• Lignocaine • Aminoglycosides
• Theophylline • Anticonvulsants e.g. phenytoin

Fore more detail refer answer no. 27

78. Ans. is b, c and d i.e. Sulfonation; Methylation; and Glucuronization


Ref. KDT 6/e, p 22 - 26

Biotransformation occurs as follows :

Mnemonics • All reactions starting with ‘De’ are phase - I reactions


– Dealkylations – Deaminations
– Desulphuration – Dechlorination
– Decyclization
• All the reactions starting with ‘G’ are phase - II reactions
– Glucoronide conjugation – Glycine conjugation
– Glutathione conjugation

52
General Phar macology
Pharmacology 1

79. Ans. is c i.e. Chloroquine Ref. Katzung 10/e, p 975, KDT 909, 910

Already explained, refer answer no. 29

80. Ans. is a, b, d and e i.e. Propranolol; Lidocaine; Imipramine; and Theophylline


Ref. KDT 5/e, p 126, 203, 409, 659

Option ‘a’ Propranolol is well absorbed after oral administration, but has low bioavailability due to high
first pass metabolism in liver. Oral : parenteral dose ratio of up to 40:1 has been found.
Option ‘b’ Lidocaine also undergoes high first pass metabolism in the liver.
Option ‘c’ Ampicillin is partly excreted in bile and reabsorbed - entero hepatic circulation occurs. However,
primary channel of excretion is kidney.
Option ‘d’ Tricyclic antidepressant (Imipramine) are extensively metabolized in the liver, the major route
for Imipramine and Amitriptyline is demethylaton whereby active metabolites-desipramine
and nortriptyline respectively are formed.
Option ‘e’ Theophyline is extensively metabolized in liver by demethylation and oxidation.

81. Ans. is b, c, d and e i.e. Warfarin; Valproic acid; Steroids; and Phenytoin
Ref. KDT 6/e, p 85; Drugs in Pregnancy & Lactation 6/e, p 888

Already explained, refer answer no. 31

53
1 Self Assessment & Review Phar macology
Pharmacology
82. Ans. is a, b, c, and d i.e. Antithistaminics; Antithyroid drugs; Penicillin; and Diazepam
Ref. KDT 6/e, p 912 - 914

There is a huge list of the drugs which are secreted in breast milk in small to large amounts and may cause
harm to foetus but we are highlighting a select few :
– Acyclovir – Benzodiazepenes – Iodine/ Iodides
– Alcohol – Chloramphenicol – Metformin
– Amiodarone – Ciprofloxacin – Metronidazole
– Amphetamines – Cyclosporine – Penicillins
– Androgens – Dapsone – Streptomycin
– Anticancer drugs – Diltiazem – Tetracyclines
– Antihistaminics (H1) – Ergotamine
– Antithyroid drugs (Carbimazole) – Estrogens
– Azathioprine

These are the drugs which are either contraindicated in breast feeding or are secreted in significant amounts to
cause harm to the foetus.

83. Ans. is a i.e. ↑ ↑ free drug plasma level Ref. KDT 6/e, p 20 - 21

There are certain concepts relating to drugs with plasma protein binding.
• Displacement of protein bound drugs leads to increased free drug plasma level.
– One drug can bind to many sites on albumin molecule Conversely, more than one drug can bind to the
same site.

This can give rise to displacement interactions among drugs bound to the same site.

Drugs bound with higher affinity will displace that bound with lower affinity.

The concentration of free (displaced drug) is increased but is transient and achieves a steady level of free
drug concentration which is only marginally higher unless displacement extends to tissue binding or there is
concurrent inhibition of metabolism or excretion. E.g. Phenylbutazone and salicylates displace tolbutamide
Indomethacin, phenylbutazone displace tolbutamide Sulphonamide and vit. K displace bilirubin (Kernicterus
in neonates).
• Highly plasma protein bound drugs are largely restricted to the vascular compartment and thus, have lower
volumes of distribution.
• The bound fraction is not available for action.
The bound fraction is in equilibrium with the free drug plasma and dissociates when the concentration of
latter is reduced.

Protein bound drug is thus equivalent to temporary storage of the drug.
• High degree of protein binding generally makes the drug long acting, because fraction is not available for
metabolism or excretion.
• Generally expressed plasma concentrations of the drug refer to bound as well as free drug.
• In hypoalbuminemia (nephrotic syndrome), binding may be reduced and high concentrations of free drug
may be attained (e.g, phenytoin and furosemide).

54
General Phar macology
Pharmacology 1
84. Ans. is a i.e. Highly lipophilic Ref. KDT 6/e, p 18 - 19

• One of the most important factor responsible for maximum distribution is lipid solubility of a drug.
• The extent of distribution of a drug depends upon :

– Lipid solubility – Ionisation at physiological pH (dependent on pka)


– Differences in regional blood flow – Extent of binding to plasma and tissue proteins

85. Ans. is a i.e. Side effects Ref. KDT 6/e, p 78-80

Both side effects and toxicity are Type - A - ADR. But side effect are seen at therapeutic doses while
toxicity is due to prolonged use or overdose.

86. Ans. is b i.e. Has affinity as well as intrinsic activity Ref. KDT 6/e, p 42

Agonists have both affinity and maximal intrinsic activity (IA = 1), e.g. adrenaline, histamine, morphine.
Competitive have affinity but no intrinsic activity (IA = 0), e.g. propranolol, atropine, chlorpheni-
antagonists ramine, naloxone.
Partial agonists have affinity and submaximal intrinsic activity (IA between 0 to 1), e.g. dichloroisoproterenol,
nalorphine.
Inverse agonists have affinity but intrinsic activity with a minus sign (IA between 0 to –1), e.g. DMCM.

87. Ans. is b i.e. Log dose response curve is sigmoid shaped Ref. KDT 6/e, p 52 - 53

• The dose response curve (DRC) is a rectangular hyperbola but if the dose is plotted on logarithmic scale,
the curve becomes sigmoid as is evident in figure below :

55
1 Self Assessment & Review Phar macology
Pharmacology

• The advantages of plotting log dose - response curves are :


– A wide range of drug doses can be easily displayed on a graph.
– Comparison between agonists and study of antagonists become easier.
• The position of DRC on the dose axis is the index of drug potency which refers to the amount of drug
needed to produce a certain response.
• The upper limit of DRC is the index of drug efficacy and refers to the maximal response that can be elicited
by the drug.

88. Ans. is a i.e. G-protein Ref. KDT 6/e, p 48, 349

GABA acts through its specific receptors GABA and GABA which are characterized as follows :
A B

GABAA receptor GABAB receptor


– Intrinsic ion channel receptor – G-protein coupled receptor
– Increases Cl conductance

– Not antagonized by Bicuculline
– Blocked by Bicuculline – Blocked by saclofen
– Facilitated by benzodiazepenes – Hyperpolarises neurons by increasing K +
conductance and altering Ca++ flux

89. Ans. is c i.e. Efficacy Ref. KDT 6/e, p 53 - 54; Katzung 10/e, p 28

Efficacy – The upper limit of Dose response curve (DRC) is the index of drug efficacy and refers to
the maximal response that can be elicited by the drug.
Potency – The position of DRC of the dose axis is the index of drug potency which refers to the
amount of drug needed to produce a certain response.

90. Ans. is a i.e. Barbiturate poisoning Ref. KDT 6/e, p 30

• Acidic drugs such a Barbiturates and Salicylates ionize more and are less reabsorbed in alkaline
urine. Therefore, urine is alkalinised in barbiturate poisoning.
• On the other hand, basic drugs such as morphine and amphetamine ionise more and are less reabsorbed in
acidic urine. Thus, urine is acidified in their poisoning.

56
General Phar macology
Pharmacology 1
91. Ans. is a i.e. Best absorbed in acidic medium Ref. KDT 6/e, p 15

• Acidic drugs e.g., salicylates, barbiturates etc. are predominantly unionised in acid gastric juice
and are absorbed from the stomach.

because acidic drugs ionise in alkaline medium in the small intestines; and once ionised, the ions being
lipid insoluble, do not diffuse across gastric mucosal cells.

• Basic drugs, on the other hand, like morphine, quinine etc. are largely ionised and are absorbed only on
reaching the duodenum.
• Acidic drugs bind to albumin wile basic drugs bind to α -acid glycoprotein.
1

57
1 Self Assessment & Review Phar macology
Pharmacology

CHAPTER REVIEW
• This section includes questions of V arious Other PGMEES from 1990 – 2008.
Various
• Questions are ar ranged in increasing order of page sequence of KDT 6 Edition. This is
arranged
done to mak e refer
make ral system more easy and uncomplicated to save the precious time
referral
of PGMEE Aspirant.

1. All are inhibitors of cytochrome P-450 enzymes 6. Therapeutic index of a drug is an indicator of :
except : (Manipal 07) a. Potency (J & K 05)
a. Amiodarone b. Safety
b. Disulfiram c. Action
c. Fluoxetine d. Efficacy
d. Rifampicin [Ref. KDT 6/e, p 55]
[Ref. KDT 6/e, p 27] 7. All are reasons for reducing drug dosage in eld-
2. When causes least “gain in weight”? erly except : (MAHE 05)
a. Olanzapine (NIMHANS 06) a. They are lean and their body mass is less
b. Risperidone b. Have decreasing renal function with age
c. Quetiapine c. Have increased baroreceptor sensitivity
d. Ziprasidone d. Body water is decreased
[Ref. CMDT, p 1063] [Ref. KDT 6/e, p 52]
3. Loading dose depends on the following factors 8. The drugs that does not cause Raynaud’s phe-
except : (APPG 06) nomenon: (APPG 05)
a. Drug concentration to be achieved a. Vincristine
b. Volume of distribution b. Bleomycin
c. Clearance of the drug c. Vinblastin
d. Bioavailability of drug d. Cisplatin
[Ref. KDT 6/e, p 34] [Ref. Laurence 9/e, p 613, 614; Goodman &
4. How long does the protection from passive immu- Gilman 10/e, p 1435]
nization using an immune globulin preparation typi- 9. Competitive inhibition : (Manipal 05)
cally last ? (MAHE 05) a. Response depends on antagonist concentra-
a. 1 to 3days tion
b. 1 to 3 weeks b. Antagonist binds to same receptor as agonist
c. 1 to 3 months c. Binds to different sites
d. 1 to 3 years d. Reduces intrinsic activity of agonist
[Ref. KDT 6/e, p 886] [Ref. KDT 6/e, p 57]
5. Half life does not help in estimating : (SGPGI 05) 10. Which one of the opioids has maximum plasma
a. Margin of safety protein binding capacity ? (Jipmer 05)
b. Time taken to eliminate the drug completely from a. Morphine
the body b. Sulfentanil
c. Dosing schedule c. Fentanyl
d. Cone, of drug in body at a particular time after its d. Pethidine
administration [Ref. KDT 6/e, p 458]
[Ref. Katzung 10/e, p 38 - 39]

Answer 1. d. Rifampicin 2. d. Ziprasidone 3. c. Clearance of ... 4. c. 1 to 3 months 5. a. Margin of ...


6. b. Safety 7. c. Have increased ... 8. a and c 9. b. Antagonist ... 10. d. Pethidine

58
General Phar macology
Pharmacology 1
11. Which of the following given in pregnancy has ter- 18. To increase the fetal lung maturity the drug to be
atogenic effects : (APPGE 04) used is : (Manipal 03)
a. Vit. B6 a. Methyldopa
b. Vit. B3 b. Propranolol
c. Folic acid c. Insulin
d. Vit. D d. Betamethasone [Ref. KDT 6/e, p 284]
[Ref. KDT 6/e, p 85; Katzung 10/e, p 975] 19. True regarding “Plasma half life” of drugs :
12. Clearance means : (Manipal 04) a. Maximum effective dose of drug
a. Rate of elimination/plasma concentration b. Max. lethal effect (Manipal 02)
b. Plasma concentration/half life c. Potency of drug
c. Time taken for the plasma concentration of the d. Dose frequency interval
drug to be reduced to half its original value [Ref. KDT 6/e, p 33-34]
d. Rate of elimination X plasma concentration 20. Downgrading of receptors is due to :(Manipal 01)
[Ref. KDT 6/e, p 31] a. Increased uptake of agonist by them
13. Steady state means : (Manipal 04) b. Increased interaction with antagonist
a. Dose rate X clearance c. Both of the above
b. Dose rate/clearance d. None of the above
c. Clearance/dose rte [Ref. KDT 6/e, p 68]
d. Rate of elimination/plasma concentration 21. Which of the following does not cross placenta ?
[Ref. KDT 6/e, p 33] a. 131I (Manipal 00)
14. Drugs which cannot be safely used in pregnancy b. Heparin
all except ? (Manipal 03) c. Warfarin
a. Tetracycline d. Dicoumarol
b. Sulphonylurea [Ref. KDT 6/e, p 85]
c. Heparin 22. All are examples of prolong except :
d. Warfarin a. Enalapril (Up 01)
[Ref. KDT 6/e, p 598] b. Cortisone
15. The antibiotic which cannot be safely used in preg- c. Azathioprine
nancy are all except : (Manipal 03) d. Mercaptopurine
a. Tetracycline [Ref. KDT 6/e, p 23 - 24]
b. Ampicillin 23. True about partial agonist is : (Up 01)
c. Streptomycin a. Produces maximal effect
d. Sulphonamides b. Supramaximal effect
[Ref. KDT 6/e, p 85] c. Produces submaximal effect
16. Zero order kinetics is not followed by : d. Effect in opposite direction
a. Phenytoin (Manipal 03) [Ref. KDT 6/e, p 40 - 41]
b. Ethanol 24. Basic drug binds to : (Up 00)
c. Barbiturates a. α-acidic glycoprotein
d. Salicylates b. Albumin
[Ref. KDT 6/e, p 31] c. Lipoprotein
17. If a drug is given repeatedly at approximately the d. Mucoprotein
same time as its half life, after how many dos-
[Ref. KDT 6/e, p 13 - 14]
ages will the drug achieve a steady a state in
25. Km value indicates : (Up 00)
plasma :
a. Purity of enzyme
a. 2 - 3 (Manipal 03)
b. Physiological role
b. 4 - 5
c. Half life enzyme drug complex
c. 8 - 9
d. Affinity
d. 6 - 8 [Ref. KDT 6/e, p 33]
[Ref. KDT 6/e, p 31 - 34]
Answer 11. d. Vit. D 12. a. Rate of ... 13. b. Dose rate/ ... 14. c. Heparin 15. b. Ampicillin
16. c. Barbiturates 17. b. 4 - 5 18. d. Betamethasone 19. d. Dose ... 20. a. Increased ...
21. b. Heparin 22. c. Azathioprine 23. c. Produces ... 24. a. α-acidic ... 25. c. Half life ...
26. b. Psychological

59
1 Self Assessment & Review Phar macology
Pharmacology
26. Placebo effect is : (UP 99) c. Phenyramine
a. Pharmacological d. Sulphonamides
b. Psychological [Ref. KDT 6/e, p 26 - 27]
c. Trial of drug effect 34. All drug can cross placenta except : (UP 95)
d. Drug dependence a. Heparin
[Ref. KDT 6/e, p 65] b. Diazepam
27. Generic drug true is : (UP 99) c. Theophylline
a. High cost d. Warfarin
b. Increased bioavailability [Ref. KDT 6/e, p 598]
c. Low cost 35. Drug contraindicated with pregnancy : (UP 95)
d. Reduced clearance a. Hydralazine
[Ref. KDT 6/e, p 63] b. Propranolol
28. Oxidative elimination of drug : (UP 99) c. Methyldopa
a. Conjugation d. Magnesium sulphate
b. Azo reduction [Ref. KDT 6/e, p 553]
c. Acetylation 36. The advantages of transdermal delivery system
d. Sulfoxide are a/e :
[Ref. KDT 6/e, p 35] a. High peak plasma level (UP 95)
29. Monitoring of plasma level is required in :(UP 99) b. Slow releasing
a. Carbamazepine c. Continuous
b. Diazepam d. Effect comes late
c. Valproic acid [Ref. KDT 6/e, p 9]
d. Phenytoin 37. Which drug causes hyperuricemia is : (UP 94)
[Ref. KDT 6/e, p 35] a. Pyrazinamide
30. Which of the following drug has high first pass b. Rifampicin
metabolism : (UP 98) c. Ethambutol
a. Propranolol d. Streptomycin
b. Phenytoin [Ref. KDT 6/e, p 742]
c. Doxycycline 38. Which is not prodrug : (UP 94)
d. Pentazocine a. Oxyphenbutazone
[Ref. KDT 6/e, p 28] b. Azathioprine
31. All are examples of microsomal inducers except : c. Cholecaflciferol
a. Phenytoin (UP 98) d. Sulindac
b. Ethanol [Ref. KDT 6/e, p 24]
c. Cimetidine 39. Plasma monitoring is essential in therapy of :
d. Rifampicin a. Ethosuccimide (UP 94)
[Ref. KDT 6/e, p 27] b. Phenobarbitone
32. All are examples of prodrug except : (UP 98) c. Phenytoin
a. Enalapril d. Sodium valproate
b. Bacampicillin [Ref. KDT 6/e, p 404]
c. Levodopa 40. Which of the following avoids lst bypass effect of
d. Phenylbutazone drug : (UP 93)
[Ref. KDT 6/e, p 24] a. Intra-arterial injection
33. Following drugs are hepatic microsomal inducers b. Rectal suppositories
except : (UP 97) c. Intravenous injection
a. Alcohol d. Oral insertion
b. Cimetidine [Ref. KDT 6/e, p 28]

Answer 27. c. Low cost 28. d. Sulfoxide 29. d. Phenytoin 30. a. Propranolol 31. c. Cimetidine
32. d. Phenylbutazone 33. b. Cimetidine 34. a. Heparin 35. b. Propranolol 36. a. High peak ...
37. a. Pyrazinamide 38. a. Oxyphenbu .... 39. c. Phenytoin 40. a. Intra-arterial ...

60
General Phar macology
Pharmacology 1
41. Serum level measurement of a drug is not useful 48. Which is the advantage of sublingual route of ad-
in : (AIIMS Dec. 94) ministration of drugs : (Jipmer 91)
a. Large person to person difference in serum lev- a. Prevents first pass effect
els b. Easy to administer
b. Drug with low margin of safety c. Lipid soluble
c. Drug activated in body d. Can be spitted out with signs of toxicity
d. To check compliance [Ref. KDT 6/e, p 7]
[Ref. KDT 6/e, p 35] 49. Drugs mostly cross biological membranes by :
42. Cy-GMP is the second messenger of : a. Passive diffusion (ROHTAK 98)
a. Growth hormone (AIIMS May 94) b. Active diffusion
b. Follicle stimulating hormone (FSH.-with AMP) c. Active transport
c. Insulin -Phosphorylate d. Carrier mediated transport
d. Thyroxin-at nuclear receptor e. All of the above
[Ref. KDT 6/e, p 48] [Ref. KDT 6/e, p 12]
43. All induce liver microsomal enzyme except : 50. The alkaline drug (Amidopyrine) is absorbed from:
a. Metyrapone (AIIMS May 94) a. Stomach (AI 91)
b. Carbamazepine b. Proximal small intestine
c. Glutethimide c. Distal small intestine
d. Phenobarbitone d. Colon
[Ref. KDT 6/e, p 419] [Ref. KDT 5/e, p 16]
44. All are nephrotoxic, except : (AIIMS Nov. 93) 51. All of the following are protein bound except :
a. Lithium a. Propranolol (Delhi 92)
b. Gentamicin b. Atenolol
c. Chlorpromazine c. Phenylbutazone
d. Cephalosporin d. Warfarin
[Ref. KDT 6/e, p 428] [Ref. KDT 6/e, p 20]
45. Half-life of a drug is a good indicator of the follow- 52. What is prodrug : (Jipmer 91)
ing, except : (AIIMS June 92) a. Drug which increase efficiency of another drug
a. Time required to reach steady state b. Metabolic end product
b. Time for a drug to be removed from the body c. Inactive drug which gets activated in the body
c. Appropriate dosing interval d. Drug which competes with another for metabo-
d. Margin of safety lism
[Ref. KDT 6/e, p 32 - 35] [Ref. KDT 6/e, p 23]
46. All of following cross plasma membrane, except: 53. Prodrugs are : (NIMHANS 01)
a. Androstenodione acts on cytoplasmic steroid a. Lisinopril
hormone or receptors (June 1991) b. Ramipril
b. M- Dopa c. Enalapril
c. Epinephrine d. Captopril
d. Thyroxine [Ref. KDT 6/e, p 24]
[Ref. KDT 6/e, p 805] 54. Which cytochrome is responsible for drugs me-
47. Which of the following is not an inducer of hepatic tabolism? (TN 95)
microsomal enzymes : (AIIMS June 91) a. P-450
a. Phenytoin b. C-3b
b. Meprobamate c. C-3A
c. Tolbutamide d. C-2a
d. Rifamipicin [Ref. KDT 6/e, p 24]
[Ref. KDT 6/e, p 26 - 27]

Answer 41. c. Drug activated ... 42. None 43. a. Metyrapone 44. c. Chlorpromazine 45. d. Margin of ...
46. b. M- Dopa 47. b and c 48. a. Prevents ... 49. a. Passive ... 50. b. Proximal ...
51. b. Atenolol 52. c. Inactive ... 53. b and c 54. a. P-450

61
1 Self Assessment & Review Phar macology
Pharmacology
55. Which of the following statements is correct ? c. Steady plasma concentration
a. Most drugs are absorbed in ionised form d. Therapeutic dosage
b. Basic drugs are generally bound to plasma al- [Ref. Katzung 10/e, p 38-39]
bumin (Karn 94) 62. Indiosyncrasy is : (Karnat 96)
c. Microsomal enzymes are located in the mitochon- a. A genetically determined abnormal reaction
dria of hepatic cells drugs
d. Blood brain barrier is deficient at the chemore- b. A characteristic toxic effect at therapeutic doses
ceptor trigger zone c. An altered physiological state produced by re-
[Ref. KDT 5/e, p 14, 12, 18, 23] peated drugs use
56. Enzyme cytochrome p450 inhibitors are : d. An immunologic ally mediated reaction
a. Sodium valproate (NIMHANS 01) [Ref. KDT 6/e, p 81]
b. INH 63. Drug which crosses the placental barrier is :
c. Rifampicin a. Phenytoin (DELHI 84, 92)
d. Ethambutol b. Diazepam
[Ref. KDT 6/e, p 27] c. Corticosteroids
57. Drug with extensive first pass metabolism : d. All of the above
a. Propranolol (Jipmer 91) [Ref. KDT 6/e, p 85; Katzung 10/e, p 975]
b. Digoxin 64. The effects of hypoglycemia is marked by :
c. Quinidine a. Warfarin (TN 90)
d. Chlorpromazine b. Beta blockers
[Ref. KDT 6/e, p 28] c. Calcium channel blockers
58. Which of the following shows high first pass me- d. Aminoglycosides
tabolism : (ROHTAK 98) [Ref. KDT 6/e, p 138]
a. Propranolol 65. The intake of which of the following is associated
b. Phenobarbitone with an increase in weight : (CUPGEE 96)
c. Phenylbutazone a. Chlorpromazine
d. Phenytoin b. Methyldopa
[Ref. KDT 6/e, p 28] c. OCP
59. Substrate concentration and velocity curve indi- d. Thyroxine
cates : (PGI 96) [Ref. KDT 6/e, p 315]
a. Michelis - Mentons equation 66. Gout is precipitated by all except : (MP 98)
b. Inverse relation a. Furosemide
c. Zero order kinetics b. Vincristine
d. All of the above c. Penicillin
[Ref. KDT 6/e, p 33; Harper 24/e, p 83] d. Sulfinpyrazone
60. Plasma concentration of drug at time 0 is 96 g/ml. [Ref. KDT 6/e, p 205]
If tl/2 is 2 hours concentration in plasma at 10 67. Which of the following drugs causes post anaes-
hours will be : (MAHE 98) thetic muscle stiffness : (AI 91)
a. 4 a. Fentanyl
b. 24 b. Pyridostigmine
c. 12 c. Suxamethonium
d. 3 d. Gallamine [Ref. KDT 6/e, p 346]
[Ref. KDT 6/e, p 32] 68. Drugs absorbed by active transport is :
61. T 1/2 of a drug can determine all the following ex- a. Propranolol (NIMS 96)
cept : (AIIMS 90)
b. Ergotamine
a. Dosing interval
c. Levodopa
b. Elimination time
d. Amantidine [Ref. KDT 6/e p 417]

Answer 55. d. Blood ... 56. d. Ethambutol 57. a. Propranolol 58. a. Propranolol 59. a. Michelis ...
60. d. 3 61. d. Therapeutic ... 62. a. A genetically ... 63. d. All of the above 64. b. Beta blockers
65. c. OCP 66. d. Sulfinpyrazone 67. c. Suxamethonium 68. c. Levodopa

62
General Phar macology
Pharmacology 1
69. Drug with extensive first pass metabolism : d. Disopyramide
a. Propranolol (Jipmer 92) [Ref. KDT 6/e, p 28]
b. Haloperidol 77. Urine is coloured due the intake of :
c. Diazepam a. Phenindione (CUPGEE 96)
d. Acetazolamide b. Phenobarbitone
[Ref. KDT 6/e, p 28] c. Phenytoin
70. Cough is an adverse reaction with intake of : d. Quinine
a. Captopril (PGI 93) [Ref. KDT 6/e, p 601]
b. Prazosin 78. Drugs which increase the metabolism of warfarin
c. Nifedipine in/are : (PGI 90)
d. Thiazide a. Rifampicin
[Ref: KDT 6/e, p 484] b. Septran
71. Cough is an adverse reaction seen with intake of: c. Phenylbutazone
a. Thiazide (PGI 93) d. Phenobarbitone
b. Nifedipine [Ref. KDT 6/e, p 28, 603; Katzung 10/e, p 55,
c. Enalapril table 4-2]
d. Prazosin 79. Which one of the following drugs dose not exhibit
[Ref: KDT 6/e, p 485] zero order kinetics : (CUPGEE 96)
72. Which of the following is not a dose related reac- a. Phenytoin
tion : (JIPMER 92) b. Ethanol
a. Myocardial irritation of quinidine c. Salicylate
b. Hypoglycemia of tolbutamide d. Ranitidine
c. Digitalis induced arrhythmia [Ref. KDT 6/e, p 31]
d. Drug fever of sulpha 80. All the prodrugs except : (Jipmer 95)
[Ref. KDT 5/e, p 643, 476, 248, 462] a. Enalapril
73. Which dose not cross placental barrier : b. Paracetamol
a. Diazepam (AIIMS 90) c. Sulphamethoxazole
b. Pethidine d. Trimethoprim
c. Atropine [Ref. KDT 6/e, p 24]
d. Dicoumerol 81. One of the following in high doses can cause con-
[Ref. Goodman & Gilman 10/e p 594] vulsions : (AI 90)
74. Urine is coloured due to the intake of : a. Penicillin
a. Phenindione (UPSC 97) b. Sulphonamides
b. Phenobarbitone c. Aminoglycosides
c. Phenytoin d. Erythromycin
d. Quinine [Ref. KDT 6/e, p 697]
[Ref. KDT 6/e, p 601] 82. Canrenone is the prodrug of : (APPGE 04)
75. Not a prodrug : (JIPMER 99) a. Ampicillin
a. Lisinopril b. Spironolactone
b. Enalapril c. Frusemide
c. Ramipril d. Acetazolamide
d. Famotidine [Ref. KDT 6/e, p 24]
[Ref. KDT 6/e p 24] 83. Which is used in drug induced ulcers : (UP 2K)
76. Which drug has a high first pass effect : a. Antacids
a. Amiodarone (Jipmer 91) b. Ranitidine
b. Phenytoin c. Omeprazole
c. Verapamil d. Misoprostol [Ref. KDT 6/e, p 632]

Answer 69. a. Propranolol 70. a. Captopril 71. c. Enalapril 72. d. Drug fever ... 73. c. Atropine
74. a. Phenindione 75. a. Lisinopril 76. c. Verapamil 77. a. Phenindione 78. a and d
79. d. Ranitidine 80. b, c and d 81. a. Penicillin 82. b. Spironolactone 83. c. Omeprazole

63
1 Self Assessment & Review Phar macology
Pharmacology
84. The following is a prodrug : (JIPMER 02) 91. Which of the following are the recognised causes
a. Lovastatin of rapidly progressive glomerulonephritis :
b. Pravostatin a. Indomethacin (UPSC 97)
c. Cetrivastatin b. Penicillamine
d. Fluvastatin c. Lead
[Ref. KDT 5/e, p 578] d. Rifampicin
85. The following has maximum propensity for [Ref. Harrison 17/e, p 1787-1788]
photodermatitis : (JIPMER 98) 92. All of the following drug can cause photo-dermati-
a. Oxytetracycline tis except : (UPSC 97)
b. Doxycycline a. Griseofulvin
c. Minocycline b. Chloroquine
e. Beta 2 agonist c. Captopril
[Ref. KDT 6/e, p 713] d. Oral contraceptive
86. Chronic ulceration would most likely occur follow- [Ref. Harrison 17/e, p 343-353]
ing administration of : (DNB 91) 93. All the following drugs cause hirsutism except :
a. Carbenicillin a. Phenytoin (TN 01)
b. Clindamycin b. Minoxidil
c. Chloramphenicol c. Corticosteroids
d. Colistin B d. Heparin
[Ref: KDT 6/e, p 731] [Ref. Harrison 17/e, p 301, table (50-1)]
87. Which is an Ototoxic drug : (MP 98) 94. The following cause macrocytic anemia except :
a. Tetracycline a. Pyrimethamine (AI 92)
b. Vincristine b. Methotrexate
c. Penicillin c. Pentamidine
d. Sulphonamide d. Trimethoprim
[Ref. KDT 6/e, p 825] [Ref: Harrison 17/e, p 649-650]
88. Psychosis can be caused by : (JIPMER 91) 95. Hypersensitive cholestatic jaundice is caused by:
a. Steroids a. Chlorpromazine (AI 92)
b. INH b. Rifampicin
c. Chloroquine c. Halothane
d. All of above d. Tetracycline
[Ref. KDT 6/e, p 286, 786, 741] [Ref: Harrison 17/e, p 265]
89. Drug which increases absorption of oral iron : 96. All cause hyperprolactinemia except :
a. Folic acid (Orissa 04) a. Metoclopramide (MAHE 98)
b. Nalidixic acid b. Haloperidol
c. Vitamin C c. Trifluperazine
d. Tetracycline d. Lasix
[Ref. KDT 6/e, p 878] [Ref. Harrison 17/e, p 2205]
90. In G-6 PD deficiency, hemolysis occurs most com- 97. Hirsutism is caused by : (MAHE 98)
monly with : (JIPMER 91) a. Minoxidil
a. Sulphonamide b. Phenytoin
b. INH c. Hydralazine
c. Amphotericin B d. All
d. Primaquine [Ref. Harrison 17/e, p 301, table (50-1)]
[Ref. Harrison 17/e, p 657, table (101-5]

Answer 84. a. Lovastatin 85. b. Doxycycline 86. b. Clindamycin 87. b. Vincristine 88. d. All of above
89. c. Vitamin C 90. a. Sulphonamide 91. d. Rifampicin 92. d. Oral contrace ... 93. d. Heparin
94. c. Pentamidine 95. a and b 96. d. Lasix 97. a and b

64
General Phar macology
Pharmacology 1
98. Drug induced hypothermia is caused by all the b. Chloramphenicol
drugs, except : (Orissa 98) c. Deceased pseudocholinesterase
a. Phenothiazines d. Old and degraded tetracyclines
b. Barbiturates [Ref: Harrison 17/e, p 1807, KDT 6/e, p 713]
c. MAO-inhibitors 105. Agranulocytosis is common with : (KERAL 94)
d. Ethanol a. Chloramphenicol
[Ref. Harrison 17/e, p 135, table (20-1)] b. Methotrexate
99. Drug implicated in the causation of pancreatitis : c. Sulpha drugs
a. L- Asparaginase (PGI 96) d. Amikacin
b. Cyclophosphamide [Ref. Harrison 17/e, p 668]
c. Cyclosporine 106. Gynecomastia is caused by : (AP 97)
d. Methotrexate a. Phenytoin
[Ref. Harrison 17/e, p 2007 KDT 6/e p 827] b. Cushing’s syndrome
100. Galactorrhea is caused by all except : c. Conn’s syndrome
a. Metoclopramide (Orissa 00) d. None
b. Digitalis [Ref. Harrison 17/e, p 2318]
c. Reserpine 107. Drug which is not a cause of a chronic active
d. Methyldopa hepatitis : (PGI 96)
[Ref. Harrison 17/e, p 2205] a. INH
101. Cholestatic jaundice occurs with intake of all b. Methyldopa
except : (AI 93) c. Chlorpromazine
a. Erythromycin d. Oxyphenacetin
b. INH [Ref. Harrison 17/e, p 1951, table (299-2)]
c. O.C. pills 108. Not an ototoxic drug : (PGI 96)
d. Nifedipine a. Neosporine
[Ref: Harrison 17/e, p 265] b. Amikacin
102. All of the following are hepatotoxic except : c. Vincristine
a. Erythromycin (A.P. 97) d. Aminoglycoside
b. Tetracycline [Ref. Harrison 17/e, p 202]
c. Choloroquine 109. Non ischaemic chest pain is caused by :
d. Rifampicin a. Bleomycin (KERALA 97)
[Ref. Harrison 17/e, p 1951, table (299-2)] b. Vincristine
103. The following drug is used for osteoporosis : c. Cyclophosphamide
a. Ranitidine (JIPMER 02) d. Cisplatinum
b. Raloxifene [Ref. Harrison 17/e, p 525]
c. Ramipril 110. Leukoencephalopathy is seen with use :
d. Riclopride a. Vincristine (PGI 93, 81)
[Ref. Harrison 17/e, p 2404-2406; Goodman & b. Cyclophosphamide
Gilman 10/e, p 1738] c. Methotrexate
104. Fanconi’s syndrome is caused by : (JIPMER 91) d. 5-FU
a. Cephalosporins [Ref: Harrison 17/e, p 2080]

Answer 98. c. MAO-inhibitors 99. a. L- Asparaginase 100. b. Digitalis 101. b. INH 102. c. Choloroquine
103. b. Raloxifene 104. d. Old and ... 105. a and d 106. a. Phenytoin 107. c. Chlorpromazine
108. a. Neosporine 109. a. Bleomycin 110. c. Methotrexate

65
NOTES
GENERAL INTRODUCTION
The Motor portion of the nervous system is divided into : • Somatic NS
• Autonomic NS
• The somatic system is concerned with consciously controlled movements such as, respiration, posture, and movement.
• The autonomic system is not under direct conscious control.
It controls mainly visceral functions such as cardiac output, blood flow to various organs, digestion, genitourinary
functions etc.

Similarities of Nervous system (both somatic and autonomic) to endocrine system


– High level of integrity in the brain
– Ability to influence functions at distant regions
– Use of negative feed back
By using drugs that mimic or blocks the actions of the chemical transmitters, we can enhance or retard the autonomic
functions.

The ANS is divided into : SYMPATHETIC PARASYMPATHETIC


– Thoracolumbar outflow – Craniosacral Includes
Cranial nerves - 3, 7, 9, 10
Sacral spinal - 3rd / 4th

This division is anatomical. It does not depend on the type of transmitter released or the kind of effect they produce i.e.
excitatory or inhibitory.
Therapy affects the following 5 potential targets of Neurotransmitter function:
1. Synthesis
2. Storage
3. Release
4. Activation of Receptors
5. Termination of Action

Neurotranmitters
• Acetylcholine
• Monoamines (5HT, Dopamine, Norepinephrine, Histomine)
• Aminodid neurotnusmitters.
• Inhibitory — GABA, Glycine
• Excitatory — Aspartate, Glutamate
2 Self Assessment & Review Phar macology
Pharmacology

Schematic illustration of generalized cholinergic junction.

CHT - Choline transporter


ChAT - Choline accetyle transferase
VAT - Vesicle-associated transporter
VAMPs - Vesicle-associated membrance proteins
SNAP’s - Synaptosome-associated proteins

70
Phar macology of ANS, PNS & A
Pharmacology utacoids
Autacoids 2
Schematic illustration of generalized noradenergic junction.

VMAT - Vesicular Monoamine transporter

CHOLINERGIC TRANSMISSION
The terminal of a cholinergic neuron contains:
• Small membrane bound clear vesicles containing most of acetylcholine, and
• Large dense cored vesicles containing peptide co transmitters.

Synthesis :
Acetate + CoA → Acetyl CoA (Synthesized in Mitochondria)
Acetyl CoA + Choline (Ch AT) → Acetylcholine (Synthesized in Cytoplasm)

• Choline is transported from the ECF into the neuron terminal by a sodium dependent membrane carrier which is blocked
by a group of drugs called HEMICHOLINIUMS.
• After synthesis, Ach is transported by an antiport carrier from the cytoplasm into the vesicle, which is blocked by VESAMICOL.
Each vesicle contains about 1,000-50,000 Ach molecules.

71
2 Self Assessment & Review Phar macology
Pharmacology
• Fusion of the vesicular membrane with the terminal membrane occurs through interaction of :
– vesicle associated membrane proteins (VAMPs) - SYNAPTOBREVIN and SYNAPTOTAGMIN with
– synaptosome associated proteins (SNAPs) - SNAP-25 and SYNTAXIN
• Ach vesicle release is inhibited by BOTULINUM TOXIN which removes 2 amino acids from these fusion proteins.
• Once released, Ach is hydrolysed by AchE to Choline + Acetate.
• True cholinesterase (AchE) is found in cholinergic synapses as well as in RBCs.
• Choline and acetate are taken by the cell only to be utilized.
– Pseudocholinesterase (Butyryl ChE) is found in liver, blood plasma, CSF, glial cells etc.

ADRENERGIC TRANSMISSION
Tyrosine
Metyrosine (-) →↓ Tyrosine hydroxylase (Rate Limiting step)
DOPA
↓ DOPA decarboxylase
Dopamine
↓ Dopamine b-hydroxylase
Norepinephrine
↓ PNMT
Epinephrine
– Metyrosine inhibits the rate limiting enzyme, tyrosine hydroxylase.
– Reserpine inhibits the high-affinity carrier for catecholamines located in wall of storage vesicles.
– Cocaine and TCAs inhibit another carrier which transports NE into cell cytoplasm (known as Uptake 1).
• Release of neurotransmitter is dependent on extracellular calcium.
Co-transmitters are substances released from the vesicle along with the neurotransmitter. They are ATP, CGRP, CCK, DA
ENKEPHALIN, GALANIN, GABA, GRP, NPY, NO S-P, VIP etc.

NANC Neurons
• Certain tissues like gut, airways and bladder respond to stimuli even after blockade of adrenergic and cholinergic
receptors. Both motor and sensory nonadrenergic, noncholinergic fibres are present here. - - The peptides are the most
common transmitter substances found in these neurons. The transmitters are CGRP, CCK, Dynorphin, Gastrin releasing
peptide, 5-HT, NPY, Somatostatin, Substance P, VIP, NO synthase etc.
• Capsaicin, a neurotoxin, derived from chilli peppers, can cause the release of substance P from such neurons, and
destruction of the neuron in high doses.
• These fibres are activated by a sensory input, followed by release of the neurotransmitter, which acts locally. They are
also called “sensory-efferent” or “sensory-local effector” fibers because when activated by a sensory input, they are
capable of releasing transmitter peptides from the sensory ending itself.

CHOLINERGIC SYSTEM
• Five types (M1 - M5) of cholinergic receptors have been identified by molecular cloning methods.
• The possibility of multiple forms was suggested by the pharmacology of pirenzapine which is an effective antimuscarinic
in blocking gastric acid secretion (selective M1 antagonist).
• The five muscarinic receptor subtypes, M1 - M5 are associated with specific anatomical sites. For example:
– M1 - ganglia; secretory glands
– M2 - myocardium, smooth muscle
– M3, M4 - smooth muscle, secretory glands
• 2 types of Nicotinic receptors exist: NM & NN

72
Phar macology of ANS, PNS & A
Pharmacology utacoids
Autacoids 2
Signal Transduction : Comparison of Muscarinic and Nicotinic receptors

Nicotinic Receptors Muscarinic Receptors


• Ligand-gated ion channels • G-protein coupled receptor system
• Receptor activation results in: • Slower responses
– rapid increase of Na+ and Ca2+ conductance • Agonist effects blocked by atropine
– depolarization
– excitation
• Subtypes based on differing subunit composition: Muscle
(NM) and Neuronal (NN) Classification
• Agonist effects blocked by D-tubocurarine (Non-depolarizing
neuromuscular B lockers)

Muscarinic receptors are located: Nicotinic receptors are located:


• in tissues innervated by postganglionic • in sympathetic and parasympathetic
parasympathetic neurons ganglia
• in presynaptic noradrenergic and cholinergic • in the adrenal medulla
nerve terminals • in the neuromuscular junction of the
• in non-innervated sites in vascular endothelium skeletal muscle
• in the central nervous system. • in the central nervous system.

Direct agonists
• Alkaloids (Nicotine / Muscarine)
• Esters (Ach / methacholine / carbachol / bethanechol)
• Carbachol / bethanechol are extremely resistant to hydrolysis
(Presence of [5-methyl group (methacholine / carbachol) reduces their potency at Nicotinic receptors]

Acetylcholine administered IM or SC produces only local vasodilatation. Small, bolus doses can lower BP for 5 -20 sec due
to release of NO. Very Large dose causes rise in BP due to release of catecholamines from adrenal Medulla.
Oxotremorine : Potent selective muscarinic agonist
DMPP : Potent selective nicotinic agonist (ganglion level)
Both are synthetic compounds and have no therapeutic potential. Used only as a research tool.

73
2 Self Assessment & Review Phar macology
Pharmacology
Mechanism of action
2 effects are produced by cholinergic agonists:
• Stimulation of muscarinic receptors directly affecting organ function
• Modulation of release of other neurotransmitter (sympathetic / parasympathetic / NANC)
All muscarinic receptors are GPCR (G-Protein coupled receptors)

MUSCARINIC RECEPTORS

M1 M2 M3

1. Location • Autonomic • Depolarization • SA node: Hyperpolarization, • Visceral smooth muscle


and ganglia: (late EPSP) ↓rate of impulse generation contraction
function Gastric glands • Hist. release, • AV node: ↓ velocity of condu- • Iris: constriction of pupil
subserved CNS: acid secretion ction • Ciliary muslce: contraction
• Learning, memory, • Atrium: shortening of APD, • Exocrine glands: secretion
motor function ↓ conractility • Vascular endothelium:
• Ventricle: ↓contractility (slight) release of NO →vaso-
(receptors sparse) dilatation
• Cholinergic nerve endings:
↓ACh release
• CNS: tremor, analgesia
• Visceral smooth muscle:
contraction
2. Nature • G-protein coupled, 7-TM • G-protein coupled, 7-TM • G-protein coupled, 7-TM
3. Transducer • IP3/DAG - ↑ cytosolic Ca2 +
• K channel opening ↓cAMP
+
• IP3/DAG – ↑ cytosolic Ca2+
mechanism • PLA2↑ - PG synthesis ↓cAMP • PLA2 ↑ – PG synthesis
4. Agonists* • MCN-343A, oxotremorine • Methacholine • Bethanechol
5. Antagonists* • Pirenzepine, Telenzepine • Methoctramine, • Hexahydrosiladifenidol,
Tripitramine Darifenacin

NICOTINIC RECEPTORS

1. Location and function Neuromuscular junction: depolari- Autonomic ganglia: depolarization -


subserved zation of muscle end plate - postganglionic impulse
contraction of skeletal muscle Adrenal medulla: catecholamine release
CNS : site specific excitation of inhibition
2. Nature Has intrinsic ion channel, pentamer Has intrinsic ion channel, pentamer of only α β
of α2 β ε or γ and δ subunits, each subunits, each subunit has 4 TM
subunit has 4 TM
3. Transducer Opening of cation (Na+, K+) channels Opening of cation (Na+, K+, Ca2+) channels
mechanism
4. Agonists PTMA, Nicotine DMPP, nicotine
5. Antagonists Tubocurarine, α-Bungarotoxin Hexamethonium, Trimethaphan

Ligand binding occurs at a subunit, causes a conformational change in ion channels (Na+ / K+) to diffuse down their
concentration gradients.
Continuous presence of nicotinic agonists cause - Inhibition of neuronal firing and relaxation of skeletal muscle by preventing
electrical recovery of post synaptic membrane (Depolarization Block). Utilized for depolarizing NM-block.

74
Phar macology of ANS, PNS & A
Pharmacology utacoids
Autacoids 2
Effects on organ system (Cholinomimetics)

Organ Response
• Eye
Sphincter muscle of iris Contraction (miosis)
Ciliary muscle Contraction for near vision
• Heart
Sinoatrial node Decrease in rate (negative chronotropy)
Atria Decrease in contractile strength (negative inotropy). Decrease
in refractory period
Atrioventricular node Decrease in conduction velocity (negative dromotropy).
Increase in refractory period
Ventricles Small decrease in contractile strength
• Blood vessels
Arteries Dilation (via EDRF). Constriction (high-dose direct effect)
Veins Dilation (via EDRF). Constriction (high-dose direct effect)
• Lung
Bronchial muscle Contraction (bronchoconstriction)
Bronchial glands Stimulation
• Gastrointestinal tract
Motility Increase
Sphincters Relaxation
Secreters Stimulation
• Urinary bladder
Detrusor Contraction
Trigone and sphincter Relaxation
• Glands
Sweat, salivary, lacrimal, nasopharyngeal Secretion

CVS
1. The vascular response is due to endothelial cell nitric oxide (NO) release following agonist interactions with endothelial
muscarinic receptors.
2. Increased NO activates guanylate cyclase which increases cyclic GMP concentrations.
3. Subsequent activation of a Ca2+ ion pump reduces intracellular Ca2+
4. Reduction in intracellular Ca2+ causes vascular smooth muscle relaxation.
5. Ca2+ complexes with calmodulin activating light-chain myosin kinase
– Increased cGMP promotes dephosphorylation of myosin light-chains.
– Smooth-muscle myosin must be phosphorylated in order to interact with actin and cause muscle contraction.

Primary effects on CVS are:


• At low doses - vasodilatation resulting in fall in BP with reflex increase in heart rate due to indirect increase in the
sympathetic tone
• At higher doses - Bradycardia and ↓↓ conduction velocity (direct action)
Mech. - ↑ inward K+ current in atrial cells
- ↓ slow inward Ca++ current in the ventricular tissues (minor effect)
Net effect depends on total concentration of muscarinic agonist in the heart

75
2 Self Assessment & Review Phar macology
Pharmacology
Ventricles have very less parasympathetic innervation.
Muscarinic agonists produce very little effect in the ventricles. However, muscarinic agonists produce greater depression of
ventricles already stimulated by sympathomimetic agents. Known as Accentuated Antagonism, (muscarinic modulation of
sympathetic effect)

On the blood vessels - Relaxation is produced if endothelium is intact (M3)


- Contraction is produced it endothelium is damaged

Vascular smooth muscle in skeletal muscle are sympathetic cholinergic (dilator) fibres, also Eccrine sweat glands
(thermoregulatory).

Clinical Uses of cholinergic drugs :


• Gastrointestinal and Genitourinary
— Bethanechol
– GI smooth muscle stimulant
- Postoperative abdominal distention (of non obstructive type)
- Paralytic ileus (of non obstructive type)
- Esophageal reflux; promotes increased esophageal motility (other drugs are more effective, e.g. dopamine
antagonist (metoclopramide) or serotonin agonists (cisapride)
– Urinary bladder stimulant
- Post-operative and post-partum urinary retention
– Alternative to pilocarpine to treat diminished salivation secondary to radiation
– Carbachol not used due to more prominent nicotinic receptor activation
– Methacholine used for diagnostic purposes
– Testing for bronchial hyperreactivity and asthma
• Ophthalmological Uses
– Acetylcholine and Carbachol may be used for intraocular use as a miotic in surgery
– Carbachol may be used also in treatment of glaucoma
– Pilocarpine is used in management of glaucoma and has become the standard initial drug for treating the open-
angle form.
– Sequential administration of atropine (mydriatic) and pilocarpine (miotic) is used to break iris-lens adhesions.

Other Uses
• Reversal of pharmacologic paralysis in anesthesia is done by :
– Neostigmine
– Edrophonium
• HEART: Presently, no cholinomimetic is used for PSVT. In the past, edrophonium was used for the condition; however
now-a-days, Adenosine and Verapamil (also diltiazem) are being employed.
• Atropine intoxication / TCA overdose
Muscarinic blockade is competitive in nature and can be effectively reversed by increased Ach concentration. The drug
employed is Physostigmine. However, Physostigmine itself can produce dangerous ADR, so it is reserved for use when
there is : – Severe elevation of body temperature
– Very rapid SVT

Cevimeline : NEW DIRECT ACTING MUSCARINIC AGONIST USED FOR TREATMENT OF DRY MOUTH IN SJOGREN’S
SYNDROME
• Myaesthenia Gravis (MG)
Edrophonium : is used to assess adequacy of treatment when MG is treated with longer acting cholinesterase inhibitors.
1. Excessive longer acting agents used – Patient becomes weak after receiving dose of edrophonium.
2. Inadequate longer acting agents used – Patient feels strength after receiving dose of edrophonium.
– Intermediate, short acting agents such as Neostigmine, Pyridostigmine and Ambenonium are employed for long-
term management of MG.

76
Phar macology of ANS, PNS & A
Pharmacology utacoids
Autacoids 2
– Long acting AChEIs such as organophosphate agents are not used in MG because the dose requirement in this
disease changes too rapidly to permit smooth control with long-acting drugs.
– Atropine is reserved if the muscarinic effects become disturbing, howevere, usually not required because of
development of tolerance.

Alzheimer’s Disease (AD)


• Drug of choice is Donepezil
• Other agents : Rivastigmine, Velnacrine, Galantamine, Metrifonate etc.

Conditions that resemble myasthenia gravis include:


• Lambert-Eaton myasthenic syndrome - a presynaptic disorder : Patients with Lambert-Eaton disorder have depressed
or absent reflexes, show autonomic changes such as xerostomia and impotence and incremental responses to repetitive
nerve stimulation. Treatment includes plamapheresis and immunosuppression.
• Neurasthenia : muscle testing indicates a nonorganic disorder, characterized by feelings of fatigue rather than by a loss
of muscle power.
• Thyroid abnormalities (either hyper or hypo- thyroidism) can increase myasthenic weakness.

Major contraindications to the use of muscarinic agonists


• Asthma : Choline esters (muscarinic agonists) can produce bronchoconstriction. In the predisposed patient, an asthmatic
attack may be induced.
• Hyperthyroidism : Choline esters (muscarinic agonists) can induce atrial fibrillation in hyperthyroid patients.
• Peptic ulcer : Choline esters (muscarinic agonists), by increasing gastric acid secretion, may exacerbate ulcer symptoms.
• Coronary vascular disease : Choline esters (muscarinic agonists), as a result of their hypotensive effects, can further
compromise coronary blood flow.

Adverse Effects of Organophosphates (Due to muscarinic excess)


• Nausea, Vomiting
• Salivation
• Diaphoresis
• Colic
• GI hyperactivity
• Headache
• Loss of accommodation

CHOLINORECEPTOR BLOCKING AGENTS


They can be : Muscarinic antagonists (Atropine and its congeners)
OR
Nicotinic antagonists : a. Ganglion blockers
b. Neuromuscular blocker
MOA of Atropine :
Atropine causes reversible (surmountable) blockade of cholinomimetics at muscarinic receptors i.e. blockade of small
dose of atropine can be overcome by large dose of Ach, suggesting a competition for the common binding site.
Tissues most sensitive to the actions of Atropine : Salivary, sweat and bronchial glands
Tissues least sensitive to the actions of Atropine : Acid secretion by parietal cells
Tissues intermediately sensitive to the actions of Atropine : Smooth Muscles and Heart

Atropine has no selectivity for M1, M2 or M3. lt acts on all the 3 types of receptors.

Effects on Organ Systems


CNS :
– Low Doses - mild stimulant effect on parasympathetic medullary centres and sedative effect on brain. In addition, causes
amnesia and drowsiness in sensitive individuals. Higher dose - excitation, agitation, hallucination and coma.

77
2 Self Assessment & Review Phar macology
Pharmacology
– Motion Sickness arising out from stimulation of muscarinic receptors can be effectively controlled by Scopolamine
(depresses vestibular excitation).
– Tremor of Parkinson’s disease is controlled by Atropine when given with Levodopa.

EYE :
– Circular muscles of iris contain M3 receptors (while radial muscles contain α Atropine blocks M3 receptors leaving the
unopposed α effect (sympathetic dilator activity) leading to pupillary dilatation (mydriasis)
– Cycloplegia : Weakening of contraction of ciliary muscles leading to a loss of ability to accommodate. The eye can’t focus
for near vision.
– ↓↓ Lacrimal Secretion : Patient complains of dry eye or “Sandy Eye” after receiving large doses of antimuscarinic drugs.
This is due to reduced lacrimal secretions.

CVS (M2 ) :
– Since the atria and SA node have rich parasympathetic supply, muscarinic blockade leads to unopposed sympathetic
action causing cardiac stimulation. It is due to blockade of M2 receptors on SA node through which vagal tone decreases
heart rate.
– Parasympathetic stimulation causes coronary dilatation and stimulation of sympathetic cholinergic fibres cause
vasodilatation of skeletal muscle. This vasodilatation can be effectively blocked by Atropine.

RESPIRATORY :
– All visceral smooth muscle cells that receive parasympathetic motor innervation are relaxed by atropine (M3 ) receptor
blocade.
– The effectiveness of unselective antimuscarinic drugs in treating COPD is limited because block of autoinhibitory M2
receptors on postganglionic parasympathetic nerves can oppose the bronchodilation caused by block of M3 receptors
on airway smooth muscle.
– Adjunct in pre-anaesthetic medication (Atropine / Glycopyrrolate)

GIT :
– Very prompt effect on gut motility & to some extent on secretion. However, complete muscarinic block fails to abolish
these activities because many local hormones and co- transmitters are known to play a role in g.i.t function. Prominent
effects are : - Dry mouth
- Reduction in volume of acid, pepsin and mucin
- Basal secretion is more effectively blocked than the stimulated secretion by alcohol, nicotine and food.
Pancreatic and intestinal secretions are little affected as they are under hormonal rather than neural
control.
- Motility is affected from stomach to colon. The walls of viscera are relaxed with reduced tone and
propulsive movements. Thus, gastric emptying is prolonged and intestinal transit time is lengthened.
In antimuscarinic drug overdose, gut paralysis occurs but peristalsis is re- established in 1-2 days due to local mechanisms
in enteric nervous system.

G.U.T. :
– Smooth muscles of bladder and ureter are relaxed.
– Trigone contracts leading to reduced voiding.
– No significant effect on uterus

SWEAT GLAND :
– Thermoregulatory sweating is suppressed as the eccrine sweat glands are innervated by sympathetic cholinergic fibers.
This leads to increased body temperature k/a Atropine fever.

USES
1. Parkinson’s disease : Oldest drug. Current status – is to reduce excessive salivary secretion in PD; given along with
levodopa.

2. Motion sickness : Scopolamine or older antihistamines with significant anticholinergic effect (meclizine, cyclizine) are
used. Scopolamine given by any route produces dry mouth and sedation.

78
Phar macology of ANS, PNS & A
Pharmacology utacoids
Autacoids 2
3. Ophthalmic uses for ciliary paralysis with pupillary dilatation.
• Adults and young children: shorter acting drugs Cyclopentolate or Tropicamide are preferred.
• Very young children : Atropine is preferred. Atropine ointment is superior to drops.
• Glycopyrrolate is as effective as atropine.
• When mydriasis is required for a short period, phenylephrine is used as it causes only cycloplegia. In contrast,
atropine and other anticholinergics cause both mydriasis and cycloplegia.
• Iridocyclitis - to break adhesions.

4. Respiratory Disorders :
• In Pre anesthetic medication: Atropine / scopolamine / glycopyrrolate
• They cause ↓ secretions, ↓ laryngospasm
• Ipratropium bromide: an inhalational agent used in bronchial asthma, COPD. The aerosol route provides the advantage
of maximal concentration at bronchial target tissue with reduced systemic effects.
(Tiotropium is a long acting aerosol therapy for above condition)

5. CVS :
• In inferior wall MI, ↑ vagal discharge due to pain can depress the heart further to reduce cardiac output. Atropine given
i.v. is given in this condition.
• In HOCM, circulating autoantibodies against 2nd extracellular loop of cardiac M2 receptors are found. They have
significant parasympathomimetic action. This can be blocked by atropine

6. GIT :
• Pirenzepine and Telenzepine (M1 antagonists) can be used in peptic ulcer; however, their efficacy is less than that of
H2 Blockers or H+ / K+ ATPase inhibitors.
• They are contraindicated in gastric ulcer as they decrease stomach motility, thereby exposing the ulcer bed to gastric
acid. (They can cause blurred vision, dry mouth, urinary retention etc) (Now Obsolete).
• For diarrhea and spasmodic conditions - anticholinergics have a very limited use.

7. Adult urinary incontinence :


• Selective M3 antagonists Tolterodine, Oxybutynin and Trospium: Used to relieve bladder spasm in urologic surgery or
children with neurologic disorder (i.e. Meningomyelocele)
– Imipramine - Nocturnal Enuresis
– Propiverine - To reduce incontinence in institutionalized patients

8. Cholinergic poisoning i.e. Organophosphate poisoning


• Antimuscarinic therapy
• Oximes (Reactivators) : The oxime group (= NOH) has high affinity for the phosphorus atom and they hydrolyse the
phosphorylated enzyme if the complex has not aged.
• Pralidoxime Carries a positive charge, so it can’t penetrate BBB, therefore it cannot reverse the CNS effects.
• Diacetyl monoxime (DAM) penetrates BBB.
• Pre-treatment with reversible AChE inhibitors such as pyridostigmine or physostigmine to prevent binding of irreversible
organophosphate inhibitor. Reserved for situations where lethal poisoning is suspected (Chemical warfare).

Atropine poisoning : Occurs as a result of attempting suicide

Clinical features : Dry mouth, mydriasis, tachycardia, hot and flushed skin / agitation and delirium for a week or so.

Treatment : • Mainly symptomatic :


– Tepid sponging and cooling blankets diazepam to control seizures
– Physostigmine use is now very much restricted because of more incidences of death.
But still sometimes given.

79
2 Self Assessment & Review Phar macology
Pharmacology
Contraindications of Anticholinergic agents
– Angle closure glaucoma
– Elderly with BPH
– Stomach ulcers

Note : Drugs having Anticholinergic adverse effects: Includes Older antipsychotics (phenothiazines, Butyrophenones), Older
antihistaminics (H1 Blockers). Tricyclic antidepressants have anticholinergic side effects and will tend to aggravate
the above conditions.

80
Phar macology of ANS, PNS & A
Pharmacology utacoids
Autacoids 2
ALL INDIA 9. Which one of the following agents has been asso-
ciated with hemorrhagic stroke? [AI 06]
a. Phenylpropanolamine
1. Mechanism of action of PAM / pralidoxime :
b. Terfenadine
a. Reactivation of cholinesterase [AI 08]
c. Quinidine
b. Synthesis of cholinesterase
d. Fenfluramine
c. Inhibition of cholinesterase
10. Which one of the following muscle relaxant has
d. None of the above
the maximum duration of action? [AI 06]
2. Ipratropium bromide is contraindicated in :
a. Atracurium
a. Urinary retention [AI 08]
b. Vecuronium
b. Asthma
c. Rocuronium
c. Hypertension
d. Doxacurium
d. All of the above
11. Which one of the following local anaesthetics be-
3. Tiotropium is used for : [AI 08] longs to the ester group? [AI 06]
a. Treating urinary retention a. Procaine
b. Treating ileus b. Bupivacaine
c. Increase salivation c. Lignocaine
d. Treating asthma d. Mepivacaine
4. Which of the following is beneficial in NSAID in- 12. Which of the following antiglaucoma medications
duced gastric ulcer ? [Al 07; AIIMS Nov. 03] can cause drowsiness? [AI 06]
a. PGE1 agonist a. Latanoprost
b. PGE2 agonist b. Timolol
c. PGD agonist c. Brimonidine
d. PGF2 agonist d. Dorzolamide
5. The following are associated with beta 2 agonist 13. Which of the following medications is contraindi-
treatment except : [Al 07] cated in patients with allergy to sulphonamides?
a. Hyperkalemia a. Levobunolol [AI 06]
b. Hyperglycemia b. Bimatoprost
c. Detrusor relaxation c. Brinzolamide
d. Relaxation of gut and bronchial muscles d. Brimonidine
6. The drug of choice for mushroom poisoning : 14. Misoprostol is a : [AI 06]
a. Atropine [Al 07] a. Prostaglandin E1 analogue
b. Physostigmine b. Prostaglandin E2 analogue
c. Adrenaline c. Prostaglandin antagonist
d. Carbachol d. Antiprogestin
7. True statement about clonidine are all except : 15. Which one of the following antibacterials should
a. Increases parasympathetic outflow [Al 07; 97] not be used with d-tubocurarine? [AI 06]
b. Decreases sympathetic outflow by blocking a. Norfloxacin
central alpha receptor b. Streptomycin
c. Used in HT c. Doxycycline
d. Prazosin is used to antagonize side effects of d. Cefotaxime
clonidine
16. Which of the following drugs is useful in prophy-
8. Which of the following drugs is not used topically laxis of migraine ? [AI 05]
for treatment of open angle glaucoma :
a. Propranolol
a. Latanoprost [AI 06; AIIMS June 98]
b. Sumatriptan
b. Brimonidine
c. Domperidone
c. Acetazolamide
d. Ergotamine
d. Dorzolamide

Answer 1. a. Reactivation ... 2. a. Urinary ... 3. d. Treating ... 4. a. PGE1 ... 5. a. Hyperkalemia
6. a. Atropine 7. d. Prazosin ... 8. c. Acetazolamide 9. a. Phenylpro ... 10. d. Doxacurium
11. a. Procaine 12. c. Brimonidine 13. c. Brinzolamide
14. a. Prostaglandin ... 15. b. Streptomycin 16. a. Propranolol

81
2 Self Assessment & Review Phar macology
Pharmacology
17. Which one of the following drugs increases gas- c. Nitroprusside
trointestinal motility ? [AI 05] d. Metyrosine
a. Glycopyrrolate 25. Diagnosis of Myasthenia Gravis is by using:
b. Atropine a. Edrophonium [AI 00, 96]
c. Neostigmine b. Neostigmine
d. Fentanyl c. Succinylcholine (SCh)
18. Sympathomimetic drugs are useful in the therapy d. Atropine
of all of the following conditions except : [AI 05] 26. A patient taking Ketoconazole and Terfenadine
a. Acute decompensated heart failure both, is prone for : [AI 00; AIIMS June 99]
b. Hypotension a. Cardiac arrhythmia
c. Hypertension b. Toxicity of ketoconazole
d. Erectile dysfunction c. Congestive cardiac failure
19. Drug implicated for prolonging QT interval in a pre- d. All of above
mature baby is : [AI 05; AIIMS June 99] 27. Cardiac or central nervous system toxicity may
a. Domperidone result when standard lidocaine doses are admin-
b. Metoclopramide istered to patients with circulatory failure. This
c. Cisapride may be due to the following reason : [AI 00]
d. Omeprazole a. Lidocaine concentration are initially higher in
20. All the following are selective beta blockers, ex- relatively well perfused tissues such as brain
cept : [AI 05] and heart
a. Atenolol b. Histamine receptors in brain and heart gets sud-
b. Esmolol denly activated in circulatory failure
c. Bisoprolol c. There is a sudden outburst of release of adrena-
line, noradrenaline and dopamine in brain and
d. Sotalol
heart
21. Beta blocker that can be used in renal failure is all
d. Lidocaine is converted into a toxic metabolite due
except : [AI 05]
to its longer stay in liver
a. Propranolol
28. Neostigmine is a : [AI 99]
b. Pindolol
a. Primary ammonium compound
c. Sotalol
b. Secondary ammonium compound
d. Oxprenolol
c. Tertiary ammonium compound
22. All of the following statements are true except :
d. Quartenary ammonium compound
a. IV noradrenaline increases systolic and dias-
29. Drug of choice in Acute central anticholinergic
tolic BP and causes tachycardia [AI 01]
syndrome is : [AI 99]
b. IV Adrenaline increases systolic, decreases di-
a. Neostigmine
astolic BP and causes tachycardia
b. Physostigmine
c. IV isoproterenol causes no change in systolic,
decreases diastolic BP and causes tachycardia c. Tacrine
d. Dopamine decreases peripheral resistance and d. 4-amino pyridine
improves renal perfusion 30. Selective α1 A - blocker is : [AI 99]
23. β blocker with peripheral vasodilator action is : a. Prazosin
a. Carvedilol [AI 01] b. Terazosin
b. Propranolol c. Tamsulosin
c. Atenolol d. Indoramine
d. Acebutalol 31. β1 selective agonist is : [AI 99]
24. Which of the following is not given in a patient with a. Terbutaline
pheochromocytoma : [AI 00; AIIMS June 99] b. Albuterol
a. Atenolol c. Dobutamine
b. Prazosin d. Isoetharine

Answer 17. c. Neostigmine 18. d. Erectile ... 19. c. Cisapride 20. d. Sotalol 21. c. Sotalol
22. c. IV isoproterenol ...23. a. Carvedilol 24. a. Atenolol 25. a. Edrophonium 26. a. Cardiac ...
27. a. Lidocaine ... 28. d. Quarternary ... 29. b. Physostigmine 30. c. Tamsulosin 31. c. Dobutamine

82
Phar macology of ANS, PNS & A
Pharmacology utacoids
Autacoids 2
32. All are classified as reversible anticholines- c. Clonidine
treases except : [AI 98] d. Reserpine
a. Ambenonium 41. Which of the following is not a catecholamine:
b. Physostigmine a. Epinephrine [AI 95]
c. Pyridostigmine b. Norepinephrine
d. Echothiophate c. Dopamine
33. In treatment of cardiac failure, dobutamine acts d. Phenylephrine
by all of the following mechanisms except : 42. All of the following statements are true except :
a. α receptors agonism [AI 98] a. PG’s and leukotrienes are derived from arachi-
b. β adrenergic receptors agonism donic acid [AI 01]
c. Dopamine receptor agonism b. COX I is an inducible enzyme
d. Increasing force of contraction c. COX II is induced by cytokines at sites of inflam-
34. Most commonly postural hypotension is seen with: mation
a. Prazosin [AI 98, 96; AIIMS June 98] d. Leukotrienes cause smooth muscle constriction
b. Nifedipine 43. In low doses aspirin acts on : [AI 01]
c. Atenolol a. Cyclooxygenase
d. ACE inhibitors b. Thromboxane A2
35. Clonidine is a : [AI 97] c. PGI2
a. α1 selective agonist d. Lipoxygenase
b. α2 selective agonist
c. α1 selective antagonist AIIMS
d. α2 selective antagonist
36. Shortest acting neuromuscular blocking agent is 44. Not used in erectile dysfunction ?
a. Pancuronium [AI 97] a. Apomorphine [AIIMS May 08]
b. Atracurium b. Taldanafil
c. Mivacurium c. Phenylephrine d. Sildenafil
d. Vecuronium 45. Which is not a 2nd generation antihistamanic ?
37. Which of the following is not an inotropic drugs : a. Loratidine [AIIMS May 08]
a. Dopamine [AI 96] b. Acrivastatine
b. Isoprenaline c. Cyclizine
c. Amrinone 46. Long acting beta agonist ? [AIIMS May 08]
d. Amiodarone a. Terbutaline
38. Actions of atropine are all except : [AI 96] b. Salmeterol
a. Bronchoconstriction c. Ciclesonide
b. Tachycardia d. Ipratropium
c. Mydriasis 47. Used for the treatment of migraine, the tripatans
d. CNS stimulation act through : [AIIMS Nov. 07]
39. Which of the following has the shortest plasma a. 5HT1A
half-life : [AI 95] b. 5HT1B
a. Propranolol c. 5HT1F
b. Esmolol d. 5HT3
c. Timolol 48. Which of the following is not sued as a sedative,
d. Atenolol but causes sedation as a side effect :
40. Which of the following does not cause bradycar- a. Antipsychotics [AIIMS Nov. 07]
dia: [AI 95] b. Antihistaminics
a. Propranolol c. Antidepressant
b. Hydralazine d. Lithium

Answer 32. d. Echothiophate 33. c. Dopamine ... 34. a. Prazosin 35. b. α2 selective ... 36. c. Mivacurium
37. d. Amiodarone 38. a. Bronchocon ... 39. b. Esmolol 40. b. Hydralazine 41. d. Phenylephrine
42. b. COX I is an ... 43. b. Thromboxane A2 44. c. Phenylephrine 45. c. Cyclizine 46. b. Salmeterol
47. b. 5HT - 1B 48. b. Antihistaminics

83
2 Self Assessment & Review Phar macology
Pharmacology
49. Impotence is caused by which of the following 56. All of the following drugs can induce Methemoglo-
drugs : [AIIMS May 07] binemia, EXCEPT : [AIIMS May 04]
a. Angiotensin antagonists a. Nitroglycerine
b. CCB b. Procaine
c. β blockers c. Prilocaine
d. ACE inhibitors d. Phenytoin
50. Which NSAID undergoes enterohepatic circula- 57. Which one on the following statements is not true
tion? [AlIMS Nov. 06] about NSAIDs ? [AIIMS May 04]
a. Aspirin a. Acetyl salicylic acid is an irreversible inhibitor of
b. Phenylbutazone COX enzyme
c. Piroxicam b. Salicylic acid reduces in vivo synthesis of pros-
d. Ibuprofen taglandins
51. Long acting β2 sympathomimetic : c. Duration of action of aspirin is primarily related
a. Salmeterol [AlIMS Nov. 06] to the pharmacokinetic clearance of the drug from
the body
b. Orciprenaline
d. Antiplatelet effect of low-dose aspirin is related
c. Penoterol
to presystemic COX inhibition
d. Pexbaterol
58. A 20 year old female weighing about 55kg is ad-
52. Neostigmine antagonizes nondepolarizing block- mitted to an emergency department having con-
ade by all of the following mechanisms, except : sumed 10g of paracetamol [acetaminophen], to-
a. Decreasing the breakdown of Acetylcholine at gether with alcohol, 6 hours earlier. A Serum
the motor end plate. [AlIMS May 06] paracetamol level is reported as 400 micrograms/
b. Preventing K efflux from the cell ml. Which one of the following is correct in this
c. Increasing the release of Acetylcholine at the case ? [AIIMS May 04]
motor end plate a. Gastric lavage is mandatory
d. Depolarization of the motor end plate b. Administration of activated charcoal
53. Which one of the following drugs does not pro- c. Abnormalities of Kidney function are likely to be
duce central anticholinergic syndrome : present.
a. Atropine sulphate [AIIMS Nov. 05] d. Hepatotoxicity is likely to occur.
b. Glycopyrrolate 59. The H3 receptor agonist causes : [AIIMS Nov. 03]
c. Antihistaminucs a. Inhibition of H1 receptor induced wakefulness
d. Tricyclic antidepressants b. Increase in H1 mediated gastrin secretion
54. All of the following drugs are recommended for c. Inhibition of H1 mediated bronchoconstriction
treatment of beta blocker induced excessive d. Negative chronotropic effect on atria
bradycardia and/or decrease in cardiac output, 60. The following is not true about the use of beta-
except : [AIIMS May 05] blockers in heart failure : [AIIMS May 03]
a. Dopamine a. It should be initiated at very low dose
b. Dobutamine b. It is most effective in new onset decompensated
c. Glucagon heart failure
d. Calcium chloride c. Slow upward titration of dose is required
55. Which one of the following is not an alpha- d. Carvedilol is most widely used in this condition
adrenoceptor agonist ? [AIIMS May 05] 61. The following statement is not true about sotalol:
a. Clonidine a. It is a nonselective beta-blocker
b. Methyldopa b. It prolongs action potential duration throughout
c. Guanabenz the heart [AIIMS May 03]
d. Guanfacine c. It is excreted through bile following hepatic me-
tabolism
d. It is low lipid soluble

Answer 49. c. β blockers 50. c. Piroxicam 51. a. Salmeterol 52. b. Preventing ... 53. b. Glycopyrrolate
54. a. Dopamine 55. b. Methyldopa 56. d. Phenytoin 57. c. Duration of ... 58. d. Hepatotoxicity ...
59. b. Increase in H1 ... 60. b. It is most ... 61. c. It is excreted ...

84
Phar macology of ANS, PNS & A
Pharmacology utacoids
Autacoids 2
62. A 60-year old hypertensive patient on Angiotensin c. Peripheral vasoconstriction and directing blood
II receptor antagonists [losartan] is posed for her- flow to heart
nia repair surgery. The antihypertensive drug d. Ratio of blood flow to epicardium and endocar-
should be : [AIIMS May 03] dium decreases
a. Continued till the day of operation 68. All of the following drugs are used in pheochro-
b. Discontinued 24 hrs preoperatively mocytoma, except : [AIIMS June 99]
c. Discontinued one week preoperatively a. Prazosin
d. Administered in an increased dosage on the day b. Metyrosine
operation c. Atenolol
63. Prostaglandins [PGs] have effects on a variety of d. Na nitroprusside
tissues. The different prostaglandins may have 69. Thromboxane A2 synthetase inhibition is by :
different effects. Which of the following statement
a. Aspirin [AIIMS Dec. 98]
is not correct : [AIIMS Nov. 02]
b. Prednisolone
a. The human arteriolar smooth muscle is relaxed
by PGE2 and PGI2, where as TXA2 and PGF2 Al- c. Dazoxiben
pha cause vasoconstriction d. Naproxen
b. PGE 1 and PGE 2 inhibit platelet aggregation, 70. Mechanism of action of Aspirin in MI is :
whereas TXA2 facilitate aggregation a. Thromboxane A2 synthesis inhibition
c. PGE2 has marked oxytocic action while PGF2 Al- b. Thromboxane A2 synthesis stimulation
pha has tocolytic action c. Antagonist of adenosine [AIIMS Dec. 98]
d. PGE2 is bronhodilator whereas PGF2 alpha is a d. Stimulation of PGF2 synthesis
bronchoconstrictor 71. ACE inhibitors are contraindicated in bilateral re-
64. A 70-year-old man was administered penicillin in- nal artery stenosis because :
travenously. Within 5 minutes, he developed a. Angiotensin II has direct effect on GFR
generalised urticaria, swelling of lips, hypotension b. Enhances bradykinin action [AIIMS June 98]
and bronchospasm. The first choice of treatment
c. Hyperreninism affects renal parenchyma
is to administer : [AIIMS Nov. 02]
d. Affects prostaglandins
a. Chlorpheniramine injection
72. Mechanism of action of Latanoprost in glaucoma
b. Epinephrine injection
is : [AIIMS June 98]
c. High dose hydrocortisone tablet
a. Increase uveoscleral flow of aqueous
d. Nebulised salbutamol
b. Trabecular absorption is increased
65. All are true about β-blockers EXCEPT :
c. Decreases aqueous formation
a. Atenolol is longer-acting than metoprolol
d. Unknown
b. Labetalol has alpha and β blocking action
73. All are true regarding prazosin, except :
c. Carvedilol has α1 agonistic and selective β1 block-
a. First dose effect [AIIMS Feb. 97]
ing action [AIIMS May 02]
b. Impaired glucose tolerance
d. Nadolol has longest half life
c. Selective alpha-1 blocker
66. Which of the following drugs is used to reverse
d. Is an antihypertensive drug
the effect of d - tubocurarin : [AIIMS Dec. 00]
74. Which is a selective cyclooxygenase-2 inhibitor :
a. Atropine
a. Ketorolac [AIIMS June 98]
b. Neostigmine
b. Tolmetin
c. Physostigmine
c. Nabumetone
d. Organophosphorus
d. Oxyprozin
67. Mechanism of action of epinephrine in cardiopul-
monary resuscitation is : 75. Hyperlipidemia is caused by intake of :
a. Increase myocardial demand [AIIMS Nov. 99] a. Calcium channel blockers [AIIMS Sep. 96]
b. Increase SA node activity b. β - blocker
c. Methyldopa
d. Reserpine

Answer 62. a. Continued till ... 63. c. PGE2 has ... 64. b. Epinephrine ... 65. c. Carvedilol ... 66. b. Neostigmine
67. b. Increase SA ... 68. c. Atenolol 69. c. Dazoxiben 70. a. Thromboxane ... 71. a. Angiotensin II ...
72. a. Increase ... 73. b. Impaired ... 74. c. Nabumetone 75. b. β - blocker

85
2 Self Assessment & Review Phar macology
Pharmacology
76. Which has maximum nicotinic effect : 84. True about TXA2 : [PGI Dec. 07]
a. Bethanechol [AIIMS Sep. 96] a. Synthesized by platelets
b. Carbachol b. Prothrombotic
c. Pilocarpine c. Formed by all cells of the body
d. Methacoline d. Vasoconstriction
77. Local anaesthetic which does not cause vasodi- e. Increase permeability of vessel wall
lation : [AIIMS Sep. 96] 85. β-blocker used in : [PGI Dec. 07]
a. Lignocaine a. Portal hypertension
b. Cocaine b. Alcohol withdrawal
c. Bupivacaine c. Anxiety
d. Tetracaine d. HOCM
78. Most potent antiemetic used in preanaesthetics 86. Aspirin toxicity true are : [PGI June 06]
medication is : [AIIMS Dec. 95] a. Tinitus early symptom
a. Atropine b. 10-30 gm causes poisoning
b. Hyoscine c. Hyperthermia, tachypnea early complications
c. Glycopyrrolate d. Cause thrombocytopenic purpura
d. Chlorpromazine 87. Esmolol true is/are : [PGI June 06, June 03]
79. Suxamethonium causes : [AIIMS Dec. 95] a. α Blocker
a. Jaundice b. Long 1/2 life
b. Splenomegaly c. Not cardioselective
c. Atrial fibrillation d. Used in LV decompensation
d. Muscle fasciculation e. Cause bradycardia
80. Vecuronium acts on : [AIIMS Dec. 95] 88. Contraindication for use of β-blocker in heart fail-
a. Cerebral cortex ure includes : [PGI Dec. 05]
b. Myoneural junction a. Asthma
c. Muscle fibres b. Heart block
d. Spinal cord c. Arrythmias
81. Gout is not caused by which of the following: d. Hypertension
a. Chlorthalidone [AIIMS May 95] e. Decompensated LVF
b. Sulfinpyrazone 89. Which of the following is paired incorrectly :
c. Aspirin a. Hemicholinium - prevents release of Ach from
d. Pyrazipine storage [PGI Dec. 05]
b. Botulinum - Increase the Ach release
PGI c. Pralidoxime - Reactivates acetyl choline es-
terases
82. Vasodilators are : [PGI Dec. 07] d. Vesamicol - Inhibits uptake of choline
a. NO e. Organophosphorus - acetylcholine esterase in-
b. CO2 hibitor
c. Minoxidil 90. All of the following are used in beta blocker over-
d. ACE inhibitor dose except : [PGI Dec. 05]
83. True about Prostacyclin : [PGI Dec. 07] a. Atropine
a. Vasoconstriction b. Norepinephrine
b. Aggrigation of platelet c. Glucagon
c. Decrease BP d. Thyroxine
d. Synthesis from vascular endothelium e. Propanolol

Answer 76. b. Carbachol 77. b. Cocaine 78. b. Hyoscine 79. d. Muscle ... 80. b. Myoneural ...
81. b. Sulfinpyrazone 82. a, b, c and d 83. c and d 84. a, b and d 85. a, c and d
86. a and b 87. e. Cause ... 88. a, b and e 89. a, b and d 90. d and e

86
Phar macology of ANS, PNS & A
Pharmacology utacoids
Autacoids 2
91. α2 agonist can be used in all of the following ex- 99. Recent antihistaminics in allergic rhinitis are :
cept: [PGI Dec. 05] a. Azelastine [PGI June 03]
a. Hypertension b. Fexofenadine
b. Glaucoma c. Chlorpheniramine maleate
c. Opiate withdrawl d. Desloratidine
d. To potentiate anaesthetic drugs 100. β - blockers with intrinsic sympathomimetic prop-
e. Ischaemia erties are : [PGI Dec. 02]
92. Which of the following is not a mixed α and β a. Propranolol
blocker : [PGI June 05] b. Oxprenolol
a. Labetalol c. Practolol
b. Pindolol d. Esmolol
c. Carvedilol e. Butoxamine
d. Levobunolol 101. Neostigmine : [PGI Dec. 02]
e. Timolol a. It is a quartenary ammonium compound
93. CNS effects of H2 receptor blockers : b. Metabolised in liver
a. Dizziness [PGI Dec. 04]
c. It can cross the blood brain barrier
b. Delirium
d. Prominent effect on smooth muscles
c. Insomnia
102. Central muscle relaxants act by : [PGI June 02]
d. Hallucination
a. Decreased nerve conduction
e. Headache
b. Inhibits spinal polysynaptic reflexes
94. Allopurinol prevents conversion of :
c. Block conduction across reflexes
a. Hypoxanthine to xanthine [PGI June 04]
d. CNS depression
b. Xanthine to hypoxanthine
e. Decreased muscle excitation
c. Hypoxanthine to I.M.P
103. Properties making cardioselective beta blockers
d. Xanthine to uric acid
desirable are : [PGI June 02]
95. Side effect of timolol maleate : [PGI Dec. 03]
a. Less bronchoconstriction
a. Hypertension
b. Adverse effect on lipid profile
b. Asthma
c. Cause glucose intolerance profile
c. Depression
d. May be used in Raynaud’s disease
d. Tachycardia
e. Less liable to impair exercise capacity
e. Hypotension
104. COX pathway is inhibited by :
96. True about Carvedilol : [PGI Dec. 03]
a. Aspirin [PGI June 02, June 01]
a. α 1 blocker
b. β 1 blocker b. Indomethacin
c. β 2 blocker c. Betamethasone
d. Antioxidant d. Calcitonin
e. Used in hypertension e. Diclofenac
97. Prophylaxis of migraine is/are : [PGI Dec. 03] 105. Ondansetron acts by : [PGI June 02]
a. Flunarizine a. Acting on CTZ
b. Cinarizine b. 5- HT3 antagonist
c. β-blocker c. D1 and D2 receptors
d. Sodium valproate d. Increase GIT motility
e. Carbamazepine 106. Atropine causes : [PGI Dec. 01]
98. True about Pseudocholinesterase : [PGI Dec. 03] a. Decreased cardiac output
a. Present in neuromuscular junction b. Heart block
b. Level is increased in pregnancy c. Hypertension
c. Succinylcholine is metabolized d. Miosis
d. Organophosphates inhibit it e. Sweating

Answer 91. e. Ischaemia 92. b, d and e 93. a, b, d and e 94. a and d 95. b, c and d
96. a, b, c, d and e 97. c and d 98. c and d 99. a, b and d 100. b. Oxprenolol
101. a. It is a ... 102. a, d and e 103. b. Inhibits ... 104. a, b and e 105. a and b
106. a. Decreased ...

87
2 Self Assessment & Review Phar macology
Pharmacology
107. True of the following is are : [PGI June 01] 115. Ganglion blocker act by : [PGI June 99]
a. β1 receptors in heart stimulate its contractions a. Inhibition of release of acetyl choline
b. β2 receptors in heart stimulate its contractions b. Competing of acetyl choline receptor
c. β receptors are present in smooth muscles c. Preventing conduction from pre post ganglionic
d. α1 receptors cause preganglionic stimulation d. Inhibition of Ach destruction
e. α2 receptors cause postganglionic feedback in- 116. α1 stimulation produce : [PGI June 98]
hibition a. HR↑
108. Drug of choice for cardiogenic shock : b. BP ↓
a. Dopamine [PGI Dec. 00; June 00] c. HR ↓
b. Propranolol d. None
c. Digitalis 117. Amatoxins in mushroom poisoning acts by Inhib-
d. Milrinone iting : [PGI June 98]
109. Dales vasomotor reversal : [PGI Dec. 00] a. DNA
a. Stimulation of alpha-1 receptors b. mRNA
b. Stimulation of alpha-2 receptors c. Adenosine
c. Stimulation of beta-1 receptors d. G - proteins
d. Stimulation of beta-2 receptors 118. β blocker acts by : [PGI Dec. 97]
110. Atropine - mechanism of action in poisoning: a. ↓ cardiac output
a. Reactivation of chronic - esterase b. ↓ HR
b. Acts on central and peripheral post. ganglionic c. ↓ BP
receptor [PGI June 00] d. All
c. Acts on central and peripheral cholinergic recep- 119. Selective alpha 1 antagonists are following ex-
tors cept: [PGI Dec. 97]
d. Acts on peripheral cholinergic receptors only a. Prazosin
111. Which prostaglandin is used in NSAID induced ul- b. Terazosin
cer : [PGI June 00] c. Butoxamine
a. Misoprostol d. Tamsulosin
b. Carboprost 120. Granulomatous hepatitis is caused by :
c. Mirtazapine a. Allopurinol [PGI Dec. 97]
d. Miprinone1 b. Methyldopa
112. The action of non - competitive muscle blockers c. Furazolidone
is affected by : [PGI Dec. 99] d. Furazolidone
a. Hypocalcemia e. Amiodarone
b. Hyponatremia 121. Max. Nicotinic effect is seen with : [PGI Dec. 97]
c. Hyperthermia a. Pilocarpine
d. All of the above b. Carbachol
113. ACE inhibitors are contraindicated in : c. Bethenachol
a. U/L renal artery stenosis with single kidney d. Methacholine
b. Proteinuria with DM [PGI Dec. 99] 122. Adrenaline cause vasoconstriction in all of the
c. MI following vessels except : [PGI June 97]
d. Hypotension a. Gut
114. Local anesthetics act by inhibiting : b. Cerebral
a. Influx of K + [PGI Dec. 99] c. Cutaneous
b. Influx of K + d. Renal
c. Influx of Na +
d. Efflux of Na +

Answer 107. a and d 108. a. Dopamine 109. d. Stimulation ... 110. c. Acts on ... 111. a. Misoprostol
112. a. Hypocalcemia 113. a. U/L renal artery ...114. c. Influx of Na + 115. c. Preventing ... 116. c. HR ↓
117. b. mRNA 118. d. All 119. c. Butoxamine 120. a, b and e 121. b. Carbachol
122. a. Gut

88
Phar macology of ANS, PNS & A
Pharmacology utacoids
Autacoids 2
ANSWERS, REFERENCES, EXPLANATIONS WITH INFORMATIVE ILLUSTR ATIONS

1. Ans. is a i.e. Reactivation of cholinesterase Ref. KDT 6/e, p 105

Cholinesterase regenerator compounds are pralidoxime (PAM), diacetyl-monoxime (DAM)


Pralidoxime is used for its effect as a reactivator of acetylcholinesterase.
The pralidoxime reactivates the enzyme AChE by attaching with the anionic site which lies vacant in the
phosphorylated enzyme. The oxime end binds to the phosphate to form an oxime-phosphate complex which
diffuses out, leaving the cholinesterase regenerated.

There are two circumstances in which this fails to work :


First : With passage of time (>48 hours), the phosphorylated enzyme undergoes a process of ‘aging’
which renders it almost completely resistant to regeneration. This aging process leaves more
stable acetylcholinesterase and rate of aging varies with different organophosphorus.
Second : No benefit as an antidote in carbamate antiChEs (physostigmine, neostigmine, propoxur) since
the oxime complex so formed does not hydrolyze rapidly to leave the regenerated enzyme.

2. Ans. is a i.e. Urinary retention Ref. Katzung 10/e, p 114, KDT 6/e, p 110

Ipratropium is a muscarinic receptor blocking drug. Its racemate is used clinically.


Antimuscarinic agents can precipitate acute urinary retention in patient with prostatic hypertrophy.

Therapeutic uses of Ipratropium :


• Treatment of acute reversible airway obstruction asthma
• Treatment of chronic reversible airway obstruction asthma option ‘d’.
• Treatment of watery rhinorrhoea associated with perennial rhinitis

Contraindication : • Hypersensitivity to atropine


Relative Contraindication : • Glaucoma
• Prostatic hypertrophy ... option ‘a’

3. Ans. is d i.e. Treating asthma Ref. Katzung 10/e, p 114, KDT 6/e, p 110

Tiotropium bromide is a recently developed congener of ipratropium bromide which binds tightly to bronchial
muscle muscarinic receptors producing long lasting bronchodilatation. It is given once daily and reduces the
incidences of COPD exacerbations and is a useful adjunct to pulmonary rehabilitation in increasing exercise
tolerance.

Tiotropium causes side effects : • Dry mouth • Constipation


• Stomach pain • Vomiting
• Indigestion • Muscle pain
• Nosebleed • Runny nose
• Sneezing • Painful white patches in mouth

89
2 Self Assessment & Review Phar macology
Pharmacology
4. Ans. is a i.e. PGE agonist 1
Ref. KDT 6/e, p 181, 632 - 633; Harrison 17/e, p 1863

The primary use of PGE analogue is prevention and treatment of NSAID associated gastrointestinal
1

injury and blood loss.

Role of prostaglandins :
• Stimulation of gastric mucus secretion.
• Stimulation of duodenal bicarbonate secretion.
• Stimulation of gastric mucosal blood flow.
• Strengthening of inter cellular tight junctions.

Prostaglandin Analogues (Misoprostol)


MOA • It is a synthetic analogue of PGE1. It produces significant reduction in basal and
food stimulated acid secretion. It is rapidly absorbed undergoes extensive first
pass metabolism, Peak effect seen in 60 min and lasts for 3 hours.
Indication • Prophylaxis of gastric and duodenal ulceration in NSAIDs users.
• Healing of established NSAIDs induced GU and DU.
• Healing of GU and DU in absence of NSAID therapy.
• Induction of abortion early pregnancy.
Contraindication • Inflammatory bowel disease
• Pregnant women
• Women and childbearing age
Adverse effects • N/V, diarrhea, abdominal cramps. It can cause bleeding and abortion in pregnancy.

Prostaglandins and their uses :


PGE Misoprostol1
– Gastric peptic ulcer NSAID induced, abortion
PGF Carboprost α

– Abortifacient
– Postpartum hemorrhage
PGF Latanoprost α

– Glaucoma
PGE Dinoprostone
2
– Induction of labour
– Abortifacient
– Bronchodilator
– Cervical ripening
PGE 1
Alprostadil – To maintain patency of ductus arteriosus, ARDS, pulmonary HT, IHD
– Intercavernous injection for impotence, erectile dysfunctions, CHF
PGI Epoprostenol
2
– To avoid damage to platelets, primary pulmonary hypertension, PVD,
Raynaud’s phenomena, anticoagulant in extra corporeal circulation,
CAD, PPHT, ARDS

5. Ans. is a i.e. Hyperkalemia Ref. KDT 6/e, p 124; Harrison 17/e, p 1603

Adverse effects of Selective β agonist (prototype salbutamol) is:


2

• Hypokalemia – when given parenterally, it decreases concentration of K in plasma, may cause cardiac
+

arrhythmia
• Hyperglycemia

90
Phar macology of ANS, PNS & A
Pharmacology utacoids
Autacoids 2
• Muscle tremor (most common), palpitations, headache
• Tachyarrhythmia
• Tolerance – on repeated use tolerance may be caused due to β agonist. 2

Mnemonic: K +
increasing agents K-BANK -
• K-sparing diuretic
• Beta blocker
• ACEI
• NSAID
• K supplement

β receptor selectivity of action of receptor agonists.


Drugs β2)
Airway smooth muscle (β β1)
Cardiac tissue (β
Isoprenalol 1 1
Salbutamol 0.25 0.004
Salmeterol 5 0.001

6. Ans. is a i.e. Atropine Ref. KDT 6/e, p 98

Mushroom poisoning is of 3 types :


Muscarine type (early mushroom poisoning) : It is due to inocybe and related species. Symptoms
characteristic of muscarinic actions appear within an hour of eating the mushroom, and are promptly reversed
by atropine.
Anticholinergic / hallucinogenic type : These have anticholinergic and hallucinogenic properties. The
manifestations are primarily central. There is no specific treatment and atropine is contraindicated.
Phalloidin type (late mushroom poisoning) : It is due to peptide toxins found in A. phalloides, Galerina and
related species. Treatment consists of supportive measures, thiotic acid may have some antidotal effect.

7. Ans. is d i.e. Prazosin is used to antagonize side effects of clonidine Ref. KDT 6/e, p 546, 552

Prazosin is a α receptor blocker. It has got poor affinity for α2 so, it cannot antagonize actions of clonidine.
1

Clonidine :
• Clonidine is a partial agonist with high affinity and high intrinsic activity at α2 receptor specially α 2A subtype
in brainstem, located presynaptically where it inhibits release of neurotransmitter.
• The major haemodynamic effects result from stimulation of α2A receptors present mainly post junctionally in
medulla (vasomotor centre) →decrease sympathetic out flow →fall in BP and bradycardia (also due to
enhanced vagal tone).
• It is a moderately potent antihypertensive.
• It does not alter plasma lipid levels, so there is no increase in LDL cholesterol on prolonged use.
• It decreases sympathetic outflow to kidney resulting in ↓ renin release, so it may be combined with drugs
which increase plasma renin activity, so clonidine can be combined with vasodilator.

91
2 Self Assessment & Review Phar macology
Pharmacology

Uses of Clonidine Adverse effects


• Moderate hypertension • Sedation
• Opioid, Nicotinic, Alcohol withdrawal • Mental depression
• Intrathecal/epidural surgical and post-op analgesia • Disturbed sleep
• To control loose motions due to diabetic neuropathy • Dry mouth, nose, eyes
• Post menopausal vasomotor symptoms • Constipation
• Clonidine suppression test for pheochromocytoma • Impotence
• Prophylaxis in migraine • Salt / H2O retention
• Preanaesthetic medication (Sedative, anxiolytic & analgesic) • Bradycardias
• Glaucoma • Rebound hypertension
• Anesthesia • Erectile dysfunction
• Psychiatric disorders • Edema

• Dexmedetomidine is selective α2A agonist used for preanaesthetic medication preoperative sedation, analgesia,
antianxiety and decrease in bronchial secretions.
• Clonidine exhibits therapeutic window phenomenon: Optimum fall in B.P. occurs between 0.2 to 2.0 ng/ml
blood levels. At higher concentration, the fall in BP is less marked.

8. Ans. is c i.e. Acetazolamide


Ref. Goodman & Gilman 11/e, p 1723; Katzung 10/e, p 152; KDT 6/e, p 146-147

Drugs that lower intraocular pressure :


Drugs Mechanisms Formulation Side effects
Acetazolamide Carbonic anhydrase inhibitor Oral Malaise, depression, hypokalemia,
acidosis, paresthesia, anorexia
Dorzolamide Reduce aqueous humor formation Topically Ocular stinging, burning, itching,
by decreasing the formation of bitter taste
HCO3– ions in ciliary epithelium
Latanoprost Prostaglandin analogue Topically Iris pigmentation, thickening and
PGF2αα Increase in uveoscleral out flow by darkening of eyelashes.
altering permeability or pressure
gradient
Pilocarpine Muscarinic agonist, contraction Topically Miosis
of sphincter pupillae ↑ drainage
Echothiophate Anticholinesterase ↑ drainage Topically Given in answer above
Timolol, Cartiolol β-Adrenoceptor antagonist Topically Systemic side effect are
reduces aqueous secretion bradycardia,
by the ciliary body bronchoconstriction
Apraclonidine α2- Adrenoceptor agonist Topically
Brimonidine Reduce the formation of aqueous Topically Drowsiness in 4-29 % cases
humor. These are potent ocular photosensitivity, conjunctival
hypotensives which do not cross hyperemia, hypersensitivity
blood-brain barrier, hence no
systemic effects (like clonidine).

92
Phar macology of ANS, PNS & A
Pharmacology utacoids
Autacoids 2
9. Ans. is a i.e. Phenylpropanolamine
Ref. Goodman & Gilman 11/e, p 260; KDT 6/e, p 128; Harrison 17/e, p 464

The factor that regulate appetite through effect on central neural circuit are as followed:

Many reports associating Phenylpropanolamine (PPA) use for weight loss with haemorrhagic stroke among
women appeared in USA.
PPA is a sympathomimetic drug that has been used most commonly in oral preparations for relief of nasal
congestion and also as an appetite suppressant.
Anorectic drugs : – Used in obesity for short period.
– Used for 2-3 months in cases of severe obesity but not for cosmetic purpose.
Anorectic agents
Sympathomimetics Serotonergic Hormonal Newer drugs
• Act on appetite centre • Act on satiety centre
– Phenteramine – Fenfluramine – Thyroxine – Orlistat
– Phenylpro- – Dexfenfluramine Role of thyroxine is – Olestra
panolamine limited as majority – Leptin
– Diethyl of obese patient are – Sibutramine
propion euthyroid.
– Mazindol

10. Ans. is d i.e. Doxacurium Ref. KDT 6/e, p 343; Katzung 10/e, p 429

Doxacurium longest acting non depolarizing muscle relaxant with duration of action is 80-120 minutes,
it has very good CVS stability used for surgery of longer duration.
Succinylcholine is most rapid onset shortest acting depolarizing muscle relaxant.
Comparative properties of neuromuscular blocking drugs
Drug Onset* (min) Duration (min)
@
Hist. Release Gang. Block Vagal Block
Long acting
• d-Tubocurarine 4-6 30-60 +++ ++ +
• Pancuronium 4-6 40-80 + +, St. +
• Doxacurium 4-8 80-120 + – –
• Pipecuronium 2-4 50-100 + – –

93
2 Self Assessment & Review Phar macology
Pharmacology

Intermediate acting
• Vecuronium 2-4 30-60 + – +
• Atracurium 2-4 20-35 + – –
• Cisatracurium 3-6 20-40 – – –
• Rocuronium 1-2 25-40 – – +
Short acting
• Mivacurium 2-4 12-20 + – –
• Succinylcholine 1-1.5 3-6 ++ St. St.

* Time to maximal block after i.v. injection


St= Stimulation
Duration of surgical grade relaxation after usual clinical doses; time to 95% recovery of muscle twitch is nearly double
@

of the figure given (specially for long acting drugs). Duration is also dose dependent.

11. Ans. is a i.e. Procaine Ref. KDT 6/e, p 352; Katzung 10/e, p 414

Procaine, cocaine, chloroprocaine, tetracaine and benzocaine are local anaesthetics that belongs to the ester
group and degraded by CYP-450 isoenzyme in liver.
Esters are hydrolysed by plasma esterases. Exception procaine and tetracaine are PABA esters procaine
antagonizes the antibacterial action of sulphonamide when given to treat the infection.

Features of amide LA’s (compared to ester LA’s)


• Produce more intense and longer lasting anaesthesia
• Bind to α1 acid glycoprotein in plasma
• Not hydrolysed by plasma esterases
• Rarely cause hypersensitivity reactions; no cross sensitivity with ester LAs.

Local anesthetics
Amide Ester
– Lignocaine – Cocaine
– Bupivacaine – Procaine
– Dibucaine – Chloroprocaine
– Prilocaine – Tetracaine
– Ropivacaine – Benzocaine
Mnemonics = 2 ‘i’ in all amides.

Xylocaine is a trade name of Lidocaine having single ‘i’

12. Ans. is c i.e. Brimonidine Ref. Khurana, p 423; KDT 6/e, p 146

The incidence of drowsiness with Brimonidine is 4-29 %.


Brimonidine is an á2-adrenergic receptor agonist. Reduces the production of aqueous humor. Other examples
include apraclonidine. Possible side effects include increased blood pressure; dizziness; red, itchy or swollen
eyes; dry mouth; and allergic reactions.

94
Phar macology of ANS, PNS & A
Pharmacology utacoids
Autacoids 2
13. Ans. is c i.e. Brinzolamide Ref. KDT 6/e, p 147

Acetazolamide, dorzolamide as well as brinzolamide are sulphonamide derivatives.

Acetazolamide should not be taken by individuals if :


• They are allergic to sulfa medications.
• They are allergic to any carbonic anhydrase inhibitor.
• They have liver or kidney disease.
• They have adrenal gland failure (i.e. Addison’s disease).
• They have diabetes insipidus.

Apraclonidine, Brimonidine : α2- Adrenoceptor agonists. Reduce the formation of aqueous humor. These are
potent ocular hypotensives which do not cross blood-brain barrier, hence no systemic effects occur (like clonidine).

14. Ans. is a i.e. Prostaglandin E analogue


1
Ref. KDT 6/e, p 634 katzung 305-306 /10 edition

Prostaglandins analogue
PGE1 – Misoprostol
PGF2αα – Carboprost
PGF2αα – Latanoprost ,bimatoprost, travaprost, unoprostone
PGE2 – Dinoprostone
PGE1 – Alprostadil ,treprostinil
PGI2 – Epoprostenol

15. Ans. is b i.e. Streptomycin Ref. KDT 6/e, p 346

Aminoglycoside group of antibiotics (like streptomycin, gentamicin, tobramycin) themselves produce


neuromuscular blockade by decreasing ACh release from the preganglionic terminals by blocking P-type Ca++
channels.
Application of following drugs with d-tubocurarine, a competitive nondepalarizing muscular blocker can potentiate
its action and prolong apnea.
Antibiotics General anesthetics Ca channel blockers
++

Streptomycin Isoflurane Verapamil


Clindamycin Halothane Diltiazem
Lincomycin Enflurane
Polymyxin B Ketamine
Colistin
Tetracycline

16. Ans. is a i.e. Propranolol Ref. Harrison 17/e, p 102; KDT 6/e, p 169

Mechanism of action of Drugs used for Prophylactic treatment of Migraine

95
2 Self Assessment & Review Phar macology
Pharmacology

• Pizotifen – 5-HT2A-2C receptor blocker


• Propranolol ⎫
• Metoprolol ⎬ β-adrenoceptor blockers
• Nadolol ⎭
• Methysergide – 5-HT2A-2C antagonist
• Cyprohepatadine – 5-HT2A receptor antagonist
• Flunarizine – Ca2+ channel blocker
• Clonidine – α2-adrenoceptor agonist
• TCA
- Amitriptyline ⎫
- Dothiepin ⎬ 5-HT uptake blocking property
- Nortriptyline ⎭
• Anticonvulsants
- Topiramate
- Valproate
- Gabapentin

Mnemonics : Very Volatile Pharmacotherapeutic Agents For Migraine Prophylaxis


• Verpamil • Flunarizine
• Valproic acid • Methysergide
• Pizotifen • Propranolol
• Amitriptyline

17. Ans. is c i.e. Neostigmine Ref. KDT 6/e, p 346

• Neostigmine is used mainly for its effects on skeletal muscle in myasthenia gravis and in anesthesia for
termination of the effects of competitive neuromuscular blocking drugs.
• Smooth muscle in treating atony of the gut and bladder in paralytic ileus and primary and secondary glaucoma.
Anticholinesterase drugs increase the level of ACh by protecting it from hydrolysis.
Neostigmine is an anticholinesterase drug, it protect acetylcholine which acts on different systems as follows:
Actions of Acetylcholine on smooth muscle

GIT Bronchial Muscle Genito Urinary Tract


– ↑ Tone Constriction – ↑ Peristalsis of ureter
– ↑ Peristalsis (Asthmatic highly – Detrusor muscle
– Sphincters relaxation sensitive) contraction
– Abdominal cramps ↓ – Sphincter and trigone
– Evacuation of bowel – Dyspnoea relaxation
– Precipitation of ↓
bronchial asthma – Voiding of bladder

Fentanyl
• It is an opioid analgesic which leads to constipation by :
– Direct action on GIT/CNS increases tone and segmentation but decreases propulsive movement.
– Spasm of pyloric / ileocaecal and anal sphincters.
– Decrease in all GIT secretions.

96
Phar macology of ANS, PNS & A
Pharmacology utacoids
Autacoids 2
– Increased due to stasis absorption of water.
– Central action causing inattention to defecation reflex.
Glycopyrrolate : • Anticholinergic drug
• Potent antisecretory
• Potent antibradycardiac drug
• Lack of central action
• Used in Preanesthetic medication

18. Ans. is d i.e. Erectile dysfunction Ref. Katzung 10/e, p 147, 188, 314; Harrison 17/e, p 298 - 299

There is no role of sympathomimetic drugs in erectile dysfunction.


A combination of the α-adrenoceptor antagonist phentolamine with the nonspecific vasodilator papaverine, when
injected directly into the penis, may cause erection in men with sexual dysfunction.
So, we use sympatholytic agent in case of erectile dysfunction.

Clinical uses of Sympathomimetic Drugs :


CVS Application – Hypotension
– Cardiogenic shock (Dopamine or Dobutamine)
– Cardiac arrest (Epinephrine; Isoproterenol IV)
Pulmonary Applications – Bronchial Asthma (Epinephrine, Isoproterenol)
Anaphylaxis – Epinephrine S/C, I/M
Hypertension – α2 agonist Clonidine, a centrally acting sympatholytic agent

19. Ans. is c i.e. Cisapride Ref. KDT 6/e, p 163, 645; Katzung 10/e, p 1088

Terfenadine, Cisapride, Astemizole blocks cardiac K+ channel in overdose and produces polymorphic ventricular
tachycardia (Torsades de pointes).

Drugs causing Prolonged QT Interval / Torsades de pointes


Antiarrhythmic Antihistaminics CYP 3A4 inhibitors Others
– Quinidine – Terfenadine – Ketoconazole – Imipramine
– Amiodarone – Astemizole – Fluconazole – Amitriptyline
– Procainamide – Erythromycin
– Disopyramide – Haloperidol
– Cisapride

20. Ans. is d i.e. Sotalol Ref. KDT 6/e, p 136, 140

Classification of β – Blockers

Cardioselective Partial agonistic Membrane


(β )1
(Intrinsic sympatho- stabilising
mimetic) activity
• Metoprolol • Pindolol • Propranolol
• Atenolol • Acebutalol • Oxprenalol

97
2 Self Assessment & Review Phar macology
Pharmacology

• Acebutalol • Alprenolol • Alprenalol


• Bisoprolol • Carteolol • Acebutalol
• Betaxalol • Dilevalol • Atenolol
• Celiprolol • Oxprenolol • Sotalol
• Esmolol • Nadolol
• Nebivolol

The advantages of cardioselective beta-blocker over non-selective blockers are :


• These are safer in asthmatics, compared to propranolol, but they are not absolutely safe.
• These are safer in diabetes as they cause less inhibition of glycogenolysis during hypoglycaemia (glucose
release from liver is controlled by β2 receptors). However, tachycardia in response to hypoglycaemia is not
blocked (β1 blockade).
• These are safer in patients with peripheral vascular disease (no β2-blockade).
• These have less deleterious effects on lipid profile.
• Less liable to impair exercise capacity.

21. Ans. is c i.e. Sotalol Ref. KDT 6/e, p 140; Katzung 10/e, p 149

• Unlike other β-blocking agents sotalol is excreted mostly unchanged in the urine and this take place by
glomerular filtration therefore contraindicated in Renal failure.
• Propranolol has Hepatic excretion.
• Pindolol has Hepatic + Renal excretion.
• Oxprenolol has Hepatic + Renal excretion.

Comparative properties of β blockers


Selectivity Partial Local Liqid Elimination Approximate
Agonist Anesthetic Solubility Half-Life Bioavailability
Activity Action

Atenolol β1
No No Low 6 - 9 hours 40
Carvedilol None No No High 7 - 10 hours 25 - 35
Esmolol β1
No No Low 10 minutes 0
Labetalol None Yes Yes Moderate 5 hours 30
Metoprolol β1
No Yes Moderate 3 - 4 hours 50
Nadolol None No No Low 14 - 24 hours 33
Propranolol None No Yes High 3.5 - 6 hours 30
Sotalol None No No Low 12 hours 90

22. Ans. is c i.e. IV isoproterenol causes no change in systolic, decreases diastolic BP and causes
tachycardia Ref. KDT 6/e, p 122 - 123

Isoprenaline and Isoproterenol are one of the same synthetic catecholamine they increase systolic &
decreases DBP and mean BP.

98
Phar macology of ANS, PNS & A
Pharmacology utacoids
Autacoids 2
Features S.B.P D.B.P TPR HR. Mean BP

Noradrenaline α-Agonist predominant ↑ ↑ ↑ ↓ (Reflex) ↑


(Vasoconstriction)
Isoprenaline or β-Agonist (predominant) ↑ ↓ ↓↓ ↑ ↓↓
Isoproterenol Vasodilatation, cardiac stimulant
Adrenaline α- Agonist (vasoconstrictor) ↑ ↓ ↓ ↑ No change
β-Agonist (cardiac stimulant)

Dopamine when given I.V. in therapeutic doses it acts on D receptor and causes dilatation of renal
1

vessels

23. Ans. is a i.e. Carvedilol Ref. KDT 6/e, p 143

Carvedilol has non-selective antagonism of β-adrenergic receptors but also possesses appreciable α1-adrenergic
antagonist activity. It also has antproliferative properties and is a scavenger of reactive free oxidant radicals.

Properties of Carvedilol
• Carvedilol is β1+ β2+ α1 adrenoceptor blocker.
– Produces vasodilatation due to α1 blockade as well as direct action.
– It also has antioxidant properties.
– Used in Hypertension, CHF and angina pectoris.

Other important points of β−


−blockers
−blockers crossing Blood Brain Barrier or Lipid soluble.
• β−
Mnemonics : PMOLA
P – Propranolol
M – Metoprolol
O – Oxprenolol
L – Labetolol
A – Alprenalol
−Blockers not crossing Blood Brain Barrier or Lipid insoluble.
• β−
Mnemonics : BANS
B – Bisoprolol
A – Atenolol
N – Nadolol
S – Sotalol

24. Ans. is a i.e. Atenolol Ref. Harrison 17/e, p 2271; KDT 6/e, p 142 - 143

β−
−blocker should be given only after α - blockade has been induced. Since administration of β-
blocker alone may cause paradoxical rise in BP by antagonising β-mediated vasodilatation.
• The induction of stable α adrenergic blockage provides the foundation for successful surgical treatment of
pheochromocytoma.

99
2 Self Assessment & Review Phar macology
Pharmacology
• Patient should be started with :
Phenoxybenzamine : It induce long lasting noncompetitive α receptor blockade. It is administered for at
least 10-14 days prior to surgery.
Before adequate α adrenergic blockade with phenoxybenzamine is achieved, paroxysms of hypertension
may be treated with oral prazosin or phentolamine.
• Other drugs which reduce the blood pressure in patient with pheochromocytoma.
– Na-nitroprusside, Ca++ channel blocker and ACE Inhibitors.

25. Ans. is a i.e. Edrophonium Ref. KDT 6/e, p 101

The two types of weakness requires opposite treatments :


They can be differentiated by edrophonium test.

⎧ Improvement → Myasthenic Crisis


Inject Edrophonium ⎨
⎩ Worsening → Cholinergic Crisis

Diagnostic tests for myasthenia gravis :


a. Ameliorative test : Edrophonium- 2-10mg IV slowly improves muscle strength only in myasthenia gravis
and not in other muscular dystrophies.
b. Provocative test : Myasthenics are highly sensitive to d-tubocurarine; 0.5mg IV causes marked weakness
in them but is ineffective in non myasthenics.
c. Demonstration of anti-NM antibodies in plasma or muscle biopsy specimen is a more reliable test.
Myasthenia gravis is an autoimmune disorder due to development of antibodies directed to nicotinic receptors
at the muscle endplate.

Reduction in No. of free NM cholinoceptors to 1/3 of normal

Weakness and easy fatiguability on repeated activity.

26. Ans. is a i.e. Cardiac arrhythmia Ref. KDT 6/e, p 158

Terfenadine is metabolized by enzyme CYP-3A4 to carboxy metabolite which is active H blocker. 1

Terfenadine (Not carboxy metabolite) blocks cardiac K+ channel in over dose and produces polymorphic ventricular
tachycardia (Torsades de pointes).
• So, when terfenadine is used with CYP 3A4 inhibitors, cardiotoxicity of terfenadine precipitates.
Torsades de pointes caused by these combinations
– Terfenadine + Any drug which inhibits CYP 3A4
– Cisapride + Erythromycin / Ketoconazole
– Quinidine + Procainamide
– Quinidine + Mefloquine
– Halofantrine + Mefloquine

• Azithromycin and Fluconazole are probably Safe.

100
Phar macology of ANS, PNS & A
Pharmacology utacoids
Autacoids 2
27. Ans. is a i.e. Lidocaine concentration are initially higher in relatively well perfused tissues such as
brain and heart Ref. Goodman & Gilman 11/e, p 962; Harrison 16/e, p 1600

“Normally when cardiac output falls, systemic vascular resistance rises to maintain a level of systemic
pressure that is adequate for perfusion of heart and brain at the expense of other tissue such as muscle,
skin especially GIT.”
In circulatory failure, three organs are saved by body homeostasis mechanism.

Ist – Heart IInd – Brain IIIrd – Kidney

– Lidocaine is a rapidly acting drug, given I.V. So, in circulatory failure, the more perfused organs get
more lidocaine, likely Heart and Brain get more Lidocaine then other organs.
– Liver is not well perfused, more blood flow to the heart and brain. Since lidocaine is predominantly metabolised
in liver, so a very less amount is eliminated from the body. Therefore, the elimination t½ is increased. The
brain/heart gets more lidocaine at (usual) normal dose which may lead to toxicity.

28. Ans. is d i.e. Quarternary ammonium compound Ref. KDT 6/e, p 99

Neostigmine is reversible anticholinesterase.


It is only physostigmine which is a naturally occurring alkaloid, a tertiary amine and being lipid soluble can
cross Blood Brain Barrier (BBB). Rest of the drugs are synthetic quarternary ammonium compounds and being
ionised can not cross BBB.

Comparative features of physostigmine and neostigmine.

Physostigmine Neostigmine

• Source Natural alkaloid Synthetic


• Chemistry Tertiary amine derivative Quatenary ammonium compound
• Oral absorption Good Poor
• CNS action Present Absent
• Applied to eye Penetrates cornea Poor penetration
• Direct action on cholinoceptors Absent Present
• Prominent effect on Autonomic effects Skeletal muscles
• Important use Miotic (glaucoma) Myasthenia gravis
• Dose 0.5-1mg oral / Parenteral 0.5 - 2.5 im/sc
0.1 - 1.0% eye drop 15 - 30 mg orally
• Duration of action Systemic 4 - 6 hrs. 3 - 4 hrs
In eye 6 - 24 hrs.

Mnemonic : Physostigmine vs neostigmine; LMNOP


Lipd soluble
Miotic
Natural
Orally absorbed

101
2 Self Assessment & Review Phar macology
Pharmacology
29. Ans. is b i.e. Physostigmine Ref. KDT 6/e, p 102

As described in above question, physostigmine penetrates cornea and cross blood brain barrier. Physostigmine,
therefore antagonises both central as well as peripheral actions.

Tacrine is a recently developed acridine compound which interacts with cholinesterase in a fashion similar to
the reversible cholinesterase inhibitors It crosses blood brain barrier and has a longer duration of action and
used in Alzheimer’s disease.
Nowadays Tacrine is not preferred because of its hepatic toxicity. Donapezil a anticholinesterases is preferred
over Tacrine.

30. Ans. is c i.e. Tamsulosin Ref. KDT 6/e, p 132

• Tamsulosin is recently introduced selective αIA blocker improving the symptoms of benign hypertrophy of
prostate. α IA subtype receptors predominate in bladder base and prostate.
• αIB receptors predominate in blood vessels.

Classification of α-adrenergic blockers :


Alpha-Adrenoceptor Antagonists
Non-selective (block α1 & α2) Selective Miscellaneous
• Ergot Alkaloids
↓ ↓ ↓ ↓
Reverisble Irreversible α1 blockers α2 blockers
• Phentolamine • Phenoxybenzamine • Prazosin • Yohimbine
• Tolazoline • Terazosin • Idazoxan
(priscoline) • Doxazosin
• Alfuzosin
• Tamsulosin
• Bunazosin
• Urapidil
• Indoramin

31. Ans. is c i.e. Dobutamine Ref. KDT 6/e, p 126

Dobutamine is cardiac stimulant, acts on both α and β adrenergic receptors. Not a D1 or D2 receptor agonist
as Dopamine.
It increases force of cardiac contraction and output, without significant change in heart rate, peripheral resistance
and BP. As such it has been considered to be a relatively selective β1 agonist.
Albuterol (salbutamol), Terbutaline and Salmeterol are all selective β2 agonists.

32. Ans. is d i.e. Echothiophate Ref. KDT 6/e, p 99

Echothiophate is irreversible anticholinesterases organophosphorus compound it is used in glaucoma.

102
Phar macology of ANS, PNS & A
Pharmacology utacoids
Autacoids 2
Anticholinesterases
Reversible Irreversible

↓ ↓ ↓ ↓
Carbamates Acridine Organophosphorus Carbamates
Physostigmine (eserine) Tacrine Dyflos Carbaryl
Neostigmine Echothiophate Propoxur
Pyridostigmine Parathion, Malathion
Edrophonium Diazinon
Rivastigmine Tabun, Sarin, Soman
Donepezil

33. Ans. is c i.e. Dopamine receptor agonism Ref. KDT 6/e, p 126

Dobutamine is a derivative of Dopamine but not a D1 or D2 receptor agonist. Though it acts on both α and β
adrenergic receptors, the only prominent action of clinically employed doses (2.5 - 10 mg / kg / min I.V.
infusion) is increase in force of contraction without significant change in heat rate, peripheral resistance and
blood pressure.

Therapeutics uses :
• Inotropic support in hypotensive shock
• Heart Failure
• As a cardiovascular stress test during ECG or echocardiographic examination of the heart.

Contraindication : – Patient with low cardiac filling pressure.


– Patient with severe ventricular arrhythmias
In addition to beneficial hemodynamic effects dobutamine enhances sodium and water excretion.

34. Ans. is a i.e. Prazosin Ref. KDT 6/e, p 134

Prazosin is the first of highly selective α1 blocker. It blocks sympathetically mediated vasoconstriction
and produces fall in BP which is attended by only mild tachycardia.
• Prazosin dilates arterioles more than veins. Postural hypotension occurs, especially in the beginning -
dizziness and fainting as “First dose effect”. This can be minimized by starting with a low dose and taking
it at bedtime.
• It is primarily used as an antihypertensive.
• ACE inhibitors induce hypotension but postural hypotension is not a problem.
• Nifedipine also induce hypotension. Postural hypotension may occur but is not as prominent as with prazosin.
• Atenolol is a selective β1 antagonist and does not dilate vessels.

35. Ans. is b i.e. α2 selective agonist Ref. KDT 6/e, p 119

Clonidine is a partial agonist with high affinity and high intrinsic activity at α2 receptor specially α2A subtype in
brainstem.

For more details, refer answer no. 7

103
2 Self Assessment & Review Phar macology
Pharmacology
36. Ans. is c i.e. Mivacurium Ref. KDT 6/e, p 343; Morgan 3/e, p 181

Neuromuscular blocking agents


Non depolarising blocker (Competitive) Depolarising blocker (Non competitive)
• Shortest acting is Mivacurium 10-20 min • Shortest acting is Succinylcholine 3-4 min

Muscle relaxant (MR)


Direct acting (MR) Neuromuscular blocking agent Centrally acting (MR)
Act on Muscle fiber Act on Nm junction Act on Cerebrospinal axis
• Dantrolene NondepolarisingDepolarising • Benzodiazepines
• Quinine (competitive) (noncompetitive) • Mephenesin
• d-TC • Suxamethonium • Baclofen
• Atracurium • Decamethonium • Tizanidine
• Mivacurium
• Vecuronium
• Pancuronium
• Rocuronium

37. Ans. is d i.e. Amiodarone Ref. KDT 6/e, p 516

• Amiodarone is an antiarrhythmic drug belonging to class III.


• Amiodarone is not an inotropic agent, but a ‘long acting antiarrhythmic’.
• It causes myocardial depression and fall in BP on i.v. injection.
• Amrinone is a selective phosphodiesterase III inhibitor. It is termed as “an inodilator”, the major effects being
positive inotropic action and vasodilatation.
• Dopamine and Isoprenaline both cause an inotropic action and may be used in cardiogenic shock.

Inotropic drugs
Sympathomimetic Phosphodiesterase III Cardiac glycosides
inotropic drugs inhibitors
• Dopamine (option “a”) • Amrinone • Digoxin
• Dobutamine • Milrinone • Digitoxin
• Isoprenaline (option “b”)

38. Ans. is a i.e. Bronchoconstriction Ref. KDT 6/e, p 107

Major action of atropine


• It has central and peripheral effects.
• Doses of 0.5 to 1.0 mg low doses
• Peripheral affects tachycardia without increase contractility, decrease saliva sweat, bronchial, nasal, lacrimal
and gastric secretions.
Blocks the response of the sphincter muscle of the iris and the ciliary muscle of the lens to cholinergic stimulation,
thus causing pupillary dilatation and paralyzing accommodation (cycloplegia) this leads to photophobia and
blurred vision.

104
Phar macology of ANS, PNS & A
Pharmacology utacoids
Autacoids 2
High doses causes excitation with restlessness, disorientation and hallucinations.
Therapeutic uses
• Anesthetic premediation - Inhibition of secretions
• Anesthetic premediation - Inhibition of cardiac reflexes
• Ophthalmic use - Iridocyclitis and choroiditis, following cataract surgery and for accurate measurement of
refractive errors
• Baradyarrhythmias and MI
• Organophosphorous poisoning
• Diarrhea and dysenteries
• Myasthenia gravis: adjunctive
• IBS
• Vestibular disorder
• Asthma
• Tetanus
Oculo-cardiac reflex during ophthalmic surgery.

39. Ans. is b i.e. Esmolol Ref. KDT 6/e, p 141

ESMOLOL is an ultrashort-acting cardioselective β antagonist. It has plasma half life of 8 to 10 min.


1

Important points • An ultrashort acting cardioselective β blocker


• No partial agonistic (intrinsic sympathomimetic) activity
• No membrane stabilising actions
• Rapidly hydrolysed by esterases in RBCs
• Plasma t½ < 10 min. Action lasts 15-20 minutes
• Precipitates heart failure
• No β2 blocking action, airway resistance not increased
Used to terminate • Supraventricular tachycardia
• Episodic AF, AFl
• Arrhythmia during anaesthesia
• Early treatment of MI
• Reduce HR and BP during and after cardiac surgery

40. Ans. is b i.e. Hydralazine Ref. KDT 6/e, p 504

• Hydralazine causes reflex tachycardia by – Directly acting ‘arteriolar’ dilatation


– Reduces Total Peripheral Resistance (TPR)
– Reflex Tachycardia
• Clonidine (Central sympatholytic) – Decrease in sympathetic out flow
– Fall in BP, bradycardia
• Reserpine (inhibits active amine transport – Decreased sympathetic outflow (depletion
and monoamines are gradually depleted) of noradrenaline at nerve endings)
– Fall in BP, bradycardia
• Propranolol (nonselective β blocker) – Blocks sympathetic activity
– Decreases BP, bradycardia

105
2 Self Assessment & Review Phar macology
Pharmacology
41. Ans. is d i.e. Phenylephrine Ref. KDT 6/e, p 120

Phenylephrine is non-catacholamine α selective agonist.


1

The three closely related endogenous catecholamine’s are Noradrenalin, Adrenaline and Dopamine.

42. Ans. is b i.e. COX I is an inducible enzyme Ref. KDT 6/e, p 185 - 186, 179

COX I COX II
Constitutive Inducible
House keeping Induced by cytokines and other signal molecules at site of inflammation

Leukotrienes : SRS - A Slow reacting substance of anaphylaxis are LTC4, LTD4


LTB4
Potent chemotactic (by WBC)
LTC 4
LTD4 Constitutive ( by macrophages)

Action on smooth muscle : LTC4 and D4 contract most of smooth muscles. They are potent broncho-
constrictors.

43. Ans. is b i.e. Thromboxane A2


Ref. KDT 6/e, p 189; Goodman & Gilman 11/e, p 675

• Aspirin inhibits COX irreversibly by acetylating one of its serine residue.


• Due to inhibition of cyclooxygenase, there is reduced synthesis of prostaglandins.

Effect on Platelets : TXA (Thromboxane A )


2 2
– Platelet aggregation ↑
PGI 2
– Platelet aggregation ↓

• If low doses of aspirin (80 - 100 mg) are used, it irreversibly inhibits TXA2 production by inhibiting COX-1 of
platelets for their total life span (7 days).
• This is because, the platelets lack nuclei and cannot synthesize COX. Such small doses do not affect PGI2
synthesis in the widely distributed vascular endothelium which can regenerate COX if at all inhibited to some
extent.
• The maximal effectiveness of aspirin therapy appears to depend upon selective blockade of TXA2 synthesis
by platelets without preventing production of PGI2 by endothelial cells. Although the optimal dosage to
prevent thrombotic events has not been firmly established, selective antiplatelet action appear to be best
achieved when the dose of aspirin is between 40 to 80mg per day, while higher doses also inhibit
PGI2 production.

44. Ans. is c i.e. Phenylephrine Ref. Katzung 10/e, p 147, 188, 314

There is no role of sympathomimetic drugs in erectile dysfunction.


A combination of the α-adrenoceptor antagonist phentolamine with the nonspecific vasodilator papaverine, when
injected directly into the penis, may cause erection in men with sexual dysfunction.
So, we use sympatholytic agent in case of erectile dysfunction.

106
Phar macology of ANS, PNS & A
Pharmacology utacoids
Autacoids 2
Clinical uses of Sympathomimetic Drugs :
CVS Application – Hypotension
– Cardiogenic shock (Dopamine or Dobutamine)
– Cardiac arrest (Epinephrine; Isoproterenol IV)
Pulmonary Applications – Bronchial Asthma (Epinephrine, Isoproterenol)
Anaphylaxis – Epinephrine S/C, I/M
Hypertension – α2 agonist Clonidine, a centrally acting sympatholytic agent

45. Ans. is c i.e. Cyclizine Ref. KDT 6/e, p 156

Clinical Classification and Pharmacological Characteristics of H1 - Receptor Antagonists


First Generation Antihistamines Second Generation Antihistamines
Highly Sedative Moderately Sedative
• Dimenhydrinate • Pyrilamine • Astemizole
• Diphenhydramine • Tripelennamine • Fexofenadine
• Doxylamine • Cyproheptadine • Cetirizine
• Hydroxyzine • Carbinoxamine (clistin) • Loratadine
• Promethazine • Clemastine • Desloratadine
• Mild Sedative • Levocetirizine
• Cyclizine
• Meclizine
• Chlorpheniramine
• Dexchlorpheniramine
• Triprolidine

46. Ans. is b i.e. Salmeterol Ref. KDT 6/e, p 222; Harrison 17/e, p 1602 - 1603

Currently available long acting beta-adrenoceptor agonists are :


• Salmeterol
• Formoterol
• Bambuterol

Drugs used in Acute Asthma :


β2 Agonists short acting Anticholinergics Systemic corticosteroids

– Salbutamol (Albuterol) – Ipratropium bromide – Hydrocortisone


– Terbutaline – Methylprednisolone
– Bambuterol

“Systemic or oral steroids are most beneficial in acute illness, when severe airway obstruction is not
resolving or is worsening despite intense optimal bronchodilator therapy.”

107
2 Self Assessment & Review Phar macology
Pharmacology
Drugs used in Chronic Asthma :
Systemic glucocorticoids Inhaled glucocorticoids Leukotriene antagonist
– Methylprednisolone – Beclomethasone – Flunisolide – Montelukast
– Prednisolone – Fluticasone – Triamcinolone – Zafirlukast
5 LOX Inhibitor
– Prednisone – Budesonide – Zileuton

Mast cell stabiliser Long acting inhaled β2- agonists Methylxanthines Combined medication
– Cromolyn sodium – Salmeterol – Theophylline – Fluticasone/
– Nedocromil sodium – Formoterol Salmeterol
– Ketotifen

47. Ans. is b i.e. 5HT - 1B Ref. KDT 6/e, p 170 - 171

• Triptans are a new class of antimigraine drugs that act by activating 5-HT1B/1D receptors.
• The antimigraine activity is due to 5-HT1B/1D receptor mediated constriction of dilated cranial extra cerebral
blood vessels, especially the arterio-venous shunts in the carotid artery, which express 5-HT1B/ID receptors.
Dilatation of these shunt vessels during migraine attack is believed to divert blood flow away from brain
parenchyma.
• Sumatriptan and allied drugs are the drugs of choice for acute attack of migraine.

5-HT receptor subtypes :


Receptor Location Acting drugs
5-HT1
5HT1A • Raphe nuclei & Hippocampus Buspirone (partial agonist)
5HT1B/1D • Cranial blood vessels Triptans (agonist)
5HT2A • Most widely expressed postjunctional receptor Ketanserin (antagonist)
(D type) • Vascular & visceral smooth muscle Cyproheptadine (antagonist)
• Platelet & cerebral neurons Methysergide (antagonist)
5HT3 • Somatic & autonomic nerve endings Ondansetron (antagonist)
(M type) • Myenteric plexus nerve endings Tropisetron (antagonist)
• Area postrema & nucleus tractus solitarious in brainstem
5HT4 • GIT (mucosa, plexus & smooth muscle) Cisapride (agonist)
• Hippocampus & colliculi Renzapride (agonist)
Metoclopramide (agonist)

Note :
• Clozapin (atypical neuroleptic) has high affinity for 5-HT6 and 5-HT7 receptors in addition to being a 5-HT2A/
2C
antagonist.
• All 5-HT receptors (except 5-HT3) are G protein coupled receptors, the 5-HT3 is a ligand gated cation (Na+,
K+) channel.
• Sumatriptan is the only triptan available for parenteral use.

108
Phar macology of ANS, PNS & A
Pharmacology utacoids
Autacoids 2
48. Ans. is b i.e. Antihistaminics Ref. Katzung 10/e, p 261

A common effect of first generation H1 antagonists is sedation Antihistaminic drugs are used as sedative and
hypnotics in children’s many are available without prescription as cold and “sleep aids” but they are unsuitable
for day time use.

For more details about Classification of Anti-histaminic drugs, refer answer no. 13

49. Ans. is c i.e. β blockers Ref. Harrion 17/e, p 298 - 299

Drugs associated with Erectile Dysfunction.


Therapeutic class Drug class / Drug
Diuretics Thiazides, Spironolactone
Antihypertensive Calcium channel blockers, Methyldopa, Clonidine, Reserpine, β-blockers,
Guanethidine
Cardiac / Digoxin, Gemfibrozil, Clofibrate
anti-hyperlipidemics
Antidepressants Selective serotonin reuptake inhibitors, Tricyclic antidepressants, Lithium,
Monoamine oxidase inhibitors
Tranquilizers Butyrophenones, Phenothiazines
H2 antagonists Ranitidine, Cimetidine
Hormones Progesterone, Estrogens, Corticosteroids, GnRH agonists, 5α-reductase inhibitors,
Cyproterone acetate
Cytotoxic agents Cyclophosphamide, Methotrexate, Roferon-A
Anticholinergics Disopyramide, Anticonvulsannts
Recreational Ethanol, Cocaine, Marijuana

50. Ans. is c i.e. Piroxicam Ref. KDT 6/e, p 194; Goodman & Gilman 11/e, p 701

Piroxicam is rapidly and completely absorbed, metabolized in liver by hydroxylation and glucoronide
conjugation, excreted in urine and bile and undergoes enterohepatic circulation. Hence, it has a long
plasma t1/2 of nearly 2 days.
It is a long acting potent NSAID with anti-inflammatory potency similar to indomethacin and good analgesic-
antipyretic action.

51. Ans. is a i.e. Salmeterol Ref. Harrison 17/e, p 1602

Long acting inhaled β2 agonists : • Salmeterol


• Formoterol
– They provide sustained effects for 9 to 12 hours.
– They are particularly helpful for nocturnal and exercise induced asthma.
– Salmeterol is not indicated for the treatment of acute episodes because of its relatively slow onset of action
(30 min) nor is it recommended as a rescue drug for breakthrough symptoms.

109
2 Self Assessment & Review Phar macology
Pharmacology

Very long acting : – Fenoterol – Albuterol


– Salmeterol – Formoterol

52. Ans. is b i.e. Preventing K+ efflux from the cell


Ref. Katzung 10/e, p 436; KDT 6/e, p 99

Reversal of Nondepolarizing Neuromuscular Blockade


• The cholinesterase inhibitors effectively antagonize the neuromuscular blockade caused by nondepolarizing
drugs.
• Neostigmine and pyridostigmine antagonize nondepolarizing neuromuscular blockade by
increasing the availability of acetylcholine at the motor end plate, mainly by inhibition of
acetylcholinesterase.
• They also cause direct nicotinic effect on motor end plates, thereby causing contraction
• To a lesser extent, these cholinesterase inhibitors also increase release of transmitter from the motor nerve
terminal.

Receptor are present on the neuromuscular junction are ligand gated ion channel, therefore the
entry of Na– through this channel causes action of acetylcholine not the K– efflux from cell.

53. Ans. is b i.e. Glycopyrrolate Ref. KDT 6/e, p 107, 110

Glycopyrolate, a quaternary ammonium anticholinergic is a more potent antisialagogue than atropine and is
less likely to cause arrhythmias. As it does not cross BBB it lacks central activity does not cause confusion
central anticholinergic effects and is now prefer.
Glycopyrrolate
• It is a synthetic anticholinergic drug
• It inhibits the action of acetylcholine at post ganglionic parasympathetic neuroeffector site including secretory
glands.
• It is potent and rapidly acting, action starts within 1 min on I.V. and in 30 min. on I.M. injection.
• It is primarily used in pre-anaesthetic medication to inhibit salivary and tracheo-bronchial secretions.

110
Phar macology of ANS, PNS & A
Pharmacology utacoids
Autacoids 2
54. Ans. is a i.e. Dopamine
Ref. CMDT ‘05, p 1571; Goodman & Gilman 11/e p 288; Katzung 10/e, p 964, 967; www.courses.ahc.umm.edu

Antidote glucagon IV bolus may reverse hypotension and bradycardia that was resistant to β-agonist.
Glucagon is act on cardiac cells to raise intracellular cAMP but does so through stimulation of glucagon
receptors rather than β adrenoceptors.

Propranolol is the most toxic drug in β – adrenergic blockers poisoning.


Clinical features of β-Blocker Toxicity
• Most common – hypotension and bradycardia.
• Seizures and coma may occur because of Na+ channel blocking action and because of hypoglycemia.
• In severe poisoning, the cardiac depression is often unresponsive to conventional therapy with β – adrenergic
stimulants such as Dopamine, Norepinephrine and Isoprenaline
Treatment : ... Harrison 16/e, p 2588
• Glucagon and calcium is used for hypotension and bradycardia.
• Initially treat bradycardia and heart block by atropine and isoproterenol and later on by external transcutaneous
cardiac pacemaker. ... CMDT ‘05

55. Ans. is b i.e. Methyldopa Ref. Goodman & Gilman 11/e, p 280, 281

• Methyldopa is a centrally acting antihypertensive agent. It is a prodrug that exerts its antihypertensive action
via an active metabolite. α-Methyl Dopa is not an alpha agonist itself while its metabolite alpha-methyl NE
behaves as an agonist.
• All the 4 drugs in the options are α-adrenoceptor agonists.
• Clonidine, Guanfacine and Guanabenz have been known to stimulate α2A subtype of α2 adrenergic receptors
in the brainstem, resulting in a reduction in sympathetic outflow from the CNS.
• Methyldopa is assumed to act as an α-adrenergic receptor agonist in the brainstem.
• So, among the options, we will like to choose Methyldopa.
α-Methyl Dopa

α Methyl Dopamine

α-Methyl norepinephrine

Here this moiety is α agonist not the methyl dopa directly and cause α agonist action
2 2

α – Agonist

α 1–Selective α 2–Selective
• Phenylephrine (directly) • Clonidine
• Methoxamine • Apraclonidine
• Mephenteramine • Guanfacine
• Metaraminol • Guanabenz
• Midodrine • Tizanidine
• Brimonidine

111
2 Self Assessment & Review Phar macology
Pharmacology
Mnemonics : Side effects of METHYLDOPA
• Mental retardation
• Electrolyte imbalance
• Tolerance
• Headache / Hepatotoxicity psYchological upset
• Lactation in female
• Dry mouth
• Oedema
• Parkinsonism
• Anaemia (haemolytic)

56. Ans. is d i.e. Phenytoin Ref. Harrison 17/e, p 640; CMDT ‘05, p 1579

Methemoglobin : – Hemoglobin in which iron is in ferric state.


– Methemoglobin cannot carry oxygen.
Drugs / Chemicals causing Methemoglobinemia

• Benzocaine, Prilocaine • Aniline


• Nitrites • Nitrogen oxide gases
• Naphthalene • Nitrobenzene
• Oxides of nitrogen • Dapsone
• Phenazopyridine (pyridium) • Primaquine & related
• Sulfonamides antimalarials

Clinical feature of Methemoglobinemia :


• Methemoglobinemia reduces oxygen carrying capacity and may cause dizziness, nausea, headache,
dyspnoea, confusion, seizures and coma.
• Victims appear cyanotic because of the “Chocolate brown” colour of methemoglobin but they have normal
pO results on arterial blood gas determinations.
2

Treatment :
• Methylene blue enhances the conversion of methemoglobin to hemoglobin by increasing the activity of the
enzyme methemoglobin reductase.
• Note that methemoglobin shifts the oxygen dissociation curve to the left limiting the release of oxygen to
tissues.
• Cyanosis : – Reduced Hb > 4 mg%
– Methemoglobin > 1.5mg%
– Sulfhemoglobin > 0.5 mg%

57. Ans. is c i.e. Duration of action of aspirin is primarily related to the pharmacokinetic clearance of the
drug from the body
Ref. KDT 6/e, p 185; Clinical Pharma. by Benett & Brown 9/e, p 282; Goodman & Gilman 11/e, p 702, 692

The dose dependent elimination of aspirin of a result of the limited capacity of liver to form salicyluric acid and
the phenolic glucoronide and that results into the large proportion of unchanged drug been excreted in the
heigher doses.

112
Phar macology of ANS, PNS & A
Pharmacology utacoids
Autacoids 2
• Aspirin inhibits COX irreversibly by acetylating one of its serine residue.
• Due to inhibition of cyclooxygenase, prostaglandin synthesis is reduced. ... Lippin. 2/e, p 404 - 405

Note : The acetylation of cyclooxygenase is irreversible because platelet lack nuclei, they can not synthesize
new enzyme and lack of thromboxane persist for the life time of platelet (3 to 7 days). This is in
contrast with endothelial cells, which have nuclei and therefore can produce new cyclooxygenase
enzyme.

“The ability of platelets to be inhibited by such low doses of aspirin is related to the presystemic inhibition
of the cyclooxygenase in the portal circulation before the aspirin is deacetylated to salicylate in the
liver.”
“In contrast to aspirin, whose duration of action is determined by the rate of synthesis of new cyclooxygenase
enzyme, the duration of action of all other NSAIDs, which are reversible inhibitors of cyclooxygenase, is
primarily related to the pharmacokinetic clearance of the drugs from the body.” (Option “a” and “b”)
Other NSAIDs are competitive and reversible inhibitor of COX, the return of activity depends on their dissociation
from the enzyme.

58. Ans. is d i.e. Hepatotoxicity is likely to occur


Ref. Harrison 17/e, p 1951; Goodman & Gilman 11/e, p 694; Katzung 10/e, p 965

After 24 - 36 hours of acetaininophen evidence of liver injury appears, with elevated aminotransferase levels and
hypoprothrombinenia.
Severe liver damage occurs in 90% of the patients with plasma concentrations of acetaminophen greater
than 300 microgram/ml at 4 hours or 45 microgram/ml at 15 hours after ingestion of the drug.
... Goodman & Gilman

↓ ↓

Paracetamol (Coal tar Analgesic) : • Toxic dose → 10 - 15 gm


• Fatal dose → 15 - 25 gm

113
2 Self Assessment & Review Phar macology
Pharmacology
Hepatic injury correlation : ... Harrison 17/e
Blood level Time after ingestion
> 300 μg/mL 4 hour
> 200 μg/mL 6 hour
> 100 μg/mL 8 hour

Symptoms of Paracetamol poisoning :


Early manifestations After 12-18hours After 2 days
– Nausea, Vomiting – Centrilobular – Jaundice start
– Abdominal pain – liver necrosis
– Liver tenderness – Renal tubular necrosis
– No impairment of consciousness – Hypoglycemia

Treatment :
1. Gastric Lavage, Supportive measures and oral administration of Activated charcoal and Cholestyramine : to
prevent absorption of residual drug.

NEITHER OF THESE AGENTS APPEAR TO BE EFFECTIVE IF GIVEN MORE THAN 30 MINUTES AFTER
PARACETAMOL INGESTION.

2. Antidote – N. Acetyl cysteine


If the Paracetamol blood level exceeds : > 200 μg/ml – 4 hour ⎫
⎬ After Ingestion
> 100 μg/ml – 8 hour ⎭
Administration of Sulfhydryl Compounds : – N-acetylcysteine (preferred)
– Cysteamine
– Cysteine

N- Acetyl cysteine reduces the severity of hepatic necrosis :


• Therapy should be given within 8 hour of intoxication.
• Administration of N-acetylcysteine →oral/I.V.
5% solution (foul smell and taste) with water, juice, soda.
140mg/kg (Loading dose) →then, 70mg/kg every 4 hours →15 to 20 doses.

114
Phar macology of ANS, PNS & A
Pharmacology utacoids
Autacoids 2
59. Ans. is a and c i.e. Inhibition of H receptor induced wakefulness; and Inhibition of H mediated
1 1

bronchoconstriction
Ref. Goodman & Gilman 11/e, p 657 - 658; KDT 6/e, p 163; katzung 258

H 3
distribution and function :
• Brain-presynaptic, decrease Histamine, Nor-epinephrine, ACh release.
• In Lung, spleen, gastric mucosa decreases Histamine release and Secretion.
• Blood vessels: Vasodilatation

In this question, 3 options are correct as follows :


H Receptor Mediated Actions :
3

• CNS – In CNS, H3 receptor agonists cause sedation by opposing H1 induced wakefulness.


• GIT – H3 receptors antagonise H1 induced ileal contraction as well as downregulate histamine
levels through autoregulatory actions in the gastric mucosa.

Decrease Gastric acid secretion.
• Respiratory – H receptor agonists inhibit H mediated bronchoconstriction.
3 1

• CVS – H1 in human atrial muscle Negative chronotropic effect occour so H3 causes presynaptic
inhibition of H1 & leads to increase heart rate
– Inhibit sympathetic outflow and norepinephrine release on human blood vessels.

Also Note : H3 agonists H3 antagonists


• α -methyl histamine • Thioperamide specific
• Imetit • Impromidine and Clobenpropit

60. Ans. is b i.e. It is most effective in new onset decompensated heart failure
Ref. KDT 6/e, p 505 - 506; Harrison 17/e, p 144 8

• Although β-blockers can worsen the heart failure, nonetheless, it is now established that these may be
beneficial in patients with mild to moderate heart failure.
• The benefit results from antagonism of damaging effects of cardiac β1 receptor over activity which promotes
unfavorable remodeling of the myocardium.
• β - blockers never given in decompensated patients

β - blockers should be started after compensation is achieved with ACE inhibitors, diuretics, digitalis.

Contraindication for β-blocker use in Heart failure (HF) :

• Unstable Heart failure • Hypotensive patients (Systolic BP < 90mm Hg)


• Severe fluid overload • Recent treatment with inotropic drugs
• Sinus bradycardia, AV-block • Bronchospastic disorder.

• Once a maintenance dose is achieved, β -blockers should be given indefinitely.

115
2 Self Assessment & Review Phar macology
Pharmacology
β - blockers used in heart failure :

• Metoprolol • Bisoprolol • Carvedilol – β1 + β2 + α1 blocker

• Greatest utility in Mild to moderate (NYHA Class II and III) cases of dilated cardiomyopathy with systolic
dysfunction.
• Abrupt administration of large doses of β - blockers.
Can worsen heart failure, so starting dose should be very low, then titrated upwards

61. Ans. is c i.e. It is excreted through bile following hepatic metabolism


Ref. KDT 6/e, p 140

Pharmacokinetics of Sotalol
• Low lipid soluble
• Incompletely absorbed orally, therefore does not undergo first pass metabolism.
• Primarily excreted unchanged in urine.
• Longer duration of action [t½ 6 - 20 hours].
• Effective in narrow dose range.
• Low to lipid insolubility, it is less likely to produce central effects.

Properties of Sotalol
• Non selective β blocker
• K+ channel blocking property
• Class III antiarrhythmic property
• It widens action potential and increase ERP
• It also prolongs APD therefore risk of torsades de pointes.

62. Ans. is a i.e. Continued till the day of operation Ref. Various Books

No Modification of Drugs before Operation :

– Antianginal drugs – ATT drugs – Antihypertensive drugs


– Antithyroid drugs – Levodopa – Progesterone only pill

Oral anticoagulants are stopped 1 week before surgery and low molecular weight heparin is started. Omit the
morning dose of heparin on the day of surgery.
Drugs/conditions which are stopped/modified before surgery.

Mnemonics = SEMLA : S : Smoking – 6 week before


E : Estrogen pills – 4 week
M : MAO-A irreversible – 3 week
L : Lithium – 2 week
A : Aspirin – 1 week

116
Phar macology of ANS, PNS & A
Pharmacology utacoids
Autacoids 2
All hypoglycemic drugs

Minor Surgery Major Surgery


< 20 min Shift to insulin 1 week
omit morning dose before (omit morning dose)

• In a patient undergoing surgery, intraoperative steroid replacement is necessary, if he has taken


steroid for more than 1 week in the last 1 year.

63. Ans. is c i.e. PGE2 has marked oxytocic action while PGF Alpha has tocolytic action
2

Ref. KDT 6/e, p 177; Katzung 10/e, p 299

The action of major prostaglandin, prostacycline and thromboxane.


Organ PGE 2
PGF α

PGI2
TXA 2

Blood vessels Vasodilation → B.P.↓


↓ Vasoconstrictor (mostly), Vasodilatation Vasoconstriction
(Smooth muscle) (Keep ductus patent) larger veins constrict, (widespread
little effect on B.P. ↓ BP)
Platelets Variable ––– Most specific Aggregation and
Antiaggregatory release reaction
Uterus – Contracts gravid Contraction of ––– –––
(Oxytocic) uterus uterus (Oxytocic)
– Relaxes non Softening of cervix
gravid uterus
Softening of cervix
Bronchial muscle Dilatation, inhibit Bronchoconstriction Dilatation Constriction
histamine release
Kidney Natriuresis ↓ Cl – ––– Natriuresis, Vasoconstriction
reabsorption. Inhibit vasodilatation,
ADH action, renin release
renin release
GIT Reduce acid Reduced acid
Eye Lower IOP Lower IOP

64. Ans. is b i.e. Epinephrine injection Ref. KDT 6/e, p 697 - 698; Katzung 10/e, p 137

Here, the patient develops hypersensitivity to penicillin, which presents as rashes, itching, urticaria and fever.
Wheezing, angioneurotic edema, serum sickness and exfoliative dermatitis are less common.
• Incidence is highest with procaine penicillin: procaine is itself allergenic. Epinephrine as the agent of choice,
presumably because epinephrine activates alpha beta one, & beta two, all of which may be important in
reversing the pathophysiologic processes underlying anaphylaxis
• Parenteral administration of epinephrine, 1:1000 epinephrine solution. Intramuscular injection may be the
preferred route of administration, since skin blood flow (and hence systemic drug absorption from subcutaneous
injection) is unpredictable in hypotensive patients.

117
2 Self Assessment & Review Phar macology
Pharmacology
65. Ans. is c i.e. Carvedilol has α agonistic and selective β blocking action
1

Ref. KDT 6/e, p 140, 143; Katzung 10/e, p 149

• Atenolol t½ is 6-9 hours and that of metoprolol is 3-4 hours.


• Nadolol has elimination t½ is 14-24 hours and is longest than others.

Properties of Carvedilol
• Carvedilol is β1+ β2+ α1 adrenoceptor blocker.
– Produces vasodilatation due to α1 blockade as well as direct action.
– It also has antioxidant properties.
– Used in Hypertension, CHF and angina pectosis.

Comparative properties of β blockers


Selectivity Partial Local Liqid Elimination Approximate
Agonist Anesthetic Solubility Half-Life Bioavailability
Activity Action
Atenolol β1
No No Low 6 - 9 hours 40
Carvedilol None No No High 7 - 10 hours 25 - 35
Esmolol β1
No No Low 10 minutes 0
Labetalol None Yes Yes Moderate 5 hours 30
Metoprolol β 1
No Yes Moderate 3 - 4 hours 50
Nadolol None No No Low 14 - 24 hours 33

66. Ans. is b i.e. Neostigmine Ref. KDT 6/e, p 101 - 102, 339 - 341

Neostigmine is an anticholinesterase drug which inhibits ChE, protects ACh from hydrolysis.
d - tubocurarine is a nondepolarizing competitive blocker (neuromuscular blocking agent).
The cholinesterase inhibitors effectively antagonize the neuromuscular blockade caused by nondepolarizing
drugs.
• They also cause direct nicotinic effect on motor end plates, thereby causing contraction
• To a lesser extent, these cholinesterase inhibitors also increase release of transmitter from the motor nerve
terminal.

For more details, refer answer no. 52

67. Ans. is b i.e. Increase SA node activity Ref. Goodman & Gilman 11/e, p 245 - 246

Epinephrine (10 ml of 1 in 10,000) is used in cardiac arrest myocardial and cerebral blood flow increase
and prevent arterial collapse and augmentation of coronary and cerebral perfusion pressure.
• Epinephrine is a powerful cardiac stimulant. It acts directly on the predominant β1 receptors of myocardium
and of the cells of the pacemaker and conducting tissues.
• Epinephrine speeds the heart by accelerating the slow depolarisation of sinoatrial (SA) nodal cells that
takes place during diastole i.e. during phase 4 of the action potential

Contraindication of epinephrine : Hyperthyroidism, IHD, Diabetes, Angle closure glaucoma, Hypertension

118
Phar macology of ANS, PNS & A
Pharmacology utacoids
Autacoids 2
68. Ans. is c i.e. Atenolol Ref. Harrison 17/e, p 2271

Best answer is none of the above, but if you have to choose one, then it should be β blocker in my opinion.
Beta antagonists should not be used prior to establishing effective α-receptor blockade, since unopposed
β-receptor blockade could theoretically causes BP elevation from increase vasoconstriction.

For more details, refer answer no. 24

69. Ans. is c i.e. Dazoxiben Ref. Goodman & Gilman 11/e, p 654

Dazoxiben inhibit thromboxane A2 synthase.


Effect of the thromboxane synthetase inhibitor (dazoxiben) upon platelet aggregation, coronary artery thrombosis
and vascular reactivity.
Dazoxiben possesses in vivo antithrombotic activity due to modification of platelet reactivity and that it can
enhance coronary vasodilator responses to exogenously administered arachidonic acid.

70. Ans. is a i.e. Thromboxane A synthesis inhibition


2

Ref. Goodman & Gilman 11/e, p 675

• If low doses of aspirin (80 - 100 mg) are used, it irreversibly inhibits TXA2 production by inhibiting COX-1 of
platelets for their total life span (7 days).
• This is because, the platelets lack nuclei and cannot synthesize COX. Such small doses do not affect PGI2
synthesis in the widely distributed vascular endothelium which can regenerate COX if at all inhibited to some
extent.
• The maximal effectiveness of aspirin therapy appears to depend upon selective blockade of TXA2 synthesis
by platelets without preventing production of PGI2 by endothelial cells. Although the optimal dosage to
prevent thrombotic events has not been firmly established, selective antiplatelet action appear to be best
achieved when the dose of aspirin is between 40 to 80mg per day, while higher doses also inhibit PGI 2

production.

71. Ans. is a i.e. Angiotensin II has direct effect on GFR Ref. KDT 6/e, p 484

ACEIs - There is decrease in Angiotensin II, whose role is to constrict efferent arterioles and maintain the
glomerular filtration pressure, even when renal perfusion is low consequently, inhibition of ACE can induce acute
renal insufficiency in such cases.
Acute renal failure is precipitated by ACE inhibitors in patients with bilateral renal artery stenosis due to
dilatation of efferent arterioles and fall in glomerular filtration pressure; contraindicated in such patients.

72. Ans. is a i.e. Increase uveoscleral flow of aqueous Ref. Khurana 4/e, p 427

Antiglaucoma drugs : Mechanism of lowering IOP at a glance

Drugs which increase trabecular outflow Drugs which increase uveoscleral outflow

• Miotics (e.g., pilocarpine) • Prostaglandins (latanoprost)


• Epinephrine, dipivefrine • Epinephrine, Dipivefrine

119
2 Self Assessment & Review Phar macology
Pharmacology

• Bimatoprost • Brimonidine
• Apraclonidine
Drugs which decrease aqueous production
• Carbonic anhydrase inhibitors (e.g., acetazolamide, dorazolamide)
• Alpha receptor stimulators in ciliary process (e.g., epinephrine, dipivefrine, clonidine, brimonidine,
apraclonidine).
• Beta blockers (e.g., timolol, betaxolol, levobunolol)
• Hyperosmotic agents (e.g., glycerol, mannitol, urea)

73. Ans. is b i.e. Impaired glucose tolerance Ref. KDT 6/e, p 134

Prazosin : • Primarily used as antihypertensive, also used in LVF which is not controlled by diuretics.
• Highly relative α1 blocker, α1 : α2 = 1000 : 1

Advantage of prazosin (selective α1 blocker)


• Improve carbohydrate metabolism • Favorable effect on lipid profile
• Symptomatic improvement in PVD & BPH • No impairment in cardiac contractility

Disadvantage : It dilates arterioles more than veins, so postural hypotension occurs which is know as first
dose effect. This can be minimized by starting with a low dose and taking at bed time.

74. Ans. is c i.e. Nabumetone Ref. KDT 6/e, p 184

Nabumetone is a preferential COX-2 inhibitor.

Preferential COX-2 inhibitors are : • Nimesulide • Meloxicam • Nabumetone

Selective COX-2 inhibitors are : • Celecoxib • Rofecoxib • Valdecoxib

75. Ans. is b i.e. β-blocker Ref. KDT 6/e, p 139 . Harrison 17/e, p 2422

Plasma lipid profile is worsened after long term use of β-blocker.


Cardioselective β blockers and those with intrinsic sympathomimetic activity have little or no deleterious
effect on lipid profile.

Effect of β blocker on lipid profile : • ↑ LDL cholesterol • ↓ HDL cholesterol


• ↑ Total triglycerides
Drugs causing hyperlipidemia :
Hypercholesteremia Lower HDL Hypertriglyceridimia
• Progesterone • β-blocker • Estrogen • β-blocker
• Cyclosporine • Anabolic steroid • Glucocorticoid • Bile acid binding resins
• Thiazide • Probucol • Isotretinoin • Thiazides

120
Phar macology of ANS, PNS & A
Pharmacology utacoids
Autacoids 2
76. Ans. is b i.e. Carbachol Ref. KDT 6/e, p 98

Carbachol has highest nicotinic activity.


Cholinesterase Susceptibility to Muscarinic Nicotinic
cholinesterase action action

ACH ++++ +++ +++


Methacholine + ++++ None
Carbachol Negligible ++ ++++
Bethanechol Negligible ++ None

Mnemonic : Methacholine has predominantly Muscarinic effects Mainly on Myocardium (note the sequence
of four Ms). Nicotinic and muscarinic actions are Common with Carbachol (note the sequence
of two Cs). Bethanechol is Bankrupt in nicotinic actions (note the sequence of two Bs). The
sound of “Ni” in Nicotinic action matches with “ne” in Bethanechol.

77. Ans. is b i.e. Cocaine Ref. KDT 6/e, p 356

All local anesthetics produce vasodilatation except cocaine which produce vasoconstriction because in the
periphery it blocks uptake of NA and adrenaline in adrenergic nerve endings.
The only indication of cocaine is ocular anesthesia.

78. Ans. is b i.e. Hyoscine Ref. KDT 6/e, p 641

Hyoscine has potent antiemetic action. It also produces amnesia which is good for anaesthesia.
• Hyoscine (Scopolamine) is the most effective drug for motion sickness.
• Dicyclomine is used for prophylaxis of motion sickness
• Domeperidone it is a prokinetic antiemetic drug acts via D antagonism.
2

• Glycopyrrolate acts rapidly and is longer acting. It is a potent antisecretory agent and prevents vagal
bradycardia effectively. It is less likely to produce central effects.

79. Ans. is d i.e. Muscle fasciculation Ref. KDT 6/e, p 344; Lee 12/e, p 223

Side effect of Suxamethonium (SCh) are :

Increase in : • Muscle tone • Muscle fasciculation


• Muscle soreness • K Hyperkalemia
+

• Intraocular pressure • Malignant hyperpyrexia


• Intraabdominal pressure • Diastolic cardiac arrest

121
2 Self Assessment & Review Phar macology
Pharmacology
Muscle effects :
Muscle spasm Muscle fasciculation
Muscle rigidity Muscle soreness
Muscle ache
↓ ↓
Fentanyl Suxamethonium

80. Ans. is b i.e. Myoneural junction Ref. KDT 6/e, p 339-340; Morgan’s 3/e, p 181

Vecuronium is a non-depolarising blocker (Competitive Blocker) Steroid derivative. Cardiovascular stability is


better due to lack of histamine release and no ganglion blockade action.

Peripherally acting muscle relaxants:


Non-depolarising Blockers (Competitive Blockers). These drugs prevent the access of ACh to Nm receptor
of motor end plate and prevent its depolarisation.

INTERMEDIATE ACTING
• Isoquinoline Derivatives : • Steroidal Derivatives :
– Atracurium – Vecuronium
– Cisatracurium – Rocuronium
Depolarising Blockers (Persistent Depolarisers).
• Succinylcholine (SCh)

81. Ans. is b i.e. Sulfinpyrazone Ref. Harrison 17/e, p2445

Sulfinpyrazone is a uricosuric drug, so it cannot cause hyperuricemia (GOUT).

Mnemonics : LEAD Poisoning Causes Stones and Nephropathy

Drug causing hyperuricemia : L Levodopa


E Ethacrynic acid
A Alcohol
D Diuretics (Chlorthalidone)
P Pyrazinamide
C Cyclosporine (Cytotoxin)
S Salicylates (low dose Aspirin)
N Nicotinic acid

82. Ans. is a, b, c and d i.e. All are correct options Ref. KDT 6/e, p 504; Guyton 11/e, p 202

NO causes vasodilatation through cyclic GMP mediated smooth muscle relaxation.

122
Phar macology of ANS, PNS & A
Pharmacology utacoids
Autacoids 2
Drugs causing vasodilatation
• Venodilation (↓ pre load) : – Glyceryl trinitrate
– Isosorbide dinitrate
• Arteriolar dilators (↓ after load) : – Hydralazine
– Minoxidil
– Ca channel blockers (nifedipine)
– Potassium channel openers (nicorandil)
• Mixed dilators (↓ pre and after load) : – ACE inhibitors
– ARBS
– α1 blocker (prazosin)
– Amrinone, milrinone
– Nitroprusside

83. Ans. is c and d i.e. Decrease BP; and Synthesis from vascular endothelium
Ref. KDT 6/e, p 177; Katzung 10/e, p 299

Prostacyclin PGI 2 causes widespread decrease in blood pressure.


A summary of the action of major prostaglandins, prostacyclin & thromboxane given in following table.

Organ PGE2 PGF2αα PGI2 TXA2

Blood vessels Vasodilation → B.P.↓


↓ Vasoconstrictor (mostly), Vasodilatation Vasoconstriction
(Smooth muscle) (Keep ductus patent) larger veins constrict, (widespread
little effect on B.P. ↓ BP)
Platelets Variable ––– Most specific Aggregation and
Antiaggregatory release reaction
Uterus – Contracts gravid Contraction of ––– –––
(Oxytocic) uterus uterus (Oxytocic)
– Relaxes non Softening of cervix
gravid uterus
Softening of cervix
Bronchial muscle Dilatation, inhibit Bronchoconstriction Dilatation Constriction
histamine release
Kidney Natriuresis ↓ Cl – ––– Natriuresis, Vasoconstriction
reabsorption. Inhibit vasodilatation,
ADH action, renin release
renin release
GIT Reduce acid Reduced acid
Eye Lower IOP Lower IOP

84. Ans. is a, b and d i.e. Synthesized by platelets; Prothrombotic; and Vasoconstriction


Ref. KDT 6/e, p 177; Katzung 10/e, p 299

Already explained, refer just above.

123
2 Self Assessment & Review Phar macology
Pharmacology
85. Ans. is a, c and d i.e. Portal hypertension; Anxiety; and HOCM Ref. KDT 6/e, p 142

Uses of beta-blocker
Mnemonics : CPMT HAS Anxiety and Mental Tension
• Cardiac arrhythmias
• Pheochromocytoma, Portal hypertension
• Myocardial infarction
• Thyrotoxicosis
• Hypertrophic obstructive cardiomyopathy
• Angina pectoris
• Supraventricular tachycardia
• Anxiety
• Migraine
• Tremor

86. Ans. is a and b i.e. Tinnitus early symptom; and 10-30 gm causes poisoning
Ref. KDT 6/e, p 189; Clinical Pharma. by Benett & Brown 9/e, p 282; Goodman & Gilman 11/e, p 702, 692

Mnemonics : ASPIRIN • Asthma in children • Intestinal blood loss


• Salicyalism • Reye’s syndrome
• Peptic ulcer disease / PPH / • Idiosyncracy
Phosphorylation-oxidation uncoupling / • Noise (tinnitus)
Platelet disaggregation /
Premature closure of PDA

Approximate relationships of plasma salicylate levels to pharmacodynamics and complications.

124
Phar macology of ANS, PNS & A
Pharmacology utacoids
Autacoids 2
Approximate relationships of plasma salicylate levels to pharmacodynamics and complications.
Increase in plasma concentration of free salicylic acid and two features of salicylism - tinnitus and
headache.
• Plasma level above 1 mg/ml of aspirin cause severe metabolic acidosis.
• Symptoms of metabolic acidosis include, loss of vision, hyperpyrexia, vasomotor collapse, renal and respiratory
failure, dehydration convulsions, coma and ultimately death. Hyperpyrexia due to aspirin, which itself is
antipyretic, sounds paradoxical.
• Plasma levels > 400 - 500 μg/ml respiratory alkalosis – Salicylism mild complication characterized by
headache, vertigo, tinnitus; hyperventilation, nausea vomiting.
• At plasma concentration 500 μg/ml medullary depression hence respiratory depression leads to respiratory
acidosis.
• At plasma level > 1mg/ml of aspirin causes lethal toxicity in the form of metabolic acidosis. Symptoms
include – hyperpyrexia, vasomotor collapse, loss of vision renal and respiratory failure, dehydration, convulsion,
coma and ultimately death.
– Aspirin inhibits COX irreversibly by acetylating one of its serine residue.
– Due to inhibition of cyclooxygenase, prostaglandin synthesis is reduced. ...Lippin. 2/e, p 404 - 405
Other NSAIDs are competitive and reversible inhibitor of COX, return of activity depends on their dissociation
from the enzyme.

87. Ans. is e i.e. Cause bradycardia Ref. KDT 6/e, p 141

ESMOLOL is an ultrashort-acting cardioselective β antagonist. It has plasma half life of 8 to 10 min.


1

Important points • An ultrashort acting cardioselective β blocker


• No partial agonistic (intrinsic sympathomimetic) activity
• No membrane stabilising actions
• Rapidly hydrolysed by esterases in RBCs
• Plasma t½ < 10 min. Action lasts 15-20 minutes
• Precipitates heart failure
• No β2 blocking action, airway resistance not increased
Used to terminate • Supraventricular tachycardia
• Episodic AF, AFl
• Arrhythmia during anaesthesia
• Early treatment of MI
• Reduce HR and BP during and after cardiac surgery

88. Ans. is a, b and e i.e. Asthma; Heart block; and Decompensated LVF
Ref. KDT 6/e, p 139; Harrison 17/e, p 1450

Contraindication of β-blockers
• Asthma option “a” (COPD)
• Decompensated heart failure option “e”
• Partial and complete heart block option “b”
• Bradycardia (H/R < 60bpm)
• Hypotensive patient (systolic B.P <100mmHg)

125
2 Self Assessment & Review Phar macology
Pharmacology
• Severe fluid overload
• Hypoglycemia
• In patient who have required recent treatment with an intravenous inotropic agent.

Mnemonic : BBC Loses Viewers Rating In 2008


Beta blockers
Broncoconstriction
Claudication
Lipids
Vivid dreams & nightmares
Inotropic action
Reduced sensitivity to hypoglycemia

89. Ans. is a, b and d i.e. Hemicholinium - prevents release of Ach from storage; Botulinum - Increase
the Ach release; and Vesamicol - Inhibits uptake of choline Ref. Katzung 10/e, p 79

• Rate limiting step of Ach synthesis is Choline uptake inhibited by hemicholinium.


• Uptake of Ach into the vesicle blocked by Vesamicol.
• Release of Ach is blocked by Botulinum toxins
• Rate limiting enzyme of Catecholamines synthesis is Tyrosine hydroxylase.

126
Phar macology of ANS, PNS & A
Pharmacology utacoids
Autacoids 2
Remember : Cholinesterase reactivators Cholinesterase inhibitors

• Pralidoxime • Organophosphorus
• Diacetyl monoxime • Carbamates
• Obidoxime • Tacrine

90. Ans. is d and e i.e. Thyroxine; and Propanolol


Ref. Harrison 17/e, p 2237; KDT 6/e, p 242; Goodman & Gilman 11/e, p 288

The most toxic b blocker is ppnl, in over dose selectivity for beta1 and beta2 is lost
Propranolol is itself a nonselective β blocker then how can it is used in β-blocker over dose.
β blocker are used in thyrotoxicosis (not thyroxine) used in β-blocker toxicity.

Clinical features of β blocker over dose :


• Physiological depression • AV block • Hypooglycemia
• Seizures • Hypotension and bradycardia • ↑ QT interval (sotalol)

Treatment of β blocker overdose :


• Glucagon and calcium for hypotension and symptomatic bradycardia.
• Atropine, isoproterenol, amrinone, dobutamine, epinephrine/ norepinephrine.
• High dose insulin with glucose and potassium.
• Electrical pacing.
• Mechanical cardiovascular support.

91. Ans. is e i.e. Ischaemia Ref. KDT 6/e, p 146, 349, 546 - 547

Uses of α2 Agonist

Brimonidine – 2nd choice drug for open angle glaucoma


Clonidine – Hypertension
– Opioid, Nicotinic, Alcohol withdrawal
– Intrathecal / epidural surgical and post-op analgesia
– To control loose motions due to diabetic neuropathy
– Post menopausal vasomotor symptoms
– Clonidine suppression test for pheochromocytoma
– Prophylaxis in migraine
– Preanaesthetic medication (Sedative, anxiolytic & analgesic)
– Glaucoma
– Anesthesia
– Psychiatric disorders
Methyldopa – Mild to moderate hypertension
– Hypertension in pregnancy
Tizanidine – Spasticity due to neurological disorder
Apraclonidine – Control of spinks of IOT after laser trabeculoplasty

Dexmedetomidine is selective α2A agonist used for preanaesthetic medication preoperative sedation, analgesia,
antianxiety and decrease in bronchial secretions.

127
2 Self Assessment & Review Phar macology
Pharmacology
92. Ans. is b, d and e i.e. Pindolol; Levobunolol; and Timolol Ref. KDT 6/e, p 143

Labetalol and carvedilol are combined α and β blocker :


Labetalol It is β1 + β2 + α1 blockers as well as weak β2 agonist.
The β blocking potency is about 1/3 that of propranolol.
The α blocking potency is about 1/10 of phentolamine.
It is 5 times more potent in blocking β than α receptor.
Carvedilol is β1 + β2 + α1, blocker.
Propranolol is nonselective β blocker.
Pindolol is potent β blocker with intrinsic sympathomimetic activity.
Timolol is non selective β blocker preferred for topical use in eye.

93. Ans. is a, b, d and e, i.e. Dizziness, Delirium, Hallucination; and Headache


Ref. KDT 6/e, p 629 katzung 263/10 edition

• H receptor antagonist are Cimetidine, Ranitidine, Famotidine etc.


2

• The CNS effect of H2 receptor blocker are headache, dizziness, confusional state, restlessness, hallucination,
delirium and coma.
• There occur especially in elderly patients, in these with renal important especially with large doses infused
I.V.

94. Ans. is a and d i.e. Hypoxanthine to xanthine; and Xanthine to uric acid Ref. KDT 6/e, p 208

Allopurinol is a uric acid synthesis inhibitor. It acts by inhibiting xanthine oxidase, the enzyme responsible for
uric acid synthesis, as shown in the figure below :

Allopurinol itself is a short acting (t½ 2 hrs.) competitive inhibitor of xanthine oxidase, but its major
metabolite alloxanthine is a long acting (t½ 24 hrs.) and noncompetitive inhibitor – primarily responsible
for uric acid synthesis inhibition in vivo.

Uricosuric drugs are : a. Probenecid


b. Sulfinpyrazone

128
Phar macology of ANS, PNS & A
Pharmacology utacoids
Autacoids 2
95. Ans. is b, c and e i.e. Asthma; Depression; and Hypotension Ref. KDT 6/e, p 144

During topical application of β blockers, significant systemic absorption can occur leading to systemic
adverse effects such as :
– Bronchospasm (Asthma) – Cold hands and feet
– CHF accentuation in elderly – Carbohydrate intolerance
– Bradycardia – Reduced exercise capacity
– Variant (Prinzmetal’s) angina – Depression, anxiety.
– Hypotension – Nightmares

• Topical β blockers (Timolol) are now the first choice drugs in open angle glaucoma.
• They lower intraocular tension by reducing aqueous formation. This probably results from down regulation of
adenylcyclase due to β2 receptor blockade in the ciliary processes.

96. Ans is a, b, c, d, and e i.e. All are correct options


Ref. KDT 6/e, p 137, 143 - 144

Carvedilol has non-selective antagonism of β-adrenergic receptors but also possesses appreciable α -adrenergic
1

antagonist activity. It also has antiproferative properties and is a scavenger of reactive free oxidant radicals.

Properties of Carvedilol
• Carvedilol is β1+ β2+ α1 adrenoceptor blocker.
– Produces vasodilatation due to α1 blockade as well as direct action.
– It also has antioxidant properties.
– Used in Hypertension, CHF and angina pectosis.

Carvedilol are antiadrenergic drugs which have both α and β blocking actions.
Carvedilol : • Competitive antagonist of α1 + β1 + β2 receptors.
• No β2 agonism.
• Antioxidant activity present.
• Bioavailability 20 - 40% due to extensive first pass metabolism.
• Useful in hypertension and cardioprotective in CHF.
• Inhibits vascular smooth muscle mitogenesis.

97. Ans. is c and d i.e. β blocker; and Sodium valproate


Ref. Harrison 17/e, p 102

Already explained, refer answer no. 16

98. Ans. is c and d i.e. Succinylcholine is metabolized; and Organophosphates inhibit it


Ref. KDT 6/e, p 95, 344; K K Sharma 1/e, p 137

Succinylcholine is rapidly hydrolysed by plasma pseudocholinesterase to succinylmonocholine and


then succinic acid + choline (action lasts 3 - 5 minutes).

129
2 Self Assessment & Review Phar macology
Pharmacology
Almost all the pharmacological effects of anti-AChE drug are due to inhibition of true Acetylcholinesterase
(AChE) rather than of Pseudocholinesterase.

Differences between the two types of Cholinesterases


Acetylcholinesterase (True) Plasmacholine Esterase /
Butyrylcholinesterase/ Pseudo
• Distribution All cholinergic sites, RBC, gray matter Plasma, liver, intestine, white matter
Membrane bound present in cholinergic
synaptic cleft
• Hydrolysis
– ACh Very fast (in μs), Highly specific Slow, Non specific action
– Methacholine Slower than ACh Not hydrolysed
– Benzoylcholine Not hydrolysed Hydrolysed
– Butyrycholine Not hydrolysed Hydrolysed
• Inhibition More sensitive to physostigmine More sensitive to organophosphates
• Function Termination of acetylcholine action Hydrolysis of ingested esters
Genetic variation

99. Ans. is a, b and d i.e. Azelastine; Fexofenadine; and Desloratidine Ref. KDT 6/e, p 156 - 159

The second generation antihistaminics are non sedative and are principally used in allergic rhinitis,
urticaria, dermographism and atopic eczema.

These drugs are : • Terfenadine • Cetrizine


• Fexofenadine • Azelastine
• Astemizole • Mizolastine
• Loratadine • Ebastine
• Desloratadine

Advantage of second generation antihistaminics :


• Higher H1 selectivity – no anticholinergic side effect
• No CNS depressant property – do not impair psychomotor performance
– no sleepiness
– no subjective effects
– do not potentiate alcohol or BZD
• Additional action on Leukotrienes

100. Ans. is b i.e. Oxprenolol Ref. KDT - 136 = 140

• Some β-adrenergic blocking agents possess β-agonist activity also known as “intrinsic sympathomimetic
activity.”
• Agents with partial agonistic activity (pindolol, alprenolol, Acebutalol, carteolol, dilevalol, oxyprenolol) cause
little or no depression of resting heart rate (partial agonistic effect) while blocking the increase in heart rate
that occurs in response to exercise or the administration of a β-agonist such as isoproterenol.

130
Phar macology of ANS, PNS & A
Pharmacology utacoids
Autacoids 2
• The presence of partial agonistic activity may be useful when bradycardia limits treatment in patients with
slow resting heart rates.
• Agents with partial agonistic activity cause less change in blood lipid levels than agents without agonistic
properties.

101. Ans. is a i.e. It is a quartenary ammonium compound Ref. KDT 6/e, p 102; K K Sharma 1/e, p 144

Neostigmine is a reversible anticholinesterase. Being quaternary compounds, these drugs are least
absorbed and do not cross blood-brain barrier.

Features of Neostigmine
Features Neostigmine
• Source Synthetic
• Chemistry Quarternary ammonium compound
• Oral absorption Poor
• CNS action Absent
• Applied to eye Poor penetration
• Direct action Present
on cholinoceptors
• Prominent effect on Skeletal muscles
• Important use Myasthenia gravis
15 - 30 mg orally
• Duration of action 3 - 4 hrs

102. Ans. is a, d and e i.e Less bronchoconstriction; May be used in Raynaud’s disease; and Less liable to
impair exercise capacity Ref. KDT 6/e, p 139 - 140

Features of Cardioselective β - blockers


• Lower propensity to cause bronchoconstriction →even these drugs should be avoided in asthmatics.
• Less interference with carbohydrate metabolism →Safer in diabetics.
• Lower incidence of cold hands and feet →Less chance of precipitating Raynaud’s phenomenon.
• No / less deleterious effect on blood lipid profile.
• Ineffective in suppressing essential tremors.
• Less liable to impair exercise capacity.
• They are more potent in blocking cardiac (β1) than bronchial (β2) receptors.
• However, selectivity is only relative and is lost at high dose.

103. Ans. is b i.e. Inhibits spinal polysynaptic reflexes Ref. KDT 6/e, p 348

• Centrally acting muscle relaxants act by a selective action in the cerebrospinal axis, without altering
consciousness.
• They selectively depress spinal and supraspinal polysynaptic reflexes without significantly affecting
monosynaptically mediated stretch reflex.
• All centrally acting muscle relaxants do have some sedative property.
• They have no effect on neuromuscular transmission and on muscle fibres, but reduce decerebrate rigidity,
upper motor neuron spasticity and hyperreflexia.

131
2 Self Assessment & Review Phar macology
Pharmacology
Centrally acting muscle relaxants :
These drugs reduce skeletal muscle tone by a selective action in the cerebrospinal axis without altering
consciousness. These belong to following cateogories :
Mephensine Group Benzodiazepine Group GABA Derivative Central α -agonist
2

• Carisoprodol • Diazepam • Baclofen • Tizanidine


• Chlorzoxazone • Clonazepam
• Chlormezanone
• Methocarbamol

104. Ans. is a, b and e i.e. Aspirin; Indomethacin; and Diclofenac Ref. KDT 6/e, p 184

Go through the classification of Cyclooxygenase (COX) inhibitors as you cannot afford to forget the names of
preferential and selective COX - 2 inhibitors.

COX Inhibitors
Non-selective Preferential COX-2 Selective COX-2 COX-3
Irreversible Reversible

– Aspirin – Phenylbutazone – Meloxicam – Rofecoxib – Paracetamol


– Sodium salicylate – Oxyphenbutazone – Etodolac – Celecoxib – Metamizol
– Sulfasalazine – Indomethacin – Nabumetone – Valdecoxib (Analgin)
– Olsalazine – Sulindac – Etoricoxib
– Methylsalicylate – Ibuprofen
– Ketoprofen
– Flurbiprofen
– Naproxen
– Mefenamic acid
– Flufenamic acid
– Tenoxicam
– Piroxicam
– Ketorolac
– Diclofenac sodium

Also remember :
• Aspirin irreversibly inhibits COX by acetylating one of its serine residues; return of COX activity depends on
synthesis of fresh enzyme.
• Other NSAIDs listed in above table are competitive and reversible inhibitors of COX; return of activity depends
on their dissociation from the enzyme.
• Corticosteroids like Betamethasone acts by inhibiting phospholipase A2.

105. Ans. is a and b i.e. Acting on CTZ; and 5 HT3 antagonist


Ref. KDT 6/e, p 648 - 649; Rang & Dale 5/e, p 374

• Ondansetron is the prototype of a new class of antiemetic drugs developed to control :


– Cancer chemotherapy induced vomiting – Radiotherapy induced vomiting
– Post operative nausea and vomiting

132
Phar macology of ANS, PNS & A
Pharmacology utacoids
Autacoids 2
• It blocks the depolarising action of 5 HT through :
– 5 HT3 receptors on vagal afferents in GIT – Peripheral function
– 5 HT3 receptors on NTS and CTZ – Central function
• Ondansetron blocks emetogenic impulses both at their peripheral origin and their central relay.
• It does not block dopamine (D1 or D2) receptors and apomorphine or motion sickness induced vomiting.

106. Ans. is a i.e. Decreases cardiac output Ref. KDT 6/e, p 107, 127; Harrison 17/e, p 1603

Option ‘a’ Atropine is a prototype anticholinergic drug. The most prominent effect is to cause tachycardia
due to blockade of M2 receptors on SA node through which vagal tone decreases heart rate.

In tachycardia, period of diastole is decreased,

Decreased ventricular filling

Decreased cardiac output
Option ‘b’ Atropine decreases refractory period of A - V node and facilitates A - V conduction, especially, if
it has been depressed by high vagal tone.
Option ‘c’ Since cholinergic impulses are not involved in maintenance of vascular tone, there is no gross
alteration of BP.
Option ‘d’ Atropine cause mydriasis, cycloplegia and abolition of light reflexes for 7 - 10 days.
Option ‘e’ Atropine markedly decreases secretion of all glands e. g. sweat, salivary, lacrimal, peptic and
tracheobronchial.

107. Ans is a and c i.e. β1 receptors in heart stimulate its contractions; and β receptors are present in
smooth muscles Ref. KDT 6/e, p 119, 121

• All cardiac actions are predominantly β1 receptor mediated. So, option ‘a’ is correct and option ‘b’ is
wrong.
• Beta receptors are present in smooth muscle and mediates various aspects of functions of intestinal, blood
vessels etc. So, option ‘c’ is correct.
All the options are clearly analysed in the following table.
β1 β2 β3

a. Location Heart Bronchi Adipose tissue


JG cells in kidney Blood vessels, uterus, urinary tract, eye
b. Agonist Dobutamine Salbutamol, Terbutaline
c. Antagonist Metoprolol α-methyl propranolol
Atenolol
Acebutalol
Betoxalol
Bisoprolol
Celiprolol
Esmolol

133
2 Self Assessment & Review Phar macology
Pharmacology

α1 α2
a. Location – Post junctional on effector organ – Prejunctional on nerve endings
– Postjunctional on brain, β- cells
of pancreas, platelets
b. Agonist – Phenylephrine, Methoxamine – Clonidine
c. Antagonist – Prazosin – Yohimbine, Rauwolscine

108. Ans. is a i.e. Dopamine Ref. Harrison 15/e, p 1693, 1705

• Dopamine is the drug of choice in cardiogenic shock because it exerts both inotropic and vasoconstrictor
actions that are useful in the presence of persistent hypotension.
• The principle for management is to modify preload, afterload and contractility. The goal is to increase contractility
without significant increases in heart rate. A pulmonary capillary wedge pressure (PCWP) of 15 to 20 mmHg
should be the initial goal

109. Ans. is d i.e. Stimulation of β2 receptors Ref. KDT 6/e, p 132

Vasomotor reversal of Dale


The α blockers if given first, blocks the pressor action of adrenaline. Now, when adrenaline is given, it produces
only fall in B.P. due to β2 - mediated vasodilation instead of the usual vasoconstriction.

110. Ans. is c i.e. Acts on central and peripheral cholinergic receptors Ref. KDT 6/e, p 104 - 105

• Atropine is highly effective in counteracting the muscarinic symptoms, but higher doses are required to
antagonize the central effects.
• It does not interact with nicotinic receptors at the neuromuscular junction, thus having no effect on muscle
weakness.
• Pralidoxime, specific antidote for anticholinesterase poisoning, is a cholinesterase reactivator and is effective
at neuromuscular junction as well as other muscarinic and nicotinic receptors.

111. Ans. is a i.e. Misoprostol Ref. KDT 6/e, p 628

Prostaglandin used in NSAID induced ulcer or in patient who continues to smoke is Misoprostol (PGE1
analogue). But DOC in NSAIDs induced ulcer is Proton pump inhibitors.

For more details, refer answer no. 1

112. Ans. is a i.e. Hypocalcemia Ref. Goodman & Gilman 11/e, p 225

Hypocalcemia causes blockade of vesicular acetylcholine release at neuromuscular junction, so it affects the
↑ ed permeability of Na+ and K+), while noncompetitive blockers act
depolarisation at end plate potential (EPP) (↑
by enhancing depolarisation at EPP, as shown in fig. below:

134
Phar macology of ANS, PNS & A
Pharmacology utacoids
Autacoids 2

113. Ans. is a i.e. U/L renal artery stenosis with single kidney
Ref. KDT 6/e, p 484, 486 - 487; Katzung 10/e, p 177

• ACE inhibitors are contraindicated in bilateral renal artery stenosis, unilateral renal artery stenosis with
single kidney and severe renal dysfunction →because ACE inhibitors cause dilatation of efferent arterioles
and fall in glomerular filtration pressure →acute renal failure is precipitated.
• Regarding other options, ACE inhibitors are used in :
– Hypertension
– Diabetic nephropathy (as it decreases proteinuria and stabilises renal function).

135
2 Self Assessment & Review Phar macology
Pharmacology
– Congestive heart failure (retards progression of left ventricular systolic dysfunction and prolongs survival
of CHF patients of all grades).
– Scleroderma crisis.

114. Ans. is c i.e. Influx of Na+ Ref. KDT 6/e, p 352

• The local anaesthetics (LAs) block nerve conduction by decreasing the influx of Na+ ions during upstroke of
action potential (AP).
• The LAs interact with a receptor situated with in the voltage sensitive Na+ channel and raise threshold of
channel opening : Na+ permeability fails to increase in response to an impulse or stimulus.
• The predominant active species is able to approach its receptor only when the channel is open at the inner
face. Thus, a resting nerve is rather resistant to blockade and blockade develops rapidly when the nerve is
stimulated repeatedly.

115. Ans. is c i.e. Preventing conduction from pre post ganglionic Ref. KDT 6/e, p 114

Ganglion blockers antagonize ganglionic transmission. Drugs that inhibit synthesis or release of
acetylcholine can interfere with ganglionic transmission, but drugs which act on cholinergic receptors
in the ganglia are more selective.

116. Ans. is c i.e. ↓ HR Ref. KDT 6/e, p 119

α1 stimulation causes : Vasoconstriction of arterioles and veins



Increase total peripheral resistance

Increase in Blood pressure

Reflex depression of SA node

Decreased heart rate

117. Ans. is b i.e. mRNA Ref. KDT 6/e, p 98

There are 3 types of mushroom poisoning depending upon the toxin present in particular species :
Poisoning Toxins
• Muscarine type (Early mushroom poisoning) • Inocybe and related species
• Anticholinergic / hallucinogenic type • Isoxazole compound in A. muscaria
• Phalloidin type (Late mushroom poisoning) • Peptide toxins in A. phalloides

The phalloidin is due to peptide toxins found in A. phalloides, galerina and related species. These inhibit RNA
and protein synthesis. The symptoms start after many hours and are due to damage to gastrointestinal mucosa,
liver and kidney.

136
Phar macology of ANS, PNS & A
Pharmacology utacoids
Autacoids 2
118. Ans. is d i.e. All Ref. KDT 6/e, p 137

Effect of β-blockers on heart and BV


– Decrease heart rate – Decrease force of contraction
– Decrease cardiac output – Prolongs systole
– Decrease cardiac work – Decreases O2 consumption
– Decreases total coronary blood flow – Decreases refractory period of myocytes
– Decreases automaticity
– Decrease both systolic and diastolic BP (only on chronic administration)

119. Ans. is c i.e. Butoxamine Ref. KDT 6/e, p 132

Classification of α-adrenergic blockers :


Nonequilibrium type
– Phenoxybenzamine
Equilibrium type (competitive)
• Nonselective – Ergotamine – Ergotoxine
– Dihydroergotamine – Dihydroergotoxine
– Tolazoline – Phentolamine
– Chlorpromazine – Ketanserin
• α1 selective – Prazosin – Terazosin
– Doxazosin – Tamsulosin
• α2 selective – Yohimbine

120. Ans. is a, b and e i.e. Allopurinol; Methyldopa; and Amiodarone Ref. Harrison 17/e, p 1949, 1950

Drugs causing Granulomatous hepatitis


Antiinflammatory Antibiotics Xanthine oxidase Anticonvulsant Antiarrythimic/
inhibitor antihypertensive
• Phenylbutazone • Sulfonamide • Allopurinol • Carbamazepine • Qunidine
• Isoniazid • Diltiazem
• Hydralazine
• Amiodarone

121. Ans. is b i.e. Carbachol Ref. KDT 6/e, p 97 - 98

Carbachol has the maximum nicotinic effect among the given choline esters.
Nicotinic effect : Acetylcholine, Carbachol > Methacholine > Bethanechol
Mnemonic : Methacholine has predominantly Muscarinic effects Mainly on Myocardium (note the sequence
of four Ms). Nicotinic and muscarinic actions are Common with Carbachol (note the sequence
of two Cs). Bethanechol is Bankrupt in nicotinic actions (note the sequence of two Bs). The
sound of “Ni” in Nicotinic action matches with “ne” in Bethanechol.

137
2 Self Assessment & Review Phar macology
Pharmacology
121. Ans. is a i.e. Gut Ref. Goodman & Gilman 11/e, p 109; KDT 6/e, p 123

Action of adrenaline is most marked on smaller arterioles and precapillary sphincters : larger arteries and veins
are affected only at higher doses.

Action of adrenaline
Vasoconstriction Vasodilatation
– Cutaneous – Skeletal muscles
– Renal – Liver arteries
– Mucous membrane – Coronary arteries and Splanchnic vessels (gut)

138
Phar macology of ANS, PNS & A
Pharmacology utacoids
Autacoids 2
CHAPTER REVIEW
• This section includes questions of V arious Other PGMEES from 1990 – 2008.
Various
• Questions are ar ranged in increasing order of page sequence of KDT 6 Edition. This is
arranged
done to mak e refer
make ral system more easy and uncomplicated to save the precious time
referral
of PGMEE Aspirant.

1. This short acting selective b-blocker used in the c. Cyclimine


treatment of arrhythmias : (Manipal 07) d. Carbachol
a. Esmolol [Ref. KDT 6/e, p 106]
b. Carvedilol 6. Which of the drug which causes post-anesthetic
c. Celiprolol muscle stiffness ? (J & K 05)
d. Bisoprolol a. Fentanyl
[Ref. KDT 6/e, p 141] b. Pyridostigmine
2. Finasteride is : (Manipal 07) c. Succinylcholine
a. 5a reductase inhibitor d. Gallamine
b. Androgen receptor inhibitor [Ref. KDT 5/e, p 84]
c. Estrogen agonist 7. Beta II receptors are found in : (SGPGI 05)
d. Steroids a. Arterioles
[Ref. KDT 6/e, p 294] b. Veins
3. Maximum anti-cholinergic action is seen with c. SA node
which H1 blocker ? (Manipal 07) d. Myocardium
a. Diphenhydramine [Ref. KDT 6/e, p 119, table (9.2)]
b. Cinnarizine 8. Timolol is preferred in glaucoma over pilocarpine,
c. Loratadine because : (J & K 05)

d. Chlorpheniramine a. It is most effective than Pilocarpine

[Ref. KDT 6/e, p 157] b. Enhances Uveo-scleral outflow

4. Sympathomimetic drugs are useful in the therapy c. Produces less ocular side effects
of all the following conditions except :(Manipal 06) d. It has no contraindications
a. Acute decompensated heart failure [Ref. KDT 5/e, p 144]

b. Hypotension 9. Shortest acting anticholinesterase is :

c. Hypertension a. Edrophonium (TN 04)

d. Erectile dysfunction b. Pyridostigmine

[Ref. KDT 6/e, p 129-131] c. Glycopyrrolate

5. All are anticholinergic drugs except :(Manipal 05) d. Neostigmine


[Ref. KDT 6/e, p 99, 101]
a. Scopolamine
b. Dicyclomine

Answer 1. a. Esmolol 2. a. 5a reductase ... 3. a. diphenydramine


4. c. Hypertension 5. d. Carbachol 6. a. Fentanyl
7. a and b 8. b. Enhances ... 9. a. Edrophonium

139
2 Self Assessment & Review Phar macology
Pharmacology
10. MOA of prototype drug propranolol : (Manipal 04) b. Atropine
a. Selective beta-1 blocker c. Sleep
b. Nonselective beta blocker d. Timolol
c. Selective beta-2 blocker [Ref. KDT 6/e, p 114]
d. Alpha+Beta blocker 17. The reason for not using prostaglandins in
[Ref. KDT 6/e, p 137] tocolysis is : (Manipal 03)
11. Salbutamol is : (Manipal 04) a. They are teratogenic and causes fetal structural
a. β2 agonsit abnormalities
b. β2 antagonist b. They may close the patent ductus arteriosis pre-
c. β1 antagonist maturely

d. β1 agonsit c. Cause placental hypoxia


[Ref. KDT 6/e, p 217] d. None of the above
12. True regarding phenylephrine are all except : [Ref. KDT 6/e, p 186]

a. Used as a nasal decongestant (Manipal 04) 18. Mechanism of action of sodium chromoglycate :

b. Causes reflex bradycardia a. Direct bronchodilator effect (Manipal 03)

c. Causes hypotension b. Inhibition of release of histamine and other in-


flammatory mediators
d. Causes mydriasis
[Ref. KDT 6/e, p 127] c. Antimuscarinic effects

13. Sumatriptan is : (Manipal 04) d. Both A and C are correct


[Ref. KDT 6/e, p 223]
a. Selective 5-HT ID/IB receptor
19. Drug useful in the medical management of BPH :
b. Useful in migraine
a. Nifedipine (Manipal 03)
c. Suppresses vomiting
b. Clonidine
d. All of he above
[Ref. KDT 6/e, p 171] c. Glycopyrrolate

14. Not an endogenous catecholamine : (APPG 03) d. Finasteride


[Ref. KDT 6/e, p 294]
a. Isoprenaline
20. Physostigmine is used in the following drug
b. Dopamine
overdosages except : (Manipal 03)
c. Noradrenaline
a. Edrophonium hydrochloride
d. Adrenaline
b. Imipramine
[Ref. KDT 6/e, p 116]
c. Thiordiazine
15. The mechanism of action of Cromolyn sodium is
by : d. Chlorpromazine

a. Leukotreine antagonist (Manipal 03) [Ref. KDT 6/e, p 104]

b. Beta 2 agonist 21. Shortest acting mydriatic is : (UPPGMEE 04)

c. Mast cell stabilizer a. Atropine

d. Inhibits neutrophils b. Homatropine


[Ref. KDT 6/e, p 223] c. Tropicamide
16. Which of the following drug precipitates an attack d. Cyclopentolate
of glaucoma in subject with narrow angle ? [Ref. KDT 6/e, p 111]
a. Dark environment (Manipal 03)

Answer 10. b. Nonselective ... 11. a. β2 agonsit 12. c. Causes ... 13. d. All of he ... 14. a. Isoprenaline
15. c. Mast cell ... 16. b. Atropine 17. d. None of the ... 18. None 19. d. Finasteride
20. a. Edrophonium ... 21. c. Tropicamide

140
Phar macology of ANS, PNS & A
Pharmacology utacoids
Autacoids 2
22. Not an endogenous catecholamine : (APPG 03) b. Migraine
a. Isoprenaline c. Hypertension
b. Dopamine d. Classical angina
c. Noradrenaline [Ref. KDT 6/e, p 142-143]
d. Adrenaline 29. About succinylcholine, false statement is :
[Ref. KDT 6/e, p 116] a. Neostigmine reversal (Manipal 02)
23. Most common receptor in heart are: b. Persistent depolarization
a. Beta 1 (Jharkhand 03) c. Short acting
b. Beta 2 d. Metabolised by pseudocholinesterase
c. Alpha 1 [Ref. KDT 6/e, p 344]
d. Alpha 2 30. Mechanism of action of sodium chromoglycate :
[Ref. KDT 6/e, p 119] a. Direct bronchodilator effect (Manipal 03)
24. Most common receptor in heart are ? b. Inhibition of release of histamine and other in-
a. beta 1 (Jharkhand 03) flammatory mediators
b. beta 2 c. Antimuscarinic effects
c. alpha 1 d. Both A and C are correct
d. alpha 2 [Ref. KDT 6/e, p 223]
[Ref. KDT 6/e, p 119] 31. Which of the following is a “5HT3 blocker” ?
25. B1 action of epinephrine is not seen in: a. Ondansetron (Manipal 01)
a. Angina pectoris (Manipal 02) b. Tamsulosin
b. Ventricular arrhythmias c. Dazoxiben
c. Slow arrhythmias d. Primagrel
d. Intestinal relaxation 32. Acid production is decreased by which class of
[Ref. KDT 6/e, p 122] drug : (Manipal 01)
26. Famotidine and cimetidine are different in their a. H1 antagonists
action of : (Manipal 02) b. H+ K+ pump blocker
a. Oral absorption c. Nicotinic receptor blockers
b. Duration of action d. All of the above
c. Mechanism of action 33. Both a mydriatic and cycloplegic : (Manipal 01)
d. Antiandrogenic action a. Tropicamide
[Ref. KDT 6/e, p 629-630] b. Homatropine
27. Epinephrine is not useful in : (Manipal 02) c. Atropine
a. Local hemostasis d. Ipratropium bromide
b. Anaphylaxis 34. Which of the following drug decreasing cholinest-
c. Local hemostasis erase activity : (UP 01)
d. All of the above a. Oxotremorine
[Ref. KDT 6/e, p 130] b. Carbachol
28. Not an indication for the use of Beta blockers : c. Bethenechol
a. Variant angina (Manipal 02) d. Edrophonium
[Ref. KDT 6/e, p 101, 104]

Answer 22. a. Isoprenaline 23. a. Beta 1 24. a. beta 1 25. d. Intestinal ... 26. None
27. a. Local ... 28. a. Variant ... 29. a. Neostigmine ... 30. None 31. a. Ondansetron
32. b. H+ K+ ... 33. c. Atropine 34. d. Edrophonium

141
2 Self Assessment & Review Phar macology
Pharmacology
35. H1 receptor blocker is : (UP 01) 41. Short acting nondepolarizing agent is : (UP 00)
a. Ranitidine a. Succinylcholine
b. Thioperamide b. Mivacurium
c. Mepyramine c. Vecuronium
d. Burimamide d. Atracurium
[Ref. KDT 6/e, p 153] [Ref. KDT 6/e, p 343]
36. β-blocker drugs are contraindicated in all except: 42. Suxamethonium is contraindicated in all except :
a. Peripheral vascular disease (UP 01) a. Pre-operative hypokalemia (UP 00)
b. Bronchial asthma b. Preoperative hyperkalemia
c. Congestive heart failure c. Open eye surgery
d. Subvalvular aortic stenosis d. Paraplegia
[Ref. KDT 6/e, p 139] [Ref. KDT 6/e, p 344]
37. β2 selective antagonist is : (UP 01) 43. Antihistaminic with least sedative action is :
a. Metoprolol a. Promethazine (UP 00)
b. Acebutolol b. Chlorpheniramine
c. Oxeprenolol c. Terfenadine
d. Butoxamine d. Dicyclomine
[Ref. KDT 6/e, p 119, 141] [Ref. KDT 6/e, p 156]
38. Sympathetic stimulation produces the following 44. Acetazolamide is : (Manipal 00)
effects except : (UP 01) a. Carbonic anhydrase inhibitor
a. Erection b. Xanthine oxidase inhibitor
b. Ejaculation c. A highly potent diuretic
c. Mydriasis d. None of the above
d. Vasoconstriction [Ref. KDT 6/e, p 568]
[Ref. KDT 6/e, p 122] 45. Which of the following drug does not cause hy-
39. Which of the following drug is not used in raised pokalemia : (AIIMS Dec. 94)
intraocular pressure : (UP 01) a. Gentamicin
a. Timolol b. Propranolol
b. Betaxolol c. Amphotericin B
c. Metipranolol d. Amiodarone
d. Atenolol [Ref. KDT 6/e, p 425]
[Ref. KDT 6/e, p 144-147] 46. H1 blocker with least sedative effect is :
40. Side effects of acetazolamide are all except : a. Chlorpheniramine (AIIMS Dec. 94)
a. Acidosis (UP 00) b. Promethazine
b. Hypokalemia c. Terfenadine
c. Depression d. Diphenhydramine
d. Dermatitis [Ref. KDT 6/e, p 156]
[Ref. KDT 6/e, p 569]

Answer 35. c. Mepyramine 36. c. congestive ... 37. d. Butoxamine 38. a. Erection 39. d. Atenlol
40. c. Depression 41. b. Mivacurium 42. a. Pre-operative ... 43. c. Tefrenadine 44. a. Carbonic ...
45. b. Propanolol 46. c. Terfenadine

142
Phar macology of ANS, PNS & A
Pharmacology utacoids
Autacoids 2
47. All are true about lidocaine, except : c. Tropicamide
a. Prolongs refractory period (AIIMS May 94) d. Cyclopentolate
b. Dose need to be altered in renal failure metabo- 54. Mediator of asthma is : (AIIMS Dec. 92)
lism by hepatic flow a. Prostacyclin (PGI )
2
c. It is very little affected by pH and heat b. PGA
2
d. Loading dose before continuous infusion c. Thromboxane A
2
[Ref. KDT 6/e, p 351-356] d. Leukotriene
48. Timolol can be given in all except selective β [Ref. KDT 6/e, p 226]
blocker : (AIIMS May 94) 55. Edrophonium test is used in : (AIIMS Dec. 92)
a. Myocardial infarction a. Cushing syndrome
b. Bronchial asthma b. Conn’s syndrome
c. Peptic ulcer c. Addison’s disease
d. Congestive heart failure (CHF) d. Myasthenia gravis
[Ref. KDT 6/e, p 95] [Ref. KDT 6/e, p 100]
49. Fasciculation are caused by : (AIIMS May 94) 56. Which causes competitive block : (AIIMS Dec. 92)
a. Vecuronium a. Pancuronium
b. Suxamethonium b. D-Tubocurare
c. Atracuronjum c. Suxamethonium
d. Pancuronium d. Gallamine
50. Dose of ketotifen is : (AIIMS May 93) [Ref. KDT 6/e, p 148]
a. 1-2 mg/d OD (1-2mg BD) 57. All are β agonist except : (AIIMS Dec. 92)
2
b. 10 mg I/V children 0.5 mg BD a. Methoxamine α agonist
1
c. 5-10 mg BD oral b. Salbutamol
d. None of the above c. Metoproterenol
[Ref. KDT 6/e, p 235] d. Terbutaline
51. d-TC (d-Tubocurare. is a : (AIIMS May 93) [Ref. KDT 6/e, p 125]
a. Ganglion blocker 58. Treatment for headache in patient of peptic ulcer
b. Depolarizing blocker is : (AIIMS June 91)
c. Competitive neuromuscular block a. Propoxyphene
d. a+c both b. Paracetamol
52. β-blockers are used in all, except : (AIIMS May 93) c. Ultrafine aspirin
a. Hypertension d. Oxyphenbutazone
b. Thyrotoxicosis [Ref. KDT 6/e, p 440, 452]
c. Variant angina 59. All are centrally acting muscle relaxant, except :
d. All of the above a. Dantrolene (AIIMS June 91)
53. Which is the shortest acting mydratic : b. Diazepam
a. Atropine (AIIMS Dec. 92) c. Mephenesin
b. Homatropine d. Methocarbamol
[Ref. KDT 6/e, p 157]

Answer 47. a. Prolongs ... 48. b. Bronchial ... 49. b. Suxamethonium 50. a. 1-2 mg/d OD ... 51. d. a+c both
52. b. Thyrotoxicosis 53. c. Tropicamide 54. d. Leukotriene 55. d. Myasthenia ... 56. b. D-Tubocurare
57. d. Terbutaline 58. a. Propoxyphene 59. a. Dantrolene

143
2 Self Assessment & Review Phar macology
Pharmacology
60. Epinephrine is drug of choice in : (UP 99) 66. Which drug has both partial agonistic action and
a. Shock membrane stabilizing action : (UP 97)
b. Bronchial asthma a. Oxprenolol
c. Hypertension b. Atenolol
d. Hyperthyroidism c. Metoprolol
[Ref. KDT 6/e, p 129] d. Esmolol
61. Action of H2 blocker is delayed by : (UP 99) [Ref. KDT 6/e, p 140]
a. Food 67. Following are competitive neuromuscular
b. Antacids blockers except : (UP 97)
c. Alcohol a. d-tubocurarine
d. NSAID b. Atracurium
[Ref. KDT 6/e, p 630] c. Gallamine
62. Latanoprost is : (UP 99) d. Suxamethonium
a. Antidiabetic [Ref. KDT 6/e, p 339]
b. Antileprotic 68. Propranolol is used in A/E : (UP 97)
c. Antihypertensive a. Hypertension
d. Antiglaucoma b. Thyrotoxicosis
[Ref. KDT 6/e, p 146-147] c. Migraine
63. Selective β1 antagonist is : (UP 99) d. Variant angina
a. Timolol [Ref. KDT 6/e, p 142-143]
b. Labetolol 69. Following drugs cross BBB except : (UP 97)
c. Propranolol a. Diphenyldramine
d. Atenolol b. Meclizine
[Ref. KDT 6/e, p 137] c. Promethazine
64. Which of the following cardioselective β blocker d. Terfenadine
has membrane stabilizing action : (UP 98) [Ref. KDT 6/e, p 156-157]
a. Atenolol 70. Dopamine used in shock causes : (UP 97)
b. Acetutolol a. Decreases resistance in renal and mesenteric
c. Esmolol vasculature
d. Sotalol b. Improves blood supply to heart
[Ref. KDT 6/e, p 140] c. Improves blood supply to brain
65. Actions of atropine are A/E : (UP 97) d. Improves blood supply to muscle and brain
a. CNS stimulant [Ref. KDT 6/e, p 126-129]
b. Mydriasis 71. Selective M1 receptor blocker is : (UP 97)
c. Bronchoconstriction a. Pirenzepine
d. Decreasing salivary secretion b. Ipratropium bromide
[Ref. KDT 6/e, p 107-108] c. Dicyclomine
d. Propantheline
[Ref. KDT 6/e, p 95]

Answer 60. a. Shock 61. b. Antacids4 62. d. Antiglaucoma 63. d. Atenolol 64. b. Acetutolol
65. c. Bronchoco ... 66. a. Oxprenolol 67. d. Suxamethonium 68. d. Varient angina 69. d. Terfenadine
70. a. Decreases ... 71. a. Pirenzepine

144
Phar macology of ANS, PNS & A
Pharmacology utacoids
Autacoids 2
72. True in cyproheptadine are A/E : (UP 97) 78. Post operative muscular pain are seen after use
a. Antihistaminic of : (UP 96)
b. Increase appetite a. Suxamethonium
c. Antipruritic b. d-tubocurarine
d. Antiadrenergic c. Gallamine
[Ref. KDT 6/e, p 156-157, 166] d. Vecuronium
73. Palidoxime acts by : (UP 97) 79. Which of the following drug have favorable effect
a. Reactivating cholinesterase on lipid profile : (UP 95)
b. Promoting synthesis of cholinesterase a. Propranolol
c. Promoting synthesis of acetylcholine b. Prazosin
d. Direct action on cholinergic receptors c. Chlorthiazide
[Ref. KDT 6/e, p 105] d. Furesemide
74. Most potent preoperative antiementic is :(UP 97) [Ref. KDT 6/e, p 545]

a. Hyoscine 80. Which drug is used to increase flow of urine in a


patient with BHP is : (UP 95)
b. Atropine
a. Propranolol
c. Glycopyrrolate
b. Imipramine
d. Chlorpromazine
c. Disopyramide
[Ref. KDT 6/e, p 112-113]
d. Prazosin
75. β1 adrenergic blocker is : (UP 96)
[Ref. KDT 6/e, p 134]
a. Atenolol
81. The drug which causes mydriasis but not cyclople-
b. Butoxamine
gia : (UP 95)
c. Propranolal
a. Phenylephrine
d. Nadolol
b. Atropine
[Ref. KDT 6/e, p 144]
c. Homatropine
76. The regional arterial resistance of mesentry and
d. Tropicamide
kidney vessels is reduced by : (UP 96)
[Ref. KDT 6/e, p 127]
a. Dopamine
82. Dopamine agonist cause : (UP 94)
b. Dobutamine
a. Inhibition of prolactin
c. Norepinephrine
b. Stimulation of prolactin
d. Isoprenaline
c. Increased production in blood
[Ref. KDT 6/e, p 126]
d. I production sensitivity
77. True about nicotinic acid are A/E : (UP 96)
[Ref. KDT 6/e, p 168]
a. Activates fibrinolysis
83. Which drug is given to a patient presenting with
b. Ganglion receptor blocker
glaucoma oedema and epilepsy is (UP 94)
c. Peripheral vasodilation
a. Carbamazepine
d. S/E is hyperpigmentation
b. Acetazolamide
[Ref. KDT 6/e, p 622]
c. Thiazide
d. Phenobarbitone
[Ref. KDT 6/e, p 568]

Answer 72. d. Antiadrenergic 73. a. Reactivating ... 74. a. Hyosine 75. a. Atenolol 76. a. Dopamine
77. b. Ganglion ... 78. a. Suxamethonium 79. b. Prazocine 80. d. Prazocine 81. a. Phenylephrine
82. a. Inhibition ... 83. b. Acetazolamide

145
2 Self Assessment & Review Phar macology
Pharmacology
84. Mechanism of action of clonidine as : (UP 94) 90. Botulinum acts by : (AIIMS 91, 80)
a. α 1-agonist a. Secretion of Ach
b. α2-agonist b. Synthesis of Ach
c. β1-agonist c. Inhibits Ach release
d. β2-agonist d. Muscle nerve block
[Ref. KDT 6/e, p 119] [Ref. KDT 6/e, p 93]
85. Drug used in hypertension with intermittent clau- 91. Sweat gland function is mediated by : (PGI 96)
dication : (UP 94) a. Sympathetic cholinergic
a. Propranolol b. Sympathetic adrenergic
b. Nadolol c. Parasympathetic
c. Atenolol d. Dopaminergic receptors
d. Practolol [Ref. KDT 6/e, p 94]
[Ref. KDT 6/e, p 140-41] 92. The sub-type of cardiac muscarinic receptor is
86. Ipatropium bromide is : (UP 94) predominantly : (Karn 95)
a. Sympatholytic a. M1
b. Anticholinergic b. M3
c. Antiemetic c. M2
d. Antipsychotic d. M1 and M3
[Ref. KDT 6/e, p 106] [Ref. KDT 6/e, p 94-95, table (7.3)]
87. Following is example of ultra short acting barbitu- 93. Nicotinic receptors are seen in : (NIMS 96)
rates are A/H (UP 93) a. Skeletal muscle
a. Thiopentone b. Visceral smooth muscle
b. Hexobarbitone c. Cardiac muscle
c. Methohexitone d. Salivary glands
d. Butobarbitone short acting + secobarbitone [Ref. KDT 6/e, p 97]
[Ref. KDT 6/e, p 374] 94. Hexamethonium is a : (Jipmer 91)
88. Intravenous Amphotericin-B- is Treatment of a. Parasympathetic blocker
choice in : (UP 93) b. Sympathetic blocker
a. Onychomycosis c. Nicotinic blocker
b. Candidiasis d. All of the above
c. Cutaneous larva migrans [Ref. KDT 6/e, p 96]
d. Cryptococcal meningitis 95. Therapeutic use of acetylcholine is not possible
[Ref. KDT 6/e, p 757] because it is : (Jipmer 92)
89. Treatment of Herpes Simplex Keratitis used A/E : a. Highly protein bound
a. Idoxuridine (UP 93) b. Rapidly degraded
b. 5-FU c. Rapidly excreted
c. Adenosine arabinoside d. Orally ineffective
d. Cytosine arabinoside [Ref. KDT 6/e, p 98, table (7.5)]
[Ref. KDT 6/e, p 125]

Answer 84. b. α2-agonist 85. c. Atenolol 86. b. Anticholinergic 87. d. Butobarbitone ... 88. d. Cryptococcal ...
89. b. 5-FU 90. c. Inhibits Ach ... 91. a. Sympathetic ... 92. c. M2 93. a. Skeletal ...
94. c. Nicotinic ... 95. b. Rapidly ...

146
Phar macology of ANS, PNS & A
Pharmacology utacoids
Autacoids 2
96. Neostigmine does not cross blood brain barrier 102. Which of the following anticholinergic is claimed
due to its : (M P 98) to act selectively on bronchial muscle :
a. Secondary structure a. Isopropamide (Karn 95)
b. Tertiary structure b. Ipratropium
c. Quartenary structure c. Benztropine
d. Primary structure d. Pirenzepine
[Ref. KDT 6/e, p 101] [Ref. KDT 6/e, p 109 - 110]
97. Timolol is : (ROHTAK 97) 103. Ganglion blockers acts by : (PGI 96)
a. Antihypertensive drug a. Preganglionic release of acetylcholine
b. Antiglaucoma drug b. Preventing conduction across synapse present
c. Cholinergic drug c. Blocks nicotinic receptors
d. None d. Preventing competition
[Ref. K.D.T. 5th/e, p 140 - 144] [Ref. KDT 6/e, p 115]
98. Antidote for organophosphorus poisoning is : 104. The classification of alpha & beta adrenergic re-
a. Atropine (Kerala 95) ceptors was given by : (Delhi PGI 96)
b. Neostigmine a. Dale
c. Succinylcholine b. Ahlquist
d. d-tubocurarine c. Sutton
[Ref. KDT 6/e, p 105] d. Lewis
99. Atropine does not inhibit which secretion : [Ref. KDT 6/e, p 118]
a. Tear (C.U.P.G.E.E. 96) 105. Most potent cardiac stimulant is :
b. Salvia a. Adrenaline (DNB 98; AIIMS 81)
c. Gl Secretion b. Propranolol
d. Bile c. Dobutamine
[Ref. KDT 6/e, p 108] d. Salbutamol
100. Hyoscine differs from atropine in that hyoscine [Ref. KDT 6/e, p 119]
does not cause : (AI 90) 106. Pure alpha 2 blocker is : (TN 04)
a. Mydriasis a. Timolol
b. Drowsiness b. Sotalol
c. Dryness of mouth c. Yohimbine
d. Tachycardia d. Carvedilol
[Ref. KDT 6/e, p 109, table (8.1)] [Ref. KDT 6/e, p 122]
101. What is the advantage of glycopyrrolate over at- 107. Dale’s vasomotor reversal phenomenon occurs
ropine : (Jipmer 91) with: (AI 91)
a. Less CNS action a. Dopamine
b. Less mydriasis b. Adrenaline
c. Less tachycardia c. Noradrenaline
d. Less neuromuscular blockade d. All of the above
[Ref. KDT 6/e, p 110] [Ref. KDT 6/e, p 123]

Answer 96. c. Quartenary ... 97. b. Antiglaucoma ... 98. a. Atropine 99. d. Bile 100. a. Mydriasis
101. a. Less CNS ... 102. b. Ipratropium 103. a and c 104. b. Ahlquist 105. a. Adrenaline
106. c. Yohimbine 107. b. Adrenaline

147
2 Self Assessment & Review Phar macology
Pharmacology
108. Dopamine is preferred in treatment of shock be 114. Which is not a selective Beta 2 agonist :
cause : (AI 92) a. Orceprenaline (Kerala 97)
a. Renal vasodilatory effect b. Terbutaline
b. Increased cardiac output c. Albuterol
c. Peripheral vasoconstriction d. Isoproterenol
d. Prolonged action [Ref. KDT 6/e, p 119, 127]
[Ref. KDT 6/e, p 126] 115. All of the following are selective B2 agonists ex-
109. Effect of Dopamine on Kidney is blocked by : cept: (A.P. 97)
a. Pindolol (Jipmer 93) a. Salmeterol
b. Phentolamine b. Isoproterenol
c. Propranolol c. Metoprolol
d. Haloperidol d. Terbutaline
[Ref. KDT 6/e, p 126] [Ref. KDT 6/e, p 127]
110. Increased renal blood flow is caused by : 116. Use of adrenaline is in : (Kerala 94)
a. Dopamine (Jipmer 98) a. Vasovagal syncope
b. Dobutamine b. Hypotension
c. Noradrenaline c. Anaphylactic shock
d. Adrenaline d. Bradycardia
[Ref. KDT 6/e, p 126] [Ref. KDT 6/e, p 129]
111. Decrease in vascular resistance of mesentric and 117. Presynaptic blocker is : (PGI 95)
renovasculature is caused by : (AI 90) a. Clonidine
a. Dopamine b. Yohimbine
b. Dobutamine c. Phenoxybenzamine
c. Ephedrine d. Methyldopa
d. Noradrenaline [Ref. KDT 6/e, p 132, 135]
[Ref. KDT 6/e, p 126] 118. Tamsulosin, a competitive α2 adrenoceptor an-
112. True about terbutaline is : (NIMS 96) tagonist has affinity for which of the following re-
a. It is an alpha agonist ceptors: (Karn. 06)
b. It is an alpha antagonist a. α1A
c. It is a β2 agonist b. α1B
d. It is a β2 blocker c. None of the above
[Ref. KDT 6/e, p 127] d. Both of a and b
113. Phenylephrine is useful for all of the following ex- [Ref. KDT 6/e, p 135]
cept : (AI 91) 119. Which of the following is beta specific blocker :
a. Anaphylactic shock a. Esmolol (Karn 94)
b. Mydriasis b. Betaxolol
c. Nasal decongestant c. Atenolol
d. Atrial tachycardia d. Sotalol
[Ref. KDT 6/e, p 127] [Ref. KDT 6/e, p 136]

Answer 108. a. Renal ... 109. None 110. a. Dopamine 111. a. Dopamine 112. c. It is a β2 ...
113. a and d 114. d. Isoproterenol 115. b. Isoproterenol 116. c. Anaphylactic ... 117. b. Yohimbine
118. d. Both of a and b 119. a, b and c

148
Phar macology of ANS, PNS & A
Pharmacology utacoids
Autacoids 2
120. Which of the following nonselective adrenergic a. Intermittent claudication
antagonists causes peripheral vasodilatation : b. Portal hypertension
a. Atenolol (Kerala 01) c. Migraine
b. Carvedilol d. Benign essential tremors
c. Sotalol [Ref. KDT 6/e, p 141, 143]
d. Nadolol 127. Propranolol is indicated in the following conditions
[Ref. KDT 6/e, p 136] except : (UPSC 04)
121. All are α blockers except : (ROHTAK 97) a. Intermittent claudication
a. Tolazoline b. Portal hypertension
b. Prazosin c. Migraine
c. Yohimbine d. Benign essential tremors
d. Butoxamine [Ref. KDT 6/e, p 139, 142 - 143]
[Ref. KDT 6/e, p 132, 136] 128. Which of the following anti-tussive drug does not
122. B-blockers are contraindicated in : act through opioid receptors ? (MAHE 02)
a. Hypertension (NIMHANS 86) a. Propoxyphene
b. Congestive cardiac failure b. Codeine
c. Anxiety states
c. Noscopine
d. Hyperthyroidism
[Ref. KDT 6/e, p 139] d. Dextromethorphan
123. Beta blocker without local anesthetic effect is : [Ref. KDT 6/e, p 214 - 215]
a. Metoprolol (Delhi 89, PGI 92) 129. Drugs used in migraine prophylaxis are all except:
b. Pindolol a. Flunarazine (NIMHANS 2K)
c. Atenolol b. Propranolol
d. Timolol
c. Cyprohetadine
[Ref. KDT 5/e, p 138, 140, table (10.2)]
d. Sumatriptan
124. Following drugs have a membrane stabilizing ef-
[Ref. KDT 6/e, p 170 - 171]
fect except : (PGI 90, 93)
130. Beta agonist used in bronchial asthma exert their
a. Atenolol
action by : (Jipmer 90)
b. Timolol
a. Selective β1 agonism
c. Metoprolol
b. Selective β2 agonism
d. Nadolol
c. Selective β1 antagonism
[Ref. KDT 6/e, p 140]
d. Selective β2 antagonism
125. Short elimination half-life (8 to 10 min) of esmolol
[Ref. KDT 6/e, p 217]
(beta 1 - adrenergic blocker) is due to :
131. Anti-inflammatory action of corticosteroids is due
a. Rapid redistribution (Karnataka 03)
to blocking of : (J & K 05)
b. Rapid elimination by kidney
a. 15 LIpo-oxygenase
c. Hydrolysis by blood esterases
b. Prostaglandin synthetase
d. Rapid protein binding
c. Thromboxane synthetase
[Ref. KDT 5/e, p 141]
d. Breakdown of phospholipids
126. Propranolol is indicated in the following conditions
[Ref. KDT 6/e, p 278]
except : (UPSC 04)

Answer 120. b. Carvidelol 121. d. Butoxamine 122. b. Congestive ... 123. d. Timolol 124. a and d
125. c. Hydrolysis ... 126. a. Intermittent ... 127. a. Intermittent ... 128. d. Dextrome ... 129. d. Sumatriptan
130. b. Selective ... 131. a. 15 LIpo- ...

149
2 Self Assessment & Review Phar macology
Pharmacology
132. Which one of the following drugs is not a long act- b. Its plasma half-life is 6 hours
ing neuromuscular blocking agent? c. Therapeutically useful in parkinsonism
a. Doxacurium (Karnataka 06) d. Acts only on dopaminergic receptors
b. Mivacurium [Ref. KDT 6/e, p 126]
c. Pancuronium 138. First does hypotension is caused by :
d. Pipecuronium a. Prazosin (MAHE 05)
[Ref. KDT 6/e, p 339] b. Clonidine
133. Which is the drug which causes post-anesthetic c. Sodium nitroprusside
muscle stiffness? d. Propranolol
a. Fentanyl (J & K 05) [Ref. KDT 6/e, p 545]
b. Pyridostigmine 139. Postural hypotension is commonly seen with :
c. Succinylcholine a. Captopril (Kerala 04)
d. Gallamine b. Sodium nitropruside
[Ref. KDT 6/e, p 3376 - 377] c. Prazosin
134. True about tyramine are all except : d. Labetalol
a. Amphetamines (AIIMS 86; Delhi 93) [Ref. KDT 6/e, p 545]
b. Biguanides 140. Mannitol decreases the intraocular pressure by :
c. Alpha blocking agent a. Withdrawing water from vitreous (COMED 06)
d. A beta blocking agent b. Increasing uveoscleral outflow
[Ref. KDT 6/e, p 120 - 121] c. Increasing aqueous through trabecular mesh-
135. Drug used in alzheimer’s disease having adverse work
effect on liver : (Jipmer 04) d. Inhibiting the enzyme carbonic anhydrase in the
a. Tacrine ciliary body
b. Donepezil [Ref. KDT 6/e, p 147, 561, 572 - 573]
c. Thioctic acid 141. Kallikerein is most commonly found in the follow-
d. Flumazenil ing part of kidney : (SGPGI 04)
[Ref. KDT 6/e, p 472] a. Cortex
136. Donepezil is used in the medication is associated b. Medulla
with an increased risk of agranulocyosis? c. Papilla
a. Schizophrenia (Karnat. 05) d. Collecting duct
b. Depression [Ref. Goodman & Gilman 10/e, p 663]
c. Anxiety 142. Which of the following drug is contraindicated in
d. Alzheimer dementia pheochromocytoma? (Delhi 93)
[Ref. KDT 6/e, p 472 - 473] a. Guanethidine
137. Which of the following statements about dopam- b. Phenoxybenzamine
ine is correct : (Karnataka 94) c. Pabelalol
a. Used in congestive cardiac failure d. Labetalol
[Ref. Harrison 16/e, p 1472, table (230.8)]

Answer 132. d. Pipecuronium 133. a. Fentanyl 134. None 135. a. Tacrine 136. d. Alzheimer ...
137. a. Used in ... 138. a. Prazosin 139. c. Prazosin 140. c. Increasing ... 141. d. Collecting ...
142. a. Guanethidine

150
INTRODUCTION TO CENTRAL NERVOUS SYSTEM
Neurotransmitters are substances synthesized in the pre-synaptic neurons and stored in vesicles, then released and
inactivated after action. When the same compound is administered exogenously, it produces the same response.
The response may be excitatory or inhibitory.
Neuromodulators are substances released by the neurons and also by astrocytes, modulate the actions of the
neurotransmitter. Latent action neither stored nor released .Related to synapticen plasticity e.g., CO2, adenosine, purines,
NO, PG, Peptides (Mainly G-Protein coupled receptors)
Neuromediators -Second messenger e.g. cAMP, cGMP, IP3

Neurotransmitters
1. Acetylcholine :
• It was the first compound to be identified pharmacologically as a CNS transmitter.
• It is the transmitter in a variety of pathways such as neostriatum, medial septal nucleus and the reticular formation.
• Loss of cholinergic neurons is associated with memory dysfunction (Pre-senile dementia of Alzheimer type). The
drugs used in Alzheimer disease are anticholinesterases as donepezil, tacrine, rivastigmine.

2. Monoamines (5-HT, Dopamine, Norepinephrine, Histamine)


• Dopamine : is the transmitter in the projection linking :
a. substantia nigra to neostriatum (the action of antiparkinson drug, levodopa is associated with this area) and
b. ventral tegmental region to limbic structures (the action of antipsychotic drug is associated with this area)
– Dopamine generally exerts a slow inhibitory action on CNS neurons.
• Norepinephrine : The noradrenergic neurons are mostly located in the locus ceruleus or the lateral tegmental area of
the reticular formation.
– Facilitates excitatory CNS function associated with attention arousal etc. However, it inhibits CNS function through
hyperpolarisation in locus ceruleus neurons.
– Cocaine inhibits re-uptake of DA and NE.
– Amphetamine causes release of DA and NE.
• 5-Hydroxytryptamine : 90% of body’s content of is localized in enterochromaffin cell in g.i.t, the rest in platelets and brain.
– The 5-HT pathways originate from neurons in the raphe or midline regions of the pons and upper brain stem.
– The 5-HT has a strong inhibitory action on most areas of the CNS mediated via the 5-HT1A receptors.
– It is associated with sleep, temperature, appetite and neuroendocrine control.
– The 5-HT agonist is metoclopramide.
– All 5-HT receptors arc GPCR except 5-HT3, which are ionotrophic.

3. Amino acid neurotransmitters


• Inhibitory - GABA, Glycine
• Excitatory - Aspartate, Glutamate
Glutamate major focus – in memory acquisition, synaptic plasticity, epilepsy, excitotoxicity
• Glutamate is present in very high concentration in the CNS.
• Glutamate receptors are both ionotropic (Kainate, AMPA and NMDA) and metabotropic (ACPD).
3 Self Assessment & Review Phar macology
Pharmacology
• The NMDA receptors play a critical role in synaptic plasticity, which is thought to underlie certain forms of learning and
memory.
• NMDA receptors are selectively blocked by Ketamine and Phencyclidine.

4. Peptides :
– Includes opioids (enkephalins, endorphins), substance-P, neurotensin, somatostatin, CCK, VIP, TRH.
– Primarily control nociception hunger, metabolism etc.

5. NO : Nitric oxide has been detected in the CNS but its exact role, remains to be elucidated.

6. Endocannabinoids :
– It affects brain through a receptor known as CB1. The drug Anandamide has been developed as a CB1 ligand.
– It has been implicated to play a role in memory, cognition and pain perception.

Benzodiazepines and Barbiturates


• The action of these drugs is mediated through the inhibitory neurotransmitter gammaaminobutyric acid (GABA), which
governs the activation of the chloride channel within the membrane.

Drug affecting GABAA-receptor gated chloride channel

GABA Endogenous agonist at GABAA receptor →promotes CI- influx.


Muscimol Agonist at GABAA site
Bicuculline Competitive antagonist at GABAA receptor
PIcrotoxin Blocks CI- channel noncompetitively; acts on picrotoxin sensitive site
Barbiturate Agonist at an allosteric site (picrotoxin site); prolong GABA action; open CI- channel
Alcohol, inhalational Open CI- channel directly; allosteric facilitation of GABA.
anesthetics, propofol
Benzodiazepine Agonist at an allosteric BZD site →facilitate GABA action
β-carboline (DMCM) Inverse agonist at BZD site →impede GABA action
Flumazenil Competitive antagonist at BZD site

• These drugs may be used as sedatives, anxiolytics, muscle relaxants, or hypnotics, depending on the individual drug
(e.g., triazolam as a hypnotic and alprazolam as an anxiolytic).
• Benzodiazepines are contraindicated in a patient who is pregnant or lactating, diagnosed with myasthenia gravis (due to
muscle relaxant activity), acute depression, or psychosis (due to general CNS depression). Because of the anticholinergic
effects of drugs of this class, therapy in patients with narrow angle glaucoma also is contraindicated.
• Most benzodiazepines first undergo microsomal oxidation through cytochrome P450 mixed function oxidases (phase I).
The metabolities then undergo phase II glucuronide conjugation and are eliminated in the urine. Metabolites produced
through phase I oxidation may have pharmacologic activity, depending on the parent drug. Secondary metabolites may
also be pharmacologically active.
• Chloriazepoxide, desmethyldiazepam, diazepam, and flurazepam have active metabolites that are subject to
enterohepatic recycling, which accounts for their relatively long half lives. Alprazolam and triazolam also have active
metabolites. However, these are rapidly inactivated by glucuronidation, which accounts for their relatively short half-lives.
• Lorazepam has been used as a hypnotic. It is useful in the “premature awakening” type of insomnia, due to its long half-
life. It is also used as an anxiolytic and skeletal muscle relaxant.
• Barbiturates (only 2 are clinically useful: Thiopentone and phenobarbitone). Elimination half-life of phenobarbitore is 4-
5 days.
• Phenobarbitone is acidic and excreted partly unchanged in urine 20-30%. Alkalinisation of urine increases elimination.
• Hepatic drug metabolism is induced by some barbiturates but not by any benzodiazepines.
• BZ receptors are classified as BZ1 (Omega 1) and BZ2 (Omega 2 receptors). ALL benzodiazepines bind to both BZ1 and
BZ2 receptors. Zolpidem, Zaleplon and Zaldiplon bind BZ1 receptors only. BZ2 receptors are associated with muscle
relaxation, amnesia and anticonvulsant actions.

154
Phar macology of Central Nervous System
Pharmacology 3
• GABA is the major inhibitory neurotransmitter in CNS. BZDs potentiate GABA mediated inhibition all over CNS. BZDs
increase the frequency of GABA mediated inhibition (i.e. chloride channel opening), while barbiturates increase the
duration of GABA inhibition. At very high concentration, barbiturates directly increase Cl- conductance (i.e., GABA Mimetic
action) while BZDs are GABA Facilitatory action at all dose ranges.

Effects
1. Sedation : Suppression of responsiveness to a constant level of stimulation with decreased spontaneous activity.

2. Amnesia : Anterograde amnesia associated with BZs.

3. Hypnosis : All sedative hypnotics induce sleep


– Sleep has 2 components REM (30%) and NREM (70%). REM and NREM occur cyclically after 90
min. Recallable dreams occur during REM.
– NREM has 4 stages (1 -4).
Greatest proportion of sleep (50%) occurs in stage-2.
Stages 3 and 4 are known as delta or slow wave.
• Sleep somnambulism and night terrors occur during slow wave.
• Secretion of growth hormone is highest and that of adrenal steroids is lowest during this phase
Effects of sedative hypnotics (all)
• Decreased latency to sleep onset. • Duration of stage 2 NREM is increased.
• Duration of REM sleep is decreased. • Duration of slow nerve sleep is decreased.
Upon withdrawal of benzodiazepines, there occurs rebound increase in REM sleep. However, sedative hypnotics do not
affect steroid and GH secretion.

4. Anesthesia : Only thiopentone is used for this purpose. High lipid solubility gives good concentration in the CNS. BZDs
are not used as they can hardly produce surgical anesthesia without any respiratory depression. They also potentiate the
respiratory depressant action of other CNS depressants.

5. Anticonvulsant effect : In suitable doses, they inhibit the development and spread of seizures
• Clonazepam, Diazepam, Lorazepam, Nitrazepam are used for this purpose.
• Among Barbiturates, only phenobarbilone is used.

6. Skeletal Muscle Relaxation : Clonazepam, Diazepam (Maximum Skeletal muscle Relaxation = Clonazepam).

7. Respiratory and Cardiovascular system : Barbiturates depress medullary centers more than benzodiazepines.
• Normal doses of sedative hypnotic may cause CVS depression in CHF, Hypovolemic states and other diseases
affecting CVS function.
• At toxic doses, cardiac contractility and vascular tone may be depressed leading to circulatory collapse.

BZD Antagonist
Flumazenil reverses many actions of BZDs and imidazopyridines but not of alcohol, opiates, General anesthetics, barbiturates.
Sedation is most promptly reversed.
Respiratory depression is not reversed completely. Flumazenil has short t½ (0.7-1.3 hrs.), needs repeated dosing.
ADRs : Agitation confusion, dizziness, nausea, seizures.
Uses : Hepatic encephalopathy

Newer Sedative Hypnotics (Imidazopyridines)


Zolpidem and Zaleplon : Binds BZ1 receptors
Minimal muscle relaxing, anticonvulsant, amnestic action, sleep duration is not increased.
• They are short acting, so no day time sleepiness
• Less psychomotor impairment
• CNS depressants like alcohol potentiate the depressant effect including respiratory depression

155
3 Self Assessment & Review Phar macology
Pharmacology
Therapeutic uses of Benzodiazepines
• Generalized anxiety disorder (GAD), Agarophobia, Panic disorder - Alprazolam
• Anticonvulsant effect (status epilepticus, clonazepam, nitrazepam, diazepam)
• Insomnia (short acting ones preferred)
• Sedation and Amnesia before medical procedure eg. endoscopy.
• Ethanol withdrawal: Chlordiazepoxide / Chlormethiazole / Diazepam
• Muscle relaxation: In sports injury; Diazepam
Situational anxiety sympathetic activation : β-Blocker + BZD. They cause memory impairment, so avoid during exams and
stage performances.

Buspirone
• Buspirone is a non-benzodiazepine (Thienopyridine agent) anxiolytic drug.
• Site of action : 5-HT1A receptor subtype agonist.
• No anticonvulsant activity.
• No interaction with benzodiazepine binding sites.
• No influence on interaction of GABA with the GABA receptor.
• Not effective in management of severe anxiety / panic disorder.
• No cross-tolerance, with other sedative-hypnotic drugs, less dependence.
• No muscle relaxant properties. Minimal adverse effects (Most common ADR is headache).
• Clinical effects seen after 1 - 3 weeks.
• Other drugs in this class include, GEPIRONE and IPSAPIRONE.
• Useful in GAD (maintenance therapy)
Buspirone undergoes an extensive first pass metabolism upon oral absorption. It is metabolized primarily by microsomal
oxidation and produces a pharmacologically active metabolite. Multiple-dose studies suggest that steady-state plasma
levels are achieved within 2 to 3 days.

PARKINSON’S DISEASE
Pathogenesis : This disease results in an imbalance between dopamine and acetylcholine levels in the nigrostriatal tract.
Thus, effective therapy results from lowering acetylcholine levels or activity to correlate with reduced dopamine levels, or
increasing dopaminergic activity to correlate with normal acetylcholine levels
• Chemically : Induced: in primates by - 1-methyl-4-phenyl-l, 2, 3, 6-tetrahydropyridine (MPTP), which is converted to the
active form by monoamine oxidase to active toxin: MPP+ – N-methyl-4-phenylpyridinium (MPP+).
• MPP : is taken up into nerve terminals by dopamine reuptake system (active transport). It inhibits oxidative phosphorylation
(possibly complex I; respiratory chain site).
• May also produce oxidative stress / free radicals.
• Consequences : – nigro-striatal neuronal death
– basal ganglia dopamine depletion
– parkinson’s disease
In idiopathic Parkinsonism, there is damage to dopaminergic neurons. Treatment is dopamine replacement.
• In drug induced Parkinsonism (dopamine blockade), antagonizing the effects of acetylcholine is required.

Drugs : L-dopa is a chemical intermediate produced in the synthesis of dopamine. It is formed from the actions of tyrosine
hydroxylase on tyrosine, and is subsequently converted into dopamine by aromatic-L-amino acid decarboxylase (LAAD, or
dopa decarboxylase). This molecule is taken up into the dopaminergic nerve terminal and converted to dopamine, which is
then released into the synoptic cleft. Unlike dopamine, dopa is in nonionized form at physiologic pH and thus will cross into
the central nervous system (CNS).
– Addition of Carbidopa or Benserazide inhibits the decarboxylation and hence enhances the effects of L Dopa.

Interactions :
– Food slows L-Dopa absorption.
– High protein diet interferes with drug transport to CNS.
– Neutral aminoacids like leucine and Isoleucine compete for L-Dopa absorption in the gut.

156
Phar macology of Central Nervous System
Pharmacology 3
Pyridoxine acts as a cofactor in the activity of LAAD and increases the peripheral metabolism of L-dopa, resulting in a
decrease in therapeutic effect

ADR :
– L-dopa stimulates vomiting center so nausea / vomiting
– Cardiovascular effects are primarily due to increased production of norepinephrine. and include atrial and ventricular
arrhythmias, ventricular tachycardia, and atrial fibrillation. These effects are markedly reduced with concurrent
administration of an inhibitor of peripheral LAAD
– Brownish discoloration of saliva / urine / tear due to oxidation of catecholamine’s to melanin
– Hallucinations (visual and auditory)
– Mood changes depression and anxiety

Other agents
Ergot derivatives
Bromocriptine : Dopamine receptor agonist used for suppression of lactation.
Cabergoline / Pergolide, lusuride ergot derivatives more side effects

ADRs of ergot derivatives : Erythromyalgia (Painful tender swollen red feet), Retroperitoneal fibrosis, vasospasm

Newer agents : Pramipexole and Ropinirole (non ergot) - Direct dopamine agonists used as monotherapy.

Selegiline, Deprenyl (MAO-B) inhibitors : Inhibits dopamine degradation. Selegiline is an inhibitor of monoamine oxidase B,
a subtype of MAO that is selective for the degradation of dopamine. By decreasing the metabolism of dopamine in the
substantia nigra, levels of dopamine increase and the DA/ACh ratio is improved.
Selegiline may directly result in decreased production of reactive oxygen species (hydrogen peroxide, superoxide anion,
hydroxyl radical) in dopaminergic neurons, resulting in a decreased rate of neuronal degeneration.

COMT Inhibitors : Tolcapone and Entacapone : The actions of COMT not only reduce circulating levels of L-dopa, which
decreases the effectiveness of the- drug, but the metabolite produced—3-methyl dopa—reduces the effect of L-dopa,
because it competes with the drug for transport across cell membranes, particularly those of the intestinal mucosa, and
structures of the blood-brain barrier. Thus, inhibition of COMT may increase the absorption of orally administered L-dopa,
and also increase the proportion of, drug entering the CNS.
Tolcapone crosses blood brain barrier, entacapone acts at the periphery. ADRs include diarrhea, orthostatic hypotension
and orange-red colored urine.

Drug induced parkinsonism : Anticholinergics such as Benzhexol (trihexiphenydyl), Benztropin, Orphenadrine, Biperiden,
and Procyclidine. Sometimes older H1 Blockers such as dimenhydrinate, triprolidine are also used as they have a strong
anticholinergic component.

Antipsychotic drugs
Besides dopamine; 5-HT, Glutamate and Norepinephrine are strongly implicated in schizophrenia.
D2, D4 and 5-HT2 receptor blockers have clinical influence in the treatment. Glutamate is an excitatory neurotransmitter that
binds to NMDA receptors. NMDA receptor blockers such as phencyclidine, ketamine and dizocilpine produce hallucinations
and thought disorders in man. This suggests reduced glutamate may be a factor in the causation of psychosis.
Dopamine excess is implicated in schizophrenia. This is accepted by most, since all antipsychotic drugs block D2 receptors
besides other receptors. The potency of antipsychotic drug effect depends on the degree of D2 blockade

Effect of antipsychotic drugs in man


– Antipsychotic drug effect takes about 3-4 weeks to develop, although dopamine blocking effect is immediate.
– Proliferation of dopamine receptors occurs in chronic treatment
• Depending on the class of drugs, varying degrees of extrapyramidal, anticholinergic, antihistaminic (including sedation
and autonomic effects can occur.
• Cardiovascular effects such as hypotension may be due to the autonomic effects, but direct cardio-depressant effects
also occur with these drugs.
• Dopamine is an inhibitor of prolactin release. Thus, a patient on antipsychotics may experience (depending on gender)
galactorrhea, amenorrhea, gynecomastia, and changes in libido.

157
3 Self Assessment & Review Phar macology
Pharmacology
Other effects
• Apathy and reduced initiative
• Slow response to external stimulus
• Tendency to drowsiness but easily aroused
• No marked loss of intellectual function
• Diminished aggressiveness

ADRs
1. Extrapyramidal effects: Results from D2 Blockade
(This is the major disadvantage of classical antipsychotics Atypical ones lack this effect) - It can be
A. Acute dystonias
• Occur in the 1st week of treatment usually
• Torticollis, protruding tongue, muscle spasms are common
• They often decline with time or after stopping the treatment
• Acute dystonic reactions are not encountered with atypical antipsychotics
B. Tardive dyskinesia: Develops after months to years
Tardive dyskinesia, which, once established, is irreversible.
Involuntary movements of face, trunk, limbs, tongue occurs - worsens on discontinuation of drug therapy
• Associated with higher dose and elderly age (above 50 years)
• Not encountered with atypical antipsychotics
C. Rabbit syndrome: Perioral tremor (after prolonged treatment)

Endocrine effects : Dopamine acting on D2 receptors inhibits prolactin secretion. Thus D2 blockers increase plasma
prolactin levels. Antipsychotic Drug-Induced endocrine Changes
In Women : • amenorrhea/galactorrhea
• false-positive pregnancy test results
• increased libido
In Men : • decreased libido
• gynecomastia
Mechanisms : • blockade of dopamine-mediated tonic inhibition of prolactin secretion
• increased peripheral androgen to estrogen conversion

Atypical antipsychotics : clozapine, olanzapine, risperidone, Quetiapine, ziprasidone, aripiprazole cause absence / minimal
prolactin increases: perhaps indicative of reduced D2 receptor blockade: consistent with reduced extrapyramidal dysfunction
(tardive dyskinesia) and reduced endocrine anomalies.
A high degree of blockade of 5HT4 receptors occurs. Clozapine may improve “negative” symptoms related to lack of motivation,
a social behavior, and poverty of speech often seen in schizophrenia.
Clozapine has more affinity for line D4 receptor than the D1 or D2 receptors. This drug also has a higher affinity for non-
dopaminergic receptors (e.g.. 5HT4, H1, M1,) than for dopaminergic receptors. This accounts for the marked sedation and
autonomic action.

Aripiprazole : – DA:5HT stabilizer partial agonist at D2, 5HT1A & antagonist at 5-HT2A
– It has no EPS, no weight gain, no endocrinal, no ECG changes, side effects
– Long acting & has high affinity

ADR
1. Obstructive Jaundice with phenothiazines
2. Hypotension (α1 block), Blurred vision, dry mouth (Muscarinic block) is commonly associated with older agents
4. Weight gain : (postulated to be 5-HT block)
5. Leukopenia and agranulocytosis. (Mainly with clozapine on idiosyncratic reaction), Agranulocytosis in a small proportion
of patients. It may also cause seizures.
6. Retinal deposits occur with thioridazine (Resembling retinitis pigmentosa)
7. Corneal / Lenticular deposits with - Cornea and lens deposits: complication of chlorpromazine treatment

158
Phar macology of Central Nervous System
Pharmacology 3
NMS is a rare but dangerous condition (muscle rigidity with ↑ ↑ body temperature)

Malignant Neuroleptic Syndrome : Life-threatening - observed in patients sensitive to antipsychotic extrapyramidal effects.
This syndrome is a combination of symptomatic effects produced by anti psychotic drug. Symptoms and signs include
hyperpyrexia, muscle rigidity, altered mental status (e.g., catatonia) and cardiovascular instability (e.g.. unstable heart rate
and blood pressure}. Acute renal failure may ultimately occur. Diagnostic signs include elevated creatinine phosphokinase
(CPK) and myoglobinuria
Accepted therapies include the administration of dantrolene to reduce muscle rigidity, and antiparkinson drugs such as
bromocriptine and amantadine to mediate extrapyramidal symptoms. Supportive therapy is also instituted. Withdrawal of the
neuroleptic drug is mandatory

8. Cardiotoxicity :
Thioridizine (minor T wave abnormality)
In over dosage: – ventricular arrhythmias
– abnormal cardiac conduction
– sudden death
• Thioridazine : drug-drug interactions
– thioridazine plus tricyclic antidepressants - requires cautious use
– possible additive antimuscarinic + quinidine-like effects
• Antipsychotic with antidepressant actions →SULPIRIDE
• Resistant psychosis responds to atypical agents

Affective Disorders
• Tricyclic Anit Depressant (TCA) : Amitriptyline imipramine, desipramine, clomipramine, protryptyline, doxepine. Between
the drugs, therapeutic differences are little.

• MOA : They block re-uptake of amines at nerve terminals


Tricyclics inhibit the reuptake of catecholamine’s Mainly NE and 5-HT but not dopamine from the synaptic cleft, thus increasing
the amount of transmitter available for postsynaptic stimulation. The neurotransmitters affected and the degree to which they
are affected varies with the individual drug
Besides these TCAs have antimuscarinic, α-blocking action. Down-regulation of β-receptors is consistent, and proposed to
be the basis of therapeutic effect.
Due to the lag time in therapeutic effect, it is thought that tricyclics manifest their true clinical effects by down regulation of beta
receptors in the limbic system. With increased concentrations in synaptic norepinephrine, over a period of time the receptors
desensitize and down regulate.

ADR
• These drugs are sedating, and have a variety of untoward effects. The most commonly seen major adverse effect is
postural hypotension. Other adverse effects include anticholinergic effects (e.g. dry mouth, constipation, urinary retention,
paralytic ileus, and tachycardia), blurred vision, increased intraocular pressure, gynecomastia, galactorrhea, and changes
in libido. Glucose intolerance Tricyclics lower seizure threshold.

USES : These drugs are used to -

– alleviate depression – diabetic neuropathy.


– obsessive compulsive disorder (OCDs) – attention-deficit hyperactivity disorder
– childhood enuresis (e.g., imipramine) – fibromyalgia (e.g., amitriptyline)
– chronic obstructive pulmonary disease (COPD) (e.g., protriptyline)

Contraindication :
• Aspirin, phenylbutazone displace from plasma binding of TCAs and produced exaggerated responds
response (This is relatively temporary).
• Imipramine is contraindicated in the presence of prostatic hypertrophy

159
3 Self Assessment & Review Phar macology
Pharmacology
Toxicity : Resembles atropine poisoning and responds to physostigmine. Death occurs from ventricular fibrillation.
Amoxapine is a heterocyclic antidepressant. It is a metabolite of the antipsychotic drug, ioxapine, and thus has the additional
action of dopaminergic blockade. This action makes it a potentially useful therapy in depressed psychotic patients
Adverse effects are similar to those seen with tricyclic antidepressants. Additionally, dopamine-related effects, such as
akathisia, galactorrhea/amenorrhea, and Parkinson-like symptoms are seen.
Mirtazapine blocks central adrenergic receptors, interrupting the negative feedback loop and increasing circulating
norepinephrine. In addition, it acts at a, receptors on central serotonergic neurons, increasing the amount of circulating
serotonin.
Nefazodone antagonizes a, receptors (heteroreceptors) which mediates an increase in serotonin outflow, in addition, the
drug is a potentiate antagonist at 5-HT, receptors, and, additionally, inhibits presynaptic reuptake of serotonin. Thus, the
actions of serotonin in mood elevation are maximally potentiated.
Unlike tricyclic drugs, nefazodone lacks the potential for major cardiotoxicity
Trazodone has substantially less anticholinergic activity than tricyclics, and no cardio depressant actions. It thus has less of
an adverse cardiovascular effect. In addition, trazodone is sedating but does not interfere with phase 4 sleep.

Selective Serotonin Reuptake Inhibitors (SSRIs)


Fluoxetine, Paroxetine, Fluvoxamine, Citalopram (Escitalopram), Sertraline
SSRls have less sedative, anticholinergic, and cardiovascular effects than do the tricyclic antidepressants

MOA : The primarily inhibit re-uptake of 5-HT


They have no effects on other receptors like α and Muscarinic.
No cheese-reaction unlike MAO-inhibitors

P/K : Fluoxetine has plasma t½ of 24-96 hours (longest)


Rest agents have plasma t½ of 15 - 24 hours
Fluoxetine and Paroxetine are hepatic microsomal enzyme inhibitors, so not to be combined with TCAs for fear of acute TCA
toxicity.

ADR : Nausea, anorexia, insomnia, failure to orgasm, ↓ libido


Serotonin syndrome - Tremor, hyperthermia, CVS collapse, muscle rigidity

Paroxetine SSRIs have the highest specificity for serotonin receptors. The elimination half-life is approximately 24 hours.
paroxetine is significantly more potent than either fluoxetine or sertraline in the inhibition of serotonin reuptake

Uses of SSRIS
• Major depression • Anorexia nervosa
• Bulimia nervosa • OCD
• Panic disorder • Pain of diabetic neuropathy
• Premenstrual syndrome

MAO :
MAO is of 2 types i.e. MAO-A and MAO-B
• 5HT is metabolized by MAO-A
• NE and Dopamine are degraded by both MAO-A and MAO-B
MAO-A is inhibited by chlorgyline, moclobemide
MAO-B is inhibited by deprenyl (selegiline)
Tyramine is degraded by both.
MAOI in a normal subject produces excitement and euphoria, while TCAs produce sedation and confusion.

ADR : Hypotension due to sympathetic block (α1)


Excessive central stimulation - Tremor, excitement, insomnia
Weight gain with increased appetite
Atropine like effects (Milder than TCAs)

160
Phar macology of Central Nervous System
Pharmacology 3
D/I : Cheese Reaction: Tyramine is metabolised by MAO. In presence of MAOI, tyramine is not degraded and
absorbed, which is stored in synaptic vesicles by displacing NE → Dangerous hypertension.
Treat cheese reaction with phentolamine / labetalol / prazosin / CCBs

RIMA : Reversible inhibitor of MAO-A


Includes moclobemide, brofaromine
They inhibit 5-HT degradation and tyramine is spared. So no cheese reaction is seen.

VENLAFAXINE : Blocks reuptake of 5-HT and NE


Mirtazapine : Central alpha-2 auto receptor antagonist,& 5HT1 heteroreceptor resulting in increased release of biogenic
amines effective.

TIANEPTINE : Antidepressant acts enhancing re-uptake of 5-HT.


(This has challenged to the 5-HT theory of depression)

MOOD Stabilizer : Lithium


Pharmacodynamics - Possible mechanisms of action:
– effects on electrolyte / ion transport
– neurotransmitter - neurotransmitter release modulation
– Influence on second messengers mediating transmitter action

Ion Transport Effects :


Lithium readily enters excitable cells via sodium channels. it is not however, removed well by the sodium/potassium ATPase
exchange mechanism. It therefore accumulates within the cell. Thus, the intracellular concentration of potassium is decreased
as the influx of potassium is reduced, both through inhibition of active transport (by Na+K+ ATPase) and the decrease in the
electrical gradient for potassium. Extracellular potassium therefore increases. The reversal in potassium levels results in a
decrease in neuronal excitability, producing a therapeutic calming effect.

Neurotransmitter Effects - variable; possible influences on noradrenergic, dopaminergic, and / or cholinergic systems.

Second Messenger Effects : Lithium influences 1P3 / DAG systems inhibiting enzymes that control normal recycling of
membrane phosphoinositol - including :
– inhibiting conversion of IP2 to IP1
– inhibiting conversion of IP to inositol
– these effects result in depletion phosphatidylinositol-4, 5 bisphosphate (PIP2)
– PIP2 : membrane precursor to inositol triphosphate and diacylglycerol (IP3 and DAG)
– Lithium may also inhibits norepinephrine-sensitive adenylyl cyclase
– Effects on the IP3 / DAG system and Adenylyl cyclase second messenger systems suggest lithium may influence G
protein coupled signal

IMPORTANT
Lithium therapeutic Plasma Cone: [0.4 - 1.2 mmol / Liter], Safe level < 1.5 mmol/L. lt is distributed in total body water (40L).
Not plasma protein binding: Biphasic elimination rate so concentrations determined 12 hours after the last dose Excretion:
urinary excretion; about 20 percent of creatinine clearance; halt-life (plasma) = 20 hours.
Lithium carbonate: an effective, probably preferred treatment for bipolar disorder (manic phase particularly)
• Valproate and carbamazepine also are used for this indication
• In severely manic cases: concurrent use of Benzodiazepines and antipsychotic agents may be required
• Remission rate for manic phase: 60%-80% – outpatients; inpatient success rate lower
• Severe mania: probably necessary to add lorazepam or clonazepam

Drug-Drug Interactions:
• Reduce lithium clearance also associated with: newer NSAIDs – not reported for either aspirin or acetaminophen
• Lithium enhances extrapyramidal syndromes associated with most classical antipsychotic agents (this finding may not
applied to the “atypical” newer antipsychotics

161
3 Self Assessment & Review Phar macology
Pharmacology
Adverse Effects :
Neurological / Psychiatric reactions
Tremor— common side effect
Atenolol or propranolol may reduce lithium-induced tremor (May aggravate Sick sinus syndrome)
Other neurological abnormalities :
• choreoathetosis
• motor hyperactivity
• ataxia
• dysarthria
• aphasia
Psychiatric manifestations: at high / toxic doses :
• mental confusion
• abnormal motor movements

Thyroid function effects :


• Typically decreases thyroid function – reversible
• Few patients show symptoms of hypothyroidism or thyroid enlargement

Renal Effects :
• Polydipsia; polyuria – frequent and reversible – occurs at therapeutic plasma concentrations
• Mechanism of action : collecting tubule does not conserve water under the influence of lithium
• Consequence : excessive free water clearance – nephrogenic diabetes insipidus resistant to vasopressin, responsive
to amiloride
• Edema : frequent adverse effects; may be due to enhanced sodium retention due to lithium

Cardiac Effects : Bradycardia / tachycardia (“sick sinus syndrome”) is the contraindication to lithium use – lithium depresses
the SA nodes

Lithium and Pregnancy :


• Special monitoring required during pregnancy (levels are likely to be unstable)
• Lithium : transferred to nursing infants through breast milk (1/3 to 1/2 of serum levels)
• Lithium toxicity in the newborn:
– Lethargy
– poor suck
– Moro reflux
• Lithium : relatively low-risk of teratogenic effects; earlier studies had reported an increased risk of Ebstein’s abnormality
(cardiac valvular defect) in lithium babies.
• Management of Lithium overdosage :
– peritoneal dialysis – effective
– hemodialysis – eflective; preferred
• Continued analysis until plasma concentration falls below normal therapeutic range

ANTIEPILEPTIC AGENTS
MOA of anti epileptic drug action
– Enhancement of GABA
– Inhibition of Na+ channel function

Other proposed MOA of drugs useful in epilepsy are:


– Calcium channel block
– Glutamate receptor block

162
Phar macology of Central Nervous System
Pharmacology 3
a) Enhancement of GABA
• Phenobarbitone and Benzodiazepine enhance activation of GABAA / receptors thus facilitating GABA mediated chloride
channels. Levetiracetam is GABA receptor modulator.
• Newer agent Vigabatrin inhibits GABA transaminase that degrades GABA, so GABA effect is enhanced
• Tiagabine inhibits GABA re-uptake.
• Gabapentin; GABA agonist but does not seem to act on GABA receptors. It affects the metabolism, re-uptake and
release of GABA

b) Inhibition of Na+ channel function


Phenytoin, Valproate, Carbamazepjne, Lamotrigine, Zonisamide affect membrane excitability by blocking inward Na+
current necessary to generate an action potential. The block shows the property of use dependence i.e those neurons
are affected most who are repeatedly tiring. They block less the less frequently firing neurons.

c) Drugs that block excitatory receptors, i.e NMDA, AMPA receptor. Have not been successful in man

d) Miscellaneous mechanism; Acetazolamide : by causing mild CNS acidosis suppresses all types of seizure, Tolerance
develops rapidly.

1. PHENYTOIN : is useful against various forms of partial and generalized seizures, but not against absence seizures which
may even get worse
P/K - 80 - 90% bound to plasma protein
Salicylates, valproate, phenylbutazone displace phenytoin and increase free plasma concentration
– It induces metabolism of other drugs such as warfarin.
– Drugs like phenobarbitone and ethanol initially inhibit metabolism of phenytoin by competition and later induce it.
Thus initially they enhance and later depress the effects of phenytoin
Phosphenytoin is a prodrug - Converted to phenytoin
This drug show 1st, zero and pseudo zero order kinetics
Safe plasma concentration is 10-20 mg/L

ARDS : MILD - Nystagmus, Vertigo, Ataxia, Headache, but no sedation


Higher dose - Marked confusion and intellectual deterioration
LATE features - gum hyperplasia / Hirsutism / Megaloblastic anemia
(Anemia can be corrected by folate)
In pregnancy - cleft palate
D/I glucose precipitates phenytoin, so use, and normal saline for infusion. Phosphenytoin does not precipitate
C/I - Heart block / hypersensitivity to phenytoin

2. CARBAMAZEPINE : Chemically derived from TCA


More effective in treating complex partial seizure
Also used in Trigeminal neuralgia, as a mood stabilizer and as a co-analgesic along with opiates
(Recently used in post herpetic neuralgia)
P/K: Initially its plasma t½ is 30 hrs. later reduced to 15 hours by auto induction of metabolism.
It induces metabolism of warfarin, corticosteroids, and oral contraceptives, phenytoin

3. SODIUM VALPROATE : Enhance GABA content in the brain by inhibiting GAT and Succinic semialdehyde
ADR : Thinning and curling of hair
Most serious – Hepatotoxicity
C/I: Spina bifida occurs if given in pregnancy.

4. ETHOSUCCIMIDE : Acts by blocking T type calcium channels. DOC of Absence seizures. It can precipitate tonic- clonic
seizures. Phensuximide acts similarly.

163
3 Self Assessment & Review Phar macology
Pharmacology
5. PHENOBARBITONE :
– An Enzyme inducer - Lowers concentration of steroids / OCs / warfarin and TCAs.
Osteomalacia and megaloblastic anemia are also seen

6. BDZs : Clobazam / Clonazepam / Diazepam / Clorzepate.


– Sedation
– Rebound seizure on sudden withdrawal
– Floppy baby syndrome (Flabby baby)
– Used in status (not for maintenance)

SAFEST ANTIEPILEPTIC IN PREGNANCY – PHENOBARBITONE

ANESTHETIC AGENTS
Balanced anesthesia should have
• Adequate analgesia with amnesia
• Reversible loss of consciousness
• Skeletal muscle relaxation
• Suppression of undesired reflexes
No anesthetic agent is ideal: so use adjuncts
• BDZs - To relieve anxiety, facilitate amnesia
• Antihistamines - To prevent allergic reaction
• H2 blockers - To reduce gastric acid secretions
• Opioids (Morphine / Fentanyl) - for analgesia
• Anticholinergics atropine, glycopyrrolate - To prevent bradycardia and facilitate drying of secretions (mainly respiratory
tract)

Pre-anesthetic Medication : Rationale


Primary Goals:
• Anxiety relief without excessive sedation
• Amnesia during perioperative period while retaining cooperation
• Relief of preoperative pain

Secondary Goals:
• Reduction in the requirement for inhalational agents
• Reduction in side effects associated with some inhalational agents
• Side effects include salivation, bradycardia, postanesthetic vomiting.
• Reduction in acidity and volume of gastric contents
• Reduction of stress in perioperative period

Induction of Anesthesia:
• Aims at reducing stage II (CNS excitation occurs resulting in delirium and combative behavior)
• Induced by Thiopentone / Propofol
• Co-administration of skeletal muscle relaxant facilitates intubation (Vecuronium / Pancuronium / Succinylcholine)

Maintenance by mixture of volatile gases / liquids:


Recovery is the reversal of induction -
• ED50 of anesthetic gases is determined by Minimum Alveolar Concentration
• MAC is an indicator of Potency
• More lipid solubility means small MAC (More potent)
MAC values typically used refer to 40 year-old individual -
• halothane (Fluothane) : 0.75%
• isoflurane (Forane) : 1.17%

164
Phar macology of Central Nervous System
Pharmacology 3
• enflurane (Ethrane) : 1.63%
• sevoflurane (Sevorane, Ult ane) : 1.8%
• desflurane (Suprane) :6.6%
• nitrous oxide : 104%
Rationale for using alveolar concentrations rather than brain levels
• Alveolar concentrations are easy to measure accurately.
• At equilibrium the partial pressure of the gas in the lung is about the same as in the brain.
• Blood and brain anesthetic concentrations rapidly equilibrate.

HALOTHANE
• Direct myocardial depressant
• Vagomimetic – causes atropine blockable bradycardia
• Sensitizes myocardium to circulating catecholamines
• Can cause arrhythmias / hypotension
• Hepatitis / Hepatic necrosis
• It is not hepatotoxic to children and its sweat odour makes it suitable for the agent of choice

ENFUIRANFT
• Metabolised to fluoride, excreted by kidney
• C/I in renal disease
• Can cause CNS excitation
• It potentiates skeletal muscle relaxation

ISOFLURANE
• Metabolized to toxic compounds
• Dose not sensitised the myocardium to circulating catecholamine’s
• Least toxic of all halogenated compounds

METHOXYFLURANE
• Most potent halogenated agent as it has highest lipid solubility
• Prolonged administration can cause renal damage by releasing Fluoride ions
• In obstetric anesthesia as it is devoid of any effect on uterus (No relaxation)

FLUORIDE ION RELEASE (NEPHROTOX1C POTENTIAL)


Meth > Hal > Enfl > Sevo > Iso = Des
Halothane produces hepatic damage by chlorotrifluoroethyl free radical
Ketamine : Dissociative Anesthesia (NMDA receptor block)
– Produce analgesia / sedation / Amnesia / Immobility
– Stimulates central sympathetic outflow (Tachycardia, ↑ BP)
Associated with post operative hallucinations

Nitrous Oxide : With a MAC value of 105%, nitrous oxide, by itself is not suitable or safe as a sole anesthetic agent.
Nitrous oxide is an effective analgesic.

Nitrous Oxide Advantages:


• Excellent analgesia
• Nonflammable
• Very rapid onset and recovery
• Little or no toxicity
• Use as an adjunct to the inhalational agents allows reduction in their dosage

165
3 Self Assessment & Review Phar macology
Pharmacology
Nitrous Oxide Disadvantages:
• No skeletal muscle relaxation
• Weak anesthetic
• Air pockets in closed spaces expand
• Post-anesthesia hypoxia (diffusion hypoxia)
• Not suitable as a sole anesthetic agent

Second Gas Effect:


• With Nitrous Oxide if second anesthetic gas is present, the rate of rise of arterial tension of the second gas is enhanced
also.
• If the first gas is nitrous oxide and the second enflurane, the concentration effect due to NO which pull more gas from the
breathing circuit into the lung, pulls both fresh NO and fresh enflurane.
• Thus the rate of rise of arterial tension of enflurane is faster as well.
• Diffusion hypoxia
• Reverse of the concentration effect: high rate of transfer of anesthetic from the blood and tissues to the alveoli.
• This additional gas dilutes alveolar oxygen and can result in postoperative hypoxia.
• This process is referred to as “diffusion hypoxia”• Lessened by administration of supplemental oxygen.

Propofol
• I.V. propofol rapidly induces anesthesia, similar to thiopental
• Duration of propofol anesthesia can be increased by repeated administration or by combination with inhalational agents,
nitrous oxide, or opioids.
• Propofol causes peripheral vasodilatation which leads to a decrease in blood pressure, but the agent is not arrhythmogenic
• Propofol does not adversely affect hepatic or renal function.
• Emergence from propofol anesthesia is rapid with minimal postoperative confusion.
• Postoperative Gl upset occur at a similar frequency to that of thiopental.
• Propofol is now commonly used in ambulatory surgery settings.

Opioid analgesic agents


• Opioid analgesics (Morphine and congeners) cause production of analgesia without loss of consciousness or production
of sleep. All effects of opioids are blocked by Nalaxone.
• Opioid agent include all natural, synthetic, semi synthetic or endogenous substances that interact with opioid receptors
• Major Substances from opium include - Morphine, codeine, thebaine, papaverine. Thebaine and papaverine have no
analgesic action per se. Thebaine is an important precursor of several semi synthetic opioid agents. Papaverine is a
smooth muscle relaxant and has no useful clinical application.

Endogenous opioid peptides


• Morphine acts on those areas of CNS that contains peptides which have opioid like properties which are named as
endogenous opioid peptides (previously known as endorphins). There are 3 classes of endogenous opioids i.e.
Enkephalins, β-endorphin and dynorphin.
• Principal precursors are Prepro-opiomelanecortin, which also serves as the precursor of ACTH and Melanocyte stimulating
Hormone (MSH), prepro dynorphin and prepro enkephalin
• Evidence suggests that the release of these endogenous opioids occurs during stress or anticipation of pain

Absorption : Absorption is rapid from most of the routes such as IM, Sc, oral and mucosal surface of nose, mouth and
rectum. From oral route some drugs undergo First-Pass metabolism. Some drugs like Fentanyl are well absorbed
transcutaneously.

Distribution : All opioids bind to plasma proteins with varying affinity, they rapidly leave the blood and localize in highly
perfused areas: such as brain, kidney, lungs, liver and spleen. Lipophilic drugs like Fentanyl which is also slowly metabolized
stored in fatty tissue.
Morphine does not penetrate the BBB as efficiently heroin or codeine. BBB is freely permeable to opioids and the BBB is not
so well developed in fetus, obstetric use of Morphine can cause CNS depression of neonate.

166
Phar macology of Central Nervous System
Pharmacology 3
Metabolism : Most opioids are converted to polar compounds following conjugation. Esters like Heroin and Remifentanyl
are hydrolyzed by plasma esterases to Morphine which is conjugated with glucuronic acid and excreted. Morphine-6-
glucuronide is also active and produces analgesia. Oxidative metabolism degrades drugs like Fentanyl, alfentanyl and
sufentanyl. Remifentanyl is degraded by non-specific plasma and tissue esterases. Morphine is metabolized to its 3-
glucuronide and 6-glucuronide forms. Morphine -6-glucuronide possesses analgesic action while M-3-G has the opposite.

Excretion : Major route of elimination of polar metabolites is urine. Glucuronide conjugated compounds are
excreted in bile.

Receptor Types : Major receptor types are μ (mu), κ (kappa) and δ (delta). Sub classes are also cloned μ1, μ2, κ1, κ2, κ3, and δ1
and δ2. All 3 types of these receptors are GPCR.

μ-Receptors Mediate : Analgesia, Euphoria, Respiratory depression and Physical dependence, Kappa and Delta receptors
also mediate analgesia.
Dysphoria is exclusively mediated by kappa receptors.

Pure agonists : Morphine and Morphine like drugs. Codejne and dextropropoxyphene are weak agonists (e.g. Morphine,
pethidine, codeine, fentanyl, sufentanyl).

Partial agonists : Nalorphine, buprenorphine

Agonists - Antagonist : agonist at one and antagonist at other. Pentazocine. cyclazocine antagonize at μ site and are
agonists at κ and δ receptors.

Antagonists : Naloxone, Naltrexone, Nalmephene, Diprenorphine (is the most potent of all pure antagonists).

Mechanism of action of OPIOIDS


All opioid receptors are GPCR and reduce cyclic, AMP. They cause opening of K + channels and inhibit opening
of Ca++ channels. This leads to reduced neuronal excitability (K+ channels opening leading to hyperpolarization) and reduced
neurotransmitter release (due to decreased calcium entry).
Certain excitatory actions that are produced are due to inhibition of inhibitory pathway. (Vomiting/pupillary constriction).

Pharmacological Actions
1. Analgesia: In many types of acute or chronic pain associated with tissue injury, tumor growth inflammation.
– Opioids are not so useful in Trigeminal neuralgia / post herpetic neuralgia
– Fantom limb pain
⎧ Spinal - Mediated by delta and Kappa receptors Analgesia
Analgesia ⎨
⎩ Supra spinal - Mediated by μ receptors
Part of antinociceptive action of opioids is due to release of endogenous opioids.

2. Euphoria : Reduces agitation and increases sense of well being. Euphoria is mediated by μ receptors and balanced by
κ receptors agonism.

3. Respiratory Depression : (μ μ effect). Increased arterial pCO2. It is better tolerated than other CNS depressants such as
barbiturates and alcohol. The cause of death in acute opioid overdose is due to Respiratory depression.

4. Suppression of Cough Reflex : MOA not clear. All opiates suppress cough. Codeine and pholecodeine suppress cough
at sub-analgesic doses. Constipation is an unwanted side effect.

5. Nausea and vomiting : (40% incidence). Opiates act on CTZ (apomorphine is used for producing vomiting).

6. Pupillary constriction : μ and κ receptor mediated effect. Pinpoint pupils are diagnostic of opiate intake.

7. GI Tract : Increases the tone of sphincters and reduces the motility of gut. These result in delayed gastric emptying and
constipation.
– Increased gall bladder contraction of sphincter of Oddi, Worsen biliary colic due to gallstones.
– Pethidine (Meperidine) causes mydriasis and does not worsen asthma or biliary colic.

167
3 Self Assessment & Review Phar macology
Pharmacology
8. Other actions : – Histamine release - Anaphylactoid reaction
– Itching at local injection site. Bronchoconstriction, hypotension can occur

9. Tolerance and dependence: They develop rapidly


Tolerance - Increased dose requirement to produce desired effect.
Dependence - can be physical - (associated withdrawal symptoms) or psychological - expressed craving for drugs.
Tolerance develops to - Analgesia
(Within 12-24 hrs) - Emesis, euphoria, dysphoria, mental clouding, sedation, respiratory, depression, cough
suppression
Tolerance does not develop to Constipation, Miosis, Convulsions and Antagonistic action.
Cross-tolerance occurs between opioids acting on same receptors.

10. Physical dependence: Marked by clear cut withdrawal syndrome that constitutes.
– Rhinorrhea
– Yawning
– Papillary dilation
– Piloerection
– Nausea, diarrhea, insomnia
– Fever

Mechanism of withdrawal syndrome


• Opiates suppress nor-adrenergic neurons of the locos ceruleus. Upon sudden withdrawal, increased Nor-adrenergic
activity results, which respond to opioids again, (clonidine blocks the excess Nor-adrenergic activity).

Uses
1. Analgesia : – Severe, constant pain is relieved better than sharp, intermittent pain
– MI, # long bones, Burns, 2nd degree, terminal cancer pain. Acute LVF

2. Acute Pulmonary edema : Postulated MOA (probably due to decreased sympathetic outflow from CNS).
– ↓ Pre load due to venodilatation
– ↓ After load due to arteriolar dilatation
– Euphoria and relief of anxiety
– ↓ Perception to shortness of breath

3. Cough suppression : codeine / pholcodeine

4. Diarrhea : loperamide, diphenoxylate, diphenoxin, (Racecadotril is an enkephalinase inhibitor, so increase enkephlin


antidiarrheal)

5. Adjunct to anesthesia : premedication for sedative, anxiolytic and analgesic properties.


– Because of their direct action on spinal cord they are used as regional analgesics, epidural or sub arachnoid routes.
Epidural route is preferred.
– Butorphanol is used intranasally (only)
– Fentanyl is adminsterd, lV, sc, subarchnoid, buccal (as lozenges) or trandermal patch.
Contra indications of Opioids :
1. Avoid using a full agonist with a partial agonist (pentazocine + morphine)
2. Head injuries : CO2 retention by opioids can cause cerebral vasodilatation in head injuries it can be fatal.
3. Pregnancy: opiates cause physical dependence in fetus produce withdrawal symptoms delivered to addict mothers
– Mild / manage with BZs
– Moderate to severe : tincture opium or methadone
4. Bronchial Asthma / COPD : acute respiratory failure.
5. Renal / hepatic impairment : In hepatic impairment metabolism is reduced and in renal impairment excretion is
reduced. Use only short acting drugs.
6. Endocrine disorders. Patients show exaggerated response to opioids in Addison’s disease and hypothyroidism.

168
Phar macology of Central Nervous System
Pharmacology 3
Drug interactions : All CNS depressants potentiate the respiratory depressant action of opioids.

Meptazinol and dezocine : Recently introduced PO, IV / IM, short t½ relatively free of respiratory depression / euphoria,
dysphoria, because of short duration short duration of action they may be used in obstetric analgesia.

NSAIDS
ACUTE INFLAMMATION (eg. Tissue injury)
Mediators of acute inflammation include histamine, 5-HT, bradykinin, prostaglandins, leukotrienes.
• Amongst all mediators of acute inflammation vasodilatation and increased vascular permeability is induced by all (i.e. H,
5-HT, BK and PCs) except LT’s.
• Chemotaxis is mediated by PC’s and LT’s and pain is mediate by BK and PGs.
• Oxygen free radicals are derived from activation of neutrophil membranes. Superoxide anion is produced by the reduction
of molecular oxygen. It stimulates the production of hydroxyl radicals and H2O2, which are also highly reactive, and they all
attack biological membranes, causing tissue damage (These free radicals are normally scavenged by SOD and catalase).

CHRONIC INFLAMMATION (e.g. Rheumatoid Arthritis)

Mediators Sources Major effects


IL 1, 2, 3 Macrophages, T- Lymphocytes Lymphocyte activation
PG production
GM - CSF T- Lymphocytes, Endothelial cells fibroblasts Macrophage and granulocyte activators
TNF - α Macrophages PG production
Interferons Macrophages, endothelial cells All of the effects
T- Lymphocytes
PDGG Macrophage, endothelial cells Fibroblast chemotaxis
Fibroblasts, Platelets Proliferation

NSAIDS primarily inhibit PG synthesis


Additional Mechanisms include:
• Inhibition chemotaxis
• Down regulation of lL-1 production
• Decreased production of superoxide
• Interference with calcium mediated intracellular events
Aspirin irreversibly activates and blocks platelet COX while other NSAlDs are reversible inhibitors.
Selective COX-2 inhibitors include Celecoxib, Rofecoxib, Parecoxib / pain is mediate by BK and PGs, COX-2 inhibitors have
no effect on platelet aggregation and cause less gastric irritation.

ALL NSAIDS
• Decrease the sensitivity of vessels to bradykinin and histamine
• Decrease lymphokine production from C-Lymphocytes
• Reverse vasodilatation

All NSAIDS are :


• Analgesic
• Antipyretic
• Anti inflammatory
• ↓↓ Platelet aggregation (except COX-2 inhibitors)

169
3 Self Assessment & Review Phar macology
Pharmacology
Aspirin :
P/K aspirin and Na salicylate are rapidly absorbed from the stomach and upper small gut. Peak 1-1.5 hours. In the plasma
aspirin is hydrolysed by esterases to acetic acid and salicylate.
Salicylate is bound to plasma albumin but this binding is saturable. So the unbound concentration increases at higher
doses (Saturation occurs above 3.6 g/day). Alkalinization of Urine increases salicylate excretion.
In addition to inhibiting PG synthesis, Aspirin also interferes with chemical mediation of kallikrein system causing:
• Inhibition of granulocyte adherence to damaged vasculature
• Stabilizes lysosomes
• Inhibits chemotaxis (migration of Polymorphs and macrophages)

Other effects:
• Aspirin is shown to decrease incidences of TIA, unstable angina, coronary artery thrombosis in post MI and thrombosis
after Angioplasty / CABG.
• Ingestion of 325 mg of Aspirin every alternate day decreases incidence of MI by 40%.
• Epidemiologic studies also suggest the ↓↓ incidence of colon cancer in low dose aspirin use.
– Gastric ulceration can be prevented by Misoprostol (PPIs like omeprazole are equally effected)

ADRS :
• Gastric upset. (Food, buffers antacids reduce it)
• GI bleed 0.5 ml to 3 ml blood loss, some adaptation occurs after 4 - 6 weeks
• Salicylism at high doses (vomiting, tinnitus, decreased hearing and vertigo)
• Metabolic acidosis with respiratory alkalosis
• Benefits in pregnancy are controversial (eclampsia)

OVERDOSE TOXICITY
• Gastric lavage
• Treat hyperthermia
• Correct acid base abnormalities
• Alkalinise urine to increase excretion of salicylate
• Maintain urine volume

DMARDS
1. Methotrexate : At anticancer doses it is a potent immunosuppressive. Its Amino Imidazole carboxamide Rihonucleotide
(AICAR), Transformylase and Thymidylate synthase plus enhanced adenosine release ↑ all cause ↓ proliferation of cells
at inflammatory site.
2. Cholorambucil : converted to phenylacetic acid mustard. Cross-links with DNA thus preventing cell proliferation.
3. Cyclophosphamide : Converted to Phosphoramide mustard. Action similar to chlorambucil. Also used to treat SLE.
4. Cyclosporine : Inhibits IL - 2 and TNF-α
ADR : Nephrotoxicity / hypertension / gingival hyperplasia / hyperkalemia / hirsutism
5. Azathioprine : Acts through its active metabolite 6TG Acid to suppress T and B cell proliferation. Also used in SLE and
Behcet’s syndrome
6. Chloroquine and Hydroxy chloroquine : They decrease the severity but not the progression of the disease, hydroxy
chloroquine.
– They suppress the responsiveness of T- lymphocytes to mitogens.
– Decrease leukocyte chemotaxis
– Stabilize lysosomal membranes
– Inhibit DNA and RNA synthesis
– Scavenge free radicals
Effects are seen after 12-24 weeks
They are used as adjuncts (OH-CHQ-SO4)
7. Gold :
– Exact MOA not Known
– Probably affects the morphology and junctional capacity of macrophages.
– Stabilization of lysosomal membrane

170
Phar macology of Central Nervous System
Pharmacology 3
– Pruritic dermatitis is the commonest ADR
– Eosinophilia, Thrombocytopenia
– Proteinuria / nephritic syndrome (Rare)
8. Penicillamine :
– It is a metabolite of penicillin and analog of amino acid cysteine
– Exact MOA not clear
– It may interfere with synthesis of DNA / collagen and mucopolysaccharides
– It is a reserved drug, in patients, who do not respond to gold
9. Sulfasalazine : suphapyridine + 5-ASA by bacteria in colon. It suppresses bone erosin. Absorbed from the colon.

ANTI TNF - α DRUGS


Cytokines play role in immune response. Wide ranges of cytokines are expressed in the joints of RA patients. TNF-α
appears to play the central role of inflammatory process. It is produced by Macrophages and activated T- lymphocytes. It
stimulates the release of other inflammatory-cytokines and proteases such as collagenasases and neutral metalloproteases.
– Infliximab and Adalimumab (TNF - α Monoclonal antibodies)
– Etanercept (TNF - α receptor blocker)

OTHERS :
• Leflunomide (Inhibits Dihydroorate dehydrogenase)
• Anakinra (IL-1 Receptor antagonist)
Paracetomol is the agent of choice of alcoholic hangovers.
But, Paracetamol can enhance hepatic injury more in alcoholics.

171
3 Self Assessment & Review Phar macology
Pharmacology

ALL INDIA a. Sodium valproate


b. Phenobarbitone
1. Ergotamine is contraindicated in all of the follow- c. Carbamazepine
ing except : [AI 08] d. Phenytoin
a. Vascular headache 9. Which of the following is the muscle relaxant of
b. Pregnancy choice in renal failure ? [AI 06]
c. Ischemic heart disease a. Rapacurium
d. Patient on erythromycin b. Pancuronium
2. Which of the following is not a sodium blocker ? c. Atracurium
a. Vigabatrin [Al 07] d. Rocuronium
b. Phenytoin 10. Which one of the following agents sensitises the
myocardium to catecholamines ? [AI 06]
c. Valproate
a. Isoflurane
d. Lamotrigine
b. Ether
3. Not an anti epileptic : [Al 07]
c. Halothane
a. Phenytoin
d. Propofol
b. Topiramate
11. All of the following are part of the treatment of
c. Flunarazine
Lithium toxicity, except : [AI 06]
d. Carbamazepine
a. Treating dehydration
4. Ethosuximide is used in the treatment of :
b. Ingestion of polystyrene sulfonate
a. Tonic-clonic seizure [AI 06]
c. Hemodialysis
b. Absence seizure
d. Using an antagonist
c. Myoclonic seizure
12. The most common side effect reported with treat-
d. Simple partial seizure ment with haloperidol is : [AI 06]
5. Which of the following statements is not true about a. Hypotension
etomidate? [AI 06]
b. Akathisia
a. It is an intravenous anesthetic
c. Dryness of mouth
b. It precipitates coronary insufficiency
d. Tic disorder
c. It inhibits cortisol synthesis
13. Rivastigmine and Donepezil are drugs used pre-
d. It causes pain at site of injection dominantly in the management of : [AI 06]
6. The following statements regarding bensodiaze- a. Depression
pines are true except : [AI 06]
b. Dissociation
a. Binds to both GABAA and GABAB receptors
c. Delusions
b. They have active metabolites
d. Dementia
c. Decreases nocturnal gastric secretion in human
14. All are side effects of Clozapine except : [AI 06]
being
a. Granulocytopenia
d. Extensively metabolized by CYP enzymes
b. Seizures
7. Which one of the following drugs does not inter-
fere with folic acid metabolism? [AI 06] c. Sedation
a. Phenytoin d. Extrapyramidal side effects
b. Gabapentin 15. Which of the following inhalational agents is the
induction agent of choice in children ? [AI 06]
c. Phenobarbitone
a. Methoxyflurane
d. Primidone
b. Sevoflurane
8. Concomitant administration of clonazepam with
which of the following antiepileptic drug can pre- c. Desflurane
cipitate absence status? [AI 06] d. Isoflurane

Answer 1. a. Vascular ... 2. a. Vigabatrin 3. c. Flunarazine 4. b. Absence ... 5. b. It precipitates ...


6. b. Streptomycin 7. a. Binds to both ... 8. a. Sodium ... 9. c. Atracurium 10. c. Halothane
11. d. Using an ... 12. b. Akathisia 13. d. Dementia 14. d. Extrapyra ... 15. b. Sevoflurane

172
Phar macology of Central Nervous System
Pharmacology 3
16. Which of the following anesthetic agents does not a. In Parkinsonism, phenothiazines reduce its effi-
trigger malignant hyperthermia? [AI 06] cacy [AI 04]
a. Halothane b. It is a prodrug
b. Isoflurane c. Pyridoxine reduces effects of levodopa in par-
c. Suxamethonium kinsonism
d. Thiopentone d. Domperidone blocks levodopa induced emesis
17. Which of the following intravenous induction and its therapeutic potential
agents is the most suitable for day care surgery? 24. Morphine can be used in all the following condi-
a. Morphine [AI 06] tions except : [AI 04]
b. Ketamine a. Head injury
c. Propofol b. Asthma
d. Diazepam c. Hypothyroidism
18. The following are the benzodiazepines of choice d. Diabetes
in elderly and those with liver disease, except : 25. Which of the following actions is ascribed to delta
a. Lorazepam [AI 06] type of opioid receptors ? [AI 04]
b. Oxazepam a. Supraspinal analgesic
c. Temazepam b. Respiratory depression
d. Diazepam c. Euphoria
19. Inverse agonist of benzodiazepine receptor is : d. Reduced intestinal motility
a. Phenobarbitone [AI 05] 26. Clinically significant drugs interaction occurs be-
b. Flumazenil tween pyridoxine and all the following drugs ex-
cept : [AI 04, 00]
c. Beta-carboline
a. Isoniazid
d. Gabapentin
b. Cyclosporine
20. Which of the following does not bind to GABA re-
ceptor chloride channels ? [AI 05] c. Levodopa
a. Ethanol d. Hydralazine
b. Alphaxolone 27. Granulocytopenia , gingival hyperplasia and facial
hirsutism are all possible side effects of one of
c. Zolpidem
the following anticonvulsant drugs : [AI 03]
d. Buspirone
a. Phenytoin
21. Oculogyric crisis is known to be produced by all
b. Valproate
of the following drugs except : [AI 05]
c. Carbamazepine
a. Trifluoperazine
d. Phenobarbitone
b. Atropine
28. Mechanism of action Tianeptin in the brain is :
c. Perchlorperazine
a. Selective serotonin reuptake inhibition [AI 02]
d. Perphenazine
b. Selective norepinephrine reuptake inhibition
22. In methyl alcohol poisoning there is CNS depres-
sion, cardiac depression and optic nerve atrophy. c. Selective serotonin reuptake enhancer
These effect are produces due to : d. Selective dopamine reuptake inhibition
a. Formaldehyde and formic acid 29. In which of the following disorders is administra-
b. Acetaldehyde [AI 05; AIIMS 04] tion of barbiturates contraindicated in :
c. Pyridine a. Anxiety disorders [AI 02]
d. Acetic Acid b. Acute intermittent porphyria
23. All the following statements regarding interactions c. Kernicterus
of levodopa are correct except : d. Refractive status epilepticus

Answer 16. d. Thiopentone 17. c. Propofol 18. a. Lorazepam 19. c. Beta- ... 20. d. Buspirone
21. b. Atropine 22. a. Formaldehyde ... 23. d. Domperidone ... 24. a. Head injury 25. a. Supraspinal ...
26. b. Cyclosporine 27. a. Phenytoin 28. c. Selective ... 29. b. Acute ...

173
3 Self Assessment & Review Phar macology
Pharmacology
30. Which of the following drugs would be removed 38. The most common side effect associated with
by dialysis ? [AI 01] chronic use of phenothiazines is : [AI 98]
a. Digoxin a. Akathesia
b. Salicylates b. Parkinsonism
c. Benzodiazepines c. Tardive dyskinesia
d. Organophosphates d. Muscular dystonia
31. False statement about selegeline is : [AI 01] 39. All of the following may be seen with Neuroleptic
a. It is a MAO-A inhibitor malignant syndrome except : [AI 98]
b. Does not cause cheese reaction a. Hypothermia
c. May be used in on-off phenomenon b. Altered consciousness
d. It is used in parkinsonism c. Muscle rigidity
32. A patient on phenytoin for treatment of seizures d. Involuntary movements
develops depression for which he is prescribed 40. Tetrahydrocannabinol is the active component of:
tricyclics. He now complains of lassitude and his a. Marijuana [AI 98]
Hb reads 8 ; next step in managing this patient b. LSD
should be : [AI 01] c. Hashish
a. Chest X-ray d. Heroin
b. MCV should be estimated 41. Which of the following antiepileptic drugs acts by
c. GGT should be estimated the release of the inhibiting transmitter GABA:
d. None of the above a. Valproic acid [AI 97]
33. A patient of parkinsonism is managed with L-dopa. b. Diazepam
If Vit. B-complex is administered concurrently : c. Ethambutol
a. The action of L-dopa in brain will be potentiated d. Phenytoin
b. Decarboxylation of L-dopa in brain will be de- 42. Which of the following drugs acts on μ receptor of
creased [AI 00] the CNS : [AI 95]
c. Side effects will be ameliorated a. Morphine
d. Decreased efficacy will result b. Buprenorphine
34. Drug of choice for Epilepsy in pregnancy is : c. Pethidine
a. Carbamazepine [AI 00] d. Pentazocin
b. Sodium valproate 43. Which of the following is false about pentazocine:
c. Phenobarbitone a. Decreased vomiting and constipation as com-
d. Phenytoin pared to morphine [AI 95]
35. Vigabatrin, a new antiepileptic agent acts by : b. Risk of addition is less than that with morphine
a. GABA - antagonism [AI 99] c. Risk of addiction is more than that with morphine
b. GABA - agonism d. It is agonist antagonist
c. NMDA antagonism
d. DA agonism AIIMS
36. Selegilline is a selective inhibitor of : [AI 99]
a. MAO-A 44. Benzodiazepine antagonist ? [AIIMS May 08]
b. MAO-B a. Flumazenil
c. Dopamine b. Beta carboline
d. Norepinephrine-uptake c. Flunarazine
37. Flumazenil is a : [AI 98] d. Diazepam
a. Benzodiazepine antagonist 45. Which statement is true about carbamazepine ?
b. Benzodiazepine agonist a. Used in trigeminal neuralgia [AIIMS May 08]
c. Adrenergic blocking agent b. Carbamazepine is an enzyme inhibitor
d. Opioid antagonist

Answer 30. b. Salicylates 31. a. It is a ... 32. b. MCV ... 33. d. Decreased ... 34. c. Phenobarbitone
35. b. GABA - ... 36. b. MAO-B 37. a. Benzodia ... 38. c. Tardive ... 39. a. Hypothermia
40. a. Marijuana 41. a. Valproic ... 42. a. Morphine 43. c. Risk of ... 44. a. Flumazenil
45. a. Used in ....

174
Phar macology of Central Nervous System
Pharmacology 3
46. Drug used in uncomplicated alcohol withdrawal? 54. The μ receptor of the opioids is responsible for
a. Diazepam [AIIMS May 08] the following clinical actions except :
b. Clonidine a. Analgesia [AIIMS Nov. 05]
c. Propanalol b. Respiratory depressions
d. Methadone c. Sedation
47. Buprenorphine is : [AIIMS May 08] d. Diuresis
a. Pure agonist 55. Which of the following is not an opioid peptide?
b. Pure antagonist a. β – Endorphin [AIIMS 05]
c. Partial agonist b. Epinephrine
d. None c. Leu 5-enkephalins
48. All but one acts via GABAA except : d. Met5-enkephalin
a. Thiopentone [AIIMS Nov. 07] 56. The following statements are true for therapy with
Lithium except : [AIIMS 04]
b. Midazolam
a. It is used in bipolar disorder
c. Zolpidem
b. Thiazides are useful in treating Lithium induced
d. Promethazine
diabetes insipidus
49. Regarding phenytoin, false is : [AIIMS Nov. 07]
c. Regular measurements of blood concentration
a. Induces microsomal enzymes of Lithium is necessary
b. At very low doses, zero order kinetics occurs d. Na+ is a specific antidote for lithium intoxication
c. Higher the dose, higher is the half life 57. An unconscious man is brought into hospital suf-
d. Highly protein bound fering from methyl alcohol poisoning. All of the
50. True about Benzodiazepines as compared to other following are correct except : [AIIMS 04]
hypnotics : [AIIMS Nov. 07] a. Kussmaul’s breathing could be expected to be
a. They alter sleep pattern more than other a feature of the condition
hypnotics b. Papilloedema would be consistent with this form
b. More sedative than other hypnotics of intoxication
c. Overdose is better tolerated compared to other c. His plasma bicarbonate might be very low
hypnotics d. Methyl alcohol would be metabolised to acetal-
d. All of the above dehyde
51. Benzodiazepine antagonist : [AlIMS Nov. 06] 58. Which one of the opioids has maximum plasma
a. Flumazenil protein binding capacity ? [AIIMS 04]
b. Fentanyl a. Morphine
c. Fomepizole b. Sufentanil
d. Flurazepam c. Fentanyl
52. Prolonged use of one of the following anti convul- d. Pethidine
sant can produce weight loss : 59. Ropinirole is the most useful for the treatment of:
a. Gabapentin [AlIMS May 06] a. Parkinson’s disease [AIIMS 04]
b. Oxcarbazepine b. Wilson’s disease
c. Topiramate c. Hoffman’s syndrome
d. Valproic acid d. Carpal tunnel syndrome
53. Which of the following antiepileptic agents act on 60. A 30 year old manic patient was prescribed halo-
the GABA ergic system to decrease the uptake of peridol one week back. For last two days he has
GABA into nervous and glial cells : become restless and kept pacing in the room for
a day. On examination he was found to have trem-
a. Vigabatrin [AIIMS Nov. 05]
ors of hand. He is most likely suffering from:
b. Progabide
a. Anhedonia [AIIMS 03]
c. Gabapentin
b. Dystonia
d. Tiagabine
c. Restless leg syndrome
d. Akathesia

Answer 46. a. Diazepam 47. c. Partial... 48. d. Promethazine ... 49. b. At very 50. c. Overdose ...
51. a. Flumazenil 52. c. Topiramate 53. d. Tiagabine 54. d. Diuresis 55. b. Epinephrine
56. d > b 57. d. Methyl ... 58. b. Sufentanil 59. a. Parkinson’s ... 60. d. Akathesia

175
3 Self Assessment & Review Phar macology
Pharmacology
61. A patient with recent-onset primary generalized 67. A female suffering from psychosis, taking phe-
epilepsy develops drugs reaction and skin rash nothiazines now complains of sudden onset of high
due to phenytoin sodium. The most appropriate grade fever, muscle rigidity and altered sensorium.
course of action is : [AIIMS 03] The diagnosis is : [AIIMS 01]
a. Shift to clonazepam a. Malignant hyperthermia
b. Restart phenytoin sodium after 2 weeks b. Neuroleptic malignant syndrome
c. Shift to sodium valporate c. Tardive dyskinesia
d. Shift to ethosuximide d. Akathesia
62. Adverse effect of phenytoin include the follow- 68. Lithium is used in a pregnant woman. Which of
ing except : [AIIMS 02] the following congenital anomaly occurs in foe-
a. Lymphadenopathy tus: [AIIMS 01]
b. Ataxia a. Tetralogy of Fallot’s
c. Hypercalcemia b. Tricuspid atresia
d. Hirsutism c. Ebstein anomaly
63. Which one of the following statements is true d. Pulmonary stenosis
regarding drugs used in treatment of parkin- 69. A new born developed cleft lip and cleft palate and
sonism ? [AIIMS 02] atrial septal defect. Which of the following drug is
a. Amantadine causes ankle oedema likely to be consumed by the mother so as to cause
b. Levodopa is particularly effective in reducing such type of congenital anomalies :
tremors a. Digoxin [AIIMS 01]
c. Amantadine is more effective then levodopa b. Methanol
d. Antimuscarinics are effective in drug induced c. ACE inhibitors
parkinsonism d. Isotretinoin
64. A patient ingested some unknown substance 70. In a chronic alcoholic patient all of the following
and presented with myoclonic jerks seizures, drugs should be avoided except : [AIIMS 01]
tachycardia and hypotension. The ECG showed a. Cefamondole
a heart rate of 120/minute with QRS interval b. Metronidazole
of 0.16 seconds. The atrial blood revealed a
c. Chlorpropamide
ph of 7.25, PCO2 of 30 mm Hg and HCO3 of
15mmol/L. the most likely cause of poisoning d. Beclomethasone
is ingestion of : [AIIMS 02] 71. What is the treatment of choice for myoclonic
a. Amantia phalloides epilepsy in children : [AIIMS 00]
b. Ethylene glycol a. Phenytoin
c. Imipramine b. Phenobarbitone
d. Phencyclidine c. Sodium valproate
65. The effective and safe drug for intractable pain in d. Ethosuximide
terminal cancer stage is : [AIIMS 02] 72. Which of the following is TRUE for morphine :
a. Injectable pethidine a. Tolerance to all its actions except sedation and
b. Injectable ketamine euphoria [AIIMS 00]
c. Oral Brufen b. Tolerance to all its actions except constipation
and miosis
d. Oral Morphine
c. It is a peripherally acting analgesic
66. The drug NOT used for analgesia in a head injury
patient is : [AIIMS 02] d. It has no abuse potential
a. Morphine 73. Alcohol intake during pregnancy causes, all, EX-
CEPT : [AIIMS 99]
b. NSAIDs
a. Brachycephaly
c. Rofecoxib
b. Microcephaly
d. Acetaminophen
c. Hyperkinetic movements
d. Congenital anomalies

Answer 61. c. Shift to ... 62. c. Hypercalcemia 63. a. Amantadine ... 64. c. Imipramine 65. d. Oral ...
66. a. Morphine 67. b. Neuroleptic ... 68. c. Ebstein ... 69. d. Isotretinoin 70. d. Beclometha ...
71. c. Sodium ... 72. b. Tolerance to ... 73. a. Brachycephaly

176
Phar macology of Central Nervous System
Pharmacology 3
74. A child complains of nocturnal enuresis. The drug PGI
given to treat this is : [AIIMS 99]
a. Imipramine 82. Which of the following match is / are correct :
b. Clomipramine a. Gabapenten-GABA transminase inhibitor
c. Fluoxetine b. Zonisamide-Inhibition of T type Ca++ current
d. Haloperidol c. Carbamazepine - Na+ channel blocker
75. An idiopathic epileptic patient is on phenytoin and d. Lamotrigine - Na+ channel blocker
tricyclic antidepressant. He becomes easily fa-
e. Valproate - K+ channel blocker [PGI Dec. 07]
tigue and lethargy. His Hb 8.2 gm%, TLC 9600 cmm.
Serum bilirubin 0.6, SGPT 25 IU/lit. The next inves- 83. Risperidone asso. with ↑ es the risk of :
tigation done is : [AIIMS 99] a. Cerebrovascular accidents [PGI Dec. 06]
a. MCV b. ↑ EPS clinical features
b. Gamma glutamyl transferase assay c. Agranulocytosis
c. Chest X-ray d. Diabetes insipidus
d. Urine culture e. Gout
76. Carbamazepine is NOT used in : [AIIMS 98] 84. Therapeutic level of phenytoin : [PGI Dec. 05]
a. Mania a. 0.9 μg/ml
b. Partial seizure b. 10 - 29 μg/ml
c. Trigeminal neuralgia c. 20 - 29 μg/ml
d. Migraine d. 30 - 39 μg/ml
77. Long term use of lithium causes : [AIIMS 98] e. 40+ μg/ml
a. Peripheral neuropathy 85. Regarding Zolpidem which of the following is not
b. Hypothyroidism true : [PGI Dec. 05]
c. Anaemia a. Act on benzodiazipine receptor type 1
d. Jaundice b. Action not reversed by flumazenil
78. Antidepressant, which is selective 5HT inhibi- c. Duration of action is less than diazepam
tor is : [AIIMS 97] d. Has sedative and hypnotic effect
a. Fluoxetine e. Good muscle relaxation
b. Imipramine 86. Antidote for Ethylene glycol poisoning :
c. Desipramine a. Methyl violet [PGI June 04]
d. Amitryptiline b. Fluconazole
79. Drug causing least physical dependence is : c. Fomepizole
a. Alprazolam [AIIMS 97] d. Ethyl alcohol
b. Fluoxetine 87. Drugs used in GTCS : [PGI June 04]
c. Dextropropoxyphene a. Ethosuximide
d. Pentazocine b. Sodium valproate
80. Extra pyramidal syndrome like side effect are c. Lamotrigine
seen in : [AIIMS 95] d. Propofol
a. Haloperidol 88. Methyl alcohol poisoning; true about :
b. Clozapine a. Ethyl alcohol is used [PGI June 04]
c. Tetracycline b. Formation of formic acid produces blindness
d. Ketoconazole c. Activated charcoal is given in all cases
81. Opioid agonist antagonist is : [AIIMS 95] d. Gastric lavage done
a. Pethidine e. Fomeprizole inhibits the formation of formic acid
b. Pentazocine
c. Buprenorphine
d. Methadone

Answer 74. a. Imipramine 75. a. MCV 76. d. Migraine 77. b. Hypothyroidism 78. a. Fluoxetine
79. b. Fluoxetine 80. a. Haloperidol 81. b. Pentazocine 82. c and d 83. b. ↑ EPS ...
84. b. 10 - 29 ... 85. b and e 86. c and d 87. b and c 88. a, b, d and e

177
3 Self Assessment & Review Phar macology
Pharmacology
89. Neurochemical mechanism of analgesia : c. Sodium valporate
a. VR - 1 [PGI Dec. 03] d. Midazolam
b. Nicotinic cholinergic e. Pethidine
c. Nocistatin pattern 97. True regarding bromocriptine : [PGI June 02]
d. Nociceptin pattern a. Natural derivative
e. Anandomide b. Synthetic derivative
90. Side effects of phenytoin are : [PGI Dec. 03] c. Has alfa - blocking action
a. Gum hypertrophy d. Decrease GI motility
b. Alopecia e. Acts on both D1 and D2 receptors
c. Subungual exostosis 98. Drugs causing parkinsonism include :
d. Onycholysis a. Bromocriptine [PGI Dec. 01]
e. Acne rosacea b. Phenothiazine
91. True about Dezocine : [PGI June 03] c. Haloperidol
a. Slow acting morphine d. Amantadine
b. Low potent morphine e. Carbidopa
c. Acts via GABA 99. Which is true regarding naltrexone :
d. Doesn’t increase histamine release a. It is an opioid antagonist [PGI Dec. 01]
e. ↑ ed plasma catecholamines b. It is an opioid agonist
92. SSRI are : [PGI June 03] c. Used in alcohol dependence
a. Citalopram d. Used to treat opioid dependence
b. Fluoxetine e. Used as a respiratory stimulant
c. Mirtazapine 100. Which is true regarding benzodiazepines :
d. Imipramine a. GABA agonist [PGI Dec. 01, Dec. 00]
e. Sertraline b. Diazepam is a short acting benzodiazepine
93. Treatment of Parkinson’s disease : c. Diazepam causes lesser respiratory depression
a. Levodopa [PGI June 03] d. Nitrazepam is metabolized in liver
b. Mazindol e. Diazepam has higher abuse potential than
c. Bromocryptine Midazolam
d. Acyclovir 101. Selective Serotonin Reuptake Inhibitor (SSRI)
e. Benserazide are : [PGI June 01]
94. Hepatotoxic drugs are : [PGI June 03] a. Fluphenazine
a. Chloroform b. Fluoxetine
b. Ether (diethyl) c. Fluvoxamine
c. N2O d. Sertraline
d. Halothane e. Fluphenthixol
e. Enflurane 102. Drugs producing antinuclear antibodies (ANA) :
95. Morphine cannot given in which of the following a. Phenytoin [PGI Dec. 00]
route : [PGI Dec. 02, June 02] b. Penicillin
a. IM c. INH
b. Transdermal d. Procainamide
c. Epidural 103. Drug used to prevent from indulging in alcoholism
d. Subarachnoid in one who has abstinence from alcohol :
e. Oral a. Clonidine [PGI Dec. 00]
96. In porphyria, drugs safe are : [PGI Dec. 02] b. Disulfiram
a. Barbiturate c. Chlordiazepoxide
b. Ketamine d. Naltrexone
e. Acamprosate

Answer 89. a, b, d and e 90. a. Gum ... 91. d. Doesn’t ... 92. a, b and e 93. a, c and e
94. a and d 93. None 94. d and e 95. b, c, d and e 96. b and c
97. a, c, d and e 98. c and d 99. b, c and d 100. b. Flumazenil 101. None
102. c and d 103. b, d and e

178
Phar macology of Central Nervous System
Pharmacology 3
104. Cheese reaction with MAO inhibitors is due to : 108. Barbuiturate is metabolized by : [PGI Dec. 97]
a. Guanethidine [PGI Dec. 99] a. Glucuronide conjugation
b. Resperpine b. Oxidation, dealkylation
c. Cough remedies c. Acetylation
d. Tyramine d. All of the above
105. Mechanism of action of GABA is on : 109. The mechanism of action of barbiturate on cere-
a. G protein [PGI Dec. 99] brum is : [PGI June 97]
b. Tyrosine kinase a. Scavenger of radicals
c. PIP/DAG b. Increase BMR in active area of brain
d. 5 - HT c. Acts on Cl- channel complex
106. Short ½ life of thiopentone is due to : d. Decrease intracranial pressure
a. Rapid excretion [PGI June. 99] 110. Alkalinity of urine is done in : [PGI June 97]
b. Rapid redistribution a. Barbiturate poisoning
c. Rapid metabolism b. Lithium toxicity
d. All of the above c. Alprazolam overdose
107. Ketanserin is : [PGI June 98] d. Diazepam toxicity
a. 5 HT2 reuptake blocker
b. Antipsychotic
c. Useful in movement disorder
d. Anti tussive

Answer 104. d. Tyramine 105. a and c 106. b. Rapid ...


107. a. 5HT2... 108. a and b 109. c. Acts on ...
110. a. Barbiturate ...

179
3 Self Assessment & Review Phar macology
Pharmacology

ANSWERS, REFERENCES, EXPLANATIONS WITH INFORMATIVE ILLUSTR ATIONS

1. Ans. is a i.e. Vascular headache Ref. Dollery 3/e, p E47; KDT 6/e, p 322

Ergotamine is a vasoconstrictor used for treatment of vascular headaches of the migraine type.
Vasoconstrictor effect is probably mediated by an interaction with serotonergic receptors for which ergotamine
is an partial agonist.
• Apart from being a parital agonist at serotonergic receptors, ergotamine is also a partial agonist at α-
adrenoceptors contributing to its vasoconstrictor effect.
• Uterotonic effect also due to serotonergic receptors.
• Emetic effect is possibly mediated through stimulation of dopaminergic receptors in the CTZ.

Contraindication :
• HT • Vascular disease
• Hepatic and renal disease • Septic condition and cachexia
• Chronic pulmonary disease • Valvular heart disease
• Collagen disease and fibrotic condition • During pregnancy and before
3rd stage of labour

2. Ans. is a i.e. Vigabatrin Ref. KDT 6/e, p 404

Phenytoin, Valproate, Carbamazepjne, Lamotrigine, Zonisamide affect membrane excitability by blocking inward
Na+ current necessary to generate an action potential. The block shows the property of use dependence i.e
those neurons are affected most who are repeatedly firing.

Drugs causing Sodium channel inactivation : Mnemonic: CP Live Via Zones :


C : Carbamazepine P : Phenytoin
L : Lamotrigine V : Valproate (Remember its not Vigabatrin)
Z : Zonisamide

Mnemonic : VI GABA TR IN
• VIgabaterine • GABA
• TRansaminase • INhibitor

Vigabatrin is an inhibitor of GABA-transaminase that degrades GABA: anticonvulsant action may be due
to increase in synaptic GABA concentration.

Mechanism of Action of Different Antiepileptic Drugs are given below.


Prolongation of Na+ Facilitation of GABA mediated Inhibition of ‘T’ type
channel inactivation Cl–– channel opening Ca2+ current
Valproate Valproate Valproate
Phenytoin Barbiturate Ethosuximide
Carbamazepine Benzodiazepine Trimethadione
Lamotrigine Gabapentin
Zonisamide

180
Phar macology of Central Nervous System
Pharmacology 3
3. Ans. is c i.e. Flunarazine Ref. Harrison 17/e, p 2507; KDT 6/e, p 411

Flunarazine is a calcium channel blocker.


Antiepileptic drugs used in various types of seizures are given below.
Primary Generalized Atypical Absence,
Tonic-Clonic Partial Absence Myoclonic, Atonic
First - Line
Valproic acid Carbamazepine Valproic acid Valproci acid
Lamotrigine Phenytoin Ethosuximide Lamotrigine
Topiramate Lamotrigine Topiramate
Oxcarbazepine
Valproic acid

Alternatives
Zonisamide Levetiracetam Lamotrigine Clonazepam
Phenytoin Topiramate Clonazepam Felbamate
Carbamazepine Tiagabine
Oxcarbazepine Zonisamide
Phenobarbital Gabapentin
Primdione Phenobarbital
Felbamate Primiddone
Felbamate

4. Ans. is b i.e. Absence seizure Ref. Goodman & Gilman 11/e, p p 502 - 514; KDT 6/e, p 406 - 407

Ethosuximide is effective against absence seizures in patient with generalized epilepsy but It does not
protect against bilateral tonic-clonic seizure that may occurs in patient with absence seizure.
So, it can be used in combination with other anticonvulsant effective against tonic-clonic seizures e.g.,
carbamazepine, phenytoin, phenobarbitone.

Mechanism of action : Ethosuximide reduces low threshold Ca++ currents (T currents) in thalamic neurons.
The thalamus plays an important role in generation of 3-Hz spike and wave rhythms typical of absence seizures.

5. Ans. is b i.e. It precipitates coronary in sufficiency Ref. Goodman & Gilman 11/e, p 351

Etomidate is primarily used for anesthetic induction of patients at risk for hypotension.
• CVS – Cardiovascular stability after induction is a major advantage of etomidate over either barbiturates
or propofol. Induction with etomidate typically produces a small increase in heart rate and little or no decrease
in blood pressure or cardiac output.
• Induction have a rapid onset and short duration of action and accompanied by a high incidence of pain on
injection and myoclonic movements as a proconvulsant. (Option ‘d’)
• Etomidate is suitable for infusion for anesthetic maintenance or sedation.

Drawbacks : – Associated with Nausea and vomiting


– Inhibits adrenal biosynthesis enzymes (Adrenal insufficiency). (Option ‘c’)

181
3 Self Assessment & Review Phar macology
Pharmacology

Indication of etomidate : • Induction of anesthesia in patient with cardiovascular disease.


• Induction of anesthesia in patient with known drug hypersensitivity.
Contraindication : • Impaired adrenocortical function
• Epilepsy
• Porphyria
Mechanism of action : • CNS depression by enhancing GABA
• No, analgesic effect
• Inhibit mitochondrial hydroxylation 11β and 17α hydroxylase activity. This result
in reduced cortical synthesis.
Adverse effect : • Pain on injection site
• Nausea and vomiting
• Involuntary muscle movements.

6. Ans. is a i.e. Binds to both GABAA & GABAB receptors


Ref. Goodman & Gilman 11/e, p 405, 407 - 408; KDT 6/e, p 394

• Benzodiazepines acts at GABAA but not GABAB receptors.


• Diazepam markedly decreases nocturnal gastric secretion in human beings. Other agents are considerably
more effective in acid-peptic disorders.
• The BZD are metabolized extensively by cytochrome P450 enzymes, particularly CYP3A4 and CYP2C19.
Erythromycin, clarithromycin, ritonavir, itraconazole, ketoconazole, nefazodone and grapefruit juice are
inhibitors of CYP3A4 and can affect the metabolism of BDZ.
Most of BZD metabolized to produce acitve metabolite and have longer t½.

Uses of diazepam : – Management of anxiety – Acute alcohol withdrawal


– Basal sedation – Adjunct for the relief of skeletal muscle spasm
– Excitation states – Treatment of status epilepticus
– Premedication

The BZD do not produced active metabolite are :


O Oxazepam
L Lorazepam
T Tamezepam, Triazolam
M Midazolam

7. Ans. is b i.e. Gabapantine Ref. Harrison 17/e, p 650; CMDT 05, p 471

Folic acid deficiency is due to


Inhibitor of folate absorption Inhibitors of dihydrofolate reductase
• Drugs : • Methotrexate
– phenytoin • Pyrimethamine
– phenobarbitone • Triamterene
– Primidone • Pentamidine
– sulfasalazine • Alcohol
• Tropical sprue • Trimethoprim

182
Phar macology of Central Nervous System
Pharmacology 3
8. Ans. is a i.e. Sodium valproate
Ref. KDT 6/e, p 407; Goodman & Gilman 11/e, p 515

Important intractions of valproate are :


• Concurrent administration of clonazepam and valproate is contraindicated because absence status
may be precipitated.
• Valproate increases plasma levels of phenobarbitone by inhibiting its metabolism.
• It displaces phenytoin from protein binding site and decreases its metabolism →phenytoin toxicity.
• Valproate and carbamazepine induce each other’s metabolism.
• Foetal abnormalities are more common if valproate and carbamazepine are given concurrently

Valproate - Important uses :


• Primary generalised seizure (PGS)
• Partial seizure
• Prophylaxis of febrile seizure
• Prophylaxis of post traumatic epilepsy
• Status epilepticus

Contraindication in liver disease and pregnancy.

Adverse effects : • Liver damage : Centrilobular necorsis, fatty changes and cholestasis.
• Hyperammonemia
• Pancreatitis
• GIT : Nausea, vomiting, diarrhoea, anorexia, dyspepsia
• Weight gain
• Skin rashes
• Menstrual disturbances
• Neurological : Tremors and sedation

9. Ans. is c i.e. Atracurium Ref. Goodman & Gilman 11/e, p 222; KDT 6/e, p 345

The unique feature of atracurium is inactivation in plasma by spontaneous nonenzymatic degradation (Hoffmann
elimination) in addition to that by cholinesterases. Therefore its duration of action is not altered with hepatic and
renal insufficiency or hypodynamic circulation. Haemodynamically it is almost neutral.
Agent Mode of elimination
Atracurium Hofmann degradation; hydrolysis by plsma esterases
Succinylcholine Hydrolysis by plasma cholinesterases shortest acting depolarising
D-Tubucurarine Renal elimination; liver clearance
Doxacurium Renal elimination long acting nondepolarising
Mivacurium Hydrolysis by plasma cholinesterases shortest acting nondepolarising
Pancuronium Renal elimination
Pipecuronium Renal elimination; liver metabolism and clearance
Rocuronium Liver metabolism
Vecuronium Liver metabolism and clearance; renal elimination

183
3 Self Assessment & Review Phar macology
Pharmacology
10. Ans. is c i.e. Halothane Ref. KDT 6/e, p 372

Inhalational anaesthetic agents that sensitises the myocardium to catecholamines are :


• Halothane • Trylene • Cyclopropane
• Methoxyflurane • CCL4

11. Ans. is d i.e. Using an antagonist Ref. KDT 6/e, p 435

Treatment in case of intoxication is symptomatic. There is no specific antidote. Osmotic diuretics and
sodium bicarbonate infusion promote Li excretion.

Haemodialysis is indicated if serum levels are >4mEq/L.


Margin of safety is narrow, monitoring serum Li concentration is essential for optimal therapy. Serum Li level is
measured 12 hours after the last dose to reflect the steady-state concentration; 0.5-0.8mEq/L is considered
optimum for maintenance therapy in bipolar disorder while 0.8-1.1mEq/L is required for episode of mania
Lithium (Li) poisoning should be assessed on the basis of symptoms and treatment should be stoped
immediately. The clinical symptoms of intoxication often only manifest themselves after hours to days.
A vicious cycle may also be initiated by temporarily reduced renal function due to kidney disorders, dehydration
due to fever, vomiting, diarrhoea, salt poor diet and heavy sweating.
Most often shows CNS symptoms.

Treatment of Lithium intoxication


• Hemodialysis until Li can < 1.0 mmol.
• IV sodium loading and forced diuresis suggested but this is not sufficiently and may evoke dangerous
hypernatremia.
• In mild poisoning - Correction of water and electrolyte balance is usually all that is required.
Toxicity symptoms occur frequently when serum levels exceed 1.5mEq/L.

Uses : • Prophylaxis in bipolar disorder


• Acute manic episode
• Cancer chemotherapy induced leukopenia and agranulocytosis
• Inappropriate ADH secretion syndrome. ... KDT 5/e, p 536

Amiloride blocks entry of Li through Na+ channels in CD (Collecting duct) cells and mitigates diabetes
insipidus induced by lithium.

12. Ans. is b i.e. Akathisia Ref. KDT 6/e, p 431

Adverse effects of Haloperidol


EPS occurred with the following frequencies:
• Tremor 39%
• Bradykinesia 48%
• Akathisia 32% (treatment small dose of PPNL)

184
Phar macology of Central Nervous System
Pharmacology 3
Tardive dyskinesia : never acute occur after 6 weeks of treatment, incidence 15%. Present as an orofacial
choreiform movement disorder but can be generalized. Female and the elderly are particularly vulnerable.
Treatment is anticholinergics.
It is characterised by uncontrollable motor restlessness without any involvement of anxiety. Mechanism is not
clear. The use of a nonselective β-blocker such as propranolol or drugs like diphenhydramine can provide
relief.

Neurological Side Effects : Six type of neurological side effects are characteristic of classical antipsychotic
drugs. Four of these (dystonias, akathisia, parkinsonism and a rare neurolept-malignant syndrome) usually
appear early during treatment while the other two (tardive dyskinesia and a rare peri-oral tremors) appear later
following prolonged treatment for more than a year.
Adverse effect of Antipsychotic drug

Dose Related Hypersensitivity Reaction


↓ ↓ ↓
Cholestatic Dermato Agranulo
Jaundice logical cyctosis

CNS α - Adr Anti Extra Endocrine Other


Blockade cholinergic Pyramidal logical
disturbances
• Appetite↑ • Postural- • Dry mouth • Amenorrhoea • Blue
• Weight hypotension • Constipation • Infertility Pigmentation
gain • Palpitation • Urinary- • Galactorrhoea of skin
• Ejaculation hesitancy • Corneal and
defect Lenticular
Opacities
• Retinal
degeneration

Perioral movement Parkinsonism No Perioral movement


It develop after
Acute muscular Tardive 1-4 weeks. Akathesia Malignant neuroleptic
dystonia dyskinesia of therapy • 1-8 weeks syndrome
• With in few hour • Late after 4 Treatment • Restlessness, Rarely with
of single dose month even – Central Anti with internal high doses of
• Mostly in Ist wk. after with drawl cholinergic discomfort, potent agent
• More common in of drug complete • Marked rigidity
children below • Mostly in desire to move • Immobility tremor
10 yrs. and in girls elderly woman about both • Fever
• Bizarre muscle • Purposeless without anxiety • Semi conscious
spasm, mostly involuntary Treatment ness
involve linguo facial & limb – Propranolol • Fluctuating BP, HR.
facial lead to : movement • Myoglobin
– Grimacing – Chewing present in blood
– Locked Jaws – Pouting Treatment
– Oculogyric crisis – uffing of – Bromocriptine
Treatment cheeks – Stop neuroleptic
– Anticholinergic – Choreoathetoid – Dantrolene
– Promethazine movement
– Hydroxyzine Im – Lip licking

185
3 Self Assessment & Review Phar macology
Pharmacology
13. Ans. is d i.e. Dementia Ref. Harrision 17/e, p 2542; KDT 6/e, p 99, 472

Rivastigmine, Tacrine, Donepezil, Galantamine are cholinergic activator.


Since brain ACh levels are markedly reduced and cholinergic neurotransmitter is the major suffer in Alzheimer’s
disease.
Tacrine, donepezil, rivastigmine and galantamine are approved by the US. Food and drug administration. for
treatment of Alzheimer’s disease. Their pharmacologic action is presumed to be inhibition of cholinesterase
with a resulting increase in carebral levels of acetylcholine and improve dementia.

Cognition enhancers list is given here.


Nootropic Metabolic enhancer Cholinergic activators Vasoactive cerebral protector
• Piracetam • Dihydrpergotoxine • Tacrine • Pvritinol
• Nicergoline • Rivastigmine • Ginkgo biloba
• Piribedil • Donepezil
• Galantamine

• Tacrine is expensive and may cause hepatotoxicity, thus it is rarely used.


• In patients with moderately advanced Alzheimer’s disease, a prospective trial of the antioxidant selegiline,
α-tocopherol (Vit. E).

14. Ans. is d i.e. Extrapyramidal side effects Ref. KDT 6/e, p 431 - 432

Clozapine is an atypical antipsychotic absence of extrapyramidal side effects because of lower affinity for D1
and D2.

Mechanism of action : 5HT2 > H1 = M1 = α1 = D4 > D2 = D1


Mainly act 5HT2 and D4 competitively block.
Superiority over haloperidol in controlling the positive as well as negative symptoms.
Also use in refractory schizophrenia.

Side effects : • Agranulocytosis and seizures.


• It also cause paradoxically hypersalivation.
• Fatigue and drowsiness most common adverse effect
• Dizziness and headache also occour
• Fever because decrease WBC - rule out agranulocytosis
• Anticholinergic effects blurring vision, drymouth and difficulty in micturition.
• Postural hypotension in initial stage of treatment.

15. Ans. is b i.e. Sevoflurane


Ref. Goodman & Gilman 11/e, p 359 - 360; KDT 6/e, p 373

Because of its non pungency and rapid increase in alveolar anesthetic concentration makes sevoflurane an
excellent choice for smooth and rapid inhalation induction in pediatrics patients.

186
Phar macology of Central Nervous System
Pharmacology 3
The speed of induction by inhalational anesthetics in descending order is :
• Nitrous oxide • Halothane
• Isoflurane • Sevoflurane
• Desflurane

16. Ans. is d i.e. Thiopentone Ref. KDT 6/e, p 372; CSDT, p 93

The drugs causing Malignant Hyperthermia are :


Muscle relaxants Inhalational agents Local anesthetics Others
• Succinylcholine • Halothane is the most • Lignocaine • Tricyclic antide-
is the most common inhalational presents
commonly agent • Mono amine
implicated drug. • Isoflurane oxidase inhibitors
• Desflurane • Phenothiazines
• Sevoflurane
• Methoxyflurane

Signs and Symtoms : • Tachycardia Tachypnea


• Hypertension Hypercapnia
• Hyperthermia Acidosis
• Skeletal muscle rigidity.

Treatment : • Immediate administr ation of dantrolene sodium


• Stop anesthetics agents
• Hyperventilation with 100% oxygen
• Control acidosis with sodium bicarbonate 2-4 mEq/kg
• Correct electrolyte imbalance (hyperkalemia)
• Maintain urine output.

17. Ans. is c i.e. Propofol Ref. KDT, 6/e, p 375

Drugs used in outpatient anaesthsia day care surgery


Group Drugs
Inducing agent • Propofol (agent of choice) • Methohexital
• Thiopentone • Etomidate
Muscle relaxant • Mivacurium • Succinyl choline
Volatile inhalational agent • Isoflurance • Sevoflurance
• Desflurance
Total intravenous • Propofol • Remifentanil
anaesthetic technique • Alfentanil • Sufentanil

187
3 Self Assessment & Review Phar macology
Pharmacology
18. Ans. is d i.e. Diazepam Ref. KDT 6/e, p 449 - 450; Goodman & Gilman 11/e, p 1105

In the question, patient is elderly and with liver disease Preferred in elderly with liver disease because hepatic
metabolism is not significant and duration of action is short.
Chloriazepoxide, diazepam, and flurazepam have active metabolities that are subject to enterohepatic recycling,
which accounts for their relatively long half lives. Alprazolam, triazolama, Oxazepam, Tamezepam, Triazolam
are rapidly inactiviated by glucuronidation, which accounts for their relatively short half-lives

The BZD do not produced active metabolite are :


O Oxazepam
L Lorazepam
T Tamezepam, Triazolam
M Midazolam

19. Ans. is c i.e. Beta-carboline Ref. KDT 6/e, p 349, 395

Benzodiazepine GABAA (GABA facilitatory)


Antagonist Inverse Agonist
Flumazenil β –Carboline
(DMCM) Dimethoxyethyl-
carbomethoxy-β carboline

Drug affecting GABA receptor


GABAA (Inotropic) GABAB (Metabotropic)
Agonist Antagonist Agonist Antagonist
• GABA • Picrotoxin • Baclofen • Saclofen
• Muscimol • Bicuculline
• Barbiturate
• Alphaxalone
• Alcohol
• BZD

20. Ans. is d i.e. Buspirone Ref. KDT 6/e, p 45; Rang & Dale 5/e, p 603

• Buspirone is an anxiolytic drug, lack of sedation and other depressing effects on CNS and no apparent
potential for abuse or dependence.
• Selective partial agonistic action on 5-HT1A receptors. By stimulating presynatpic 5-HT1A autoreceptors, it
reduces the activity of dorsal raphe serotonergic neurones.
• Antagonism at certain postsynaptic 5-HT1A receptors has also been demonstrated.
• Buspirone has weak dopamine D2 blocking action but no antipsychotic or extrapyramidal effects.
Option “a” ethanol acts as an agonist of GABAA receptors.
Option “b” Etomidate is a steroidal anaesthetic agent and acts as GABAA receptor agonist.
Option “c” Zolpidem nonbenzodiazepine hypnotics acts on ω1 subtype of BZD (GABA) receptor.
Option “d” Buspirone is azopirone antianxiety drug.

188
Phar macology of Central Nervous System
Pharmacology 3
21. Ans. is b i.e. Atropine Ref. KDT 6/e, p 341

• Oculogyric crisis is a form of acute dystonic reaction seen after neuroleptic drug treatment.
• Trifluperazine, perphenazine, perchloroperazine all are neuroleptic agent and causes oculogyric crisis by
inhibiting dopaminergic system where as atropine is anticholinergic agent it will certainly not cause oculogyric
crisis.

Oculogyric crisis (OGC) : The term “Oculogyric” refers to rotating of eyeballs, but several other responses are
associated with the crisis.
Drugs that can trigger an oculogyric crisis include neuroleptics, amantadine, benzodiazepines, carbamazepine,
chloroquine, cisplatin, diazoxide, influenza vaccine, levodopa, lithium, metoclopramide, nifedipine, pemoline,
phencyclidine, reserpine, and tricyclics.

Treatment : Immediate treatment of drug induced OGC can be achieved with intravenous antimuscarinic
benztropine or procyclidine; which usually are effective within 5 minutes, although may take as long as 30
minutes for full effect.

22. Ans. is a i.e. Formaldehyde and Formic acid


Ref. KDT 6/e, p 386; Goodman & Gilman 11/e, p 599, 600; Harrison 16/e, p 2591

Methanol is toxic by two mechanisms. Firstly, methanol (whether it enters the body by ingestion, inhalation, or
absorption through the skin) can be fatal due to its CNS depressant properties in the same manner as ethanol
poisoning. Secondly, it is toxic by its metabolism (toxication) via the enzyme alcohol dehydrogenase in the liver
by forming formic acid and formaldehyde which cause permanent blindness by destruction of the optic nerve[
The usual fatal dose is 100–125 mL (4 fl oz)

Clinical Symptom : 1. Ist 6 - 18 hours – Direct depressant action on CNS.


2. After 6 - 18 hours – Symptoms due to formic acid.
Metabolic Acidosis Kussmaul breathing but not common.
Respiration is slow, shallow gasping, “fishmouth”.
Cause of death - Respiratory failure.
Retinal Congested edematous, blurred edges of optic disc.
Centrocecal scotoma.
Ganglion cell of retina is most affected.
Dilated unreacted pupils.
Final result is bilateral, permanent blindness.
Ocular toxicity is directly related to blood formic acid level.
Abdominal pain Due to pancreatic necrosis.

Treatment :
• Keep patient in quiet and dark room to protect the eyes from light.
• Gastric lavage – by sodium bicarbonate
• I.V. sodium bicarbonate – Correct acidosis
• Potassium chloride – If hypokalemia occurs
• Ethanol : Ethanol has 100 fold greater affinity for alcohol dehydrogenase than methanol. So, the formation
of formic acid and formaldehyde is prevented.

189
3 Self Assessment & Review Phar macology
Pharmacology
Ethanol concentration should be 1gm/liter. For this, the loading dose of 6g/kg is given as soon
as possible and then, the continuous infusion of 10gm/hour.
• 4 – methyl pyrazole (Fomepizole) – Specific inhibitor of alcohol dehydrogenase.
• Folate Therapy – Folic acid enhance the metabolism of formic acid and formate to CO2 + H2O (Calcium
leucovorin).
• Hemodialysis : Indications : – Acidosis even after giving ethanol therapy
– Methanol blood level > 500mg/litre
– Osmolar gap > 10mosm/l
• Disulfiram : Inhibitor of Aldehyde dehydrogenase

23. Ans. is d i.e. Domperidone blocks levodopa induced emesis and its therapeutic potential
Ref. KDT 6/e, p 417 - 418, 645

Domperidone : Domperidone blocks levodopa induced nausea and vomiting. Without abolishing its
antiparkinsonian effect because domperidone does not cross blood brain barrier.

Interactions of levodopa :
• PYRIDOXINE : Abolish therapeutic effect by enhancing peripheral decarboxylation of levodopa, less is
available to cross to the brain.
• Phenothiazines, butyrophenones, metoclopromide reverse therapeutic effect by blocking DA receptors

24. Ans. is a i.e. Head injury Ref. KDT 6/e, p 457, 461; Katzung 10/e, p 503

Morphine is contraindicated in all the 4 options given but best is Head injury.

Morphine is contraindicated in Head injury because :


• By retaining CO2, it ↑ intracranial tension which will add to that caused by head injury itself. (Option ‘a’)
• Even therapeutic doses can cause marked respiratory depression in these patients.

190
Phar macology of Central Nervous System
Pharmacology 3
• Vomiting, miosis and altered mentation produced by morphine interfere with assessment of progress in
head injury cases.
• Bronchial Asthma (Option ‘b’)
Morphine can precipitate an attack by its histamine releasing action, but morphine affords dramatic
relief in cardiac asthma (acute left ventricular failure) by :
– Reducing preload on heart due to vasodilatation and peripheral pooling of blood.
– Tends to shift blood from pulmonary to systemic circuit, relieves pulmonary congestion and edema.
– Allays air hunger by depressing respiratory centre.
– Cuts down sympathetic stimulation by calming the patient, reduces cardiac work.

Use of morphine in patient with endocrine disease. (Option ‘c’)


Patient with adrenal insufficiency (Addison disease) and those with hypothyroidism (myxedema) may have
prolonged and exaggerated responses to opioids.

Inference : Thus, morphine is given in asthma (cardiac asthma) and also can be given in hypothyroidism but
cautiously. So, clear cut contraindication of morphine is head injury.

Mnemonic : MORPHINES effects


• Miosis • Histamin release/ • Euphoria
• Orthostatic hypotension Hormonal alterations • Sedation
• Respiratory depression • Increased ICT
• Pain supression • Nausea

25. Ans. is a i.e. Supraspinal analgesia Ref. Goodman & Gilman 11/e, p 551

The δ receptor is involved in analgesia, both spinal and supraspinal (mainly the affective component), as well as
in causing opioid reinforcement and reward (along with μ receptors).

Classification of opioid Receptor and actions


Receptor subtype Action on agonist Antagonist

Analgesia
– Supraspinal μ,k,δ Analgesic
– Spinal μ,k,δ Analgesic No effect
– Respiratory function μ Decrease (Bradypnea)
– GIT μ,k Decrease (Constipation)
– Psychotomimetic (Euphoria) k Increase
– Feeding μ,k,δ Increase feeding Decrease Feeding
– Sedation μ, k Increase No effect
– Diuresis k Increase
Hormone Regulation
– Prolactin μ Increase Release Decrease Release
– GH μ/δ Increase Release Decrease Release

Neurotransmitter release
– Ach μ Inhibit
– Dopamine μ,δ Inhibit

191
3 Self Assessment & Review Phar macology
Pharmacology
Mnemonic : PM CARE
Acts through μ receptors PM : Prolactin Miosis Euphoria
CAR : Constipation Analgesia Respiratory function

26. Ans. is b i.e. Cyclosporine Ref. KDT 6/e, p 876

Important Drug Interactions of Pyridoxine


• Pyridoxal + INH Pyridoxal –hydrazone is formed which is rapidly excreted in urine, pyridoxine
deficiency occurs
So, pyridoxal can’t get converted to pyridoxal - phosphate (the active form of enzyme)
• P – P enicillamine ⎫
H – H ydralazine ⎬ Interfere with pyridoxine utilisation lead to vitamin B6 deficiency
C – C ycloserine ⎭

Note : In question, Cyclosporine is given in place of Cycloserine


(Same type of names – Confusion occurs)

Levodopa decarboxylase Dopamine


⎯⎯⎯⎯⎯⎯⎯⎯⎯→
B6

Pyridoxine enhances peripheral decarboxylation of Levodopa →Dopamine. Less levodopa remains to


cross blood brain barrier.

Therapeutic effect ↓↓ed in Parkinson’s disease.
• Oral Contraceptives – decrease pyridoxal – PO4 levels

Pyridoxine Dependent Enzymes

• Decarboxylases • Transaminases
• Threonine aldolase • ALA synthase (1st step in heme biosynthesis)

27. Ans. is a i.e. Phenytoin Ref. KDT 6/e, p 405

Adverse Effects of Phenytoin


Over-Dose Toxicity :
• Nystagmus, ataxia, CNS depression lethargy, and blurred vision.
• Fast I.V. injection cardiovascular collapse and coma.
Chronic Toxicity :
• Gingival hyperplasia and coarsening of facial features; because phenytoin after being secreted through
saliva, inhibits the enzyme collagenase which is responsible for the breakdown of collagen of connective
tissue.
• Megaloblastic anaemia, because phenytoin induces hydroxylating enzyme activity for which folates are the
cofactors. As serum folic acid levels go down, the body’s demand for folic acid increase.

192
Phar macology of Central Nervous System
Pharmacology 3
• Vitamin K deficiency, caused by phenytoin leads to hypoprothrombinemia and consequently haemorrhage
in the newborn of the mother who received phenyotin during pregnancy.
• Phenytoin increase the metabolism of calciferol and produces vitamin-D deficiency. This leads to rickets in
malnourished children and osteomalacia in adults.
• Hirsutism (in females) and acne may appear due to increased secretion of androgens.
• Phenytoin inhibits insulin secretion and may cause hyperglycaemia. It also decreases ADH release.
• Use of phenytoin during pregnancy increases the risk of congenital malformation (cleft lip, cleft palate and
congenital heart disease.
• Other adverse effects hypersensitivty reactions skin rashes, fever, hepatitis and some CNS effects such as
vertigo, nausea, tremors and confusion.
• Phenytoin should not be discontinued suddenly as it may precipitate withdrawl seizures.

28. Ans. is c i.e. Selective serotonin reuptake enhancer Ref. KDT 6/e, p 447-448

Atypical antidepressant drugs

Tianeptine Trazodone Mianserin Bupropion

Tianeptine is Inhibit uptake : 5-HT Does not Inhibit DA,


selective serotonin Also blocks H1 & α1 inhibit either NA or NA uptake
reuptake enhancer receptors 5HT uptake but blocks, No, weight gain
5HT2,5HT1C,& H2 used for Cessation
Side effect : receptor presynaptic
Sedation, Hypotension, - ↑ release and NA turnover of smoking
Priapism - Cardio safe in elderly
with depression and CHD

29. Ans. is b i.e. Acute intermittent porphyria Ref. KDT 6/e, p 392; Harrison 17/e, p 2439

Acute intermittent Porphyria (AIP) : Barbiturates exacerbate it by inducing microsomal enzymes and δ
amino levulinic acid synthetase and increasing porphyrin synthesis.

Contraindications of barbiturates : • Liver and Kidney disease


• Severe pulmonary insufficiency eg. emphysema
• Obstructive sleep apnoea.
Categories of unsafe and safe drugs in AIP, HCP and VP (Hereditary coproporphyria, variegated porphyria).
UNSAFE • Barbiturates • Sulfonamides antibiotics • Meprobamate
• Glutethimide • Ethchlorvynol • Mephenytoin
• Succinimides • Carbamazepine • Valproic acid
• Griseofulvin • Ergots • Synthetic Estrogens
• Synthetic Progestogens • Danazol • Alcohol
SAFE • Narcotic analgesics • Aspirin • Acetaminophen
• Phenothiazines • Penicillin derivatives • Streptomycin
• Glucocorticoids • Bromides • Insulin
• Atropine
Porphyria with epilepsy – DOC for epilepsy – BROMIDES

193
3 Self Assessment & Review Phar macology
Pharmacology
30. Ans. is b i.e. Salicylates Ref. Katzung 10/e, p 963; KDT 6/e, p 190

Hemodialysis not indicated

Mnemonic : DKBOs D - Digoxin B - Benzodiazepine


K - Kerosine O - Organophosphorus

Indications for Hemodialysis (HD) and Hemoperfusion (HP) in drug poisoning :


Hemodialysis (HD) Hemoperfusion (HP)
Ethylene glycol HD Carbamazepine HP
Lithium HD Ethchlorynol HP
Methanol HD Phenobarbital HP
Metformin HD Meprobamate HP
Salicylate HD Procainamide HD or HP
Valproic acid HD Theophylline HD or HP
Procainamide HD or HP
Theophylline HD or HP

31. Ans. is a i.e. It is a MAO-A inhibitor Ref. KDT 6/e, p 420

Selegiline
It is a selective MAO-B inhibitor (not MAO - A inhibitor as given in option ‘a’)
MAO-B predominates in brain and blood platelets. Selegiline in low doses (10mg/day), does not interfere with
peripheral metabolism of dietary amines; catecholamine accumulation and hypertensive reaction (cheese reaction)
does not develop. Option ‘b’
– It retards intracerebral degradation of DA responsible for therapeutic effect in parkinsonism. Option ‘d’
– Administered with levodopa it prolongs levodopa action, attenuates motor fluctuations and decreases ‘Wearing
off effects. Option ‘c’.
– Clinical benefits derived from selegeline are short lived (6-26 months).
– Selegiline by inhibiting MAO, prevent formation of free radical and delay the progression of the
disorder (Parkinsonism).

Adverse effect : Postural hypotension, nausea, confusion, accentuation of levodopa induced involuntary
movement and psychosis.

Contraindication : Patients with convulsive disorders.

32. Ans. is b i.e. MCV should be estimated Ref. KDT 6/e, p 405

This patient on phenytoin complains of lassitude and has Hb of 8 which denotes patient is anemic.
Phenytoin is known to cause megaloblastic anemia, by decreasing folate absorption and decreasing its excretion
therefore here we do the investigations MCV, mean corpuscular volume to determine if patient is having a
megaloblastic type picture.
• Megaloblastic anaemia, because phenytoin induces hydroxylating enzyme activity for which folates are the
cofactors. As serum folic acid levels go down, the body’s demand for folic acid increases.

For more details about Side effects of Phenytoin, kindly see answer no. 27

194
Phar macology of Central Nervous System
Pharmacology 3
33. Ans. is d i.e. Decreased efficacy will result Ref. KDT 6/e, p 875 - 876

Levodopa decarboxylase Dopamine


⎯⎯⎯⎯⎯⎯⎯⎯⎯→
B6

Pyridoxine enhances peripheral decarboxylation of Levodopa →Dopamine. Less levodopa remains to


cross blood brain barrier.

Therapeutic effect ↓↓ed in Parkinson’s disease.

34. Ans. is c i.e. Phenobarbitone


Ref. Goodman & Gilman 11/e, p 511; Harrison 17/e, p 2507-2510

A casual role of antiseizure drug is suggested by association of congenital defects with higher concentrations
of drug or with polytherapy as compared monotherapy.
Phenobarbitone is drug of choice of epilepsy in pregnancy.
Phenobarbitone can be used but Vit.K 20mg/day during last month of gestation has been recommended for
prophylaxis because phenobarbitone causes transient and reversible deficiency of Vit. K dependent clotting
factors in 50% of infants. Infant should receive Vit. K 1mg at birth.
For more details about Antiepileptic drugs used in different epilepsies, kindly see answer no. 2

35. Ans. is b i.e. GABA-agonism Ref. Goodman & Gilman 11/e, p 523; KDT 6/e, p 410

Vigabatrin acts via inhibiting GABA transaminase enzyme which degrades GABA which is an inhibitory
neurotransmitter.

Newer Anticonvulsant Drugs :


Drugs Possible Mechanism Indications Toxic Potential
Vigabatrin Inhibits GABA transaminase Partial seizures Depression, psychosis,
visual dysfunction
Gabapentin ↑ GABA effect Dipolar disorder Sedation, ataxia,
neuropathic pain cognitive change.
Tiagabine Blocks GABA transporter Partial seizures Sedation, dizziness,
“Flu-Like” symptoms
Lamotrigine Blocks Na channels and Absence and partial Sedation, ataxia,
glutamate receptors seizures diplopia, Stevens-
Jhonsons syndrome
Topiramate Blocks glutamate (AMPA) Partial seizures Renal stones
receptors, ↑ GABA effects.
Felbomate Blocks Na+ and Ca++ Partial seizures Aplastic anemia, acute
channels, blocks glutamate liver failure
receptors.

195
3 Self Assessment & Review Phar macology
Pharmacology
36. Ans. is b i.e. MAO-B Ref. KDT 6/e, p 420

There are two types of MAO :


MAO A : MAO-B
Predominates in peripheral adrenergic nerve Predominates in brain and in platelets
endings, intestinal mucosal, human placenta
Deaminates 5HT and NA
Inhibited by : • RIMA’s (Reversible inhibitors of MAO-A) Inhibited by : Selegiline
– Clorgylline
– Moclobemide

Selegiline already explained in answer no. 32

37. Ans. is a i.e. Benzodiazepine antagonist Ref. KDT 6/e, p 395, 399 - 400

FLUMAZENIL is Benzodiazepine receptor antagonist

Effects
– Little intrinsic activity (no effect in normal people).
– It competes with BZD agonists and BZD inverse agonists for BZD receptor and reverses their depressant
and stimulant effects respectively.
– It abolishes hypnogenic, psychomotor, cognitive and EEG effects of BZDs.
– Does not antagonise CNS effects of other sedatives-hypnotics, opioids, alcohol and GA.

Pharmacokinetics :
• Oral – High first pass metabolism in liver.
– Oral bioavailability is 16%, so not used orally.
• IV – Action starts in seconds.
– Action lasts for 1-2 hours.
– Elimination t½ is 1 hours.

USES : – To reverse BZD anaesthesia


– BZD overdose
In patients who have ingested BZD with TCAs, seizures and cardiac arrhythmias can occur when they ingest
flumazenil.

38. Ans. is c i.e. Tardive dyskinesia Ref. KDT 6/e, p 431 - 432

Remember : Tardive dyskinesia is most common side effect on chronic use.

For more details, refer answer no. 12

39. Ans. is a i.e. Hypothermia Ref. Katzung 10/e, p 439; KDT 6/e, p 431

Neuroleptic malignant syndrome is a life threatening disorder that occurs in patients who are extremely
sensitive to the extrapyramidal effects of antipsychotics.

196
Phar macology of Central Nervous System
Pharmacology 3
This syndrome is believed to result from an excessively rapid blockage of post synaptic dopaminergic receptors.

Features : • Initial symptoms is marked muscular rigidity.


• Fever may ensue often reaching dangerous levels (Hyperthermia).
• Autonomic instability and altered BP and Pulse rate (is a mid brain manifestation).
• Stress Leukocytosis.
• Elevated creatine kinase isoenzymes (due to muscle damage).
Treatment : Muscle relaxants such as dantrolene, or dopamine agonists such as bromocriptine.

40. Ans. is a i.e. Marijuana Ref. KDT 6/e, p 438

THC (delta-9-tetrahydrocannabinol) : It is the active principle of cannabis indica (marijuana).


It is used in the following forms :
• Bhang : from dried leaves
• Ganja : from dried female inflorescences (smoked)
• Charas : is dried resinous extract from the flowering tops and leaves (most potent) also called hashish.
• LSD : D-lysergic Acid Diethylamide
• Hashish : Canabinoids variety
• Heroin : Monphine

41. Ans. is a i.e. Valproic Acid Ref. KDT 6/e, p 404

Valproic acid is increase concentrations of GABA, which decreases the frequency of neuronal firing. It may
also have a direct effect on the neuronal membrane by the inhibition of voltage-gated sodium channels and the
resulting decrease in sodium influx
Valproate appears to act by multiple mechanisms :
• Prolongation of Na+ channel opening A • Facilitation of GABA mediated

• Inhibition of ’T’ type Ca++ current C Cl– channel opening B

197
3 Self Assessment & Review Phar macology
Pharmacology
Valproic acid is effective in the control of simple or complex absence seizures and partial seizures, it is also
useful as adjunct therapy in patients who exhibit multiple seizure types, including absence or tonic-clonic
seizures
Children under valproate therapy have an increased incidence orofacial hepatotoxicity, particularly if the drug
is used in combination with other anticonvulsant agents. When used as monotherapy. the risk declines
substantially. Children under two years old have the greatest risk of hepatotoxicity (approximately 20-fold),
so if the drug is to be administered, it should be used as monotherapy and carefully monitored.
Divalproex sodium drug is a 1:1 combination of valproic acid and sodium valproate dispensed in an enteric-
coated tablet for treatment of absence or partial seizures of the tonic-clonic type. Divalproex Na is also useful
in the therapy of bipolar disorder

42. Ans. is a i.e. Morphine Ref. KDT 6/e, p 463

Characteristics and Comparison of Opioid Analgesics


Drugs μ κ δ
• Morphine (prototype) +++ + +
• Codeine ++ + +
• Dextropropoxyphene ++ + +
• Etorphine +++ +++ +++
• Fentanyl +++ – +
• Heroin (Diamorphine) +++ + +
• Levorphanol +++ + +
• Methadone +++ – –
• Meperidine (Pethidine) ++ + +
• Tramadol + – –

43. Ans. is c i.e. Risk of addiction is more than that with morphine
Ref. Rang & Dale 5/e, p 575; KDT 6/e, p 464 - 465

Pentazocin Morphine

• Classified – Agonist antagonist • Agonist


• Action – Agonist on κ receptor • Agonistic action on μ receptor
(weak μ – Antagonist / partial
antagonist) agonist at μ receptor
• Route / Use – Oral / Parenteral • Rarely oral
• Analgesia – At spinal cord • At supraspinal
• CVS – Tachycardia ↑ ed BP
• Sed. / Resp. dep. – Less • More
• Vomiting – Less • More
• Biliary spasm – Less severe • More
• Abuse liability – Less • More

198
Phar macology of Central Nervous System
Pharmacology 3
44. Ans. is a i.e. Flumazenil Ref. KDT 6/e, p 349, 395, 399 - 400

Benzodiazepine GABAA (GABA facilitatory)


Antagonist Inverse Agonist
Flumazenil β –Carboline
(DMCM) Dimethoxyethyl-
carbomethoxy-β carboline

FLUMAZENIL
BDZ Antagonist
Flumazenil reverses many actions of BDZs and imidazopyridines but not of alcohol, opiates, GA,
barbiturates. Sedation is most promptly reversed. Respiratory depression is not reversed completely.
Flumazenil has short t½ (0.7-1.3 hrs.) needs repeated dosing.
ADRs : Agitation confusion, dizziness, nausea, seizures.
Other uses : Hepatic encephalopathy
– Little intrinsic activity (no effect in normal people).
– It competes with BZD agonists and BZD inverse agonists for BZD receptor and reverses their depressant
and stimulant effects respectively.
– It abolishes hypnogenic, psychomotor, cognitive and EEG effects of BZDs.
– Does not antagonise CNS effects of other sedatives-hypnotics, opioids, alcohol and GA.

Pharmacokinetics : • Oral – High first pass metabolism in liver.


– Oral bioavailability is 16%, so not used orally.
• IV – Action starts in seconds.
– Action lasts for 1-2 hours.
– Elimination t½ is 1 hours.

USES : – To reverse BZD anaesthesia


– BZD overdose
In patients who have ingested BZD with TCAs, seizures and cardiac arrhythmias can occur when they ingest
flumazenil.

45. Ans. is a i.e. Used in trigeminal neuralgia Ref. KDT 6/e, p 406 - 407

Uses of Carbamazepine
• It is the most effective drug for chronic partial seizure and shares first choice drug status with phenytoin for
GTCS and SPS.
• Trigeminal and related neuralgias : Carbamazepine is the drug of choice.
• Manic depressive illness and acute mania : As an alternative to lithium.

Interactions : Carbamazepine is an enzyme inducer; can reduce efficacy of haloperidol, oral contraceptives,
lamotrigine and topiramate. Metabolism of carbamazepine is induced by phenobarbitone, phenytoin, valproate
and vice versa.

199
3 Self Assessment & Review Phar macology
Pharmacology
46. Ans. is a i.e. Diazepam Ref. KDT 6/e, p 385

Alcohol withdrawal syndrome consists of anxiety, sweating, tremor, impairment of sleep, confusion,
hallucinations, delirium tremens, convulsions and collapse.
Treatment :
• Psychological and medical supportive measures are needed during withdrawal.
• Benzodiazepines (chordiazepoxide, diazepam) are the preferred drugs now. These have a long duration of
action and can be gradually withdrawn later.
• Naltrexone helps prevent relapse of alcoholism. It reduced alcohol craving, number of drinking days and
chances of resumed heavy drinking.

47. Ans. is c i.e. Partial agonist Ref. Rang & Dale 5/e, p 575

Buprenorphine is partial agonist of opioid receptor.


This drug is a morphine-type mixed agonist-antagonist opioid. It exerts analgesic effects by acting as
partial agonist at the m receptor, and has little affinity for k receptors
Selective of opioid drugs and peptides for receptor subtypes
Opiates drugs
Pure agonist Partial mixed agonists Antagonist Endogenous peptides
• Morphine, codeine, • Pentazocine • Naloxone • β-Endorphin
oxymorphone • Nalbuphine • Naltrexone • Leu-enkephalin
dextropropoxyphene • Nalorphine • Met-enkephalin
• Methadone • Buprenorphine • Dynorphin
• Meperidine
• Fentanyl, sulfentanil

48. Ans. is d i.e. Promethazine Ref. KDT 6/e, p 390 - 395

Looking at each option separately.


Thiopentone : Barbiturates act at GABAA : BZD receptor–CI– channel complex and potentiate GABAergic
inhibition by increasing the lifetime of CI– channel opening induced by GABA (contrast
BZDs which enhance frequency of CI– channel opening).
At high concentrations, barbiturates directly increase CI– conductance (GABA-mimetic
action).
Midazolam : BZDs act by enhancing presynaptic / postsynaptic inhibition through a specific BZD receptor
(α1 2β2, 2γ 2 pentamer isoform) which is an integral part of the GABAA receptor–CI– channel
complex.
Zolpidem : The newer non BZD hypnotics Zaleplon, Zolpidem, etc. have high affinity for a1 subunit
isoform of GABAA : BZD receptor–CI– channel complex and exert selective hypnotic-amnesic
effect, but have little antiseizure or muslce relaxant property.
Promethazine : Promethazine is an antihistaminic drug and does not ineract with GABAA receptors.

Note : Barbiturate – GABA facilitatory action


– GABA mimetic action
Benzodiazepines – GABA facilitatory action

200
Phar macology of Central Nervous System
Pharmacology 3
49. Ans. is b i.e. At very low doses, zero order kinetics occurs
Ref. KDT 6/e, p 405; Katzung 10/e, p 375 - 377

Option “a” Phenytoin induces microsomal enzymes and increases degradation of steroids (failure of oral
contraceptives), digitoxin, doxycycline, theophylline.
Option “b” The elimination of phenytoin is dose-dependent. At very low blood levels, phenytoin metabolism
follows first-order kinetics.
As blood levels rise within the therapeutic range, the maximum capacity of the liver to metabolize
phenytoin is approached and the kinetics changes from first-order to zero-order.
Option “c” The half-life of phenytoin varies from 12 hours to 36 hours, with an average of 24 hours for most
patients in the low to mid therapeutic range. Much longer half-lives are observed at higher
concentrations.
Option “d” Phenytoin is highly (90%) bound to plasma proteins, other highly bound drugs, such as
phenylbutazone and sulfonamides, can displace phenytoin from its binding site.

50. Ans. is c i.e. Overdose is better tolerated compared to other hypnotics Ref. KDT 6/e, p 393

Advantages of Benzodiazepines (BZDs) over other hypnotics :


• BZDs have a high therapeutic index. Ingestion of even 20 hypnotic doses does not usually endanger life–
there is no loss of consciousness (though amnesia occurs) and patient can be aroused.
• Hypnotic doses do not affect respiration or cardiovascular functions.
• BZDs have practically no action on other body systems.
• BZDs cause less distortion of sleep architecture; rebound phenomena on discontinuation of regular use are
less marked.
• They have lower abuse liability: tolerance is mild, psychological and physical dependence and withdrawal
syndrome are less marked.
• A specific BZD antagonist flumazenil is available which can be used in case of poisoning.

51. Ans. is a i.e. Flumazenil Ref. KDT 6/e, p 399

Already explained, kindly see answer no. 37

52. Ans. is c i.e. Topiramate Ref. Goodman & Gilman 11/e, p 520

• Topiramate is an anti-epileptic drug whose side effects are somnolence, fatigue, weight loss, and nervousness.
• It can precipitate renal calculi, and a change in the taste of carbonated beverages.
• Valproic Acid causes weight gain with chronic use.
• Topiramate to be equivalent to valproate and carbamazepine in children and adults with newly diagnosed
partial and primary generalized epilepsy.

53. Ans. is d i.e. Tiagabine Ref. KDT 6/e, p 409 - 410; Goodman & Gilman 10/e, p 522

Tiagabine inhibits the GABA transporter GAT-1 and inhibition of GABA transporter reduces the uptake of GABA
into neurons and Glia.

201
3 Self Assessment & Review Phar macology
Pharmacology
Newer Anticonvulsant Drugs :
Drugs Possible Mechanism Indications Toxic Potential
Vigabatrin Inhibits GABA transaminase Partial seizures Depression, psychosis,
visual dysfunction
Gabapentin ↑ GABA effect Dipolar disorder Sedation, ataxia,
neuropathic pain cognitive change.
Tiagabine Blocks GABA transporter Partial seizures Sedation, dizziness,

For more details, kindly see answer no. 37

54. Ans. is d i.e. Diuresis Ref. KDT 6/e, p 462; Goodman & Gilman 10/e, p 573

Diuresis is mediated by Kappa receptor not μ receptor

Mnemonic : PM CARE
Action through μ receptors PM : Prolactin Miosis Euphoria
CAR : Constipation Analgesia Respiratory function

55. Ans. is b i.e. Epinephrine Ref. KDT 6/e, p 467; Goodman & Gilman 11/e, p 548; Katzung 10/e, p 489

• Human body produces opium like analgesics but they are chemically peptides, hence called endogenous
opioids.
• Adrenaline / Epinephrine is a neurotransmitter secreted by adrenal medulla are amines.

Endogenous Opioid Peptides are:

Endorphine μ Dynorphins κ Enkephalin δ

• β Endorphin • Dynrophin A • Leu-enkephalin


• Neoendorphin α 1 β • Dynrophin B • Met-enkephalin
• Enkephalin

Novel Endogenous Opioid Related Peptides : Endogenous Opioid Peptide Affinity on different receptors
• Orphanin/ Nociceptin • μ Endorphin > Enkephalins > Dynorphins
• Endomorphin – 1 • δ Enkephalins >> Endorphins > Dynorphins
• Endomorphin – 2 • κ Dynorphins >> Endorphins/Enkephalin

56. Ans. is d > b i.e. Na+ is a specific antidote for Lithium intoxication
Ref. KDT 6/e, p 435

Treatment in case of intoxication is symptomatic. There is no specific antidote. Osmotic diuretics and
sodium bicarbonate infusion promote Li excretion.
Haemodialysis is indicated if serum levels are >4mEq/L.

202
Phar macology of Central Nervous System
Pharmacology 3
Margin of safety is narrow, monitoring serum Li concentration is essential for optimal therapy. Serum Li level is
measured 12 hours after the last dose to reflect the steady-state concentration; 0.5-0.8mEq/L is considered
optimum for maintenance therapy in bipolar disorder while 0.8-1.1mEq/L is required for episode of mania.
Thiazide diuretics have a significant potential to increase serum lithium concentrations. These diuretics induce
a natriuresis that leads to a compensatory increase in the reabsorption of sodium (and lithium) in the proximal
tubule. This effect of thiazide diuretics has been suggested in many case reports, describing lithium toxicity
subsequent to thiazide initiation, and has also been documented in a handful of small controlled studies. In
general, therapeutic doses of thiazide diuretics result in a 25 to 40% decrease in lithium clearance with a
concomitant increase in serum lithium levels. The nature of this interaction is quite variable and the most
conservative approach is simply to avoid the use of thiazide diuretics if possible. Option “b”

Toxicity symptoms occur frequently when serum levels exceed 1.5mEq/L.


For more details, refer answer no. 11

Doses of Lithium : – Maintenance - 0.5-0.8mEq/L


– Mania - 0.8-1.1mEq/L
– Toxicity - 1.5 mEq/L

Uses of Lithium : • Prophylaxis in bipolar disorder


• Acute manic episode
• Cancer chemotherapy induced leucopenia and agranulocytosis
• Inappropriate ADH secretion syndrome. ... KDT 5/e, p 536
+
Important point : Amiloride blocks entry of Li through Na channels in CD cells and mitigates diabetes
insipidus induced by lithium.

57. Ans. is d i.e. Methyl alcohol would be metabolised to acetaldehyde


Ref. KDT 6/e, p 386; Goodman & Gilman 11/e, p 599, 600; Harrison 16/e, p 2591

Already explained, kindly see answer no. 22

58. Ans. is b i.e. Sufentanil


Ref. Lee 12/e, p 79 - 80, 190, Morgan’s Anaesthesia 3/e, p 167 (table 8.5); Goodman & Gilman 11/e, p 595

Sufentanil is highly bound to plasma protein principally to α-1-acid glycoprotein and it is highly lipophilic therefore
transfer rapid to CNS.

Characteristic of Opioids :

Drugs Nonionized Fraction Protein Binding Lipid Solubility

Sufentanil ++ ++++ (90%) ++++


Remifantanil +++ +++ ++
Fentanyl + +++ ++++
Alfentanil ++++ ++++ +++
Morphine ++ ++ (40%) +
Pethidine 64%

Potency of Analgesic effect : Sulfentanil > Remifentanil > Fentanyl > Alfentanil > Morphine > Pethidine

203
3 Self Assessment & Review Phar macology
Pharmacology
59. Ans. is a i.e. Parkinson’s disease Ref. KDT 6/e, p 417

Ropinirole is dopamine agonist used in Parkinson’s disease.

204
Phar macology of Central Nervous System
Pharmacology 3
Metabolic pathways of levodopa in the periphery and the brain.
3-OMD–3-O-methyldopa; COMT–Catechol-O-methyl transferase; MAO–monoamine oxidase; 3-MT–3-
methoxytyramine; DOPAC–3, 4 dihydroxy phenylacetic acid; HVA–Homovanillic acid (3-methoxy-4-hydroxy
phenylacetic acid), DDC–Dopa decarboxylase

60. Ans. is d i.e. Akathesia Ref. KDT 6/e, p 431

Already explained, kindly see answer no. 12

61. Ans. is. c i.e. Shift to sodium valporate Ref. Goodman & Gilman 11/e, p 515; Harrison 17/e, p 2509

This patients should shift to valproate. Sodium valproate is now considered the drug of choice for all kind of
seizures except partial seizures.
Discontinuation of the anticonvulsant is essential in patients who develop symptoms compatible with
anticonvulsant hypersensitivity syndrome. A minimum battery of laboratory tests, such as liver transaminases,
complete blood count and urinalysis and serum creatinine, should be performed. Corticosteroids are usually
administered if symptoms are severe.
Patients with anticonvulsant hypersensitivity syndrome should avoid all aromatic anticonvulsants;
benzodiazepines, valproic acid (sodium valproate) or one of the newer anticonvulsants can be used for seizure
control.
However, valproic acid should be used very cautiously in the presence of hepatitis. There is no evidence that
lamotrigine cross-reacts with aromatic anticonvulsants. In addition, family counselling is a vital component of
patient management.

62. Ans. is c i.e. Hypercalcemia Ref. KDT 6/e, p 405

Phenytoin increase the metabolism of calciferol and produces vitamin-D deficiency. This leads to
rickets in malnourished children and osteomalacia in adults.
For more details, kindly see answer no. 27

63. Ans. is a i.e. Amantidine causes ankle oedema Ref. KDT 6/e, p 421

A characteristic side effect due to local release of catecholamines resulting in vasoconstriction, is


livedo reticularis and edema of ankles.
Amantidine is used in Parkinsonism.

Amantidine is a dopamine facilitator : It is an antiviral drug used for prophylaxis of influenza A2. It appears
to act by promoting presynaptic synthesis and release of dopamine in brain.
It acts rapidly but has lower efficacy than levodopa, though higher than anticholinergics.

64. Ans. is c i.e. Imipramine Ref. KDT 6/e, p 444

Imipramine Toxicity : Serious cardiovascular disturbances are frequently present with imipramine toxicity,
including tachycardia, cardiac arrhythmia (flutter, atriofibrillation, ventricular premature beats, and ventricular
tachycardia), as well as impaired myocardial conduction.

205
3 Self Assessment & Review Phar macology
Pharmacology
ECG abnormalities (such as widened QRS complexes and marked S-T shifts and signs of congestive heart
failure and cardiac arrest). Hypotension and initial hypertension may occur. However, the usual finding is increasing
hypotension which may lead eventually to shock. Coma may ensue.
The prominent symptoms of drug toxicity :
Imipramine
CNS CVS Respiratory Metabolic ECG changes
• Convulsion • Tachycardia • Depression • Acidosis • R-axis deviator
• Coma • Hypotension • QRS > 100m sec.
• Myoclonus • Arrhythmia i.e. prolonged

Patients signs and symptoms matches with toxicity of imipramine


Ethylene Glycol (Toxicity)
CNS CVS General Metabolic Lab finding
• Coma • Hypotension • Lethargy • Acidosis • Leucocytosis
• Seizures • Back pain • Increased • ↑ BUN &
anion gap Creatinine
• Proteinuria
• Oxalate
• Crystalluria

• Amanta phylloides – Main toxicity-Gastrointestinal


• Phenycyclidine – Toxicity symptoms resemble clinical features of schizophrenia

65. Ans. is d i.e. Oral morphine Ref. Harrison 16/e, p 58

For pain control in cancer patients


• Pharmacologic interventions follow the WHO three step approach (WHO ladder) involving :
Non opioids analgesics Mild opioids Strong opioids
• NSAIDs • Morphine • Increase the potency of opioids or
• Ibuprofen 1600mg/d 5-10mg/4hourly orally • Higher doses of morphine
If continous pain, regular is given
round the clock basis

66. Ans. is a i.e. Morphine Ref. KDT 6/e, p 45, 74, 61; Katzung 10/e, p 501

Morphine is contraindicated in head injury because :


• By retaining CO2 it ↑ intracranial tension which will add to that caused by head injury itself. Option ‘a’
• Even therapeutic doses can cause marked respiratory depression in these patient.
• Vomiting, miosis and altered mentation produced by morphine interfere with assessment of progress in
head injury cases

For more details, kindly see answer no. 24

206
Phar macology of Central Nervous System
Pharmacology 3
67. Ans. is b. i.e. Neuroleptic malignant syndrome Ref. Katzung 10/e, p 468

Neuroleptic malignant syndrome is a life threatening disorder that occurs in patients who are extremely
sensitive to the extrapyramidal effects of antipsychotic.
This syndrome is believed to result from an excessively rapid blockade of post synaptic dopaminergic receptors.

Features :
• Initial symptom is marked muscular rigidity.
• Fever may ensue often reaching dangerous levels (Hyperthermia).
• Autonomic instability and altered BP and pulse rate (is a mid brain manifestation).
• Stress leukocytosis.
• Elevated creatine kinase isoenzymes (due to muscle damage).
Treatment : Muscle relaxants such as dantrolene, or dopamine agonists such as bromocriptine.

68. Ans. is c i.e. Ebstein anomaly Ref. KDT 6/e, p 435 - 436

• Lithium causes : Ebstein anomaly, fetal goiter when used in a pregnant women.
• Isotretinon causes : Craniofacial defect, heart anomaly, abortion.
Drugs and their congenital malformation when taken in pregnancy are given below :
Drugs Congenital malformation
• Penicillamine • Cutis lexa
• Warfarin • Contradi syndrome
• Chlorambucil • Genitourinary anomaly (Renal agenesis)
• Valproic acid • Neural tube defect
• Carbamezapine • Cleft lip and palate, Cardiac defect
• Costicosteroid • Congenital heart disease, cleft lip and palate
• Barbiturates • Respiratory depression
• Carbimazole • Foetal Aplasia Cutis
• Lithium • Ebstein anomaly, Foetal goiter.
• DES • Vaginal adenosis (Clear cell Ca)
• Thalidomide • Phocomelia
• Chloramphenicol • Grey baby syndrome
• Androgens • Virilization; Limb, oesophagus and cardiac defects
• Isotretinoin • Craniofacial defect, Heart anomaly, Abortion
• Indomethacin • Premature closure of DA
• Tamoxifen • Risk of Abortion and Foetal damage

69. Ans. is d i.e. Isotretinoin Ref. KDT 6/e, p 54

The teratogenic effects mentioned in the question are characteristically produced by Isotretinoin.
• Craniofacial abnormality (cleft lip, cleft palate)
• CVS abnormality
• CNS abnormality.
– Also known as Accutane embryonopathy.

207
3 Self Assessment & Review Phar macology
Pharmacology
Ingestion of alcohol fetal alcohol syndrome which is characterized by triad of abnormalities.
• Craniofacial - Microcephaly and maxillary hypoplasia
• Prenatal and postnatal growth retardation
• Mental retardation.
ACE inhibitor usually cause renal damage in fetus.
Digitalis has no teratogenic effect.

70. Ans. is d. i.e. Beclomethasone Ref. KDT 6/e, p 386

The aldehyde dehydrogenase enzyme (ALDH) is inhibited by disulfiram and by other drugs including metronidazole,
oral hypoglycaemics, certain cephalosporines like cifotetan and cefoperazone and urinary anti antiseptics like
nitrofurantoin. When alcohol is consumed in presence of disulfiram, conversion of acetaldehyde to acetic acid
is stopped. Hence acetaldehyde accumulates to cause distressful effects comprising facial flushing, nausea,
vomiting, dizziness and headache.
This type of reaction is extremely unpleasant, but not life threatening, and hence disulfiram can be used as an
aversion therapy to discourage people from consuming alcohol.

The drugs causing Disulfiram like actions :

• Metronidazole • Chlorpropamide
• Citrated calcium carbamide • Flurazolidine
• Griseofulvin • Cephalosporins (Cefoperazone, moxalactam cefamondole)

71. Ans. is c i.e. Sodium valproate Ref. Katzung 10/e, p 391

Specific myoclonic syndromes are usually treated with valproate; an intravenous formulation can be used
acutely as needed.
Another specific myoclonic syndrome, juvenile myoclonic epilepsy, can be aggravated by phenytoin or
carbamazepine; valproate is the drug of choice followed by lamotrigine and topiramate.
For more details, kindly see answer no. 35 of All India of Chapter 3

72. Ans. is b i.e. Tolerance to all its actions except constipation and miosis Ref. KDT 6/e, p 457

Degree of tolerance that may develop to some of the effects of the opioids. ..... Katzung p 504
High Moderate Minimal or None
↓ ↓ ↓ ↓
Analgesia Respiratory depression Bradycardia Miosis
Euphoria, dysphoria Antidiuresis Constipation
Mental clouding Nausea and vomiting Convulsions
Sedation Cough suppression

Tolerance is exhibited to most of actions except to consipation, miosis and convulsive actions.
CNS Action :
• The analgesic action of morphine has spinal and supraspinal components. Perception of pain and reaction
to it are both altered.

208
Phar macology of Central Nervous System
Pharmacology 3
It acts in the substantia gelatinosa of dorsal horn.
• Morphine produces pronounced psychological and physical dependence, its abuse liability is rated high.

73. Ans. is a i.e. Brachycephaly Ref. Katzung 10/e, p 368 - 369; KDT 6/e, p 384 - 385

• Alcohol intake during pregnancy cause Fetal Alcohol syndrome. The abnormalities that have been characterized
as Fetal Alcohol syndrome include :

– Intrauterine growth retardation – Microcephaly


– Poor coordination – Minor joint anomalies
– Cardiac malformation (ASD, PDA) – Hyperkinetic movements
– Underdevelopment of midfacial region (appearing as a flattened face)

74. Ans. is a i.e. Imipramine Ref. Harrison 17/e, p 178

Imipramine should be used in children aged 6 years and over for nocturnal enuresis.
Base of bladder has primarily α-adenoreceptors while body has β-adrenoreceptor, nor-adrenaline stimulation
produced β-adrenoreceptor inhibiton of the tone of the bladder body and an α-adrenoreceptor constraction of the
base.
DOC of nocturnal enuresis is imipramine.
Before the age of 5 - 6 years, nocturnal enuresis is consider normal phenomenon.

Uses of Imipramine : • Panic attack • Enuresis • Cataplexy

It is not given in children < 6 years of age because of cardiovascular side effects.

75. Ans. is a i.e. MCV Ref. KDT 6/e, p 404

The only abnormality revealed by the investigation of the patient is anaemia (Hb level 8.2). All other serum
values are in normal range.
Phenytoin decreases folate absorption and increases its excretion leads to Megaloblastic anaemia. So
next regional investigation should be to check mean corpuscular volume (MCV), which in all probability
should be increased.

76. Ans. is d i.e. Migraine Ref. KDT 6/e, p 405 - 406

Carbamezapine not used in myoclonic and absence seizure.


• DOC Trigeminal neuralgia
• Most effective drug in Complex partial seizures
• DOC along with Phenytoin Generalized tonic clonic and simple partial seizures
• Bipolar disorder As an alternative to lithium
Manic depressive illness Also use in diabetes insipidus
and acute mania

209
3 Self Assessment & Review Phar macology
Pharmacology
77. Ans. is b i.e. Hypothyroidism Ref. KDT 6/e, p 416, 418, 436

Adverse effects of Lithium : • Tremor most common – Drug of choice propranolol


• GI intolerance
• Endocrinology hypothyroidism – Drug of choice Thyroxine
• During 1st trimester of pregnancy – Ebsteins anomaly.

78. Ans. is a i.e. Fluoxetine Ref. KDT 6/e, p 439

Antidepressant Drugs

Reversible inhibitors Selective serotonin Tricyclic antide Atypical Antidepressant


of MAO - A (RIMA’s) reputake inhibitors (SSRI’s) pressants TCA’s
• Maclobamide Fluoxetine MOA : ↑ NE or ↑ 5HT or both
• Clorgyline • Cardio Safe S/E : Priapism
• S/E : Nausea, vomiting Mianserin :
Fluvoxamine Cardio safe elderly with
Sertraline depression and CHD
Citalopram Tianeptine :
Paroxetine Selective serotonin reuptake
enhancer
Bupropion :
No weight gain, cessation of
smoking
↓ ↓
NA + 5HT Reuptake Inhibitors Predominantly NA Reuptake Inhibitor
Imipramine : DOC in – Panic attack Amoxapine : Antidepressant with antipsychotic
– Enuresis property can have tardive dyskinesia
– Cataplexy Desipramine
Amitriptyline : DOC in – Depression with multiple Nortriptyline
somatic complaints
– Neurological pain
(diabetes)
Cloimpiramine : DOC in – OCD
Doxepin : – Peptic ulcer
– Disease with depression

79. Ans. is b i.e. Fluoxetine Ref. KDT 6/e, p 475

Dextropropoxyphene > Pentazocin > Diagepam


Drugs of Abuse that Engender Dependence

210
Phar macology of Central Nervous System
Pharmacology 3
Abuse potential

Fluoxetine is Cardio safe antideprassant (SSRI)


Most common side effects are : Nausea, vomiting, GI disturbance and weight loss.

80. Ans. is a i.e. Haloperidol Ref. KDT 6/e, p 431

Drugs causing extrapyramidal effects :


Mnemonics :
Extra PMT Require fOr MLA

• Extrapyramidal • Phenothiazines • Reserpine • OCPs • M Dopa


• Metoclopromide • L Dopa
• Tricyclic antidepressants • Antipsychotic

81. Ans. is b i.e. Pentazocine Ref. Rang & Dale 5/e, p 575; KDT 6/e, p 464 - 465

Already explained, refer answer no. 43

82. Ans. is c and d i.e. Carbamazepine-Na+ channel blocker; and Lamotrigine - Na+ channel blocker
Ref. Goodman & Gilman 11/e, p 523; KDT 6/e, p 410

Vigabatrin acts via inhibiting GABA transaminase enzyme which degrade GABA which is a inhibitory
neurotransmitter.
Newer Anticonvulsant Drugs :
Drugs Possible Mechanism Indications Toxic Potential
Vigabatrin Inhibits GABA transaminase Partial seizures Depression, psychosis,
visual dysfunction
Gabapentin ↑ GABA effect Dipolar disorder Sedation, ataxia,
neuropathic pain cognitive change.
Tiagabine Blocks GABA transporter Partial seizures Sedation, dizziness,
“Flu-Like” symptoms
Lamotrigine Blocks Na channels and Absence and partial Sedation, ataxia,
glutamate receptors seizures diplopia, Stevens-
Topiramate Blocks glutamate (AMPA) Partial seizures Renal stones
receptors, ↑ GABA effects.
Folbomate Blocks Na+ and Ca++ Partial seizures Aplastic anemia, acute
channels, blocks glutamate liver failure
receptors.

211
3 Self Assessment & Review Phar macology
Pharmacology
83. Ans. is b i.e. ↑ EPS clincial features Ref. KDT 6/e, p 429; CMDT ‘07, p 1086; Harrison 17/e, p 2722

Risperidone is a antipsychotic activity has been ascribed to a combination of D2 + 5-HT2 receptor blockade. In
addition it has high affinity for α1, α2 and H1 receptors : blockade of these may contribute to efficacy as well as
side effects like postural hypostension.

Disadvantage : • EPS in high doses • Weight gain


• Gynaecomastia / ↑ prolactin • Postural hypotension
• Less epileptogenic than clozapine • Agitation

Extrapyramidal side effect : Acute dystonia, Parkinsonism’s, Tardine dyskinesia.

84. Ans. is b i.e. 10 - 29 μg / ml Ref. Katzung 10/e, p 377; KDT 6/e, p 405

The therapeutic plasma level of phenytoin for most patient is between 10 and 20 μg/ml.
Therapeutic level of some important drugs characterized by their low therapeutic index.

• Digoxin – 0.8 - 2 ng/ml


• Clonidine – 0.2 - 2.0 ng/ml
• Lithium – 0.6 - 1.2 mEq/L (for prophylaxis)
– 0.8 - 1.2 mEq/L (for maintenance)
• Theophyline – 5 - 20 μg/ml
• Quinidine – 2 - 6 μg/ml
• Valproic acid – 50 - 100 μg/ml

85. Ans. is b and e i.e. Action not reversed by flumazenil; and Good muscle relaxation
Ref. KDT 6/e, p 398; Katzung 10/e p 356

Non benzodiazepine hypnotics : – Zopiclone – Zolpidem – Zalpelon


Zolpidem
• Iminopyridine derivative that selectively acts on ω -1 subtype of BZD receptor. Option ‘a’
• Like benzodiazepines action are not antagonized by flumazenil. Option ‘b’ wrong
Pronounced hypnotic effect. Option ‘d’
Minimal anticonvulsant, muscle relaxation, antianxiety effect. Option ‘e’ wrong

• Used for : – Short term insomnia.


– They not antagonize the central nervous system effects of sedative-hypnotics, ethanol
opioids or general anaesthesia.
• Advantage : – Minimal effect on sleep stages and minimal day time sedation
– No / little rebound insomnia on discontinuation
– Absence of tolerance or physical dependence.
• Kinetics : – Short duration of action
– t½ - 2.5 hours and t½ of Diazepam - 20 - 80 hours. (Option ‘c’)

212
Phar macology of Central Nervous System
Pharmacology 3
86. Ans. is c and d i.e. Fomepizole; and Ethyl alcohol Ref. KDT 6/e, p 387

The antidote of Ethylene glycol poisoning is Fomepizole and Ethyl alcohol.

Ethylene glycol

Oxidation ↓ Alcohol dehydrogenase ← Fomepizole
Glyceraldehyde

Glycolic acid Glyoxalic acid


CNS depression ↓
↑ with anion gap)
Met. Acidosis (↑ Oxalic acid + Glycine
Tubulointerstitial damage Hypocalcaemia

Ethylene Glycol (Toxicity)


CNS CVS General Metabolic Lab finding
• Coma • Hypotension • Lethargy • Acidosis • Leucocytosis
• Seizures • Back pain • Increased • ↑ BUN &
anion gap Creatinine
• Proteinuria
• Oxalate
• Crystalluria

Ethyl alcohol is also metabolised by alcohol dehydrogenase with higher affinity than ethylene glycol. So, in
ethylene glycol poisoning, ethyl alcohol divert the metabolism of ethylene glycol leading to the reduction in
formation of toxic metabolites.

87. Ans. is b and c i.e. Sodium valproate; and Lamotrigine Ref. KDT 6/e, p 404

Already explained, refer answer no. 2

88. Ans. is a, b, d and e i.e. Ethyl alcohol is used; Formation of formic acid produces blindness; Gastric
lavage done; and Fomepizole inhibits the formation of formic acid
Ref. KDT 6/e, p 386; Goodman & Gilman 11/e, p 599; Harrison 16/e, p 2591

• Ethanol : Ethanol has 100 fold greater affinity for alcohol dehydrogenase than methanol. So, the formation
of formic acid and formaldehyde is prevented. (Option “a”)
Ethanol concentration should be 1gm/liter. For this, the loading dose of 6g/kg is given as soon
as possible and then, the continuous infusion of 10gm/hour.
• Folate Therapy – Folic acid enhance the metabolism of formic acid and formate to CO2 + H2O (Calcium
leucovorin). Acetaldeyde is main toxic product in ethanol Poisoning. (Option “b”)
Treatment :
• Keep patient in quiet and dark room to protect the eyes from light.
• Gastric lavage – by sodium bicarbonate

213
3 Self Assessment & Review Phar macology
Pharmacology
• I.V. sodium bicarbonates – Correct acidosis
• Potassium chloride – If hypokalemia occurs
• 4 – methyl pyrazole (Fomepizole) – Specific inhibitor of alcohol dehydrogenase.
• Folate Therapy – Folic acid enhance the metabolism of formic acid and formate to CO2 + H2O (Calcium
leucovorin).
• Hemodialysis : Indications :
– Acidosis even after giving ethanol therapy
– Methanol blood level > 500mg/liter
– Osmolar gap > 10mosm/l.

The main toxic product in methanol poisoning is formaldehyde and formic acid.
Toxic Dose : 15 ml → Caused blindness
75 ml → Fatal dose
Clinical Symptom : 1. Ist 6 - 18 hours – Direct depressant action on CNS.
2. After 6 - 18 hours – Symptoms due to formic acid.

89. Ans. is a, b, d and e i.e. VR-1; Nicotinic cholinergic; Nociception pattern; and Anandomide
Ref. Ganong 21/e; Rang & Dale 5/e, p 568, 609

A VR-1 vanilloid receptors / VRL-I are new receptors that are associated with pain mechanism. VR-I
produces pain with capsicin but VRL-I does not. VR-I is clearly a nociceptor while VRL-I is probably a
receptor. Nociceptors mediate potentially harmful stimuli such as pain, extreme heat and extreme cold.
• Nicotinic cholinergic mechanism is involved in the regulation of pain as the analgesic effect of nicotine is
reduced in mice lacking α4 and β2 nicotinic cholinergic receptor subunits.
• There are anandamide containing neurons in the periaqueductal grey mater and other areas concerned the
pain. Anandamide have definite analgesic effects.

214
Phar macology of Central Nervous System
Pharmacology 3
90. Ans. is a i.e. Gum hypertrophy Ref. KDT 6/e, p 404

Chronic Toxicity : Gingival hyperplasia and coarsening of facial features; because phenytoin after being
secreted through saliva, inhibits the enzyme collagenase which is responsible for the
breakdown of collagen of connective tissue.
For more details, kindly see answer no. 27

91. Ans. is d i.e. Does not increases histamine release Ref. Katzung 9/e, p 512

Difference between Morphine and Dezocine :


Morphine Dezocine
• Less potent • More potent
• Slower acting • Faster acting
• Opioid agonist • Opioid agonist / antagonist
• ↑ Histamine release • Less histamine release
• More hypotensive • Less hypotensive

92. Ans. is a, b and e i.e. Citalopram; Fluoxetine; and Sertraline Ref. KDT 6/e, p 439

Classification of Antidepressant Drugs


• NE and 5-HT Reuptake inhibitor – Imipramine – Clomipramine
– Amitriptyline – Doxepin
• NE Reuptake inhibitor – Desipramine – Nortriptyline
– Protriptyline – Maprotiline
– Amoxapine – Lofepramine
• Selective Serotonin Reuptake – Sertraline – Fluoxetine
Inhibitors (SSRIs) – Fluvoxamine – Paroxetine
– Citalopram
• Atypical Antidepressants – Trazodone – Nefazodone
– Bupropion – Mirtazapine
– Mianserin – Venlafaxine
• MAO Inhibitors – Non-selective (MAO-A & MAO-B inhibitors) : Tranylcypromine
– Selective MAO-A Inhibitors : Moclobemide

93. Ans. is a, c and e i.e. Levodopa; Bromocriptine; and Benserazide Ref. KDT 6/e, p 415

Classification of Drugs used in Parkinson are :


1. Drugs affecting brain dopaminergic system :
• Dopamine precursor : Levodopa (L-dopa)
• Peripheral decarboxylase inhibitors : Carbidopa, benserazide
• Dopaminergic agonists : Bromocriptine, Pergolide, Peribedil, Robinirole, Pramipexole
• MAO-B inhibitor : Selegiline
• COMT inhibitors : Entacapone, Tolcapone
• Dopamine facilitator : Amantadine

215
3 Self Assessment & Review Phar macology
Pharmacology
2. Drugs affecting brain cholinergic system
• Central anticholinergics : Trihexyphenidyl (Benzhexol), Procyclidine, Biperiden.
• Antihistaminics : Orphenadrine, Promethazine.

94. Ans. is a and d i.e. Chloroform; and Halothane Ref. Various books

Hepatotoxic drugs are : • Halothane • Chloroform • Niacin


• Carbon tetrachloride • Statins

Other options : Ether – Sensitize heart to Adr and not hepatotoxic.


N2O – Is nontoxic to liver, kidney, brain.
Enflurane – eliminated mostly via lungs, contraindicated in epilepsy. It causes adrenal
suppression, used in pheochromocytoma.

95. Ans. is None Ref. Goodman & Gilman 11/e, p 564, 567; KDT 6/e, p 456 - 458

Option ‘a’ : Although it is mentioned that the more lipophilic opioids are absorbed through nasal or buccal
mucosa

but inhalation is not an established route of morphine administration.
Option ‘b’ : Few agents (e.g. morphine, hydromorphone) are available in suppositories.

Absorption through rectal mucosa is adequate

Other important routes of morphine administration


– Subcutaneous (SC) – Intravenous (IV) – Intramuscular (IM)
– Transdermal – Epidural/Intrathecal – Rectal
– Oral

• Morphine is very rarely used orally because of :


– Unreliable oral absorption
– High and variable first pass metabolism

Oral bioavailability is ¼ of parenterally administered drug.

96. Ans. is d and e i.e. Midazolam; and Pethidine Ref. KDT 6/e, p 397, 393; Harrison 17/e, p 2439

Acute intermittent Porphyria : Barbiturates exacerbate it by inducing microsomal enzymes and δ amino
levulinic acid synthetase and increasing porphyrin synthesis.

Contraindications of barbiturates : • Liver and kidney disease


• Severe pulmonary insufficiency e.g. emphysema
• Obstructive sleep apnoea.

216
Phar macology of Central Nervous System
Pharmacology 3
Categories of unsafe and safe drugs in AIP, HCP and VP (Hereditary coproporphyria, variegated porphyria).
... Harrison 17/e, p 2439

SAFE • Narcotic analgesics • Aspirin • Acetaminophen


• Phenothiazines • Penicillin derivatives • Streptomycin
• Glucocorticoids • Bromides • Insulin
• Atropine

97. Ans. is b, c, d and e i.e. Synthetic derivative; Has alfa - blocking action; Decrease GI motility; and Acts
on both D1 and D2 receptors Ref. KDT 6/e, p 236

• Bromocriptine is a synthetic ergot derivative, 2 - bromo - α - ergocryptine.


• It is a potent dopamine agonist.
• It has greater action on D2 receptors, while at certain dopamine sites in brain, it acts as a partial agonist or
antagonist of D1 receptor.
• It is a weak α blocker.

Features of Bromocriptine :
Action • Decrease prolactin release from pituitary-strong antigalactoopoietic
• Increase growth hormone release in normal individuals
• Decrease growth hormone release from pituitary tumours
• Has levodopa like actions in CNS - antiparkinsonian effects
• Produces nausea and vomiting by stimulating CTZ
• Decreases gastrointestinal motility (D2 inhibition)
• Hypotension
Uses : • Hyperproteinemia
• Acromegaly (inhibition growth hormone)
• Parkinsonism (D2 agonism)
• Hepatic coma
• Suppression of lactation and breast engorgement
• Infertility
Side effects • Nausea, vomiting
• Postural hypotension
• Mental confusion
• Psychosis

98. Ans. is b and c i.e. Phenothiazine; and Haloperidol Ref. Harrison 17/e, p 2553

The other three options given i.e. Bromocriptine, Amantadine and Carbidopa are used in the treatment
of parkinsonism.

Drugs causing parkinsonism

• Phenothiazines • Butyrophenones e.g. Haloperidol


• Metoclopramide • Lithium carbonate
• Reserpine • Alpha methyldopa
• Valproic acid • Fluoxetine

217
3 Self Assessment & Review Phar macology
Pharmacology
99. Ans. is a, c, d and e i.e. It is an opioid antagonist; Used to treat alcohol dependance; and Used to treat
opioid dependance and Used as a respirator stimulant
Ref. KDT 6/e, p 464, 467; Katzung 10/e, p 494

The mixed agonist-antagonist opiates produce negligible euphoric effects. However, other receptor-mediated
effects are similar to those of morphine. Therefore, the physical manifestations of opiate withdrawal are minimized,
and psychologic dependence is not exacerbated.
The agonist-antagonist receptor properties of the nalorphine-type drugs act as agonists at k receptors and
antagonists at m receptors. However, the level of potency of these drugs is generally less than that produced by
a pure agonist or antagonist. Therefore, because these drugs are competitive in binding, they may act as
partial agonists or antagonists as well.

Naltrexone is a pure opioid antagonist.


Naltrexone differs from Naloxone in 3 points
• Naltrexone is orally active
• Naltrexone has a long duration of action ( 1- 2 days)
• Naltrexone is more potent.

Options “c”, “d” and “e” relate to use of naltrexone and the USES are :
• To treat opioid dependance.
– It is used to treat acute intoxication as well as maintenance therapy due to long duration of action.
– It is used for ‘Opioid blockade’ therapy of post-addicts.
• To treat alcohol dependance.
– It prevents relapse of alcoholism, reduce alcohol craving and number of drinking days - thus approved by
US -FDA in comprehensive treatment.
• As a respiratory stimulant.
– It is used to reverse respiratory depression due to intraoperative use of opioids.
– Other respiratory stimulants are Doxapram, Nikethamide, and Ethyl and propyl butamide.
• For reversing neonatal asphyxia due to opioid use during labour.
– Dose is 10 ug/kg injected in the cord.

Respiratory stimulant : • Doxapram


• Nikethamide
• Ethyl/propyl butamide

Here, it is mandatory to revise classification of opioid antagonists.


Agonist-Antagonists Partial / Weak Agonist Pure Antagonists
• Not used as analgesics – Buprenorphine – Naloxone
– Nalorphine – Dextropropoxyphene – Naltrexone
• Used as analgesics – Nalmefene
– Pentazocaine
– Nalbuphine
– Butorphanol

218
Phar macology of Central Nervous System
Pharmacology 3
100. Ans. is c and d i.e. Diazepam causes respiratory depression; and Nitrazepam is metabolised in liver
Ref. KDT 6/e, p 393 - 394

Let’s look at each option individually.


Option ‘a’ : Benzodiazepines (BZDs) act by enhancing presynaptic /postsynaptic inhibition through a specific
BZD receptor which is an integral part of the GABAA – BZD receptor, increases the frequency of
Cl- channel opening induced by submaximal concentration of GABA. Thus, it is not a GABA
agonist. It has GABA facilitatory action.

Option ‘b’ : Diazepam is a long acting benzodiazepine as follows :


Long acting BZDs Short acting BZDs Ultra short acting BZDs
a. Chlordiazepoxide a. Alprazolam a. Estazolam
b. Clonazepam b. Flunitrazepam b. Midazolam
c. Diazepam c. Lorazepam c. Temazepam
d. Flurazepam d. Oxazepam d. Triazolam

Option ‘c’ : “Coma, respiratory depression though rare, can occur with ultra short acting agents like
Midazolam, Triazolam.” ..... Harrison 15/e
Option ‘d’ : Benzodiazones are metabolised in liver by dealkylation and hydroxylation to many metabolites,
some of which may be active.
Option ‘e’ : The dependence producing liability of BZDs is low. They are seldom abused alone. Drug abusers
find them bland except rapidly absorbed midazolam which has higher abuse potential.

101. Ans. is b, c, and d i.e. Fluoxetine; Fluvoxamine; and Sertraline Ref. KDT 6/e, p 445 - 446

Options “a” and “e” i.e. Fluphenazine and Fluphenthixol are antipsychotic drugs and not antidepressant
drugs.

Selective Serotonin Reuptake Inhibitors (SSRIs) : – Sertraline – Fluoxetine


– Fluvoxamine – Paroxetine
– Citalopram

Advantages of SSRIs over TCAs


• Produce little or no sedation.
• Do not interfere with cognitive or psychomotor functions.
• Do not produce anticholinergic side effects.
• Are devoid of a adrenergic blocking action-postural hypotension does not occur.
• Have no seizure precipitating propensity.
• Do not inhibit cardiac conduction-overdose arrhythmias are not a problem.

102. Ans. is c and d i.e. INH; and Procainamide Ref. Various book

Both Procainamide and INH causes SLE like syndrome by inducing antinuclear antibodies (ANA).

219
3 Self Assessment & Review Phar macology
Pharmacology
Please remember the drug causing SLE like syndrome (Mnemonic - Plz Check BOM In SHIP).

Plz Procainamide In Interferon α


Check Chlorpropamide S Sulphonamide
B Barbiturates H Hydralazine
O OCP I INH
M Methyl Dopa P Phenytoin-Penicillamine

103. Ans. is b, d and e i.e. Disulfiram; Naltrexone; and Acamprosate Ref. Harrison 17/e, p 2728

This question has been directly picked up from Harrison’s 17/e, p 2728 which says : “While the mainstay of
alcoholic rehabilitation involves counselling, education, and cognitive approaches, several medications might
be useful as follows :
Naltrexone Acamprosate Disulfiram
– An opioid antagonist – Given 2 mg/day. – Given 250mg/day.
– Given 50 - 100 mg/day. – Results similar to naltrexone – Inhibits aldehyde dehydrogenase
– Decrease the probability of – Produces unpleasant side
return to drinking effects in presence of alcohol.
– Short term periods of
rehabilation

104. Ans. is d i.e. Tyramine Ref. KDT 6/e, p 440

• Certain varieties of cheese, beer, wines, pickled meat and fish contain large quantities of tyramine, dopa etc.

• In MAO inhibited patients, these indirectly acting sympathomimetic amines in cheese etc. escape degradation
in the intestinal wall and liver.

• They reach the systemic circulation and displace large amounts of NA from transmitter loaded adrenergic
nerve endings.

• Hypertensive crisis and cerebrovascular accidents occurs.
• Cough remedies, Guanethidine and Reserpine when given along with MAO also lead to hypertensive reaction,
but Cheese contains tyramine, Dopa and cheese is due to Tyramine.

105. Ans. is a and c i.e. G protein; and PIP / DAG Ref. KDT 6/e, p 48, 349

GABAA GABAB
is ligand gated ion channel is G-protein coupled receptor

220
Phar macology of Central Nervous System
Pharmacology 3
106. Ans. is b i.e. Rapid redistribution Ref. KDT 6/e, p 375 - 376, 389 - 391

There are three process involved in termination of action of barbiturates.


Redistribution – It is important in the case of highly lipid soluble thiopentone and other ultrashort action
barbiturates.
– After their I.V. injection, consciousness is regained in 8 - 12 minutes due to redistribution
while the ultimate disposal occurs by metabolism (t½ of elimination phase is 9 hours).
Metabolism – Drugs with intermediate lipid solubility (short acting barbiturates) are primarily
metabolized in liver by oxidation, dealkylation and conjugation.
– Their plasma t½ ranges from 12 - 40 hours.
Excretion – Barbiturates with low lipid solubility (long acting agents) are significantly excreted
unchanged in urine.
– The t½ of phenobarbitone is 80 -120 hours.

107. Ans is a i.e. 5-HT2 receptor blocker Ref. KDT 6/e, p 167

Regarding ketanserin
• It is a selective 5-HT2 receptor blocker.
• Among 5-HT2 receptors, blockade of 5-HT2A is stronger than 5-HT2C blockade.
• 5-HT induced vasoconstriction, platelet aggregation and contraction of airway smooth muscle are antagonised.
• It is an effective antihypertensive.
• It has also been shown to be effective in Raynaud’s disease.
• Side effects are mild : Dizziness, tiredness, nausea.

108. Ans. is a and b i.e. Glucuronide conjugation; and Oxidation, dealkylation Ref. KDT 6/e, p 389 - 391

Short acting barbiturates (Drugs with intermediate lipid solubility) are primarily metbolized in liver by oxidation,
dealkylation and conjugation.

Classification of Barbiturates according to their duration of action :


Long acting Short acting Ultra short acting
• Phenobarbitone • Butobarbitone • Thiopentone
• Mephobarbitone • Secobarbitone • Methohexitone
• Pentobarbitone • Hexobarbitone

109. Ans. is c i.e. Acts on Cl–channel complex Ref. KDT 6/e, p 391, 395

• Barbiturates act primarily at the GABA : BZD receptor - Cl- channel complex and potentiate GABA ergic
inhibition by increasing the life time of Cl- channel opening induced by GABA.
• These do not bind to the BZD receptor, but bind to another site (probably the picrotoxin sensitive site) on the
same macromolecular complex to exert the GABA facilitatory action.

221
3 Self Assessment & Review Phar macology
Pharmacology

Barbiturates Action

• At high concentration • CI– conductance (GABA mimetic). Inhibit Ca++


dependent release of neurotransmitters.
• At very high concentration • It depress Na+ and K+ channels.

110. Ans. is a i.e. Barbiturate poisoning Ref. KDT 6/e, p 26, 392

• Alkalinisation of urine increases ionisation and excretion of barbiturates and other long acting agents.
• Alkalinisation of urine is also helpful in excretion of acidic drugs as salicylates. On the other hand, urine
is acidified in poisoning due to alkaline drugs such as morphine and amphetamine.

222
Phar macology of Central Nervous System
Pharmacology 3
CHAPTER REVIEW
• This section includes questions of V arious Other PGMEES from 1990 – 2008.
Various
• Questions are ar ranged in increasing order of page sequence of KDT 6 Edition. This is
arranged
done to mak e refer
make ral system more easy and uncomplicated to save the precious time
referral
of PGMEE Aspirant.

1. Ketamine is contraindicated in : a. GABAA agonist


a. Hypertension (Manipal 07) b. Central a2 agonist
b. Bronchial asthma c. Anti-muscaranic agent
c. Hypovolaemia d. GABA B agonist [Ref. KDT 6/e, p 349]
d. All of the above 7. NSAIDs which does not inhibit prostaglan’din’
[Ref. KDT 6/e, p 376] synthesis : (Manipal 06)
2. Morphine cannot be used in all the following con- a. Sulindac
ditions except : (Manipal 06) b. Nefopan
a. Head injury c. Ketorolac
b. Asthma d. Mefenamic acid
c. Hypothyroidism [Ref. KDT 6/e, p 199]
d. Diabetes 8. Mechanism of action Tianeptin in the brain is :
[Ref. KDT 6/e, p 457] a. Selective scrotonin reuptake inhibition
3. Clinically significant drug interaction occurs be- b. Selective norepinephrine reuptake inhibtion
tween pyrodoxine and all the following drugs ex- c. Selective serotonin reuptake enchancer(MAHA
cept : d. Selective dopamine reuptake inhibition 05)
a. Isoniazid (Manipal 06) [Ref. KDT 6/e, p 445]
b. Cyclosporins 9. Moclobemideis : (APPG 06)
c. Levodopa a. SSRI
d. Hydralazine b. Antipsychotic drug
[Ref. KDT 6/e, p 876] c. MAO inhibitors
4. Which of the following is the best indication for d. Prevent recyclation of adrenaline
propofol as an intravenous induction agent ? [Ref. KDT 6/e, p 440]
a. Neurosurgery (Manipal 06) 10. Which of the following is true regarding olazapint
b. Day care surgery except ? - (Manipal 06)
c. Patients with coronary artery disease a. It caused marked anti-muscaranic action
d. In neonates b. It causes weight gain
[Ref. KDT 6/e, p 375] c. It cause mild increases in prolactin
5. AH are side effects of phenytoin except : d. Less epileptogenic action than phenothiazine
a. Hirsutism (NINHAMS 06) [Ref. KDT 6/e, p 429]
b. Gum hypertrophy 11. All are side effects of valproic acid except :
c. Ataxia a. Polycystic ovaries (NIMHANS 06)
d. Hypoglycemia b. Fulminant hepatitis
[Ref. KDT 6/e, p 404 - 405] c. Tremor
6. Tiazinadine is : (Manipal 06) d. Ataxia [Ref. KDT 6/e, p 407]

Answer 1. a. Hypertension 2. d. Diabetes 3. None 4. b. Day care ... 5. d. Hypogly ...


6. b. Central ... 7. b. Nefopan 8. c. Selective ... 9. c. MAO ... 10. d. Less ...
11. a. Polycystic ...

223
3 Self Assessment & Review Phar macology
Pharmacology
12. Carbamazepine in elderly causes : (COMED 06) c. Benzodiazepenes block glutamate receptors in
a. Hypernatremia hierarchical neural pathways in the brain
b. Hyponatremia d. They increase the frequency of opening of chlo-
c. Hyperkalemia ride ion channels that are coupled to GABA re-
d. Hypokalemia ceptors
[Ref. KDT 6/e, p 405 - 406] [Ref. KDT 6/e, p 41]
13. Which of the following drugs exerts its antisei- 19. For treatment of cognitive symptoms of demen-
zures activity mainly as a result of interference tia, the drug most effective is : (COMEDK 05)
with ion conductance through sodium channels, a. Serotonergic drugs
which results in inhibition of high frequency re- b. Dopaminergic drugs
petitive firing of neurons : (MAHA 05 c. Catecholamines
a. Carbamazepine d. Cholinesterase inhibitors
b. Clonazepam [Ref. KDT 6/e, p 101]
c. Ethosuximide 20. Sedatives that can be safely used in the elderly :
d. Gabapentin a. Phenobarbitone (Manipal 04)
[Ref. KDT 6/e, p 405] b. Midazolam
14. Drug of choice in simple partial seizure is : c. Triazolam
a. Carbamezapine (SGPGI 05) d. All of the above
b. Phenobarbitone [Ref. KDT 6/e, p 397]
c. Diazepam 21. Prolonged allopurinol therapy in a patient with
d. Phenytoin gout is not indicated for : (UPSC 04)
[Ref. KDT 6/e, p 411] a. Acute gouty arthritis
15. Mirtizapine is useful in : (Manipal 05) b. Tophi
a. Depression c. Urate nephropathy
b. Insomnia d. Evidence of bone/joint damage
c. Schizophrenia [Ref. KDT 6/e, p 205 - 206]
d. Epilepsy 22. In brain the breakdown of L-DOPA is inhibited by :
[Ref. KDT 6/e, p 439] a. Deprinyl (Manipal 03)
16. ‘Dantrolene’ is useful in all except : (Manipal 05) b. Pyridoxine
a. Malignant neuroleptic syndrome c. Haloperidol
b. Malignant hypertension d. Bromocriptine
c. Malignant hyperthermia [Ref. KDT 6/e, p 420]
d. UMN disorders 23. Selective reuptake inhibition of serotonin and not
[Ref. KDT 6/e, p 347] norepinephrine is caused by which of the follow-
17. All except one are anti-depressants :(Manipal 05) ing:
a. Reserpine a. Protryptilline (Manipal 03)
b. Trazodone b. Fluoxetine
c. Imipramine c. Meprotriline
d. Moclobemide d. Amytriptiline
[Ref. KDT 6/e, p 439] [Ref. KDT 6/e, p 439]
18. Which of the following statements best describes 24. Drug used in case of chronic alcoholics to aid their
mechanism of action of benzodiazepenes : rehabilitation : (Manipal 03)
a. Benzodiazepenes activate GABA receptor in the a. Dysulfiram
spinal cord (Karnat 05) b. Sucralfate
b. Their inhibition of GABA transminase lead to in- c. Thiamine
creased levels gaba d. Methanol
[Ref. KDT 6/e, p 386]

Answer 12. b. Hyponatremia 13. a. Carbamaz ... 14. a. Carbame ... 15. a. Depression 16. b. Malignant ...
17. a. Reserpine 18. d. They incr ... 19. None 20. c. Triazolam 21. a. Acute ...
22. a. Deprinyl 23. b. Fluoxetine 24. a. Dysulfiram

224
Phar macology of Central Nervous System
Pharmacology 3
25. DOC for complex partial seizures : (Manipal 03) c. Useful in spasticity of cerebral origin
a. Haloperidol d. Very useful in parkinsonism
b. Imipraine [Ref. KDT 6/e, p 349]
c. Alprazolam 33. Vigabatrin acts as : (Manipal 00)
d. Carbamazepine a. GABA transminase inhibitor
[Ref. KDT 6/e, p 406] b. GABA antagonist
26. Aspirin is contraindicated in: (Manipal 02) c. GABA mimetics
a. Dysmenorrhea d. None of the above
b. Viral infection [Ref. KDT 6/e, p 410]
c. Ischemic heart disease 34. A-parkinsonism patient treated with combination
d. Inflammation of levodopa and carbidopa therapy patient develop
[Ref. KDT 6/e, p 189] late onset grimacing and facial tics probable cause
27. Regarding meperidine all are true except : is : (Up 00)
a. Drug dependence (Manipal 02) a. Ataxia
b. Houring short half life than morphine b. Dementia
c. More spasmogenic than morphine c. Tremor
d. Action is antogonise by naloxone d. Chorio-athetoid movement
[Ref. KDT 6/e, p 458] [Ref. KDT 6/e, p 418]
28. Dantrolene acts by : (Manipal 01) 35. True about cyclo-oxygenase-II inhibitor except :
a. GABA-A inhibitor a. Less GI irritation (Up 00)
b. GABA - B inhibitor b. Produces prostaglandins at the site of inflam-
mation
c. Direct action on Ca++ channel
c. Constructive enzyme COX I indu..... COX II
d. Uncopules contraction from depolarization of
muscle membrane d. Glucocorticoids regulates it
29. Selective COX-II inhibitor is : (J & K 01) [Ref. KDT 6/e, p 185 - 186]
a. Diclofenac 36. NSAID induced gastric irritation prevention by :
b. Mefanemic acid a. Pirenzepine (Up 00)
c. Nimesulide b. Roxatidine
d. Sulindac c. Mesopristol
[Ref. KDT 6/e, p 196] d. None
30. Which is a 5-HT receptor agonist ? (Orissa 00) [Ref. KDT 6/e, p 111]
a. Ondansetron 37. Malignant hyperthermia caused by : (Up 00)
b. Ketanserine a. Suxamethonium
c. Sumatriptan b. Gallamine
d. Risperidone c. Atracurium
[Ref. KDT 6/e, p 163, 170 - 171, 167] d. d-Tubocurarine [Ref. KDT 6/e, p 347]
31. Metabolism of ethanol is by : (Manipal 00) 38. Which of the following is not seen during extradu-
ral block : (Up 00)
a. First order kinetic
a. Hypotension
b. Zero order kinetic
b. Headache
c. Plasma cone depende
c. Backache
d. Microsomal enzymes in the liver
d. Meningitis
[Ref. KDT 6/e, p 383]
39. Following are used preanesthetic agent except :
32. Baclofen : (Manipal 00)
a. Morphine (Up 00)
a. Reduces the release of excitatory transmitters
in spinal cord b. Hyosine
b. Reduces the release of excitatory transmitters c. Neostigmine
in brain d. Scopolamine [Ref. KDT 6/e, p 378 - 379]

Answer 25. d. Carbama ... 26. b. Viral ... 27. c. More ... 28. d. Uncopules ... 29. c. Nimesulide
30. c. Sumatriptan 31. b. Zero order ... 32. a. Reduces ... 33. None 34. d. Chorio-...
35. b. Produces ... 36. c. Mesopristol 37. a. Suxametho... 38. a. Hypotension 39. c. Neostigmine

225
3 Self Assessment & Review Phar macology
Pharmacology
40. Increase intracranial tension occurs with : 48. Which anticonvulsant is safe during pregnancy :
a. Ketamine (Up 00) a. Valproic acid (AIIMS May 93)
b. Halothane b. Carnamazepine least
c. Ether c. Ethosuximide
[Ref. KDT 6/e, p 376] d. Phenytoin max
41. Hallucination are seen in : (Up 00) [Ref. KDT 6/e, p 316]
a. Ketamine 49. Malignant hyperthermia is seen with :
b. Halothane a. Gallamine (AIIMS May 93)
c. Ether b. Lignocaine
d. Thiopentone c. Scoline (SCH.
[Ref. KDT 6/e, p 376] d. Bupivacaine
42. Selective 5-HT re uptake blocker is : [Ref. KDT 6/e, p 156]
a. Desipramine (AIIMS May 94) 50. Tolerance occur to all side effects of morphine,
b. Amitryptiline except : (AIIMS May 93)
c. Fluoxetine a. Sedation
d. Dothiepin b. Constipation
[Ref. KDT 6/e, p 419] c. Pain relieving
43. Pain is caused by injection of : (AIIMS May 94) d. Euphoric effect
a. Ketamine [Ref. KDT 6/e, p 436]
b. Methohexitone 51. Antagonist o Benzodiazepine is : (AIIMS May 93)
c. diisopropyl alcohol a. Nalorphine
d. Thiopentone b. Carbamazepine
[Ref. KDT 6/e, p 357] c. Naloxone
44. All are true regarding …… except : (AIIMS Nov. 93) d. Flumazenil
a. Respiratory depression is a side eccect [Ref. KDT 6/e, p 399 - 400]
b. More potent than aspirin 52. Flushing on consumption of alcohol seen with :
c. Effect is prolonged a. Sulfonamide (AIIMS Dec. 92)
d. This is an analgesic b. Metonidazole
45. Hemodialysis is useful in poisoning with all, e x - c. Phenothiazine
cept : (AIIMS Nov. 93) c. Penicillin
a. Salicylate [Ref. KDT 6/e, p 799]
b. Methyl alcohol 53. Hallucinations are caused by : (AIIMS Dec. 92)
c. Diazepam a. Ethionamide
d. Barbiturates b. Chlorpromazine
[Ref. KDT 6/e, p 529] c. Ketamine
46. All are predominant arterior dilater, except : d. Propanadid [Ref. KDT 6/e, p 354]
a. Diazoxide (AIIMS Nov. 93) 54. What is treatment for morphine withdrawal :
b. Minoxidil a. Clonidine (AIIMS Dec. 92)
c. Hdralazine b. Nalorphine
d. No nitropruside c. Pentazocine
47. Mechanism of action of clonidine in opioid d. Buprenorphine [Ref. KDT 6/e, p 437]
withdrawl syndrome is : (AIIMS Nov. 93) 55. All are side effects of phenytoin ecept :
a. β blocking effect a. Hypotension rapid (AIIMS Dec. 92)
b. Inhibition of opioid receptor b. Acne
c. Action of a2 presynaptic nerve ending c. Glucose intolerance
d. Post synaptic action d. Nephrotic sydrome
[Ref. KDT 6/e, p 546] [Ref. KDT 6/e, p 385]
Answer 40. a. Ketamine 41. a. Ketamine 42. c. Fluoxetine 43. d. Thiopentone 44. a. Respiratory ...
45. c. Diazepam 46. d. No nitropru ... 47. c. Action of a2 ... 48. b. Carnama ... 49. c. Scoline ...
50. b. Constipation 51. d. Flumazenil 52. b. Metonidazole 53. c. Ketamine 54. a. Clonidine
55. d. Nephrotic ...

226
Phar macology of Central Nervous System
Pharmacology 3
56. Emetic action of morphine is due to action on : c. Flurazepam is longest acting
a. Gastric mucosa (June 1991) d. Doses should be increased if used with
b. Chemprecetor trigger zone cimetidine
c. Vomiting center in medulla [Ref. KDT 6/e, p 396 - 397]
d. Unknown mechanism 64. Site of action of morphine is : (UP 99)
[Ref. KDT 6/e, p 434] a. Brain stem
57. Intravenous dose of ketamine is : (AIIMS June 91) b. Cortex cerebri
a. 2mg/kg c. Spinal cord
b. 5mg/kg d. All of the above
c. 0.5mg/kg [Ref. KDT 6/e, p 376]
d. 1mg/kg 65. Shortest acting benzodiazepines : (UP 99)
[Ref. KDT 6/e, p 350] a. Diazepam
58. Dose of pancuronium for intubation is : b. Midazolam
a. 0.02 mg/kg (AIIMS June 91) c. Trizolam
b. 0.04 mg/kg d. Diazepam
c. 0.06 mg/kg [Ref. KDT 6/e, p 396]
d. 0.01 mg/kg 66. Methadone therapy used in A/E : (UP 99)
[Ref. KDT 6/e, p 53] a. Excruciating pain
59. Concentration of lignocaine for spinal anaes thesia b. Antitussive
is : (AIIMS June 91) c. Opiate withdrawal
a. 0.05% d. Alcohol withdrawal
b. 1.00% [Ref. KDT 6/e, p 459 - 450]
c. 2.00% 67. Side effect of amantidine are all except : (UP 98)
d. 5.00% a. Postural hypotension
[Ref. KDT 6/e, p 167] b. Ankle oedema
60. Which is not a ganglion blocker : c. Hepatotoxicity
a. Pancuronium (AIIMS June 91) d. Hallucination
b. Halothane [Ref. KDT 6/e, p 421, 777]
c. Trimethaphan 68. Busprione act on : (UP 98)
d. Pentolinium a. 5 HT1a 5HT1a
[Ref. KDT 6/e, p 113] b. 5HT1a
61. True statement about relationship of NSAID and c. GABA
gastric ulcer all except : (Up 01) d. 5HT3
a. Incidence increased if taken empty stomach [Ref. KDT 6/e, p 451]
b. Perrectum administration does not produce ul- 69. Baclofen is : (UP 98)
cer a. GABA derivative
c. IV drug produces less gastric ulcer than oral b. Benzodiazepines derivative
d. Incidence rate is decreasing in India c. 5HT1A
62. Aspirin inhibits : (UP 99) d. 5HT1B
a. β-oxygenases [Ref. KDT 6/e, p 349]
b. Uric acid excretion 70. Benzoisoquinoloine derivative is : (UP 98)
c. Cycloxygenase a. Morphine
d. Platelet aggregation b. Codeine
[Ref. KDT 6/e, p 185 - 187] c. Thebaine
63. True about benzodiazepines are A/E : (UP 99) d. Noscapine
a. Diazepam is shortest acting [Ref. KDT 6/e, p 453]
b. Midzolam is shortest acting

Answer 56. b. Chemprecetor ... 57. a. 2mg/kg 58. c. 0.06 mg/kg 59. d. 5.00% 60. b. Halothane
61. d. Incidence ... 62. c. Cycloxygenase 63. None 64. d. All of the ... 65. b. Midazolam
66. d. Alcohol ... 67. c. Hepatoto ... 68. a. 5 HT1a ... 69. a. GABA ... 70. d. Noscapine

227
3 Self Assessment & Review Phar macology
Pharmacology
71. Which of the following antipsychotic has least c. NO2
extrapyramidal symptoms side effect : (UP 98) d. Chloroform
a. Fluphenazine [Ref. KDT 6/e, p 371 - 372]
b. Thioridazine 79. Accidental intraaterial infection of thiopentone
c. Trifluperazine casues : (UP 97)
d. Triflupromazine a. Vaso spasm
[Ref. KDT 6/e, p 428] b. Necrosis of vessels
72. Which of the following antidepressants has least c. Vasodilation
anticholinergic side effect : (UP 98) d. Hypotension
a. Amoxapine [Ref. KDT 6/e, p 374]
b. Mianserine 80. Flushing is common a patient takin gwith oral hy-
c. Maprotiline poglycemic drug with alcohol is : (UP 96)
d. Trazosdone a. Chlorpropamide
[Ref. KDT 6/e, p 442] b. Phenformin
73. Analgesic effect of aspirin is due to : (UP 97) c. Glibenclamide
a. Inhibits prostaglandin synthesis d. Tolazamide
b. Prostacycline [Ref. KDT 6/e, p 278]
c. Heat production 81. Side effect of phenytoin are : (UP 96)
d. Inhibition of cylokines a. Gynaecomastia
[Ref. KDT 6/e, p 187 - 188] b. Osteomalacia
74. Aspirin inhibits : (UP 97) c. Megaloblastic anemia
a. Cyclooxygenase d. Gum hyperplasia
b. Lipooxygenase [Ref. KDT 6/e, p 385]
c. Prostacycline 82. The drug induced parkinsonism is best treated
d. Isomerase by: (UP 96)
[Ref. KDT 6/e, p 185 - 187] a. Benzhexol
75. Pure opoid antagonist is : (UP 97) b. Deprenyl
a. Nalophine c. Levodopa
b. Pentazocine d. Procyclidine
c. Naloxone [Ref. KDT 6/e, p 401]
d. Buprenorphine 83. Comparision of heroin from morphine are A/E :
[Ref. KDT 6/e, p 466] a. Slowly metabolished to morphine (UP 96)
76. Dissociate anaesthesia is caused by : (UP 97) b. More euphoria than morphine
a. Ketamine c. More constipation and nausea than morphine
b. Halothane d. Less hypotension
c. d-TC [Ref. KDT 6/e, p 436]
d. Gallamine 84. Physical with drawl symptoms are absent in pa-
[Ref. KDT 6/e, p 376] tient abusing : (UP 96)
77. Least cardiotoxic anaeshtetic agent is : (UP 97) a. Alcohol
a. Enflurane b. Cannabis
b. Isoflurane c. Opium
c. TCE d. Pethidine
d. Halothane 85. First anaesthetic used is : (UP 96)
[Ref. KDT 6/e, p 372] a. Procain
78. Maximum uterine relaxation is seen with :(UP 97) b. Cocaine
a. Ether c. Bupivacaine
b. Halothane d. Chloroprocaine [Ref. KDT 6/e, p 357]

Answer 71. b. Thioridazine 72. d. Trazosdone 73. a. Inhibits ... 74. a. Cyclooxy ... 75. c. Naloxone
76. a. Ketamine 77. b. Isoflurane 78. b. Halothane 79. a. Vaso spasm 80. a. Chlorpropamide
81. a, b, c and d 82. a. Benzhexol 83. c. More ... 84. b. Cannabis 85. b. Cocaine

228
Phar macology of Central Nervous System
Pharmacology 3
86. Which anesthetic agent caused : (UP 96) a. IDU (UP 94)
a. Buprenorphine b. Amantadine
b. Fentanyl c. Metrisazone
c. Prostigmine d. Acyclovir
d. Thiopentone [Ref. KDT 6/e, p 777]
87. Not involved sigment in epidural anesthesia is : 95. Drug of choice in trigemmal neuralgia is :
a. C4 (UP 96) a. Carbamazepine (UP 94)
b. T6 b. Phenobarbitone
c. L3 c. Phenytoin
d. S1 d. Valproic acid
88. Last CVS effect seen in : (UP 96) [Ref. KDT 6/e, p 405-406]
a. Enflurane 96. Drugs NOT to be given with alcoholic patients all
b. Isoflurane EXCEPT : (UP 94)
c. Trichloroethyline a. Latamoxel
d. Halothane b. Metronidazole
[Ref. KDT 6/e, p 373] c. Sulpride
89. All are causing Gum hyperplasia except :(UP 95) d. Chlorpropamide
a. Cyclosporin A [Ref. KDT 6/e, p 383]
b. AML 97. Which muscle relaxant is choice in bronchial
c. Carbamazepine asthama patient is : (UP 94)
d. Vitamin c deficiency a. d-tubocurarme
[Ref. Harrison 14/e, p 427] b. Scoline
90. All are antidepressent drugs except : (UP 95) c. Decamethonium
a. Trazadone d. Gallannne
b. Fluxetine 98. True about tricyclic anti depressert is (UP 94)
c. Amitriptyline a. Photosensitivity
d. Pimozide b. Effectcomes after 10 days
[Ref. KDT 6/e, p 439] c. + lead to cheese reaction
91. Cautry not used with : (UP 95) d. Diarrhoea
a. Diethylether [Ref. KDT 6/e, p 190]
b. Ketamine 99. Surgery is usually done in which stage of ether
c. Halothane anesthesia : (UP 94)
d. Sodium thiopentone a. I
[Ref. KDT 6/e, p 37] b. II
92. The drug contraindicated in porphyria is :(UP 95) c. III
a. Sodium thiopentone d. IV [Ref. KDT 6/e, p 367]
b. Methylsergide 100. Following is example of Lipid insoluble : (UP 93)
c. Ketamine a. Nadoicl
d. Atracurium b. Atenolol
[Ref. KDT 6/e, p 375] c. Sotalol
93. Haffman elimination occurs in : (UP 95) d. Propranol [Ref. KDT 6/e, p 140]
a. Atracurium 101. Increase Prolactin level by (UP 93)
b. d-TC a. Chlorpramazine
c. gallium b. Dopamine
d. Pancurorium c. Bromocriptine
[Ref. KDT 6/e, p 345] d. Apomorphine
94. Which antiviral drug is used in Parkmsonism : [Ref. KDT 6/e, p 431]

Answer 86. b. Fentanyl 87. a. C4 88. b. Isoflurane 89. c. Carbama ... 90. d. Pimozide
91. a. Diethylether 92. a. Sodium ... 93. a. Atracurium 94. b. Amantadine 95. a. Carbam ...
96. c. Sulpride 97. d. Gallannne 98. b. Effectcomes ... 99. c. III 100. d. Propranol
101. a. Chlorpramazine

229
3 Self Assessment & Review Phar macology
Pharmacology
102. Minimum Glucocorticoids activity found in : 110. Chemically paracetomol is : (AIIMS 90)
a. Hydrocortisone (UP 93) a. N-acetyl-p-aminophenol
b. Cortisone b. Chloroquine
c. Prednisolone c. Acetyl salicylate
d. Methyl prednisolone d. Acetophen
[Ref. KDT 6/e, p 291] [Ref. KDT 6/e, p 184, 196]
103. Anti Spasmodic effect is seen with : (Jipmer 93) 111. Drug which is used in rheumatoid arthritis and
a. Neostigmine heavy metal poisoning : (Kerala 97)
b. Propantheline a. Desferrioxamine
c. Nikethamide b. Gold
d. Ambenonium [Ref. KDT 6/e, p 106] c. Pencillamine
104. Ibuprofen acts by inhibiting : (Kerala 94) d. Chloroquine
a. Lipoxygenase [Ref. KDT 6/e, p 201 - 202, 867; Harrison 16/e, p
b. Cyclo oxygenase 1974]
c. Resetting the hypothalamic thermostat 112. Mandrax is combination of : (JIPMER 90)
d. Increases the pain threshold a. Hypnotic and analgesic
[Ref. KDT 6/e, p 175, 192] b. Hypnotic and antihistamine
105. Aspirin has all the properties except : (TN 91) c. Hypnotic and antipyretic
a. Brings down elevated temperature d. Hypnotic and antiinfluenzal
b. Brings down normal temperature [Ref. KDT 5/e, p 340]
c. Prevents platelet aggregation 113. Use of preoperative morphine is to : (Kerala 94)
d. Used in rheumatoid arthritis a. Decrease anxiety
[Ref. KDT 6/e, p 185] b. To produce analgesia
106. Aspirin is useful in all except : (MP 2K) c. For sedation
a. Pre-eclampsia d. For muscle relaxation
b. Deep ven thrombosis [Ref. KDT 6/e, p 378 - 379]
c. Myalgia 114. Most potent opioid is : (KERALA 01)
a. Fentanyl
d. Deep seated abdominal pain
b. Sulfentanyl
[Ref. KDT 6/e, p 190 - 191]
c. Pethidine
107. Single dose NSAID : (TN 96)
a. Aspirini d. Morphine
b. Diclofenac [Ref. KDT 6/e, p 378]
c. Naproxen 115. Flushing occurs following alcohol ingestion in
patents taking : (Kerala 95)
d. Piroxicam
a. Metronidazole
[Ref. KDT 6/e, p 194]
b. Penicillin
108. True about ketorolac is all except : (M.P. 98)
c. Tetracyclin
a. It inhibit cyclo-oxygenase
d. Chloramphenicol
b. It is as effecitive for pain as morphine
[Ref. KDT 6/e, p 383 - 386]
c. It acts on opoid receptor
116. Methanol causes blindnes due to its :
d. It acts only as an injectable
a. Hypersensitivity reactions (Kerala 90; Al 89)
[Ref. KDT 6/e, p 194 - 195]
b. Conjugated pr^Hucts
109. Which drug does not cause platelet dysfunction :
a. Aspirin (Kerala 97) c. Idiosyncratic reaction
b. Ibuprofen d. Oxidative products
c. Acetaminiphen [Ref. KDT 6/e, p 386]
d. All of the above
[Ref. KDT 6/e, p 198]

Answer 102. b. Cortisone 103. b. Propantheline 104. b. Cyclo ... 105. b. Brings ... 106. d. Deep ...
107. d. Piroxicam 108. c and d 109. c. Acetaminiphen 110. a. N-acetyl-... 111. c. Pencillamine
112. b. Hypnotic ... 113. a and b 114. b. Sulfentanyl 115. a. Metronidazole 116. d. Oxidative ...

230
Phar macology of Central Nervous System
Pharmacology 3
117. Primarily anxiolytic benzodiazepine is : (MAHE 98) a. Primidone
a. Flurazepam b. Diphenil hydantoin
b. Oxazepam c. Diazepam
c. Temazepam d. Carbamazepine
d. Alprazolam [Ref. KDT 6/e, p 404]
[Ref. KDT 6/e, p 389, 450] 125. With chronic use is seizure state, the adverse ef-
118. Secobarbitone is : (DNB 90) fects of this drug include coarsening of facial fea-
a. Long acting barbiturate tures, hirsutism, gingival hyperplasia and osteo-
b. Intermediate acting barbiturate malacia : (Karnat 05)
c. Short acting barbiturate a. Carbamazepine
d. Ultra short acting barbiturate b. Ethosuc.cimide
[Ref. KDT 6/e, p 389] c. Gabapentin
119. Barbiturates are derivatives of : d. Phenytoin
a. Urea (Bihar 90; JIPMER 80) [Ref. KDT 6/e, p 404]
b. Ethyl alcohol 126. Phenytoin Causes : (Assam 95)
c. Tungstic acid a. Paracortical Hyperplasia
d. Opium b. Follicular Hyperplasia
e. Cinnamon c. Osteoporosis
[Ref. KDT 6/e, p 390] d. Paracortical Hypertrophy
120. Which one of the following drugs produce rapid [Ref. KDT 6/e, p 404 - 405; Robbins 6/e, p 650;
induction of sleep with least effect on sleep Harrison 16/e, p 2271]
structure : (St. Johns 02) 127. Phenytoin does not cause : (U.P. 96)
a. Zolpidem a. Osteomalacia
b. Diazepam b. Gynacomastia
c. Chlorpromazine c. Gum hyperplasia
d. Haloperidol d. Megaloblastic anemia
[Ref. KDT 6/e, p 398] [Ref. KDT 6/e, p 404 - 405; Harrison 15/e, p 432]
121. The specific antagonist for Benzodiazepines is : 128. Carbamazepine is useful in the following condi-
a. Flumazenil (Kerala 90) tions except : (Karn 95)
b. Alprozolam a. Grandmal epilepsy
c. Di-Isoprophyl phenol b. Petitmal epilepsy
d. Cremopher-ei c. Psychomotor epilepsy
[Ref. KDT 6/e, p 399 - 400] d. Trigeminal neuralgia
[Ref. Harrison 15/e, p 2363; KDT 6/e, p 405 -
122. Which of the fllowing are side effect of flumazenil:
406]
a. Nausea (PGI 92)
129. When carbamazepine is administered one should
b. Headache avoid : (JIPMER 95)
c. Withdrawal seizures a. Erythromycin
d. Dizziness b. Amoxycillin
[Ref. KDT 6/e, p 400] c. Ceftriaxone
123. Which antiepileptic drug in therapeutic doses d. Sulfamethoxazole
causes least sedation ? (TN 95)
[Ref. KDT 6/e, p 406]
a. Primidone
130. Least sedating drug among the following is :
b. Clonazepam
a. Clonazepam (MAHE 98)
c. Phenytoin
b. Phenytoin
d. Phenobarbitone [Ref. KDT 6/e, p 403]
c. Primidone
124. Use of which antiepileptic drug causes gingival
d. Phenobarbital
hypertrophy and hirusitism ? (TN 95)
[Ref. KDT 6/e, p 403, 405, 408]
Answer 117. b and d 118. b. Intermediate 0 119. a. Urea 120. a. Zolpidem 121. a. Flumazenil
122. c. Withdrawal ... 123. c. Phenytoin 124. b. Diphenil ... 125. d. Phenytoin 126. a. Paracortical ...
127. None 128. b. Petitmal ... 129. a. Erythromycin 130. b. Phenytoin

231
3 Self Assessment & Review Phar macology
Pharmacology
131. All of the following are indicated in the treatment 138. Drug of choice in drug induced parkinsonism is :
of petitmal epilepsy except : (SGPGI 05) a. Levodopa (AI 90)
a. Ethosuximide b. Benzhexol
b. Sodium-valproate c. Amantidine
c. Phenytoin d. Carbidopa
d. Clonazepam [Ref. KDT 6/e, p 421]
[Ref. KDT 6/e, p 411] 139. Which of the following is a longest acting
132. Drug of choice of psychomotor epilepsy is : butyrophenone : (PGl 90, 93)
a. Valproic acid (Rajasthan 97) a. Haloperidol decanoate
b. Carbamazepine b. Triperidol
c. Ethosuccimide c. Flupenthixol
d. Phenytoin d. Penfluridol
[Ref. KDT 6/e, p 411; Harrison 16/e, p 2367 - [Ref. KDT 6/e, p 427, table (32.1), 429]
2369] 140. Refractory schizophrenia - What is the treatment
133. Which is not used in status epilepticus? of choice : (JIPMER 03)
a. Lorazepam (Burdwan 00) a. Haloperidol
b. Phenytoin b. Flupenthixol
c. Phenobarbitone c. Trifluoperazine
d. Valproate d. Clozapine
[Ref. KDT 6/e, p 411; Harrison 16/e, p 2371] [Ref. KDT 6/e, p 429]
134. Absence seizures is treated by- (AI 9I) 141. Clozapine (a newer antipsychotic. is commonly
a. Phenobarbitone associated with : (PGl 80; NIMHANS 90)
b. Ethosuximide a. Seizures
c. Diazepam b. Depression
d. Phenytoin c. Agranulocytosis
[Ref. KDT 6/e, p 411; Harrison 16/e, p 2367] d. Tardive dyskinnesia
135. In the basal ganglia, the dopaminergic activity is [Ref. KDT 6/e, p 429]
balanced by : (UPSC 00) 142. Which of the following antipsychotic drugs gives
a. Serotonergic system rise to agranulocytosis ? (UPSC 01)
b. Cholinergic system a. Risperidone
c. GABA b. Clozapine
d. Glutamate c. Olanzapine
[Ref. KDT 6/e, p 414] d. Pimozide
136. Which sf the following is contraindicated with [Ref. KDT 6/e, p 429]
levodopa ? (Karnat 96) 143. Antipsychotic with less extra pyramidal side
a. Carbidopa effects : (Jipmer 04)
b. Vitamin B Complex a. Loxapine
c. Selegilline b. Pinozide
d. Methyldopa c. Qutetapiqe
[Ref. KDT 6/e, p 418] d. Resperidone [Ref. KDT 6/e, p 429]
137. Pergolide used in the treatment of Pasrkinsonism 144. Which of the following medication is associated
acts as : (J & K 01) with an increased risk of agranulocyosis :
a. Dopamine agonist a. Clozapine (Karnat 05)
b. MAO - B inhibitor b. Imipramine
e. MAO - A inhibitor c. Lithium
d. Anticholinergic d. Haloperidol
[Ref. KDT 6/e, p 415 - 419] [Ref. KDT 6/e, p 429]

Answer 131. c. Phenytoin 132. b. Carbama ... 133. d. Valproate 134. b. Ethosuximide 135. b. Cholinergic ...
136. b. Vitamin B ... 137. a. Dopamine ... 138. b. Benzhexol 139. a. Haloperidol ... 140. d. Clozapine
141. c. Agranulo ... 142. b. Clozapine 143. d. Resperidone 144. a. Clozapine

232
Phar macology of Central Nervous System
Pharmacology 3
145. Rabbit syndrome is a : (Karnat 99) b. Haloperidol
a. Late onset. Drug induced extra pyramidal symp- c. Clozapine
toms d. Doxepin
b. Late onset, drug induced anticholinergic side [Ref. KDT 6/e, p 433]
effect 152. Phenothiazines are used in treatment of :
c. Intermediate onset, drug induced extra pyrami- a. Depression (Delhi 96)
dal symptoms b. Organic brain syndrome
d. Late onset effect of Parkinson s disease c. Anxiety neurosis
[Ref. KDT 6/e, p 431] d. All
146. Irresistable urge to move about and increased mo [Ref. KDT 6/e, p 433]
tor activity is : (TN 97)
153. The following drug has anxiolytic with lea?’
a. Rabbits syndrome sedation : (JIPMER 02)
b. Malignant neuroleptic syndrome a. Chlordiazepoxide
c. Akathisia b. Buspirone
d. Tardive dyskinesia c. Triazolam
[Ref. KDT 6/e, p 431] d. Alpra
147. Akathisia is best treated with : (Jipmer 02) [Ref. KDT 6/e, p 450]
a. Haloperidol 154. An anxiolytic benzodiazepine that has additional
b. Fluoxetine antidepressant property is : (TNPSC 2K)
c. Lithium a. Chlordiazepoxide
d. Propranolol b. Oxazepam
[Ref. KDT 6/e, p 431] c. Alprazolam
148. The rabit syndrome (consisting of fine, rapid move- d. Lorazepam
ment of the lips that mimic chewing move ments [Ref. KDT 6/e, p 450]
of a rabit) is effectively treated by :
155. All are true about Alprazolam except : (TN 97)
a. Trihexyphenidyl (PGI 81; NIMHANS 90)
a. Anxiolytic
b. Lorazepam
b. Is a Benzodiazepine
c. Lithium
c. Causes sedation
d. Levodopa
d. Very short ½ life
[Ref. KDT 6/e, p 431]
[Ref. KDT 6/e, p 450]
149. A patient on haloperidol for psychosis develops
156. Drugs of choice for insomnia in elderly :
fever, altered sensorium and labile hypertension.
Drug useful in treatment of this patient is : a. Diazepam (Manipal 04)
a. Cefotaxime (UPSC 02) b. Fluoxetin
b. Bromocriptine c. Flurazepam
c. Largactil d. Oxazepam
d. Propanolol [Ref. KDT 6/e, p 450]
[Ref. KDT 6/e, p 432; Goodman Gillman 10/e, p 157. Anxiolytic without benzodiazepineola structure :
501] a. Alprazolam (Jipmer 03)
150. Long term antipsychotic use definitely causes : b. Chlordiazepoxide
a. Depression (APPG 03) c. Diazepam
b. Mania d. Zolpidem
c. Schizophrenia [Ref. KDT 6/e, p 450, 398]
d. Tardive dyskinesia 158. Non-sedating nonhabit forming anxiolytic is :
[Ref. KDT 6/e, p 432] a. Triazolam (PGI 92, 93)
151. Drug of choice for the treatment of negative b. Buspirone
symptoms of schizophrenia is : (Karnataka 02) c. Alprazolam
a. Chlorpromazine d. Oxazepam [Ref. KDT 6/e, p 450]

Answer 145.a. Late onset ... 146. c. Akathisia 147. d. Propranolol 148. a. Trihexy ... 149. b. Bromocriptine
150. d. Tardive ... 151. c. Clozapine 152. b and c 153. b. Buspirone 154. c. Alprazolam
155. Very short ½ ... 156. d. Oxazepam 157. None 158. b. Buspirone

233
3 Self Assessment & Review Phar macology
Pharmacology
159. Lorazepam differs from diazepam in that it is : c. Fluoxetine
a. More lipid soluble (AIIMS 90) d. Amitryprylin
b. Fast entry into brain [Ref. KDT 6/e, p 442, table (33.1)]
c. Short - acting 167. 62 year old male who was on antidepressant treat-
d. Active metabolic produced ment was brought to the hospital with historyof
[Ref. KDT 6/e, p 450] having taken overdose. He was complaining of dry
160. Newer nonsedative, non hypnotic anxiolytic is : mouth, blurring of vision and history of having not
passed urine for 12 hours.The bladder is palpable.
a. Diazepam (Assam 95)
The medication could be :
b. Lorazepam
a. Lithium (UP 2K)
c. Buspirone
b. Selengilline
d. Alprazolam
c. Amitrytiline
[Ref. KDT 6/e, p 450]
d. Dexamphetamine
161. For detoxification of alcohol withdrawal drug used
[Ref. KDT 6/e, p 444]
in : (JIPMER 99)
168. Imipramine is contra indicated in : (AI 92)
a. Disufiram
a. Diabetes
b. Oxazepam
b. Glaucoma
c. Naltrexone
c. Hypertension
d. Adrenaline
d. Peptic ulcer [Ref. KDT 6/e, p 442, 444]
[Ref. KDT 6/e, p 450; Ahuja 4/e, p 38]
169. Which does not compete with protein binding of
162. Central Inhibitor of dopamine metabolism is :
imipramine : (Al 93)
a. Carbidopa (AI 93)
a. Ethosuximide
b. Deperenyl
b. Phenytoin
c. Orphenadrine
c. Aspirin
d. Bromocriptine
d. Phenyl butazone
[Ref. KDT 6/e, p 420, 439]
[Ref. KDT 6/e, p 444]
163. Anti depressant is : (Kerala 96)
170. Tricyclic antidepressants are contra indicated in
a. Chlorpropamide
a. Gastric ulcer (AP 97)
b. Imipramine
b. Angle Closure Glaucoma
c. Benzhexol
c. Depression
d. Lithium
d. Anxiety
e. Haloperidol [Ref. KDT 6/e, p 439]
[Ref. KDT 6/e, p 444, 113]
164. Newer MAO inhibitors are useful in the treatment
171. Anti-derpessant causing Tardive dyskinesia
of : (Kerala 94)
is : (AP 96)
a. Mania
a. MAO inhibitors
b. Schizophrenia
b. Mianserin
c. Hypertension
c. Imipramine
d. Depression
d. Amoxapine
[Ref. KDT 6/e, p 439 - 440]
[Ref. KDT 6/e, p 445]
165. Specific MAO-B inhibitor is : (Jimper 95)
172. Tianeptine is an antidepressant which is very popu-
a. Phenelzine
lar nowadays. Its mechanism of action is :
b. Tranylcypromine
a. Norepinephrine agonist (Kerala 04)
c. Selegeline
b. Selective serotonin reuptake inhibitor
d. Haloperidol
c. Selective serotonin reuptake enhancer
[Ref. KDT 6/e, p 440]
d. Selective non-epinephrine reuptake inhibitor
166. Safest anti depressant in hypertension is :
[Ref. KDT 6/e, p 447]
a. Imipramine (Kerala 95)
b. MAO inhibitors

Answer 159. a, c and d 160. b. Lorazepam 161. b. Oxazepam 162. b. Deperenyl 163. b. Imipramine
164. d. Depression 165. c. Selegeline 166. c. Fluoxetine 167. c. Amitrytiline 168. b. Glaucoma
169. a. Ethosuximide 170. b. Angle ... 171. d. Amoxapine 172. c. Selective ...

234
Phar macology of Central Nervous System
Pharmacology 3
173. Antiepileptic which is also a mood stabilizer 180. Morphine causes vomiting by stimulating :(A1 91)
is : (TN 04) a. Medullary centre
a. Phenytoin b. Spinal cord
b. Phenobarbitone c. Hypothalamus
c. Sodium valproate d. Chemoreceptor trigger zone
d. Diamox [Ref. KDT 6/e, p 455]
[Ref. KDT 6/e, p 436 - 437] 181. Morphine : (Kerala 97)
174. The commonest symptom of lithium Toxicity is - a. Stimulates CTZ zone of medulla
a. Nephrotic syndrome (AIIMS 81, AMC 92) b. Stimulates respiratory centre of medulla
b. Polyurea c. Depresses CTZ zone of medulla
c. ARF d. None of the above
d. Renal calculi [Ref. KDT 6/e, p 455]
[Ref. KDT 6/e, p 435] 182. Which is not a feature of opioid withdrawal :
175. AH of the following are recognised side effects of a. Rhinorrhoea (PGI 93)
lithium carbonate therapy except : (UPSC 95) b. Constipation
a Hypothyroidism c. Yawning
b. Leucocytosis d. Pilo erection
c. Polyuria [Ref. KDT 6/e, p 457
d. Hypercalcaemia 183. Heroin differs from morphine in all respects ex-
[Ref. KDT 6/e, p 435] cept: (AI 90)
176. Lithium clearance is decreased by : a. Causes more euphoria
a. Hyperkalemia (Karnataka 01) b. More constipation
b. Hypokalemia c. Slowly metabolised to morphine
c. Hypernatremia d. Synthetic congener of morphine
d. Hyponatremia [Ref. KDT 6/e, p 458; Parikh 6/e, p 107; Laurence
[Ref. KDT 6/e, p 435 - 436] 9/e, p 340]
177. Lithium level is increased by administration of 184. Which one of the following drug is not a non-opioid
which of the following drugs : (SRMC 02) analgesic drug ? (COMED 06)
a. NSAIDs a. Meloxicam
b. Diuretics b. Methadone
c. Verapamil c. Nimesulide
d. Bronchodilators d. Nabumetone
[Ref. KDT 6/e, p 435 - 436] [Ref. KDT 6/e, p 459 - 460, 184]
178. Young female with bipolar disorder who failed 185. Methadone is : (TN 90)
to respond to adequate lithium therapy should a. Useful for long term treatment
be prescribed : (MAHE 98) b. No addiction liability
a. Chlorpromazine c. Better compliance
b. Amitryptilne d. Used in maintenance of treatment
c. Carbamazepine [Ref. KDT 6/e, p 459 - 460]
d. Haloperidol 186. Therapeutic indications of morphine : (PGI 90)
[Ref. KDT 6/e, p 436; Harrison 16/e, p 2557] a. Preterm labour
179. Mood stabilizing drugs are all except : (Jipmer 03) b. Pain
a. Sodium valproate c. Intractaole diarrhoea
b. Carbamazepine d. Abolishes rigors
c. Lithium [Ref. KDT 6/e, p 461]
d. Ehosuximide
[Ref. KDT 6/e, p 436 - 437]

Answer 173. c. Sodium ... 174. b. Polyurea 175. d. Hypercal ... 176. d. Hyponatremia 177. a and b
178. c. Carbamazepine 179. d. Ehosuximide 180. d. Chemore ... 181. a. Stimulates ... 182. b. Constipation
183. a and b 184. b. Methadone 185. d. Used in ... 186. b. Pain

235
3 Self Assessment & Review Phar macology
Pharmacology
187. Wrong about pentazocine : (TN 99) 194. Doxapramis drug : (TN 99)
a. Orally effective a. Respiratory stimulant
b. Decreases B.P. b. Antiepileptic
c. Tachycardia c. Sedative
d. Addictive d. Antidiabetic
[Ref. KDT 6/e, p 464 - 465] [Ref. KDT 6/e, p 470]
188. The most potent narcotic analgesic is : (AI 91) 195. The drug of choice for attention deficit hyperactiv-
a. Morphine ity disorder is : (COMEDK 05)
b. Pethidine a. Haloperidol
c. Bupheronorphine b. Imipramine
d. Pentazocine c. Methyl phenidate
[Ref. KDT 6/e, p 465] d. Lecithin
189. Opioid Specific antagonist is : (Al 93) [Ref. KDT 6/e, p 470]
a. Naloxone 196. Drug used in alzheimer’s disease having adverse
effect on liver : (Jipmer 04)
b. Naltrexone
a. Tacrine
c. Methadone
b. Dionezepil
d. Nalorphine
c. Thioctic acid
[Ref. KDT 6/e, p 466]
d. Flumazenil
190. None of the following analgesics is available in
sublingual formulation except : (Karn 94) [Ref. KDT 6/e, p 472]
a. Piroxicam 197. Donepezil is used in the treatment of which of the
b. Pentazocine following of the following conditions :
c. Buprenorphine a. Schizophrenia (Bihar 02)
d. Ketorolac b. Depression
[Ref. KDT 6/e, p 465] c. Anxiety
191. An orally effective opoid agonist without an d. Alzheimer dementia
antagonistic action : (JIPMER 90) [Ref. KDT 6/e, p 472]
a. Dextropropoxyphene 198. AH are used in treatment of epilepsy except :
b. Buprenorphine a. Ethosuximide (Kerala 94)
c. Pentazocine b. Reserpine
d. Cyclazocine c. Acetazolamide
[Ref. KDT 6/e, p 459, 464 - 465] d. Vigabatrine
192. The most efficient respiratory stimulant is : [Ref. KDT 6/e, p 549; Harrison 16/e, p 2367-2369]
a. Nikethamide (Bihar 98) 199. In case of CCF with gout which diuretic is preferred:
b. Amiphenazole a. Lasix (AI 89)
c. Protheamide b. Thiazide
d. Doxapram c. Ticaynafen
[Ref. KDT 6/e, p 470] d. Ethacrynic acid
193. The term “soporofic” is synonymous with all Aph- [Ref. KDT 6/e, p 467]
rodisiac : (BIHAR 90) 200. All of the following are risk factors for NSAID in-
b. Psychotomimetic duced mucosal injury except : (ICS 2K)
c. Hypnotic a. Old age and female sex
d. Euphoric b. Prolonged use of high dose therapy
e. CNS stimulant c. NSAID with steroid therapy
[Ref. KDT 6/e, p 470] d. NSAID with misoprostol
[Ref. KDT 6/e, p 633 - 634]

Answer 187. b. Decreases B.P. 188. c. Bupheronor ... 189. a. Naloxone 190. c. Buprenorp ... 191. a and b
192. d. Doxapram 193. None 194. None 195. None 196. a. Tacrine
197. b. Depression 198. b. Reserpine 199. c. Ticaynafen 200. d. NSAID ...

236
Phar macology of Central Nervous System
Pharmacology 3
201. Addition of which of the following agent can help 202. Patient with hypotension and cardiac disease best
in elimination of 15-20 Tablets of 10 mg dextroam- anti depressant is : (NIMHANS 06)
phetamine : (Bihar 01; AIIMS 90) a. Venlafaxine
a. Acetazolamide b. Mirtazapine
b. NH4Cl c. Duloxetine
c. Penicillamine d. Cifaloprax
d. NaHCO3 [Ref. Goodman Gillman 10/e, p 454]
[Ref. Goodman Gillman 10/e, p 236]

Answer 201. b. NH4CI 202. b. Mirtazapine

237
NOTES
THE THERAPY OF ANGINA
Angina is a result of hypooxygenation of the myocardium. It is a function of the ratio of oxygen demand to oxygen delivery.
Thus, the goals of therapy are to increase oxygen delivery to the myocardium, or, alternatively, to decrease oxygen demand
commensurate with oxygen delivery

Classification of angina
Unstable angina (resting angina) and stable (exercise-induced) angina are both thought to be due to the formation of arterial
sclerotic plaques. Unstable angina occurs at rest, i.e., anginal pain and myocardial deoxygenation occur during a relaxed
state. This form of angina is mos t serious, as it indicates a probable infarct.
Stable angina is more predictable, as is exercise-induced angina events are precipitated by an increase in cardiac work.
Variant (Prinzmetal’s) angina is less common, and is due to coronary vasospasm.
The therapy for resting angina may be an agent that increases coronary perfusion, such as a nitro compound. Alternatively,
a drug that depresses cardiac function (e.g., a calcium channel blocker or beta-blocker) may be used to decrease the work
of the heart
Rarely angina occurs in angiographically normal vessels without other identifiable cause - Syndrome X. It is most likely
due to inadequate flow in the coronary microvasculature

Control of coronary blood flow involves:


• Local metabolism (Major Determinant)
• Nervous system regulation.
Candidates for these vasodilatory substances include: (Local metabolic factors)
• adenosine
• prostaglandins (PGI2)
• bradykinin
• hydrogen ions
• carbon dioxide
• lactate

NITRIC OXIDE ( most important coronary vasodilator)


• Cyclic GMP Facilitates dephosphorylation of Myosin-Light Chain Po4
• Calcium triggers actin / myosin complex →contraction
• PDEI enhances cyclic GMP concentration
• Decreasing intracellular calcium concentration by CCBS and stabilizing myocardial membrane potential at near
RMP by enhancing K+ channel openers, Nicorandil, Chromakalin etc

DRUGS USED IN ANGINA : 3 Major drug classes


Nitrates in the treatment of angina : Nitrates are useful in treating angina because they both decrease the work of the heart
and increase myocardial perfusion. In addition, they help prevent the formation of thrombi, which potentially could cause
infarction.
4 Self Assessment & Review Phar macology
Pharmacology
Nitrates decrease myocardial oxygen consumption : These drugs decrease ventricular filling and increase stroke volume,
effectively decreasing the work of the heart. Nitrates are inhibitors of platelet function, by virtue of increased synthesis of cyclic
guanylate monophosphate (cGMP), which is antagonistic toward cyclic adenosine monophosphate (cAMP), a necessary
component of the platelet activation cascade.

Adverse effects commonly seen with nitrate therapy :


• Adverse effects include postural hypotension, syncope, nausea and vomiting, and muscle weakness. Severe hypoten-
sion and syncope are seen with less than 1 percent of patients.
• Methemoglobinemia is a rare occurrence with nitrate therapy and is characterized by cyanosis, and nausea/vomiting,
progressing to shock and coma.
• The structure of nitroprusside contains cyanide (HCN). After administration, the drug rapidly interacts with sulfhydryl
groups in cell membranes, and. in erythrocytes, reacts with thiols and hemoglobin to form methemoglobin. This results
in the release of cyanide, which is extremely toxic. The toxicity of the released cyanide is prevented by the buffering of the
cyanide radicals by the methemoglobin, resulting in the formation of cyanomethemoglobin. Cyanide radicals remaining
are converted hepatically to thiocyanate, which is then excreted renally.
• Tolerance and attenuation of the vasodilatory effects of the drug may be seen. This event appears to be related to elevated
plasma levels of drug.
K+ Channel Openers (Cromakalim, Bimakalim, Nicorandil, Pinacidil)

Hydralazine not effective in anginal therapy : Drugs that potently dilate arterioles can cause a phenomenon known as
“coronary steal,” whereby the increase in flow caused by the arteriolar dilation draws blood from unobstructed vessels and
increases flow to areas already well perfused. This “steals” blood preferentially from vessels which may be obstructed (have
lesser flow), and decreases perfusion in those areas which may already be ischemic.

CALCIUM CHANNEL ANTAGONISTS


Amongst 3 classes of CCBs, Dihydropyridines (eg Nifedipine) is most vasculoselective, Phenylalkylamines (verapamil) is
more cardioselective while Benzothiazepine (Diltiazem) is intermediate DHPs also vary in selectivity in different vascular
beds. (NIMODIPINE acts on cerebral vessels)

Benefits of nifedipine therapy in angina : Nifedipine is a vasoactive drug and thus decreases coronary spasm, allowing a
greater degree of myocardial perfusion. In- addition, the drug decreases afterload, which decreases the work of the heart,
and preload, which decreases myocardial stretch, as well as passive coronary vascular compression. Thus, myocardial
oxygen consumption is decreased and perfusion is increased.

Antianginal therapy for a patient with hypotension : A drug that has reduced vasoactive action is most appropriate in such
a patient. Verapamil, diltiazem, bepridil, or a beta1 antagonist would be useful.
Amlodipine is a potent peripheral vasodilator, which decreases cardiac afterload and thus decreases cardiac work and
myocardial oxygen consumption. Coronary arteries are also dilated, increasing oxygen delivery to the myocardium. Amiodipine
is useful in both chronic stable angina and Prinzmetal’s angina.
Amlodipine has little effect on myocardial tissues, It does not significantly reduce conduction, nor does-it have significant
negative inotropic activity.

Amlodipine not induce significant reflex tachycardia : The drug has a gradual onset, and thus does not cause significant
baroreceptor stimulation.

Nicardipine useful in the therapy of angina : Nicardipine selectively dilates cerebral and coronary vessels. Thus, myocardial
blood flow and oxygen delivery are increased, but performance is not decreased.
Nicardipine does not decrease myocardial contractility and so would be especially useful for antianginal therapy in a patient
with CHF.

BETA-RECEPTOR ANTAGONISTS
These drugs block sympathetic stimulation of the myocardium, resulting in decreased heart rate and force of contraction.
The decreased rate results in an increase in diastolic perfusion time and, therefore, increased oxygenation of the myocar-
dium, while the negative inotropic effect results in a decrease in oxygen demand.

242
Phar macology of Car
Pharmacology diovascular & Renal System
Cardiovascular 4
Efficacy of propranolol and atenolol in the therapy of angina : Atenolol is a selective â1, receptor antagonist, and mediates
a decrease in heart rate and force of contraction. This results in increased oxygen delivery and decreased myocardial oxygen
consumption. | â2 receptors are relatively unaffected, so sympathetic activity results in vasodilatation and decreased afterload.
Propranolol blocks vasodilatory effects with sympathetic stimulation. This may result in an increase in afterload (as com-
pared to the effects of atenolol) and increased myocardial oxygen consumption.
Pindolol causes a net vasodilatation, and thus decreases afterload, resulting in decreased myocardial oxygen consumption

COMBINATION THERAPY
Advantages and disadvantages of combination therapy with a beta-blocker and nitrate in angina : This combination is
useful, as the two effects of the drugs synergize, allowing lower doses of each drug to be used. The cardio depressant
actions of the beta-blocker decrease the rate and force of contraction. Concurrently administered, the nitrates decrease
cardiac afterload, which increases the ejection fraction of the heart and reduces end diastolic volume, in addition to decreas-
ing myocardial oxygen consumption. Additionally, the compensatory sympathetic reflexes produced by the nitrates are
inhibited by the presence of the beta receptor blockers.
Combination therapy with atenolol and nifedipine : These drugs synergize effects well—atenolol decreases myocardial
oxygen consumption through negative inotropic and chronotropic effects, while nifedipine mediates coronary arteriolar
dilatation and increased oxygen delivery. In addition, nifedipine decreases myocardial oxygen consumption by decreasing
afterload. The presence of the beta-blocker also blocks reflex sympathetic effects caused by the nifedipine-mediated,
vasodilatation

DRUGS USED IN THE THERAPY OF CONGESTIVE HEART FAILURE (CHF)


In heart failure, contractility is decreased, filling is increased, and ejection fraction is decreased. Thus, desirable changes
include lowering afterload (to increase ejection fraction), lowering preload (to decrease filling), lowering of fluid volume, and
increasing cardiac contractility
Conventional therapy combines digoxin, diuretics, and angiotensin-converting enzyme (ACE) inhibitors. Digitalis glycosides
increase cardiac contractility, diuretics decrease fluid volume and, therefore, ventricular fill volume, and ACE inhibitors (or
vasodilators) decrease afterload without interfering with myocardial function. ACE inhibitors also decrease fluid volume.

DIURETICS
Diuretics lower fluid volume, which decreases both preload and afterload. This decreases myocardial stretch and increases
ejection fraction.
Effects of torsemide or bumetanide in CHF therapy : These drugs are loop diuretics and are extremely efficient in the
lowering of fluid volume and decreasing myocardial stretch. They are also useful in reducing edema associated with CHF.
The adverse effect of these drugs is the lowering of serum potassium, which may interfere with cardiac conduction in an
already compromised heart
Amiloride has mild hypotensive effects, which are beneficial in treatment of CHF. It is also potassium sparing, and may be
useful as adjunct therapy with loop diuretics
The advantages of using ACE inhibitors in CHF : ACE inhibitors decrease arterial tone by decreasing the production of
angiotensin II and increasing bradykinin. In addition, a decrease in sodium retention by aldosterone is seen, which limits
fluid retention and decreases blood volume. This in turn decreases ventricular filling and myocardial stretch

DRUGS THAT INCREASE CONTRACTILITY


Mechanism of action of digitalis glycosides in the therapy of CHF : Digitalis glycosides block the activity of sodium-
potassium adenosine triphosphatase (ATPase). This inhibits the recovery of the cardiac myocyte from depolarization in a
dose-dependent manner. This, in turn, results in a buildup of sodium within the cell and potassium outside of the cell, with
successive depolarizations. The increase in intracellular sodium inhibits the membrane sodium-calcium transporter, allow-
ing accumulation of calcium within the cell. The transporter may eventually reverse, and intracellular sodium is exchanged
for extracellular calcium. The resulting increase in intracellular calcium mediates an increase in the force of contraction of
the cardiac muscle.
Consequence of increased intracellular calcium on cardiac conduction : Calcium overload is manifested as an early
afterdepolarization of the cardiac action potential, which may result in premature depolarization of Purkinje fibers and the
appearance of ventricular arrhythmias. With substantially increased calcium load, this may lead to ventricular tachyarrhythmias
and death

243
4 Self Assessment & Review Phar macology
Pharmacology
Glycosides increase intracellular sodium and also extracellular potassium by inhibitory actions on sodium potassium
ATPase. Thus, with chronic therapy, because intracellular sodium is increased, phase 0 may be blunted slightly. The
increase in extracellular potassium results in a decrease in the rate of repolarization and thus a decrease-in the slope of
phase 3 of the action potential, and a “skewed” appearance. This results in a longer time for the membrane to repolarize and,
accordingly, an increase in the effective refractory period. Because the objective of the drug is to increase the influx of
calcium, the faster influx of calcium results in a shortened phase 2, because a shorter time is required for the membrane to
reach the equilibrium potential for calcium. Finally, phase 4 is elevated and increased in slope, due to the alterations in
sodium and potassium concentration. This, with chronic therapy, and particularly as toxicity is approached, results in
delayed oscillatory afterpotentials.

Effects of cardiac glycosides : These drugs decrease conduction in the AV node, and, thus, in therapeutic doses, mediate
a partial blockade of the node. This decreases the frequency and number of ectopic foci reaching the ventricle.
These drugs decrease the rate of conduction in the AV node, due to the changes in the cardiac action potential as discussed
above (increased effective refractory period, ERP).
Increase urinary output that is mainly due to improved renal circulation.
These drugs sensitize nodal tissue to parasympathetic activity, resulting in a decreased rate of firing of the sinoatrial (SA)
node, and a decrease in conduction.
The first sign of toxicity is bradycardia, due to the effects of the drug on the cardiac action potential. This may be accompanied
by fatigue, drowsiness, mental confusion, and blurred vision. Further toxicity results in tachycardia.

Cardiac glycosides result in bradycardia : Digitalis glycosides cause an influx of extracellular calcium and an accumulation
of extracellular potassium. The increased levels of extracellular potassium decrease the rate of re-polarization of the cardiac
cell. Thus, as levels of the drug approach the toxic range, accumulated extracellular potassium causes a lengthening of
phase 3 of the cardiac action potential and an increase in the length of time between depolarizations, Thus, bradycardia is
seen.
Digoxin is excreted unchanged in the urine, which accounts for its relatively short half-life.
Digitoxin is metabolized in the liver to active metabolites. These are eliminated in the bile, and subject to enterohepatic
recycling. This accounts for its relatively long half-life.

BIPYRIDINES
Mechanism of action of bipyridine-type drugs (amrinone and milrinone) : These drugs mediate an inward current of
calcium, possibly due to the inhibition of phosphodiesterase. In addition, they cause a release of intracellular calcium from
stores in the cardiac sarcoplasmic reticulum. This increases troponin-tropomyosin interactions, and, consequently, force of
contraction.

HYPERTENSION
The increased pressure against vascular walls, combined with normal blood flow, produces 3 shearing effect on the
vascular endothelium. This results in damage to endothelial cells, production of paracrine mediators, and decreased
production of vasodilator substances such as those involved in the nitrous oxide/guanylate cyclase cascade. This may result
in increased incidence of vasospasm (e.g., stroke, myocardial infarct [Ml]). The endothelial damage produced by the in-
creased shear may also provide a reptatorory for lipid substances, facilitating the formation of sclerotic plaques, which
decrease vessel diameter and reduce blood flow to critical organs. In addition, increased pressure, flow rate and total
peripheral resistance (TPR) (afterload) increase ventricular stretch and may result in ventricular hypertrophy and congestive
heart failure. End organ failure (e.g., liver and kidney) may also occur.

Initial therapy normally begins with a reduction of plasma volume, using a diuretic.
Diuretics (also ACE inhibitors and β blockers) are 1st line agents for the control of mild to moderate hypertension
Combination with β blockers / ACEI / sympathoplegic drug potentiate therapeutic effect.
Thiazides are not effective in patients with inadequate renal function i.e. CLCR < 50 ml/ min (Loop diuretics are effective).
Thiazides reduce calcium excretion in kidney. So they can be beneficial in osteoporosis and idiopathic hypercalcuria.
Advantages of using a selective beta-receptor antagonist (e.g., atenolol, metoprolol) in the therapy of hypertension.
The beta receptor is involved in peripheral arterial vasodilatation, as well as bronchiolar dilatation. Thus, with the use of a
nonselective antagonist, blockade of the β receptor may not only decrease the amount of vasodilatation produced, and thus
decrease the efficacy of the drug, but may also precipitate bronchoconstriction in susceptible individuals. Use of a selective
{β1 receptor antagonist, which by definition has negligible effects on the (β2 receptor, would preclude these events).

244
Phar macology of Car
Pharmacology diovascular & Renal System
Cardiovascular 4
Pindolol is a partial agonist at the β2-adrenergic receptor, and has antagonistic actions at the β1 -receptor. Thus, cardiac
output is reduced with the drug, due to β receptor antagonism, and vasodilatation is increased, due to the effects of
[β1receptor stimulation].

Use of calcium channel blockers in the therapy of hypertension.


These drugs block voltage-gated (type “L”) calcium channels in a dose-dependent manner.

Role of calcium in atrioventricular (AV) nodal depolarization.


Conduction through the calcium-dependent portions of the AV node is dependent on extracellular calcium flux. The initial AV
nodal depolarization results in changes in membrane potential that result in the opening of voltage-gated (type “L”) calcium
channels. This allows the rapid influx of calcium into the cell, resulting in changes in membrane potential. Because the
membrane potential for calcium is more positive than the resting potential, the membrane becomes more excitable as
calcium enters the cell.

The therapeutic use of calcium channel blockers.


These agents are useful in the therapy of hypertension and angina (see Table). Cardio-selective drugs (e.g., verapamil) may
be useful in the therapy of arrhythmias. Lesser uses include therapy of drug resistance in oncologic therapy (e.g., verapamil)
and prevention of ischemic damage in stroke victims (e.g., lipophilic drugs such as nimodipine).

Drug Class Primary use Duration Protein Binding


Nifedipine Dihydropyridine Angina, hypertension 4 hours 90%
Nimodipine Dihydropyridine CNS 2 hours
Nicardipine Dihydropyridine Angina, hypertension., Raynaud’s 5 hours 95%
Nisoldipine Dihydropyridine Angina, hypertension 7 - 12 hours 99%
Nitrendipine Dihydropyridine 5 - 12 hours
Felodipine Dihydropyridine Hypertension 11 - 16 hours 99%
Amlodipine Dihydropyridine Angina, hypertension 35 hours 93%
Isradipine Dihydropyridine Hypertension 12 hours 99%
Verapamil Diphenylalkylamine Angina, hypertension, SVT 8 - 10 hours 90%
Diltiazem Benzothiazepine Angina, hypertension, PST 4 - 6 hours 70 - 80%
Bepridil Not classified — Stable angina 24 - 64 hours 99%
class I & 111
antiarrhythmic
properties

If the combination of a Ca++ blocker and a diuretic does not produce sufficient lowering of pressure, a calcium channel
blocker (e.g., verapamil, nifedipine) may be used. This class of drugs blocks type “L” calcium channels in the myocardium
and peripheral vasculature in a dose-dependent manner, resulting in decreased cardiac output and peripheral resistance.
The individual drugs-within the class vary as to effect on cardiac muscle and/or vasculature. Therefore, the drug used
dictates the relative decrease in cardiac output and / or TPR.
Nifedipine acts primarily at the level of the vasculature, and has little effect on the myocardium. Thus, the efficiency of the
heart would be increased, and the work of the heart decreased. With verapamil therapy, both cardiac conduction and
contractile force would be affected, resulting in a decrease in myocardial efficiency.

Bepridil, an atypical calcium channel antagonist, has antiarrhythmic activity, in addition to actions on the calcium channel.
In addition to blockade of calcium channels, blockade of both” sodium and potassium channels is manifested, which
result in unique actions on cardiac tissue.

Verapamil and diltiazem, as they have the highest efficacy in blockade of nodal calcium channel

Members of the dihydropyridine class of calcium channel blocker drugs (e.g., nifedipine, nisoldipine) have intrinsically
less effect on the cardiac conduction system than other classes. In particular, verapamil has a high degree of SA nodal

245
4 Self Assessment & Review Phar macology
Pharmacology
and AV nodal blockade, and diltiazem has a somewhat lower effect. Bepridil blocks both sodium and potassium chan-
nels, and also causes both. SA nodal and AV nodal blockade

Verapamil has a high degree of effect on nodal conduction, and therefore affects the action potential. Blockade of calcium
channels results in an increase in the time required to-reach .the membrane potential for calcium, resulting in a dose-
dependent lengthening of phase 2.

Common adverse effects of verapamil : Constipation, headache, pruritis, mild nausea, nervousness and peripheral
edema are most common. More serious effects, such as hypotension, bradycardia, and asystole may also result, particu-
larly if the drug is administered in combined therapy with a p-blocker.

Major drugs used as vasodilators, and the levels at which they act.
• Drugs that act primarily at the venous level. These include most nitrates.
• Drugs that act primarily at the arterial level (e.g., hydralazine and calcium channel antagonists). These are potent
vasodilators.
• Drugs that have actions at both arterial and venous levels. This group includes sodium nitroprusside beta-receptor
antagonists, and angiotensin-converting enzyme (ACE) inhibitors.

Use of ACE inhibitors in antihypertensive therapy : ACE inhibitors (e.g., captopril, enalapril, lisinopril, etc.) compete with
angiotensin I for binding of ACE, thus reducing the formation of angiotensin II. This results in decreased vasoconstriction.
Secretion of aldosterone is also reduced, resulting in decreased fluid volume, and vasodilatation is promoted through an
inhibition of the metabolism of bradykinin, a potent vasodilator. This produces a net vasodilatation, accompanied by a
decrease in plasma volume, resulting in a fall in blood pressure.
Clinically useful ACE inhibitors do NOT discriminate between KININASE and ACE.

CENTRALLY ACTING ANTIHYPERTENSIVE AGENTS


The mechanism of action of á-2 receptor agonists (e.g., clonidine) in the therapy of hypertension : These drugs act as
agonists the regulatory á 2-adrenergic receptor, both at the neuronal ending and at the level of the nucleus tractus solitarius
(NTS). Thus, release of norepinephrine is decreased.

The activation and mechanism of action of á-methyldopa : á-Methyldopa is taken into the adrenergic nerve terminus by the
catecholamine reuptake system. It is then taken into the adrenergic vesicle and enters the catecholamine synthesis path-
way. It is acted upon by dopa decarboxylase, and converted to a-methylnorepinephrine, a potent á-receptor agonist. The net
mechanism of the drug is therefore a centrally-mediated reduction in the release of norepinephrine.
This drug is particularly useful by decreasing renal vascular resistance. Blood flow to the kidney is thus not compromised,
and less end-organ damage is produced

Mechanism of action of reserpine : This drug depletes neuronal stores of norepinephrine. The drug enters the adrenergic
nerve terminus through the synaptic reuptake mechanism, and is transported into the adrenergic vesicle. Once in the
vesicle, it displaces norepinephrine from the vesicle to the cytoplasm, where it is degraded by monoamine oxidase (MAO)

DRUGS THAT ACT BY GANGLIONIC BLOCKADE :


Mechanism of action of trimethaphan : Trimethaphan is an antagonist at nicotinic ganglionic synapses. The drug thus
blocks transmission of information through autonomic pathways in a dose-dependent manner.

Adverse effects of trimethaphan : This drug is both sympathoplegic and parasympathoplegic, due to blockade of nicotinic
ganglia in both parasyrnpathetic and sympathetic pathways. Thus, effects on both systems are seen. Effects due to sympa-
thetic blockade include sedation, hypotension, and cardiovascular abnormalities, whereas those due to parasympathetic
blockade include dyspepsia and constipation.

Combination therapy in the treatment of essential hypertension : The first line therapy is usually a diuretic. The addition of
a beta-receptor antagonist to the regimen is preferred, although calcium channel blockers, ACE inhibitors, and angiotensin-
II-receptor antagonists may also be used, depending on the properties of the actual drug chosen. The important things to
consider are the age and state of health of the patient, as well as the mechanism of action of the drugs. Those drugs with
similar mechanisms of action (e.g., beta blockers and calcium channel blockers) would not be a good choice for combina-
tion therapy, while drugs with disparate mechanisms (e.g., ACE inhibitors and diuretics) would synergize therapeutically, and
allow lower doses of drug to be used. This would allow for greater efficacy and fewer adverse effects. In addition, doses

246
Phar macology of Car
Pharmacology diovascular & Renal System
Cardiovascular 4
would have to be adjusted for combination therapy, particularly in the geriatric or pediatric patient, and further adjusted in the
event that the physiologic effects of one drug would affect the clearance of the other (e.g., thiazides and ACE inhibitors).

HYPERTENSIVE EMERGENCIES
Def : (BP > 210/150mm Hg or DBP > 130 with end organ damage)
• Require MBP reduction of 20%-25% in first 1-2 hours
• Associated with significant organ damage
• Includes (Associated with elevated BP)

Encephalopathy Nitroprusside/ Nicardipine


Nephropathy Nitroprusside, Fenoldopam
Intracranial hemorrhage Nicardipine
AORTIC dissection Nicardipine/ Nitroprusside / Trimethaphan and 3 - blockers (esmolol)
Pre eclampsia / Eclampsia Labetalol / hydralazine
Pulmonary oedema Loop Diuretics/ Nitroprusside
Unstable angina IV NTG
MI Nitroprusside / esmolol

Drugs like Sublingual Nifedipine are contra-indicated (Increased MORTALITY)

THERAPY OF CARDIAC ARRHYTHMIAS


Mechanism of action of an antiarrhythmic used in the therapy of atrial arrhythmias : A drug used in the therapy of atrial
arrhythmias should decrease atrial conduction and should also cause partial blockade of the atrioventricular (AV) node. This
confines the arrhythmia to the atria and prevents the induction of ventricular arrhythmias. These drugs normally decrease
conduction in the sodium-dependent portion of the AV node

CLASS I ANTIARRHYTHMICS
Classified as class la antiarrhythmics : The prototypes for this class are quinidine, disopyramide, and procainamide. This
class also includes amiodarone and the tricyclic antidepressant imipramine, which has antiarrhythmic properties.

Mechanism of action of class I antiarrhythmics : These drugs block sodium channels. They essentially act as local
anesthetics, reducing the conduction rate and amplitude of electrical impulses,

Blood dyscrasias seen with class I antiarrhythmic drugs : Hematologic conditions caused by class I agents include
hemolytic anemia (especially in patients with glucose-6-phosphate dehydrogenase [G6PD1 deficiency), aplastic anemia,
leukopenia, agranulocytosis, and thrombocytopenic purpura.

Quinidine and procainamide have both been shown to cause a syndrome resembling lupus, Symptoms may include
polyarthritis, fever, and pleuritic chest pain.

These drugs lengthen the duration of the action potential.

The indirect actions of quinidine on the myocardium : Quinidine has antimuscarinic actions, and inhibits vagal tone, thus
the increase in sympathetic activity may precipitate an increase in conduction rate and ventricular tachycardia.

Cardiac reflex actions induced by quinidine : This drug mediates alfa-adrenoceptor blockade, particularly when adminis-
tered rapidly. This results in vasodilatation and reflex tachycardia.

Interaction of quinidine and drugs such as warfarin, digoxin, and carbamazepine : Quinidine is highly plasma protein-
bound (> 85 percent) and thus competes with other drugs [hat are highly protein-bound for storage sites on plasma proteins.
Thus, concurrent administration of quinidine with highly plasma protein-bound drugs such as benzodiazepines,
carbamazepine, warfarin and digoxin increases the plasma concentration of these drugs, leading to toxicity.

247
4 Self Assessment & Review Phar macology
Pharmacology
Major therapeutic use of lidocaine, as a class Ib antiarrhythmic : Lidocaine is used in the therapy of non-life-threatening
ventricular arrhythmias, such as non-sustained ventricular tachycardia and frequent premature ventricular beats. It is also
used as an adjunct to de-fibrillation and cardiopulmonary resuscitation (CPR) in patients with ventricular tachycardia and/or
fibrillation.
This drug has a very high first-pass effect and is degraded rapidly if given orally.

lidocaine toxicity : Initially, with high dose, the patient may experience drowsiness, vertigo, twitching, and dis-orientation. At
higher plasma concentrations (> 9 microgram/ml) psychosis, convulsions, and respiratory depression are seen.
Tocainide and mexiletine are orally active derivatives of lidocaine recently introduced.
This drug binds to both sodium and potassium channels, inhibiting recovery after repolarization of the membrane.

Potentially lethal hematologic effects seen with tocainide therapy : These effects are rare. They include neutropenia,
leukopenia, agranulocytosis, bone marrow depression, hemolysis, hypoplastic anemia, aplastic anemia, eosinophilia, or
thrombocytopenia.

Mechanism of action of mexiletine : This drug inhibits fast sodium channels in the myocardial cell membrane. Thus,
automaticity is decreased in His-Purkinje fibers. Conduction velocity is, however, not affected.

CLASS II ANTIARRHYTHMIC DRUGS


Mechanism of action of class II antiarrhythmics : These drugs decrease calcium flux in the AV node, and decrease the
excitability of cardiac tissue. This class includes (beta-receptor antagonists (e.g., esmolol) and pan-beta receptor antago-
nists (e.g., propranolol).

Antiarrhythmic therapy with nadolol : Nadolol has no partial agonist activity, no local anesthetic action, and is long-acting
properties desirable in a beta-receptor antagonist used in the therapy of arrhythmias.
Ideally, these drugs should be potent, selective for the beta receptor, have low to moderate lipid solubility, and be devoid of
agonist activity.

CLASS III ANTIARRHYTHMIC DRUGS


The mechanism of action of antiarrhythmics classified as class III : The drugs prolong the cardiac action potential through
increases in intracellular potassium. These drugs may achieve this result either by enhancing inward flux of potassium
through changes in sodium or calcium flux, or by blocking the outward flux of potassium.

Effect of class III antiarrhythmics on the cardiac action potential : Phase 2 of the cardiac action potential is prolonged, due
to the blockade of potassium flux decreasing the excitability of tissue and thus inhibiting the spontaneous formation of
ectopic foci.

Actions of bretylium on noradrenergic terminals : Bretylium releases norepinephrine from the sympathetic ganglia and
postganglionic adrenergic nerve terminals, and blocks norepinephrine reuptake.

The mechanism of action of amiodarone with those of quinidine and disopyramide : Quinidine primarily block’s sodium
channels in the depolarized state; disopyramide has an affinity for sodium channels in both depolarized and inactive states;
and amiodarone primarily blocks inactive sodium channels.

Adverse effects are seen with amiodarone : A multitude of adverse effects are common, including cardiac effects, neuro-
logic dysfunction (paresthesias, tremor, ataxia, cephalalgia), gastrointestinal (GI) dysfunction, lung dysfunction and hepato-
cellular necrosis.
CLASS IV ANTIARRHYTHMIC DRUGS
Mechanism of action of class IV antiarrhythmics : These drugs are cardiac calcium channel blockers. By decreasing
calcium conduction in the calcium-dependent portions of the AV node, His’ bundle and in the myocardium itself, ectopic foci
can be suppressed.

Effect of class IV antiarrhythmics on the cardiac action potential : These drugs block the influx of calcium. Thus, phase 2
of the cardiac action potential is prolonged, because an increased amount of time is required to reach the calcium potential.

248
Phar macology of Car
Pharmacology diovascular & Renal System
Cardiovascular 4
Calcium channel blocker is most effective in the therapy of paroxysmal atrial tachycardia : Verapamil, because it slows
conduction in the AV node, and prolongs the refractory period.

Actions of verapamil in the therapy of atrial flutter and atrial fibrillation : Due to partial blockade of the AV node, verapamil
slows ventricular response in atrial flutter and fibrillation.

MISCELLANEOUS ANTIARRHYTHMIC AGENTS


Adenosine is a potent vasodilator that is produced endogenously. It mediates an outward flow of potassium from adenos-
ine-sensitive potassium channels, stabilizing cardiac membranes. This results in a decrease in the duration of the atrial
action potential, as well as negative chronotropic and inotropic actions. In addition, by stabilizing excitable tissue in the AV
node, the drug effectively inhibits the conversion of paroxysmal atrial tachycardia to ventricular tachycardia, which could lead
to fibrillation.
Adenosine is most useful in the therapy of supra ventricular tachycardias such as Wolff-Parkinson-White syndrome. It is not
effective against “normal” atrial arrhythmias such as atrial flutter and atrial fibrillation

Duration of action of intravenous adenosine : The documented half-life is approximately 10 seconds.

Adenosine eliminated : Adenosine is taken up into erythrocytes and vascular endothelial cells and degraded by nucleases.

Magnesium as an antiarrhythmic : Magnesium has been-shown to affect the conduction of both sodium and potassium in
the heart, which may be due, at least in part, to the dependence of cardiac adenosine triphosphatase (ATPase) on magne-
sium. It also competes with calcium and thus decreases calcium conductance across the cardiac membranes, resulting in
a prolongation of phase II of the cardiac action potential. Given intravenously, it has been shown to decrease digitalis-
induced arrhythmias and torsades de pointes in susceptible patients, as well as arrhythmias produced by myocardial
ischemia (e.g., due to infarction).

DRUGS USED IN THE THERAPY OF HYPERLIPIDEMIA


Dietary cholesterol may be taken up to form a core lipoprotein (e.g., very-low-density lipoprotein [VLDL] core) and catabolized
in the liver to release free cholesterol, which is used in the synthesis of cell membranes and steroid hormones.
• Cholesterol may be synthesized in the liver by a synthetic pathway involving the actions of 3-hydroxy-3-methylglutaryl
coenzyme A (HMG-CoA) reductase.
• The production of both LDL receptors and HMG-CoA reductase is regulated at the level of transcription, by a feedback
loop involving levels of intracellular cholesterol.
• Elimination of cholesterol occurs mainly by production of bile by the liver. Free cholesterol is eliminated by hepatic
secretion into bile, and is also incorporated into bile salts.
• Niacin increases the clearance of VLDL, and increases levels of high-density lipoproteins (HDL). It also decreases the
synthesis of VLDL, which in turn results in lower levels of LDL.
• Niacin causes peripheral vasodilatation, resulting in flushing of the skin and lower pelvic area. This effect radically
decreases patient compliance, the gastrointestinal effects of niacin.
• Niacin may cause diarrhea and gastrointestinal (GI) discomfort, including nausea and vomiting.
• Niacin may cause hepatotoxicity with prolonged therapy. This is usually manifested as nausea and vomiting, with altered
liver enzyme profile.

BILE-BINDING RESINS
The bile-binding resins currently in use : Gemfibrozil, colestipol, and colesevelam

Mechanism of action of bile-binding resins : These drugs bind bile salts (which are cholesterol-based) in the small
intestine, preventing their reabsorption. This disruption of the enterohepatic recycling of bile salts forces the liver to use
exogenous cholesterol stores for the synthesis of new bile salts. This result in a cascade effect: the decrease in hepatic
cholesterol triggers an increase in LDL receptor expression, which results in removal of increased amounts of LDL from the
blood.

249
4 Self Assessment & Review Phar macology
Pharmacology
HMG-COA REDUCTASE INHIBITORS
Mechanism of action of HMG-CoA reductase inhibitors (e.g., simvastatin, atorvastatin) : These drugs inhibit HMG coen-
zyme A reductase, which is essential for the conversion of 3HMG coenzyme A to mevalonate, a precursor of cholesterol
inhibition is dose-dependent. The formation of HDL is decreased, resulting in a decrease in formation of sclerotic plaques.

Effect of statins (e.g., pravastatin, lovastatin) on carotid artery intima media thickness : This class of drugs has been
shown to significantly reduce intima media thickness with long-term therapy. Incidence of stroke, cardiovascular events, and
mortality are correspondingly decreased.

Adverse effects tend to be mild GI effects : dyspepsia, constipation and flatulence. More serious effects, such as renal
tubular obstruction, rhabdomyolysis and myopathy, have been reported. These are most likely to occur with oncurrent
therapy of other drugs that inhibit the metabolism of the drug (e.g., systemic anti-fungals or macrolide antibiotics) or with
consumption of grapefruit. Elevated liver enzymes (e.g., transaminase) may also be present.

DRUGS THAT AFFECT THE CLOTTING CASCADE


The clotting cascade is a sequential conversion of inactive plasma factors to active factors, resulting in the ultimate conversion
of soluble fibrinogen to insoluble fibrin. This fibrin forms the basis of clot formation

Clotting inhibited by:


The clotting cascade can be inhibited through several pathways.
These include :
• The inhibition of the conversion of profactors to active factors, which results in disruption of the clotting cascade.
• The inactivation of activated factors, which results in disruption of the clotting cascade.
• Interference with vitamin K recycling or reduction in vitamin K stores. Because the majority of clotting factors are vitamin
K-dependent, reduction of vitamin K levels will result in anticoagulation.
• Interference with platelet activation. This will result in inhibition of the formation of a platelet plug and lack of activation of
platelet activated clotting factors.
An antithrombotic drug is one that affects platelets. These drugs decrease platelet aggregation, and affect the activation of
platelets, thus decreasing the initiation of the clotting cascade.

Thrombolytic drug : A thrombolytic drug is one that actually disrupts an established clot. Unlike an anticoagulant, these
drugs are not prophylactic, but actually dissolve the fibrin web that forms the basis of the clot, causing dissolution

Hemostatic drug : These drugs decrease blood flow and thus blood loss

A hemoperfusion agent : These drugs decrease blood viscosity and increase the flow of blood, resulting in increased tissue
perfusion.

ANTICOAGULANT DRUGS
Primary mechanism of action of heparin : Heparin acts to potentiate antithrombin III, an endogenous substance that
inhibits the conversion of fibrinogen to fibrin. The drug induces a conformational change in antithrombin III, and binds
simultaneously to thrombin and antithrombin III, facilitating the inactivatton of thrombin

The secondary mechanisms of action of heparin : The conformational change in antithrombin III induced by heparin allows
the molecule to bind to, and inactivate, factors involved in the clotting cascade. Primary factors affected are the activated
forms of factor IX (“Christmas factor,” or plasma thromboplastin component) and X (“Stuart-Prower factor)
Heparin is a proteoglycan and would be destroyed by digestive enzymes. It is thus given by parenteral route

Types of heparin and their actions : Heparin and heparin-like compounds are available in a variety of molecular weight
fractions. The lower molecular weight heparins are not able to bind both antithrombin III and thrombin simultaneously, which
is necessary for the inactivation of thrombin. These drugs, consequently, affect the actions of factors IXa and Xa, rather than
thrombin.

Effect of heparin on bone : Heparin inhibits the carboxylation of bone proteins, and decreases the activity of vitamin K in
osteoblasts. It may increase the rate of bone loss with chronic therapy, through binding to osteoblasts and subsequent
decrease in osteoblast activity. Heparin may also inhibit the formation of bone in the fetus

250
Phar macology of Car
Pharmacology diovascular & Renal System
Cardiovascular 4
Half-life of heparin as a function of metabolism : Heparin is metabolized according to a saturable pathway. Thus, half-life
varies with dose. The half-life within the low therapeutic dose range is approximately 30 minutes. This may increase to as
much as 3 hours, with doses in the high therapeutic range. These values may increase in individual patients, in proportion
to the degree of plasma protein binding

Adverse effects of heparin therapy : The major adverse effect is bleeding. This may range from mild bruising to subarachnoid
hemorrhage. Thrombocytopenia is also a common adverse effect, particularly with the use of bovine heparin.

Heparin is known to inhibit aldosterone synthesis. This is not considered clinically significant. However the effects on
aldosterone may lead to hyperkalemia and metabolic abnormalities with long term therapy.

Antidote to heparin overdose : Protamine sulfate is used. This molecule is highly charged and binds tightly to the heparin
molecule, thus inhibiting pharmacological action

The advantage of therapy with enoxaparin : Enoxaparin and other low molecular weight heparins (e.g., ardeparin, dalteparin)
produce a more predictable anticoagulant response than (unfractionated) heparin. This is a result of increased bioavailability,
particularly after subcutaneous injection, longer half-life, and dose-independent clearance.

Dalteparin and enoxaparin : These drugs have much less plasma protein binding activity than do the higher molecular
weight (unfractionated) heparins. These drugs also have decreased binding to platelets, and factors IV and V, thus producing
-less thrombocytopenia.

Mechanism of action of warfarin : Warfarin disrupts the recycling of vitamin K epoxide form low active vitamin K. This results
in depletion of available vitamin K stores, and eventually to lack of activation of vitamin K-dependent clotting factors—
particularly factors II, VII, IX and X (recall that vitamin K is converted to an inactive epoxide form after interaction with platelet
factors, and must be “recycled” back to its active form in the liver.

Paradox of warfarin actions : Warfarin, like other vitamin K inhibitors, inhibits the activity of platelet-derived clotting factors
(II, VII. IX and X). This results in anticoagulant action. Paradoxically, it also inhibits the synthesis of endogenous proteins with
anticoagulant activity such as protein C and protein S.

The drug may take up to 4 days to produce a pharmacologic effect. Two things must happen physiologically before the effects
of the drug are seen:
• Existing hepatic stores of vitamin K must first be depleted. Inhibition of recycling by the drug will only deplete circulating
vitamin K, and the pharmacologic action of the drug will not be seen all the stores of active vitamin K have been
exhausted.
• Activated platelet-derived factors must undergo normal catabolism. Factors that are already activated do not depend on
the presence of vitamin K, so the effect of the drug is only seen after normal catabolism of activated factors.
The anticoagulant effects of warfarin are stereoselective; the S-isomer of warfarin is 3 to 5 times more potent than the R-
isomer

Protein binding of warfarin and its ramifications.


Warfarin is heavily plasma protein-bound (> 97 percent). It therefore, interacts with other protein-bound drugs (e.g.,
carbamazepine, phenytoin, and benzodiazepines) in that binding of warfarin to plasma proteins may displace bound drug
that is administered as concurrent therapy. Thus, toxicity may result, due to elevated plasma levels, and the clearance rate
of the drug may (increase to compensate)
Several drugs may increase or decrease the activity of warfarin—primarily through similar actions or by the alteration of
warfarin metabolism. These include:
• Phenylbutazone and related (sulfapyridine) drugs. The activity of these drugs on platelet action is synergistic with the
actions of warfarin. In addition, these drugs increase the pharmacologic effects of warfarin by displacement of protein
binding.
• Aspirin and other antipiatelel drugs synergize with the actions of warfarin.
• Barbiturates may increase the metabolism of warfarin through induction of hepatic cytochrome P450.
• Cimetidine, metronidazole and fluconazole affect metabolism of warfarin. Some antibiotics (e.g., third generation
cephalosporins) affect the populations of vitamin K-producing bacteria in the gut, causing increased amounts of vitamin
K to become available. This interferes with the action of warfarin.

251
4 Self Assessment & Review Phar macology
Pharmacology
Heparin acts very quickly to inhibit clot formation, whereas warfarin has a slow onset. Conversely, upon withdrawal of the
drug, heparinized patients quickly recover, whereas the withdrawal of warfarin will have no appreciable effect for as much
as several days.
• Both drugs have an indirect action on the clotting cascade—heparin must form a complex with antithrombin III in order to
produce an effect, and warfarin acts through depletion of vitamin K, which affects specific clotting factors.
• Warfarin may be administered orally or parenterally, whereas heparin must be administered by slow infusion (e.g.,
subcutaneous injection or intravenous [IV] admixture).
• Both drugs are highly plasma protein-bound. However, the binding of heparin is less specific than that of warfarin in that
the drug will bind to any positively charged protein, including those on cell membranes.
• Warfarin is metabolized in the liver, whereas heparin is depolymerized in endothelial cells and within macrophages. The
elimination of heparin is primarily renal.

ANTITHROMBOTIC AGENTS
Aspirin irreversibly acetylates cyclo-oxygenase, the rate-limiting enzyme in the arachidonic acid cascade (prostanoid synthesis)
Actions of aspirin on the production of platelet thromboxane : By inhibiting cyclooxygenase (COX-1), aspirin and similar
drugs (e.g., indomethacin) inhibit the synthesis of thromboxanes by platelets in a dose-dependent manner. This results in
a decrease in platelet aggregation and decreased local vasoconstriction

Effect of aspirin on the vascular endothelium : The vascular endothelium preferentially produces prostacyclins (e.g., PGI2)
rather-than prostaglandins or thromboxanes. Prostacyclins mediate a decrease in platelet aggregation (“stickness”) and
local vasodilatation. Aspirin decreases the synthesis of prostacyclins by the vascular, endothelium.

Net vasodilatation and decreased platelet activation be seen with low-dose aspirin-but not high-dose aspirin : Aspirin
irreversibly acetylates cyclooxygenase. Thus; enzyme exposed to the drug is permanently nonfunctional. Also, platelets are
not entire-cells—they are fragments of megakaryocytes, .and thus have no cellular “machinery;” they are not capable of
synthesizing new enzyme.
With either low-dose or high-dose aspirin, both endothelial and platelet cyclooxygenase are platelet thromboxane synthesis
is permanent—new platelets must be produced in order for thromboxane to be synthesized. The net effect is vasodilatation
with a decrease in platelet aggregation. With high-dose aspirin, both platelet and endothelial cell prostanoid production is
inhibited, as new enzyme produced by endothelial cells is rapidly acetylated by circulating levels of drug.
Abciximab inhibits platelet glycoprotein (GP) Ilb/IIIa binding to fibrinogen, as well as von Willebrand’s factor and other
adhesive molecules. This decreases platelet aggregation (“stickiness”).

Mechanism of action of ticlopidine : Ticlopidine interferes with the ADP-induced binding of fibrinogen to the platelet membrane
at specific receptor sites. This interferes with platelet aggregation

THROMBOLYTIC DRUGS
Alteplase is an enzyme derived from human melanoma- It is a tissue plasminogen activator.
Mechanism of action of alteplase : This enzyme binds to fibrin in an existing clot, and activates the conversion of plasminogen
to plasmin. The plasmin then lyses the fibrin. The clot, without its fibrin support, disintegrates.
Reteplase is a “fibrin selective” tissue plasminogen activator (TPA). It is activated by the presence of fibrin. Thus, if no clot is
present, the drug is inactive.
Reteplase acts from within the clot, and so must penetrate the clot before it is active. TPA binds to the fibrin matrix on the
outside of the clot. Thus, higher clot lysis rates are reported for reteplase than for TPA

Mechanism of action of streptokinase : Streptokinase forms an activator complex with plasminogen, which results in the
cleavage of the Arg-Val bond in plasminogen, and conversion of plasminogen to plasmin. The activator complex also
diffuses into the clot and activates preplasmin-2, a mediator that also lyse fibrin.

252
Phar macology of Car
Pharmacology diovascular & Renal System
Cardiovascular 4
ALL INDIA c. Hyperthyroidism
d. Systemic lupus erythematous
1. Torsades de pointes is caused by all except : 8. Bosentan is a : [AI 03]
a. Quinidine [AI 08] a. Serotonin uptake inhibitor
b. Disopyramide b. Endothelin receptor antagonist
c. Procainamide c. Leukotriene modifier
d. Lidocaine d. Calcium sensitizer
2. Gpllb/IIIa antagonist are all except : [Al 07] 9. All of the following are useful intravenous therapy
for hypertensive emergencies except : [AI 03]
a. Abciximab
a. Fenodolpam
b. Clopidogrel
b. Urapidil
c. Tirofiban
c. Enalapril
d. Eptifibatide
d. Nifedipine
3. Nesiritide is : [Al 07; 05]
10. Haemorrhage secondary to heparin adminis-
a. Brain natriuretic peptide analogue tration can be corrected by administration of :
b. Endothelin receptor antagonist a. Vitamin K [AI 03]
c. Gp IIb/IIIa antagonist b. Whole blood
d. TNF alpha antagonist c. Protamine
4. One of the following is not true about nesiritide : d. Ascorbic acid
a. It is a brain natriuretic peptide analogue 11. The mechanism of action of sodium nitroprusside:
b. It is used in acutely decompensated heart fail- a. Increased cAMP [AI 02]
ure
b. Increased guanylate cyclase
c. It has significant oral absorption [AI 05]
c. Calcium channel blockage
d. It has a short half-life
d. K+ Channel opener
5. The following drugs have significant drug interac-
tion with digoxin except : [AI 05] 12. The primary mechanism of action of fluoride on
topical application is: [AI 02]
a. Cholestyramine
a. Conversion of hydroxyapatite to fluoroapatite by
b. Thiazide diuretics replacing the OH ions
c. Quinidine b. Inhibition of plaque bacteria
d. Amlodipine c. Form a reservoir in saliva
6. A 60 year old man with rheumatic mitral stenosis d. Improvement in tooth morphology
with atrial fibrillation is on therapy for a fast ven-
tricular rate. While on treatment, he developed a 13. Digoxin is contraindicated in: [AI 02]
regular pulse of 64/min. The most likely drug be- a. Supraventricular tachycardia
ing administered was : [AI 04] b. Atrial fibrillation
a. Verapamil c. Congestive heart failure
b. Digoxin d. Hypertrophic obstructive cardiomyopathy
c. Carvedilol 14. All of the following statements are true regarding
d. Propranolol Losartan except: [AI 02]
7. Which of the following is not an adverse effect of a. It is a competitive angiotensin receptor antago-
chronic amiodarone therapy : [AI 04] nist
a. Pulmonary fibrosis b. It has a long acting metabolite
b. Hypothyroidism c. Associated with negligible cough
d. Causes hyperuricemia

Answer 1. d. Lidocaine 2. b. Clopidogrel 3. a. Brain ... 4. c. It has ... 5. d. Amlodipine


6. b. Digoxin 7. d. Systemic ... 8. b. Endothelin ... 9. d. Nifedipine 10. c. Protamine
11. b. Increased ... 12. a. Conversion ... 13. d. Hypertrophic ... 14. d. Causes ...

253
4 Self Assessment & Review Phar macology
Pharmacology
15. Digoxin is not indicated in: [AI 01] 22. Spironolactone is contraindicated which of the
a. Atrial flutter following: [AI 00]
b. Atrial fibrillation a. Enalapril
c. High output failure b. Atenolol
d. PSVT c. Verapamil
16. A diabetic female on INH and rifampicin for TB d. None of the above
suffers DVT: She is started on warfarin : PT is not 23. Enalapril is contraindicated in all of the following
raised; next step should be : except: [AI 00]
a. Long term heparin therapy [AI 01] a. Diabetic nephropathy with albuminuria
b. Replace warfarin with acesoumarin b. Single kidney
c. Switch ethambutol for rifampin c. Bilateral renal artery stenosis
d. Use LMW heparin d. Hyperkalemia
17. All of the following statements about ticlopidine 24. Quinidine is a: [AI 99]
are true except: [AI 01] a. Na+ channel blocker
a. Directly interacts with platelet membrane, Gp IIb b. K+ channel blocker
/ IIIa receptors
c. Ca+ channel blocker
b. Duration of action is long
d. CI– channel blocker
c. It is used as an alternative to aspirin in patients
25. All of the following statements are true about ni-
with cerebrovascular disease
trates except: [AI 99]
d. Onset of action is delayed
a. It releases NO
18. Coronary steal phenomenon’ is caused by:
b. It causes vasodilatation
a. Dipyridamole [AI 00]
c. It decreases AV conduction
b. Diltiazem
d. It has high first pass metabolism
c. Propranolol
26. Dipyridamole acts by: [AI 99]
d. Verapamil
a. Adenosine uptake inhibition
19. The Nitrate which does not undergo first pass
b. Inhibiting thromboxane A2
metabolism is: [AI 00]
c. Stimulating PGI2 synthesis
a. Isosorbide mononitrate
d. Inhibiting PGI2 synthesis
b. Nitroglycerine
27. Which of the following antiarrhythmic drugs
c. Pentaerythritol tetranitrate
causes prolonged repolarization of ventricles and
d. Isosorbide dinitrate ERP: [AI 98]
20. A 6 year old child presents with malignant hyper- a. Amiodarone
tension. The drug of choice is: [AI 00]
b. Propranolol
a. Na nitroprusside
c. Verapamil
b. Sublingual nifedipine
d. Quinidine
c. Frusemide
28. Which of the following statements regarding ad-
d. Enalapril enosine is not true: [AI 98]
21. Which one of the following drug causes increased a. Used in PSVT
concentration of Na+ and Cl- in urine with normal
b. Administered as rapid I.V. infection
bicarbonate: [AI 00]
c. Has short lived side effects
a. Ethacrynic acid
d. Disopyramide increases its therapeutic effect
b. Frusemide
c. Acetazolamide
d. Bumetanide
Answer 15. c. High output ... 16. c. Switch ... 17. a. Directly ... 18. a. Dipyridamole 19. a. Isosorbide ...
20. a. Na nitroprusside 21. a. Ethacrynic ... 22. a. Enalapril 23. a. Diabetic ... 24. a. Na+ channel ...
25. c. It decreases ... 26. a. Adenosine ... 27. a. Amiodarone 28. d. Disopyramide ...

254
Phar macology of Car
Pharmacology diovascular & Renal System
Cardiovascular 4
29. Low molecular weight heparin therapy is associ- b. ACE inhibitors
ated with all except: [AI 98] c. Hydralazine
a. Less chances of bleeding d. Sodium nitroprusside
b. Single dose per day 37. Drugs not used in myocardial infarction are:
c. Easy filterability by glomerular capillaries a. Inhibitors of platelet aggregation [Al 96]
d. High biological interaction to plasma proteins b. Thrombolytics
30. All of the following statements about antianginal c. Anticoagulants
action of nitrates are true except: [AI 97]
d. Inhibitors of Plasminogen activator
a. ↓ Myocardial O2 consumption
38. Which of the following is NOT an inotropic drug:
b. ↓ Both pre and after load
a. Dopamine [Al 95; AIIMS June 97]
c. ↑ Total coronary flow
b. Isoprenaline
d. Cause favorable redistribution of coronary flow
c. Amrinone
31. Drug of choice in PSVT is: [AI 97; 95]
d. Amiodarone
a. Verapamil
b. Propranolol
AIIMS
c. DC Shock
d. Digoxin
39. Free water clearance decreased by:
32. Predominant arteriolar dilators include all of the
a. Vincristine [AIIMS May 08]
following except: [AI 97]
b. Vinblastine
a. Sodium Nitroprusside
c. Chlorpropamide
b. Diazoxide
d. Freusamide
c. Hydrallazine
40. About Furesemide true is: [AIIMS Nov. 07]
d. Minoxidil
a. Acts on PCT
33. Which of the following antihypertensive drugs is
devoid of any central action: [AI 97] b. Given only by parenteral route
a. Clonidine c. Used in pulmonary oedema
b. α methyl dopa d. Mild diuretic
c. Propranolol 41. Thiazide causes all except: [AIIMS Nov. 07]
d. Indapamide a. Hyperglycemia
34. All of the following are calcium channel blockers b. Increased calcium excretion
except: [Al 96] c. Increased blood uric acid levels
a. Nimodipine d. Used in CCF
b.. Verapamil 42. Mechanism of action of nitric oxide is :
c. Flunarizine a. Increase CAMP [AIIMS Nov. 07]
d. Pirenzepine b. Increase cGMP
35. ADH acts on: [Al 96] c. Increase IP3/DAG
a. Proximal convoluted tubule d. Non-adrenergic/non-cholinergic
b. Distal convoluted tubule 43. All of the following utilize NO except:
c. Loop of Henle a. Hydralazine [AIIMS Nov. 07]
d. Collecting duct b. Sildenafil
36. All the following drugs decrease the preload ex- c. Glyceryl nitrate
cept: [Al 96] d. Minoxidil
a. Glyceryl tri-nitrate

Answer 29. d. High biological ... 30. c. ↑ Total ... 31. a. Verapamil 32. a. Sodium ... 33. d. Indapamide
34. d. Pirenzepine 35. d. Collecting duct 36. c. Hydralazine 37. d. Inhibitors ... 38. d. Amiodarone
39. d. Freusamide 40. c. Used in ... 41. b. Increased ... 42. b. Increase ... 43. d. Minoxidil

255
4 Self Assessment & Review Phar macology
Pharmacology
44. All of the following cause hypertension except: 52. All of the following are the indication for use of
a. NSAID [AIIMS May 07] ACE inhibitors, except: [May 05]
b. Erythropoietin a. Hypertension
c. Cyclosporine b. Myocardial infarction
d. L-dopa c. Left Ventricular Dysfunction
45. All cause acute interstitial nephritis except: d. Pheochromocytoma
a. Diuretics [AIIMS May 07] 53. Use of the following drug to treat hypertension
with pregnancy is contraindicated : [Dec. 04]
b. β lactam antibiotics
a. Enalapril
c. Allopurinol
b. Methyldopa
d. INH
c. Nifedipine
46. Digoxin action is influenced by all except:
d. Labetolol
a. Electrolyte disturbances [AIIMS May 07]
54. One of the following statements about hydrala-
b. Myocardial ischemia
zine is not true: [Dec. 04]
c. Liver failure
a. It causes direct relaxation of blood vessels
d. Renal failure
b. It causes dilatation of both arteries and veins
47. Drug used to prevent renal toxicity in case of alco-
c. Postural hypotension is not a common problem
holic hepatitis: [AIIMS May 07]
d. It increases plasma renin activity
a. Silymarin
55. Urgent reversal of Warfarin therapy can be done
b. S adenosyl methionine
by administration of: [Dec. 04]
c. Thalidomide
a. Cryoprecipitates
d. Pentoxyphylline
b. Platelet concentrates
48. All of the following act by increasing nitric oxide
c. Fresh frozen plasma
except: [AIIMS May 07]
d. Packed red blood cells
a. Sodium nitroprusside
56. One of the following diuretics does not require its
b. Hydralazine
presence in the tubular lumen for its pharmaco-
c. Fenoldopam logical effects: [Dec. 04]
d. Nitroglycerine a. Thiazide diuretics
49. Antiarrhythmic that does not belong to class IC: b. Loop diuretics
a. Propafenone [AlIMS Nov. 06] c. Carbonic anhydrase inhibitors
b. Flecainide d. Aldosterone antagonists
c. Tocainide 57. Sparfloxacin and terfenadine can cause:
d. Encainide a. Ventricular arrhythmia [Nov. 03]
50. The most significant adverse effect of ACE inhibi- b. Myopathy
tion is: [AlIMS May 06]
c. Electrolyte imbalance
a. Hypotension
d. Nephropathy
b. Hypertension
58. The diuretic group that does not require access
c. Hypocalcemia
to the tubular lumen to induce diuresis is:
d. Hypercalcemia rate a. Carbonic anhydrase inhibitor [May 03]
51. Inhaled nitric oxide is used: [AlIMS May 06] b. Na-Cl symport inhibitor
a. For stabilizing systemic heamodynamics c. Mineralocorticoid antagonist
b. In case of jaundice d. Na-K symport inhibitor
c. To prevent CNS complication
d. For reducing pulmonary hypertension

Answer 44. d. L-dopa 45. d. INH 46. c. Liver failure 47. d. Pentoxyphylline 48. c. Fenoldopam
49. c. Tocainide 50. a. Hypotension 51. d. For reducing ... 52. d. Pheochromo ... 53. a. Enalapril
54. b. It causes ... 55. c. Fresh frozen ... 56. d. Aldosterone ... 57. a. Ventricular ... 58. c. Mineralocor ...

256
Phar macology of Car
Pharmacology diovascular & Renal System
Cardiovascular 4
59. The following statement is not true about the use b. Mexilitine
of clonidine in the treatment of hypertension: c. Beta - blocker
a. Reduction in the treatment of hypertension
d. Radio - frequency ablation technique
b. Increase in LDL-cholesterol on prolonged use
66. All are true about streptokinase and urokinase,
c. Sedation and xerostomia are common side ef- except: [Nov. 99]
fects [May 03]
a. Infection with streptococcus causes beneficial
d. It can be combined with vasodilators dual effect
60. Which of the following diuretics decrease the re- b. Reduces chances of arterial and venous throm-
nal lithium clearance: [Nov. 03] bosis
a. Acetazolamide c. Control is done with thrombin time
b. Hydrochlorthiazide d. Mechanism of action is activation of plasmino-
c. Furosemide gen
d. Spironolactone 67. All are true regarding antiarrhythmic verapamil
61. The following drugs cause hypertensive crisis in except: [June 99]
pheochromocytoma except: [May 02] a. Verapamil belongs to second group drugs
a. Phenoxybenzamine b. It is drug of choice for PSVT
b. Propranolol c. It is contraindicated in complete heart block
c. Saralasin d. It depresses heart rate
d. Captopril 68. A lady Sati getting rifampicin and warfarin devel-
62. A patient is having benign prostatic hyperplasia. ops pulmonary thromboembolism. Which of the
He is having a blood pressure of 180/110 mm-Hg following is most suitable management:
and a serum creatinine of 3.5 Which of the follow- a. Substitute warfarin for long term heparin therapy
ing drugs is NOT to be used to treat him: b. Low molecular weight heparin [June 99]
a. Prazosin [Dec. 00] c. Replace warfarin for acetacumarin
b. Enalapril d. Replace rifampicin for ethambutol
c. Amlodepine 69. All are true about losartan, except: [June 98]
d. Metoprolol a. Angiotensin II antagonist
63. If a patient on digitals develops ventricular tachy- b. Causes hyperuricemia
cardia, which of the following drugs you will NOT
c. Does not cause cough
use in management: [Dec. 00]
d. Long acting metabolites
a. Amiodarone
70. Regarding milrinone, all are true except:
b. Quinidine
a. Thrombocytopenia is rare [June 98]
c. Phenytoin
b. Phosphodiesterase 3 inhibitor
d. Lidocaine
c. Antiarrhythmic action
64. Chronic thiazide therapy causes persistent hyper-
calcemia due to: [June 00] d. Given for long duration
a. Renal tubular acidosis 71. All are fibrinolytic, except: [June 98]
b. Fanconi’s syndrome a. Streptokinase
c. Hypervitaminosis D b. Urokinase
d. Hyperparathyroidism c. Alteplase
65. A 50 year old man had an attack of myocardial d. Epsilon amino caproic acid
infarction arid developed ventricular ectopics and
low ejection fraction. Which of the following anti-
arrhythmic drug to be given: [June 00]
a. Flecainide
Answer 59. b. Increase in ... 60. b. Hydrochlor ... 61. a. Phenoxyben ... 62. b. Enalapril 63. b. Quinidine
64. d. Hyperparathy ... 65. c. Beta - blocker 66. a. Infection with ... 67. a. Verapamil ... 68. c. Replace ...
69. b. Causes ... 70. d. Given for ... 71. d. Epsilon ...

257
4 Self Assessment & Review Phar macology
Pharmacology
72. Nifedipine and beta - blockers are given together: 79. Mg ++ administered in: [PGI June 06]
a. To decrease pedal edema due to nifedipine a. Eclampsia
b. To overcome increased sympathetic activity of b. Cardiac arrhythmia
nifedipine [Sep. 96] c. Seizure
c. Anti CHF action of propranolol d. Tetany
d. Antiarrhythmic effect of nifedipine 80. Which of the following drugs causes neutrophilia?
73. Altered taste sensation is caused by: [May 95] a. Epinephrine [Dec. 05]
a. Pefloxacin b. Glucocorticoids
b. Rifampicin c. NSAID
c. Ciprofloxacin d. Clozapine
d. Captopril e. Methotrexate
81. Drug to be given within 6 hours of MI:
PGI a. Aspirin [Dec. 04]
b. Metoprolol
74. Digoxin is used in all except: [PGI Dec. 07] c. Streptokinase
a. Decompensated heart failure d. Diltiazem
b. HOCM e. Statin
c. Supraventricular tachycardia 82. Antihypertensive drugs beneficial or neutral role
d. Myocarditis in Lipid metabolism: [Dec. 03]
75. ECG changes in digitalis toxicity all except: a. Prazosin
a. T wave inversion [PGI Dec. 07] b. Propranolol
b. Diminished T wave amplitude c. Furosemide
c. Conduction block d. Losartan
d. ST depression in proximal part e. Chlorthiazide
76. Drugs causing pericarditis all except: 83. True about Quindine : [Dec. 03]
a. Methysergide [PGI Dec. 06] a. It increases effective refractory period
b. Hydralazine b. Used in hypertension.
c. Amiodarone c. Causes paradoxical tachycardia
d. Bretylium d. It decreases absolute refractory period
e. Minoxidil e. Cinchonism is seen
77. Carbonic anhydrase inhibitor not given in: 84. True about vitamin K: [June 03]
a. Sulfonamide hypersensitivity [PGI Dec. 06] a. ↑ ed synthesis of factor II, VII, IX & X
b. Glaucoma b. Doesn’t require exposure to sunlight
c. Ccontraindicated in high altitude sickness c. Cause hemolytic anemia
d. Contraindicated in metabolic acidosis d. T ½ is < 6 hour
e. COPD 85. Arteriolar dilators used in treatment of CCF include:
78. Digoxin toxicity aggravated in: [PGI June 06] a. Hydralazine [June 02]
b. Nifedipine
a. Hypokalemia
c. Prazosin
b. Hyperkalemia
d. Enalapril
c. Hypercalcemia
e. Nitrates
d. Hypermagnesemia

Answer 72. b. To overcome ... 73. d. Captopril 74. a, b and d 75. b. Diminished ... 76. c and d
77. a, d and e 78. a and c 79. a, b, c and d 80. a, b, and c 81. a, b and c
82. a and d 83. a, c and e 84. a and c 85. a and b

258
Phar macology of Car
Pharmacology diovascular & Renal System
Cardiovascular 4
86. A patient was started on antihypertensive medi- a. HOCM
cations, developed renal failure, the drug offended b. High output failure
is: [June 02]
c. AF with high ventricular rate
a. Beta - blocker
d. All
b. Alfa - blocker
94. Pharmacological defibrillator is: [Dec. 99]
c. Calcium channel blocker
a. Adenosine
d. ACE inhibitor
b. Bretylium
e. Clonidine
87. Combination use of beta blockers and calcium c. Lignocaine
channel blockers cause: [Dec. 01] d. Amiodarone
a. Heart block 95. True about triamterene is A/E: [Dec. 99]
b. Hypertension a. Saluretic effect is greater than thiazide
c. Hypotension b. Triamterene is often combined with thiazide
d. Bradycardia c. Action is just like amiloride
e. Tachyarrhythimias d. It is a K+ sparing diuretic
88. Potassium sparing diuretics include: 96. K+ channel opener is: [Dec. 99]
a. Spironolactone [Dec. 01] a. Verapamil
b. Triamterene b. Nicorandil
c. Amiloride c. Nitroprusside
d. Ethacrynic acid
d. Amrinone
e. Bumetanide
97. Enalapril acts by: [June 99]
89. ACE inhibitors cause: [Dec. 01]
a. Angiotensin converting enzyme inhibition
a. Persistent cough
b. Angiotensin receptor blockade
b. Taste changes
c. Calcium channel blocker
c. First dose hypotension
d. Direct vasodilation
d. Ankle edema
98. Mannitol when given I/V causes: [June 98]
e. Angioedema
a. ↑ Blood viscosity
90. About the use of nitrates all are true except:
b. ↓ Blood viscosity
a. In HOCM - not given [June 00]
c. ↓ GFR
b. Acute M.I. can be given
d. ↑ ICT
c. CHF and acute LVF
99. Not true about heparin is: [June 98]
d. Methemoglobinemia
a. Activates antithrombin III
91. About plasma expanders all are true except
b. Small unionized molecule are not absorbed
a. Dextran interferes with platelet aggregation
orally
b. Albumin is have 69 AA [June 00]
c. Protamine sulphate always used to reverse its
c. Half life of albumin - 15 hours action
d. Amylopectin d. Release liproprotein lipase from vessel wall and
92. Among following least anaphylaxis is seen with: tissues
a. Streptokinase [June 00] 100. Osmotic diuretics cause following except:
b. Urokinase a. Inhibit Renin release [June 98]
c. Anisolated streptokinase b. ↑ intravascular volume
d. tPA c. Reduce salt reabsorption in ascending limb
93. Digoxin is used in CHF due to: [Dec. 99] d. None of the above

Answer 86. d. ACE inhibitor 87. a and d 88. a, b and c 89. a, b, c and e 90. d. Methemoglo ...
91. c. Half life of ... 92. d. tPA 93. c. AF with ... 94. b. Bretylium 95. a. Saluretic ...
96. b. Nicorandil 97. a. Angiotensin ... 98. a. ↑ Blood ... 99. b. Small ... 100. d. None of the ...

259
4 Self Assessment & Review Phar macology
Pharmacology
101. In chronic renal failure which one is contraindi- 103. Digitalis has inotropic action due to: [Dec 97]
cated: [Dec 97] a. Initiation of Na K ATPase
+ +

a. Furosemide b. Trapping Ca++ release


b. Ethacrynic acid c. Inhibiting Na+ K+ ATPase
c. Triamterene d. Increase in intra cellular K+
d. Bumetanide 104. Best used in digoxin induced arrhythmia:
102. Most powerful coronary vasodilator is: a. Phenytoin [June 97]
a. Adenosine [Dec 97] b. Lignocaine
b. CO2 c. Quinidine
c. Hypoxia d. Procainamide
d. Hypertension

Answer 101. c. Triamterene 102. c. Hypoxia 103. c. Inhibiting Na+ ...


104. b. Lignocaine

260
Phar macology of Car
Pharmacology diovascular & Renal System
Cardiovascular 4
ANSWERS, REFERENCES, EXPLANATIONS WITH INFORMATIVE ILLUSTR ATIONS

1. Ans. is d i.e. Lidocaine Ref. KDT 6/e, p 158, 692, 510, 512, 516, 519; Katzung 9/e, p 1118

Drugs causing Prolonged QT Interval / Torsades de pointes


Antiarrhythmic Antihistaminics CYP 3A4 inhibitors Others

– Quinidine – Terfenadine – Ketoconazole – Imipramine


– Amiodarone – Astemizole – Fluconazole – Amitriptyline
– Procainamide – Erythromycin
– Disopyramide – Haloperidol
– Cisapride

Torsades de pointes caused by these combinations

– Terfenadine + Any drug which inhibits CYP 3A4


– Cisapride + Erythromycin / Ketoconazole
– Quinidine + Procainamide
– Quinidine + Mefloquine
– Halofantrine + Mefloquine

2. Ans. is b i.e. Clopidogrel Ref. KDT 6/e, p 610

Drugs which selectively blocks GPIIb/ IIIa receptor : • Abciximab


Mnemonic: ATE or EAT • Eptifibatide
• Tirofiban
• Ticlopidine and clopidogrel block the ADP receptor on platelets and thus prevent the cascade resulting in
activation of the glycoprotein IIb/ IIIa receptor that leads to fibrinogen binding to the platelet and consequent
platelet aggregation.

Inhibits fibrinogen binding to platelets without altering GPIIb/ IIIa receptor.
• Ticlopidine is more effective than aspirin, however, it has the disadvantage of causing diarrhea, skin rash, a
low incidence of neutropenia and TTP.
• Ticlopidine accumulates in the body with peak effect after 8-10 days therapy.

t½ – 8 hrs after single dose
t½ – 8 days after multiple doses
• It can be used in CVA as an alternative to aspirin.

261
4 Self Assessment & Review Phar macology
Pharmacology

3. Ans. is a i.e. Brain natriuretic peptide analogue


Ref. CMDT ‘05, p 383; Harrison 17/e, p 1453; KDT 6/e, p 507

• Nesiritide is a recombinant form of human BNP (Brain natriuretic peptide) which is an endogenous peptide
secreted primarily from the left ventricle in response to an increase in wall stress. It dilates the arterial and
venous circulation in a balanced manner.
• Nesiritide is a synthetic B-type natriuretic peptide manufactured from E. coli by recombinant DNA technology.

Option ‘c’ It is only available for parenteral administration (low oral bioavailability). It is given as a bolus
(2 μg/Kg) followed by a fixed dose infusion (0.01-0.03μg/Kg/min).
Option ‘d’ It has a short half life :
– Distribution half life of 2 min with steady serum levels reaching after 90 min of continuous
infusion (at a rate ranging from 0.01 to 0.03 μg/kg/min).
– Mean terminal half life of 20 min.
Option ‘b’ The primary indication is hospitalized patients with acutely decompensated CHF who have
failed intravenous diuretic therapy, but have severe fluid and sodium retention and a systolic
blood pressure > 90 mmHg.

262
Phar macology of Car
Pharmacology diovascular & Renal System
Cardiovascular 4
Natriuretic family

ANP BNP CNP Urodilatin


(Cardiac atrial) (Cardiac ventricle) • No Natriuresis • Extended form of ANP
• Atrial natriuretic • Brain natriuretic • No diuresis
peptide peptide • Potent vasodilator

• Diuretic
• Natriuretic
• Vascular relaxation
• Inhibition or RAS, SNS

Adverse effects of Nesirtide is : • Hypotension


• Renal dysfunction

BNP dilates coronary arteries but is not associated with coronary steal phenomenon.
- Endothelin receptor antagonist : Bostentan
- GPIIb / IIIa antagonist : Abciximab, Eptifibatide
- TNFα antagonist : Etanercept, Infliximab, Adalimumab

4. Ans. is c i.e. It has significant oral absorption Ref. CMDT ‘05, p 383; Harrison 17/e, p 1453

Already explained, refer answer no. 3

5. Ans. is d i.e. Amlodipine Ref. KDT 6/e, p 499

No book mentions any drug interactions of amlodipine with digoxin.


Cholestyramine binds digoxin and reduces absorption of digoxin.

Digitoxin metabolism accelerated by Hypokalemia precipitate digitalis induced arrhythmias


– Phenytoin – Diuretics (thiazides)
– Rifampin – Amphotericin B
– Corticosteroids

Drugs ↑ serum digoxin concentration Destroy intestinal flora ↓ Clearance & volume of distri.
– Verapamil – Itraconazole – Erythromycin – Quinidine
– Diltiazem – Propafenone – Tetracycline
– Cyclosporine – Spironolactone
– Amiodarone – Flecainide

263
4 Self Assessment & Review Phar macology
Pharmacology
6. Ans. is b i.e. Digoxin Ref. Harrison 17/e, p 1433

If in the presence of atrial fibrillation, the ventricular rhythm becomes regular and slow (e.g. 30-60 beats/min);
complete heart block is suggested and if the ventricular rhythm is regular and rapid (e.g. ≥ 100 beats/min), a
tachycardia arising in the AV junction or ventricle should be suspected. Digitalis intoxication is a common
cause of both the phenomenon.
Above patient has following complaints Rheumatic mitral stenosis and Atrial fibrillation.
The patient in question has atrial fibrillation and while on treatment for fast ventricular rate develops a regular
and slow pulse of 64/min, thus suggesting a complete heart block secondary to digoxin administration.

7. Ans. is d i.e. Systemic Lupus erythematous Ref. KDT 6/e, p 515 - 516

SLE is not an adverse effect of amiodarone.


Adverse effects of amiodarone : To remember adverse effects, learn the following Mnemonic:

Mnemonic : Before Using Amiodarone Plz Check PFT, LFT, TFT


Plz Photosensitivity and Pigmentation of skin
Check Corneal deposition
PFT Pulmonary fibrosis (Option ‘a’) and Peripheral neuropathy
LFT Liver damage, Pseudoalcoholic liver injury and Mallory hyaline bodies
TFT Thyroid dysfunction, hypothyroidism (Option ‘b’), rarely hyperthyroidism (Option ‘c’) and inhibit
peripheral conversion of T4 to T3

Dose related adverse effects:


• Fall in B.P • Myocardial depression occurs on IV Injection
• Torsades de pointes • Nausea, GI upset • Increase LDL

Amiodarone :
• It is class III antiarrhythmic drug.
• Amiodarane has antianginal as well as antiarrhythmic action.
• Mechanism of Action : K channel blocker prolongs phase III of repolarization
+

• It has complex effects showing class :


I – Blocks inactivated Na+ channels
II – Non competitive β adrenergic blocking property
III – K+ channel blocker prolongs phase III of repolarization
IV – Inhibit myocardial Ca++ blocker

Pharmacokinetics : Incompletely absorbed after oral administration, full effects may not be achieved until 6
weeks after initiation of treatment.

Uses: • Severe refractory supraventricular and ventricular tachyarrhythmias


• Tachycardia associated with the WPW syndrome

264
Phar macology of Car
Pharmacology diovascular & Renal System
Cardiovascular 4
8. Ans. is b i.e. Endothelin receptor antagonist Ref. Harrison 17/e, p 1578; Katzung 10/e, p 287 - 288

• BOSENTAN is a nonselective endothelin receptor (i.e. both ETA and ETB receptor) antagonist.
• Endothelins cause →Transient vasodilatation →Then prolonged vasoconstriction (as in Fig.). So, Bostentan
finally will block vasoconstriction.
• Bosentan ‘a non peptide molecule’ acts on both ETA and ETB receptors:
– It lowers pulmonary artery pressure.
– It lowers vascular resistance in patients with CHF and increase cardiac index.
• It is approved for treatment of Primary Pulmonary Hypertension for patients of NYHA Class III and IV.
Side effects : Abnormal hepatic function tests (Hepatotoxic) →Increase transaminases
Contraindication : Concurrent use with cyclosporine or glyburide
Mechanism of action of Bosentan:

9. Ans. is d i.e. Nifedipine Ref. CMDT’ 05, p 426; Katzung 10/e, p 179; KDT 6/e, p 553-554

Nifedipine is not used for hypertensive emergencies because of its inability to control rate and degree
of fall in BP as well as mortality.
IV drugs with controllable actions are now used. Mean BP should not be lowered by more than 25%
over minutes or a few hours and then gradually to not lower than 160/100 mm Hg.

265
4 Self Assessment & Review Phar macology
Pharmacology

Parenteral Agents use for hypertensive emergencies


• Nitroprusside sodium • Nitroglycerin IV • Esmolol • Labetalol
• Fenoldopam (Option ‘a’) • Nicardipine – IV • Urapidil (Option “b” ) • Diazoxide
• Enalaprilat (Option ‘c’) • Hydralazine • Trimethaphan • Furosemide

Nitroprusside sodium:
• DOC in most hypertensive emergencies.
• Combination with β blocker is especially useful in patient with aortic dissection.
• Esmolol is often used to manage intraoperative and postoperative hypertension.
• Diuretics such as furosemide are administered to prevent the volume expansion that typically occurs during
administration of powerful vasodilators.

10. Ans. is c i.e. Protamine Ref. KDT 6/e, p 600

• Bleeding due to overdose is the most serious complication of heparin therapy. Haematuria is generally the
first sign.
• Heparin’s anticoagulant effect can be rapidly reversed by administration of protamine sulphate, hence used
as specific antidote in heparin overdose.

Remember:

Specific antidote Bleeding

Protamine sulfate Heparin therapy


Vitamin K1 (Phytonadione) Anticoagulants (Warfarin)

Antidote Type of Poisoning


• Protamine sulphate Heparins
• Vitamin K Warfarin and Coumarin anticoagulants
• N-acetylcystiene Acetaminophen (paracetamol poisoning)
• Atropine AChE inhibitors – Physostigmine, neostigmine, organophosphates,
including, insecticides such as malathion and parathion
• Desferrioxamine Iron and Iron salts
• Digoxin Immune Fab Digoxin and digitoxin
• Esmolol Theophylline, beta agonist glucagon
• Ethanol Methanol or ethylene glycol; Fomepizole (ADH inhibitor) was recently
approved for ethylene glycol poisoning
• Flumazenil Benzodiazepines, Zolpidem, Zopidone
• Naloxone Opioid analgesics

11. Ans. is b i.e. Increased guanylate cyclase


Ref. Goodman & Gilman 11/e, p 863 - 864; KDT 6/e, p 548-549

Endothelial cells, RBC’s split nitroprusside to generate NO which activates cytosolic guanylyl cyclase →
increase cGMP →causes dephosphorylation of myosin light chain kinase (MLCK) through a cGMP dependent
protein kinase →vascular smooth muscle relaxation.

266
Phar macology of Car
Pharmacology diovascular & Renal System
Cardiovascular 4

CAM Calmodulin
MLCK Myosin Light chain kinase
MLCK - P Phosphorylated
GTP Guanosine Triphosphate
C - GMP Cyclic guanosine monophosphate
NO Nitric oxide

Use of sodium nitroprusside:


• In management of hypertensive emergencies
• To lower BP in acute aortic dissection (always given with a β blocker)
• To increase cardiac output in CHF
• To decrease myocardial O2 demand after acute MI
• To induce controlled hypotension during anesthesia to decrease bleeding in surgical procedure

12. Ans. is a i.e. Conversion of hydroxy apatite to fluroapetite by replacing the OH ions
Ref. Essentials of preventive and Community Dentisry Dr. Soben Peter 1/e, p 220

• When hydroxypatite is exposed to low phosphate concentration (about 1ppm), a layer of fluoropatite forms
on the hydroxyapatite crystals. The thin layer governs the rate of enamel dissolution and reduces rate of
caries progression.
• During caries attack, the enamel is exposed to acidic pH of about 5.5 or lower, thereby leading to its
dissolution.
• The presence of fluoride reduces the solubility of enamel by promoting the precipitation of hydroxyapatite
and phosphate mineral.
Ca10 (PO) 6 (OH)2 + 2F → Ca10 (PO) 6F2 + 20H (OH ions are replaced)
(hydroxy-apatite) (fluro-apatite) Increase pH

Agents proved to be most active in inhibiting dental caries are :


• Neutral sodium fluoride (NaF) • Acidulated phosphate fluoride (APF) • Stannuous fluoride (SnF )2

13. Ans. is d i.e. Hypertrophic obstructive cardiomyopathy


Ref. Harrison 16/e, p 1411; KDT 6/e, p 499

Digitalis, diuretics, nitrates, vasodilators and β adrenergic agonists are best avoided if possible, particularly in
a patient with known left ventricular outflow tract pressure gradients (HOCM).
Regarding Digoxin : It is a cardiac glycoside, exerts positive inotropic action on heart.

267
4 Self Assessment & Review Phar macology
Pharmacology

Their initial action is to inhibit


cardiac membrane. Na+ / K± ATPase

↓ Na+ / Ca+ exchange

↑ Ca++ in SR

↑ Ca++ release and binding to troponin

Tropomyosin moves

↑ Actin and Myosin interaction

↑ Contractile force

Binding of digitalis to the “Pump” is inhibited by K+, so hyperkalemia decreases the effects and hypokalemia
may increase the effects and cause toxicity of digitalis.
Toxicity is increased by : ↓K+, ↓Mg++, ↑ Ca++

Electrophysiological effects : The effects on heart via parasympathetic autonomic nervous system (PANS)
include : • Central vagal stimulation
• Facilitation of muscarinic activity
• Sensitization of baroreceptors

Differences between Digitoxin and Digoxin


Parameters Digitoxin Digoxin
• Oral absorption 95% – 100% 75% – 90%
• Administration Oral Oral
• aVd (L/kg) 0.6 6–7
• Protein binding 90 30
• Plasma half life 6 – 7 days 38 – 40 hrs
• Onset of action 2 hours 1/2 hour
• Duration of action Very long Intermediate
• Metabolised (%) 80 (liver) 20 (liver)
• Excretion Mainly bile, also urine Urine (unchanged)
• Doses
– Digitalising dose 1.0 mg in 24 hr or 0.4 mg 0.5 – 0.75 mg 8 hrly. 3 doses
every 12 hr. 3 doses orally doses orally
– Maintenance dose 0.1 mg single dose per day 0.25 – 0.5 mg per day

Mode of action Atria AV Node Ventricles and purkinje

• Direct inhibition of ↓ APD and ERP ↓ Conduction velocity ↓ APD and ERP less significant
Na K -ATPase,
+ +
↑ abnormal autom- reinforced by cholinomi- than atrial actions
↑ Intracellular Na +
aticity (possible metic action
arrhythmias)

268
Phar macology of Car
Pharmacology diovascular & Renal System
Cardiovascular 4
• Indirect cholinomi- ↓ SA nodal rate ↑ ERP, ↓ Conduction No significant action
metic actions, vagal velocity (possible AV
stimulation, barore- block)
ceptor sensitization
• Sympathomimetic β 1
↑ SA rate ↓ RP (Possible AV nodal ↑ Conduction velocity, ↑ phase 4
like positive chrono- ↑ Conduction tachycardia) slope, ↑ abnormal automaticity
tropy, dromotropy velocity
and inotropy

Other Uses : The vagomimetic properties of digoxin may be used prophylactically in supraventricular tachycardia
(SVT) including atrial fibrillation, and for slowing the increase in AV conduction cause by quinidine.

Some points about disitalis toxicity you must know:

Early signs : Anorexia, Nausea, ECG changes, ↓QT interval, T wave inversion, PVBs
Late signs : CNS effects, disorientation, visual effects (halos) and hallucinations
More severe : SVTs, AV nodal tachycardias or AV block and ventricular tachycardias or VF
cardiotoxicity

Management of digitalis toxicity:


• Adjustment of electrolytes • Use of antiarrhytmics (Lidocaine, Phenytoin)
• Use of digitalis Fab antibodies and pacemakers. • Cardioversion is usually reserved for VF
• Avoid digitalis in Wolff-Parkinson-White arrhythmias

14. Ans. is d i.e. Causes hyperuricemia Ref. KDT 6/e, p 488

• Losartan is a mild uricosuric i.e. promotes excretion of uric acid rather than causing hyperuricemia.
• Losartan is a competitive antagonist of angiotensin II at AT1 receptors.

Angiotensin converting Angiotensin Antagonists


enzyme inhibitors (ACEI) (Losartan)
ACE
Mechanism of Action Angiotensin - I ⎯→ Angiotensin-II Competitive antagonist of angiotensin-
Side Effects • Cough : Brassy cough caused by II, more selective for AT1 than
inhibition of bradykinin/subs-P AT2 receptor
breakdown in the lungs • Does not potentiate Bradykinin
• Hypotension Ist dose • Ist dose hypotension is uncommon
• Rashes, Urticaria • Does not provoke cough
• Angioedema • Angioedema is rare
• Dysguesia • It is mild uricosuric
• Fetopathic • It is carboxylated to an active
metabolite in liver which is more
• Granulocytopenia and proteinuria potent and has a longer half life
• Precipitates acute renal failure in • Fetopathic
bilateral renal artery stenosis

269
4 Self Assessment & Review Phar macology
Pharmacology
15. Ans. is c i.e. High output failure Ref. KDT 6/e, p 499

Cardiac glycosides primarily mitigate systolic function. Digitalis causes a dose dependent increase in force of
contraction of heart i.e. a positive iontropic action.
Best result are obtained when Poor response and more toxicity is likely when
• Myocardium is not primarily damaged • Myocardium has been damaged : e.g. by ischaemia,
inflammation (Myocardial) or thiamine deficiency
• Failure is of low output type • High output failure (e.g. anemia)

Digitalis is used in AFL, AFI and PSVT


Arital fibrillation • Digitalis is DOC for controlling venticular rate in AF whether associated with CHF or
not, though it is incapable of curing AF.
• Digitalis reduces ventricular rate in AF by decreasing the number of impuses that are
able to pass down the A-V node and bundle of His.
• When digitalis is given in AF, average ventricular rate decreases in a dose dependent
manner and pulse deficit is abolished because ventricle does not receive an impulse
very early in diastole before it has had time to fill up reasonably.
Arital flutter • Digitalis enhances the degree of AV block, reduces ventricular rate and prevents sudden
shift of block to lower degree.
• May convert AFI to AF, which is a welcome response as control of ventricular rate is
easier in AF than in AFI.
PSVT Digitalis may be used, however it is now reserved for preventing recurrence.

Adenosine (DOC) and Verapamil are effective, less toxic and acts faster than digitalis.

16. Ans. is c i.e. Switch ethambutol for rifampin Ref. KDT 5/e, p 607

The fact that PT remains unchanged indicates ineffectiveness of warfarin.


This patient was on rifampicin when warfarin is started. As can be expected, rifampicin will induce the metabolism
of warfarin which increases its elimination and render it ineffective. The possible alternatives could be either we
increase the dose of warfarin (not given in options) or replase rifampicin with ethambutol. So, here the answer is
replace the rifampicin with ethambutol.
Contraindications of heparin are :
• Bleeding disorders, thrombocytopenia • Severe hypertension (risk of cerebral haemorrhage)
• Subacute bacterial endocarditis (embolism) • Tuberculosis (hemoptysis)
• Ocular and neurosurgery, lumbar puncture • Chronic alcoholics, cirrhosis, renal failure
• Aspirin and other antiplatelet drugs should be used very cautiously during heparin therapy.

17. Ans. is a i.e. Directly interacts with platelet membrane, Gp IIb / IIIa receptor
Ref. KDT 5/e, p 572

Already explained, kindly see answer no. 2

270
Phar macology of Car
Pharmacology diovascular & Renal System
Cardiovascular 4
18. Ans. is a i.e. Dipyridamole Ref. KDT 6/e, p 525, 609

Dipyridamole

• Dipyridamole is classified as an antianginal drug.


• The pharmacological success but therapeutic failure of dipyridamole has been explained on the basis of
“coronary steal phenomenon”; by dilating resistance vessels in nonischemic zone as well, it diverts the
already reduced blood flow away from ischemic zone.
• It is powerful coronary dilator.
• It acts by preventing uptake and degradation of adenosine which is a local mediator involved in autoregulation
of coronary flow in response to ischemia.
• It inhibits platelet aggregation by potentiating PGI and increasing cAMP in platelets.
2

• Though not useful as an antianginal drug, it is being employed for prophylaxis of coronary and cerebral
thrombosis in post MI and post stroke patients as well as to prevent thrombosis in patients with prosthetic
heart valves.

19. Ans. is a i.e. Isosorbide mononitrate Ref. KDT 6/e, p 28, 56

First pass metabolism or presystemic metabolism.


• Metabolism of a drug during its passage from the site of absorption into systemic circulation.
• All nitrates except isosorbide mononitrate undergo extensive but variable first pass metabolism in liver.
• First pass metabolism generally causes problems such as:
– A much larger dose of drug is needed when it is given orally than when it is given by other routes.
– Marked individual variation occurs in the extent of first pass metabolisms as given in following table.

Extent of first pass metabolism of important drugs:


Low Intermediate High
Not given orally Given orally
– Phenobarbitone – Aspirin – Isoprenaline – Propranolol
– Phenylbutazone – Quinidine – Lignocaine – Alprenolol
– Tolbutamide – Desipramine – Hydrocortisone – Verapamil
– Theophylline – Nortriptyline – Testosterone – Salbutamol
– Pindolol – Chlorpromazine – Nitroglycerine
– Pentazocine – Morphine
– Metoprolol – Pethidine
– Methyltestosterone
– Propoxyphene

20. Ans. is a i.e. Sodium Nitroprusside


Ref. Harrison 17/e, p 1562; 16/e, p 1478 - 1480; Nelson 17/e, p 1596

Drug of Choice are IV Labetalol or Nitroprusside and both are equally effective.
• Nitroprusside is given by continuous intravenous infusion at a dose of 0.25 to 8.0 μgm/kg per min. It is the
agent of choice, since it dilates both arterioles and veins.
• It has advantage over the ganglion blockers of not being associated with the development of tachyphylaxis
and can be used for days with few side effects.

271
4 Self Assessment & Review Phar macology
Pharmacology

Parenteral Agents use for hypertensive emergencies


• Nitroprusside sodium • Nitroglycerin IV • Esmolol • Labetalol
• Fenoldopam (Option ‘a’) • Nicardipine – IV • Urapidil (Option “b”) • Diazoxide
• Enalaprilat (Option ‘c’) • Hydralazine • Trimethaphan • Furosemide

21. Ans. is a i.e. Ethacrynic acid Ref. KDT 6/e, p 573

Urinary excretion of electrolytes and water with diuretics uses :


Diuretics Na +
K +
Cl -
HCO 3
-
HO 2

1. Frusemide ↑↑↑ ↑ ↑↑ ↑ ↑↑↑


2. Ethacrynic acid ↑↑↑ ↑ ↑↑↑ −
3. Thiazide ↑↑ ↑ ↑ ↑ ↑↑
4. Acetazolamide ↑ ↑↑ ↓↑ ↑
5. Spironolactone ↑ ↓ ↑ −↑ ↑
6. Triamterene ↑ ↓ ↑ −↑
7. Mannitol ↑↑ ↑ ↑ ↑ ↑↑↑↑

Maximum Na Excretion +
• Frusemide
• Ethacrynic acid
Maximum Cl Excretion by
-
• Ethacrynic acid
HCO is not Excreted by
3
• Ethacrynic acid
Maximum K Excretion by
+
• Acetazolamide

22. Ans. is a i.e. Enalapril Ref. KDT 6/e, p 484

Hyperkalaemia : Spironolactone should not be given along with potassium supplements or with ACE inhibitors
to avoid exacerbation of hyperkalemia.
Quinidine with amiloride may increase the risk of arrhythmias.
• Spironolactone is a K+ sparing diuretic.
• Hyperkalemia is the most serious side effect with spironolactone therapy.
• Hyperkalemia is also an important risk in patients on ACE inhibitors and if the two are given concomitantly,
significant rise of K+ to dangerous levels can occur.

23. Ans. is a i.e. Diabetic nephropathy with albuminuria Ref. KDT 6/e, p 487

Clinical uses of angiotensin converting enzyme inhibitors (ACEI)


– Diabetic nephropathy: Prevent or delay end stage renal disease in type I as well as type II diabetics.
Albuminuria remains stable in those treated with ACEI, but aggravates in untreated diabetics.

– Hypertension - advantages:
• Lack of postural hypotension, electrolyte disturbances, feeling of weakness and CNS effects.
• Safety in asthmatics, diabetics and peripheral vascular disease patients.

272
Phar macology of Car
Pharmacology diovascular & Renal System
Cardiovascular 4
• Prevention of secondary hyperaldosteronism and K+ loss due to diuretics.
• Renal blood flow is well maintained.
• Reverse LVH.
• No hyperuricemia and no deleterious effect on plasma lipid profile.
• No rebound hypertension on withdrawal.

– CHF : Congestive heart failure


– MI : Myocardial infarction
– Scleroderma Crisis : ACEI produces dramatic improvement and are life saving in this condition.

Adverse effects of ACEI : (Captopril, Enalapril, Lisinopril, Perindopril and Ramipril)


• Hypotension – An initial sharp fall in BP occurs specially in diuretic treated and CHF patients.
• Hyperkalemia – Is an important risk in patients with impaired renal function and in those taking K+ sparing
diuretics. NSAID or β blockers.
• Cough – Caused by inhibition of bradykinin/Substance P breakdown in the lungs of susceptible individuals.
• Rashes, urticaria
• Angioedema
• Dysguesia (Altered taste sensation)
• Fetopathic – Fetal growth retardation, hypoplasia of organs and fetal death may occurs if ACE inhibitors are
given during later half of pregnancy but not teratogenic in the first half; must be stopped when the woman
conceives.
• Headache dizziness, nausea and bowel upset.
• Granulocytopenia and proteinuria are rare.
• Acute renal failure – Precipitated by ACEI in patient with bilateral renal artery stenosis and single function
kidney due to dilatation of efferent arterioles and fall in GF pressure; contraindicated in such patients.

24. Ans. is a i.e. Na+ channel blocker Ref. KDT 6/e, p 511

Quinidine is Na+ channel blocker. It is a class 1A antiarrhythmic agent


The indirect actions of quinidine on the myocardium : Quinidine has antimuscarinic actions, and inhibits
vagal tone, thus the increase in sympathetic activity may precipitate an increase in conduction rate and ventricular
tachycardia.
Cardiac reflex actions induced by quinidine : This drug mediates alfa-adrenoceptor blockade, particularly
when administered rapidly. This results in vasodilatation and reflex tachycardia
Class Antiarrythmic drugs
I. Na channel blockers (membrane stablizing agents)
+

A. Quinidine, Procainamide Disopyramide, Moricizine


B. Lignocaine, Mexiletine, Phenytoin, Tocainide
C. Propafenone, Flecainide
II. β blockers (Antiadrenergic agents)
Propranolol, Esmolol, Sotalol (also class III)
III. Agents widening AP (Prolong repolarization and ERP)
Amiodarone, Dofetilide, Bretylium (also class II)
IV. Calcium channel blockers
Verapamil, Diltiazem

273
4 Self Assessment & Review Phar macology
Pharmacology
25. Ans. is c i.e. It decreases A.V. conduction Ref. KDT 6/e, p 523 - 524

Option ‘a’ • Nitrates are rapidly denitrated enzymatically in the smooth muscle cell to release the reactive
free radical nitric oxide (NO).
Option ‘b’ • The major action of nitrate is direct nonspecific smooth muscle relaxation.
• Nitrates dilate veins more than arteries.
Option ‘d’ • All nitrates except Isosorbide mononitrate undergo extensive but variable first pass metabolism
in liver.

26. Ans. is a i.e. Adenosine uptake inhibition Ref. KDT 6/e, p 535

Already explained, refer answer no. 18

27. Ans. is a i.e. Amiodarone Ref. KDT 6/e, p 515 – 516

Amiodarone is antiarrhythmic agent that prolongs repolarization and ERP, belongs to class III.
Effect of class III antiarrhythmics on the cardiac action potential : Phase 2 of the cardiac action potential
is prolonged, due to the blockade of potassium flux decreasing the excitability of tissue and thus inhibiting the
spontaneous formation of ectopic foci
This class include : – Amiodarone
– Bertylium (this also has class II properties)

28. Ans. is d i.e. Disopyramide increases its therapeutic effect


Ref. KDT 6/e, p 512, 513

• It selectively depresses AV node by acting on specific cell surface adenosine receptor.


• It terminates reentrant circuit through AV node and restores normal sinus rhythm in patients with PSVTs.

Adverse effects : The main adverse effects are facial flushing, shortness of breath, bronchospasm, nausea and
metallic taste but these side effects are normally short live.

Drug interactions : It include theophylline or caffeine which reduce its effectiveness, being adenosine receptor
antagonist. On the other hand, its effects are potentiated by adenosine uptake inhibitors such as dipyridamole.

Dipyridamole (and not disopyramide) increases the therapeutic effect. It does so by inhibiting the
uptake and degradation of adenosine.
Important points regarding adenosine

• Adenosine is the D.O.C. for PSVT.


• Adenosine is administered by rapid iv injection (over 1 - 3 seconds).
• Adenosine has a rapid action. It terminates more than 90% episodes of PSVT within 30 sec.
• Adenosine has a very short t½ in blood (10 sec) due to uptake into RBC’s and endothelial cells.
• Almost complete elimination occurs in single passage through coronary circulation.
• Action lasts less than 1 minute, therefore adverse effects (even cardiac arrest, if it occurs) are transient.
• No hemodynamic deterioration; can be given in hypotension, CHF or those receiving β blockers.

274
Phar macology of Car
Pharmacology diovascular & Renal System
Cardiovascular 4
29. Ans. is d i.e. High biological interaction to plasma proteins
Ref. KDT 6/e, p 599 - 600

Option ‘a’ : LMW heparin have a different anticoagulant profile :


– They selectively inhibit factor Xa with little effect on antithrombin and coagulation in general.
– They act only by inducing conformational change in AT III.
– They appear to have lesser antiplatelet action.
As a result, they have a small effect on aPTT and whole blood clotting time.

• Since a PTT/clotting times are not prolonged much, laboratory monitoring is not needed, and the incidence
of haemorrhagic complication is less.

Option ‘b’ : These have a longer and more consistent monoexponential t½. Once daily subcutaneous
administration is sufficient and advised.

Option ‘c’ : They are easily filtered from glomerular capillaries because of their smaller molecular weight.

30. Ans. is c i.e. Increase total coronary flow Ref. KDT 6/e, p 523

Pharmacological action of nitrates

Preload reduction : • Nitrates dilate veins more than arteries →peripheral pooling of blood →
decreased venous return i.e preload on heart is reduced →end diastolic
size and pressure are reduced → decreased cardiac work (Laplase
law) → hence decreased oxygen consumption.
After load reduction : • Nitrates also produce some arteriolar dilatation → slightly decrease
total peripheral resistance or afterload on heart → BP falls.
• This action contributes to the reduction of cardiac work – hence
decreased oxygen consumption.
Favourable redistribution : • Nitrates cause favourable redistribution of blood flow to ischaemic
of coronary flow areas in angina patients without increasing total coronary flow.

31. Ans. is a i.e. Verapamil Ref. KDT 6/e, p 518

Drugs for PSVT

• First choice – Adenosine


• Second choice – Verapamil
• Others – β blocker; Digoxin

Advantages of adenosine for termination of PSVT are :


• Efficacy equivalent to or better than verapamil.
• Action lasts < 1min; adverse effects (even cardiac arrest, if it occurs) are transient.
• No haemodynamic deterioration; can be given to patients with hypotension, CHF or those receiving β blockers.

275
4 Self Assessment & Review Phar macology
Pharmacology
32. Ans. is a i.e. Sodium Nitroprusside Ref. KDT 4/e, p 504

Arteriolar dilators Venodilators Mixed dilators


(Primarily ↓ after load) (primarily ↓ preload) ↓pre and after load)
(↓
• Hydralazine • Nitrates : • ACE inhibitors
• Minoxidil – Glyceryl trinitrate • Losartan (AT1 antagonist)
• Ca2+ channel blockers – Isosorbide dinitrate • Prazosin (α1 blocker)
(Nifedipine) – Isosorbide mononitrate • Phentolamine
• Pot. channel openers – Erythrityl tetranitrate • Nitroprusside
(Nicorandil) – Penta erythrital
• Diazoxide tetranitrate

33. Ans. is d i.e. Indapamides Ref. KDT 6/e, p 507 - 542

Indapamide is a weak thiazide diuretic and it reduces BP at much lower dosage. It has no central
action.
Propranolol and reserpine may exert some sympatholytic effect at the CNS level.

Antiadrenergic agents with central action i.e. central sympatholytics are :

• Clonidine • Methyldopa • Guanabenz • Guanfacine

34. Ans. is d i.e. Pirenzepine Ref. KDT 6/e, p 111, 633, 628

Pirenzepine : • Anticholinergic drug which acts by selectively blocking M1 muscarinic receptors (i.e.
It is not a calcium channel blocker)
• It inhibits gastric secretions without producing the typical atropinic side effects.

Calcium channel blockers :


Phenyl alkylamine (hydrophilic) – Verapamil
Benzothiazepine (hydrophilic) – Diltiazem
Dihydropyridine (lipophilic) – Nifedipine – Felodipine
– Amlodipine – Nitredipine
– Lacidipine – Nimodipine

Dihydropyridines (DHP) are the most potent Ca++ channel blocker.

Nimodipine : • Selectively relaxes central vasculature.


Flunarizine : • Is a relatively weak Ca++ channel blockers that also inhibit Na+ channels.
• It is a cerebro-selective Ca++ channel blocker and may benefit migraine by reducing
intracellular Ca++ overload due to brain hypoxia.

35. Ans. is d i.e. Collecting duct Ref. KDT 6/e, p 559

The characteristic feature of cells lining the collecting duct is their responsiveness to antidiuretic hormone
(ADH) i.e. ADH - act on collecting duct.

276
Phar macology of Car
Pharmacology diovascular & Renal System
Cardiovascular 4

36. Ans. is c i.e. Hydralazine Ref. KDT 6/e, p 523

Hydralazine is a directly acting arteriolar dilator and thus acts by reducing afterload.

Vasodilator drugs
↓ ↓
Arteriolar dilator Venodilator
Dilate arterioles Dilate veins
↓ ↓
↓ Total peripheral resistance Peripheral pooling of blood
↓ ↓
↓ Afterload ↓ Venous return

↓ Preload

Arteriolar dilators (Primarily ↓ afterload)


• Hydralazine • Minoxidil
• Ca2+ channel blockers (Nifedipine) • K+ channel openers (Nicorandil)
• Diazoxide

37. Ans. is d i.e. Inhibitors of Plasminogen activator Ref. KDT 6/e, p 537

• An acute thrombus at the site of atherosclerotic obstruction is the usual cause of MI.

277
4 Self Assessment & Review Phar macology
Pharmacology
• Plasminogen activators convert plasminogen to plasmin which converts insoluble fibrin into soluble fibrin
fragments.
• Plasminogen activation inhibitors inhibit dissolution of clot which leads to worsening of MI.

38. Ans. is d i.e. Amiodarone Ref. KDT 6/e, p 515 - 516, 519

• Amiodarone is an antiarrythmic drug belonging to class III.


• Amiodarone is not an inotropic agent, but a ‘long acting anti-arrhythmic’ belonging to class III antiarrhythmic
drugs.
• It causes myocardial depression and fall in BP on i.v. injection.
• Amrinone is a selective phosphodiesterase III inhibitor. It is termed as “an inodilator”, the major effects being
positive inotropic action and vasodilation
• Dopamine and Isoprenaline both cause an inotropic action and may be used in cardiogenic shock.
Ionotrotic drugs
Sympathomimetic inotropic drugs Phosphodiesterase III inhibitors Cardiac glycosides
• Dopamine option ‘a’’ • Amrinone (inamirone) – option ‘c’ • Digoxin
• Dobutamine • Milrinone • Digitoxin
• Isoprenaline option ‘b’

39. Ans. is d i.e. Furesamide

In the physiology of the kidney, free water clearance (CH2O) is the volume of blood plasma that is cleared of
solute-free water per unit time. An example of its use is in the determination of an individual’s state of hydration
and diuretics decreases free water clearance.
Free water clearance can be used as an indicator of how the body is regulating water. A free water clearance of
zero means the kidney is producing urine isosmotic with respect to the plasma. Values greater than zero imply
that the kidney is producing dilute urine through the excretion of solute-free water. Values less than zero imply
that the kidney is conserving water (likely under the influence of antidiuretic hormone, ADH), resulting in the
production of concentrated urine.
So diuretic like Furesamide causes water loss from the body in the form of, free water clearance and
decreases it.

40. Ans. is c i.e. Used in pulmonary oedema Ref. KDT 6/e, p 562 - 563; Goodman & Gilman 11/e, p 749

Option “a” The major site of action of furesemide is the thick ascending limb of the loop of Henle where
it inhibits Na+ - K+ - 2CI– cotransport.
Option “b” It can be given by oral, intramuscular, and intravenous routes.
Option “c” A major use of loop diuretics is in the treatment of acute pulmonary edema. A rapid increase
in venous capacitance in conjunction with a brisk natriuresis reduces left ventricular filling
pressures and thereby rapidly relieves pulmonary edema.
Option “d” It is a highly efficacious diuretic.
The efficacy of inhibitors of Na+ - K+ - 2CI– symport in the thick ascending limb of the loop of
Henle is due to a combination of two factors :

278
Phar macology of Car
Pharmacology diovascular & Renal System
Cardiovascular 4
• Approximately 25% of the filtered Na+ load normally is reabsorbed by the thick ascending
limb.
• Nephron segments past the thick ascending limb do not possess the reabsorptive capacity
to rescure the flood of rejectate exiting the thick ascending limb.

41. Ans. is b i.e. Increased calcium excretion Ref. Goodman & Gilman 11/e, p 755

• The thiazide diuretics decrease Ca2+ excretion when administered chronically. The mechanism involves
increased proximal reabsorption owing to volume depletion, as well as direct effects of thiazides to increase
Ca2+ reabsorption in the DCT.
• The inhibition of the Na+ - CI– symporter in the luminal membrane decreases intracellular Na+ levels, thereby
increasing the basolateral exit of Ca2+ via enhanced Na+ - Ca2+ exhange.

42. Ans. is b i.e. Increase cGMP Ref. KDT 6/e, p 523 - 524

NO activates cytosolic guanylyl cyclase → increase cGMP → causes dephosphorylation of myosin light
chain kinase (MLCK) through a cGMP dependent protein kinase →vascular smooth muscle relaxation.

Cal. Channel
Blockers Ca2+

Stored
Ca2+ MLCK cGMP
Ca2+
MLCK - P GTP
Ca + CAM NO
Myosin Myosin - P
+
Actin

CONTRACTION

Nitrodilators

43. Ans. is d i.e. Minoxidil Ref. Goodman & Gilman 11/e, p 862

• Minoxidil sulfate activates the ATP-modulated K+ channel. By opening K+ channels in smooth muscle and
thereby permitting K+ efflux, it causes hyperpolarization and relaxation of smooth muscle. Other K+ channel
openers are diazoxide, nicorandil and pinacidil.
• Hydralazine, Sildenafil, and Glyceryl nitrate involves generation of NO and activation of cGMP to cause
smooth muslce relaxation.

44. Ans. is d i.e. L-dopa Ref. Goodman & Gillman 11/e, p 534 - 535, 685, 1411

Levodopa may form dopamine in the periphery. It can act on receptors in the blood vessels and causes orthostatic
hypotension and not hypertension.

279
4 Self Assessment & Review Phar macology
Pharmacology

Drugs causing Hypertension


• Cocaine • MAO inhibitors
• Cyclosporine • Glucocorticoids
• Oral contraceptives • TCAs
• Rofecoxib • Valdecoxib
• Clonidine withdrawal • Sympathomimetics

Drugs causing Hypotension


• Theophylline • Adenosine
• Morphine • Quinidine
• Fosphenytoin (I.V.) • Amiodarone
• IL - 2 • Levodopa
• Alpha blockers • Guanethidine
• Bretylium • β-blocker (I.V.)
• Glyceryl trinitrate • Chloropromazine
• Diuretics • Clonidine
• Calcium channel blockers

45. Ans. is d i.e. INH Ref. Harrison 17/e, p 1807

Drugs causing interstitial nephritis


• Cephalosporin • Penicillins especially - Methicillin • Phenytoin
• Allopurinol • Ciprofloxacin • Phenobarbital
• Phenindione • Sulphonamides • Valproicacid
• Thiazides • NSAIDs • COX2 inhibitors
• Rifampicin • Vancomycin

46. Ans. is c i.e. Liver failure Ref. KDT 6/e, p 498 - 499

Digoxin : is primarily excreted by the kidney so dose need not be altered in hepatic disease.
Digitoxin : is primarily metabolized in liver so its dose has to be altered in hepatic disease.

Other important precautions and contraindications:


• Hypokalemia : enhances digitalis toxicity by increasing its binding to Na+ K+ ATPase.
• Elderly, renal or severe hepatic disease : patients are more sensitive.
• Myocardial infarction : arrhythmogenic dose of digitalis may be reduced.
• Thyrotoxicosis : reduces responsiveness to digitalis.
• Myxoedema : these patients eliminate digoxin more slowly; cumulative toxicity can occur.
• Partial A-V block : may be converted to complete A-V block.
• Calcium : synergises with digitalis →precipitates toxicity.
• Quinidine : reduces binding of digoxin to tissue proteins as well as its renal and biliary clearance.
• Adrenergic drugs : can induce arrhythmias in digitalized patients; both increase ectopic automaticity.

280
Phar macology of Car
Pharmacology diovascular & Renal System
Cardiovascular 4
47. Ans. is d i.e. Pentoxyphylline Ref. Schiff’s Disease of Liver 10/e, Chapter 32

All above drugs are used in alcoholic hepatitis, but to prevent renal toxicity in a case of alcoholic hepatitis,
pentoxyphylline is given.
Option “a” Silymarin is a free radical scavenging, antioxidant and anti-inflammatory agent which stimulate
hepatocyte proliferation.
Option “b” S adenosyl methionine in alcoholic cirrhosis S - adenosyl methionine synthase activity is
reduced, so its supplementation is given.
Option “c” Thalidomide prevent liver damage by suppression of TNF-α production by degrading its
mRNA.
Option “d” Pentoxifylline is a phosphodiesterase inhibitor used in the treatment of peripheral vascular
disease on the basis of its ability to increase erythrocyte flexibility, reduce blood viscosity, and
inhibit platelet aggregation.

48. Ans. is c i.e. Fenoldopam Ref. Goodman & Gillman 11/e, p 863, 249, 825; KDT 5/e, p 512

Fenoldopam is a selective D1 agonist useful in hypertensive emergencies by dilating blood vessels of coronary,
renal, mesenteric vessels.
Drug which donates nitric oxide
• Sodium nitroprusside • Organic nitrates
• Nitrites • Hydralazine
• Propofol • Nebivolol

49. Ans. is c i.e. Tocainide Ref. KDT 6/e, p 511

Tocainide belongs to 1B class of antiarrhythmic drugs.


Class Antiarrythmic drugs
I. Na channel blockers (membrane stablizing agents)
+

A. Quinidine, Procainamide Disopyramide, Moricizine


B. Lignocaine, Mexiletine, Phenytoin, Tocainide
C. Propafenone, Flecainide
II. β blockers (Antiadernergic agents)
Propranolol, Esmolol, Sotalol (also class III)
III. Agents widening AP (Prolong repolarization and ERP)
Amiodarone, Dofetilide, Bretylium (also class II)
IV. Calcium channel blockers
Verapamil, Diltiazem

50. Ans. is a i.e. Hypotension Ref. KDT 6/e, p 484

Adverse effect of ACEI : (Captopril, Enalapril, Lisinopril, Perindopril and Ramipril)

• Hypotension – An initial sharp fall in BP occurs specially in diuretic treated and CHF patients.

281
4 Self Assessment & Review Phar macology
Pharmacology

• Hyperkalemia – Is an important risk in patients with impaired renal function and in those taking K sparing
+

diuretics. NSAID or β blockers.


• Cough – Caused by inhibition of bradykinin/Substance P breakdown in the lungs of susceptible individuals.
• Rashes, urticaria
• Angioedema
• Dysguesia (Altered taste sensation)
• Fetopathic – Fetal growth retardation, hypoplasia of organs and fetal death may occurs if ACE inhibitors are
given during later half of pregnancy but not teratogenic in the first half; must be stopped when the woman
conceives.
• Headache dizziness, nausea and bowel upset.
• Granulocytopenia and proteinuria are rare.
• Acute renal failure – Precipitated by ACEI in patient with bilateral renal artery stenosis and single function
kidney due to dilatation of efferent arterioles and fall in GF pressure; contraindicated in such patients.

51. Ans. is d i.e. For reducing pulmonary hypertension Ref. Harrison 17/e, p 1683

Pharmacological implications of Neurogenic Nitric oxide


• Respiratory Distress :
– Inhaled NO has been approved by the FDA only for use in newborns with persistent pulmonary hypertension
and has become the first line therapy for this disease.
– Potential benefits of inhaled NO in a variety of conditions, including weaning from cardiopulmonary bypass
in adult and congenital heart disease patients, primary pulmonary hypertension, pulmonary embolism,
acute chest syndrome in sickle-cell patients, congenital diaphragmatic hernia, high-altitude pulmonary
edema, and lung transplantation.
Other conditions where NO play a crucial role are
• Vascular effects • Atherosclerosis
• Platelet aggregation • Organ transplant
• Host defence • Nonadrenergic-Noncholinergic (NANC) Nervous System
• Central nervous system • Inflammation

52. Ans. is d i.e. Pheochromocytoma Ref. KDT 6/e, p 486 - 487

Clinical uses of angiotensin converting enzyme inhibitors (ACEI)


1. Hypertension (Option ‘a’)
• Are the first line drugs in all grades of hypertension.
• They offer the following advantages :
– Lack of postural hypotension, electrolyte disturbances, feeling of weakness and CNS effects.
– Safety in asthmatics, diabetics and peripheral vascular disease patients.
– Prevention of secondary hyperaldosteronism and K+ loss due to diuretics.
– Renal blood flow is well maintained.
– Reverse LVH (Option ‘c’)
– No hyperuricemia and no deleterious effect on plasma lipid profile.
– No rebound hypertension on withdrawal.

282
Phar macology of Car
Pharmacology diovascular & Renal System
Cardiovascular 4
2. CHF : Unless contraindicated, ACE inhibitors are now advocated as first line drugs in all patients with
symptomatic as well as asymptomatic left ventricular inadequacy; a diuretic with or without digitalis may be
added according to need.
3. MI (Option ‘b’)
• Oral ACE inhibitors administered while MI is evolving (within 24 hrs. of an attack) and continued for 6
weeks reduce early as well as long term mortality, irrespective of presence or absence of systolic
dysfunction, provided hypotension is avoided.
• If there are no contraindications, all MI patients stand to gain from ACE inhibitor therapy, though magnitude
of benefit is greatest in those having associated hypertension and/or diabetes.
4. Diabetic nephropathy
• Prevent or delay end stage renal disease in type I as well as type II diabetics.
• Albuminuria remains stable in those treated with ACEI, but aggravates in untreated diabetics.
• All patients with diabetic nephropathy, whether hypertensive or normotensive, deserve ACE inhibitor
therapy.
5. Scleroderma Crisis
• ACEI produces dramatic improvement and are life saving in this condition.
6. Prophylaxis
It is in high cardiovascular risk subject - Protective effect exerted both on myocardium and vasculature
independent of hypotensive action.

53. Ans. is a i.e. Enalapril Ref. KDT 6/e, p 553

Enalapril is an ACEI which is fetopathic


Antihypertensives
Avoided during pregnancy Safer during pregnancy
• Diuretics • Hydralazine
• ACE inhibitor (enalapril) • Dihydropyridine CCBs
• AT antagonist (losartan)
1
• Labetalol
• Reserpine • Prazosin
• Nonselective β blockers • Clonidine
• Sodium nitroprusside • Methyldopa

Important : Cardioselective β blocker and those with ISA activity are used when no other choice is present.

54. Ans. is b i.e. It causes dilatation of both arteries and veins Ref. KDT 6/e, p 503 - 504, 547 - 548

• Hydralazine is a directly acting arteriolar dilator. It is not a mixed dilator.



• It causes greater reduction of diastolic than systolic BP.

• Powerful reflex compensatory mechanisms are evoked, causes:
tachycardia, ↑ cardiac output, ↑ renin release

283
4 Self Assessment & Review Phar macology
Pharmacology
• Increased aldosterone secretion

• Salt and water retention

• Tolerance to the hyotensive action develops unless diuretics or β blocker or both are given together to block
the compensatory mechanisms.

Adverse effects of Hydralazine :


• Facial flushing, conjunctival injection, throbbing headache, dizziness, palpitation, nasal stuffiness fluid
retention, edema, CHF.
• Angina and MI may be precipitated in patients with coronary artery disease.
• Postural hypotension is not prominent because of little action on veins.
• Lupus erythematosus or rheumatoid arthritis like symptoms develop on prolonged use of doses above
100 mg / day. It is more common in women and in slow acetylators.

55. Ans. is c i.e. Fresh frozen plasma Ref. Goodman & Gilman 11/e, p 1529; KDT 6/e, p 601

If immediate hemostatic competence is necessary because of serious bleeding or profound warfarin overdosage,
adequate concentrations of vitamin K–dependent coagulation factors can be restored by transfusion of fresh-
frozen plasma (10 - 20 ml/kg), supplemented with 10 mg of vitamin K1 by slow intravenous infusion.

Blood Components

Component Volume Content Clinical Response/uses

1. Packed Red 180-200ml RBC+WBC+ Increases Hb 10gm/L and Hematocrit 3%


Blood cells Small Plasma
2. Platelets 5.5X 10 RD unit
19
Increase platelet count 5000-10000/uL
3. Fresh frozen 200-250ml Stable coagulation Increase coagulation factors about 2%
plasma factor and plasma • Initiation
Protein : • Rapid Reversal of warfarin induce
Fibrinogen antithrombin bleeding
albumin, Protein c/5 • Treatment of TTP
4. Cryoprec- 10 - 15ml • Cold insoluble plasma Topical fibrin glue
ipitates • protein fibrinogen • Increases factor VIII 80 IU.
• factor VIII
• VWF

Warfarin Heparin
Action Warfarin inhibit vitamin K Activate antithrombin III. The heparin-antithrombin
dependent factor II, VII, complex then inactivates factor Xa, IIa, IXa, XIIa,
IX, X, Protein C and S XIIIa but not factor VIIa.
Monitoring PT APTT
Antidote Ist is fresh frozen plasma Protamine sulphate
and 2nd is Phytonadione(K ) 1

284
Phar macology of Car
Pharmacology diovascular & Renal System
Cardiovascular 4
56. Ans. is d i.e. Aldosterone antagonists Ref. KDT 6/e, p 561

• Spironolactone acts from the interstitial side of the tubular cell, combines with the mineralocorticoid receptor
and inhibits the formation of AIPs in a competitive manner. It has no effect on Na+ and K+ transport in the
absence of aldosterone.
• All other diuretics act from the luminal side of the membrane.

Effects of aldosterone on late distal tubule and collecting duct


and diuretic mechanism of aldosterone antagonists.
1. Activation of membrane-bound Na+ channels; 2. Redistribution of Na+ channels from cytosol to membrane;
3. de novo synthesis of Na+ channels; 4. Activation of membrane-bound Na+, K+–ATPase; 5. Redistribution of
Na+, K+–ATPase from cytosol to membrane; 6. de novo synthesis of Na+, K+–ATPase; 7. Changes in permeability
of tight junctions; 8. Increased mitochondrial production of ATP.
Diuretics and their mechanism of action :
Inhibitors of Na –K –2Cl cotransport
+ + –
• Furosemide
• Ethacrynic acid
Inhibitors of Na –Cl symport
+ –
• Thiazides
Carbonic anhydrase inhibitors • Acetazolamide
Potassium sparing diuretics • Spironolactone
• Triamterene
• Amiloride

285
4 Self Assessment & Review Phar macology
Pharmacology
57. Ans. is a i.e. Ventricular arrhythmia Ref. KDT 6/e, p 692; Katzung 9/e, p 780

Drugs causing prolonged QT interval / Torsades de pointes :

• Amiodarone • Amitriptyline • Cisapride • Erythromycin


• Haloperidol • Imipramine • Terfenadine

Terfenadine, Cisapride, Astemizole (not carboxy metabolite) blocks cardiac K+ channel in over dose and
produces polymorphic ventricular tachycardia (Torsades de pointes).

58. Ans. is c i.e. Mineralocorticoid antagonist Ref. KDT 6/e, p 561

Spironalactone (mineralocorticoid antagonist) acts from the interstitial side of the tubular cell while
all other diuretics act from the luminal side of the cell membrane.
For more details, refer answer no. 56

59. Ans. is b i.e. Increase in LDL-cholesterol on prolonged use Ref. KDT 6/e, p 546

Clonidine :
• Clonidine is a selective α 2A adrenergic receptor agonist in the brainstem.
• It is a moderately potent antihypertensive.
• It does not alter plasma lipid levels, so there is no increase in LDL cholesterol on prolonged use.
• It decreases sympathetic outflow to kidney, results in ↓ renin release, so it may be combined with drugs
which increase plasma renin activity.
• So, clonidine can be combined with vasodilator.
• It exhibits therapeutic window phenomenon: Optimum fall in B.P. between 0.2 to 2.0 ng/ml blood levels. At
higher concentration, fall in BP is less marked.
Uses of Clonidine Adverse effects
• Moderate hypertension • Sedation
• Opioid, Nicotinic, Alcohol withdrawal • Mental depression
• Intrathecal/epidural surgical and post-op analgesia • Disturbed sleep

286
Phar macology of Car
Pharmacology diovascular & Renal System
Cardiovascular 4
• To control loose motions due to diabetic neuropathy • Dry mouth, nose, eyes
• Post menopausal vasomotor symptoms • Constipation
• Clonidine suppression test for pheochromocytoma • Impotence
• Prophylaxis in migraine • Salt/H2O retention
• Preanaesthetic medication (Sedative, anxiolytic & analgesic) • Bradycardias
• Rebound hypertension

60. Ans. is b i.e. Hydrochlorthiazide


Ref. Goodman & Gilman 11/e, p 509; Katzung 8/e, p 493; KDT 6/e, p 436

Hydrochlorthiazide, furosemide and spironolactone : All decrease renal lithium clearance but thiazide
causes greatest retention among diuretics, so the answer is thiazide.
• Li+ is eliminated in urine.
• Li+ is handled by the kidney in the same way as Na+. Most of the filtered Li+ is reabsorbed in the proximal
tubule.
• Diuretics (thiazide, furosemide) by causing Na+, loss promote proximal tubular reabsorption of Na+ as well
as Li+ →plasma levels of Li+ rise.
• Li+ retention can be increased by : • Li+ excretion can be increased by :
– Indomethacin – Tetracycline – Osmotic diuretics – Amionophylline
– ACE inhibitors – Thiazides – Acetazolamide – Triamterene

61. Ans. is a i.e. Phenoxybenzamine Ref. Harrison 17/e, p 2271; KDT 6/e, p 142 - 143

• Patient should be started with :


Phenoxybenzamine : It induce long lasting noncompetitive α receptor blockade. It is administered for at
least 10-14 days prior to surgery.
Before adequate α adrenergic blockade with phenoxybenzamine is achieved, paroxysm of hypertension
may be treated with oral prazosin or phentolamine.

• Other drugs which reduce the blood pressure in patient with pheochromocytoma.

– Na-nitroprusside – Ca++ channel blocker – ACE Inhibitors

β−
−Blocker should be given only after α - blockade has been induced. Since administration of β-
blocker alone may cause paradoxic rise in BP by antagonising β-mediated vasodilatation.
• The induction of stable α adrenergic blockage provides the foundation for successful surgical treatment
of pheochromocytoma.
• Phenoxybenzamine is the DOC for hypertension in pheochromocytoma, so it cannot cause hypertensive
crisis.
• Phenoxybenzamine indues a long-lasting non competitive α - receptor blockade.
• Propranolol (β-blocker) and saralasin (an angiotensin receptor antagonist) are known to precipitate hypertensive
crisis in pheochromocytoma.

287
4 Self Assessment & Review Phar macology
Pharmacology
62. Ans. is b i.e. Enalapril Ref. Katzung 10/e, p 176-177

• Here, the patient is having hypertension and is suffering from impaired renal function (Serum creatinine >
3.5mg / dl) due to benign prostatic hyperplasia.
• ACE inhibitors are contraindicated as this patient, having serum creatinine > 3.5 mg/dL (which indicate renal
dysfunction) might land up in renal failure.

63. Ans. is b i.e. Quinidine Ref. KDT 6/e, p 499

Interaction of quinidine and drugs such as warfarin, digoxin, and carbamazepine : Quinidine is highly
plasma protein-bound (> 85 percent) and thus competes with other drugs [that are highly protein-bound for
storage sites on plasma proteins. Thus, concurrent administration of quinidine with highly plasma protein-
bound drugs such as benzodiazepines, carbamazepine, warfarin and digoxin increases the plasma concentration
of these drugs, leading to toxicity
Quinidine and Procainamide are contraindicated in digitalis induced ventricular arrhythmias.

For digitalis induced ventricular arrhythmias :


• Lignocaine I/V repeated as required, is the drug of choice.
• Phenytoin is also useful but not preferred now, because sudden deaths have occurred when it was injected
I/V in digitalis intoxicated patients.
Digitalis toxicity DOC
• For Tachyarrhythmias KCL
• For Ventricular arrhythmias Lignocaine
• For Supraventricular arrhythmias Propranolol
• For AV block and bradycardia Atropine

64. Ans. is d i.e. Hyperparathyroidism Ref. Goodman & Gilman 11/e, p 775; Harrison 16/e, p 2259, 17/e, p-2380

• Thiazides are associated with aggravation of hypercalcemia in primary hyperparathyroidism. Some of the
renal effect is due to augmentation of PTH action and is more pronounced in individuals with intact PTH
secretion. ...Harrison
++
• When administered chronically, thiazide diuretics decreases Ca excretion. The mechanism is unknown
but may involve increased proximal reabsorption due to volume depletion as well as direct effects of thiazide
to increase Ca++ reabsorption in the DCT. ... Goodman & Gilman

65. Ans. is c i.e. β- blocker Ref. CMDT’ 05, p 349; Harrison 16/e, p 1456

CMDT writes : • I/V β- blockers (metoprolol, 5mg every 5 min for three doses) are given immediately
after acute myocardial infarction.
• It reduces the incidence of ventricular fibrillation.
Harrison writes : • Prophylactic antiarrhythmic therapy (either I/V lidocaine early or oral agents later) is
contraindication for ventricular premature beats in the absence of clinically important
ventricular tachyarrhythmias, as such therapy may actually increase the mortality rate.
• β - blocking agents are effective in abolishing ventricular ectopic activity in patient with
MI and in the prevention of ventricular fibrillation.

288
Phar macology of Car
Pharmacology diovascular & Renal System
Cardiovascular 4
66. Ans is a i.e. Infection with streptococcus causes beneficial dual effect Ref. KDT 6/e, p 606

Regarding Streptokinase :
• It is a fibrinolytic drug obtained from β haemolytic streptococci group C.
• Antistreptococcal antibodies present due to past infection inactivate considerable fraction of the initial dose
of streptokinase, so previous infection with streptococcus has harmful effect on streptokinase.
• It is a plasminogen activator, so used as thrombolytic.
• Haemorrhage is the main complication.
• Monitoring should be done with thrombin time because thrombin time is used to assess fibrin status
and the fibrinolytics act by degrading fibrin.

Streptokinase

• Source Group C streptococcus


• T½ 20 minutes
• Clot selectivity Low
• Bleeding +
• Hypotension +++
• Allergic reaction ++

67. Ans. is a i.e. Verapamil belongs to second group drugs Ref. KDT 6/e, p 530

Calcium channel blocker is most effective in the therapy of paroxysmal atrial tachycardia : Verapamil,
because it slows conduction in the AV node, and prolongs the refractory period.
• Verapamil belongs to class IV antiarrythmic drug (So, option ‘a’ is absolutely wrong).
• It can accentuate conduction defects (contraindicated in 2nd and 3rd degree AV - block).
• Drug of choice in PSVT is adenosine but verapamil is used as an alternative.
• It should not be given with β blockers - additive sinus depression, conduction defects or asystole may occur.
Cardiac effects of Verapamil
Heart Rate ↓
A - V Conduction velocity ↓ ↓
Contractility ↓
Output ↓

289
4 Self Assessment & Review Phar macology
Pharmacology
68. Ans. is c i.e. Replace warfarin for acetacumarin Ref. KDT 6/e, p607; Harrison 15/e, p 58

This patient was on rifampicin when warfarin is started. As can be expected, rifampicin will induce the metabolism
of warfarin which increases its elimination and render it ineffective. The possible alternatives could be either we
increase the dose of warfarin (not given in options) or replace rifampicin with ethambutol. So, here the answer is
replace the rifampicin with ethambutol.
Contraindications of Heparin are :

• Bleeding disorders, thrombocytopenia • Severe hypertension, (risk of cerebral haemorrhage)


• Subacute bacterial endocarditis (embolism) • Tuberculosis (hemoptysis)
• Ocular and neurosurgery, lumbar puncture • Chronic alcoholics, cirrhosis, renal failure
• Aspirin and other antiplatelet drugs should
be used very cautiously during heparin therapy.

69. Ans. is b i.e. Causes hyperuricemia Ref. KDT 6/e, p 488

• Losartan is an angiotensin receptor antagonist.


• It is 10,000 times more selective for AT1 than AT2 receptor.
• Pharmacologically, AT1 antagonists differ from ACE inhibitors in following ways :
– They don’t interfere with degradation of bradykinin and other ACE substrates; no rise in level or potentiation
of bradykinin, so does not cause cough.
– They result in more complete inhibition of AT1 receptor activation, because alternative pathway of All
generation and consequent AT1 receptor activation remain intact with ACE inhibitors.
– They result in indirect AT2 receptor activation. Due to blockade of AT1 receptor mediated feed back
inhibition-more. All is produced which acts on AT2 receptors that remain unblocked.

• Pharmacokinetics : It is partially carboxylated in liver to an active metabolite (E 3174) which is 10-30 times
more potent noncompetitive AT1 antagonist.

70. Ans. is d i.e. Given for long duration Ref. KDT 6/e, p 507; Goodman & Gilman 11/e, p 890

• Milrinone and Amrinone are recently developed inotropic drugs used in cardiac failure.
• They act by inhibiting phosphodiesterase III.
Phosphodiesterase III Inhibitors
Amrinone (Inamrinone) Milrinone
• Inotropic and direct vasodilator • Similar action but 10 times more potent
• Thrombocytopenia is prominent • Not significant
• Less arrhythmogenic • More arrhythmogenic
• Long term action (t½ 2 - 5 hrs.) • Short term action (t½ 40 - 80 min)

Because of its greater selectivity for PDE III isoenzymes, shorter half life and fewer side effects, milrinone is the
agent of choice among currently available PDE inhibitors for short term parenteral inotropic support in severe
heart failure.

290
Phar macology of Car
Pharmacology diovascular & Renal System
Cardiovascular 4
71. Ans. is d i.e. Epsilon amino caproic acid Ref. KDT 6/e, p 608

Thrombolytic drug : A thrombolytic drug is one that actually disrupts an established clot. Unlike an anticoagulant,
these drugs are not prophylactic, but actually dissolve the fibrin web that forms the basis of the clot, causing
dissolution.
Haemostatic drug : These drugs decrease blood flow and thus blood loss.
A haemoperfusion agent : These drugs decrease blood viscosity and increase the flow of blood, resulting in
increased tissue perfusion.

Fibrinolytic drugs Antifibrinolytic drugs


• Streptokinase • Epsilon amino caproic acid (EACA)
• Urokinase • Tranexaemic acid
• t-PA (Alteplase) • Aprotinin

72. Ans. is b i.e. To overcome increased sympathetic activity of nifedipine Ref. KDT 6/e, p 531

• β-blocker is given along with nifedipine because nifedipine causes tachycardia, ↑ contractility and
cardiac output due to reflex sympathetic stimulation which is prevented by β-blocker.
• Note β-blocker is not given with verapamil or diltiazem (both are also CCBs) because both drugs are
cardiodepressant and β- blocker is also cardiodepressant which may lead to bradycardia, heart block and
cardiac arrest.

73. Ans. is d i.e. Captopril Ref. KDT 6/e, p 484

Drug causing altered taste sensation are : • Metronidazole • Rifampicin


• Astrazolamide • Biguanide
• Captopril • Penicillamine
• Grisofulvin • Lithium
Mnemonic : MR ABC PG Li

74. Ans. is a, b and d i.e. Decompensated heart failure; HOCM; and Myocarditis
Ref. KDT 6/e, p 499, 501

Digitalis have little or no value in patient with decompensated heart failure, sinus arrhythmias, HOCM,
myocarditis, mitral stenosis, chronic constrictive pericarditis and any form of diastolic heart failure.

Uses of digitalis : • Congestive heart failure • Cardiac arrhythmias


• Atrial fibrillation • Atrial flutter
• Paroxysmal supraventricular tachycardia

75. Ans. is is b i.e. Diminished T wave amplitude Ref. See below

Digitalis effects : The therapeutic serum levels of digitalis may produce subtle alteration of ECG called digitalis
effect ECG. Manifestations of digitalis effect are follows :
• Inverse check mark configuration of S.T. segment is the characteristic of digitalis effects.

291
4 Self Assessment & Review Phar macology
Pharmacology
ST-Segment : Digitalis in therapeutic dosage does not depress the proximal part of ST segment. Therefore, if
inverse check mark sign is associated with depression of ST-segment then either digitalis toxicity or associated
myocardial involvement due to ischemia or coronary insufficiency may be suspected.
• T wave : Therapeutic doses of digitalis diminish the amplitude of T wave without any changes its directions.
T wave inversion indicates digitalis toxicity rather than effect.
• Short QTc interval
• Prolong PR interval
• Slowing of heart rate with prolongation of PR interval
Digitalis induced arrhythmias and conduction disturbances
• Non-paroxysmal atrial tachycardia with second degree AV block is the most common arrhythmia. Atrial
ectopics and / atrial bigeminy are common.
• Accelerated junctional rhythm with AV dissociation.
• Junctional (idionodal) tachycardia with AV dissociation.

76. Ans. is c and d i.e. Amiodarone; and Bretylium Ref. Harrsion 17/e, p 1489

Drugs causing Pericarditis : • Procainamide • Anticoagulants


• Hydralazine • Methysergide
• Phenytoin • Minoxidil
• Isoniazid • Penicillins
• Minoxidil

77. Ans. is a, d and e i.e. Sulfonamide hypersensitivity; Contraindicated in metabolic acidosis; and COPD
Ref. KDT 6/e p 569; KDT 5/e, p 529, 533; Goodman & Gilman 11/e, p 746

Non-Diuretic uses of Carbonic Anhydrase inhibitors are :


• Glaucoma
• Metabolic alkalosis
• To treat urate calculi or cystinuria
• High altitude sickness
• Epilepsy
• They produce allergic reaction in patients who are sulfonamide hypersensitive.

78. Ans. is a and c i.e. Hypokalemia; and Hypercalcemia Ref. KDT 6/e, p 498 - 499

Important precautions and contraindications in which Digoxin toxicity is aggravated :


• Hypokalemia : enhances digitalis toxicity by increasing its binding to Na+ K+ ATPase.
• Elderly, renal or severe hepatic disease : patients are more sensitive.
• Myocardial infarction : arrhythmogenic dose of digitalis may be reduced.
• Thyrotoxicosis : reduces responsiveness to digitalis.
• Myxoedema : these patients eliminate digoxin more slowly; cumulative toxicity can occur.
• Partial A-V block : may be converted to complete A-V block.

292
Phar macology of Car
Pharmacology diovascular & Renal System
Cardiovascular 4
• Calcium : synergises with digitalis →precipitates toxicity.
• Quinidine : reduces binding of digoxin to tissue proteins as well as its renal and biliary clearance.
• Adrenergic drugs : can induce arrhythmias in digitalized patients; both increase ectopic automaticity.
• Avoid digitalis in Wolff-Parkinson-White arrhythmias

79. Ans. is a, b, c and d i.e. Eclampsia; Cardiac arrhythmia; Seizure; Tetany


Ref. Davidson 19/e, p 172, 520, 717, 626; Harrison 16/e, p 44

• Mg++ used in Eclampsia and Eclamptic seizure. The magnesium may prevent seizure by interacting with N–
methyl D-aspartate receptors.
• Magnesium is used for ventricular tachycardia and Torsades de pointes. It is also used in tetany due to
magnesium deficiency.
• Mg can be used in acute severe asthma in the dose of 25 mg/kg iv.

80. Ans. a, b and c i.e. Epinephrine; Glucocorticoids; and NSAID Ref. Harrison 17/e, p 1536-1537

Drugs causing Neutrophilia :


• Glucocorticoids
• Epinephrine
• NSAID
• Lithium

81. Ans. is a, b and c i.e. Aspirin, Metoprolol; and Streptokinase Ref. Harrison 16/e, p 1454 - 1455

Aspirin :
• The use of antiplatelet and antithrombotic therapy in early stage of STEMI plays an important role to establish
and maintain patency of the infarct - related artery.
• Aspirin is the standard antiplatelet agent for patients with STEMI (ST elevation MI).

Metoprolol :
• β - blockers like metoprolol in acute condition improves the myocardial oxygen demand, decreases pain,
reduces infarct size and decreases the incidence of serious ventricular arrhythmias.
• ACE inhibitors reduce mortality rate after STEMI, and the mortality benefits are additive to those achieved
with aspirin and beta blockers. The mechanism involves a reduction in infarct size. They should be given
within 24 hours to all patients with STEMI.

Steptokinase :
• Fibrinolysis therapy by plasminogen activators like streptokinase / urokinase should be initiated within 30
mins of presentation.
• Calcium channel blockers have little value in the acute phase of MI.

293
4 Self Assessment & Review Phar macology
Pharmacology

82. Ans. is a and d i.e. Prazosin; and Losartan Ref. KDT 6/e, p 488, 545, 543

• Prazosin and Losartan have beneficial or no effect on lipid profile while the other drugs in options
have deleterious effect as shown :
Drug Lipid Metabolism
Prazosin – Favourable effect on metabolism
– Lowers LDL cholesterol and triglyceride levels
– Increases HDL cholesterol
Losartan – No significant effect on plasma lipid profile
Propranolol – Unfavourable effect on lipid profile
– It raises triglyceride level and LDL/HDL ratio
Calcium channel blockers – Deleterious effect on plasma lipid profile

83. Ans. is a, c and e i.e. It increases effective refractory period; Causes paradoxical tachycardia; and
Cinchonism is seen Ref. KDT 6/e, p 511 - 512

• Quinidine has the following properties :


Actions • It has class IA + Class III antiarrhythmic properties + antivagal action.
• It decreases automaticity in atrial fibrilations and other ectopic foci; this serves to extinguish
extrasystoles and is important in prophylaxis of reentrant arrhythmias.
• It increases effective refractory period (ERP) more than action potential duration (APD) so
that the ratio ERP / APD is increased. (Options ‘a’ and ‘d’)
• It depresses myocardial contractility in damaged hearts.
MOA • It blocks myocardial Na+ channels in the open state - reduces automaticity and maximal rate of
0 phase depolarisation in frequency dependent manner.
• APD is increased due to K+ channel block.
• ERP is increased by its moderate effect on recovery of Na+ and K+ channels.
USES • To maintain sinus rhythm after AF or AFl has been terminated by DC shock.
• To prevent recurrences of VT.
S/E • Gastrointestinal intolerance is the most common side effect (dose limiting)
• Cinchonism occurs at higher dose - ringing in ears, deafness, vertigo, headache, visual
disturbances, mental changes and delirium – Option ‘e’
• Idiosyncrasy and hypersensitivity
• It can precipitate failure in dose dependent manner, esp. in patients with low cardiac reserve
• Paradoxical tachycardia : when used for conversion of AF or AFl, it reduces atrial rate

Less number of impulses enter AV node, concealed conduction is reduced

Refractoriness of the A-V node is decreased; antivagal action adds to this

More impulses pass to the ventricle

Tachycardia occurs
(this is called ‘paradoxical’ because quinidine,
a cardiac depressant, is not expected to increases heart rate) – Option ‘c’

294
Phar macology of Car
Pharmacology diovascular & Renal System
Cardiovascular 4
84. Ans. is a and c i.e. ↑ synthesis of factor II, VII, IX and X; and Causes hemolytic anemia
Ref. KDT 6/e, p 595

Vitamin K

• Vit K acts as a cofactor in the synthesis of coagulation proteins by the liver :


– Factor II (prothrombin) – Factor VII
– Factor IX – Factor X
• Menadione (K3) – can cause hemolysis
• Patients with G-6-PD deficiency and neonates are specially susceptible for haemolysis
• Half life of vit. K is 72 hours

85. Ans. is a and b i.e. Hydralazine; and Nifedipine Ref. KDT 6/e, p 504

Arteriolar dilators Venodilators Mixed dilators


(Primarily ↓ after load) (primarily ↓ preload) ↓pre and after load)
(↓
• Hydralazine • Nitrates : • ACE inhibitors
• Minoxidil – Glyceryl trinitrate • Losartan (AT1 antagonist)
• Ca channel blockers
2+
– Isosorbide dinitrate • Prazosin (α blocker)
1

(Nifedipine) – Isosorbide mononitrate • Phentolamine


• Pot. channel openers – Erythrityl tetranitrate • Nitroprusside
(Nicorandil) – Penta erythrital
• Diazoxide tetranitrate

86. Ans. is d i.e. ACE Inhibitors Ref. KDT 6/e, p 484

• Acute renal failure (ARF) is precipitated by ACE inhibitors in patients with bilateral renal artery stenosis due
to dilatation of efferent arterioles and fall in glomerular filteration pressure. So, ACE inhibitors are
contraindicatated in such patients.
• Other drugs in the options are not known to cause renal failure.

295
4 Self Assessment & Review Phar macology
Pharmacology

Drugs safe in Renal failure


– Rifampicin – Doxycycline
– Metronidazole – Erythromycin
– Cefoperazone – Mezlocillin
– Ceftriaxone – Cefaclor
– Nafcillin

87. Ans is a and d i.e. Heart block; and Bradycardia Ref. KDT 6/e, p 139, 529, 530

When β blockers are given with


Verapamil or Diltiazem
↓ leads to
Additive sinus depression, conduction
defects or asystole may occur
(both drugs are cardiodepressant)
↓ leads to
Bradycardia, Heart block
and Cardiac arrest

Nifedipine (it is also a CCB) can be safely used with β - blockers because nifedipine causes tachycardia,
increases contractility and cardiac output due to reflex sympathetic stimulation. So, there is no gross alteration
on blood pressure by the above drug combination.

88. Ans is a, b, and c i.e. Spironolactone; Triamterene; and Amiloride Ref. KDT 6/e, p 560

Spironolactone, Triamterene, and Amiloride are the 3 potassium sparing diuretics.

Classification of Diuretics
• High Efficacy Diuretics a. Sulphamoyl derivatives
(Inhibitors of Na K 2Cl cotransport)
+ + –
– Furosemide
– Bumetanide
b. Phenoxy acetic acid derivative
– Ethacrynic acid
c. Organomercurials
– Mersalyl
• Medium Efficacy Diuretics a. Thiazides
– Chlorothiazide
– Hydrochlorothiazide
– Benzthiazide
– Hydroflumethiazide
– Clopamide
b. Thiazide like
– Chlorthalidone
– Metolazone
– Xipamide
– Indapamide

296
Phar macology of Car
Pharmacology diovascular & Renal System
Cardiovascular 4
• Weak Diuretics a. Carbonic anhydrase inhibitors
– Acetazolamide
b. Potassium sparing diuretics
– Spironolactone (Aldosterone antagonist)
– Triamterene
– Amiloride
c. Osmotic diuretics
– Mannitol
– Glycerol

89. Ans. is a, b, c and e i.e. Persistent cough; Taste changes; First dose hypotension; and Angioedema
Ref. KDT 5/e, p 451 - 452

Already explained, refer answer no. 50

90. Ans. is d i.e. Methemoglobinemia Ref. KDT 6/e, p 523 - 524, 526 - 527

• Analysing each option separately:

Option ‘a’ Nitrates are not given in hypertrophic obstructive cardiomyopathy as it causes
exaggeration of the murmur gradient.
Option ‘b’ Nitrates used in acute MI have been shown to reduce the area of necrosis by favourably
altering O2 balance in the marginal partially ischemic zone by reducing cardiac work.
Option ‘c’ Nitrates have been used in CHF and acute LVF. They afford relief by venous pooling of
blood →decreased end diastolic volume →improvement in left ventricular function by
Laplace law and regression of pulmonary congestion.
Option ‘d’ Methemoglobinemia is an adverse effect of nitrates and so they are avoided.

91. Ans is c i.e. Half life of Albumin – 15 hrs. Ref. KDT 6/e, p 622

Plasma expanders Substances used


• High molecular weight substances. • Human albumin
• Exert colloidal osmotic (oncotic) pressure • Dextran
• When infused i.v., retain fluid in the • Degraded gelatin polymer
vascular compartment. • Hydroxyethyl starch
• Polyvinyl pyrrolidone

Human Albumin
• It is obtained from pooled human plasma.
• 100 ml of 20% human albumin solution is osmotic equivalent to 400 ml of fresh frozen plasma or 800ml of
whole blood.
• It can be used regardless of patient’s blood group.
• It does not interfere with coagulation.
• Unlike whole blood or plasma, it is free of risk of transmitting serum hepatitis because the preparation is
heat treated.

297
4 Self Assessment & Review Phar macology
Pharmacology
• There is no risk of sensitization with repeated infusions.
• Half life is 20 days
• Is expensive
• Used in burns, hypovolemia, shock, acute hypoprotenemia, acute liver failure and dialysis.
Dextran

• It is a polysaccharide obtained from sugar beet .


• Available in 2 forms :
– Dextran 70 (more commonly used)
– Dextran 40
• It expands plasma volume for 24 hrs.
• Dextran has nearly all properties of ideal plasma expander except :
a. It may interfere with blood grouping and cross matching.
b. Some polysaccharide reacting antibodies, if present in the patient in sufficient quantities, may cross react
with dextran and trigger anaphylactic reaction.
c. It can interfere with coagulation and platelet function → prolong bleeding time →not to be used in :
1. Hypofibrinogenemia
2. Thrombocytopenia
3. Presence of bleeding

92. Ans. is d i.e. tPA Ref. CMDT 05, p 347

Comparision between various fibrinolytic agents given here t-PA has least anaphylatic.

Streptokinase Alteplase Reteplase Tenecteplase


(tPA) (TNK-tM)
• Source Group C Recombinant Recombinant Recombinant
streptococcus DNA DNA DNA
• T1/2 60 - 80 min. 5 - 10 min. 15 - 20 min. 120 min.
• Clot selectivity Low High High High
• Fibinogenolysis +++ + + +
• Bleeding + + + +
• Hypotension +++ + + +
• Allergic reaction ++ 0 0 +
• Re-occlusion 5-20% 10-30% – 5-20%

Note that no allergic reaction are seen with : Alteplase (t-PA) and Reteplase

93. Ans. is c i.e. AF with high ventricular rate Ref. KDT 6/e, p 500 - 501

• Digitalis is the drug of choice for controlling ventricular rate in atrial fibrillation, whether associated with CHF
or not.
• Digitalis reduces ventricular rate in AF by decreasing the number of impulses that are able to pass down the
A-V node and bundle of His.

298
Phar macology of Car
Pharmacology diovascular & Renal System
Cardiovascular 4
– It increases ERP of A-V node by direct, vagomimetic and antiadrenergic actions.
– A degree of AV block is naturally established in AF. Because of the relatively long ERP of A-V node,
many of the atrial impulses (~ 500/min) impinge on it while it is still refractory.
• When digitalis is given in AF, average ventricular rate decreases in a dose dependent manner.
• Dose should be adjusted to a ventricular rate of 70 - 80/min at rest. If this is not possible with digitalis alone,
a β blocker or verapamil is added.
• Digitalis has poor response and more toxicity in high output failure.
• In HOCM, β blockers is safe and is recommended (Digoxin is contraindicated).

94. Ans. is b i.e. Bretylium Ref. KDT 6/e, p 511, 516; Goodman & Gilman 11/e, p 921

• The chief use of bretylium is in ventricular fibrillation refractory to electrical defibrillation. So, it is
assumed to be a pharmacological defibrillator.
• Lignocaine and amiodarone are also used in ventricular fibrillation but they are not called as defibrillators.

Other important points about Bretylium :


• Bretylium prolongs action potential in normal purkinje cells to a greater extent than ischemic cells (in which
action potentials already are prolonged abnormally). Thus, it reduces heterogeneity of repolarisation times,
an effect that may suppress reentry and thus suppress origination of arrhythmia.
• Bretylium initially cause increased norepinephrine release from sympathetic neurons in heart and
then, inhibition of subsequent uptake. Interfering neuronal release of NE (Nor Epinephrine).
• Oral absorption is erratic and it is excreted unchanged in urine.

95. Ans. is a i.e. Saluretic effect is greater that thiazide Ref. KDT 6/e, p 561

Saluretic effect of Triamterene is 3%.


Saluretic effect of Thiazide is 8%.
So, saluretic effect of triamterene is less than thiazide.
Potassium sparing diuertics

1. Aldosterone antagonist : • Spironolactone


2. Directly acting (Inhibitors of renal epithelial Na channel) :
+
• Triamterene, Amiloride

• Triamterene is a K+ sparing diuretic and mechanism of action (inhibitors of renal epithelial Na+ channel) is
same as amiloride.
• Triamterene is often combined with thiazide because it is a K+ sparing diuretic which reduces risk of
hypokalemia caused by thiazide.

96. Ans. is b i.e. Nicorandil Ref. KDT 6/e, p 534

K+ channel openers

• Nicorandil • Cromakalim
• Pinacidil • Minoxidil
• Diazoxide

299
4 Self Assessment & Review Phar macology
Pharmacology

MOA of K+ channel openers


Since intracellular K concentration is much higher (150 mmol)
+

compared to extracellular (4-5 mmol), K channel opening results


+

in outflow of K ions and hyperpolarisation.


+


Smooth muscle relaxation (both vascular and visceral)

97. Ans. is a i.e. Angiotensin converting enzyme inhibition


Ref. KDT 6/e, p 480, 542; Lippincot’s ‘Pharma’ 2/e, p 186

Enalapril acts by Angiotensin converting enzyme inhibition.


The different effects of angiotensin converting enzyme (ACE) inhibitors are summarised in the figure
below :

98. Ans. is a i.e. ↑ blood viscosity Ref. KDT 6/e, p 572

Mannitol
• It is an osmotic diuretic.
• It increases GFR and inhibits renin release.
• It decreases intracranial tension, intraocular pressure.
• It increases plasma osmolality, blood viscosity.

300
Phar macology of Car
Pharmacology diovascular & Renal System
Cardiovascular 4
99. Ans. is b i.e. Small unionised molecule is not absorbed orally Ref. KDT 6/e, p 597 - 598

Option ‘b’ Heparin is a very large, highly ionised molecule, therefore not absorbed orally.
Option ‘a’ Heparin acts indirectly by activating antithrombin III (AT III, a serine proteinase inhibitor). The
heparin - AT III complex then binds to the clotting factors of intrinsic and common pathways (Xa,
IIa, IXa, XIIa, XIIIa) and inactivates them but not factor VIIa operative in the extrinsic pathway.
Option ‘c’ Protamine sulphate is an antagonist of heparin. However, it is needed infrequently because
the action of heparin disappears by itself in a few hours.
It is more commonly used when heparin action needs to be terminated rapidly e.g., after
cardiac or vascular surgery.
Option ‘d’ Following injection of heparin, lipoprotein lipase is released from its heparan sulfate binding
into the circulation and is accompanied by clearing of lipaemia. (Harper)

100. Ans. is d i.e. None of the above Ref. KDT 6/e, p 572

• Actions of Osmotic Diuretics (e.g., Mannitol)


– Expands extracellular fluid volume.
– Increases glomerular filteration rate (GFR).
– Inhibits renin release.
– Increases renal blood flow especially to medulla - medullary hypertonicity is reduced- corticomedullary
osmotic gradient is dissipated - passive salt reabsorption is reduced.
– Retains water isoosmotically in proximal tubule.
– Inhibits salt reabsorption in the thick ascending limb of the loop of Henle.

101. Ans. is c i.e. Triamterene Ref. KDT 6/e, p 571

Triamterene causes rise in blood urea and serum K+, so it is contraindicated in patients with CRF.

Potassium sparing diuertics

1. Aldosterone antagonist : • Spironolactone


2. Directly acting (Inhibitors of renal epithelial Na channel) :
+
• Triamterene, Amiloride

102. Ans. is c i.e. Hypoxia Ref. Ganong 19/e, p 592

Among the given options, hypoxia is the potent vasodilator of coronary circulation.
Coronary vasodilators

• O lack
2
• ↑ CO 2

• ↑ H ,K+ +
• ↑ lactate
• ↑ Prostaglandins

301
4 Self Assessment & Review Phar macology
Pharmacology
103. Ans. is c i.e. Inhibiting NA+K+ ATPase Ref. Katzung 9/e, p 206

Cardiac glycosides (Digitalis) increase the intensity of the interaction of actin and myosin filaments of
cardiac sarcomere by increasing free calcium concentration in the vicinity of the contractile proteins
during systole.
• The increase in calcium concentration is the result of a two step process:
First, an increase of intracellular sodium concentration because of Na+/K+ ATPase inhibition and second,
a relative reduction of calcium expulsion from the cell by the sodium-calcium exchanger caused by the
increase in intracellular sodium.

Their initial action is to inhibit cardiac


membrane. Na / K ATPase+ ±


↓ Na / Ca exchange
+ +


↑ Ca in SR
++


↑ Ca release and binding to troponin
++


Tropomyosin moves

↑ Actin and Myosin interaction

↑ Contractile force

104. Ans. is b i.e. Lignocaine Ref. KDT 6/e, p 498 - 499

For digitalis induced ventricular arrhythmias:


– Lignocaine IV repeated as required, is the drug of choice.
– Phenytoin is also useful but not preferred now, because sudden deaths have occurred when it was injected
IV in digitalis intoxicated patients.
– Quinidine and Procainamide are contraindicated in digitalis induced ventricular arrhythmias.

Digitalis toxicity DOC

• For Tachyarrhythmias KCL


• For Ventricular arrhythmias Lignocaine
• For Supraventricular arrhythmias Propranolol
• For AV block and bradycardia Atropine

302
Phar macology of Car
Pharmacology diovascular & Renal System
Cardiovascular 4
CHAPTER REVIEW
• This section includes questions of V arious Other PGMEES from 1980 – 2006.
Various
• Questions are ar ranged in increasing order of page sequence of KDT 5 Edition. This is
arranged
done to mak e refer
make ral system more easy and uncomplicated to save the precious time
referral
of PGMEE Aspirant.

1. Dipyridamol is a drug which acts by inhibiting up- 6. Adenosine is the drug of choice in : (Manipal 05)
take and degradation of adenosine; adenosine a. PSVT
here acts by : (Manipal 07) b. Heart block (type-1)
a. ↑ total coronary flow c. Heart block (type-2)
b. ↑ mycocardial contractility d. Atrial fibrillation
c. Smooth mucle constriction [Ref. KDT 6/e, p 518-519]
d. Smooth muscle 7. Orlistat is effective in obesity by : (COMEDK 05)
[Ref. KDT 6/e, p 525] a. Reduction of food intake
2. Drug of choice in a patient with diabetic nephr- b. Beta-1 adrenoreceptor agonist activity
opathy and hypertension : (Manipal 07) c. Inhibition of pancreatic lipase
a. Enalapril d. 5 HT receptor agonist activity
b. Verapamil [Ref. KDT 6/e, p 131
c. CCB 8. Site of action of ACE inhibitors : (Manipal 04)
d. Diuretics a. Proximal tubules
[Ref. KDT 6/e, p 484] b. Distal tubules
3. All are true about mannitol except : c. Henles loop
a. ↓ GFR (Manipal 07) d. Collecting ducts
b. Osmotic diuretic [Ref. KDT 6/e, p 481]
c. Non-electrolyte 9. True regarding “endothelial dependent relaxing
d. Renal vasodilator factor” : (Manipal 04)
[Ref. KDT 6/e, p 572] a. Also known as NO
4. All of the following may be used in pregnancy as- b. Acts on receptors present on the endothelial
sociated hypertension except : (Manipal 06)4 surface
a. Nifedipine c. Kinins; AcH and Histamine trigger them
b. Captopril d. All of the above
c. Methyldopa [Ref. KDT 6/e, p 97]
d. Hydralazine 10. Drug causing osmotic diresis : (Manipal 04)
[Ref. KDT 6/e, p 553] a. Mannitol
5. Mechanism of action of ACE inhibitors in ↓BP : b. Isosorbide
a. ↓Cardiac output (Manipal 05) c. Glycer
b. ↓Peripheral resistance d. All of the above
c. Dilatation of capacitance vessels [Ref. KDT 6/e, p 561]
d. None of the above
[Ref. KDT 6/e, p 483]

Answer 1. d. Smooth ... 2. a. Enalapril 3. a. ↓ GFR 4. b. Captopril 5. b. ↓Peripheral ...


6. a. PSVT 7. c. Inhibition ... 8. a. Proximal ... 9. d. All of the ... 10. d. All of the ...

303
4 Self Assessment & Review Phar macology
Pharmacology
11. Dry cough is an adverse effect seen in adminis- 18. A common side effect of sodium nitroprusside :
tration of which of the following : a. Metabolic acidosis (Manipal 01)
a. Nifedipine (Manipal 03) b. Hypotension
b. Enalapril c. Thiocyanate toxicity
c. Prazocin d. Thrombocytopenia
d. Furesimide [Ref. KDT 6/e, p 524]
[Ref. KDT 6/e, p 484] 19. Vassodilators are all except : (Up 01)
12. Digoxin is used in the management of : a. Xanthines
a. HOCM (Manipal 03) b. Potassium - infusion
b. Atrial flutter c. Losartan
c. Ventricular arrythmia’s d. Flecanide
d. Ectopics [Ref. KDT 6/e, p 504 - 505]
[Ref. KDT 6/e, p 502 20. Mechanism of action of cardiac glycosides are all
13. Which is the primary anatomical site of action of except : (Up 01)
hydrochlorthiazide ? (Manipal 03) a. Increases force of cardiac contraction
a. Proximal tubles b. Negative inotropriacation cardiac glycosides
b. Early distal tubles c. Binding of glycoside to Na+ K+ ATPase is slow
c. Late distal tubles d. Progressive accmlation of Na+ intracellularly
d. Thick ascending loop of health [Ref. KDT 6/e, p 496]
[Ref. KDT 6/e, p 564] 21. Mechanism of action of cyanide poison :
14. Most comon side effects of oral nitrates : a. Inhibit cytochrome oxidase (Up 01)
a. Bradycardia (Manipal 03) b. Inhibit cytochrome P45 enzyme
b. Headache c. Inhibit phosphodiesterases
c. Hypotension d. Increasing intracellular cGMP
d. Oral ulceration [Ref. KDT 6/e, p 527]
[Ref. KDT 6/e, p 524] 22. Mechanism of action of moricizine :
15. MOA of amodarone is : (Manipal 02) a. Prolongs action potential duration(Manipal 00)
a. Prolongs APD b. Shortens action potential duration
b. Blocks inactivated Na+ channel c. No change in A.P.
c. Inhibits myocardial Ca++ channel d. None of the above
d. All of the above [Ref. KDT 6/e, p 511 - 513]
[Ref. KDT 6/e, p 515-516] 23. Drug effect correlates with plasma concetration
16. Treatment of Nephrogenic diabets insipidus is by: with:
a. Thiazides (Manipal 02) a. Digoxin (Manipal 00)
b. Carbamazepine b. Hit and run drugs
c. Vasopressin c. Warfarin
d. Cyclophosphamide d. Diuretics
[Ref. KDT 6/e, p 564 - 565] [Ref. KDT 6/e, p 498]
17. MOA of adenosine is by : (Manipal 01) 24. Nitric oxide acts by : (Manipal 00)
a. Increases K+ Effux and decreasing Ca++ re- a. Inhibiting Cox 1
lease in heart muscle b. Inhibiting Cox 2
b. Inhibiting K+ channels c. Increasing cGMP
c. Inhibiting Na+ K+ ATPAse d. Decreasing cGMP
d. Increases Ca++ influx [Ref. KDT 6/e, p 523 - 524]
[Ref. KDT 6/e, p 518 - 519]

Answer 11. b. Enalapril 12. b. Atrial ... 13. b. Early ... 14. b. Headache 15. d. All of the ...
16. a. Thiazides 17. a. Increases... 18. b. Hypotension 19. d. Flecanide 20. b. Negative ...
21. a. Inhibit ... 22. a. Prolongs... 23. a. Digoxin 24. c. Increasing ...

304
Phar macology of Car
Pharmacology diovascular & Renal System
Cardiovascular 4
25. Which of the following do not reduce preload on 32. Absolute contra indicated anti hypertensive drug
heart : (AIIMS Dec. 94) in pregnancy is : (AIIMS Nov. 93)
a. Glycery trinitrate a. Enalapril-ACE inhibitors
b. Isosorbid dinitrate b. Diazoxide-Na nitrop
c. ACE inhibitor c Atenelol-Non selective β-blockers
d. Hydralazine reduce after load d. Nifedipine -Reserpine
[Ref. KDT 6/e, p 547 - 548] [Ref. KDT 6/e, p 554]
26. All are true about calcium channel blocker except 33. Which of the following intracts with digitalis
a. Dilitiazen, does not cause tachycardia only in- mamximally : (AIIMS Nov. 93)
fection (AIIMS May 94) a. Frusemide
b. Hyperkalemia may occur with Nifedipine mild b. Triamterene
nitriuretic action c. Amiloride
c. Verapmil is good antiarrythmic used in PSVT d. Spironlactone
d. Nifedipine may be give in congestive heart fail- [Ref. KDT 6/e, p 499]
ure 34. Which drug has wide therapeutic index :
[Ref. KDT 6/e, p 530] a. Digoxin (AIIMS Nov. 93)
27. Buprenorphine is partial agonist of : b. Lithium
a. Mu (m. recptor (AIIMS May 94) c. Phenytoin
b. Delta (d. receptor d. Penicillin
c. Kappa (k. receptor [Ref. KDT 6/e, p 35]
d. Sigma (s. receptor 35. Pulmonary fibrosis side effect of : (AIIMS Nov. 93)
[Ref. KDT 6/e, p 443] a. Methotrexate
28. Heparin effect is increased by : (AIIMS May 94) b. Vincristine
a. Narcotic analgesic c. Bleomycin
b. Non steroidal anti inflammatory drug d. Cyclophosphamide
c. Oral contraceptive [Ref. KDT 6/e, p 823 - 827]
d. Inhalation agent 36. Long term use of nitrates lead to decreased e f -
[Ref. KDT 6/e, p 602] fect because of : (AIIMS Nov. 93)
29. Digoxin dose has to be regulated in all, except : a. Development of resistance
a. Hepatotoxicity (AIIMS May 94) b. SH group in the enzyme
b. Old age c. Decreased oral absorption
c. Hypokalemia d. Increased resistance
d. Hypoxia [Ref. KDT 6/e, p 524]
[Ref. KDT 6/e, p 499] 37. All are used in digoxin toxicity, except :
30. Verapamil is contraindicated in all except : a. Lignocaine (AIIMS Nov. 93)
a. PSVT with AV block (AIIMS May 94) b. Hemodialysis
b. Variant angina c. Phenytoin
c. Beta adrenergic block C/1 d. Potassium
d. Digitalis toxicity C/1 [Ref. KDT 6/e, p 407]
[Ref. KDT 6/e, p 529 - 530] 38. Gancyclovir is used in treatment of it is more a c -
31. Levostatin causes : (AIIMS May 94) tive against : (AIIMS Nov. 93)
a. Increased LDL receptor in cell a. Adeno virus
b. Decrease HDL cholesterol b. Cytomegalo virus
c. Decrease LDL cholesterol c. EB virus
d. Increase SGOT/SGPT ratio d. Arena virus
[Ref. KDT 6/e, p 619] [Ref. KDT 6/e, p 769 - 770]

Answer 25. d. Hydralazine ... 26. b. Hyperkalemia... 27. a. Mu ... 28. b. Non steroidal ... 29. a. Hepatotoxicity
30. b. Variant ... 31. a and c 32. a. Enalapril-... 33. a. Frusemide 34. c. Phenytoin
35. c. Bleomycin 36. b. SH group ... 37. b. Hemodialysis 38. b. Cytomegalo ...

305
4 Self Assessment & Review Phar macology
Pharmacology
39. Verapamil is used in all, except : (AIIMS May 93) c. Nausea and vomiting
a. Angina pectoris d. Pulsus bigeminus
b. Atrial fibrillation [Ref. KDT 6/e p 497]
c. Ventricular tachycardia 47. Which of the following is wrong about calcium
d. None of the above hannel blacker :
[Ref. KDT 6/e, p 530] a. Verapamil is useful in sick sinus syndrome
40. What is true about levostatin : (AIIMS May 93) b. Nifedipine is useful in exercise induced asthma
a. Inhibit HMG CoA synthetase c. Nifedipine is good for CHF [June 1991]
b. Inhibit HMG-CoA reductase d. Verapamil has good antiarrhythmic action
c. Mysitis may result from prolonged use [Ref. KDT 6/e, p 530]
d. Decrease synthesis of cholesterol and causes 48. In proximal convoluted tubule, sodium reabsorb
lenticular opacity ption is prevented by : (AIIMS June 1991)
[Ref. KDT 6/e, p 618] a. Triametrene
41. All are contraindications for use of streptokinase b. Acetazolamide
in management of MI is all, except : (AIIMS May 93) c. Ethacrynic acid
a. Active peptic ulcer d. Mannitol asmotic
b. Coagulopathy [Ref. KDT 6/e, p 568]
c. NIDDM 49. Dose of digoxin is reduced in all, except :
d. Hypotension a. Hypokalemia (AIIMS June 91)
[Ref. KDT 6/e, p 617] b. Hepatic disease
42. Digoxin induced arrythmia is treated by : c. Old person
a. Quinidine (AIIMS Dec. 92) e. Hypoxia in
b. Procainamide [Ref. KDT 6/e, p 499]
c. Phenytoin not preferred now 50. Digitalis complication are all excpet : (Up 00)
d. Lidocaine Doc a. Atrial fibrillation
[Ref. KDT 6/e, p 498] b. Atrial flutter
43. All are side effects of lisinopril except : c. Paroxymal atrial tachycardia
a. Dizziness (AIIMS Dec. 92) d. Congestive heart failure
b. Jaundice [Ref. KDT 6/e, p 498]
c. Cough 51. Thromboxane-A synthetase is inhibited by :
d. Angioneurotic edema a. Dazoxiben (Up 00)
Ref. KDT 6/e, p 205] b. Dipyramidole
44. Which calcium channel blocker : (AIIMS Dec. 92) c. PGI2
a. Nifedipine 52. Thrombolytic therapy is contraindicated in all ex-
b. Verapamil cept:
c. Dilitiazem smoth a. Diebetes (Up 00)
d. Nimodipine b. Hypertension
45. In case of hypertensive emergency all are given c. Peptic ulcer
except : (AIIMS Dec. 92) d. Acute MI
a. Indapamide [Ref. KDT 6/e, p 606]
b. I/V Hyralazine 53. Digoxin is not used with A/E : (UP 99)
c. Sublingual Nifedipine a. Spiranolactone
d. Trimethaphan b. Thiazides
[Ref. KDT 6/e, p 554] c. Quinidine
46. All are adverse effect of digitalis, except : d. Verapamil
a. Ventricular tachycardia [June 1991] [Ref. KDT 6/e, p 499]
b. Vasodilation

Answer 39. d. None of the ... 40. b. Inhibit HMG-... 41. c. NIDDM 42. d. Lidocaine ... 43. b. Jaundice
44. a. Nifedipine 45. a. Indapamide 46. b. Vasodilation 47. a. Verapamil ... 48. b and d
49. b. Hepatic ... 50. b. Atrial flutter 51. a. Dazoxiben 52. d. Acute MI 53. b. Thiazides

306
Phar macology of Car
Pharmacology diovascular & Renal System
Cardiovascular 4
54. Heparin induced thrombocytopenia, true is : b. Nifidipine
a. Generally severe (UP 99) c. Nimodipine
b. Due to platelet function abnormalities d. Ditiazem
c. Antibodies are formed to heparin platelet com- [Ref. KDT 6/e, p 529]
plex 62. Following are potassium sparing diuretics except:
d. Should be treated by heparin antagonist a. Spironolactone (UP 97)
[Ref. KDT 6/e, p 599] b. Trimeterene
55. High ceiling diuretic is : (UP 99) c. Indapamidwe
a. Acetazolamide d. Amiloride
b. Spironaloctone [Ref. KDT 6/e, p 569]
c. Furesemide 63. Drug of choice in patient of cirrhosis with portal
d. Mannitol hypotension : (UP 97)
[Ref. KDT 6/e, p 562] a. Furosemide
56. Antirrhythmic which causes tachycardia : b. Spironolactone
a. Nifedipine (UP 99) c. Benzthiazide
b. Verapamil d. Chlorthiazide
c. Diltiazem [Ref. KDT 6/e, p 563]
d. Quinidine 64. Streptokinase act : (UP 97)
[Ref. KDT 6/e, p 511] a. Plasminogen activator
57. Both pre & after load is decreased by : (UP 98) b. Anti fibrinolytics
a. Hydralazine c. Direct dissolves thrombus
b. Nifedipine d. Oral anticoagulants
c. Na-nitropruside [Ref. KDT 6/e, p 606]
d. Minoxidil 65. True about minoxidil are : (UP 96)
[Ref. KDT 6/e, p 523] a. K+ channel opener
58. The function of nitrates are A/E : (UP 97) b. Used in hypertension
a. Decreases total peripheral resistancw c. Calcium channel blocker
b. Act on both preload and after load d. Used in alopecia
c. 1Total coronary blood flow [Ref. KDT 6/e, p 549]
d. Used in angina 66. All are K+ sparing diuretics except : (UP 96)
[Ref. KDT 6/e, p 524] a. Spirolactone
59. Drug of choice in digitalis induced arrythmias : b. Propenone
a. Phenytoin (UP 97) c. Bumetamide
b. Qunidine d. Amiloride
c. Procainamide [Ref. KDT 6/e, p 561]
d. Pyritinol 67. Increased uric acid secretion by : (UP 96)
[Ref. KDT 6/e, p 498] a. Bumetamide
60. The drug induced metabolism occure with digi- b. Furesemide
talis are A/E : (UP 97) c. Acetozolamide
a. Amidarone d. Hydrochlorthiazide
b. Quinidine [Ref. KDT 6/e, p 570]
c. Verapamil 68. ACE inhibitor is : (UP 96)
d. Captopril a. Captopril
[Ref. KDT 6/e, p 499] b. Saralasin
61. Which CCB has maximum conduction effect on c. Diltiazem
heart is : (UP 97) d. Encanimide
a. Verapamil [Ref. KDT 6/e, p 203]
Answer 54. c. Antibodies ... 55. c. Furesemide 56. d. Quinidine 57. c. Na-nitrop ... 58. c. 1Total ...
59. a. Phenytoin 60. d. Captopril 61. a. Verapamil 62. c. Indapami ... 63. b. Spironol ...
64. a. Plasminogen ... 65. c. Calcium ... 66. c. Bumetamide 67. c. Acetozolamide 68. a. Captopril

307
4 Self Assessment & Review Phar macology
Pharmacology
69. The CCB with predominant peripheral effect is : d. Furosemide
a. Verapamil (UP 96) [Ref. KDT 6/e, p 561, 572]
b. Nifedipine 77. Centrally acting antihypertensive daigs are A/E
c. Dilitazem a. α-methyl dopa (UP 93)
d. Galapamil b. Clonidine
[Ref. KDT 6/e, p 530] c. Guonabenz
70. In congestive heart failure, all drug causing d. Trimethaphan
aggrevation and precipitation except : (UP 96) [Ref. Harrison 14/e, p 439]
a. Theophylline 78. Potassium sparing diuretics is : (UP 93)
b. Propranolol a. Benzthiazide
c. Carbenoxolone b. Bumetanide
d. Phenyl butazone c. Etliacrynic acid
[Ref. KDT 6/e, p 233] d. Triamterene
71. Enalapril have advantage over captopril in : [Ref. KDT 6/e, p 570]
a. Less hypotensive first dose effect (UP 95) 79. Carbenoxolone sodium is : (UP 93)
b. More hypotensive first dose effect a. Ulcer healing drug
c. Loss of taste are more comon b. Ulcer protective drug
d. Asumption is affected by food c. Prostaglandin analogues
[Ref. KDT 6/e, p 485] d. Proton pump inhibitor
72. ACE inhibitor is : (UP 95) [Ref. KDT 6/e, p 640]
a. Captopril 80. Winch of the following drug inhibits VLDL produc-
b. Nefedipine tion and lipoK sis (UP 93)
c. Seralasin a. Nicotinicacid
d. Pirenzepine b. Clofibrate
[Ref. KDT 6/e, p 483] c. Probucol
73. Inodilator drug is : (UP 95) d. Cholestyramine
a. Amiodarone [Ref. KDT 6/e, p 622]
b. amirinone 81. Which of the following antihypertensives is an
c. Enoxmone alpha-adrenoceptor blocker : (KARN 95)
d. Thevetin a. Clonidine
[Ref. KDT 6/e, p 507] b. Alpha-methyldopa
74. Diuretic of choice in cirrhosis is : (UP 94) c. Atenolol
a. Spironolactone d. Prazosin
b. Frusemide [Ref. KDT 6/e, p 132]
c. Bumetamide 82. K channel openers are all except : (MAHE 01)
+

d. Thiazide a. Minoxidil
[Ref. KDT 6/e, p 570] b. Nicorandil
75. Action of digitalis isprolonged because of : c. Glebenclamide
a. Slow metabolism (UP 94) d. Penacidil
b. Highly protein bound [Ref. KDT 6/e, p 266]
c. Enterohepatic circulation 83. New drug for increasing sensitivity of cells to in-
d. Slow distribution [Ref. KDT 6/e, p 497] sulin : (CUPGEE 02)
a. Glienclamide
76. Osmotic diuretics is : (UP 94)
b. Velnafaxine
a. Spironolactone
c. Pioglitazone
b. Mannitol
d. Glipizide
c. Indapamide
[Ref. KDT 6/e, p 260, 447; Harrison 15/e, p 2544]

Answer 69. b. Nifedipine 70. a. Theophylline 71. a. Less ... 72. a. Captopril 73. b. amirinone
74. a. Spironol ... 75. None 76. b. Mannitol 77. d. Trimethaphan 78. d. Triamterene
79. a. Ulcer ... 80. a. Nicotinicacid 81. d. Prazosin 82. c. Glebencl ... 83. c. Pioglitazone

308
Phar macology of Car
Pharmacology diovascular & Renal System
Cardiovascular 4
84. The following drugs are useful in diabetes 91. All of the following are true about Enalapril
insipidus except : (Karnat 96) except : (Delhi 93)
a. Desmopressin a. A prodrug
b. Lithium b. Used in treatment of hypertension
c. Chlorpropamide c. Blocks angiotensin I
d. Hydrochlorothiazide d. Inhibits ACE
[Ref. KDT 6/e, p 435 - 436] [Ref. KDT 6/e, p 485]
85. First drug of choice for acute pulmonary edema 92. ACE inhibitors are given in Diabetes mullitus in
is : (Karnat 05) which situation : (Jharkand 03)
a. Frusemide a. Type I nephropathy
b. Morphine b. Type II nephropahty
c. Aminophyline c. Both
d. Oxygen d. None
[Ref. KDT 6/e, p 454, 461] [Ref. KDT 6/e, p 487]
86. hich of the following is an ACE inhibitor : 93. About Losartan - true is : (Calcutta 2K)
a. Saralasin (Al 90) a. Cough
b. Diltiazem b. No angioedema
c. Captopril c. No hypotension with high renin hypertension
d. Spirinolactone d. No foetopathic effect
[Ref. KDT 6/e, p 483] [Ref. KDT 6/e, p 488 - 489]
87. Hyperkalemia may occur if potassium sparing 94. The mechanism of action of digitalis is :
diureties are given alongwith : (Karnat 96) a. Increase in heart rate with increase in oxygen
a. Captopril consumption (JIPMER 90)
b. Atenolol b. Increase in heart rate without increase in oxy-
c. Prazosin gen consumption
d. Clonidine c. Decrease in heart rate with out decrease in oxy-
[Ref. KDT 6/e, p 584] gen consumption
88. Spirinolactone & ACE inhibitors are not used d. Decrease in heart rate with decrease in oxygen
concurrently because of danger of : (JIPMER 98) consumption
a. Hypergiycemia [Ref. KDT 6/e, p 493 - 495]
b. Hypokalemia 95. Digoxin acts on the heart by : (Kerala 94)
c. Hyperkalemia a. Decreasing preload
d. Hypogycemia b. Increasing the force of contraction
[Ref. KDT 6/e, p 484] c. Decreasing the after load
89. Contraindication of lisinopril includes : (UPSC 93) d. Decreasing heart rate
a. Renovascular hypertension [Ref. KDT 6/e, p 493 - 494, 500 - 501; Harrison
b. Cardionegaly 15/e, p 1326]
c. Bilateral renal artery stenosis 96. Digoxin : (TNPSC 2K)
d. Unilateral renal artery stenosis a. Acts by blocking cardiac beta adrenergic recep-
[Ref. KDT 6/e, p 484; Harrison 16/e, p 1478 table tors
(230.11)] b. Acts by direct inhibition of Na+- K+- AT Pase pump
90. Which is correct of Enalapril as compared to c. Decreases intracellular calcium
captopril : (PGI 93) d. Acts on contractile proteins
a. Longer action [Ref. KDT 6/e, p 496]
b. More potent
c. Faster onset of action
d. More adverse reactions
[Ref. KDT 6/e, p 485]
Answer 84. b. Lithium 85. b. Morphine 86. c. Captopril 87. a. Captopril 88. c. Hyperkalemia
89. c. Bilateral ... 90. a and b 91. c. Blocks ... 92. c. Both 93. b. No angioedema
94. d. Decrease ... 95. b. Increasing ... 96. b. Acts by ...

309
4 Self Assessment & Review Phar macology
Pharmacology
97. The cardiac inotropic actions of digoxin are : 104. Diagoxin levels are increased by addition of which
a. A consequence of Na+/K+adenosine triphosp- of the following : (SGPGI 05)
hatase (ATPase. inhibition) (MAHA 05) a. Pheneytoin
b. A result of decreased calcium ion b. Quinidine
c. Due to accumulation of cyclic adenosine mono- c. Furesemide
phosphate (cAMP) d. Steroids
d. Due to changes in the electrical properties of [Ref. KDT 6/e, p 499]
cardiac muscle cells 105. Which of the following drug is contraindicated in
[Ref. KDT 6/e, p 496] hypertrophic cardiomyopathy : (SGPGI 05)
98. The toxic level of digitalis is : (BIHAR 91) a. Digoxin
a. 0.1-0.5 ng/ml b. Propranolol
b. 0.5-1.5 ng/ml c. Verapamil
c. 1.5-2.0ng/ml d. Phenylephrine
d. More than 2.4 ng/ml [Ref. KDT 6/e, p 500]
[Ref. KDT 6/e, p 498] 106. Drug which reduces afterload but not preload :
99. Which of the following is false about digoxin : a. Captopril (DELHI 92; UPSC 93)
a. Does reduced when used with quinidine (AI 90) b. Isosorbide dinitrate
b. Dose reduced in liver disease c. Nitroglycerin
c. Characteristic arythmia is PAT with block d. Hydralazine
d. Anorexia, nausea and vomiting are common [Ref. KDT 6/e, p 504]
side effects 107. All of the following are predominant arteriolar di-
[Ref. KDT 6/e, p 498; Harrison 16/e, p 1375] lators except : (AI 91)
100. The dosage of digoxin should be reduced in all a. Sodium Nitroprusside
except : (AI 91) b. Hydralazine
a. Old age c. Nifecfipine
b. Liver failure d. Nitrates [Ref. KDT 6/e, p 504]
c. Hypercalcemia 108. Class III antiarrythmic agents are all except :
d. Renal fairure a. Sotalol (Delhi 92)
[Ref. KDT 6/e, p 498; Harrison 16/e, p 1375] b. Amiodarone
101. Digoxin is eliminated from the body by : c. Verapamil
a. Tubular filtration (JIPMER 92) d. Bretylium
b. Hepatic Conjugation [Ref. KDT 6/e, p 511]
c. Glomerular filtration 109. A Patient presents to the casulaty with the
d. Oxidation [Ref. KDT 6/e, p 498] historyof ringing in the ears, headache for taking
102. True about Digoxin is all except : someheart drug’ the name of which is he does not
a. High therapeutic index (AIMS 87; Delhi 93) know. Examination reveals incresed QT in
b. Absorbed better than digitoxin ECG,theprobabledrug is : (NIMHANS 01)
c. Decreases AV conduction a. Quindine
d. Dose to be decreased in elderly b. Lignocaine
[Ref. KDT 6/e, p 498] c. Digoxin
103. Drug of choice for Digitalis Induced arythmia is : d. Amiodarone
a. Verapamil (JIPMER 92) [Ref. KDT 6/e, p 511 - 512]
b. Dilantin 110. The false statement amongst following is :
c. Quinidine a. Intravenous Procainamide causes hypotension
d. Encainide b. Quinidine causes cinchonism (Delhi PG 96)
[Ref. KDT 6/e, p 499; Harrison 16/e, p 1375] c. Lidocaine causes lupus like syndrome
d. Bretylium causes orthostatic hypotension
[Ref. KDT 5/e, p 477]
Answer 97. a. A consequence ...98. d. More than... 99. b. Dose ... 100. b. Liver ... 101. c. Glomerular ...
102. a and b 103. b. Dilantin 104. b. Quinidine 105. a. Digoxin 106. d. Hydralazine
107. d. Nitrates 108. c. Verapamil 109. a. Quindine 110. c. Lidocaine ...

310
Phar macology of Car
Pharmacology diovascular & Renal System
Cardiovascular 4
111. The ERP is reduced by (effective refractive period 118. False about calcium channel blockers is :
ERP) : (JIPMER 98) a. Prevents leak of calcium channels (Kerala 97)
a. Procainamide b. Causes increase in coronary blood flow
b. Amiodarone c. Cause negative inotropic effect
c. Lignocaine d. Closes voltage operated calcium channels
d. Sotalol [Ref. KDT 6/e, p 528]
[Ref. KDT 6/e, p 513] 119. Calcium channel blocker with maximum periph-
112. All of the following drugs are used in the prophy- eral action : (Al 91)
laxis of ventricular tachycardia except : a. Verapamil
a. Verapamil (UPSC 95) b. Nifedipine
b. Amiodarone c. Diltiazem
c. Diagoxin d. Dilazine
d. Mexiletine [Ref. KDT 6/e, p 529, table (39.2), 530]
[Ref. KDT 6/e, p 519 table (38.3)] 120. Following side effects of diltiazem differs from
113. Quinidine is contra indicated in : (Al 93) lisinopril : (AP 91)
a. Atrial fibrillation a. Bradycardia
b. Atrial flutter b. Ankle oedema
c. Digitoxicity c. Headache
d. Supraventicjular tachycardia d. Nausea
[Ref. KDT 6/e, p 511 - 513, 519] [Ref. KDT 6/e, p 449, 450, 494 & 495]
114. Drug of choice for Paroxysmal supraventricular 121. Calcium channel blocker with predominant periph-
tachycardia(PSVT) is : (Karnat 05) eral action is : (Al 90)
a. Verapamil a. Verapamil
b. Digitalis b. Diltiazem
c. Quinidine c. Depranil
d. Diphenylthydantion d. Nifidepine
[Ref. KDT 6/e, p 518] [Ref. KDT 6/e, p 529, table (34.2), 530]
115. Supra-ventricular arrhythmia can be treated with 122. Following are side effects of Nimodipine expect-
a. Adenosine (Karnataka-PGMEE 06) a. Hypotension (PGI 89, 91)
b. Flecainide b. Flushing
c. Tocainide c. Headache
d. Disopyramide d. Marked changes in GIT motility
[Ref. KDT 6/e, p 519] [Ref. KDT 6/e, p 532]
116. The beneficial effects of nitrates in the treatment 123. The major clinical use of nimodipine is in :
of angina include the following except : a. Hypertension (Kanataka PGMEE 06)
a. Decreased arterial pressure (KERALA 2K) b. Angina pectoris
b. Decreased ventricular volume c. Subarachnoid haemorrhage
c. Decreased diastole perfusion time d. Raynaud’s phenomenon
d. Decreased ejection time [Ref. KDT 6/e, p 532]
e. Decreased left ventricular diastolic 124. Calcium channel blocker with maximum effect on
[Ref. KDT 6/e, p 523 - 524] conduction of heart is : (J & K 05)
117. Longest acting Nitro glycerin in preparation is - a. Verapamil
a. Glyceryltrinitrate (JIPMER 93) b. Nifedipine
b. Amyl Nitrate c. Diltiazem
c. Penta Erythrital tetranitrate d. Phenylanine
d. Isosorbide dinitrate [Ref. KDT 6/e, p 529 - 532]
[Ref. KDT 6/e, p 526, table (37.1)]

Answer 111. c. Lignocaine 112. a. Verapamil 113. c. Digitoxicity 114. a. Verapamil 115. b. Flecainide
116. c. Decreased ... 117. c. Penta ... 118. a. Prevents ... 119. b. Nifedipine 120. a. Bradycardia
121. d. Nifidepine 122. d. Marked ... 123. c. Subarachnoid ... 124. a. Verapamil 125. c. Xylocaine

311
4 Self Assessment & Review Phar macology
Pharmacology
125. Drug of choice for ventricular arrhythmias due to 132. Postural hypotension and failure of ejaculation is
Myocardial infarction (MI) is : (Karnat 05) most commonly seen in treatment with :
a. Quinidine a. Guanethidine (KERALA 91)
b. Amiodarone b. Minoxidil
c. Xylocaine c. Propranolol
d. Diphenylthydantion d. Sodium Nitroprusside
[Ref. KDT 6/e, p 537] [Ref. KDT 6/e, p 549 - 550]
126. Drug of choice in Renovascular hypertension 133. Drug Contra indicated in Hypertensive cardiac
is : (JIPMER 93) failure is : (JIPMER 93)
a. Captropril a. Nifedepine
b. Aldomet b. Captopril
c. Nifedipine c. Atenolol
d. Propranolol d. Hydrallazine
[Ref. KDT 6/e, p 486, 540; Harrison 16/e, p 1479] [Ref. KDT 6/e, p 551]
127. Which antihypertensive does not have central ac- 134. ACE inhibitors are contraindicated in all of the
tions : (Al 92) following except : (Karnat. 96)
a. Clonidine a. Pregnancy
b. Indapamide b. Renal failure
c. Propranolol c. Single kidney
d. Alpha Methyl dopa d. Congestive cardiac failure
[Ref. KDT 6/e, p 542] [Ref. KDT 6/e, p 483 - 487, 503 - 505, 551 - 553]
128. All are true of minoxidil except : (AI 90) 135. Which antihypertensives is contraindicatd in
a. Increased potassium permiability pregnancy : (JIPMER 98)
b. Calcium channel blocker a. Methy dopa
c. Useful in hypertension b. Beta-Blockers
d. Increase hair synthesis c. Alpha and Beta Blockers
[Ref. KDT 6/e, p 534, 548] d. ACE inhibitors
129. True about minoxidil is : (KERALA 95) [Ref. KDT 6/e, p 551 - 552]
a. Increase hair growth 136. All of the following drugs are used in hypertensive
b. Antihypertensive emergencies except : (Delhi 93; AllMS 92)
a. Hydrallazine
c. Both
b. Labetolol
d. None
c. Minoxidil
[Ref. KDT 6/e, p 548]
d. Thiazide
130. K+Channel opener is : (MP 98)
a. Verapamil [Ref. KDT 6/e, p 554]
b. Sodium nitropruside 137. Diuresis is caused by : (AIMS 81; Delhi 92)
a. Mannitol
c. Minoxidil
b. Glycerol
d. Amrinone
c. Urea
[Ref. KDT 6/e, p 548, 534]
d. All of the above [Ref. KDT 6/e, p 560]
131. All the antihypertensive drugs has central effects
except : (AI 91) 138. Which of the following diuretics promote
a. Clonidine calcium reabsorption : (Karnt. 94)
b. Alpha methyldopa a. Chlorothiazide
c. Propranolol b. Frusemide
d. Sodium nitroprusside c. Acetazolamide
[Ref. KDT 6/e, p 548 - 549; Harrison 16/e, p 1473, d. Ethacrynic acid
1474] [Ref. KDT 6/e, p 564]

Answer 126. a. Captropril 127. b. Indapamide 128. b. Calcium ... 129. c. Both 130. c. Minoxidil
131. d. Sodium ... 132. a. Guanethidine 133. c. Atenolol 134. d. Congestive ... 135. d. ACE ...
136. d. Thiazide 137. d. All of the ... 138. a. Chlorothiazide

312
Phar macology of Car
Pharmacology diovascular & Renal System
Cardiovascular 4
139. The following thiazide diuretic is active if the GFR b. Spironolactone
is 30 - 40 mm Hg. : (JIPMER 02) c. Frusemide
a. Metalozone d. Acetazolamide
b. Chlorthiazide [Ref. KDT 6/e, p 570]
c. Chlorthalidone 146. All of the following occurs when aldosterone
d. Benzthiazide antagonist is given except : (JIPMER 95)
[Ref. KDT 6/e, p 564 - 565] a. Hyponatremia
140. The toxic effects produced by the thazide diuret- b. Hyperkalemia
ics include the following except : (Kerala 2K) c. Hypovolemia
a. Metabolic alkalosis d. Hyperuricemia
b. Metabolic acidosis [Ref. KDT 6/e, p 569 - 570]
c. Impaired carbohydrate tolerance 147. Side effects of Spironolactone are accelarated by
d. Hyperlipidemia concurrent administration by : (JIPMER 98)
e. Hyponatremia a. Thyroxine
[Ref. KDT 6/e, p 567 - 568] b. ACE inhibitor
141. Which does not cause hypercalcemia : (Kerala 03) c. Salicylates
a. Thiazides d. Paracetamol
b. Loop diuretics [Ref. KDT 6/e, p 484, 570]
c. Sipronolactanone 148. Drug which is useful in neuronal diabetes
d. All inspidus in both children and adults given intra
[Ref. KDT 6/e, p 568] nasally : (MAHE 01)
a. Vasopressin
142. Thiazide diuretics are best avoided in which of
the following conditions when associated with b. Desmopressin
hypertension? : (UPSC 01) c. Lypressin
a. Heart failure d. Presselin [Ref. KDT 6/e, p 576 - 577]
b. Diabetes mellitus 149. All of the following antihypertensives decrease
c. Heart block plasma renin activity except : (Karn 94)
d. Gout a. Clonidine
[Ref. KDT 6/e, p 568; Harrison 16/e, p 1478 table b. Methyldopa
(230.11)] c. Atenolol
143. Mineralocorticoid antagonist is : (TN 03) d. Chlorothiazide
a. Frusemide [Ref. KDT 4/e, p 542]
b. Cryptoterpne acetate 150. Synthetic factor Xa inhibitor is : (COMED 06)
c. Spironolactone a. Enoxaparin
d. Thiazides b. Dalptrin
[Ref. KDT 6/e, p 569] c. Fondaparinux
144. One of the following diuretics doesnot require its d. Heparin
presence in the tubular lumen for its [Ref. Katzung 9/e, p 547, 548]
pharmacological effects : (JIPMER 05) 151. In Dicumarol poisoning , which Vit K is :
a. Thiazide diuretics a. Menadione (JIPMER 98)
b. Loop diuretics b. Menaqunone
c. Carbonic anhydrase inhibitors c. Phytonadione
d. Aldosterone antagonists d. None of the above
[Ref. KDT 6/e, p 569] [Ref. KDT 6/e, p 595]
145. Canrenone is the prodrug of : (APPGE 04)
a. Ampicillin

Answer 139. a. Metalozone 140. b. Metabolic ... 141. b and c 142. d. Gout 143. c. Spironol ...
144. c. Carbonic ... 145. b. Spironol ... 146. d. Hyperuricemia 147. b. ACE ... 148. b. Desmopressin
149. d. Chlorothiazide 150. a and b 151. c. Phytonadi ...

313
4 Self Assessment & Review Phar macology
Pharmacology
152. Heparin is : (ROHTAK 97) 159. Streptokinase overdose is treated with :
a. Polysaccharide a. Warfarin (TNSC 2K)
b. Lipoprotein b. Aspirin
c. Monosaccharide c. Epsilon aminocaproic acid
d. Polyenoic acid d. Vit.K
e. Ceramide [Ref. KDT 6/e, p 597] [Ref. KDT 6/e, p 608]
153. Which one of the following statements is NOT 160. Which does not have antiplatelet action :
correct- (UPSC 02) a. Paracetamol (Kerala 95)
a. Unfractionated Heparin does not cross the pla- b. Dipiridamole
centa c. Aspirin
b. Low molecular Heparins can cross the placenta d. Ibuprofen
c. Warfarin crosses the placenta [Ref. KDT 6/e, p 609]
d. Breast feeding is safe for the infant of a mother 161. Platelet aggregation is inhibited by : (JIPMER 90)
taking Warfarin a. Clofibrate
[Ref. KDT 6/e, p 598] b. Aspirin
154. Antitode for heparin is : (Kerala 95) c. Dipyridamole
a. Protamine d. All of the above
b. EDTA [Ref. KDT 6/e, p 609, 617]
c. Vit.K 162. The HMG CoA reductase inhibitor is : (TN 98)
d. Desferrioxamine a. Gemfibrozil
[Ref. KDT 6/e, p 600, 604] b. Clofibrate
155. The tissue plasminogen activator produced by c. Lovastatin
recombinant DNA technology is : (JIPMER 02)
d. None of the above
a. Anestreplase
[Ref. KDT 6/e, p 614]
b. Reteplase
163. Machanism of action of nicotinic acid are all ex-
c. Altepase
cept: (CUPGEE 01)
d. Abciximab
a. Increased catabolism of HDL
[Ref. KDT 6/e, p 606]
b. Catabolism of LDL
156. Drug of choice in Acute myocardial infarction is :
c. Catabolism WVLDL
a. Morphine (JIPMER 93)
d. Catabolism of cholesterol
b. I.V. heparin
[Ref. KDT 6/e, p 614, table (45.3)]
c. Nitroglycerine
164. Drug acting on rate - limiting step in cholesterol
d. Streptokinase synthesis is : (Jipmer 03)
[Ref. KDT 6/e, p 606 - 607; Harrison 16/e, p 1453] a. Nicotinic acid
157. Which of the following is an inhibitor of b. Simvastatin
fibrinolysis : (JIPMER 02) c. Gemfibrozil
a. Tranexemic acid d. Cholestryamine
b. Tictopidine [Ref. KDT 6/e, p 614]
c. Alteplase 165. For tratment of hypertriglyceridemia drug used
d. Urokinase inthe primary stage : (JIPMER 98)
[Ref. KDT 6/e, p 608] a. Cholestyramine
158. Antifibrinolytic agent is : (DNB 01) b. Nicotinic acid
a. Aspirin c. Gemfibrozil
b. Aminocaproic acid d. Resins
c. Ticlopidine [Ref. KDT 6/e, p 617 - 618]
d. Abciximab
[Ref. KDT 6/e, p 608; Harrison 15/e, p 761]

Answer 152. a. Polysaccharide 153. b. Low ... 154. a. Protamine 155. c. Altepase 156. d. Streptokinase
157. a. Tranexemic ... 158. b. Aminocaproic ... 159. c. Epsilon ... 160. a. Paracetamol 161. b and c
162. c. Lovastatin 163. a. Increased ... 164. b. Simvastatin 165. c. Gemfibrozil

314
THE THERAPY OF ASTHMA
Effective agents may include glucocorticoids, cholinergic agents, leukotriene-receptor antagonists, and adrenergic receptor
agonists. Methylxanthines, such as theophylline, are also useful.

Drugs for obstructive airway diseases: asthma/COPD


Adrenergics, inhalants
Short acting â2-agonists
• Salbutamol/Levosalbutamol • Fenoterol
• Terbutaline • Pirbuterol
• Procaterol • Bitolterol
• Rimiterol • Carbuterol
• Tulobuterol • Reproterol

Long acting â2-agonists (LABA)


• Arformoterol • Bambuterol
• Clenbuterol • Formoterol
• Salmeterol

Ultra LABA

• Indacaterol

Other:
• Epinephrine • Hexoprenaline
• Isoprenaline (Isoproterenol) • Orciprenaline (Metaproterenol)

Glucocorticoids
• Beclometasone • Budesonide
• Ciclesonide • Fluticasone
• Mometasone

Anticholinergics/ muscarinic antagonist


• Ipratropium • Tiotropium

Mast cell stabilizers


• Cromoglicate • Nedocromil

Xanthines
• Aminophylline • Doxofylline
• Theobromine • Theophylline
5 Self Assessment & Review Phar macology
Pharmacology
Leukotriene antagonists
• Montelukast • Pranlukast
• Zafirlukast

Lipoxygenase inhibitor
• Zileuton

Thromboxane receptor antagonists


• Ramatroban • Seratrodast

Combination products
• Budesonide/formoterol • Fluticasone/salmeterol
• Ipratropium/salbutamol

Mechanism of action of drugs used in asthma

Bronchodilation is promoted by cAMP. Intracellular levels


of cAMP can be increased by β-adrenoceptor agonists,
which increase the rate of its synthesis by adenylyl
cayclase (AC); or by phosphodiesterase (PDE) inhibitors
such as theophyline, which slow the rate of its degradation.
Bronchoconstriction can be inhibited by muscarinic
antagonists and possible by adenosine antagonists.

Role of corticosteroid s in the therapy of asthma


• Corticosteroids can decrease inflammatory responses in the lung by several mechanisms. These include the inhibition
of synthesis of inflammatory prostanoids, potentiating of â2-adrenergic receptors, and the inhibition of histamine release
through stabilization of mast cells.
• Patients who require inhaled beta2 adrenergic agonists 4 or more times/week are the candidates for inhaled
glucocorticoids.
• Agents available are : Beclomethasone, triamcinolone, flunisolide, budesonide, ciclesonide and fluticasone.
• The main caution in switching patients from oral to inhaled steroids is to taper oral therapy slowly to avoid precipitation n
of adrenal insufficiency.

Role of inhaled adrenergic agonists in asthma treatment


• Although human bronchial smooth muscle receives little or no sympathetic innervation, it nevertheless contains large
number of beta2 adrenergic receptors.
• The stimulation of these receptors activates cAMP pathway thereby, relaxing airway smooth muscle, inhibiting mediator
release and causing tachycardia and skeletal muscle tremor as toxic effects.
• Long-term treatment with these agents leads to receptor desensitization and a diminution of effect. The beta2 receptors
on bronchial smooth muscle are relatively resistant to desensitization, while receptors on mast cells and lymphocytes
are desensitized rapidly.

Advantage of inhaled salmeterol therapy over therapy with other agents


• Salmeterol and formoterol are potent selective beta2 agonists that achieve their long duration of action (12 hours or
more) as a result of high lipid solubility, which permits them to dissolve in the smooth muscle cell membrane. The local
drug, then functions as a slow release depot that provides drug to adjacent beta receptors over a long period.
• In addition, because of their long duration of action, these drugs are useful in preventing late phase bronchoconstriction.
• The current management guidelines for asthma recommend that long-acting beta2 adrenergic agonists be added if
symptoms persist in patients on low or medium doses of inhaled steroids.

318
Phar macology of Respiratory
Pharmacology Respiratory,, Endocrinal & Gastr ointestinal System
Gastrointestinal 5
Role of oral therapy with beta adrenergic receptors agonists
Oral therapy is used in 2 primary situations:
1. In young children (<5yrs old), who cannot manipulate metered dose inhalers and yet have occasional wheezing and viral
upper respiratory infections. In these cases, brief courses of oral therapy (albuterol or metaproterenol syrups are used).
2. In some patients with severe asthma exacerbations, any aerosol, whether delivered via a metered-dose inhaler or a
nebuliser, can worsen cough and bronchospasm owing to local irritation.
The frequencies of systemic side effects are with oral agents are higher in adults than in children.

Mechanism of action of montelukast and zafirlukast


• Montelukast inhibits the actions of leukotriene D4 (LTD4) at the CysLT1 receptor. This results in a decrease in sensitivity to
antigen challenge, and decreased bronchoconstriction.
Monoclonal antibody, Omalizumab as a therapy for asthma
• Omalizumab is a recombinant humanized monoclonal antibody targeted against IgE. IgE bound to omalizumab but
cannot bind to IgE receptors on mast cells and basophils, thereby preventing allergic reaction at a very early step in the
process.
• It is indicated for adults and adolescents older than 12 years of age with allergies and moderate-to-severe asthma.

DRUGS USED IN THE THERAPY OF DIABETES MELLITUS


Type 1 diabetes is characterized by lack of insulin. Thus, the therapy for this condition is daily insulin therapy.
Insulin is typically complexed with zinc and/or protamine, a charged protein. This complexing allows the hormone to be
slowly dissociated from the complex, making the preparation longer-acting.
U-100 insulin is normal, uncomplexed insulin. It is termed U-100 because it contains 100 units of insulin per milliliter.
The plasma half-life of U-100 insulin is approx. 9 minutes.

Short-acting insulins:
• It should be injected 30-45 minutes before meal. It is usually given subcutaneously in combination with an intermediate
or long acting preparation.
• The IV route is preferred only in ketoacidosis or when the requirement of insulin changes rapidly, such as during
perioperative period, during labour and delivery and in ICU situations.
• It is the only form of insulin that can be used in subcutaneous infusion pumps.
• Insulin lispro (lysine occurs at B28 and proline occurs at B29) and insulin aspart (replacement of proline at B28 with
aspartic acid) exists as hexamers in commercial formulations like regular insulin; however, unlike regular insulin, lispro
and aspart dissociates into monomers instantaneously following injection. This property results in rapid absorption and
shorter duration of action than regular insulin. Injection of these analogs 15 minutes before a meal affords glycemic
control similar to that of regular insulin given 30 minutes before a meal.
• Therapeutic advantages of lispro and aspart insulins over regular insulins:
1. The prevalence of hypoglycaemia is reduced by 20 to 30%.
2. The glucose control, as assessed by HbA1C is modestly but significantly improved (0.3 to 0.5%).
• Insulin glulysine (glutamic acid replaces lysine at B29 and lysine replaces asparagines at B23) is also a rapid acting
analog with similar features as those of lispro and aspart.

Intermediate acting insulins:


• NPH (isophane insulin suspension) and lente insulin (insulin zinc suspension) are 2 commonly used ones.
• It is usually given either once a day before breakfast or twice a day.
• NPH and regular insulin can be easily mixed together while mixing of lente with regular insulin leads to formation of
complex between regular insulin and protamine or zinc.

Long acting insulins:


• Ultralente, PZI and insulin glargine are the agents available as long-acting insulins.
• It has a slower onset and a prolonged peak of action (contrast glargine, which has a sustained peakless absorption
profile).
• Insulin glargine : It is the 1st long acting analog of human insulin approved for use in USA. It is a clear solution at pH 4.0.
This pH stabilizes the insulin hexamer and results in a prolonged and predictable absorption from subcutaneous tissue.
It cannot be mixed with any other insulin due to low pH. The glucose-lowering effects of this form do not peak, thus
mimicking the patient’s basal secretion of insulin. It provides a better once daily control than ultralente or PZI. It also does
not accumulate after repeated injections

319
5 Self Assessment & Review Phar macology
Pharmacology
Insulin detemir (rDNA origin) Antidiabetic, long-acting insulin analogue
Indicated for once- or twice-daily SC administration in treatment of adult patients with type 1 or type 2 diabetes mellitus who
require basal insulin for control of hyperglycemia.

Common multidose insulin regimens. A. Typical split-mixed regimen consisting of twice-daily injections of a mixture of regular (regular/lispro/
aspart) and intermediate-acting (NPH or lente) insulin. B. A variation in which the evening dose of intermediate-acting insulin is delayed until
bedtime to increase the amount of insulin available the next morning. C. A regimen that incorporates ultralente or glargine insulin. D. A variation
that includes premeal short-acting insulin with intermediate-acting insulin at breakfast and bedtime. E. Patterns of insulin administration with
a regimen of continuous subcutaneous insulin infusion.

Adverse effects of insulin


The major adverse effect is, predictably, hypoglycemia. In addition, patients may have food allergies that can cause reactions
to insulins of animal origin (i.e., an allergy to pork may limit the use of porcine insulin, etc.). Patients may also develop
abnormalities in fat deposition with repeated injections at the same site (lipodystrophy),

Result of administering propranolol with insulin


Beta-receptor antagonists mask the symptoms of hypoglycemia (e.g., tachycardia, diaphoresis), and thus mask the symptoms
of insulin overdose.

320
Phar macology of Respiratory
Pharmacology Respiratory,, Endocrinal & Gastr ointestinal System
Gastrointestinal 5
TYPE II (ADULT ONSET) DIABETES
In type II (adult onset, non-insulin-dependent diabetes mellitus [NIDDM]), the problem lies at the level of the receptor rather
than the islet cell. Typically, these patients produce insulin, but the receptors are either desensitized or are associated with
a defective transduction.

Pharmacologic therapies for NIDDM : Pharmacologic therapies include drugs that elicit an increase in insulin secretion in
order to maximally stimulate desensitized receptors (e.g., sulfonylureas), and drugs that alter glucose utilization or production
(e.g., biguanides, troglitazone), or alter the breakdown of carbohydrates (e.g., acarbose).

Administration of insulin useful in the control of blood glucose in NIDDM : This depends on the etiology. If the condition is
caused by overstimulation of receptors, insulin administration can be useful. If there were a genetic defect in receptor
transduction, administration of insulin would not produce benefit in control of blood glucose.

Mechanism of action of sulfonylureas : These agents are believed to trigger the release of insulin by the blockade of ATP-
sensitive potassium-channel receptors on the pancreatic cell. This blockade results in a decrease in potassium conductance
and subsequent depolarization of the cell membrane, stimulating calcium ion influx and release of insulin.

Drug Duration of Action Half-life Production of Metabolites


Tolbutamide Short (6-8h) <1h No
Acetohexamide Intermediate (12-18 h) 1.5 h Yes
Glyburide Intermediate (10-16 h) 10 h No
Glipizide Intermediate (24 h) 4h No
Chlorpropamide Long (> 40 h} 36 h No
Tolazamide Long (> 20 h) 7h Yes

Second-generation sulfonylureas (Glyburide and glipizide) : The first generation drugs are significantly decreased in
potency, as compared to second generation drugs. In addition, plasma protein-binding is much greater with first-generation
drugs, so drug interactions are more likely to occur.

Action of phenylbutazone on the actions of sulfonylureas : Phenylbutazone increases the potency of sulfonylureas as it
displaces these drugs from plasma protein-binding sites. Thus, lower dosage is required.

Sulfonylureas on the heart :


• These drugs affect potassium conductance, not only in islet cells, but also in coronary arteries and the myocardium.
Therefore, they can exacerbate ischemic damage in the compromised heart as a result of a reduced ability to relax
cardiac and vascular smooth muscle.
• Glimepiride, the most recent sulphonylurea, exerts beneficial effects with regards to ischemic preconditioning as compared
with glyburide. The physiologic response to an ischemic event in the coronary vasculature is a reflex vasodilatation to a
subsequent ischemic episode. This reflex is preserved with glimepiride but reduced with glyburide.

Mechanism of action of biguanide antidiabetic agents : Biguanides (e.g., metformin, phenformin) enhance the metabolism
of glucose by peripheral tissues. In addition, these drugs can inhibit hepatic gluconeogenesis and increase the rate of
glycolysis.

Compare the hypoglycemic effects of biguanides and sulfonylureas : Sulfonylureas can promote hypoglycemia, because
they increase the production of insulin. Biguanides do not affect insulin levels, but simply promote glucose utilization by
tissues. They therefore do not promote hypoglycemia.
Acarbose is an alpha-glucosidase inhibitor that slows the breakdown of complex carbohydrates in the intestine. Thus,
increases in postprandial serum glucose are inhibited, resulting in both a decrease in insulin requirement and a decrease
in insulin receptor stimulation.

Action of thiazolidinediones (Rosiglitazone, pioglitazone)


• It is a selective agonist for nuclear peroxisome proliferator-activated receptor (PPAR-gamma) which activates insulin
responsive genes that regulate carbohydrate and lipid metabolism.
• It requires insulin to be present for its action.
• This drug lowers insulin resistance in muscle and adipose tissue, reducing the need for insulin.
• As it acts at the nuclear level, it may take upto 1 to 3 months to be effective.

321
5 Self Assessment & Review Phar macology
Pharmacology
Adverse effects of troglitazone
Adverse effects include weight gain, anemia, fluid retention, and rare idiosyncratic hepatic necrosis. Troglitazone has
been withdrawn from use due to its association with severe hepatic toxicity. Monthly assessment of liver function is required
with the use of other 2 agents of this class.

Role of GIP and GLP-1 in the management of diabetes:


• Glucose dependent insulinotropic peptide (GIP) and glucagon like peptide (GLP-1), also termed as incretin hormones,
are released from the upper and lower bowel that augments glucose dependent insulin secretion.
• GLP-1 has a much larger effect in this regard. GLP-1 also reduces glucagons secretion, slows gastric emptying and
decreases appetite. However, the disadvantage is that circulating GLP- is rapidly (1-2 min) inactivated by the dipeptidyl
peptidase IV enzyme (DPP-IV).
• Exenatide (exendin-4) has been developed has been derived from salivary gland of Gila monster and has 53% homology
with human GLP-1. It is resistant to DPP-IV and has full agonistic activity at GLP-1 receptors. It lowers HbA1C by 1-1.3%
and also promotes weight loss in type 2 DM.
Exenatide As adjunctive therapy to improve glycemic control in patients with type 2 diabetes mellitus who are taking metformin,
a sulfonylurea, or a combination of metformin and a sulfonylurea but have not achieved adequate glycemic control.
• FDA has recently approved exenatide as twice daily injections in combination therapy with other agents in subjects with
type 2 DM.
• A recent approach is to inactivate the DPP-IV protease, thereby increasing endogenous circulating GLP-1 levels. A
number of DPP-IV inhibitors have entered clinical trials. The major concern with these agents is that DPP-IV can metabolise
a wide range of peptides, it can theoretically have effects on many other body systems.

STEROIDS AND IMMUNOSUPPRESSANT AGENTS


Glucocorticoids interfere with macrophage action, decrease capillary permeability, and decrease the activity of local mediators.
Action of glucocorticoids on lysosomes
• Glucocorticoids stabilize lysosomal membranes, resulting in an inhibition of lysosomal action and, thus, an inhibition of
phagocytic activity.
• Glucocorticoids inhibit the release of leukocytic acid hydrolases, and also inhibit the adhesion of leukocytes to the
vascular endothelium.
Actions of corticosteroids on the immune system
• Corticosteroids stabilize lymphocytic membranes, reducing the production of immunoglobulins and their complement.
Lymphocyte production is also inhibited, producing lymphocytopenia.
• Exogenously administered glucocorticoids produce a negative feedback inhibition on the production of adrenocorticotroplc
hormone (ACTH) from the anterior pituitary. This results in a decrease in stimulation of the adrenal cortex and a decrease
in the production of cortisol and aldosterone.
• Prolonged therapy, in particular, will cause adrenal atrophy. The degree of atrophy is dose-dependent, and, therefore,
administration of progressively smaller doses over time will allow the adrenal cortex to regenerate, while supplementing
endogenous cortisol levels.

Major actions of glucocorticoids

Gene mediated cellular actions of glucocorticoids


Mechanism Action
• Translocation of glucose transporters from • ↓ glucose uptake & utilization in peripheral
plasma membrane to deeper sites. tissues.
• Induction of hepatic gluconeogenetic enzymes. • ↑ production of glucose from amino acids.
• Induction of hepatic glycogen synthase. • Deposition of glycogen in hepatocytes.
• Site specific changes in sensitivity of adipocytes • Altered distribution of body
to GH, Adr, insulin.
• ↑ expression of vascular adrenergic & AT1 receptor • Enhanced reactivity to vasopressor substances.
• ↓ expression of POMC gene in pituitary corticotropes • ↓ production of ACTH

322
Phar macology of Respiratory
Pharmacology Respiratory,, Endocrinal & Gastr ointestinal System
Gastrointestinal 5
Antiinflammatory & Immunosuppressant actions
• Induction of lipocortins in macrophages, • Lipocortins inhibit phospholipase A2 → decreased
endothelium & fibroblasts production of PGs, LTs & PAF.
• Negative regulation of COX-2 • ↓ inducible PG production
• Negative regulation of genes for cytokines in • ↓ production of IL-1, IL-2, IL-3, IL-6, TNFα, GM-CSF,
γ interferon fibroblast proliferation & T-lymphocyte
function are suppressed, chemotaxis interfered.
• ↓ production of acute phase reactants from • Complement function is interfered.
macrophages & endothelial cells
• ↓ production of ELAM-1 & ICAM-1 in endothelial cells. • Adhesion & localization of leukocytes is interfered.
• ↓ production of collagenase & stromolysin • Prevention of tissue destruction

Short-acting corticosteroids are: Hydrocortisone and prednisolone

Adverse effects of glucocorticoid therapy


In addition to the suppression of the hypophyseal-pituitary axis and adrenal atrophy, these drugs can cause a variety of
adverse effects, including osteoporosis, pancreatitis, steroid-induced diabetes mellitus, cataracts, glaucoma, psychosis/
oral candidiasis and other opportunistic infections, immunosuppression, infertility, weight gain, and skin atrophy. Severe
edema may also be produced, particularly in the face, depending on the degree of mineralocorticoid activity.
• Dexamethasone crosses easily into the central nervous system (CNS), in contrast to most other steroids. In addition, it
is a potent and long-acting corticosteroid.
Glucocorticoid antagonists : Metyrosine, Aminoglutethimide, Ketoconazole, Mifepristone, Mitotaine, Trilostaine
Mineralocorticoid antagonists : Spironolactone, Eplerenone, Drospirenone

DRUGS THAT AFFECT THE THYROID GLAND


Increased metabolism, increased heat production, weight loss, protrusion of the eyes (exophthalmia), and an increase in
bone and muscle turnover are the major symptoms. In addition, increased levels of thyroid hormone have detrimental effects
on cardiac muscle and cardiac conduction, as well as on vascular smooth muscle function.
Thyroid hormone L-triiodothyronine (T3) sensitizes the myocardium to the effects of catecholamines. This can facilitate the
production of cardiac arrhythmias.
Sodium iodide inhibits the proteolysis of thyroglobulin stored in the thyroid gland.
Iodide decreases the release of thyroid hormone and also decreases the synthesis of iodinated tyrosine and thyronine
residues.
Iodine is primarily used to induce a euthyroid state before surgery, it increases vascularity and increase the density of the
thyroid gland, which decreases postoperative complications.

Iodism
This condition presents as a consequence of iodide or iodine administration. Its symptoms resemble a low-level upper
respiratory infection—burning of the mouth and throat, swollen sub-maxillary glands, and painful teeth and gums. A metallic
taste may also be present, caused by the metallic nature of iodine.

Therapeutic properties of methimazole


Methimazole acts as a substrate for thyroid peroxidase, an enzyme that acts as a catalyst in the iodination of thyroglobulin.
Thus, there is a dose-dependent competition between the drug and thyroglobulin for iodination. The iodination of thyroglobulin
is thus decreased, resulting in a reduced production of thyroid hormone.

Mechanism of action of propothiouracil (PTU)


Like methimazole, PTU acts as a substrate for thyroid peroxidase, and thus inhibits the iodination of thyroglobulin. In
addition, the drug inhibits the conversion of T3 to T4 in the periphery.
Propothiouracil has a shorter duration of action and has significantly less transfer across the placenta and into breast milk
than methimazole.

PTU is a safer drug for administration in pregnancy


The drug is highly protein-bound and is also significantly ionized at physiologic pH. Thus, the drug does not cross the fetal-
placental barrier well.

323
5 Self Assessment & Review Phar macology
Pharmacology
DRUGS FOR TREATMENT OF OSTEOPOROSIS
• The drugs for this condition act either by a) decreasing the rate of bone resorption and slowing the rate of bone loss
(antiresorptive therapy) and b) by promoting bone formation (anabolic therapy)
• The antiresorptive therapy includes biphosphonates, thiazide diuretics, calcium, vitamin D, estrogens, selective estrogen
receptor modulators and calcitonin whereas anabolic therapy includes teriparatide (a biologically active PTH fragment
PTH1-34).

DRUGS THAT AFFECT GASTRIC ACID SECRETION

Physiological and pharmacological regulation of gastric secretion: the basis for therapy of acid-peptic disorders. Shown are the interactions
among an enterochromaffin-like (ECL) cell that secretes histamine, a parietal cell that secretes acid, and a superficial epithelial cell that
secretes cytoprotective mucus and bicarbonate. Physilogical pathways, shown in solid black, may be stimulatory (+) or inhibitory (–). 1 and
3 indicate possible inputs form postganglionic cholinergic fibers, while 2 shows neutral input form the vagus nerve. Physiological agonists
and their respective membrane receptors include: acetylcholine (Ach), muscarinic (M), and nicotinic (N) receptors; gastrin, cholecystokinin
receptor 2 (CCK2); histamine (HIST), H2 receptor; and prostaglandin E2 (PGE2), EP3 receptor. Drug actions are indicated by dashed lines. A blue
X indicates targets of pharmacological antagonism. A light blue dashed arrow indicates a drug action that mimics or enhances a physiological
pathway. Shown in blue are drugs used to treat acid-peptic disorders. NSAIDs are nonsteroidal anti-inflammatory drugs and are ulcerogenic.

PROTON-PUMP INHIBITORS
Mechanism of action of omeprazole
• Omeprazole binds irreversibly to the H+/K+ adenosine triphosphatase (ATPase) (proton pump) on the secretory surface
of the parietal cell membrane. The secretion of hydrogen ions into the gastric lumen is thus inhibited in a dose-dependent
manner, decreasing the production of gastric acid.

324
Phar macology of Respiratory
Pharmacology Respiratory,, Endocrinal & Gastr ointestinal System
Gastrointestinal 5
• Omeprazole blocks basal and stimulus-induced acid secretion (e.g., stress-related).
• It is a prodrug that requires activation in an acid environment. After absorption, the prodrug diffuses into the parietal cells
of the stomach and accumulates in the acidic secretory canaliculi. Here, it is activated by proton catalysed formation of
tetracyclic sulphenamide.trapping the drug so that it cannot diffuse back across the canalicular membrane. The activated
form then binds covalently with sulfhydryl groups in H+K+ATPase, irreversibly inactivating the proton pump.
• The acid secretion resumes only after new pump molecules are synthesized and inserted into luminal membrane,
providing a prolonged (upto 24 hours) suppression of acid secretion, despite the much shorter plasma half lives (0.5-2
hours) of the parent comounds.
• Omeprazole inhibits the actions of cytochrome P450, which may result in drug interactions.
• Systemic alkalosis caused by an excessive decrease in gastric acid production is unlikely, because the drug must be
activated by an acid environment within the parietal ceil. Thus, the conversion of the drug to the active form is related to the
amount of carbonic acid available

HISTAMINE-RECEPTOR ANTAGONISTS

Actions of H2 antagonists : These drugs inhibit the release of gastric acid via antagonism of the H2 receptor.

Efficacy of H2 antagonists is less than that of proton-pump inhibitors :


• Multiple stimuli for the release of gastric acid exist, so blockade of the histamine receptor only decreases secretion to the
extent that other stimuli are present or absent. Proton-pump inhibitors act at the last step of gastric acid synthesis and
thus are not subject to the effects of other mediators.
• These agents predominantly inhibit basal acid secretion, which accounts for their efficacy in suppressing nocturnal acid
secretion. Because the most important determinant of duodenal ulcer healing is the level of nocturnal acidity, evening
dosing of these agents is an adequate therapy in most cases.
• Famotidine has been shown to be as much as 100 times as potent as cimetidine. Ranitidine is somewhat less potent
than famotidine but is more potent than cimetidine
• Cimetidine inhibits the actions of cytochrome P450. Cimetidine is the most potent of the three. Ranitidine has substantially
less inhibitory action, and famotidine has negligible effect on the enzyme.
• Cimetidine has weak antiandrogenic activity, which can affect the male reproductive system with frequent or prolonged
use.
Famotidine be especially useful in ulcer prophylaxis. This drug appears to have cytoprotective properties. Famotidine is
primarily eliminated by the kidney, so the elimination half-life is significantly prolonged with renal impairment.

ANTACIDS
Calcium carbonate, magnesium carbonate, and sodium bicarbonate are commonly used. Aluminum hydroxide is still found
in some preparations, but has become less popular, as a result of the possible role of aluminum in the etiology of Alzheimer’s
disease.

“Calcium rebound” with respect to antacids : Recall that calcium is a factor in the stimulation of the release of gastric acid.
Thus, a calcium-containing antacid will first neutralize stomach acid, bringing the pH up, but the calcium may then produce
a “rebound” effect by stimulating release of additional acid.

Advantage of using magnesium-containing antacids : These compounds are not absorbed well, as compared to other
antacids, making them longer-lasting and decreasing the possibility of alkalosis.

CYTOPROTECTIVE AND HORMONAL AGENTS


Misoprostol is a synthetic, oral prostaglandin E analog, which acts as a cytoprotective agent for the gastric epithelium.

Actions of misoprostol
• This drug exhibits a cytoprotective effect, increases gastric mucosal secretions, and may also decrease the time of
gastric emptying. In addition, it may also decrease-the secretion of gastric acid and pepsin, in a dose-dependent
manner.
• The drug is very effective as a prophylactic, but is also very useful in protecting an existing ulcerated epithelium from
further damage, due to its mucus-producing effects
• Misoprostol contracts smooth muscle, and thus causes uterine contractions. Its secondary use is actually as an
abortifacient

325
5 Self Assessment & Review Phar macology
Pharmacology
• Sucralfate is an adhesive carbohydrate polymer that adheres to damaged gastric endothelium and affords protection
from further damage, allowing the tissue to heal.

Bismuth subsalicylate (“Pepto Bismol”) in the prophylaxis and therapy of ulcers


• This drug coats the gastric epithelium, decreasing acid damage and irritation. It is also protective against ulcer formation,
in combination with a drug such as metronidazole.
• Octreotide inhibits gastrin secretion, whereas drugs such as cimetidine and omeprazole increase it. Octreotide is also
extremely costly, making combination therapy with these agents of maximum benefit.

326
Phar macology of Respiratory
Pharmacology Respiratory,, Endocrinal & Gastr ointestinal System
Gastrointestinal 5
ALL INDIA 9. Which enzyme is inhibited by Aminophylline?
a. Monoamine oxidase [AI 06; AIIMS May 04]
1. A child was given leukotriene receptor antagonist b. Alcohol dehydrogenase
for prophylaxis of asthma. The drug is: [AI 07] c. Phosphodiesterase
a. Montelukast d. Cytochrome P - 450
b. Zileuton 10. Bisphosphonates acts by: [AI 06]
c. Nedocromil a. Increasing the osteoid formation
d. Verapamil b. Increasing the mineralisation of osteoid
2. The leukotriene receptor antagonist used in bron- c. Decreasing the osteoclast mediated resorption
chial asthma is: [Al 07] of bone
a. Zafirlukast d. Decreasing the parathyroid hormone secretion
b. Zileuton 11. All of the following appear to decrease hot flushes
c. Ketotifen in menopausal women except: [AI 05]
d. Nedocromil a. Androgens
3. All are short and rapid acting insulins except: b. Raloxifene
a. Lispro [Al 07] c. Isoflavones
b. Aspart d. Tibolone
c. Glargine 12. Which one of the following drugs has been shown
to offer protection from gastric aspiration syn-
d. Glulisine
drome in a patient with symptoms of reflux ?
4. Finasteride is: [Al 07]
a. Ondansetron [AI 05]
a. 5 alpha reductase inhibitor
b. Metoclopramide
b. PDE inhibitor
c. Sodium citrate
c. Alpha 1a blocker
d. Atropine
d. Androgen receptor blocker
13. With which of the following theophylline has an
5. The following drugs are used in obesity except: antagonistic interaction ? [AI 05]
a. Orlistat [Al 07] a. Histamine receptors
b. Sibutaramine
b. Bradykinin receptors
c. Olestra
c. Adenosine receptors
d. Neuropeptide Y analogues
d. Imidazoline receptors
6. Which one of the following is true about heparin?
14. The following statements regarding finasteride are
a. Hyperkalemia [Al 07] true except: [AI 05]
b. Does not cause alopecia a. It is used in the medical treatment of benign pro-
c. Teratogenic static hypertrophy (BPH)
d. Does not cause osteoporosis b. Impotence is well documented after its use
7. All are anti H. pylori drugs except: [Al 07] c. It blocks the conversion of dihydrotestosterone
a. Oxytetracycline to testosterone
b. Amoxicillin d. It is a 5- α reductase inhibitor
c. Bismuth Subcitrate 15. All of the following are hormonal agents used
d. Omeprazole against breast cancer except: [AI 04]
8. Uses of octreotide are all except: [Al 07] a. Letrozole
a. Acromegaly b. Exemestane
b. Is used orally c. Taxol
c. Given for secretory diarrhea d. Tamoxifen
d. Used in portal hypertension

Answer 1. a. Monteleukast 2. a. Zafirleukast 3. c. Glargine 4. a. 5 alpha ... 5. d. Neuropeptide ...


6. a. Hyperkalemia 7. a. Oxytetracycline 8. c. Given for ... 9. c. Phosphodiest ....10. c. Decreasing ...
11. b. Raloxifene 12. b. Metoclo ... 13. c. Adenosine ... 14. c. It blocks ... 15. c. Taxol

327
5 Self Assessment & Review Phar macology
Pharmacology
16. A patient of peptic ulcer was prescribed Ranitidine a. After meals
and Sucralfate in the morning hours. Why is this b. Shortly before meals
combination incorrect: [AI 04] c. Along with H2 blockers
a. Ranitidine combines with sucralfate and pre- d. During prolonged fasting periods
vents its action
23. In a patient taking oral contraceptive, the chance
b. Combination of these two drugs produces seri- of pregnancy increases after taking any of the fol-
ous side effects like agranulocytosis lowing drugs except : [AI 02; AIIMS Feb. 97,
c. Ranitidine decreases the gastric pH so a. Phenytoin Dec. 94]
sucralfate is not able to act
b. Griseofulvin
d. Sucralfate inhibits absorption of ranitidine
c. Ampicillin
17. All of the following statements about an alpha glu-
d. Cimetidine
cosidase inhibitor are true except: [AI 04]
24. All the following belong to the steroid receptor
a. Reduces intestinal absorption of carbohydrates
superfamily except: [AI 02]
b. Effective in both type 1 and 2 diabetes
a. Vitamin D3 receptor
c. Hypoglycemia is a common and serious side
b. Thyroid receptor
effect
c. Retinoid receptor
d. Can be used with other oral Hypoglycemic
Agents d. Epinephrine receptor
18. All of the following are advantages of using 25. All of the following are correct about steroids ex-
Raloxifene over estrogen in post menopausal cept: [AI 01]
women except ? [AI 04] a. Inhibits the release of arachidonic acid from ves-
a. Reduces fracture rates sel wall through action of phospholipase A2
b. Avoids endometrial hyperplasia b. Bind plasma membrane receptors and follow-
ing internalization influence nuclear changes
c. Reduces incidence of venous thrombosis
c. Inhibit vascular membrane permeability
d. No increase in incidence of breast carcinoma
d. Increase glucose synthesis, glycogen deposi-
19. Which of the following statements about
tion in liver
biguanides is not true ? [AI 04]
26. All of the following statements are true about theo-
a. Don’t stimulate insulin release
phylline except:
b. Decrease hepatic glucose production
a. Increase in dose is required in cardiopulmonary
c. Renal dysfunction is not a contraindication for disease [AI 01; AIIMS Sep. 96,
their use
b. Increases cAMP Nov. 00, June 00]
d. Can be combined with sulfonylureas
c. Increase in dose is required in smokers
20. Which of the following is present intracellularly in
d. Inhibits phosphodiesterase
muscle cells: [AI 03]
27. A patient is taking 40mg Famotidine OD, Sucralfate
a. Insulin
and antacid tablets TDS. This treatment is irratio-
b. Corticosteroid nal, because of: [AI 00]
c. Epinephrine a. Sucralfate decreases the absorption of
d. Glucagon famotidine
21. A highway truck driver has profuse rhinorrhea and b. Sucralfate increases the toxicity of famotidine
sneezing. Which amongst the following drugs c. Sucralfate decreases absorption of antacids
would you prescribe him ? [AI 03]
d. Sucralfate polymerises only when gastric pH is
a. Pheniramine less than 4
b. Promethazine 28. Drug causing Hirsutism and Gynaecomastia:
c. Dimenhydrinate a. Spironolactone [AI 00; AIIMS Jun 98, May 07]
d. Cetrizine b. Rifampicin
22. Proton pumps inhibitors are most effective when c. Penicillin
they are given: [AI 02]
d. Bumetanide
Answer 16. d. Sucralfate ... 17. c. Hypoglycemia ... 18. c. Reduces ... 19. c. Renal ... 20. b. Corticos ...
21. d. Cetrizine 22. b. Shortly ... 23. d. Cimetidine 24. d. Epinephrine ... 25. b. Bind ...
26. a. Increase ... 27. d. Sucralfate ... 28. a. Spironolactone 29. a. Calcium-...

328
Phar macology of Respiratory
Pharmacology Respiratory,, Endocrinal & Gastr ointestinal System
Gastrointestinal 5
29. Milk-Alkali syndrome may be caused by ingestion 37. Which of the following steroids can be adminis-
of: [AI 99] tered by inhalation: [Al 95]
a. Calcium-carbonate a. Beclomethasone
b. Magnesium sulphate b. Betamethasone
c. Aluminium trisilicate c. Prednisolone
d. Aluminium hydroxide d. Hydrocortisone
30. Which of the following drug acts as a HMG-CoA 38. All the following are true of cholestyramine ex-
reductase inhibitor: [AI 98] cept: [Al 95]
a. Gemfibrozil a. Are basic ion exchange resins
b. Clofibrate b. Cause compensatory increase in HMG CoA re-
c. Lovastatin ductase activity
d. Probucol c. May cause constipation, steatorrhea
31. Which of the following has least glucocorticoid d. Patient acceptability is good
activity: [AI 98; 96]
a. Fludrocortisone AIIMS
b. Dexamethasone
c. Triamcinolone 39. Which teratogen causes deafness?
d. Betamethasone a. Valproate [AIIMS May 08]
32. True statement about Omeprazole is: [AI 97] b. Chloroquine
a. It may cause leiomyosarcoma c. Alcohol
b. It is a nitrocellulose d. Warfarin
c. May induce carcinoid tumors in rats 40. Most potent anti-emetic is: [AIIMS Nov. 07]
d. It is more frequently used by the I.V. route than a. Ondansertron
orally b. Granisertron
33. Prolonged use of steroids may cause: [AI 97] c. Dolasetron
a. Decrease in bone matrix protein d. Palonosetron
b. Hypoglycemia 41. All of the following are the side effects of Clomi-
c. Hypotension phene citrate except: [AIIMS Nov. 07]
d. Early healing of wound a. Polycystic ovary
34. All are examples of gastro-kinetic drugs except: b. Multiple pregnancies
a. Cisapride [Al 96] c. Ovarian cancer
b. Domperidone d. Teratogenicity
c. Erythromycin 42. Uterine relaxant with least side effects:
d. None a. Ritodrine [AIIMS Nov. 07]
35. Estrogen acts on: [Al 95] b. Nifedipine
a. Cellular membrane receptors c. Progesterone
b. Cytoplasmic receptors d. Magnesium sulphate
c. Nuclear receptors 43. Drug causing hyperglycemia are all except:
d. Mitochondria a. Corticosteroids [AIIMS Nov. 07]
36. Which drug causes osteoporosis on long-term b. Frusemide
use: [Al 95] c. Indomethacin
a. Estrogen d. CCB
b. Progesterone
c. GnRH analogues
d. Warfarin

Answer 30. c. Lovastatin 31. c. Triamcinolone 32. c. May ... 33. a. Decrease ... 34. c. Erythromycin
35. c. Nuclear ... 36. c. GnRH ... 37. a. Beclome ... 38. d. Patient ... 39. d. warfarin
40. d. Palonosetron 41. d. Teratogenicity 42. b. Nifedipine 43. c. Indomethicin

329
5 Self Assessment & Review Phar macology
Pharmacology
44. Bisphosphonates are not used in: 51. The loading dose of Aminophylline is:
a. Hypercalcemia [AIIMS Nov. 07] a. 50-75 mg/kg [AlIMS May 06]
b. Osteoporosis b. 0.5-1.0 mg/kg
c. Cancer c. 2.0-3.5 mg/kg
d. Vitamin D intoxication d. 5-6 mg/kg
45. All are true about Clofibrate except: 52. Which of the following drugs is known to cause
a. Drug of choice for increased TG granuloma in the liver: [Nov. 05]
b. It can cause myalgia [AIIMS Nov. 07] a. Allopurinol
c. Contraindicated in gallstones b. Nifedipine
d. More effective empty stomach c. Tetracycline
46. OC pills are contraindicated in pateints receiving: d. Methyl testosterone
a. Rifampicin [AIIMS Nov. 07] 53. Which of the following is the established biologi-
b. Ethambutol cal therapy for Crohn’s disease:
c. Streptomycin a. Anti TNF α – antibody [Nov. 05]
d. Pyrazinamide b. IL – 1 antagonist
47. A patient is posted for elective surgery. Which of c. IL – 6 antagonist
the following drugs should be stopped on the day d. IL – 8 antagonist
of surgery: [AIIMS Nov. 07] 54. Heparin is the commonly used anticoagulant in
a. Atenolol cardiac surgery. All of the following are true about
b. Amlodipine heparin except: [Nov. 05]
c. Statins a. Weakest acid found in living things
d. Metformin b. Most commercial preparation of heparin now
utilize pig intestinal slimes
48. All are antiemetic except: [AIIMS May 07]
c. Acts via antithrombin activation
a. Domperidone
d. Produce thrombocytopenia
b. Cyclizine
55. In the treatment of papillary Carcinoma thyroid,
c. Phenazocine
Radioiodine destroys the neoplastic cells predomi-
d. Ondansetron nantly by: [Nov. 05]
49. Anticoagulant effect of warfarin is increased by a. X rays
all of the following except: [AlIMS May 06]
b. β rays
a. Cimetidine
c. α rays
b. Phytonadione
d. α particles
c. Amiodarone
56. Which of the following is the drug of choice for
d. Phenylbutazone syndrome of inappropriate secretion of Antidi-
50. A patient of thromobosis of hepatic veins has been uretic Hormone [SIADH]? [May 05]
receiving coumarin therapy for duration of three a. Demeclocycline
years. Recently, she has developed bleeding ten-
b. Vasopressin
dency. How will you reverse the effect of cou-
marin? [AlIMS May 06] c. Thiazide diuretic
a. Protamine injection d. Chlorpropamide
b. Vit. K injection 57. All of the following statements about HRT [hor-
mone replacement therapy] are true except:
c. Infusion of fibrinogen
a. It increases the risk of coronary artery disease
d. Whole blood transfusion
b. It increases bone mineral density [May 05]
c. It increases the risk of breast cancer
d. It increases the risk of endometrial cancer

Answer 44. d. Vitamin D 45. d. More ... 46. a. Rifampacin 47. d. Metformin 48. c. Phenazocine
49. b. Phytonadione 50. b. Vit K ... 51. d. 5-6 mg/kg 52. a. Allopurinol 53. a. Anti TNF ...
54. a. Weakest ... 55. b. β rays 56. a. Demec ... 57. a. It increases ...

330
Phar macology of Respiratory
Pharmacology Respiratory,, Endocrinal & Gastr ointestinal System
Gastrointestinal 5
58. Aldosterone antagonists are NOT useful in the 65. Prolactin secretion will be inhibited by:
treatment of: [May 05] a. Haloperidol [Nov. 02]
a. Hypertension b. GABA (Gamma aminobutyric acid.
b. Congestive heart failure c. Neurophysin
c. Gynaecomastia d. Dopamine
d. Hirsutism 66. All are true about HMG CoA reductase inhibitors
59. Which of the following substances is the most except: [May 02]
potent androgen: [May 04] a. The CNS accumulation of simvastatin and
a. Dehydroepiandrostenedione lovastatin is high and less for Pravastatin and
b. Dihydrotestosterone Fluvastatin
c. Androstenedione b. Simvastatin is rapidly metabolised and
d. Testosterone pravastatin is least
60. Women receiving oestrogen therapy have an in- c. Bioavailability is minimally modified when
creased risk of developing all of the following can- pravastatin is taken with food
cers, except: [Nov. 04] d. Fibrinogen levels are increased by pravastatin
a. Breast cancer 67. Inhibition of 5-lipoxygenase is useful in:
b. Endometrial carcinoma a. Cardiac failure [Nov. 02]
c. Carcinoma of gall bladder b. Bronchial asthma
d. Hepatocellular carcinoma c. Hepatic failure
61. Prolactin secretion is inhibited by: d. Arthritis
a. Dopamine antagonist 68. The drug NOT used in prostatic carcinoma:
b. GABA [Nov. 03, Nov. 02, Dec. 95] a. Finasteride [May 02]
c. Neurophysin b. Diethyl stilbestrol
d. Bromocripitine c. Testosterone
62. Which one of the following is not an adverse ef- d. Flutamide
fect of salbutamol: [Nov. 03] 69. The drug NOT used in acute asthma is:
a. Tachycardia a. Salbutamol [May 02]
b. Tolerance b. Ipratropium
c. Hypokalemia c. Montelukast
d. Hypoglycemia d. Hydrocortisone
63. If a diabetic patient being treated with an oral hy- 70. All are true about Metformin EXCEPT : [May 02]
poglycemic agent, develops dilutional hyponatre- a. Causes little or no hypoglycemia in non diabetic
mia, which one of the following could be respon- subjects
sible for this effect: [Nov. 03] b. Acts by increased insulin secretion
a. Chlorpropamide
c. Increases peripheral utilisation of glucose and
b. Tolazamide decreases absorption of glucose from intestine
c. Glyburide d. When given with alcohol, increases risk of lactic
d. Glimepiride acidosis.
64. The drug of choice for the treatment of Thyrotoxi- 71. A patient presents with nephrotic syndrome and
cosis during pregnancy is: [Nov. 03] hypoalbuminemia. Protein binding of which drug
a. Carbimazole is not affected : [May 02]
b. Iodine therapy a. Tolbutamide
c. Propyl thiouracil b. Morphine
d. Metimazole c. Diazepam
d. Valproate

Answer 58. c. Gynaecomastia 59. b. Dihydrot ... 60. c. Carcinoma ... 61. d. Bromocripitine 62. d. Hypoglycemia
63. a. Chlorpropamide 64. c. Propyl ... 65. d. Dopamine 66. d. Fibrinogen ... 67. b. Bronchial ...
68. c. Testosterone 69. c. Monteleukast 70. b. Acts by ... 71. b. Morphine

331
5 Self Assessment & Review Phar macology
Pharmacology
72. Statin induced myopathy is NOT exacerbated by 80. Androgen receptor blocking drug is :
which of the following drugs : [Nov. 00] a. Tamoxifen [Feb 97]
a. Nicotinic acid b. Cyproterone acetate
b. Enalapril c. Mifepristone
c. Erythromycin d. Nalondrone
d. Clofibrate 81. All are used to treat hypercalcemia, except :
73. Zileuton is : [Nov. 00] a. D-penicillamine [Sept. 96]
a. 5 lipooxygenase inhibitor b. Corticosteroid
b. TXA2 inhibitor c. Biphosphonate
c. Leukotriene receptor antagonist d. Mithramycin
d. ACE inhibitor 82. Lactic acidosis is a side effect of : [Dec. 95]
74. Drug of choice of ulcerative colitis is : [Nov. 99] a. Phenformin
a. 5 Amino salicylic acid b. Metformin
b. Sucralfate c. Chlorpropramide
c. Metronidazole d. Glibenclamide
d. Sulfasalazine
75. All are side effects of steroids given in antiinflam- PGI
matory doses, except : [Nov. 99]
a. Diabetes mellitus 83. Long acting Insulin are : [PGI Dec. 07]
b. Hyperkalemia a. Glargine
c. Osteoporosis b. Lispro
d. Na+ and water retention c. Detemir
76. Administration of betamethasone during delivery d. Aspart
causes all, except : [June 99] e. Glulisine
a. Decrease neonatal mortality 84. Drugs causing metallic taste : [PGI Dec. 06]
b. Decrease incidence of ARDS a. Antimicrobials
c. Decrease incidence of intraventricular b. Angiotensin receptor blockers
haemorrhage c. Anticancer drugs
d. Decrease incidence of hyperbilirubinemia d. NSAID’s
77. All are true regarding oral hypoglycemic agent, e. Gold
except : [Nov. 99] 85. Glucose intolerance seen with drugs :
a. Effective only after total pancreatectomy a. Thiazide diuretics [PGI Dec. 06; June 02]
b. Metformin causes lactic acidosis b. β -blockers
c. Causes release of insulin from beta-cells c. Verapamil
d. Useful in obese maturity onset diabetics d. ACE inhibitors
78. Oral anticoagulants are stopped prior to surgery 86. Glucocorticoids act in inflammation by :
for at least : [June 98] a. ↓ Lipocortin [PGI June. 2006]
a. 1-3 days b. ↑ I L2
b. 3-5 days c. ↑ Lipocortin
c. 8-10 days d. ↑ CRP
d. 6-8 days 87. Reduced inflammation in airway produced by :
79. All are long acting bronchodilators, except : a. Fluticasone [PGI June 06]
a. Salbutamol [June 97] b. Budesonide
b. Salmaterol c. Theophylline
c. Terbutaline d. Salbutamol
d. Adrenaline e. Ipratropium
Answer 72. b. Enalapril 73. a. 5 lipooxy ... 74. a. 5 Amino ... 75. b. Hyperkalemia 76. d. Decrease ...
77. a. Effective ... 78. d. 6-8 days 79. d. Adrenaline 80. b. Cyproterone ... 81. a. D-penicillamine
82. a. Phenformin 83. a and c 84. a, c and e 85. a, b & c 86. c. ↑ Lipocortin
87. a&b

332
Phar macology of Respiratory
Pharmacology Respiratory,, Endocrinal & Gastr ointestinal System
Gastrointestinal 5
88. Drugs causing Addison’s disease are : 95. Hormone used in breast cancer are : [Dec. 02]
a. Ketoconazole [PGI June 06] a. Danazol
b. Aminoglutethimide b. Cyproterone acetate
c. Cyclosporine c. Tamoxifen
d. Glucocorticoids d. LHRH analogue
89. True about Zafirlukast is : [Dec. 05] 96. Antidiabetic used safely in renal failure :
a. It inhibit lipooxygenase pathway a. Metformin [Dec. 02]
b. It blocks LTB4 receptor b. Glimepiride
c. It is given orally c. Glibenclamide
d. It is effective in acute bronchial asthma d. Rosiglitazone
e. Efficacy is equal to glucocorticoid in decreasing e. Repaglinide
the frequency of asthmatic attack 97. Antiemetic action is through : [Dec. 02]
90. Drug used in obesity is/are : [Dec. 05] a. ↓ Chemoreceptor trigger zone (CTZ)
a. Sibutramine b. H1 agonist
b. Orlistat c. D1 antagonist
c. Insulin d. Olfactory apparatus
d. Mephentramine e. 5 HT4 agonist
e. Thyroxine 98. True regarding bromocriptine : [June 02]
91. Anti inflammatory agent in management of acute a. Natural derivative
asthma includes : [June 05] b. Synthetic derivative
a. Ipratopium bromide c. Has alfa - blocking action
b. Budesonide d. Decrease GI motility
c. Fluticasone and salmetrol combination e. Acts on both D1 and D2 receptors
d. Cromolyn sodium 99. Isotretinoin is : [June 02]
e. Salbutamol a. A vitamin A analogue
92. Antilipidemic drugs that prevent hypercholester- b. Used in cystic acne
olemia by inhibiting absorption : [Dec. 04]
c. Safe in pregnancy
a. Ezetimibe
d. Used in psoriasis
b. Orlistat
e. Bony hyperostosis is a side effect
c. Cholestyramine
100. Regarding oxytocin : [June 02]
d. Statins
a. Secreted by anterior pituitary
93. In antacid preparation aluminum hydroxide is
b. Acts on myoepithelial cells of breast
added with magnesium salt because : [Dec. 04]
c. Causes contraction of uterus during labour
a. Magnesium caused constipation
d. May cause retention of water
b To counteract constipating effect of aluminum
e. Has sympatholytic activity
d. Aluminum salt causes diarrhea
101. Androgen antagonists includes : [Dec. 01]
e. To counteract the constipating effect of magne-
sium salt a. Cyproterone
94. True about Lispro - Insulin : [Dec. 03] b. Spironolactone
a. Action is faster and short in duration than regu- c. Cimetidine
lar insulin d. Progesterone
b. It is given 15 minutes prior to meal e. Minocycline
c. Source is Lamb
d. Action is faster and long in duration than regular
insulin

Answer 88. a & b 89. b. It blocks ... 90. a & b 91. b&c 92. a&b
93. b To counteract ... 94. a & b 95. a. Danazol 96. b, d & e 97. a&e
97. b, c, d & e 99. a, b & e 100. a, c & d 101. b, c & d

333
5 Self Assessment & Review Phar macology
Pharmacology
102. Steroids in asthma : [Dec. 00] 110. Intracellular receptor activation is by :
a. Decreases sensitivity of bronchial epithelium to a. Thyrotropin [Dec. 99]
allergen b. Noradrenaline
b. Decreases inflammatory response c. Estradiol
c. Increases action of bronchodilators d. Glucagon
d. Causes potent bronchodilation 111. Glucocorticoids causes : [June 98]
103. Uses of salbutamol are/is : [Dec. 00] a. ↓ osteoid formation
a. Acute asthma b. IgF stimulation
b. Premature labour c. ↑ Ca++ absorption from stomach
c. Bronchitis d. Calcification of bone
d. Asthma prophylaxis 112. Hypothyroidism may be caused by :
104. Absolute contraindication to OC pills : a. Ethionamide [June 98]
a. Epilepsy [Dec. 00] b. Thiocetazone
b. Migraine c. Ethambutol
c. Pregnancy d. Streptomycin
d. Thromboembolism 113. True about lactulose is : [June 98]
e. Renal failure a. Blood ammonia
105. Cyt - P - 450 is inhibited by : [June 00] b. 10% absorbed from gut
a. Phenobarbitone c. Osmotic laxative
b. Cimetidine d. Disaccharide of lactose and sucrose
c. Phenytoin 114. OCP produces in liver : [June 98]
d. CCl4 a. Cholangio carcinoma
106. True of tamoxifen : [June 00] b. Adenoma
a. Binds to estrogen receptors c. Cirrhosis
b. ↑ estrogen production d. Atrophic changes
c. ↓ FSH production 115. Granulomatous hepatitis is caused by :
d. LH secretion a. Allopurinol [Dec. 97]
107. Which of (enzyme. inhibits initial step of choles- b. Methyldopa
terol synthesis: [June 00] c. Furazolidone
a. Lovastatin d. Amiodarone
b. Gemfibrozil 116. Cromolyn sodium is used for following actions :
c. Probucol a. Mast cell stabilisation [Dec. 97]
d. Cholestyramine b. H1 antihistamine
108. A young lady on OCP develops sever abdominal c. Phosphodiesterase inhibitor
pain and frank psychosis. Most likely cause is :
d. Inhibition of cyclooxygenase
a. Hysteria [Dec. 99]
117. Biguanides acts by following except: [Dec. 97]
b. Porphyria
a. ↑ Insulin release from pancreas
c. Thrombosis
b. ↑ Glycolysis
d. Mesenteric infarct
c. ↓ Gluconeogenesis
109. True about metabolism of vitamin D in kidney is :
d. ↑ Insulin binding to its receptors
a. Conversion of 25 to 1, 25 dihydroxycholecalciferol
b. Formation of 25 deactivated [Dec. 99]
c. Activated form is deactivated
d. None

Answer 102. a and b 103. a and b 104. b and d 105. a. Cemitidine ... 106. a. Binds to
107. a. Lovastatin 108. b. Porphyria 109. a. Conversion 110. a and c 111. a. Osteo...
112. a, b & c 113. c and d 114. b. adenoma 115. a. Allopurinol 116. a. Mast cell
117. a. ↑ Insulin

334
Phar macology of Respiratory
Pharmacology Respiratory,, Endocrinal & Gastr ointestinal System
Gastrointestinal 5
ANSWERS, REFERENCES, EXPLANATIONS WITH INFORMATIVE ILLUSTR ATIONS

1. Ans. is a i.e. Montelukast Ref. KDT 6/e, p 217, 222 - 223; O.P. Ghai 6/e, p 361

Montelukast and Zafirlukast are leukotriene receptor antagonist used for the treatment of mild to moderate
persistent asthma and excercise - induced asthma.
They are also indicated for prophylactic therapy of mild to moderate asthma as alternatives to inhaled
glucocorticoids.
Montelukast, Zafirlukast received approval for the use in pediatric asthma patients more then 6 years of age.
Their main advantage is that they are given orally once daily and they do not induce tachyphylaxis.
Montelukast: - It is a cystenyl leukotriene (LTC and LTD ) antagonist
4 4

Zileuton : - It is a potent and selective 5- Lipooxygenase inhibitor, blocks


LTC /LTD /LTB synthesis.
4 4 4

- Its clinical efficacy in asthma is similar to Montelukast.


- Its duration of action is short.
- It has hepatotoxic potential. There are significant interactions with other drugs metabolized
in liver
- Its use is therefore restricted.
Nedocromil : - It has mast cell stabilizing action.

2. Ans. is a i.e. Zafirlukast Ref. KDT 6/e, p 222; Harrison 17/e, p 1604

Zafirlukast is LTD receptor antagonist used in prophylactic therapy for mild to moderate asthma.
4

LEUKOTRIENE ANTAGONISTS

Leukotriene receptor antagonist 5-Lipoxygenase inhibitor


• Montelukast • Zileuton
• Zafirlukast
– Competitively blocks leukotriene receptors – Blocks enzyme lipooxygenase
↓ ↓
– Bronchodilatation, ↓ sputum eosinophil count, – Decrease LTC /LTD /LTB synthesis
4 4 4

↓ inflammation and reactivity ↓


– Indicated for prophylactic therapy of mild to – Decrease features of asthma
moderate asthma

– Not used in acute asthma

Montelukast and Zafirlukast


• Indication – Prophylaxis of mild to moderate asthma
• Efficacy of Zafirlukast is less than that of steroids
• It is not effective in terminating acute episode
• Pharmacology – Well absorbed orally – t1/2 - Montelukast - 3-6 hours
– t1/2 - Zafirlukast - 8-12 hours

335
5 Self Assessment & Review Phar macology
Pharmacology
• Adverse effects
– Headache, Rashes
– Eosinophilia, Neuropathy
– Churg strauss syndrome (vasculitis and eosinophilia)

3. Ans. is c i.e. Glargine


Ref. Goodman & Gilman’s 11/e, p 1626-1627; KDT 6/e, p 259

FEATURES OF INSULIN GLARGINE


• It is the first long acting analog of human insulin.
• It is a clear solution of pH 4.0. This pH stabilizes the insulin hexamer and results in a prolonged and
predictable absorption from subcutaneous tissues.
• It cannot be mixed with short-acting insulin preparations due to its acidic pH.
• It results in less hypoglycemia, has a sustained peakless absorption profile, provides a better once
daily 24-hour insulin coverage than ultralente or NPH insulin.
• Injection of Glargine forms microfined precipitate in subcutaneous tissue as the pH rises from 4.0 to 7.4. As
a result, a sustained release of insulin takes place from the site of injection which mimics the basal secretion.

Types Onset Duration Comments


Ultrashort - acting insulins
1. Insulin lispro 10-20 min 3-4 hrs Clear solutions; Given S.C;
can be given I.V.
2. Insulin aspart 10-20 min 3-4 hrs
3. Insulin glulisine 10-20 min 3-4 hrs

Short - acting Insulins


1. Regular insulin (crystaline Zn-insulin) 30-60 min 5-6 hrs Clear solutions; given S.C or I.V.
2. Prompt insulin-Zn suspension – do – – do – Cloudy; given S.C, not I.V.
(semilente)

Intermediate - acting insulins


1. Neutral protamine Hagedorn (NPH) or 1-2 hrs 20-24 hrs Cloudy; given S.C, not I.V.
isophane insulin)
2. Lente insulin (30% semilente mixed with 1-3 hrs 20-24 hrs Cloudy; given S.C, not I.V.
70% ultralente insulin), it is
Zn-insulin suspension

Long-acting insulins
1. Ultra-lente insulin (Protamine Zinc insulin) 4-6 hrs 20-36 hrs Cloudy; given S.C, not I.V.

2. Insulin glargine 1.5 hrs 15-24 hrs Clear solution at pH 4; given S.C
3. Insulin detemir 1-4 hrs 24 hrs

336
Phar macology of Respiratory
Pharmacology Respiratory,, Endocrinal & Gastr ointestinal System
Gastrointestinal 5
4. Ans. is a i.e. 5-alpha reductase inhibitor Ref. KDT 6/e p 294, 778

• Finasteride is a competitive inhibitor of the enzyme 5-alpha reductase which converts testosterone
into more active dihydrotestosterone responsible for androgen action in many tissues including prostate
gland and hair follicles.
• It is more active for the type 2 isoenzyme which predominates in the male urogenital tract.
• It is used for benign hypertrophy of prostate and male pattern baldness.
• It is the only drug to retard disease progression in BPH.
Looking at other options:
GnRH analogue : Leuprolide, Buserelin, Goserelin, Nafarelin
Selective estrogen receptor modulator : Tamoxifen, Raloxifene, Toremifene
Non-steroidal anti-androgen drug : Flutamide, Bicalutamide

5. Ans. is d i.e. Neuropeptide Y analogues Ref. Harrison 17/e, p 464, KDT 6/e, p 130-131

Neuropeptide Y analogues are not used, instead the Neuropeptide Y antagonists are used in the
treatment of obesity. It can easily be interpreted by the figure below:
Anorectic drugs: – Used in obesity for short periods. Do not improve long term outlook.
Anorectic agents
NorAdrenergic Serotonergic Hormonal Newer drugs
• Act on Apetite centre • Act on Satiety centre
• Ex. – Phentermine – Fenfluramine – Thyroxine – Orlistat
– Phenylpropanolamine – Dexfenfluramine Role of thyroxine is – Olestra
– Diethylpropion limited as majority – Leptin
– Mazindol of obese patient are – Sibutramine
euthyroid – Neuropeptide
Y antagonists
– β agonist
3

– Rimonabant

Drugs Mechanism of action


Orlistat Inhibit gastric and pancreatic lipase; hence it interferes with digestion and
absorption of dietary triglycerides.
Olestra Sucrose polyester which can be used as a cooking medium in place of fat but
is neither digested nor absorbed.
Sibutramine Combined serotonergic and noradergenic reuptake inhibition plus thermogenesis
stimulation by b3 activation in adipose tissue.

6. Ans. is a i.e. Hyperkalemia


Ref. KDT 6/e, p 599; Harrison 17/e, p 739-741

Adverse effects characteristic of Heparin:


• Bleeding due to overdose
• Heparin-induced thrombocytopenia

337
5 Self Assessment & Review Phar macology
Pharmacology
• Osteoporosis and spontaneous fractures on prolonged use of high dose
• Transient alopecia
• Hypersensitivity
Drugs causing Hyperkalemia
• ACE inhibitors • NSAIDs
• Amiloride • Pentamidine
• Cyclosporine • Pot. salts of drugs
• Cytotoxics • Spironolactone
• Digitalis overdose • Succinylcholine
• Heparin • Triamterene
• Lithium • Trimethoprim

7. Ans. is a i.e. Oxytetracycline


Ref. KDT 6/e, p 637; Goodman & Gilman 11/e, p 979 – 980; Harrison 17/e, p 948-949

Although Tetracycline is used in the treatment, I will still exclude it, as Option C mentions it as Oxytetracycline
and other options are more obviously correct answers.
TRIPLE THERAPY for 14 Days (BD)
PCM : PPI + Clarithromycin 500 mg + Metronidazole 500 mg
PCA : PPI + Clarithromycin 500 mg + Amoxicillin 1 gm
QUADRUPLE THERAPY for 14 Days
PMBT : PPI (BD) + Metronidazole 500 mg (TDS) + Bismuth subsalicylate 525 mg
(QID) + Tetracycline 500 mg (QID)
OR
H receptor antagonist (BD) + Metronidazole 250 mg (QID) + Bismuth
2

subsalicylate 525 mg (QID) + Tetracycline 500 mg (QID)


Note : H. pylori, a gram negative rod , has been associated with gastritis and the subsequent development of
Q

gastric and duodenal ulcers, gastric adenocarcinoma, and gastric B-cell lymphoma.

8. Ans. is b i.e. Is used orally Ref. KDT 6/e, p 235; Goodman Gilman 11/e, p 149; Katzung 9/e, p 1048

“Gastrointestinal side effects-including diarrhea, nausea and abdominal pain occur in up to 50% of
patients receiving octreotide. In most patients, these diminish over time and do not require cessation of
therapy.” ...... Goodman 11/e

Octreotide is not given orally; when given i.v, it has a serum half life of 1.5 hours. It also may be administered by
s.c injection having a duration of action of 6-12 hours.
Gallstones occur in 25% of patients receiving octreotide due to decreased gallbladder contraction and
gastrointestinal transit time.
Note :
• Although Octreotide has diarrhea as an adverse effect, it is also used for the treatment of acromegaly and
secretory diarrhea associated with carcinoid, AIDS, cancer chemotherapy or diabetes. The control of diarrhea
is due to suppression of hormones which enhance intestinal mucosal secretion.

338
Phar macology of Respiratory
Pharmacology Respiratory,, Endocrinal & Gastr ointestinal System
Gastrointestinal 5
• Octreotide is injected i.v. reduces hepatic blood flow and helps stop esophageal variceal bleeding.
• Octreotide is a synthetic octapeptide surrogate of somatostatin and 40 times more potent in suppressing
GH secretion and longer acting.

9. Ans. is c i.e. Phosphodiesterase


Ref. KDT 6/e, p 220 - 221; Goodman & Gilman 11/e, p 744; Katzung 9/e, p 325-326

Aminophylline is a theophylline preparation (theophylline-ethylenediamine complex) commonly used


for therapeutic purposes.
Mechanism of Action of Theophyllines
• Release of Ca from sarcoplasmic reticulum specially
++

in skeletal and cardiac muscle


• Nonselective Inhibition of phosphodiesterase

ATP Adenyl cyclase cAMP Phosphodiesterase 5-AMP (causes bronchodilation)


or ⎯⎯→ or ⎯⎯→ or
GTP Guanyl cyclase cGMP ↑ (–) 5-GMP (causes bronchodilation)
↑ Theophylline
β agonist
2

Blockade of adenosine receptors


• Adenosine acts as an autocoid and transmitter with myriad biological actions.
• In bronchial asthma, adenosine can cause bronchoconstriction in asthmatics and potentiate immunologically
induced mediators release from human lung mast cell.

Selective PDE Inhibitors


• PDE III : Amrinone, Milrinone
• PDE IV : Drotaverine
• PDE III + IV : Cilomilast
• PDE V : Sildenafil
• PDE VI : Sildenafil

10. Ans. is c i.e. Decreasing the osteoclast mediated resorption of bone


Ref. KDT 6/e, p 333 - 334

Mechanism of action of Bisphosphonates


The mechanism of action of BPNs is not fully understood, but two facets of action have been delineated :
Bisphosphonates inhibits bone resorption by:
a. • Accelerating apoptosis of osteoclast reducing their number
• Disruption of cytoskeleton and ruffled border of osteoclasts
• Affect osteoclast precursors and inhibit their differentiation by suppressing IL-6

339
5 Self Assessment & Review Phar macology
Pharmacology
b. It has been shown recently that bisphosphonates, specially the second and third generation potent amino-
derivatives like alendronate, have important metabolic effects in the mevalonate pathway for isoprenoid lipid
synthesis. They inhibit prenylation of certain GTP-binding protein involved in cytokeletal organization, membrane
ruffling and vesicle movement. The net result is inactivation of osteoclasts, impaired vesicle fusion and
enhanced apoptosis.
Uses of bisphosphonates
• Osteoporosis • Osteolytic bone metastasis
• Paget’s disease • Hypercalcemia of malignancy

11. Ans. is b i.e. Raloxifene Ref. KDT 6/e, p 305

Raloxifene do not relieve vasomotor symptoms of menopause like hot flushes; rather hot flushes may
be induced in some women.
Mechanism of action and effects of Raloxifene:
Raloxifene
Selective estrogen reuptake modulators (SERMs)

Partial Agonist Antagonist

Bone C.V.S. Endometrium Breast


• Prevent bone • ↓ LDL by ↑ hepatic • No risk of • E.R. +ve • E.R. –ve
loss in postmeno- LDL receptor endometrial Protection No pro-
pausal women • Antiatherogenic carcinoma from breast tection
• Risk of vertebral carcinoma
fracture is reduced
but not in long
bones except ankle

Adverse effects : • Hot flushes • Leg cramps • DVT • Pulmonary embolism

Raloxifene is an effective alternative HRT to prevent and treat osteoporosis in postmenopausal women.
• Tibolone: It is a SERM whose metabolites have mixed estrogenic, progestogenic and weak androgenic
activity. It is comparable to hormone replacement therapy for the treatment of climacteric-related complaints
(includes hot flushes). Option ‘d’.
• Isoflavones: are phytoestrogens that bind weakly to estrogen receptor but have over 80% of estrogen’s
affinity for estrogen receptor. Option ‘c’.
- Soy and red clover isoflavones are selective estrogen receptor modulators, which act as estrogen on bone,
brain, and vasculature but have no stimulatory effects upon the uterus or breast.
- Daily intake of 60 gm of isolated soy protein reduces hot flushing by about 45% within 12 weeks. They also
improve plasma lipid profile and inhibit development of coronary atherosclerosis in male monkeys.
• Androgens: Androgen deficiency contributes to hot flushes loss of lipids and sexual hair, muscle atrophy
and osteoporosis. Selected women may be treated with low-dose methyltestosterone, which is available in
combination with conjugated estrogens.

340
Phar macology of Respiratory
Pharmacology Respiratory,, Endocrinal & Gastr ointestinal System
Gastrointestinal 5
12. Ans. is b i.e. Metoclopramide Ref.Dutta 4/e, p 557; KDT 6/e, p 643 - 644

Metoclopramide is a prokinetic drug; it promotes gastrointestinal transit and speed gastric emptying.

Aspiration of gastric contents (MENDELSON’S SYNDROME)


Cause: Delayed gastric emptying due to ↑ serum progesterone, ↓ motility

Gastric acid content pH < 2.5

Chemical pneumonitis / Bronchopneumonia

Signs and Symptoms: Tachycardia, hypotension, dyspnoea, cyanosis, bronchospasm.

Prevention:
• H blocker – Ranitidine (150mg orally) a night before and repeated one hour before GA to raise the pH of
2

stomach.
• Metoclopramide - to decrease gastric volume and increase the tone of lower esophageal sphincter.
• Intubation with adequate cricoid pressure. (Sellick’s manoeuvre)

13. Ans. is c i.e. Adenosine receptors Ref. KDT 6/e, p 220; Goodman & Gilman 11/e, p 728

“Theophylline is a competitive antagonist at adenosine receptors.”


Mechanism of action:
Three distinct cellular actions of methylxanthines:
a. Blockade of adenosine receptors
- Adenosine acts as an autacoid and transmitter with myriad biological actions.
- In bronchial asthma, adenosine can cause bronchoconstriction in asthmatics and potentiate
immunologically induced mediators release from human lung mast cell.
b. Release of Ca from sarcoplasmic reticulum, especially in skeletal and cardiac muscle.
++

c. Inhibition of phosphodiesterase.

14. Ans. is c i.e. It blocks the conversion of dihydrotestosterone to testosterone


Ref. Harrison 17/e, p 488-489; KDT 6/e, p 294

Finasteride is 5-α α-reductase inhibitor and blocks the conversion of Testosterone to DHT.
• Circulating and prostatic dihydrotestosterone concentration is lowered but plasma LH and testosterone
levels are unchanged because testosterone level is mediated by feed back pituitary LH inhibition.
• Finasteride decreases prostate size and increases peak urinary flow rate in 50% patients with symptomatic
benign hypertrophy of prostate (BHP).
• Beneficial effect is not immediate, need 6 months for maximum symptomatic relief.
• Patients with large prostate (volume > 40ml) obtain greater relief than those with smaller gland, but it is the
only drug which can retard disease progression.
• It is effective orally, extensively metabolized in liver and excreted in urine and faeces. t½ is - 4-8 hours.
• It is well tolerated

341
5 Self Assessment & Review Phar macology
Pharmacology

• Side effects: • Decreased libido • Impotence


• Decreased volume of ejaculate • Skin rash and swelling of lips

15. Ans. is c i.e. Taxol Ref. Katzung 10/e, p 896 – 897; KDT 6/e, p 305-306

Taxol: is a brand name for paclitaxel and is not a hormonal agent. It is indicated for metastatic ovarian and
breast cancer. Major toxicity of paclitaxel is reversible myelosuppression and “stocking and glove” neuropathy.
Now, let us consider the other 3 options separately.
Letrozole and • They are nonsteroidal competitive inhibitor of aromatase that has no inhibitory effect
anastrozole on adrenal glucocorticoid or mineralocorticoid synthesis.
• They are presently approved for :
– First-line treatment of postmenopausal women with metastatic breast cancer that
is ER-positive.
– Treatment of postmenopausal women with metastatic breast cancer that is ER-
positive and has progressed while on tamoxifen therapy.
– An adjuvant therapy of postmenopausal women with hormone-positive, early-stage
breast cancer.
Exemestane • It is a steroidal hormonal agent that binds to and irreversibly inhibits aromatase.
• There appears to be a lack of cross resistance between exemestane and nonsteroidal
aromatase inhibitors.
• It is indicated for the treatment of advanced breast cancer in postmenopausal women
whose disease has progressed on tamoxifen therapy.
Tamoxifen • It is a selective estrogen receptor modulator (SERM) and is useful in estrogen receptor
expressing breast cancer.

16. Ans. is d i.e. Sucralfate inhibits absorption of ranitidine Ref. KDT 6/e, p 636

Sucralfate adsorbs many drugs and interferes with the absorption of tetracyclines, fluoroquinolones, cimetidine,
phenytoin and digoxin.

Sucralfate is an ulcer protective drug. It is basic aluminium salt of sulfated sucrose.


– It polymerizes at pH<4 by cross linking of molecules, assuming a sticky gel like consistency.
– It precipitates surface proteins at ulcer base and acts as a physical barrier preventing acid, pepsin and bile
from coming in contact with ulcer base.
– It augments gastric mucosal PG synthesis.
– It is considered to be superior in patients who continue to smoke.

17. Ans. is c i.e. Hypoglycemia is a common and serious side effect


Ref. KDT 6/e, p 270; Katzung 9/e, p 710

Prominent adverse effects of α glucosidase inhibitors include flatulence, diarrhea and abdominal
discomfort due to fermentation of unabsorbed carbohydrates.

Classification : α – glucosidase inhibitors – Acarbose – Miglitol

342
Phar macology of Respiratory
Pharmacology Respiratory,, Endocrinal & Gastr ointestinal System
Gastrointestinal 5
Mechanism of Action :
Is a complex oligosaccharide

Reversibly inhibits α - glucosidases, the final enzymes in digestion of carbohydrates in the brush
border of small intestinal mucosa

Decreases digestion and absorption of polysaccharides and sucrose

Post prandial hyperglycemia is reduced without increasing insulin levels i.e., has insulin sparing effect
– It is a mild antihyperglycemic and not hypoglycemic.

Contraindications : – Inflammatory bowel disease


– Hepatic insufficiency
– Renal insufficiency (serum creatinine > 2mg%)
Uses : – It is FDA approved for use in Type 2DM both as monotherapy and in combination
with sulphonylureas, where the glycemic effect is additive.
– It is unique because it reduces post prandial glucose rise even in Type 1DM patients.

18. Ans. is c i.e. Reduces incidence of venous thrombosis Ref. KDT 6/e, p 303-305

Agents Affecting Estrogenic System in Females


Oestrogen Selective Oestrogen Receptor
Modulators (SERMs)
– Oestradiol – Tamoxifen – Raloxifene
– Oestrone
Breast • ↑ ed risk of breast cancer • ↓ed risk of breast cancer • ↓ed risk of breast cancer
(2% / year of HRT use) • It is the 1 choice
st
• Used in the treatment of
hormonal agent of breast ER + ve breast cancers.
cancer in both pre and
post menopausal women.
Lipid Profile • ↓ LDL cholesterol • ↓ LDL cholesterol • ↓ LDL cholesterol
↓ Total cholesterol ↓ Total cholesterol ↓ Total cholesterol
↑ HDL cholesterol (No effect on HDL (No effect on HDL
↑ Triglycerides cholesterol & triglycerides) cholesterol & triglycerides)
Endometrial • ↑ ed risk of endometrial • ↑ ed risk of endometrial • No increased risk
carcinoma (5-15 fold) carcinoma (2-3 fold) of endometrial carcinoma
Deep Venous • ↑ ed risk 3 times • ↑ ed risk 3 times • ↑ ed risk 3 times
Thrombosis
Bone • ↓ ed • ↓ ed • ↓ ed
Resorption
Vascular • ↓ ed • ↑ ed • ↑ ed
Symptoms
(Hot Flushes)

343
5 Self Assessment & Review Phar macology
Pharmacology
19. Ans. is c i.e. Renal dysfunction is not a contraindication for their use
Ref. KDT 6/e, p 269; Harrison 17/e, p 2300 - 2301
Biguanides are contraindicated in patients with renal failure, congestive heart failure, alcoholism
and hepatic disease because of increased risk of lactic acidosis.
Classification of Biguanides
• Phenformin
• Metformin
Differences from Sulphonylureas
• No hypoglycemia in normal and non diabetic subjects
• Do not stimulate pancreatic β cells to release insulin

Mechanism of action
• Do not stimulate pancreatic β cells to cause insulin release.
• Suppress hepatic gluconeogenesis and glucose output from the liver: the major action.
• Enhance insulin mediated glucose disposal in muscle and fat (so, insulin is necessary for its action)

Do not alter translocation of GLUT4 (the major glucose transporter)

Increase GLUT1 transport from intracellular site to plasma membrane (thus, effect differs from insulin)

• Interfere with mitochondrial respiratory chain



Promote peripheral glucose utilisation by enhancing anaerobic glycolysis (Phenformin, more prominent in
this effect)
• Inhibit intestinal absorption of glucose, hexoses, vitamin B , amino acids
12

Adverse Effects:
• GI side effects in 20% of patients
• Do not cause hypoglycemia
• Lactic acidosis – most common and most serious - Phenformin
• Vit. B deficiency (Megaloblastic anaemia) - Metformin
12

Contraindications:
• Renal insufficiency • Alcoholism • Hepatic disease
• Severe hypoxia • CHF • Any form of acidosis

Use:
Type 2DM – used either alone or in combination

Combination therapy with Glucose lowering agents in Type 2 diabetes mellitus


• Sulphonylurea + Metformin or Thiazolidinedione
• Sulphonylurea + α - glucosidase inhibitor
• Insulin + Metformin or Thiazolidinedione
• Metformin + Thiazolidinedione

In long term, when type 2 diabetes mellitus enters phase of relative insulin deficiency, insulin is ultimately
required.

344
Phar macology of Respiratory
Pharmacology Respiratory,, Endocrinal & Gastr ointestinal System
Gastrointestinal 5
20. Ans. is b i.e. Corticosteroid Ref. KDT 6/e, p 232, 48

Drug acting on Drugs acting at Drugs acting on


cell membrane receptors cytoplasmic receptor nuclear receptor
Intracellular Intracellular
IP -DAG
3
Activation of c-GMP
Tyrosine kinase
• Adrenergic α 1
• Insulin • ANP Steroid Thyroid hormones
• Histamine-H 1
• IGF- I • BNP – Glucocorticoids – Thyroxine
• Muscarine-M ,M 1 3
• GH (Nesiritide) – Mineralocorticoids – Triiodothyronine
• 5-HT 2
• Prolactin • EDRF – Estrogens
• Vasopressin V 1
• Nitric oxide – Progestins
• Oxytocin – Androgens
• Bradykinin–B2 – Calcitriol
• Angiotensin
• Prostaglandin FP
• EP 1

• EP 3

Adenyl cyclase
↑ c-AMP ↓c-AMP

• Adrenergic-β • Adrenergic -α 2

• Histamine-H 2
• Muscarinic-M 2

• Dopamine-D 1
• Dopamine-D 2

• Glucagon • 5HT1
• TSH/FSH/LH • GABA B

• ACTH • Opioid-μ,δ
• Prostaglandin-EP 2
• Angiotensin
• Prostacyclin-IP • Prostaglandin EP3
• Adenosine-A 2
• Somatostatin
• Adenosine-A 1

21. Ans. is d i.e. Cetrizine Ref. KDT 6/e, p 158 - 159

The patient is a highway truck driver and has profuse rhinorrhoea and sneezing. Therefore, he must be prescribed
a drug with least or no sedative effect.
Cetrizine, from the given options is least sedative, so it is preferred.
• Second Generation / Nonsedative antihistaminic / Antiallergic
Drugs Important points
Terfenadine Most rapid onset
Associated with Torsades de pointes (ventricular arrhythmia) when given with drugs that
inhibits metabolism e.g. erythromycin, clarithromycin, ketoconazole, itraconazole.
Astemizole Longest duration of action
Associated with Torsades de pointes, same as above
Loratidine Least / absent CNS depression, long and fast acting
Cetrizine No topical activity

345
5 Self Assessment & Review Phar macology
Pharmacology

Azelastine Good topical activity


Ebastine Active metabolic carbastine
Fexofenadine Is an active metabolite of terfenadine. Does not prolong QTc interval. Although erythromycin
and ketoconazole increase its blood levels, no arrhythmias have been observed.

Other Options: • Dimenhydrinate and promethazine are highly sedative


• Pheniramine is moderately sedative

22. Ans. is b i.e. Shortly before the meals Ref. Katzung 9/e, p 1039; KDT 6/e, p 631 - 632

The plasma level of proton pump inhibitor (PPI) peaks 1 hour after oral intake of the drug and maximum
proton pumps are activated at meals. So, for PPIs to be maximally effective, they should be given 1 hour
before taking meals.
The bioavailability of proton pump inhibitors decreases 50% by the food, hence drug should be administered in
an empty stomach followed 1 hour later by a meal to activate the H+K+ATPase and make it more susceptible
to the PPI.

About proton pump in stomach:


Fasting state Fed state
Only 10% proton pumps are active Maximum proton pumps are active
So, maximum efficacy of PPI occurs when:
• Maximum proton pumps are active and blocked by PPI
• Plasma level of PPI is maximum for proton pumps

23. Ans. is d i.e. Cimetidine Ref. KDT 6/e, p 317

Drugs causing oral contraceptive failure:


Mnemonics : Poor Contraceptive Pehli RAT Par Tare Gine
• Phenobarbitone • Ampicillin
• Carbamazepine • Tetracyclin
• Phenytoin • Primadone
• Rifampicin • Griseofulvin

• Rifampicin, ampicillin and tetracycline cause OCP failure by suppressing intestinal flora, thereby decreasing
enterohepatic circulation of OCP.
• All the other mentioned drugs decrease the effectiveness of OCP by inducing hepatic microsomal enzymes,
leading to increased metabolism of OCP.

24. Ans. is d i.e. Epinephrine receptor Ref. Harper 25/e, p 584

Steroid receptor supergene family hormones


• Glucocorticoid • Vitamin D • Mineralocorticoid • Thyroid hormone
• Progestins • Retinoic acid • Oestrogen • Vitamin A

346
Phar macology of Respiratory
Pharmacology Respiratory,, Endocrinal & Gastr ointestinal System
Gastrointestinal 5
25. Ans. is b i.e. Bind plasma membrane receptors and following internalization influence nuclear changes
Ref. KDT 6/e, p 48, 276 - 280

Mechanism of action of steroids:


• Steroids bind to cytoplasmic receptors and inhibit the release of arachidonic acid from membrane
phospholipids by inhibiting the enzyme phospholipase A . 2

• Various drugs inhibit the biosynthetic pathway of prostaglandins and leukotrienes as depicted in flow chart
below :

Option ‘c’ : The antiinflammatory action is non specific and covers all components and stages of inflammation
which includes: – Reduction of increased capillary permeability
– Local exudation
– Cellular infiltration
– Phagocytic activity
Option ‘d’ : Action on carbohydrate metabolism : It maintains the blood glucose level during starvation by
following actions: – Glycogen deposition in liver by activation of glycogen synthetase
– Gluconeogenesis
– Inhibition of glucose utilization by peripheral tissues

26. Ans. is a i.e. Increase in dose is required in cardiopulmonary disease


Ref. KDT 6/e, p 221; Goodman & Gilman 11/e, p 728

Agents which induce theophylline metabolism Agents which inhibit theophylline metabolism

↓↓ plasma level : Dose to be increased ↑ ↑ plasma level : Dose to be decreased to 2/3


• Smoking • Erythromycin

347
5 Self Assessment & Review Phar macology
Pharmacology

• Phenytoin • Ciprofloxacin
• Rifampicin • Cimetidine
• Phenobarbitone • Oral contraceptives
• Charcoal broiled meat meal • Allopurinol
• Elderly, CHF, pneumonia
• Liver disease

Theophylline increases cAMP by inhibiting the enzyme phosphodiesterase.

27. Ans. is d i.e. Sucralfate polymerises only when gastric pH is less than 4 Ref. KDT 6/e, p 636

Famotidine is an H receptor blocker, which increases the gastric pH by blocking histamine induced gastric
2

secretion. The polymerization of sucralfate is dependent on acidic pH.


Sucralfate is an ulcer protective drug. It is basic aluminium salt of sulfated sucrose.
– It polymerizes at pH<4 by cross linking of molecules, assuming a sticky gel like consistency.
– It precipitates surface proteins at ulcer base and acts as a physical barrier preventing acid, pepsin and bile
from coming in contact with ulcer base.
– It augments gastric mucosal PG synthesis.
– It is considered to be superior in patients who continue to smoke.

28. Ans. is a i.e. Spironolactone Ref. Harrison 15/e, p 430; Harrison 17/e, p 2318

Drugs causing gynaecomastia:


A • ACE Inhibitors G • Griseofulvin
B • Busulfan H • Heroine
C • Ca channel blockers
++
I • INH
C • Clomiphene K • Ketoconazole
D • Digitalis, Diethylstilbestrol, Diazepam P • Phenytoin
E • Ethionamide, Estrogen R • Reserpine
F • Frusemide, Flutamide S • Spironolactone
T • Testosterone

29. Ans. is a i.e. Calcium carbonate Ref. KDT 6/e, p 635

Milk Alkali Syndrome: In the past, large quantity of milk was prescribed with CaCO (or NaHCO ) for peptic 3 3

ulcer. Such regimen often produced a syndrome characterized by headache, anorexia, weakness, abdominal
discomfort, Ca deposition – renal stones due to concurrent hypercalcemia and alkalosis. Therefore, called milk
++

alkali syndrome.

Note: • Al salts are constipating


+++
• Mg salts are laxatives
++
• CaCO is constipating
3

30. Ans. is c i.e. Lovastatin Ref. KDT 6/e, p 612 - 617

Lovastatin and other statins inhibit HMG-CoA reductase, the rate limiting step in cholesterol synthesis.

348
Phar macology of Respiratory
Pharmacology Respiratory,, Endocrinal & Gastr ointestinal System
Gastrointestinal 5

Effects of Statins, Niacin and Gemfibrozil on the synthesis of lipids

31. Ans. is c i.e. Triamcinolone Ref. KDT 6/e, p 282; Katzung 10/e, p 646

Points to remember: Agents with zero mineralocorticoid activity and increasing glucocorticoid activity are:
Mnemonic: Try Partnership with Dexa Beta for Glucocorticoid activity
Triamcinolone < Paramethasone < Dexamethasone < Betamethasone

Agent with zero Glucocorticoid activity is ‘DOCA’.


Compound Glucocorticoid Mineralocorticoid Forms available : Oral (O),
Injectable (I), Topical (T)
Short -acting (t½<12 hr.)
• Hydrocortisone 1 1 O/I/T
• Cortisone 0.8 0.8 O/I/T
Intermediate acting
(t½12-36 hr.)
• Prednisone 4 0.8 Oral
• Prednisolone 4 0.8 O/I/T
• Methylprednisolone 5 0.5 O/I/T
• Triamcinolone 5 0 O/I/T
Long acting (t½>36 hr.)
• Paramethasone 10 0 O/I/
• Betamethasone 25 0 O/I/T
• Dexamethasone 25 0 O/I/T
• Fludrocortisone 10 50 O/I/T
• Desoxycorticosterone 0 100 Injectable pellets
acetate (DOCA)
• Aldosterone 0.3 3000 Not used clinicaly

349
5 Self Assessment & Review Phar macology
Pharmacology
32. Ans. is c i.e. May induce carcinoid tumours in rats Ref. KDT 6/e, p 631 - 633

Omeprazole is a proton pump inhibitor (PPI) used for acid peptic disease..
-It is a substituted benzimidazole which binds covalently to the SH groups of the proton pump (H+K+ATPase)
and inactivates it irreversibly, especially when 2 molecules of omeprazole reacts with 1 molecule of the enzyme.
-It induces proliferation of parietal cells & gastric carcinoid tumours in rats but not in human beings (Option ‘c’)
- Oral route is preferred in most of the cases.
- From a pharmacokinetic point of view, PPI are the ideal drugs; they have a short serum half life, they are
concentrated and activated near their site of action and they have a long duration of action.
Uses of omeprazole: • Peptic ulcer
• Gastroesophageal reflux disease
• Zollinger Ellison syndrome
Adverse effects : • Nausea • Loose stools • Headache
• Abdominal pain • Muscle pain • Joint pain
• Dizziness • Rashes • Leucopenia

33. Ans. is a i.e. Decrease in bone matrix protein Ref. KDT 6/e, p 285 - 286

Important side effects of corticosteroids are:

C – Cushing’s habitus S – Suppression of hypophyseal pituitary axis


O – Osteoporosis Ter – Ulcer
R – Retarded growth O – Optic glaucoma (Topical)
Tic – Diabetic D – Delayed wound healing
O – Optic post. subcapsular cataract I – Infection
S – Skin lesions-purple striae

34. Ans. is c i.e. Domperidone Ref. KDT 6/e, p 641

Prokinetic drugs : • Metoclopramide • Cisapride


• Domperidone • Mosapride
• Tegaserod
Indications of Domperidone
• Prevention and symptomatic relief of acute nausea and vomiting in adults from any cause but specifically.
– Cytotoxic therapy and radiotherapy
– N and V associated with L-dopa
• Stimulation of gut mobility
– non-ulcerdyspepsia
– esophageal reflux
– functional dyspepsia
Domperidone increase spontaneous gastric activity and autogoniges dopamino inhibition of gastric emptying
(Peripheral activity mainly).
• Increase lower esophageal sphincter pressure (LOSP) and promotes esophageal and antral peristatsis.

350
Phar macology of Respiratory
Pharmacology Respiratory,, Endocrinal & Gastr ointestinal System
Gastrointestinal 5
• Increase pyloric dilatation and gastric emptying.
• Motion sickness, can result during space flights, taking off and landing of an aeroplane, viewing of a moving
horizon or waves of the sea from the dock of a rolling ship.
• It is a labyrinthine vomiting via stimulation of vestibular nuclei, one can use a H receptor antagonist or a
1

centrally acting anticholinergic drug.


• Hyoscine (Scopolamine) is the most effective drug for motion sickness.
• Dicyclomine is used for prophylaxis of motion sickness.
• H antagonists with anticholinergic properties like diphenhydramine, cyclizine, meclizine commonly used for
1

prevention of motion sickness.


• Promethazine so of its antihistaminic - anticholinergic properites used for antimotion sickness.
• Cirmarizine - Antivertigo drug has antihistaminic, anticholinergic, antiserotonin and CCB’s properties.

35. Ans. is c i.e. Nuclear receptors

• Estrogen binds to specific nuclear receptors in target cells and produce effects by regulating protein
synthesis. .....KDT 6/e, p 299
• The estrogen receptors are found predominantly in the nucleus. ..... Katzung 9/e, p 665
• Hormones (estrogen) binds to an estrogen receptor in the nucleus. .....Goodman, Gilman 11/e, p 1548
• Estrogen exerts their effect by interaction with receptors that are members of the superfamily of nuclear
receptors. Goodman & Gilman 11/e, p 1548

36. Ans. is c i.e. GnRH analogues Ref. Harrison 16/e, p 1027; Harrison 17/e, p 2397-2400

Risk factors for Osteoporosis and Osteoporotic Fractures:


Drugs Substance misuse
• Corticosteroids • Heparin • Alcoholism
• Anticonvulsants • Thyroxine • Smoking
• Sedatives - Lithium - Aromatase inhibitors
• Gonadotrophin-releasing hormone (GnRH) agonists - Cyclosporine
Genetic Lifestyle
• Race • Diet / calcium intake
• Low body weight • Exercise / immobility
• Family history • Highly trained athletes
Endocrine Inflammatory disease
• Pituitary disease • Ankylosing spondylitis
• Early menopause • Rheumatoid arthritis
• Thyrotoxicosis • Inflammatory bowel disease
• Hyperparathyroidism
Gastrointestinal disease Others
• Malabsorption • Anorexia nervosa
• Chronic liver disease • Myeloma
• Homocystinuria
• Mastocytosis
• Gaucher’s disease

351
5 Self Assessment & Review Phar macology
Pharmacology
37. Ans. is a i.e. Beclomethasone Ref. KDT 6/e, p 217

Inhaled steroids are : • Beclomethasone • Budesonide • Fluticasone


• Trimacinolone • Flunisolide • Ciclesonide

38. Ans. is d i.e. Patient acceptability is good Ref. KDT 6/e, p 616

• Cholestyramine is a bile acid sequestrant (resins). These are basic ion exchange resins supplied in the
chloride form. They are neither digested nor absorbed in the gut: bind bile acids in the intestine interrupting
their enterohepatic circulation.
• Faecal excretion of bile salts and cholesterol increase

Enhanced hepatic metabolism of cholesterol to bile acid

More LDL receptors are expressed on liver cells;
increased clearance of LDL, IDL and VLDL

Adverse effects: • Nausea • Steatorrhoea


• Flatulence • Deficiency of fat soluble vitamins
• Heart burn • Patient acceptability is poor
• Constipation

Interactions : Reduce the absorption of many orally administered drugs eg. digoxin, warfarin, statins, thiazides,
barbiturates.
A synergistic lowering of plasma cholesterol occurs when given with nicotinic acid.

39. Ans. is d i.e. Warfarin Ref. Harrison 17/e, p 745

• Warfarin crosses placenta and cause fetal abnormalities or bleeding.


• Risk of embryopathy is highest if warfarin is given in first trimester of pregnancy.
• Central nervous system abnormalities can also occur with exposure to coumorins at any time during pregnancy.

The fetal abnormalities includes:


– Nasal hypoplasia
– CNSabnormalities
– Stippled epiphysis (chondrodysplasia)
– Warfarin is contraindicated in 1 and 3 trimesters of pregnancy.
st rd

– Warfarin does not pass into breast milk and can safely be given to nursing mothers.

Other fetal abnormalities reported with maternal warfarin use include absent or non-functioning kidneys, anal
dysplasia, deafness, seizures, Dandy-Walker syndrome, and focal cerebellar atrophy. Use of warfarin throughout
pregnancy has been associated with hemorrhagic complications, premature births, spontaneous abortions,
stillbirths, and death.

352
Phar macology of Respiratory
Pharmacology Respiratory,, Endocrinal & Gastr ointestinal System
Gastrointestinal 5
40. Ans. is d i.e. Palanosetron Ref. Katzung 10/e, p 1027

• The first three agents (ondansetron, granisetron, and dolasetron) have a serum half-life of 4-9 hours and may
be administered once daily by oral or intravenous routes. All three drugs have comparable efficacy and
tolerability when administered at equipotent doses.
• Palonosetron is a newer intravenous agent that has greater affinity for the 5-HT receptor and a long serum
3

half-life of 40 hours.

41. Ans. is d i.e. Teratogenicity Ref. KDT 6/e, p 304

Side effects of Clomiphene citrate


• Polycystic ovaries • Multiple pregnancy • Hot flushes • Gastric upset
• Vertigo • Allergic dermatitis • Ovarian tumour

42. Ans. is b i.e. Nifedipine

• Nifedipine has been used frequently as a tocolytic (agent that delays premature labor). A Cochrane review
has concluded that it is comparable with magnesium sulfate and beta-agonists (such as ritodrine) with fewer
side-effects Its role vis à vis atosiban is not established.
• The approved uses for nifedipine are the long-term treatment of hypertension (high blood pressure) and
angina pectoris. In hypertension, recent clinical guidelines generally favour diuretics and ACE inhibitors,
although calcium channel antagonists are still favoured as primary treatment for older black patients.

Sublingual nifedipine has previously been used in hypertensive emergencies. This was found to be dangerous,
and has been abandoned.

43. Ans. is c i.e. Indomethacin Ref. Goodman & Gilman 11/e, p 1633; Harrison 17/e, p 2276

Glucose intolerance will be caused by drugs causing hyperglycemia.


Drug causing Hypoglycemia Drug causing Hyperglycemia
Mnemonics : DD PaPa GOA Gye
• Quinine • Salicylates D : Diazoxide G : Glucocorticoids
• β blocker • Ethanol D : Diuretics O : Oral contraceptives
• Clofibrate • ACE inhibitors P : Phenytoin A : Asparaginase
• Lithium • Theophylline P : Pentamidine G : Growth hormone
• Bromocriptine • Tetracycline Others :
• Pyridoxine • Pentamidine • Epinephrine • Atypical antipsychotics
• Indomethacin • Clonidine • HIV-1 protease inhibitors
• Morphine • β2 receptor agonists
• Heparin • Ca channel blockers
2+

• Nalidixic acid • H -receptor blockers


2

• Sulfinpyrazone

353
5 Self Assessment & Review Phar macology
Pharmacology
44. Ans. is d i.e. Vitamin D intoxication Ref. KDT 6/e, p 334

Bisphosphonates are synthetic organic compounds that inhibit osteoclasts-mediated bone resorption by binding
to the hydroxyapatite crystals of bone.

Uses of Bisphosphonates
Osteoporosis Paget’s disease Hypercalcemia of Osteolytic bone
malignancy metastasis
– Alendronate (oral) – Tiludronate (oral) – Ibandronate (oral) – Pamidronate (iv)
– Risedronate (oral) – Etidronate (oral / iv) – Pamidronate (iv) – Zoledronate (iv)
– Risedronate (oral) – Zoledronate (iv)

45. Ans. is d i.e. More effective empty stomach


Ref. Goodman & Gilman 11/e, p 958

Option “a” The fibrates are the drugs of choice for treating hypertriglyceridaemias, particularly those
associated with low levels of HDL-C. These are mainly used to treat two hyperlipidaemias-
familial hypertriglyceridaemia (type IV) and dysbetalipoproteinemia (type III).
Option “b” The fibrates cause reversible myopathy, GIT distress and increase in aminotransferases levels.
Their use with statins should be avoided as the combination may increase the risk of myositis
and rhabdomyolysis.
Option “c” The fibrates increase the lithogenicity of bile. Clofibrate use has been associated with increased
risk of gallstone formation; gemibrozil and fenofibrate reportedly do not increase biliary tract
disease. ...........
... Goodman & Gilman
Option “d” Fibrates are well absorbed from GIT and their absorption is improved when they are taken with
food.

46. Ans. is a i.e. Rifampicin Ref. KDT 6/e, p 741

• Rifampicin is a microsomal enzyme inducer - increases several CYP450 isoenzymes, enhances its own
metabolism as well as that of many drugs including oral contraceptives, warfarin, corticosteroids etc.
• Contraceptive failures have occured: advised to switch over to an oral contraceptive with higher dose (50μg)
of estrogen or alternative method of contraception.
• Ampicillin can also cause contraceptive failure by impairing enterohepatic circulation of estrogens mediated
by bacteria.

47. Ans. is d i.e. Metformin Ref. Goodman & Gilman 11/e, p 1638; Harrison 17/e, p 2300

Metformin should be discontinued temporarily prior to the administration of intravenous contrast media and
prior to any surgical procedure. The drug should not be readministered any sooner than 48 hours after such procedures
and should be withheld until renal function is determined to be normal. These conditions all predispose to increased
lactate production and hence to the potentially fatal complication of lactic acidosis.

354
Phar macology of Respiratory
Pharmacology Respiratory,, Endocrinal & Gastr ointestinal System
Gastrointestinal 5
Contraindications of Metformin therapy
• Renal impairment • Hepatic impairment
• H/o lactic acidosis (of any cause) • Cardiac failure
• Chronic hypoxic lung disease • Prior to any surgery
• Prior to iv contrast media administration • Seriously ill patient
• Nil orally

48. Ans. is c i.e. Phenazocine Ref. KDT 6/e, p 641

Class of Emetics Examples


Anticholinergics Hyoscine, Dicyclomine
H antihistaminics
1
Promethazine, Diphenhydramine, Dimenhydrinate, Cyclizine, Meclizine, Cinnarizine
Neuroleptics Chlorpromazine, Prochlorperazine, Haloperidol
Prokinetic drugs Metoclopramide, Domperidone, Cisapride, Mosapride, Tegaserod
5-HT antagonists
3
Ondansetron, Granisetron
Adjuvant antiemetics Dexamethasone, Benzodiazepines, Cannabinoids
Aprepitant Substance-P/Neurokinin Receptor Antagonist (used in chemotherapy induced vomiting)

49. Ans. is b i.e. Phytonadione Ref. Katzung 10/e, p 551; KDT 6/e, p 602

Warfarin activity is potentiated with concomitant use of:


• Enzyme inhibitors such as: Metronidazole, chloramphenicol, disulfiram, erythromycin, cimetidine, allopurinol,
amiodarone and celecoxib.
• Drugs which displace warfarin from protein binding sites such as: Cotrimoxazole, indomethacin, phenytoin,
probenecid, phenylbutazone and ethacrynic acid.
• Liquid paraffin (habitual use): Due to emulsification and excretion of vit. K leading to vit. K deficiency
• Inhibitors of platelet aggregation: Aspirin
• Drugs which inhibit gut flora and reduce vit. K synthesis: e.g., broad spectrum antibiotics.
• Drugs causing hypoprothrombinaemia: such as 3 generation cephalosporins like cefoperazone, cefamandole
rd

and moxalactam.
• Miscellaneous factors: Hepatic disease (decrease the synthesis of clotting factors), hyperthyroidism (enhances
catabolism of clotting factors).

Increased prothrombin time Decreased prothrombin time

Pharmacokinetic Pharmacodynamic Pharmacokinetic Pharmacodynamic


Amidarone Drugs Barbiturates Drugs
Cimetidine Aspirin (high doses) Cholestyramine Diuretics
Disulfiram Cephalosporins 3 generation
rd
Rifampin Vitamin K
Metronidazole Heparin Body factors
Fluconazole Body factors Hereditary resistance
Phenylbutazone Hepatic disease Hypothyroidism
Sulfinpyrazone Hyperthyroidism
Trimethoprim-
Sulfamethoxazole

355
5 Self Assessment & Review Phar macology
Pharmacology
50. Ans. is b i.e. Vit. K injection Ref. Goodman & Gilman 11/e, p 1478

To reverse the effect of overdose of oral anticoagulants, phytonadione is the preparation of choice
because it acts most rapidly.
• Bleeding is the major toxicity of oral anticoagulant drugs.
• If the INR is > 5, vitamin K (phytonadione, Mephyton, Aquamephyton) can be given orally at a dose of 1 to
1

2.5 mg.
• If immediate hemostatic competence is necessary because of serious bleeding or profound warfarin
overdosage (INR > 20), adequate concentrations of vitamin K-dependent coagulation factors can be restored
by transfusion of fresh frozen plasma (10 to 20 ml per kg), supplemented with 10 mg of vitamin K , given by
1

slow intravenous infusion.

51. Ans. is d i.e. 5-6 mg/kg Ref. Goodman & Gilman 11/e, p 730

When the time to reach steady state is appreciable, as it is for drugs with long half-lives, it may be desirable to
administer a loading dose that promptly raises the concentration of drug in plasma to the target concentration.
Therapeutic concentrations are achieved with loading doses of about 5 mg/kg of theophylline (calculated as the
free base) and can be maintained with 2 mg/kg given every 12 or 24 hours.
Loading dose = Maintenance dose x Accumulation factor.

52. Ans. is a i.e. Allopurinol Ref. Harrison 16/e, p 1840; Harrison 17/e, p 1983

Among the given options, Allopurinol is the only drug which causes liver granuloma
Liver Granuloma Fatty Liver Cholestasis Necrosis of Liver
Carbamazepine Amiodarane Chlorpromazine Acetaminophen
Phenylbutazone Valproate Erythromycin Yellow phosphorus
Sulfonamides Tetracycline Chlorpropamide Carbon tetrachloride

List of drugs causing granulomatous hepatitis is given in answer no. 118 from Chapter 2

53. Ans. is a i.e. Anti TNF α antibody Ref. Harrison 16/e, p 1929; Harrison 17/e, p 1896

Anti TNF α antibody (infliximab) is used in Ulcerative colitis and Crohn’s disease
Except IL -11 (which is under trial) none of the IL antagonist is used in treatment of Crohn’s disease

TNF - α BLOCKING DRUGS • ADALIMUMAB • INFLIXIMAB • ETANERCEPT

INFLIXIMAB
Mechanism of action : – It is a chimeric (25% mouse, 75% human) monoclonal antibody that binds
with high affinity to soluble and possibly membrane-bound TNF-α.
– It complexes with solube TNF-α and prevents its interaction with p55 and
p75 cell surface receptors. This results in down-regulation of macrophage
and T cell function.

356
Phar macology of Respiratory
Pharmacology Respiratory,, Endocrinal & Gastr ointestinal System
Gastrointestinal 5
Indications : – Rheumatoid arthritis (MC indications) – Ulcerative colitis
– Psoriasis – Psoriatic arthritis
– Juvenile chronic arthritis – Wegener’s granulomatosis
– Giant cell arteritis – Sarcoidosis
Adverse Effects : – Upper respiratory tract infections
– Nausea, – Sinusitis
– Headache – Rash, Cough

54. Ans. is a i.e. Weakest acid found in living organism Ref. KDT 6/e, p 597

Heparin (anticoagulant) is the strongest acid in the body and carries electronegative charge in solution.
Contraindications and adverse effect of heparin are:
• Bleeding disorders, thrombocytopenia
• Severe hypertension (risk of cerebral hemorrhage)
• Subacute bacterial endocarditis (embolism)
• Tuberculosis (hemoptysis)
• Ocular and neurosurgery, lumbar puncture
• Chronic alcoholism, cirrhosis, renal failure
• Aspirin and other antiplatelet drugs should be used very cautiously during heparin therapy
Warfarin Heparin
Action Warfarin inhibits vitamin K Activate antithrombin III. The heparin-antithrombin
dependent factor II, VII, complex then inactivates factor Xa, IIa, IXa, XIIa,
IX, X, Protein C and S XIIIa but not factor VIIa.
Monitoring PT APTT
Antidote Ist is fresh frozen plasma Protamine sulphate
and 2nd is Phytonadione (K ) 1

55. Ans. is b i.e. β rays Ref. Schwartz 8/e, p 1421; William’s Endocinology 10/e, p 479; KDT 6/e, p 252

I is an effective agent for delivering high radiation doses to the thyroid tissue with low spillover to other portions
131

of the body.
I emits both X-rays and β rays. β rays are utilized for their destructive effects on thyroid cells and X-rays are
131

useful for tracer studies.

Radioiodine therapy
Indication Complications
• Patient at high risk for Acute : – Neck pain
mortality or recurrence – Vocal cord paralysis – Sialadenitis
• Incomplete tumour resection Chronic : – Leukemia & Bone marrow suppression
• FTC/PTC in age group < 16years – Pulmonary fibrosis
• Tall or collumnar cell variant of PTC – ↑ risk of cancer of - Lung, Breast, Bladder, Stomach

357
5 Self Assessment & Review Phar macology
Pharmacology
56. Ans. is a i.e. Demeclocyline Ref. Harrison 16/e, p 2103; Harrison 17/e, p 619, 2222-2223

Demeclocyline is used in the treatment of SIADH because it antagonises ADH action and reduces
urine concentrating ability of the kidney.
SIADH is characterized by increased vasopressin release leading to increased absorption of water producing a
concentrated or hyperosmolar urine. → WEAK ANDROGENS
SIADH (Syndrome of inappropriate ADH)
• The term SIADH is applied to conditions with vasopressin excess. Vasopressin excess is termed inappropriate
as this increase occurs despite decreased plasma osmolality resulting in hypervolemia, concentrated urine
and decreasing serum osmolality.
• Edema does not occur despite hypervolemia(due to unknown reasons).

Cardinal features of SIADH include :


• Hyponatremia (dilutional hyponatremia with Na+ < 135 mmol/l)
• Decreased plasma osmolality (<280 mosm/kg) with inappropriately increased urine osmolality > 150
mosm)
• Urine sodium over 20 meq/l
• Low blood urea nitrogen < 10mg/L
• Hypouricemia (< 4mg/L)
• Absence of cardiac, liver or renal disease
• Normal thyroid and adrenal function

A high BUN suggests a volume contracted state and excludes a diagnosis of SIADH
Drugs causing SIADH are
• Vasopressin or DDAVP
• Phenothiazines
• Chlorpropamide
• Tricyclic antidepressants
• Oxytocin, high, dose
• Monoamine oxidase inhibitors
• Vincristine
• Serotonin reuptake inhibitors
• Carbamazepine
• Nicotine

Treatment :
• Desmopressin has no role in the management of SIADH.
• In acute SIADH, the keystone of treatment is to restrict total intake of fluids to less than the sum total
of insensible losses and urinary output.
• If more rapid correction of hyponatremia is desired, the fluid restriction can be supplemented by intravenous
infusion of hypertonic saline.
• In chronic SIADH, the hyponatremia can be minimized by treatment with demeclocycline or fludrocortisone.
Another nonpeptide AVP (combined V2/V1a) antagonist, Conivaptan has been approved for short term in-
hospital treatment of SIADH

358
Phar macology of Respiratory
Pharmacology Respiratory,, Endocrinal & Gastr ointestinal System
Gastrointestinal 5
57. Ans. is a i.e. It increases the risk of coronary artery disease
Ref. KDT 6/e, p 301 - 302; CMDT ‘05, 1151; Harrison 16/e, p 2210

Hormone replacement therapy


Variable Effect
• Definite Benefits
– Symptoms of menopause - Definite Improvement
(Vasomotor)
– Osteoporosis - Definite increase in bone mineral density & decrease in fracture risk

• Definite Risk
– Endometrial cancer - Definite 8 to 10 fold increase in risk (with estrogen alone)
- No increase with use of estrogen plus progestin
– Venous thromboembolism - Definite increase in risk ≥ 2 fold
– Breast cancer - Increase in risk with long term use ≥ 5 year
• Probable and Uncertain Risk and benefit
– CVS disease - Probable decrease in risk (according CMDT, KDT, Goodman &Gilman)
- HRT increase risk of coronary artery disease (according to Harrison)
– Gallbladder cancer - Probable Increase in risk
– Colorectal cancer - Probable decrease in risk
– Cognitive dysfunction - Unproven decrease in risk
and dementia

58. Ans. is c i.e. Gynaecomastia Ref. KDT 6/e, p 571

Gynaecomastia is an important adverse effect and not a use of aldosterone antagonist (i.e. Spironolactone).
List of drug causing gynaecomastia given in answer no. 40

Sprionolactone :
Mechanism of action :
– Aldosterone acts on the late DT and CD cells by combining with mineralocorticoid receptor (MR)

Induces formation of Aldosterone induced protein (AIP)

AIP promotes – Na+ reabsorption – K+ excretion
Now,
– Spironolactone acts from the interstitial side of the tubular cell

– Combines with mineralocorticoid receptor

Inhibits formation of AIP in a competitive manner.
– It has no effect on Na+ and K+ transport in the absence of aldosterone.

359
5 Self Assessment & Review Phar macology
Pharmacology
USES : Spironolactone is a weak diuretic and can be used only in combination with other more efficacious
diuretics.
Edema : – It is more useful in cirrhotic and nephrotic edema because in these
conditions, aldosterone levels are generally high.
– It is breaks resistance to thiazide diuretics that develops due to secondary
hyperaldosteronism.
To counteract K+ loss due to – It antagonises K+ loss due to other diuretics, and
thiazide and loop diuretic : hence also known as K+ sparing diuretic.
Hypertension : – It is indicated only at low doses in combination with a thiazide to prevent
hypokalemia.
CHF : – It is used as an additional drug to conventional therapy in moderate to
severe CHF.
– It retards disease progression and lowers mortality.

Adverse Effects of Spironolactone


• Hyperkalemia : – Most serious side effect specially if renal function is inadequate
• Acidosis : – Particularly in cirrhosis
• Gynaecomastia
• Hirsutism
• Impotence
• Menstrual irregularities

59. Ans. is b i.e. Dihydrotestosterone Ref. KDT 6/e, p 288

The most potent androgen is Dihydrotestosterone.


Cholesterol

Pregnenolone

17(OH) Pregnenolone

Dihydroepiandrostenedione ⎫
↓ ⎬ → WEAK ANDROGENS
Androstenedione ⎭

TESTOSTERONE

TESTOSTERONE (5mg/d) →Excretion 90%

5α Reductase (6-8%) Aromatase (0.3%)


Dihydrotestosterone (Most Potent) Testosterone (Potent) Estradiol
• External genitalia formation • Wolffian duct derivatives • Hypothalamic/
in fetus (internal genitalia) pituitary feedback
• Prostate growth & BPH • Bone formation • Epiphyseal closure
• Acne • Muscle mass • Gynaecomastia
• Facial/body hair • Spermatogenesis • Vascular and
• Erythropoiesis behavioral effects

360
Phar macology of Respiratory
Pharmacology Respiratory,, Endocrinal & Gastr ointestinal System
Gastrointestinal 5
60. Ans. is c i.e. Carcinoma of gall bladder Ref. Harrison 16/e, p 2212; Sabiston 16/e, p 1021

Definite risk of estrogen therapy Probable risk of estrogen therapy


• Breast Ca. • CHD
• Endometrium Ca. • Gall bladder disease (Gall stones)
• Venous thromboembolism • Colorectal cancer
• Ovarian cancer
• Cognitive decline / dementia

“Hepatocellular carcinoma may occur with long term androgenic steroid administration with exposure
to thorium dioxide or vinylchloride and possibly with exposure to estrogens in OCP.” ..... Harrison
“The association of oral contraceptives with HCC is probably due to their role in causing hepatic adenoma to
develop. Adenoma will undergo malignant transformation is probably about 10% in 10 year period.”

61. Ans. is d i.e. Bromocriptine Ref. Harrison 16/e, p 548; KDT 6/e, p 297 - 303

• Prolactin is under the inhibitory control of hypothalamus through prolactin releasing inhibitory hormone
(PRIH).
• PRIH is in fact, dopamine which acts on pituitary lactotrope D receptor.
2

• Thus, dopamine and its agonists decrease prolactin level and vice versa.
DOPAMINE
Agonists Antagonists Depleters
Decrease Prolactin Increase Prolactin Increase Prolactin
• Dopamine • Chlorpromazine • Reserpine
• Bromocriptine • Haloperidol • Methyldopa
• Apomorphine • Metoclopramide
• Pergolide
• Piribedil
• Ropinirole
• Pramipexole

62. Ans. is d i.e. Hypoglycemia Ref. KDT 6/e, p 217

• Salbutamol is a selective β agonist.


2

• The major adverse effects of salbutamol are due to excessive activation of β adrenergic receptors.

Adverse effects of salbutamol


• Tachycardia • Skeletal muscle tremor • Glucose intolerance
• Hypokalemia • Hyperglycemia

All of these adverse effects are far less likely with inhalation therapy than with parenteral or oral therapy.

63. Ans. is a i.e. Chlorpropamide Ref. KDT 6/e, p 266 - 267

Sulphonylureas, especially Chlorpropamide induces dilutional hyponatremia by potentiating the effects of


antidiuretic hormone (ADH) on renal collecting duct.

361
5 Self Assessment & Review Phar macology
Pharmacology
– There are certain adverse effects which are seen only with chlorpropamide. Other sulphonylureas are very
less prone to cause them.
These are : • Cholestatic juandice
• Dilutional hyponatremia
• Disulfiram like reaction (intolerance to alcohol)
Also remember other side effects of sulphonylureas :
a. Hypoglycaemia : • It is the commonest problem with use of oral hypoglycaemics
• Chlorpropamide is the most frequent culprit
• Tolbutamide carries lowest risk due to its low potency and short duration of action
• Lower incidence also with glipizide, glibenclimide, glimepiride
• It is more common in elderly, lV and kidney disease patients
b. Hypersensitivity : • Rashes • Photosensitivity • Purpura
• Transient leucopenia • Agranulocytosis

Also remember : • Safety of sulphonylureas during pregnancy is not established →change over to insulin.
• They are secreted in milk →should not be given to nursing mothers.

64. Ans. is c i.e. Propylthiouracil Ref. KDT 6/e, p 250; Harrison 16/e, p 2116; Harrison 17/e, p 2236

All antithyroid drugs can cross the placental barrier and are concentrated by the fetal thyroid, so
there is always a risk of fetal hypothyroidism.
Drug of Choice is Propylthiouracil

because, it is more strongly protein- bound

therefore, crosses the placenta less readily and in low doses
• It is not secreted in sufficient quantity in breast milk to preclude breast feeding.
• The lowest effective dose of propylthiouracil should be used, and it is often possible to stop treatment in the
last trimester, because TSH- R antibodies decline in pregnancy.
• Carbimazole and Methimazole, if used during pregnancy may lead to :
– Fetal aplasia cutis
– Choanal atresia
• Radioactive I and surgery are contraindicated during pregnancy. Fetal thyroid will be destroyed →cretinism
2

You should also know the differences between propylthiouracil and carbimazole, because they are very very
frequently asked.
Propylthiouracil Carbimazole

• Dose to dose less potent. • About 3 times more potent


• Highly plasma protein bound (so, used in pregnancy) • Less bound
• Less transferred across placenta and milk • Larger amounts cross to fetus and in milk
• Plasma t½ – 1-2 hours • Plasma t½ – 6-10 hours
• Single dose acts for 4-8 hrs • Single dose acts for 12-24 hrs.
• No active metabolite • Produces active metabolite → methimazole

362
Phar macology of Respiratory
Pharmacology Respiratory,, Endocrinal & Gastr ointestinal System
Gastrointestinal 5
• Multiple (2-3) daily doses needed • Single daily dose required
• Inhibits peripheral conversion of T →T
4 3
• Does not inhibit conversion of T →T
4 3

• Used only in : – Thyroid storm • More commonly used.


– Pregnancy
– Patient with allergy to carbimazole

65. Ans. is d i.e. Dopamine Ref. Harrison 16/e, p 548; KDT 6/e, p 297 - 303

Already explained, refer answer no . 61

66. Ans. is d i.e. Fibrinogen levels are increased by pravastatin


Ref. KDT 6/e, p 615; Goodman & Gilman 11/e, p 987

Option ‘d’ : It is clearcut that this option is incorrect because fibrinogen levels are decreased by pravastatin.
Looking at the other options :
Option ‘a’ : • Lovastatin and simvastatin are lipophilic i.e. their CNS penetration is better.
• Pravastatin and fluvastatin are hydrophilic.
Option ‘b’ : • This option is slightly controversial because all the statins undergo rapid first pass metabolism.
• But Goodman/Gilman also says “Pravastatin is excreted without extensive metabolic
modification”.
• So, this option can be true if we look from this angle.
Option ‘c’ : • Absorption of the statins is enhanced by taking the dose with food.....Katzung 9/e, p 568
• With the exception of pravastatin whose absorption is minimally affected.

67. Ans. is b i.e. Bronchial asthma Ref. KDT 6/e, p 223; Harrison 16/e, p 1513

5-Lipoxygenase inhibitor
• Zileuton
– Blocks enzyme lipooxygenase

– Decrease LTC /LTD /LTB synthesis
4 4 4


– Decrease features of bronchial asthma

68. Ans. is c i.e. Testosterone Ref. Harrison 16/e, p 548; KDT 6/e, p 272, 278

• The answer is obvious because increased testosterone levels play a causative factor in prostatic cancer, so
it cannot be used in the treatment.
• The aim is to either decrease testosterone levels or decrease testosterone production leading to chemical
castration.

363
5 Self Assessment & Review Phar macology
Pharmacology
Drugs used in Prostatic Cancer :
First Line Second Line
• Estrogens – Diethylstilbesterol (DES) • Ketoconazole
• Anti androgens – Flutamide • Aminoglutethimide
– Bicalutamide
– Nilutamide
• GnRH analogues – Leuprolide acetate
– Goserelin acetate
• α Reductase inhibitor
5-α – Finasteride

Important point of Finasteride :


• A selective inhibitor of 5α reductase type 2 isoenzyme which predominates in male urogenital tract.

Prevents conversion of testosterone to the more active compound, dihydrotestosterone.

69. Ans. is c i.e. Montelukast Ref. KDT 6/e, p 222; Harrison 16/e, p 1513; Harrison 17/e, p 1602-1604

Drugs used in Acute Asthma :


β Agonists short acting
2
Anticholinergics Systemic corticosteroids
– Salbutamol (Albuterol) – Ipratropium bromide – Hydrocortisone
– Terbutaline – Methylprednisolone
– Bambuterol

“Systemic or oral steroids are most beneficial in acute illness, when severe airway obstruction is not
resolving or is worsening despite intense optimal bronchodilator therapy.” ..... Harrison 16/e, p 1513
Drugs used in Chronic Asthma:
Systemic glucocorticoids Inhaled glucocorticoids Leukotriene modifiers
– Methylprednisolone – Beclomethasone – Flunisolide – Montelukast
– Prednisolone – Fluticasone – Triamcinolone – Zafirlukast
– Prednisone – Budesonide – Zileuton

Mast cell stabiliser Long acting inhaled β - agonists


2
MethylxanthinesCombined medication
– Cromolyn sodium – Salmeterol – Theophylline – Fluticasone/
– Nedocromil sodium – Formoterol Salmeterol
– Ketotifen

70. Ans. is b i.e. Acts by increased insulin secretion Ref. KDT 6/e, p 267, 269; Harrison 16/e, p 2176 - 2177

Already explained, refer answer no. 19

71. Ans. is b i.e. Morphine Ref. KDT 6/e, p 30

Morphine, in contrast, binds only 30% to plasma proteins. All others in the options are bound 90-99% to plasma
proteins.

364
Phar macology of Respiratory
Pharmacology Respiratory,, Endocrinal & Gastr ointestinal System
Gastrointestinal 5
• The patient is having nephrotic syndrome and hypoalbuminemia.
• So, the drugs which are highly bound to plasma protein albumin, will be affected the most.
• Now, look at the drugs binding to albumin :
– Barbiturates – Benzodiazepines – NSAIDs – Valproic acid – Phenytoin
– Penicillins – Sulphonamides – Tetracyclines – Tolbutamide – Warfarin

Also remember that : • Acidic drugs bind to albumin • Basic drugs bind to α - acid glycoprotein
1

72. Ans. is b i.e. Enalapril Ref. KDT 6/e, p 615

Statin induced myopathy is increased by : Drug causing myopathy


• Erythromycin C Chloroquine, Clofibrate, Cimetidine
• Gemfibrozil H HMG CoA reductase inhibitors
• Cyclosporine I Interferon α
• Nicotinic acid L L-asparaginase
• Ketoconazole G Glucocorticoid
O OCS
Z Zidovudine
A Amphoterecin-B

73. Ans. is a i.e. 5 lipooxygenase inhibitor Ref. KDT 6/e, p 223; Harrison 16/e, p 1514

• Zileuton is 5-lipooxygenase inhibitor and is a modest bronchodilator that reduces asthma morbidity,
provides protection against exercise induced asthma and diminishes nocturnal symptoms, but it has
limited effectiveness against allergens.
• 5 - Lipooxygenase : It is an enzyme involved in production of particularly LTB (potent chemotactic), LTC ,
4 4

LTD and LTF , LTE . These are most important mediators of inflammatory reactions occuring in asthma.
4 4 4

74. Ans. is a i.e. 5 Amino salicylic acid Ref. Goodman & Gilman 11/e, p 1012; Harrison 16/e, p 1784

Sulfasalazine is the parent compound of 5 - Aminosalicyclates or mesalamine.


Sulfasalazine :

5 ASA Sulfapyridine
Mesalamine Responsible for adverse effects
Main active therapeutic moiety of this preparation
against inflammatory bowel disease

365
5 Self Assessment & Review Phar macology
Pharmacology
• Though 5 ASA is the main active therapeutic moiety, it is not used alone because when given orally
(alone) it is : – Poorly absorbed
– Inactivated before reaching colon.
Therefore, it is given along with sulfapyridine as sulfasalazine, which prevents its early absorption and inactivation.

75. Ans. is b i.e. Hyperkalemia Ref. KDT 6/e, p 285; Katzung 10/e, p 645

When given in greater than physiologic amounts, steroids such as cortisone and hydrocortisone, which have
mineralocorticoid effects in addition to glucocorticoid effects, causes some sodium and fluid retention and loss
of potassium. This leads to a hypokalemic hypochloremic alkalosis.

76. Ans. is d i.e. Decrease incidence of hyperbilirubinemia Ref. Katzung 10/e, p 644

Lung maturation in fetus is regulated by the fetal secretion of cortisol. Treatment of the mother with large doses
of glucocorticoid reduces the incidence of respiratory distress syndrome (RDS) in infants delivered prematurely.
Dose : When delivered before 34 weeks of gestation, IM betamethasone 12mg is given followed by an additional
dose of 12mg 18 - 24 hours later.
• Intraventricular hemorrhage is common among very low birth weight infants.
• It is also given in preterm premature rupture of membrane at less than 32 week gestation.
• Contraindication of prenatal corticosteroids is chorioamnionitis

77. Ans. is a i.e. Effective only after total pancreatectomy Ref. KDT 6/e, p 266

• Presence of at least 30% functional pancreas is necessary for the action of oral hypoglycemics (Sulfonylureas).
• They do not cause hypoglycemia in pancreatectomised animals and in type-1 diabetics. This confirms their
indirect action through pancreas.
• Both phenformin and metformin causes lactic acidosis (phenformin > metformin).

78. Ans. is d i.e. 6-8 days Ref. Various Books

Oral anticoagulants are stopped 1 week before surgery and low molecular weight heparin is started. Omit the
morning dose of heparin on the day of surgery.
Drugs/conditions which are stopped/modified before surgery.
Mnemonics = SEMLA : S Smoking – 6 week before
E Estrogen pills – 4 week before
M MAO-A irreversible – 3 week before
L Lithium – 2 week before
A Aspirin – 1 week before

All hypoglycemic drugs


Minor surgery Major surgery

< 20 min Shift to insulin 1 week


omit morning dose before (omit morning dose)

366
Phar macology of Respiratory
Pharmacology Respiratory,, Endocrinal & Gastr ointestinal System
Gastrointestinal 5
A pateint undergoing surgery is given intraoperative steroid replacement, if he has taken steroids for
more than one week in the last one year.
No modification :
– Antianginal drugs – Antitubercular drugs – Antihypertensive drugs
– Antithyroid drugs – Levodopa – Progesterone only pill

79. Ans. is c i.e. Terbutaline Ref. KDT 6/e, p 218

Short acting : Selective β agonists are, terbutaline, salbutamol, bitolterol, remiterol and pirbuterol.
2

Long acting : Salmeterol, bambuterol and formoterol.


Adrenaline now rarely used for bronchial asthma.

80. Ans. is b i.e. Cyproterone acetate Ref. KDT 6/e, p 293 - 294

Cyproterone acetate is a potent antiandrogen. It competes with dehydrocorticosterone for the intracellular
androgen receptor and inhibits its binding.

Adverse effects : It can cause gynaecomastia.


Option ‘a’ : Tamoxifen – It is selective estrogen receptor modulator (SERMs).
Option ‘c’ : Mifepristone – It is nonsteroidal antiprogestational with significant antiglucocorticoid as
well as antiandrogenic activity.
Option ‘d’ : Nalondrone – These are synthetic androgens with higher anabolic and lower androgenic
activity.

81. Ans. is a i.e. D-penicillamine Ref. KDT 6/e, p 332 - 333

D-penicillamine has no role in the treatment of hypercalcemia.


Treatment of Severe Hypercalcemia

• Rehydration with normal saline


• Forced diuresis, saline plus loop diuretic
• Bisphosphonates – Etidronate, pamidronate, alendronate
• Calcitonin
Special therapy used

• Glucocorticoids • Gallium nitrate


• Phosphates • Dialysis
• Plicamycin

82. Ans. is a i.e. Phenformin Ref. KDT 6/e, p 267; Harrison 16/e, p 2176 - 2177

Adverse Effects : • GI side effects in 20% of patients and does not cause hypoglycemia
• Lactic acidosis most common and most serious
• Vitamin B deficiency (Megaloblastic anaemia)
12

• Alcohol ingestion precipitates severe lactic acidosis.

367
5 Self Assessment & Review Phar macology
Pharmacology
83. Ans. is a and c i.e. Glargine; and Detemir
Ref. Katzung 10/e, p 689; Harrison 17/e, p 2297; Goodman & Gilman’s 11/e, p 1626-1627

Features of Insulin Glargine


• It is the first long acting analog of human insulin.
• It is a clear solution of pH 4.0. This pH stabilizes the insulin hexamer and results in a prolonged and
predictable absorption from subcutaneous tissues.
• It cannot be mixed with short-acting insulin preparations due to its acidic pH.
• It results in less hypoglycemia, has a sustained peakless absorption profile, provides a better once
daily 24-hour insulin coverage than ultralente or NPH insulin.
• Injection of Glargine forms microfined precipitate in subcutaneous tissue as the pH rises from 4.0 to 7.4. As
a result, a sustained release of insulin takes place from the site of injection which mimics the basal secretion.

Features of Insulin Detemir


• Insulin detemir has the most reproducible effect of the intermediate and long-acting insulins, and its use is
associated with less hypoglycemia than NPH insulin. Insulin detemir has a dose-dependent onset of action
of 1-2 hours and duration of action of more than 24 hours. It is given twice daily to obtain a smooth background
insulin level.

For more details, refer answer no. 3

84. Ans. is a, c and e i.e. Antimicrobials; Anticancer drugs; and Gold Ref. Harrison 16/e, p 1273

Drugs causing Metallic taste are :

• Metronidazole • Penicillamine
• Rifampicin • Griseofulvin
• Acetazolamide • Lithium
• Biguanide
• Captopril
Mnemonic : MR. ABC PG Li

85. Ans. is a, b and c i.e. Thiazide diuretics; β -blockers; and Verapamil


Ref. Goodman & Gilman 11/e, p 1633; Harrision 16/e, p 2153

Already explained, refer answer no. 43

86. Ans. is c i.e. ↑ Lipocortin Ref. KDT 6/e, p 48

Drugs inhibiting the biosynthetic pathway of prostaglandins and leukotrienes as depicted in flow chart :

368
Phar macology of Respiratory
Pharmacology Respiratory,, Endocrinal & Gastr ointestinal System
Gastrointestinal 5

87. Ans. is a and b i.e. Fluticasone; and Budesonide Ref. KDT 6/e, p 225 - 226

Glucocorticoid by inhibiting phospholipase A and by inducing lipocortin they act as an antiinflammatory agent
2

and reduced inflammation in air ways.

88. Ans. is a and b i.e. Ketoconazole; and Aminoglutethimide Ref. Harrison 16/e, p 2141; 17/e, p 2263

Drugs causing Addison’s disease are :


• Metyrapone, ketoconazole, aminoglutethimide
• Cytotoxic drugs (mitotane)

89. Ans is b i.e. It blocks LTB receptor


4
Ref. KDT 6/e, p 222; Harrison 16/e, p 1514; Harrison 17/e, p 1604

Zafirlukast is LTD4 receptor antagonist used in prophylactic therapy for mild to moderate asthma.
LEUKOTRIENE ANTAGONISTS
Leukotriene receptor antagonist 5-Lipoxygenase inhibitor
• Montelukast • Zileuton
• Zafirlukast
– Competitively blocks leukotriene receptors – Blocks enzyme lipooxygenase
↓ ↓
– Bronchodilatation ↓ sputum eosinophil count – Decrease LTC /LTD /LTB synthesis
4 4 4

↓ inflammation and reactivity ↓


– Indicated for prophylactic therapy of mild to – Decrease features of asthma
moderate asthma

– Not used in acute asthma

369
5 Self Assessment & Review Phar macology
Pharmacology

Montelukast and • Indication : – Prophylaxis of mild to moderate asthma


Zafirlukast • Efficacy of Zafirlukast is less than that of steroids (option ‘e’)
• It is not effective in terminating acute episode (option ‘d’)
• Pharmacology – Well absorbed orally (option ‘c’)
– t1/2 - Montelukast - 3-6 hours
– t1/2 - Zafirlukast - 8-12 hours
• Adverse affect – Headache, Rashes
– Eosinophilia, Neuropathy
– Churg strauss syndrome (vasculitis and eosinophilia)

90. Ans. is a and b i.e. Sibutramine; and Orlistat


Ref. KDT 6/e, p 128 - 131; Harrison 16/e, p 429; Harrison 17/e-464

Already explained, refer answer no. 5

91. Ans. is b and c i.e. Budesonide; and Fluticasone and salmeterol combination
Ref. KDT 6/e, p 223, 225; Harrison 16/e, p 1513

Cromolyn sodium : It is not a bronchodilator and does not antagonize constrictor action of histamine, Ach, LTs
etc. Therefore, it is ineffective if given during an asthmatic attack. ... KDT 6/e, p 223
• Budesonide and fluticasone both are inhalational steroid and inhaled steroids have no role during an acute
attack or in status asthmaticus. ..... KDT 6/e, p 225
• Further, salmeterol is used for maintenance therapy and for nocturnal asthma but not for acute symptoms.

Drugs used in Acute Asthma :


β Agonists short acting
2
Anticholinergics Systemic corticosteroids
– Salbutamol (Albuterol) – Ipratropium bromide – Hydrocortisone
– Terbutaline – Methylprednisolone
– Bambuterol

92. Ans. is a and b i.e. Ezetimibe; and Orlistat


Ref. Katzung 10/e, p 571, 1041; Harrison 16/e, p 2279; Harrison 17/e, p 2427-2428

• Ezetimibe is the first member of a new group of antilipidemic drugs that inhibit intestinal absorption of
cholesterol and phytosterols.
• Other important points about Ezetimibe :
– It is readily absorbed and conjugated in the intestine to an active glucoronide with half life of 22 hours.
– It undergoes enterohepatic circulation, 80% of the drug is excreted in faeces.
– It is effective even in the absence of dietary cholesterol because it inhibits reabsorption of cholesterol
excreted in the bile.
– Its primary clinical effect is reduction of LDL cholesterol.
– Average reduction in LDL with ezetimibe is about 18%, with minimal increases in HDL cholesterol.
– It is apparently synergistic with reductase inhibitors, producing decrements as great as 25% in LDL
cholesterol beyond that achieved with reductase inhibitor alone.

370
Phar macology of Respiratory
Pharmacology Respiratory,, Endocrinal & Gastr ointestinal System
Gastrointestinal 5
• Orlistat is an antiobesity drug. It is an inhibitor of gastric and pancreatic lipase; it interferes with digestion
and absorption of dietary triglycerides. Absorption of cholesterol and fat soluble vitamins is also impaired.
• Statins act by inhibiting HMG - CoA reductase.
• Cholestyramine is a bile acid sequestrant.

93. Ans. is b i.e. To counteract constipating effect of aluminium Ref. KDT 6/e, p 634 - 635

Mnemonic : Magnesium causes Megastool

• Al+ salts are constipating • Mg ++ salts are laxatives • CaCO3 is constipating

94. Ans. is a and b i.e. Action is faster and short induration than regular insulin; and It is given 15 minutes
prior to meal Ref. Goodman & Gilman 11/e, p 1625; KDT 6/e, p 559

• Lispro insulin is a rapid acting insulin analogue produced by recombinant DNA technology.

Difference between Regular insulin and Lispro insulin :


Features Regular insulin Lispro insulin
• Source Human Human or Pork
• Onset 30 - 60min 10 - 20min
• Chemistry Exist as hexamer. Do not Exist as hexamer but dissociates
dissociate immediately into instantaneously into monomers
monomers
• Absorption Less absorption as compared 3 times more rapid absorption than
to lispro insulin regular insulin from subcutaneous sites
• Duration of Action 5 - 8 hours 3 - 4 hours
• Glycemic control Achieved when drug is given Achieved when drug is given 15
30 minutes before a meal minutes before a meal
• Comment Clear solution; can be mixed with Clear solution; can be given I.V.;
all types of insulins given S. C. or I. V. can be mixed with all types of insulins

Theraputic advantages of lispro insulin as compared to regular insulin :


• The prevalence of hypoglycemia is reduced by 20-30%.
• Glucose control, as assessed by HbA , is modestly but significantly improved (0.3-0.5%).
1c

Future insulins : • Lispro insulin • Aspart insulin • Glargine insulin

95. Ans. is a i.e. Danazol Ref. Harrison 16/e, p 521; Harrison 17/e, p 720

Danazol with weak progestational, androgenic and glucorticoid activities is used to suppress ovarian
function.
Agents used for hormonal management of metastatic breast cancer
Drugs Action
• Tamoxifen – Antiestrogen
• Megestrol acetate – Progestin
• Diethylstilbesterol (DES) – Estrogen
• Anastrozole – Aromatase inhibitors

371
5 Self Assessment & Review Phar macology
Pharmacology
Endocrine therapies for Breast cancer
Therapy Comments
• Castration – For Premenopausal women
– Surgical
– LHRH agonists
• Antiestrogens – Useful in pre and postmenopausal women
– Tamoxifen
• Pure antiestrogens – Promising early clinical response in tamoxifen resistant patient
• Surgical adrenalectomy – Rarely employed, second line choice
• Aromatase inhibitors – Low toxicity and superior to additive hormone therapy : now,
first choice for metastatic disease
• High dose progesterones – Common third choice
• Hypophysectomy – Rarely used

96. Ans. is b, d and e i.e. Glimeperide; Rosiglitazone; and Repaglinide


Ref. CMDT ‘05, p 1172; KDT 6/e, p 268

Meglitinide are nonsulphonylurea, contains repaglinide and nateglinide. Repaglinide undergoes complete
metabolism in liver, therefore useful in patient with renal impairement or in elderly.
Groups Mechanism of Action Example Clearance Contraindication
Secretagogue ↑ insulin release Chlorpropamide Renal Renal/liver disease
Sulfonylurea Tolbutamide Hepatic/Renal
Non-sulfonylurea Insulin release
Meglitinide Repaglinide
Nateglinide
Biguanides ↓ Hepatic glucose Metformin Serum creatinine
production, weight loss > 1.5mg/dl “men”
↑ glucose utilization > 1.4mg/dl “women”
↓ insulin resistance
α glucosidase ↓ Glucose absorption Acarbose Renal/liver disease
inhibitors Miglitol
Thiazolidinedione ↓ insulin resistance Rosiglitazone Liver disease, CHF
↑ glucose utilization Pioglitazone

97. Ans. is a and e i.e. ↓ Chemoreceptor trigger zone (CTZ); and 5HT agonist 4

Ref. KDT 6/e, p 640; Rang & Dale 5/e, p 374

372
Phar macology of Respiratory
Pharmacology Respiratory,, Endocrinal & Gastr ointestinal System
Gastrointestinal 5

98. Ans. is b, c, d and e i.e. Synthetic derivative; Has alfa - blocking action; Decrease GI motility; and Acts
on both D and D receptors
1 2
Ref. KDT 6/e, p 236
• It is a potent dopamine agonist.
• It has greater action on D receptors, while at certain dopamine sites in brain, it acts as a partial agonist or
2

antagonist of D receptor.
1

• It is a weak α blocker.
Actions :
Strong antigalactopoietic Acromegaly → Decrease GH release
– Decrease prolactin release from pituitary – In contrast, in normal persons, it ↑ es GH release
Anti parkinsonian Nausea and vomiting
– Has levodopa like actions in CNS – By stimulating dopaminergic receptors in CTZ
Decrease g.i.t. motility Hypotension α blocking

99. Ans. is a, b and e i.e. A vitamin A analogue; Used in cystic acne; & Bony hyperostosis is a side effect
Ref. KDT 6/e, p 854

• Isotretinoin (accutane) is a synthetic vitamin A analogue with restricted use for the treatment of severe cystic
acne that is resistant to standard therapies.

373
5 Self Assessment & Review Phar macology
Pharmacology
• It inhibits production of sebum (skin bacteria decrease secondarily).
• It is absolutely contraindicated in pregnancy, is highly teratogenic.

Upto 25% of exposed fetus develop Accutane embryopathy (craniofacial, heart and CNS abnormalities)

Side effects : – Hypervitaminosis A – Corneal opacities


– Inflammatory bowel disease – Anorexia
– Alopecia – Headache, muscle and joint pains

• All the above side effects are reversible.


• It also causes : – Skeletal hyperostosis (premature closure of epiphysis in children) (Option ‘e’)
– Increased lipids (triglycerides, HDL)

100. Ans. is b, c and d i.e. Acts on myoepithelial cells of breast; Causes contraction of uterus during
labour; and May cause retention of water Ref. KDT 6/e, p 319 - 320

Oxytocin :
• Oxytocin is an octapeptide secreted by the posterior pituitary along with ADH.
• Oxytocin and ADH are synthesized within the nerve cell bodies in supraoptic and paraventricular nuclei of
hypothalamus.
Actions of Oxytocin
Uterus Breast CVS Kidney
• ↑ ↑ses force and Contracts myoepithelium • Conventional • In high doses
frequency of contractions of mammary alveoli doses → no effect → ADH like action
• With low doses, full ↓ • Higher doses ↓
relaxation occurs in forces milk into bigger → vasodilation by mild water retention
between, unlike in milk sinusoides → K/a direct action ↓
higher doses Milk Ejection Reflex Urine output ↓↓
• Estrogens sensitise (initiated by suckling) ↓
uterus to oxytocin by may prepicitate Pulm.
increasing no. of edema if large doses
oxytocin receptors of Oxytocin and i.v.
• Non pregnant and early fluids infused together
pregnant uterus are
nearly resistant to oxytocin
• Sensitivity to oxytocin ↑ es
progressively during
3 trimester
rd

Falls sharply during


puerperium
• Increased contractility is
restricted to fundus and
body; lower segment not
contracted unlike ergome-
trine & methyl ergometrine

374
Phar macology of Respiratory
Pharmacology Respiratory,, Endocrinal & Gastr ointestinal System
Gastrointestinal 5
101. Ans. is a, b and c i.e. Cyproterone; Spironolactone; and Cimetidine Ref. KDT 6/e, p 293 - 294

Androgen antagonists include


• Cyproterone acetate
• Flutamide
• Bicalutamide
• Ketoconazole : Inhibits androgen production
• Cimetidine : Inhibits androgen action
• Spironolactone

102. Ans. is a and b i.e. Decreases sensitivity of bronchial epithelium; and Decreases inflammatory response
Ref. KDT 5/e, p 206

Steroids in asthma benefit by


• Reducing bronchial hyperreactivity
• Reducing mucosal oedema
• Suppressing inflammatory response to antigen - antibody reaction or other trigger stimuli
• Steroids are neither bronchodilators nor do they increase action of bronchodilators.

103. Ans. is a and b i.e. Acute asthma; and Premature labour Ref. KDT 6/e, p 127

• Salbutamol is a selective β - agonist. 2

• It produces bronchodilatation, vasodilation and uterine relaxation without producing significant cardiac
stimulation.
• It has β : β action ratio or 10 : 1
2 1

Uses of Salbutamol : • Acute asthma


• As uterine relaxant to delay premature labour : Ritodrine is the preferred drug
Option ‘d’ : • Incorrect because the main drugs used in asthma prophylaxis are :
– Most cell stabilisers (Na -chromoglycate)
+

– Glucocorticoids

104. Ans. is b and d i.e. Migraine; and Thromboembolism Ref. Dutta 6/e, p 543; KDT 6/e, p 317

CONTRAINDICATION OF ORAL PILLS


A. Absolute- Contraindications
Circulatory Liver diseases Others
• Arterial/venous thrombosis • Active liver disease • Pregnancy
• Severe hypertension • Liver adenoma, carcinoma • Undiagnosed UTI
• Valvular heart disease, • History of cholestatic • Breast cancer (estrogen
ischemic heart disease, angina jaundice in pregnancy development)
• Hyperlipidaemia
• Focal migraine

375
5 Self Assessment & Review Phar macology
Pharmacology

B. Relative - Contraindication
• Obesity • Varicose vein • Epilepsy
• Asthma • Depression/Mood changes • Age > 35 year
• Smoking • Gall bladder disease • Nursing mother the first six months

Side effects of
PROGESTERONE ESTROGEN
Pr - Prolongation of anovulation and infertility after T/t E – Endometrial carcinoma
O - ↑ Ovarian cyst S – Stroke
G - Gain in weight T – Thromboembolism
Ester - Ectopic pregnancy R – Rectal/ Colo-rectal carcinoma
One - Osteoporosis O – Ovarian cancer
+ - Irregular menstrual bleeding Ge – Gall bladder and liver disease
N – DemeNtia
+ – Breast carcinoma

105. Ans. is b i.e. Cimetidine Ref. KDT 6/e, p 27

Cytochrome P-450 is inbibited by : • Cimetidine • Ciprofloxacin


• Oral contraceptive • Allopurinol
• Erythromycin • Valproate

106. Ans. is a i.e. Binds to estrogen receptors Ref. KDT 6/e, p 304

Tamoxifen is a selective estrogen receptor modulator (SERM). It is act as potent estrogen antagonist in breast
carcinoma cells, blood vessels and some peripheral sites, but as partial agonist in uterus, bone, liver and
pituitary.

107. Ans. is a i.e. Lovastatin Ref. KDT 6/e, p 615

Effects of Statins, Niacin and Gemfibrozil on the synthesis of lipids.

376
Phar macology of Respiratory
Pharmacology Respiratory,, Endocrinal & Gastr ointestinal System
Gastrointestinal 5
108. Ans. is b i.e. Porphyria Ref. KDT 6/e, p 397; Harrison 16/e, p 2306; Harrison 17/e 2439

Acute intermittent Porphyria : Barbiturates exacerbate it by inducing microsomal enzymes and δ amino
levulinic acid synthetase and increasing porphyrin synthesis.

Contraindications of barbiturates : • Liver and kidney disease


• Severe pulmonary insufficiency e.g. emphysema
• Obstructive sleep apnoea.

109. Ans. is a i.e. Conversion of 25 to 1, 25 dihydroxycholecalciferol Ref. KDT 6/e, p 331

Vitamin D is of following types :


D3 : Cholecalciferol – synthesized in skin under the influence of UV rays.
D2 : Calciferol – present in irradiated food – yeasts, fungi, bread, milk.
D1 : Mixture of antirachitic substances found in food – only of historic interest.

110. Ans. is a and c i.e. Thyrotropin; and Estradiol Ref. KDT 6/e, p 48, 232

Drug acting on Drugs acting at Drugs acting on


cell membrane receptors cytoplasmic receptor nuclear receptor
Intracellular Intracellular
IP -DAG
3
Activation of c-GMP
Tyrosine kinase
• Adrenergic α 1
• Insulin • ANP Steroid Thyroid hormones
• Histamine-H 1
• IGF- I • BNP – Glucocorticoids – Thyroxine
• Muscarine-M ,M 1 3
• GH (Nesiritide) – Mineralocorticoids – Triiodothyronine
• 5-HT 2
• Prolactin • EDRF – Estrogens
• Vasopressin V 1
• Nitric oxide – Progestins
• Oxytocin – Androgens
• Bradykinin–B2 – Calcitriol
• Angiotensin
• Prostaglandin FP
• EP 1

• EP 3

377
5 Self Assessment & Review Phar macology
Pharmacology

Adenyl cyclase
↑ c-AMP ↓c-AMP

• Adrenergic-β • Adrenergic -α 2

• Histamine-H 2
• Muscarinic-M 2

• Dopamine-D 1
• Dopamine-D 2

• Glucagon • 5HT1
• TSH/FSH/LH • GABA B

• ACTH • Opioid-μ,δ
• Prostaglandin-EP 2
• Angiotensin
• Prostacyclin-IP • Prostaglandin EP3
• Adenosine-A 2
• Somatostatin
• Adenosine-A 1

111. Ans. is a i.e. ↓ osteoid formation Ref. KDT 6/e, p 277 - 278, 285 - 286

Glucocorticoids causes decreased osteoid formation and increased resorption of bone.


• It inhibits intestinal absorption of Ca and enhance renal excretion of Ca .
++ ++

• Loss of calcium from bone is indirectly due to loss of osteoid.


• Spongy bones (vertebrae, ribs etc.) are more sensitive.

112. Ans. is a i.e. Ethionamide Ref. Harrison 16/e, p 950, 2109; KDT 6/e, p 743

Adverse effects of ethionamide


• Intense gastrointestinal intolerance (anorexia, vomiting) • Serious neurological reactions
• Hypothyroidism • Reversible hepatitis (5% of cases)
• Hypersensitivity reactions

Drugs causing hypothyroidism


• Iodine excess • Antithyroid drugs
• Iodine contrast media • p-aminosalicylic acid
• Amiodarone • Interferon α and other cytokines
• Lithium • Aminoglutethimide

113. Ans. is c and d i.e. Osmotic laxative and Blood ammonia Ref. CMDT’ 05, p 525

Lactulose is an osmotic laxative and is used to soften stools. They are commonly employed in older
nonambulatory patients to prevent constipation and fecal impaction.
• It is neither digested nor absorbed in gut.
• Lactulose decreases the blood NH concentration by 25-50% in patient with hepatic encephalopathy.
3

378
Phar macology of Respiratory
Pharmacology Respiratory,, Endocrinal & Gastr ointestinal System
Gastrointestinal 5
114. Ans. is b i.e. Adenoma

Absolute- Contraindications
Liver diseases : • Active liver disease
• Liver adenoma, carcinoma
• History of cholestatic jaundice in pregnancy

115. Ans. is a i.e. Allopurinol

Granulomatous hepatitis is caused by


Antiinflammatory Antibiotics Xanthine oxidase Anticonvulsant Antiarrythimic/
inhibitor antihypertensive

• Phenylbutazone • Sulfonamide • Allopurinol • Carbamazepine • Qunidine


• Isoniazid • Diltiazem
• Hydralazine
• Amiodarone

116. Ans. is a i.e. Mast cell stabilisation Ref. KDT 6/e, p 222-223

Mast cell stabilisation


• Sodium cromoglycate (cromolyn sodium)
• Nedocromil sodium
• Ketotifen
Mechanism of Action
• Inhibits degranulation of mast cells by trigger stimuli.
• Inhibit chemotaxis of inflammatory cells.
• Not a bronchodilator, not used in acute asthma.
Uses
• Bronchial asthma
• Allergic rhinitis
• Allergic conjunctivitis

117. Ans. is a i.e. ↑ Insulin release from pancreas Ref. KDT 6/e, p 267, 269; Harrison 16/e, p 2176 - 2177

Classification of Biguanides :
• Phenformin
• Metformin

Differences from Sulphonylureas :


• No hypoglycemia in normal and non diabetic subjects
• Do not stimulate pancreatic β cells to release insulin

379
5 Self Assessment & Review Phar macology
Pharmacology
Mechanism of action :
• They do not stimulate pancreatic β cells to cause insulin release.
• Suppress hepatic gluconeogenesis and glucose output from the liver : the major action.
• Enhance insulin mediated glucose disposal in muscle and fat (so, insulin is necessary for its action).

Do not alter translocation of GLUT4 (the major glucose transporter).

Increase GLUT1 transport from intracellular site to plasma membrane (thus, effect differs from insulin).

• Interfere with mitochondrial respiratory chain.

Promote peripheral glucose utilisation by enhancing anaerobic glycolysis (Phenformin, more prominent in
this effect).
• Inhibit intestinal absorption of glucose, hexoses, vitamin B12, amino acids.
In long term, when type 2 diabetes mellitus enters phase of relative insulin deficiency, insulin is
ultimately required.

380
Phar macology of Respiratory
Pharmacology Respiratory,, Endocrinal & Gastr ointestinal System
Gastrointestinal 5
CHAPTER REVIEW
• This section includes questions of V arious Other PGMEES from 1990 – 2008.
Various
• Questions are ar ranged in increasing order of page sequence of KDT 6 Edition. This is
arranged
done to mak e refer
make ral system more easy and uncomplicated to save the precious time
referral
of PGMEE Aspirant.

1. False statement about lactulose : (Manipal 07) 5. Neither addictive nor constipation : (HP 06)
a. Not effec tive in opium induced constipation a. Codeine
b. Flatulence is common b. Dextromethorphan
c. Not absorbed c. Diphenoxylate
d. Osmotic purgative d. Levophenol
[Ref. KDT 6/e, p 655-656] [Ref. KDT 6/e, p 215]
2. All of the following statement about an alpha 6. The antagonism of leukotriene is an example of :
glucosidase inhibitor are true except : a. Pharmacological antagonism (COMEDK 05)
a. Reduces intestinal absorption of carbohydrates b. Physical antagonism
b. Effective in both type 1 and 2 serious side effect c. Physiological antagonism
c. Hypoglycemia is a common and serious side d. Chemical antagonism
effect (Manipal 06) [Ref. KDT 6/e, p 217]
d. Can be used with other oral hypoglycemic 7. Which of the following leucotrine is used in
agents treatment of Asthma : (SGPGI 05)
[Ref. KDT 6/e, p 268-270] a. Zilutin
3. Which one of the following statements about b. Zafirlukast
biguanides is not true ? (Manipal 06) c. Cromoglycate
a. Do not stimulate insulin release d. Ipratropium
b. Decrease hepatic glucose production [Ref. KDT 6/e, p 222]
c. Renal dysfunction is not a contraindication for 8. The drug NOT used in acute asthma is:
their use
a. Salbutamol (MAHE 05)
d. Can be combined with sulfonylureas
b. Ipratorpium
hypooglycemic agents
c. Monteleukast
[Ref. KDT 6/e, p 267, 270]
d. Hydrocortisone
4. A patient of peptic ulcer was prescribed ranitidine
and sucralfate in the morning hours. Why is th is [Ref. KDT 6/e, p 223]
combination incorrect ? (Manipal 06) 9. Budesonide is : (J & K 05)
a. Ranitidine combines with sucralfate and a. A new bronchodilator
prevents its action b. Used in A.I.D.S
b. Combination of these two drugs produces c. A new antiepleptic
serious side effects like agranulocytosis d. New antihypertensive
c. Ranitidine decreases the gastric pH so [Ref. KDT 6/e, p 226]
sucralfate is not able to act
d. Sucralfate inhibits absorption of ranitidine
[Ref. KDT 6/e, p 636]

Answer 1. a. Not effective ... 2. c. Hypoglycemia ... 3. None 4. d. Sucralfate ... 5. b. Dextrome ...
6. c. Physiological ... 7. b. Zafirlukast 8. c. Monteleukast 9. a. A new ...

381
5 Self Assessment & Review Phar macology
Pharmacology
10. Orlistae is effective in obesity by : (COMEDK 05) 17. Theophyline levels are increased by which of the
a. Reduction of food intake following drugs ? (Manipal 03)
b. Beta - 1 adrenoreceptor agonist activity a. Ciprofloxacin
c. Inhibition of pancreatic lipase b. OCP’s
d. 5HT receptor agonist activity c. Erythromycin
[Ref. KDT 6/e, p 131] d. All of the above
11. New drug for increasing sensitivity of cells insulin: [Ref. KDT 6/e, p 221]
a. Glibenclamide (J & K 05) 18. All the following statements regarding insulin is
b. Velnafaxine true except : (Manipal 03)
c. Proglitazone a. In liver insulin increases Glycogenis
d. Glipizids b. Its a polypeptide hormon with A and B chain
[Ref. KDT 6/e, p 269] c. Its a action of anabolic increases glucose
storage
12. Hypoglycemia is most common with :(Jipmer 04)
d. It gfacilitates glucose entry in to red blood cells
a. Glyburide
[Ref. KDT 6/e, p 257]
b. Metformin
19. Antagonist of warfarin : (Manipal 03)
c. Rosiglitazone
a. Protamine sulfare
d. Acarbose
b. Phospholipase
[Ref. KDT 6/e, p 267]
c. Vitamin - K
13. Drug reducing post prandial blood glucose :
d. Ethacrylate sulfate
a. Repaglinide (Jipmer 04)
[Ref. KDT 6/e, p 602]
b. Glibenclaomide
20. B1 action of epinephrine is not seen in:
c. Rosiglitazone
a. Angina pectoris (Manipal 02)
d. Glyburide
b. VEntricular arrythmias
[Ref. KDT 6/e, p 269]
c. Slow arrythmias
14. Prolonged allopurinol therapy in a patient with gout
is not indicated for : (UPSC 04) d. Intestinal relaxation
a. Acute gouty arthritis [Ref. KDT 6/e, p 122]
b. Tophi 21. Moa of biphosphonates is by: (Manipal 02)
c. Urate nephropathy a. Increased urinary excretion of calcium
d. Evidence of bone/joint damage b. Osteoclastmediated resorption inhibition
[Ref. KDT 6/e, p 209] c. Decreased absorption of calcium from gut
15. True about lispro Insulin is : (SGPGI 04) d. Increased absorption of calcium from gut
a. Action is faster and longer in duration than [Ref. KDT 6/e, p 333-335]
regular insulin 22. Prolactin inhibition is caused by : (Manipal 02)
b. It is given after meals a. Dopamine
c. Source is lamb b. Bromocriptine antagonist
d. Action is faster and short in duration regular c. CRH
insulin d. Acetylcholine
[Ref. KDT 6/e, p 259m 261] [Ref. KDT 6/e, p 235-236]
16. Hirudin acts as a : (Manipal 03) 23. Which of the following can be given in diabetes
a. Blood coagulant insipidus in both children and adults ?
b. Found in human saliva a. 8a lysin vasopressin (Manipal 01)
c. Inhibits thrombin b. 8a arginine vasopressin
d. Is an antidote for heparin c. Desmopressin acetate
[Ref. KDT 6/e, p 600] d. Desmopressin lactate
[Ref. KDT 6/e, p 474-475]

Answer 10. c. Inhibition ... 11. d. Glipizids 12. a. Glyburide 13. a. Repaglinide 14. a. Acute ...
15. d. Action ... 16. c. Inhibits ... 17. d. All of the ... 18. d. It gfacilitates ... 19. c. Vitamin - K
20. d. Intestinal ... 21. b. Osteoclast ... 22. a. Dopamine 23. c. Desmopress ...

382
Phar macology of Respiratory
Pharmacology Respiratory,, Endocrinal & Gastr ointestinal System
Gastrointestinal 5
24. Abciximab is : (Manipal 01) c. Acts on itracytoplasmic receptors
a. Platelet inhibitor d. Prevents fertilization by aseptic inflammation
b. Anticancer [Ref. KDT 6/e, p 310-311]
c. Immunosupperssant 32. Prokinetic drug given with IM : (Up 00)
d. Anti hypertensive a. Promethazine
[Ref. KDT 6/e, p 610-611] b. Metoclopramide
25. Finasteride can be grouped under : (Manipal 01) c. Ondasetron
a. Antiestrogen d. Dicyclamine
b. 5a reductase inhibitor [Ref. KDT 6/e, p 643]
c. Antiprogestogen 33. Side effects of progesterone are all except :
d. 5 hydroxytestosterone inhibitor a. Hyperglycemia (Up 00)
[Ref. KDT 6/e, p 294] b. Irregular bleding
26. Levels of theophylline in blood is not increased by: c. Hirsutism
a. Erythromycin (Manipal 12) d. Venous thromboses
b. Cimetidine [Ref. KDT 6/e, p 309]
c. Carbamazepine 34. All of the following acts protein regulation through
d. Propanolol nuclear rceptor except : (UP 98)
[Ref. KDT 6/e, p 221] a. GnRH
27. Whch of the following is a “K+ channel blocker” ? b. Estrogen
a. Minoxidil (Manipal 01) c. Androgen
b. Cromokalin d. Progesterone
c. Phenacidil [Ref. KDT 6/e, p 232]
d. Sulfonyl ureas 35. Insulin in diabetes mellitus acts by : (UP 98)
[Ref. KDT 6/e, p 268] a. Increase glucose transport to liver
28. MOA of bromcriptine is by : (Manipal 01) b. Increases peripheral utilisation of glucose
a. Incrasing dopamine in brain c. Stimulates glycogenolysis in liver
b. Hyperprolactinemia in brain d. Stimulates gluconeogenesis in liver
c. Increases GH level in acromegaly [Ref. KDT 6/e, p 258]
d. None of the above 36. Theophylline metabolism is affected in : (UP 97)
[Ref. KDT 6/e, p 236-237] a. Ciprofloxacin
29. A patient taking regular luxative and develop b. Steroid
hypokalemia, probable drug is : (Up 00) c. Oxazepam
a. Bisacodyl d. Cisapride
b. Liquid paraffin [Ref. KDT 6/e, p 221]
c. Phenolphthalein 37. Theophylline clearance is increased in : (UP 96)
d. isosyllium a. Chronic liver disease
[Ref. KDT 6/e, p 653, 656] b. Smokers
30. Serum cortisol level is decreases with : c. Congestive heart failure
a. Thiopentone (Up 00) d. Concomittant erythromycin therapy
b. Propofol [Ref. KDT 6/e, p 232]
c. Etomidate 38. % of retinoic used in cream : (UP 96)
d. Methathexitone a. 5%
[Ref. KDT 6/e, p 375] b. 0.5%
31. True about mifepristone : (Up 00) c. 1%
a. Can be uses for early abortion d. 01%
b. Midluteal defect [Ref. KDT 6/e, p 858]
Answer 24. a. Platelet ... 25. a. Antiestrogen 26. d. Propanolol 27. d. Sulfonyl ... 28. a. Incrasing ...
29. a. Bisacodyl 30. c. Etomidate 31. a. Can be ... 32. b. Metoclopramide 33. d. Venous ...
34. a. GnRH 35. a. Increase ... 36. a. Ciprofloxacin 37. b. Smokers 38. b. 0.5%

383
5 Self Assessment & Review Phar macology
Pharmacology
39. Corticosteroids aggrevates are : (UP 95) b. Noscapine
a. Congenital adrenal hyperplasia c. Codme
b. Peptic ulcer d. Guaiacol
c. Hypertension [Ref. KDT 6/e, p 618]
d. Diabetes 47. Cyproheptadine is used in : (UP 94)
[Ref. KDT 6/e, p 283] a. Cold urticaria
40. Drug used for to reduce ammonia formation: b. Solar urticaria
a. Magnicium sulphate (UP 95) c. Exercise urticaria
b. Lactulose d. Icthyosis
c. Bisacodyl [Ref. KDT 6/e, p 190]
d. Biphosphonates 48. In tolbutamide, hypoglycemia is potentiated by :
[Ref. KDT 6/e, p 655] a. Paracetamol (UP 94)
41. Antacids not used with : (UP 95) b. Aspirin
a. Cimetidine c. Clofibrate
b. Omeprazole d. Phenylbutazone [Ref. KDT 6/e, p 267]
c. Pirenzepime 49. Theophylline acts by inhibiting which enzyme :
d. Sucralfate a. Phosphodiestrase (UP 94)
[Ref. KDT 6/e, p 636] b. Monoaminooxidase
42. Which of the following drug not used with : c. Na+ K+ ATPase
a. Salbutamol (UP 95) d. Aldehyde dehydrogenase
b. Budesemide [Ref. KDT 6/e, p 220]
c. Ipratropium bromide 50. Action of Atropine are A/E : (UP 93)
d. Theophylline a. CNS stimulation
[Ref. KDT 6/e, p 221] b. Bronchoconstriction
43. The maximum glucorticoid activity in : (UP 95) c. Mydriasis
a. Cortisone d. Tachycardia
b. Deoxygycortisone [Ref. KDT 6/e, p 105]
c. Corticosterone 51. Direct arterial vasodilators are A/E : (UP 93)
d. Cortisole a. Hydralazme
[Ref. KDT 6/e, p 282] b. Minoxidil
44. Synthesis of adrenal harmone blocked by : c. Diazoxide
a. Metyrapone (UP 95) d. Nitropruside
b. Methylsergide [Ref. KDT 6/e,]
c. Hydrocortisone 52. Skin hyperpignientation is caused by : (UP 93)
d. Diazepam a. Busulphan
[Ref. KDT 6/e, p 287] b. Cisplatinum
45. Drug used to lower LDL cholesterol is : c. Doxorubicin
a. Cholestyramine (UP 94) d. Vincristine
b. Clofibrate [Ref. Harrison 14/e, p 425]
c. Nicotinicacid 53. Example of Third generation cephalosponns :
d. β sitosterol a. Cephalexin (UP 93)
[Ref. KDT 6/e, p 618] b. Cefuroxime
46. Which is a central as well peripheral antitussive c. Cefoxitine
agent: (UP 94) d. Cefotaxime
a. Benzonatate [Ref. KDT 6/e, p 712]

Answer 39. a. Congenital ... 40. b. Lactulose 41. d. Sucralfate 42. d. Theophylline 43. d. Cortisole
44. a. Metyrapone 45. a. Cholestyramine 46. a. Benzonatate 47. a. Cold ... 48. a. Paracetamol
49. a. Phosphod ... 50. b. Bronchoc ... 51. d. Nitropruside 52. a. Busulphan 53. d. Cefotaxime

384
Phar macology of Respiratory
Pharmacology Respiratory,, Endocrinal & Gastr ointestinal System
Gastrointestinal 5
54. Theophyllme act on : (UP 93) sensation : (AIIMS Dec. 92)
a. Phosphodiesterase a. Rifampicin
b. Na+ K+ ATPase b. Ciprofloxacin
c. Peroxidase c. Captoprill
d. Aldehyde dehydrogenase d. Quinolone derivative
[Ref. KDT 6/e, p 220] [Ref. KDT 6/e, p 427]
55. All are used in management of hypercalcemia 62. In hyperthyroidism lugol’s lodine acts mainly by
except : (AIIMS Nov. 93) blocking the : (AIIMS June 92)
a. Mithramycin a. Uptake of iodide by the gland step-1
b. Penicillamine b. Release of thyroid hormone from the gland step-
c. Disphosphonate 5
d. Steroids c. Oxidation of iodide inons step -2
[Ref. KDT 6/e, p 335] d. Coupling of iodotyrosyl residues step-2
56. Therapeutic effects of theophylline is enhaced by [Ref. KDT 6/e, p 258]
all except : (AIIMS Nov. 93) 63. Sodium cromoglycate : (AIIMS June 92)
a. Climetidine a. Is an orally administered bronchodilator
b. Smoking b. Is effective in an acute attack of asthama
c. Erythromycin c. Inhibits the release of chemical mediators from
d. Congestive heart failure mast cells
[Ref. KDT 6/e, p 221] d. Inhibits response of chemical mediators on
57. Hyper natremia is caused by : (AIIMS Nov. 93) bronchial smooth muscles
a. Prednisolone [Ref. KDT 6/e, p 223]
b. Fludrocortisone 64. Hymoxyethyl starch is : (June 1991)
c. hydrocortisone a. Vasodilator
d. Dexamethasone b. Plasma expander
[Ref. KDT 6/e, p 291] c. Inotropic agent
58. Which anti thyroid drug can be safely used in : d. Type of dextran
pregnancy : (AIIMS Nov. 93) [Ref. KDT 6/e p 625]
a. Propylthiouracil 65. In a case of diabetes mellitus, lactic acidosis in
b. I131 due to : (AIIMS June 91)
c. Methimazole a. Insulin treatment
d. Carbimazole b. Sulfonylureas
[Ref. KDT 6/e, p 259-260] c. Phenformin
59. All increase the blood theophylline, except : d. Metformin
a. Smoking (Increase blood theophyllin level [Ref. KDT 6/e, p 279]
b. Erythromycin (AIIMS May 93) 66. Parenteral route may be employed in A/E :(UP 99)
c. Allopurinol a. Vitamin B3
d. Cimetidine b. Vitamin C
[Ref. KDT 6/e, p 221] c. Vitamin D
60. What is mode of action of sodium chromoglycate d. Vitamin E
a. Mast cell stabilization (AIIMS May 93) [Ref. KDT 6/e, p 875]
b. Antihistaminic 67. Abrupt withdrawal is not done in : (UP 99)
c. Anticholinergic a. Penicillin
d. None of the above b. Antihistaminics
[Ref. KDT 6/e, p 223] c. Corticosteroids
61. Which of following drug causes altered taste d. Antiulcer
[Ref. KDT 6/e, p 286]
Answer 54. a. Phosphodi ... 55. None 56. b. Smoking 57. b. Fludrocortisone 58. a. Propylthio ...
59. a. Smoking ... 60. a. Mast cell ... 61. c. Captoprill 62. b. Release ... 63. c. Inhibits ...
64. None 65. c. Phenformin 66. a. Vitamin B3 67. c. Corticost ...

385
5 Self Assessment & Review Phar macology
Pharmacology
68. Which of the following hormone dose not need a 75. Blood glucose levels in diabetics may be lowered
second messenger : (Delhi 84, 90) by : (BHU 90)
a. Oestrogen a. Codeine administration
b. LH b. Insulin administration
c. FSH c. Subcutaneous injection of adrenaline
d. Glucagon d. All of the above
[Ref. KDT 6/e, p 232] [Ref. KDT 6/e, p 254]
69. Galactorrhoea can be seen with all except : 76. Insulin causes : (Bihar 01)
a. Methyldopa (Bihar 99) a. Na+ entry into cells
b. Metaclopramide b. K+ exit from cells
c. Phenothiazine c. Na+ exti/K+ entry
d. Bromocriptine d. K+ entry into cells
[Ref. KDT 6/e, p 236] [Ref. KDT 6/e, p 257]
70. Bromocriptine is not used in : (Jipmer 92) 77. Insulin having the longest duration of action is :
a. Shock a. lsophane insulin (Karna 90)
b. Suppression of lactation b. PZI
c. Parkinsonism c. Insulin-zinc suspension
d. Proactinoma d. Crystaline insulin
[Ref. KDT 6/e, p 236] [Ref. KDT 6/e, p 259, table (19.1)]
71. Mechanism of action of bromocrptine is : 71. Best preparation of insulin for IV injection is :
a. Action of postsynaptic dopamine receptors a. Lente (Kerala 95)
b. Inhibition of reuptake (JIPMER 95) b. Semi lente
c. Inhibition of hydrolysis of dopamine c. Regular insulin
d. Increase of dopamine synthesis d. Humulin
[Ref. KDT 6/e, p 236] [Ref. KDT 6/e, p 259]
72. Drug of choice for hyperprolactinemia is : 72. T 1/2 of plasma insulin is : (Calcutta 2K)
a. Bromocriptine (MAHE 98) a. 3-5 min
b. Dopamine b. 20-40 min
c. Maprotiline c. 6 hour
d. Carbidopa d. 48 hrs
[Ref. KDT 6/e, p 236] [Ref. KDT 6/e, p 259]
73. Which of the following is orally effective : 73. True about lente insulin : (TN 98)
a. Thyroxine (JIpmer 81, 90) a. Duration of action is 36 hours
b. TSH b. Clear alkaline solution
c. FSH c. Cloudy acidic solution
d. Insulin [Ref. KDT 6/e, p 248] d. Made up of 3 parts of insulin zinc suspension
74. Methimazole differ from propylthiouracil in all (amorphous) and 7 parts of insulin zinc
except : (JIPMER 98) suspension (crystaline.
a. Is secreated into milk [Ref. KDT 6/e, p 259]
b. Has more volume of distribution 74. Least antigenic type of lnsulin is : (JIPMER 92)
c. More Protein bound a. PZI
d. Degrades faster b. Lente insulin
[Ref. KDT 6/e, p 250] c. Humilin
d. Isophane insulin
[Ref. KDT 6/e, p 260]

Answer 68. a. Oestrogen 69. d. Bromocrip ... 70. a. Shock 71. a. Action of ... 72. a. Bromocri ...
73. a. Thyroxine 74. c and d 75. b. Insulin ... 76. d. K+ entry ... 77. b. PZI
71. c. Regular ... 72. a. 3-5 min 73. d. Made ... 74. c. Humilin

386
Phar macology of Respiratory
Pharmacology Respiratory,, Endocrinal & Gastr ointestinal System
Gastrointestinal 5
75. Indications of newer insulins include all of the 82. Sulfonylureas are mainly used in : (Delhi PGI 96)
following except : (Karn 96) a. Juvenile onset diabetes
a. Insulin resistance b. Maturity onset diabetes
b. Pregnancy c. Diabetes in pregnancy
c. Diabetic kidney disease d. Diabetes resistant to conventional insulin
d. Insulin lipodystophy [Ref. KDT 6/e, p 271]
[Ref. KDT 6/e, p 260] 83. True about lispro insulin is : (Calcutta 2K)
76. The main advantage of human insulin is : a. Given within 1/2 hour of meal
a. Can be given to patients allergic to animal insulin b. Reduces late onset postprandial hypoglycemic
b. Less side effects (Delhi PG 96) episodes
c. Greater t1/2 c. Reduces preprandial glucose
d. Greater efficiency [Ref. KDT 6/e, p 259, 261]
[Ref. KDT 6/e, p 260] 84. New drug for increasing sensitivity of cells to
77. Which of the following drugs enhance insulin insulin : (CUPGEE 02)
release : (JIPMER 95) a. Gliebenclamide
a. Phenytion b. Venlafaxine
b. Carbamazepine c. Proglitazone
c. Sodium valproate d. Glipizide [Ref. KDT 6/e, p 273]
d. Diazoxide 85. Long term steroid use has following effect on bone:
None a. Decrease matrix of bone (Bihar 97)
[Ref. KDT 6/e, p 261] b. Disassociation of matrix to mineral ratio
78. Side effect of clomiphine citrate is : (Kerala 95) c. Marked demineralisation
a. Alopecia d. All of the above
b. Hot flushes [Ref. KDT 6/e, p 277 - 278]
c. Hyperstimulation syndrome 86. Antithyroid drug propylthiouracil acts by all except:
d. All a. Inhibiting iodinisation (AI 91)
[Ref. KDT 6/e, p 304; Shows 12/e, p 242] b. Inhibiting conversion of mono lodotyrosine to Di
79. Which one of the following insulin secretagogue lodo tyronine
has shortest duration of action : (UPSC 01) c. Inhibiting peripheral deiodination of T3 and T4
a. Glibenclamide d. Inhibiting the release of the hormone
b. Glipizide [Ref. KDT 6/e, p 279]
c. Gliclazide 87. Maximum mineralocorticoid activity is seen in :
d. Repaglinide [Ref. KDT 6/e, p 268] a. Prednisolone (Bihar 93)
80. Mechanism of action of Tolbutamide is : (PGI 95) b. Aldosterone
a. Production of insulin c. Dexamethasone
b. Glucose absorption from intestine d. Betamethasone
c. Glycogenesis [Ref. KDT 6/e, p 282, table (20.1)]
d. None of the above 88. Which of the following is the most potent
[Ref. KDT 6/e, p 266] mineralocorticoid : (TN 01)
81. Chlorpropamide metabolism is enhanced by : a. 11 -deoxycorticosterone
a. Ethyl alcohol (TN 90) b. Fluoro cortisone
b. Diazepam c. Aldosterone
c. Lorazepam d. Triamcinolone
d. Chlordiazepoxide [Ref. KDT 6/e, p 282, table (20.1)]
[Ref. KDT 6/e, p 267]

Answer 75. c. Diabetic ... 76. a. Can be ... 77. None 78. d. All 79. d. Repaglinide
80. a. Production ... 81. a. Ethyl ... 82. b. Maturity ... 83. b. Reduces ... 84. c. Proglitazone
85. c. Marked ... 86. d. Inhibiting ... 87. b. Aldosterone 88. c. Aldosterone

387
5 Self Assessment & Review Phar macology
Pharmacology
89. Steroids are not indicated in treatment of : a. Inhibition of ovulation
a. Hypercalcemia (Kerala 01) b. A change in the cervical mucus
b. Respiratory distress c. Inhibition of motility of the uterine tubes
c. Intraventricular hemorrhage d. Inhibition of motility of sperms
d. Enterocolitis e. Changes in the uterine endometrium
[Ref. KDT 6/e, p 286] [Ref. KDT 6/e, p 314 - 315]
90. All of the following are side effects of steroids 97. The following are likely adverse reactions of oral
except : (J & K 01) contraceptives except : (Karn 94)
a. Osteoporosis a. Thromboembolism
b. Muscular hypertrophy b. Weight gain
c. Cataracts c. Carbohydrate intolerance
d. Psychoses [Ref. KDT 6/e, p 286] d. Galactorrhea
91. Glucocorticoids are contraindicated in all except: [Ref. KDT 6/e, p 316]
a. Organ transplant (MAHE 98) 98. Oral contraceptives produce all of the following
b. Peptic ulcer side effects except : (Delhi PG 96)
c. Hypertension a. Chloasma
d. Diabetes mellitus b. Hyperlipidemia
[Ref. KDT 6/e, p 287] c. Hypotension
92. Anabolic steroids may produce the following side d. Breakthrough bleeding
effects except : (Karnat 96) [Ref. KDT 6/e, p 315 - 316]
a. Precocious puberty in children 99. Which steroid is used as an aerosol :
b. Cholestatic jaundice a. Prednisolone (JIPMER 91)
c. Delayed closure of epiphysis in children b. Triamcinolone
d. Acne in males and females c. Beclomethasone
[Ref. KDT 6/e, p 292] d. Betamethasone
93. Finasteride is : (Mahe 01) [Ref. KDT 6/e, p 317 - 318]
a. 5 alpha reductase inhibitor 100. Which of the following substances does not bind
b. Anti androgen to activated charcoal ?
c. Immunosuppressant a. Methanol (Bihar 95)
d. Antiemetic b. Carbamazepine
[Ref. KDT 6/e, p 294] c. Dapsone
94. Not a progesterone is : (TN 99) d. Salicylates
a. Mestranol [Ref. KDT 6/e, p 386 - 387]
b. Megestrol 101. Which of the following is a tertiary bile acid :
c. Dehydrogesterone a. Cholic acid (J & K 01)
d. Norgesterol [Ref. KDT 6/e, p 297, 307] b. Chenodeoxycholic acid
95. Use of oral contraceptives pills prophylactically is c. Urso-deoxycholic acid
useful in ? (Karnt 05) d. 7-keto deoxycholic acid
a. Carcinoma endometrium [Ref. KDT 6/e, p 616]
b. Carcinoma deficiency 102. Carbenoxolone sodium is used in peptic ulcer
c. C5 deficiency because it : (AI 91)
d. C3b inhibitor a. Systemic antacid
[Ref. KDT 6/e, p 301; Harrison 15/e, p 427, CMDT b. Orally used antacid
p 1151] c. Promotes the ulcer healing
96. Mechanism of action of an oral contraceptive d. Defoaming agent
containing a combination of oestrogen-progestin [Ref. KDT 6/e, p 636, 628]
is the following except : (Kerala 2K)
Answer 89. d. Enterocolitis 90. b. Muscular ... 91. a. Organ ... 92. c. Delayed ... 93. a and b
94. a. Mestranol 95. c. C5 deficiency 96. c and d 97. d. Galactorrhea 98. c and d
99. c. Beclomethasone100. a. Methanol 101. a. Cholic acid 102. c. Promotes ...

388
Phar macology of Respiratory
Pharmacology Respiratory,, Endocrinal & Gastr ointestinal System
Gastrointestinal 5
103. Decreased gastric acid secreation is by : c. Cisapride
a. PGE2 (Jipmer 99) d. Eposom salt
b. PGF2 [Ref. KDT 6/e, p 644]
c. PGI 111. Prokinetic drug without D2 blockade is :
d. Thromboxane A2 a. Metoclopramide (Kerala 03)
[Ref. KDT 6/e, p 633] b. Cisapride
104. Treatment of drug induced gastritis : (Jipmer 99) c. Domperidome
a. Mesoprostol d. Chlorpromazine
b. H2 receptor blockers [Ref. KDT 6/e, p 645]
c. Antacids 112. Ondansetron acts through : (MANIPAL 01)
d. Famotidine a. 5 HT3 receptors
[Ref. KDT 6/e, p 633] b. 5 HT3 A receptors
105. Constipation is caused by all except : c. Na-K channel
a. Morphine (AIIMS 80, Delhi 92) d. D2 receptors
b. Codeine [Ref. KDT 6/e, p 646]
c. Verapamil 113. Antiemetic used to prevent aspiration in
d. Magnesium Sulphate emergency surgery : (UP 00)
[Ref. KDT 6/e, p 634] a. Ondansterone
106. Of the following which is a constipating antacid : b. Promethezine
a. Calcium carbonate (Delhi 86, 92) c. Metoclopramide
b. Silicon dioxide d. Ranitidine [Ref. KDT 6/e, p 644, 646]
c. Sodium bicarbonate 114. Granisetron causes antiemetic action due to
d. Magnesium trisilicate blockade of : (MP 98)
[Ref. KDT 6/e, p 635] a. Dopamine
107. The primary aim of duodenal ulcer management b. 5HT
is : (JIPMER 93) c. Alpha-Receptors
a. To excise the ulcerogenic mucosa d. Opoid receptors [Ref. KDT 6/e, p 647]
b. To prevent complications 115. Methyl Poly Siloxane is (MPS) : (AI 91)
c. To rule out malignancy a. Sysntemic antacid
d. To derease acid output b. Orally used antacid
[Ref. KDT 6/e, p 638] c. Promotes ulcer healing
108. Which drug does not cause prolactin release : d. Defoaming agent
a. Metoclopramide (KERALA 01) [Ref. KDT 6/e, p 649]
b. Reserpine 116. Which of the following drug is useful in dissolving
c. Methyldopa Gall bladder stone : (Delhi 93)
d. Apomorphine a. Clofibrate
[Ref. KDT 6/e, p 639 - 640] b. Chenodeoxycholic acid
109. Metoclopramide is useful for all except : (TN 98) c. Lactulose
a. Motion sickness d. Lithocholic acid
b. Chemotherapy induced vomiting [Ref. KDT 6/e, p 649]
c. Anti emetic 117. Which purgative causes hypokalemia : (UP 00)
d. Promoting lactation a. Liquid parffin
[Ref. KDT 6/e, p 644] b. Bisacodyl
110. The of choice in the treatment of gastroparesis is c. Castor oil
a. Bisacodyl (TNPSC 00) d. Osmotic purgatives
b. Liquid paraffin [Ref. KDT 6/e, p 653]

Answer 103. a and c 104. a, b and d 105. d. Magnesium ... 106. a. Calcium ... 107. b. To prevent ...
108. d. Apomorphine 109. d. Promoting ... 110. c. Cisapride 111. b. Cisapride 112. a. 5 HT3 ...
113. c. Metoclopr... 114. b. 5HT 115. d. Defoaming ... 116. b. Chenode ... 117. b and c

389
5 Self Assessment & Review Phar macology
Pharmacology
118. Constipation is caused by all except : c. Travellers diarrhoea
a. Morphine (AIIMS 80; Delhi 92) d. Salmonella diarrhoea
b. Codeine [Ref. KDT 6/e, p 660; Harrison 15/e, p 1676]
c. Verapamil 123. Stress increases all hormones level except :
d. Magnesium Sulphate a. ACTH (Bihar 99)
[Ref. KDT 6/e, p 655] b. Noradrenaline
119. True about lactulose are all except : (MP 00) c. Insulin
a. It is a cathartics d. Glucagon
b. Inhibits absorption of ammonia [Ref. Schwartz Surgery 7/e, p34, 37]
c. Inhibits absorption of amino acids 124. Least half life is for which hormone : (Jipmer 99)
d. Causes bacteria to utilize ammonia a. Noradrenaline
[Ref. KDT 6/e, p 655 - 656]
b. Renin
120. Concentration of Potassium in ORS is : (Al 92)
c. Aldosterone
a. 10mEq
d. Thyroxine
b. 20mEq
[Ref. Ganong Physio 19/e, p125, 200, 350, 254]
c. 40mEq
125. The drug that is not effective against
d. 90mEq
streptococcus : (APPGE 05)
[Ref. KDT 6/e, p 659]
a. Ciprofloxacin
121. Which of the following is true of oral rehydration
b. Azithromycin
solution concentration : (AI 90)
c. Cloromphenicol
a. Sodium 70 mEq/litre
b. Potassium 20 mEq/litre d. Vancomycin
c. Chloride 30 mEq/litre [Ref. KDT 6/e, p 732]
d. Bicarbonate 80 mEq/litre 126. Drug with highest emetogenic potential is :
[Ref. KDT 6/e, p 659; Ghai 6/e, p 272] a. Cis-platinum (J & K 01)
122. Diarrhoea which respond to antibiotic therapy b. Emetine
is : (JIPMER 92) c. Belomycin
a. Tropical sprue d. Vancomycin
b. Irritable bowel syndrome [Ref. KDT 6/e, p 827]

Answer 118. d. Magnesium ... 119. c. Inhibits ... 120. b. 20mEq 121. b. Potassium ... 122. a, c and d
123. c. Insulin 124. a. Noradrenaline 125. d. Vancomycin 126. a. Cis-platinum

390
Antimicrobial Drugs

TRUE ANTIBIOTIC
A true antibiotic is derived from a natural source (e.g., bacteria). It is an agent that reduces the organism proliferation or
function (i.e., bacteriostatic) or resulting in organism death (i.e., bactericidal).

ANTIMICROBIAL AGENT
The term antimicrobial agents include drugs synthesized in labs (i.e., antibacterial agents) as well as those obtained
naturally fro fermentation of micro organisms (i.e., antibiotics).
Semisynthetic antibiotics are ones where key portion of the chemical structure is derived from microbial source and its
chemical structure is then modified by attaching different chemical moieties.
An anti-infective should be used with evidence. At times, laboratory tests may be deceptive. Irrational antibiotic therapy is
that:
• When the dose and duration are inadequate
• Improper selection of the drug (wrong drug)
• Prolongation of hospital stay
• Development of “new” infection
• Development of resistance
• Increase in health care costs

TYPE OF THERAPY : Bactericidals for all emergency situations


Pregnancy and Lactation : No antibiotic is safe in both, commonly used are Penicillins, Cephalosporins. The agents
contraindicated are: Tetracyclines, Fluoroquinolones, Aminoglycosides, Sulfonamide. Most antibiotics appear in breast
milk.

Pharmacodynamics of antimicrobial drugs


A. Concentration dependent killing (CDK)
The antimicrobials which are much more effective with higher drug concentrations in the blood, exhibit CDK e.g.
Aminoglycosides, Fluoroquinolones, Metronidazole.
B. Time dependent killing (TDK) : The antimicrobials which are much more effective when blood levels are maintained
above the minimum inhibitory concentration for as long a duration as possible, exhibit TDK. The peak bactericidal action
does not increase with increasing the drug concentration above a particular level e.g. Vancomycin, beta-lactams.

All antimicrobials can be divided into :


• Bacteriostatic - Usually inhibit protein synthesis in the organism
6 Self Assessment & Review Phar macology
Pharmacology
• Bactericidal - Inhibit cell wall synthesis
Certain drugs have both properties of cidal and static. Chloramphenicol is bactericidal to pneumococci, H. influenzae
and meningococci. While Vancomycin, ampicillin, penicillins used as single agent are bacteriostatic against enterococci.

Chemotherapeutic index
The object of chemotherapy is to destroy the foreign invading organisms without affecting the host (selective toxicity).
Chemotherapeutic index (CTI) is a measure of selective toxicity. It is defined as LD50 for the host divided by LD50 for
microorganisms.

POST ANTIBIOTIC EFFECT (PAE)


• Persistent suppression of bacterial growth after brief exposure of antimicrobial drug is known as PAE, the mechanism
of PAE is postulated to be:
– Inadequate recovery of cell structures after a non-lethal insult
– Persistence of the drug in the bacterial cell
• PAE represents the extension of lag phase of bacterial growth. The longer the time the bacteria requires to return to
logarithmic growth after exposure to antimicrobial, the greater is the PAE of that antimicrobial agent.
• In-vivo PAE is much longer than in-vitro PAE. This difference may be due to post antibiotic leukocyte enhancement
(PALE) and/or due to post antibiotic bacterial suppression effect (PASE).
– In PALE, there is greater susceptibility of microorganisms to leucocytes after exposure to an antimicrobial that
exhibits PAE.
– In PASE, the humoral and cellular immune processes undergo alteration in morphological characteristics and lose
their adhesiveness with the bacteria as happens in bacterial endocarditis.

* Minimum antibacterial concentration (MAC): The lowest drug concentration required to induce morphologic change in
bacteria.
* Minimum inhibitory concentration (MIC): The lowest antibiotic concentration that prevents the growth of microorganism
after a 24-hour incubation period with a standard organism inoculation of 10000 to 100000 cfu/ml.
* Minimum bactericidal concentration (MBC): The lowest concentration of an antibiotic that causes complete destruction
of the organism or permits survival of less than 0.1% of the inoculums.

Drugs showing PAE


a. Agents showing CDK with prolonged PAE: Aminoglycosides, fluoroquinolones, metronidazole, rifampicin
b. Agents showing TDK with shorter PAE: Beta-lactams, clindamycin, macrolides (except azithromycin and clarithromycin)
c. Agents showing TDK with prolonged PAE: Azithromycin, clarithromycin, vancomycin, tetracyclines

Pharmacokinetics of antibacterial agents


• In critically ill patients, only IV route is indicated as oral and parenteral routes are unreliable
• Drugs like vancomycin and antipseudomonal drugs should be used IV, as oral route is slow.
• For Meningitis and endocarditis, IV route is preferred.

For some antimicrobials, P/K in oral and parenteral route is similar, e.g.
- Tetracyclines - Metronidazole
- Fluoroquinolones - Rifampicin
- TMP-SMZ - Fluconazole

ANTIBIOTIC RESISTANCE
The resistance in bacterial populations can spread from person to person by bacteria; from bacteria to bacteria by plasmids
and from plasmid to plasmid by transposons and integrons.

394
Antimicr obial & Antineoplastic Dr
Antimicrobial ugs
Drugs 6
Mechanisms :
a. Mutation-selection:
• a stable and heritable genetic change that occurs spontaneously and randomly among microorganisms. It is not
induced by AMA.
• It is the basis for resistance to streptomycin (ribosomal mutation), fluoroquinolones (gyrase or topoisomerase IV
gene mutation), rifampicin (RNA polymerase gene mutation) and linezolid (ribosomal RNA mutation).
• Mutation is the basis for all drug resistance in M. tuberculosis.
b. Gene transfer: - Plasmids are the vectors serving as a carrier of DNA molecule.
• Most bacteria contain such extrachromosomal genetic elements that can replicate independently and freely in the
cytoplasm.
• Plasmids which carry genes resistant to antibiotics are called R-plasmids. These R-genes can get transferred from
one R-plasmid to another plasmid or to chromosome.

Methods of transfer of R-genes from one bacteria to another :


a. Conjugation :
• is gene transfer by sexual method i.e. direct cell-to-cell contact through a sex pilus or bridge. This is important
because multiple resistance genes can be transferred in a single event.
• It commonly occurs in colon where a large variety of gram negative bacilli come in close contact. Vancomycin resistance
in enterococci and staph. aureus, chloramphenicol resistance in typhoid bacilli, streptomycin resistance in E. coli,
penicillin resistance in hemophilus, gonococci etc. are examples of conjugation.
b. Transduction :
• is transfer of gene carrying resistance through bacteriophage
• is important in many staph. aureus strains.
c. Transformation : is the release of resistance carrying DNA in the medium which may be imbibed by another sensitive
organism; is the least important mechanism.

MULTIPLE ANTIBIOTIC RESISTANCE


The 2 major mechanisms include:
a. Acquisition of multiple unrelated resistance genes: it occurs by sequential steps of gene transfer and environmental
selection in areas of high-level antimicrobial use. E.g., hospital associated strains of gram negative bacteria, enterococci,
staphylococci and community acquired salmonella, gonococci and pneumococci.
b. Mutation in a single gene that mediate resistance to unrelated compounds: it occurs in genes encoding outer membrane
porins and efflux proteins of gram-negative bacteria. These mutations decrease periplasmic accumulation of beta-
lactam, quinolones, tetracyclines, chloramphenicol and aminoglycosides.

MANAGEMENT OF ANTIBACTERIAL TOXICITY


• Obtain drug history- of Allergy, ADRs and adverse drug interaction (ADI)
• Avoid offending drug as far as possible. If the drug must be used then desensitize. Eg. Penicillin allergic patient having
neurosyphilis.
• Avoid drugs having cross reactivity
E.g. • Penicillin-Cephalosporins (10% incidence)
• Ceftazidime - Aztreonam (100% Avoid aztreonam if allergic to ceftazidime)
• Penicillin - Imipenem (50%)

ANTIBIOTIC COMBINATIONS
Most infections can be effectively treated with single agents. However, in certain situations, antibiotics can be rationally
combined:
1. To provide broad-spectrum empirical therapy in seriously ill patients, (e.g. sepsis)
• Since bacterial diagnosis is not known, drugs covering gram positive, gram negative and sometimes anaerobes are
combined. E.g., penicillin + streptomycin; cephalosporin + aminoglycoside + metronidazole. In community acquired
pneumonia, a macrolide is used for atypical organisms and cefuroxime for pneumococci and gram negative microbes.

395
6 Self Assessment & Review Phar macology
Pharmacology
• It should be continued only till bacterial sensitivity becomes available as it may lead to resistance, toxicity, superinfection
etc.
2. To treat polymicrobial infection - In intra-abdominal abscess or rupture of gut into peritoneum, hepatic and brain
abscess, genital tract infections, the rational combination may be an Aminoglycoside or 3rd generation Cephalosporin
plus Metronidazole, after surgical drainage.
3. To decrease emergence of resistance
• The classis example is TB where monotherapy rapidly leads to resistance.
• The other examples are infections treated with rifampicin such as Staphylococcal osteomyelitis or prosthetic valve
endocarditis, in which a 2nd agent is added to prevent emergence of rifampicin resistant mutants. It is also employed
in treatment of H. pylori and HIV infections.
4. To decrease dose related toxicity
• This occurs only when the combination is synergistic so that the doses can be reduced, it is needed for antimicrobials
with low safety margin.
• Example : Amphotericin B + Flucytosine in treating cryptococcal meningitis in HIV-infected patients. This allows
reduction in Amphotericin dose, thus decreased nephrotoxicity.
• Example : Streptomycin + Penicillin G for SABE due to Strept. faecalis
5. To achieve synergism
• Synergism is said to occur, when the MIC of each AMA is reduced to 25% or less. Thus, a synergistic drug sensitizes
the organisms to the action of other member of the pair. This may manifest as a more rapid lethal action of the
combination than either of the individual members.
Example
• Penicillin + gentamicin in enterococcal endocarditis
• Beta-lactam + Aminoglycoside in Febrile Neutropenic cancer patients or pseudomonas Infection
• TMP-SMZ due the sequential block in folate metabolism

Antibiotic prophylaxis
It is indicated in the following situations :
a. The risk or severity of infection should outweigh the risk of side effects from AMA
b. The AMA should be given for the shortest period necessary to prevent infections.
c. The AMA should be given before the expected period of risk (e.g., within 1 hr of incision before elective surgery) or as soon
as possible after contact with infected patient.
d. The antibiotic must be active against the most likely contaminating microorganism. Thus, cephalosporins are used most
commonly in this form of chemoprophylaxis.
e. The prolonged administration of drugs after surgical procedure is potentially harmful; hence, it is not used beyond 24
hours to prevent development of resistance and superinfections.

The prophylaxis is of 2 types :


A. Surgical prophylaxis
B. Non-surgical prophylaxis

Condition Antibacterial agent Timing or duration

Nonsurgical

• Cardiac lesions susceptible to Amoxicillin Before and after procedures causing


bacterial endocarditis bacteremia
• Recurrent s. aureus infections Mupirocin 5 days (intranasal)
• Contact with patient with meningo- Rifampin 2 days
coccal meningitis Fluoroquinolone Single dose
• Bite wounds Penicillin V or amoxicillin/clavulanic acid 3 - 5 days
• Recurrent cystitis Trimethoprim-sulfamethoxazole or a 3 times per week for up to 1 year or after
fluoroquinolone or nitrofurantoin sexual intercourse

396
Antimicr obial & Antineoplastic Dr
Antimicrobial ugs
Drugs 6
Surgical

• Clean (cardiac, vascular, neuro- Cefazolin (vancomycin) Before and during procedure
logic, or orthopedic surgery)
• Ocular Topical combinations and subconjun- During and at end of procedure
ctival cefazolin
• Clean-contaminated (head and neck Cefazolin (or clindamycin for head and Before and during procedure
high-risk cesarean cesarean section; neck)
hysterectomy)
• Clean-contaminated (vaginal or Cefazolin or cefoxitin or cefotetan Before and during procedure
abdominal hysterectomy)
• Clean-contaminated (high risk Fluoroquinolone Before and during procedure
genitourinary surgery)
• Clean-contaminated (colorectal Cefoxitin or cefotetan (add oral neom- Before and during procedure
surgery or appendectomy) mycin+erythromycin for colorectal)
• Dirty (ruptured viscus) Cefoxitin or cefotetan + gentamycin Before and for 3-5 days after procedure
clindamycin + gentamycin, or another
appropriate regimen directd at anero-
bes.
• Dirty (traumatic wound) Cefazolin Before and for 3-5 days after trauma

IMPORTANT DRUGS USED COMMONLY IN NON SURGICAL PROPHYLAXIS:


• Dental, Oral, Or Upper Respiratory Procedures In High Risk Patients - Amoxycillin or
Clindamycin
• Genitourinary, GI procedures in High Risk group - Ampi./Vanco./genta
• Group B streptococci (GBS) - Ampicilln/Pencillin
• H.influenzae - Rifampicin
• Meningococci - Rifampicin
• MAC - Clarithro/Azithromycin
• Otitis media - Amoxycillin
• Pertusis - Erythromycin
• Plague - Tetracycline
• Pneumococcal infection - Penicillin
• PCP - TMP-SMZ
• Rheumatic Fever - Benzathine penicillin
• T. Gondii - TMP-SMZ

DURATION OF AMA TREATMENT


• Recent investigations have evaluated shorter duration of therapy and treatment duration guidelines have been established
for common bacterial infections as mentioned in the following table.
• Re-treatment of infections for which therapy has failed usually requires a prolonged course (>4 weeks) with combination
of antibacterial agents.

Duration of Therapy Infections

Single dose Gonococcal urethritis, streptococcal pharyngitis (penicillin g benzathine), primary and secondary syphilis
(penicilin G benzathine)
3 days Cystitis in young women community-or travel-acquired diarrhea
3-10 days Community-acquired pneumonia (3-5 days), community-acquired meningitis (pneumococcal or
meningococcal), antibiotic-associated diarrhea (10days)

397
6 Self Assessment & Review Phar macology
Pharmacology

2 weeks Helicobacter pylori- associated peptic ulcer, neurosyphilis (penicillin IV), penicillin-succeptible viridans
streptococcal endocarditis (penicillin plus aminoglycoside) disseminated gonoccal infection with arthritis,
acute pyelonephritis, uncomplicated s. aureus catheter-associated bacteremia.
3 weeks Lyme disease, septic arthritis (nongonococcal)
4 weeks Acute and chronic prostatitis, infective endocarditis (penicillin-resistant streptococcal)
>4 weeks Acute and chronic osteomyelitis, S.aureus endocarditis, foreign-body infections (prosthetic-valve and
joint infections), relapsing pseudomembranous colitis.

BETA-LACTAM ANTIBIOTICS
Penicillin G : It has greatest activity against gram +ve bacteria, non beta-lactamase producing staphylococci, streptococci,
Bacillus anthracis, Enterococci, Meningococci, Actinomyces, Spirochetes, Clostridium, Gram-positive rods.
Antistaphylococcal penicillins : Nafcillin, cloxacillin, dicloxacillin, oxacillin. These are resistant to staphylococcal B-
lactamases. They are effective against Staphylococci, Streptococci but inactive against Enterococci, anaerobic bacteria and
Gram -ve cocci and rods.
Extended spectrum penicillins, Ampicillin, Amoxycillin, Antipseudomonal (e.g. Azlocillin, Mezlocillin, Piperacil]jn, Ticarcillin)
They have the properties of natural penicillin but have activity against Gram -ve organisms.
MOA: Inhibit cell wall synthesis by binding PBP. Cell death occurs due to osmotic lysis.
Both penicillins and cephalosporins are bactericidal only when the bacteria are actively multiplying (i.e. synthesizing cell
wall).

Resistance: Develops due to one or more of the following 4 mechanisms :

i. Inactivation of antibiotic by B- Lactamase


ii. Modification of target PBP
iii. Impaired penetration of the drug to PBPs
iv. Presence of efflux pump

So far, around 150 B- lactamases have been identified. Those produced by S. aureus, H. influenzae, E. coil have narrow
substrate specificity. They hydrolyse penicillins, but not cephalosporins easily. Those produced by Pseudomonas and
Enterobacter have wider substrate specificity and can hydrolyse both cephalosporins and penicillins (except
antipseudomonas).
Carbapenems are highly resistant to hydrolysis by penicillinase and cephalosporinase and are hydrolysed by a Metalo-
beta-lactamases.
Resistance to beta-lactam antibiotics may be acquired either by mutation of existing PBP genes or, more importantly, by
acquiring new PBP genes (e.g. staphlococcal resistance to methicillin) or by acquiring new “pieces” of PBP genes (e.g.
pneumococcal, gonococcal and meningococcal resistance).

Pharmacokinetics
• All are poorly absorbed from the gut except Penicillin V, Amoxycillin, Ampicillin and Cloxacillin which are acid stable.
• Food retards the absorption of all except Amoxycillin. Oxacillin is most strongly bound to food followed by dicloxacillin. All
are bound to plasma proteins in variable quantities. Highly protein bound agent is Nafcillin.
• Penicillins are polar molecules and the concentration in the cell is less than ECF.
• Procaine and Benzathine penicillins are especially formulated to delay the absorption, hence to maintain prolonged
plasma concentration.
• Penicillins are excreted into sputum and milk. Penetration into eye, prostate and CNS is poor. Inflammation facilitates it.

Excretion is mainly in urine: 10% by Filtration and 90% by Secretion.


Nafcillin is mainly excreted in bile. Oxacillin, Cloxacillin and Dicloxacillin are excreted by both routes. Dose adjustments are
necessary in renal/hepatic failure.
In the new born, Renal tubular secretion is less efficient, hence dose adjusted per kg body wt. will result in higher plasma
penicillin concentration.

398
Antimicr obial & Antineoplastic Dr
Antimicrobial ugs
Drugs 6
CLINICAL USES of Penicillin G, Benzathine penicillin:
• Many gram + ve, - ve organisms, actinomyces, spirochetes
• Administration of aminoglycosides is necessary to achieve desired bactericidal effect in infective endocarditis.
* Antistaphylococcal - Oxa/Cloxa/Dicloxa given orally for mild staphylococcal infections .
- For serious systemic staph. infections, Oxa/nafcillin are give I.V.
* Extended spectrum penicillins
• Are susceptible to beta-Iactamases
• Have enhanced activity against gram - ve bacteria
Ampicillin/Amoxycillin - For UTI, LRI, Sinusitis, Otitis
– Used for treating penicillin resistant pneumococci
– For shigellosis - Ampicillin is DOC
– Avoid Ampicillin in uncomplicated salmonella gastroenteritis, as it may prolong the carrier state.

Anti Pseudomonal
• Carbenicillin Indanyl Na -Orally active (acid stable) antipseudomonal drug (Least used).
• Ticarcillin is useful in lower doses.
Piperacillin, Azlocillin, Mezlocillin resemble ticarcillin, but their spectrum is wider, active against Klebsiella.
A single agent should not be used to treat pseudomonas. Always combine an aminoglycoside.
Addition of beta laclamase inhibitor further enhances the spectrum (Clavulanate, Tazobactam, Sulbactam) i.e. B-lactarnase
producing S. aureus and gram-ve bacteria.

Adverse events :
• Hypersensitivity reaction - due to penicilloic acid. Total dose used in the past determines the magnitude of hypersensitivity
(except anaphylactic shock; incidence 1%)
• Seizures are seen in high dose penicillin, in renal failure
• Nafcillin - Neutropenia
• Methicillin - Interstitial nephritis
• Oxacillin - hepatitis
• Pseudo membranous colitis seen with all penicillins (mostly with Ampicillin)
• Secondary infection such as vaginal candidiasis
• Skin rash of non-allergic in nature seen with Ampicillin/Amoxycillin

CEPHALOSPORINS
• Similar to penicillins (Chemicals, MOA, Toxicity)
• More stable than penicillins
• Not active against Enterococci, Listeria
• 4 generations. 1st generation has action against Gram + ve

1st GENERATION CEPHALOSPORINS


• Cefazolin, Cefadroxil, cephalexin, cephradine, Cephalothin etc. Cefazolin is given IV, IM. Enterococi and
Pseudomonas are resistant. All are excreted by glomerular filtration and tubular- secretion.
• Dose adjeustment is necessary in renal failure.
• Probenecid inhibits tubular secretion.
USES :
They (except cefazolin) are rarely the drug of choice in any serious infection.
• CEFAZOLIN is the agent of choice in surgical prophylaxis as it penetrates well in most tissues.

399
6 Self Assessment & Review Phar macology
Pharmacology

• Cefazolin is the only 1st generation parenteral cephalosporin still in general use.
• Cefazolin does not penetrate CNS and cannot be used for meningitis.
• Cephazolin may be a choice in infections for which it is a least toxic drug (e.g., K pneumoniae) and in patients with
staphylococcal or streptococcal infections who have a history of mild penicillin hypersensitivity but not anaphylaxis.

2nd GENERATION CEPHALOSPORINS


• Cefaclor, cephamandole, cefonicid, cefuroxime, cefprozil, laracarbef as well as the structurally related cephamycins
cefoxitin, cefmetazole, cefotetan, which have activity against anaerobes belongs to the 2nd generation cefalosporins.
• They have spectrum similar to 1st generation agents plus activity against gram-negative organisms. As like 1st
generation agents, noneof the agents are active against enterococci and pseudomonas.
• Cefaclor, cefuroxime axetil, cefprozil and loracarbef are given orally. Cefuroxime axetil is only active against
penicillin resistant pneumococci.
• Cefotetan, cefonicid, cefoxitin, cefprozil are given parenterally (IM-is painful)
• Cefuroxime axetil is the only 2nd generation drug that crosses BBB But not used to treat meningitis as it is less
effective than ceftriaxone and cefotaxime
Uses :
• Active against beta- lactamase producing H. influenzae
• For treatment of sinusitis, otitis, LRTI
• Cefuroxime is used to treat community acquired pneumonias where beta- lactamase producing H. influenzae
and Klebsiella is a consideration.

3rd GENERATION CEPHALOSPORINS

Cefoperazone, cefotaxime, ceftazidime, ceftizoxime, ceftriaxone, cefixime, cefpodoxime proxetil, ceftibuten, moxalactam
Features
• They have expanded gram negative action
• Ceftazidime and cefoperazone are active against Pseudomonas
• Ceftizoxime and moxalactam are active against B. fragilis
• Cefixime, ceftibuten and cefpodoxime proxetil are orally active
• Cefotaxime, ceftriaxone, cefixime can cross BBB and are useful in treating meningitis
• Ceftriaxone and cefoperazone are excreted in bile, so no dose adjustment required in renal insufficiency
• Ceftriaxone has longest tl/2 (7.5 hrs) amongst cephalosporins requiring OD dosing

Uses : Gonorrhea (PPNG and TRNG) - ceftriaxone and cefixime.


• Meningitis caused by meningococci, pneumococci, H. influenzae and susceptible enteric gram negative rods
(except listeria).
• For pseudomonas infections, antipseudomonal cephalosporins should be combined with an aminoglycoside.
• Penicillin resistant pneumococci - ceftriaxone, cefotaxime
• Empirical therapy of sepsis both in immunocompetent and immunecompromised hosts
• Empirical treatment in immunocompromised febrile, neutropenic patients 3rd generation cephalosporins and
aminoglycoside.

4TH GENERATION: CEFPIROME AND CEFEPIME


Are more resistant to hydrolysis by beta- lactamases produced by enterobacter. Good activity against Pseudomonas
aeruginosa, staph. aureus, enterobacteriacae. Highly effective against Hemophilus and Nisseria. They penetrate
well into CNS, are excreted by kidney.

ADR of Cephalosporins:
• Allergy : Including anaphylaxis, nephritis, granulocytopenia, hemolytic anemia 5-10% of penicillin
allergic people are allergic to cephalosporins

400
Antimicr obial & Antineoplastic Dr
Antimicrobial ugs
Drugs 6
• Toxicity : – Pain at IM site
– Thrombophlebitis i.v.
– Renal toxicity including Interstitial nephritis and tubular necrosis
Agents containing Methyl thiotetrazole ring (MTT) cause hypoprothrombinemia and bleeding disorders- These are
cefamandole, Moxalactam, cefmetazole, cefotetan and cefoperazone. Can be prevented by vitamin K 10 mg twice
weekly. Moxalaclam also causes platelet dysfunction and can induce severe bleeding (banned now). MTT ring containing
drugs causes severe disulfiram like reactions.
Super infection : 2nd and 3rd generation are not active against gram-positive organisms during the course of
treatment super infection can occur with them and fungi.

Other (beta-lactam Drugs (Aztreonam)


• Aztreonam is highly resistant to beta-lactamases. This agent inhibits synthesis of bacterial cell wall by high-
affinity binding to penicillin-binding protein (PBP) which is found primarily in aerobic, Gram-negative microbes
• Spectrum of activity includes aerobic, Gram-negative bacteria and is similar in activity to aminoglycosides without
causing ototoxicity or nephrotoxicity.
• Aztreonam is effective in treating Gram-negative urinary tract infections, lower respiratory tract, skin,
intraabdominal, gynaecologic infections and septicemia.
• This drug may be used in combination with other antibiotics which are active against Gram-positive microbes
and anaerobes in mixed infections
• Active against gram negative organisms including pseudomonas
• No activity against gram positive or anaerobes
• Excreted by kidney, t1/2 (1-2h) prolonged in RF

Beta- Lactamase inhibitors e.g. Clavulanic acid, Sulbactam, Tazobactam


• All have very weak antibacterial action : They are potent inhibitors of a several β- lactamases and thus can
protect penicillins / cephalosporins
• They are active against plasmid mediated β- lactamases, but not against chromosomally mediated (e.g
pseudomonas)
Uses: Ampicillin + sulbactam, Amoxycillin + clavulanate in â- lactamases producing S. aureus and H. influenza.
Piperacillin + Tazobactam used empirically in treating mixed aerobic and anaerobic infections irrespective of the
immune status.

Carbapenems: e.g. Impipenem, Meropenem, Ertapenem, biapenem.


• Have activity against Gram positive, negative bacteria and Anerobes.
• Inhibit bacterial cell wall mucopeptide synthesis and is bactericidal
• Organism resistant to it are : - Enterococci
- MRSA
- C. Difficile
- Burkholderia cepacia
• Imipenem is inactivated by dehydropeptidaseI in renal tubule; hence combined with dehydropeptidase inhibitor
(cilastatin) which protects it.
• Meropenem and ertapenem are not degraded by dehydropeptidase, hence does not require cilastatin
• They penetrate all tissues including CNS
• Dose reduction in renal failure
• They are the DOC of enterobacterial infection
• Combination of an aminoglycoside for pseudomonas infection
• Patients allergic to penicillins may be allergic to carbapenem

401
6 Self Assessment & Review Phar macology
Pharmacology

Other cell wall inhibitors:

Vancomycin
• Mainly active against gram positive organisms especially MRSA and C. difficile
• Vancomycin acts synergistically with gentamicin/ streptomycin
• Orally poorly absorbed - only used for C. difficile
• Parenteral i.v
• Drug accumulation occurs in renal insufficiency
• Functionally anephric patents t l/2 may be 10 days

Resistance: Vancomycin resistance is an important problem in antibiotic chemotherapy. For example, vancomycin-
resistant enterococci strains are worldwide.
• The resistance mechanism involves transfer between cells and is plasmid mediated.
• The specific alteration is a change in the peptidoglycan stem peptide which causes a loss of the vancomycin
binding target.
• Often significant vancomycin resistance is observed with enterococci strains all the same time, most staphylococci
aureus and staphylococci epidermidis remain vancomycin (Vancocin) sensitive

Major Clinical Use (It is basically a “reserved drug”)


• With gentamicin for enterococcal endocarditis in patients allergic to penicillin
• Meningitis caused by resistant pneumococci with Cefotaxime, Ceftriaxone or Rifampicin
• Pseudomembranous Colitis (Antibiotic associated colitis) - DOC is Metronidazole Vancomycin is the 2" drug
(VRE has emerged)
• Sepsis along with other agents
• Endocarditis due to methicillin resistant staphylococci
• Treatment alternative enterococcal endocarditis.: Vancomycin with gentamicin: for patients allergic to penicillin.
• Vancomycin in combination with cefotaxime, ceftriaxone or rifampim: appropriate for treatment of meningitis
when the suspected infecting agent is thought/known to be highly penicillin resistant. ADR
• Pain and thrombophlebitis
• Ototoxicity and nephrotoxicity (Mild) but can be severe when an aminoglycoside is combined
• Red man (Red Neck) syndrome - flushing due to histamine release (can be prevented by prolonging the infusion
or increasing the interval.

Teicoplanin : Similar to vancomycin in MOA and spectrum. It has a long t 1/2 45-70 hrs (longest for any antibiotic)
• Can be given IM and IV less thrombophlebitis less pain.

Fosfomycin: analog of phosphoenol pyruvate inhibits enol pyruvate transferase


• Acts synergistically with aminoglycosides/ FQs / beta- lactam drugs
• Safe in pregnancy
• Used for UTI with pregnancy with resistant or allergic to penicillins and cephalosporias

Bacitracin
• Obtained from bacteria Bacillus subtilis
• Mainly active against gram positive organism
• Markedly nephrotoxic (so contraindicated for systemic use)
• Used only locally as skin ointments with neomycin/ polymyxin. Alone it is used to irrigate joints / pleural cavities, wounds
etc.

Cycloserine
• Acts against both gram positive and gram negative organism

402
Antimicr obial & Antineoplastic Dr
Antimicrobial ugs
Drugs 6
• It is a structural analog of D-alanine component of cell wall
• Acts by inhibiting alanine recemase
• Mainly used in MDR- TB 2nd line tt
• Toxicity is CNS related headache, psychosis, memory loss, neuropathy
(give pyridoxin 100–250 ingld)

TETRACYCLINES
• Tetracyclines chelate divalent/ trivalent cations
• Glycylcyclines are derivatives of Minocyline. Are not affected by tetracycline resistance. They are active against both TC
susceptible and TC- resistant gram positive and negative aerobes and anaerobes. Glycylcyclines are under development.

Tigecyclin

Spectrum : Truly broad spectrum. Acting on Gram positive and negative bacteria, anaerobes, rickettsiae Chlamydia,
Mycoplasma, some protozoas and amoeba

Resistance :
• Ribosomal protection in bacteria
• Active efflux
• Enzymatic inactivation

P/K
• Doxy and minocycline are well-absorbed 95-100%
• Food retards absorption of all except Doxy / Mino (others that retard absorption are Ca++ Al++) dairy products, Milk, antacids,
alkaline pH
• Minocycline reaches high concentration, in tear and saliva useful in meningococcal carrier state eradication
• They chelate calcium and damage Bones/Teeth

Carbamazepine, phenytoin, barbiturates, chronic alcoholism shorten t l/2 by 50% by hepatic enzyme induction.
Doxycycline mainly eliminated by bile. No dose adjustment in renal failure

USES
• DOC for Mycoplasma, Chlamydia, Rickettsiae
• For Helicobacter pylori, with other drugs
• Vibrio cholerae (Rapidly stops shedding of bacteria)
• Plague/tularemia/brucellosis - TC + Aminoglycoside
• E. histolytica, malaria, acne, exacerbation of bronchitis, community acquired pneumonia/Lyme disease/Relapsing fever/
Leptospirosis
• Minocycline 200mg/day x 5 days used eradicates meningococcal carrier state (But rifampicin is preferred).
• Demeclocycline inhibits the action of ADH in renal tubules used in SIADH
C/I - Pregnancy, Lactation, Children < 8 yrs of age, when given in pregnancy it is deposited in teeth giving rise
to Fluorescence. Discoloration and Enamel dysplasia

ADR
• Bone → Deformity and growth inhibition.
• Hepatotoxicity: In pre existing liver disease or when given in high doses (> 4rng/day IV)
• Nephrotoxicity: Fanconi like syndrome - with expired drugs, accumulation occurs in renal failure
(except doxy)

403
6 Self Assessment & Review Phar macology
Pharmacology
• Photosensitization: Demeclocycline - sensitizes UV/sunlight esp. in fair skinned people.
• Vestibular - Dizziness/vertigo (Nausea/vomiting)

MISUSE : Used in animal feed to increase growth - leads to resistance in man

MACROLIDES
• Large lactone ring (14 -16 atoms) with deoxy sugars attached.
• Spectrum: They are mainly effective against Gram + ve bacteria.

Also in mycoplasma, chlamydia, helicobacter, listeria


• Erythromycin base is destroyed by gastric acid
• Estolate salt is best absorbed
• Erythromycin is NOT removed by dialysis
• Large amount excreted in bile except clarithromycin excreted in urine
• Does not penetrate CNS

USES
• DOC for corynebacterial infections (diphtheria/erythrasma)
• Chlamydial infections
• Community acquired pneumonias
• Mycoplasma and Legionella
• 2nd choice of prophylaxis against endocarditis during dental procedures with prosthetic valve (DOC-CLINDAMYCIN)

Highest risk of ADRs - Estolate


ADR:
• Cholestatic jaundice
• Gastro intestinal → motilin agonist - diarrhea

Interactions : Erythromycin inhibits CYP-450 - thus increasing concentration of theophylline/ cyclosporine/oral


anticoagulants and methylprednisolone
Digoxin level is increased by increasing bioavailability (increase rate of absorption)

CLARITHROMYCIN : Increase ACID STABILITY and increase oral absorption Spectrum similar to erythromycin but has
more activity against Mycobacterium avium complex.
• It has long T ½ permits BD dosing
• It has lesser GI adverse effect
• Used against H. pylori

AZITHROMYCIN
Clinical use/spectrum of activity are similar to erythromycin/clarithromycin, Long elimination half-life 2-3 days - OD
dosing
- Well absorbed orally
- Food retard absorption hence given I hour before or 2 hr after it. Al+3 and Mg+2, retard the rate of absorption

Does NOT inhibit CYP-450 enzyme system. So, free of drug interaction seen with clarithro/Erythromycin.

404
Antimicr obial & Antineoplastic Dr
Antimicrobial ugs
Drugs 6
CLINDAMYCIN
• Derivative of Lincomycin (Lincomycin is toxic, resembles erythromycin - and not used).
• Inhibit protein synthesis, similar to erythromycin

Uses
• DOC for severe infections caused by bacteroides and other anaerobes
• With cephalosporin/gentamicin to treat penetrating wound of abdomen/septic abortion/pelvic abscess/Aspiration
pneumonia
• Dental procedures in prosthetic valve patients
• With Primaquine for PCP- as alternative to TMP-SMZ
• With pyrimethamine for Toxoplasmosis
ADR :
• Mainly GI, Diarrhea/nausea/enterocolitis
• Pseudo membranous colitis - Caused by Toxigenic C. difficile
• Treated with MNZ/VANCO

NEWER Agents
A. Streptogramins - Quinupristin/Dalfopristin
Prolonged Post antibiotic effect (10 hrs) – It is given i.v.
Used against resistant strains of :
– Pneumococci
– S. aureus (MRSA)
– VISA (GISA)
– Vancomycin resistant strain of MRSE and E. faecium
• Dose adjustment is not required in renal failure
• Inhibits CYP 3A4

B. OXAZOLIDINONES
• Linezolid
• Active against gram + ve bacteria
• Eperezolid
• Given orally (Eperezolid not used yet)

USES : Approved for vancomycin resistant E. faecium, as a last resort for multiple drug resistant organisms.

Daptomycin : Cyclic lipopeptidase inhibitor


• Daptomycin has a spectrum of activity similar to quinupristin/dalfopristin and linezolid. Daptomycin is active against
S. aureus, including MSSA and MRSA.
MOA : • Rapid depolarization of bacterial cell membrane due to influx-efflux of ions.
• The FDA-approval of daptomycin for complicated skin and skin structure infections

AMINOGLYCOSIDES
• They are effective against gram -ve enteric bacteria
• Streptomycin, gentamicin, tobramycin, amikacin, netilmycin - Systemic
• Kanamycin and Neomycin - Topical
• They are water soluble, stable in solution and are more active in alkaline pH than in acidic pH

405
6 Self Assessment & Review Phar macology
Pharmacology
• They exhibit synergism with beta-lactam antibiotics and vancomycin (But should not be mixed together as they form
chemical complex, which causes loss of activity)
MOA : • They are irreversible inhibitors of protein synthesis. The drugs are actively transported by a carrier.
• Low pH and anaerobic conditions decrease transport. Beta-lactams and vancomycin, enhance the transport
• In the cell all bind to 30-S subunit (except strepto that binds to 12S) of ribosomal proteins, causing inhibition of
protein synthesis.
Resistance
• By developing enzymes that inactivate aminoglycoside by adenylation, acetylation or phosphorylation
• Impaired entry into bacteria
• The receptor protein on 30-S subunit may be genetically modified or deleted. Aminoglycoside-modifying enzymes occur
in both gram-negative and gram-positive bacteria.
P/K : Not absorbed from gut. May be absorbed from gut, if ulcers are present
I M or I.V. route is used clinically.
2-3 times/day dosing is now replaced by OD dosing ↑ efficacy, less side effects.
They are highly polar and so don’t readily enter cells
Eye and CNS penetration is less - inflammation T penetration in CNS.
Cleared by kidney . Excretion is proportional to creatinine clearance
In normal renal functions t 1/2 is 2-3 hrs. It may increase up to 24-48 hrs in patients with significant renal impairment
ADR : Nephrotoxic, ototoxic: both are likely if therapy is contained beyond 5 days, in the elderly, on the setting of renal
insufficiency and in presence of nephrotoxic / ototoxic drugs (loop diuretics, Vanco, Ampho-B are nephrotoxic and
should be avoided). Vestibular toxicity manifests as vertigo and loss of balance
• Neuromuscular blockade, (curare like action) can be reversed by calcium gluconate or neostigmine
• Aminoglycosides given in pregnancy can cause deafness in the new born
Uses : Gentamicin + vancomycin (or other B-lactam) for sepsis for Pseudomonas, enterobacter, serratia, klebsiella,
proteus, OD dosing is equally useful.
• Enterococcal endocarditis due to viridans streptococci / enterococci
• Staphylococcal endocarditis with nafcillin
• No Aminoglycoside to be used as a single agent
• Plasma concentration should be monitored in renal failure
Intrathecal administration of aminoglycosides are not popular today due to availability of 3rd generation cephalosporins.
Amikacin - also useful in MDR TB (even streptomycin resistance cases respond ) Gentamicin, tobramycin and netilmicin
completely interchangeable clinically.

Least ototoxic/Nephro Netilmicin preferred for prolonged use


Least vestibulotoxic Amikacin (But, Most ototoxic to AUDITORY component)
Most nephrotoxic Gentamicin / Tobramycin
Most vestibulotoxic Streptomycin, Gentamicin, Neomycin,
Most auditory Toxic Amikacin, Streptomycin, Gentamicin

Spectinomycin : Aminocyclitol antibiotic structurally related to Aminoglycosides It is given EvI


Uses : Sole use as an alternative treatment of gonorrhea in patients allergic to penicillin. No cross-resistance between
Spectinomycin and other drugs used to treat gonorrhea

SULFONAMIDES and TRIMETHOPRIM


Oldest class of antibacterial drugs
Modification of chemical structure gives rise to :
Diuretics - Thiazides, Loop diuretics

406
Antimicr obial & Antineoplastic Dr
Antimicrobial ugs
Drugs 6
Anti-glaucoma - Acetazolamide
Hypoglycemic - Sulfonylureas
Anti -leprotic - Sulfones
NSAIDs - Rofecoxib

Sulfonamides are bacteriostatic : They inhibit many Gram + ve, Gram-ve bacteria, Chlamydia and some protozoa.

Rickettsias are NOT inhibited by sulfa drugs. Their growth is stimulated by these drugs.
P/K
After absorption they bind to plasma albumin (Displacement interactions are there) partly acetylated and partly glucuronidated
(Important in premature newborns).
USES : • SDZ for meningococcal prophylaxis. I DOC is Rifampcin
• Rarely used alone, combined with TMP
• SDZ + Pyrimethamine is 1st line therapy for toxoplasmosis
• SDX + Pyrimethamine for malaria
• 5-ASA- Ulcerative colitis
Topical use : sulfacetamide eye drops/mafenide, Ag-SDZ. Mafenide can cause carbonic anhydrase inhibition, leading
to metabolic acidosis.
ADRS : Allergy 5% incidence
All sulfonamides and their derivatives eg. Acetazolamide, thiazides, Furosemide, bumetamide, torsemide, diazoxide and
the sulfonylurea oral hypoglycemic drugs are cross-allergenic.
Common ADR : Fever, skin rash, exfoliative dermatitis.
Most serious ADR - Hemolytic anaemia, aplastic anaemia, granulocytopenia, thrombocytopenia If taken during pregnancy -
Risk of hemolytic anemia kernicterus in newborn. Hemolysis is most common ADR in G6 PD deficiency patients.

Ascertain that G-6 PD deficiency or Sulpha allergy is not there


TMP and Pyrimethamine
TMP inhibits bacterial DHFR more efficiently than the mammalian while pyrimethamine has more affinity for protozoal DHFR.
DHFR converts DHF to THF that ultimately leads to synthesis of pyrimidines for DNA synthesis combination of sulfa causes
sequential block - synergism.
USES : Alone TMP is seldom used. Only for acute uncomplicated UTI
• TMP-SMZ - for PCP, shigellosis, (chloramphenicol, ampicillin in salmonellosis) complicated UTI, prostatitis.
• Respiratory tract infection caused by pneumococci, hemophilus, klebsiella, moraxella
• Single strength TMP-SMZ for recurrent UTI prophylaxis in women.
Pyri + SDZ For toxoplasmosis and leishmaniasis
Pyri + SDX for malaria.
ADRs_of Pyri/TMP: As they are antifolate drugs, they can cause
• Megaloblastic anaemia, leukopenia, granulocytopenia
• Use of folinic acid can cause treatment failure and therefore not recommended

FLUOROQUINOLONES
A. Cipro/Oflox/Lome/Levofloxacin
• They are fluorinated analogs of Nalidixic acid
• They block bacterial DNA synthesis by blocking DNA-gyrase (Topoisomerase II and Topoisomerase IV)
• They have excellent activity against Gram -ve bacteria; some newer ones have activity against gram + ve cocci.
• Levofloxacin (L-isomer of ofloxacin) has superior activity against gram + ve organism.

407
6 Self Assessment & Review Phar macology
Pharmacology
B. Cline/Gati/SPAR : More activity against gram + ve than previous group Clina has best activity against gram + ve cocci. C.
Moxi/trova - have enhanced gram + ve activity byut more hepatotoxic.
– Have excellent activity against Anaerobes
P/K :
• Longest t)/2: 18h - sparfloxacin (banned by FDA) dlt photosensitivity
• Highest oral bioavailability 98% - Gatifloxacin
• Most FQS are excreted renally
• Moxi and trova excreted non-renally
• Spar excreted 50% renally and 50% non-renally
• Alatrovafloxacin is the prodrug of trovafloxacin used parenterally.
• Moxi/Trova/Spar are contraindicated in hepatic Failure
• Bivalent and Trivalent cations decrease Quinolone absorption.
Quinolone-resistance :
• Quinolone resistance including resistance to newer fluoroquinolones has developed quickly in Staphylococcus and
Pseudomonas strains.
• The resistance mechanism involves a mutation in the drug target site, which is DNA gyrases. The mediated DNA
gyrases are not susceptible to inhibition by quinolones.
• Other resistance mechanisms involve in gram-negative strains, mutations in porins that cause the bacterial membrane
to lose its permeability to the drug in gram-positive organisms, resistance follows the development of the capability
that enables the cell to actively pump out the drug from the cell.
ADR : Trovafloxacin is associated with hepatic failure/acute hepatitis
Theophylline and quinonlone - seizures due to –I decreases theophylline metabolism
Cartilage damage
Tendon rupture (Achilles tendon)
C/I pregnancy/Lactation/Below < 8 yrs.

GREPAFLOXACIN HAD BEEN WITHDRAWN FROM THE MARKET because of cardiovascular Toxicity

Antifungal Agents
Classification of antifungal drugs based on their mechanism of action
• Inhibition of fungal cell wall synthesis – Caspofungin, Micafungin
• Bind to fungal cell membrane ergosterol – Amphotericin B, Nystatin, Hamycin
• Inhibition of ergosterol + Ianosterol synthesis – Terbinafine, Naftifine, Butenafine
• Inhibition of ergosterol synthesis – Miconazole, Clotrimazole, Ketoconazole, Fluconazole, Itraconazole and Voriconazole.
• Inhibition of nucleic acid synthesis – 5-Flucytosine
• Disruption of mitotic spindle and inhibition of fungal mitosis – Griseofulvin
• Miscellaneous agents – Ciclopirox, Tolnaftate, Haloprogin, and Undecylenic acid.

SYSTEMIC ANTIFUNGAL DRUGS

Amphotericin-B
It is a polyene macrolide (other one is Nystatin). It is insoluble in water and is therefore prepared as a colloidal suspension
with sodium desoxycholate for i.v. injection.
P/K • Not absorbed from gut. So, as an oral agent, it is useful only in the treatment of intestinal fungal infection.
• Mostly metabolized in the liver and partly excreted unchanged in urine.
• Dose adjustment is NOT required in Renal / Hepatic impairment

408
Antimicr obial & Antineoplastic Dr
Antimicrobial ugs
Drugs 6
• Only 2-3 % reaches CNS; T1/2 is 15 days.
MOA • Binds to ergosterol and alters permeability of the cell by forming Amphotericin-B associated pores in cell membrane.
• The pores allow leakage of vital ions and macromolecules from the cell.
• Human toxicity can result from binding to cholesterol or cell membrane and damaging it only in very high doses.
ADRs
Immediate (following IV infusion)
• Fever, chills, muscle spasm, vomiting and hypotension
• Premedication with paracetamol, corticosteroids, pethidine and antihistamincs are useful.
Late • Renal damage is the most significant reaction
- Occurs when the total cumulative dose exceeds 4 gm in a course of treatment
- Manifests as renal tubular acidosis and severe K+ and Mg+ wasting
- Infusion of normal saline decreases this toxicity.
• Anemia can result from decreased erythropoietin production in the damaged renal tubular cells
• Arachnoiditis, seizures are associated with intrathecal administration

Antifungal spectrum : Truly Broad spectrum antifungal agent. Candida, cryptococcus, histoplasma, blastomyces, coccidioides,
aspergillus
USES : • Due to Broad spectrum of action, Amphotericin B remains the antifungal agent of choice in nearly all life
threatening mycotic infections.
• Often used as an inducing agent in serious infections and later replaced by other agents (e.g., azoles) for
chronic therapy especially in fungal pneumonia, cryptococcal meningitis, sepsis syndrome due to fungal
infections.
• Often also used for empiric therapy for selected patients in whom the risks of leaving a systemic fungal
infection untreated are high. The most common such patient is the cancer patient with neutropenia who
remains febrile on broad-spectrum antibiotics.
• Mycotic corneal ulcers and keratitis - topical drops and sub conjunctival injections.
• Candiduria responds to bladder irrigation and reduces systemic toxicity.
• Intrathecal therapy for fungal meningitis is now used only in cases who have not responded to other agents.

5- Flucytosine
• It is related to the anticancer agent, 5-FU structural similarity
• Its spectrum of activity is narrower than Amphotericin B
P/K • Available for oral use
• Well absorbed orally, peaks 1-2 hr after dose and penetrates well into all tissues including CSF
• Excreted in urine
• Toxicity is more in renal insufficiency and AIDS patients
MOA • Enters the fungal cell by enzyme, cytosine permease in fungus not in mammal cells
• Converted to 5-FdUMP and FUTP that inhibit DNA and RNA synthesis respectively (Human cells can’t convert the
parent drug into active metabolites)
• Synergy seen with Amphotericin B due to enhanced penetration of 5-FU in Amphotericin B damaged cells
• Resistance develops to monotherapy rapidly. So, it should always be combined with other drugs.
ADRs • Occurs due to metabolism to the anticancer drug, 5-FU by gut flora
• Bone narrow toxicity, anemia, neutropenia, thrombocytopenia are common
Uses • Restricted to meningitis by Cryptococcus neoformans and Chromoblastomycosis.
• Combined with Amphotericin B for cryptococcal meningitis and itraconazole for chromoblastomycosis.

409
6 Self Assessment & Review Phar macology
Pharmacology
AZOLES
Synthetic compounds:
a) Imidazoles - Ketoconazole, miconazole (Topical), clotrimazole (Topical)
b) Triazoles – Fluconazole, itraconazole, voriconazole
MOA • Reduction of fungal ergosterol synthesis by inhibition of fungal cytochrome P450 enzymes. Imidazoles have a
lesser degree of specificity than the triazoles accounting for higher degree of drug interactions and side effects.
P/K • All are orally absorbed, fluconazole can be given i.v. Absorption is very high for Fluconazole.
Fluconazole attains high CSF concentration, 70% of plasma
Fluconazole is eliminated in urine. Others are eliminated in bile.
USES • Broad spectrum
• Candida, cryptococcus, blastmycosis, coccidioidomycosis histoplasmosis and dermatophytes are sensitive.

Features of different azoles:


A. Ketoconazole
- It is much less toxic than amphotericin B but more toxic than fluconazole or itraconazole.
- The most common side effects are nausea and vomiting.
- Causes maximum inhibition of mammalian CYP450 enzymes; the dangerous interaction with terfenadine, astemizole
and cisapride resulting in torsades de pointes has resulted in withdrawal of the drug in many countries.
- It interferes with adrenal and gonadal steroid synthesis resulting in gynaecomastia, infertility and menstrual
irregularities.
USES : Dermatophytosis, mucocutaneous candidiasis and coccidioidomycosis
D/I : • Ketoconazole is absorbed best at lower gastric pH; drugs like H2 blockers, PPIs, antacids decrease absorption.
• Inhibition of metabolism of certain drugs causing toxicity e.g. cisapride / cyclosporin-A
• Rifampicin enhances the metabolism of ketoconazole

B. Itraconzole
• Orally absorbed, absorption is increased by food and low gastric pH
• Inhibits CYP-450 but to a lesser degree than ketoconazole
• Does not affect mammalian steroid metabolism
• Rifampicin reduces its bioavailability
• Most potent of available azoles
• Like Ketoconazole, its CNS penetration is poor
• Agent of choice for dimorphic fungi histoplasma / blastomyces and sporothrix.
• It is useful in Aspergillus but the agent of choice for this disease is Voriconazole.
C. Fluconazole :
• High water solubility and good CSF penetration
• Maximum CNS penetration among available azoles (70%)
• Least effect on hepatic enzymes, so minimum drug interactions
• Widest therapeutic index among azoles
• the azole of choice for treatment and prophylaxis of cryptococcal meningitis; is also the agent of choice for
mucocutaneous candidiasis.
• Used both orally or l.V.
• Activity against dimorphic fungi is less
D. Voriconazole :
- High water solubility; good oral absorption with bioavailability of 90%
- Used both orally or I.V.

410
Antimicr obial & Antineoplastic Dr
Antimicrobial ugs
Drugs 6
- Visual disturbances including blurring and changes in color vision occur in upto 30% patients. These changes occur
immediately after a dose and resolve within 30 minutes.
- Similar to itraconazole in spectrum of action having excellent activity against candida species.
- Drug interaction profile due to hepatic enzyme inhibition is similar to itraconazole.

Drug interactions involving oral azole antifungal drugs


Effect Azole(s)
Decreased absorption of azole
Antacids Ketoconazole, itraconazole
H2 receptor antagonists Ketoconazole, itraconazole
Sucralfate Ketoconazole, itraconazole
Omeprazole Ketoconazole, itraconazole
Didanosine (oral) Itraconazole

Increased metabolism of azole


Isoniazid Ketoconazole
Rifampin Ketoconazole, itraconazole, fluconazole
Phenytoin Ketoconazole, itraconazole
Carbamazepine Itraconazole
Phenobarbital Ketoconazole, itraconazole

SYSTEMIC DRUGS FOR MUCOCUTANEOUS INFECTIONS


A. Griseofulvin :
• Fatty foods increase absorption
• Binds to developing skin, nail and hair and protects them for fungal infection
• MOA: It interferes with mitosis – leading to multinucleate and stunted fungal hyphae; also causes abnormal metaphase
configurations. It does not cause typical metaphase arrest like vinca alkaloids and colchicines; rather the daughter nuclei fail
to move apart.
• Deposited in newly forming skin where it binds to keratin and protects skin from new infection (fungistatic agent). As
the action is to prevent infection of new skin structures, it must be administered for 2-6 weeks to allow replacement of
infected keratin by the resistant structures. Nail infection requires months of treatment.
D/I : with warfarin, phenobarbital as Griseofulvin inhibits their metabolism
B. Terbinafine :
• Very useful in treating onychomycosis (more effective than griseofulvin or itraconazole)
• Binds to the keratin and is fungicidal
• Inhibits -ergosterol biosynthesis by inhibiting the fungal enzyme squalene epoxidase (No effect in CYP -450 system)
• Leading to accumulation of squalene which is toxic to the fungi.
• Most serious ADR - is HEPATITIS

Topical agents
Nystatin
• Polyene macrolide like amphotericin B
• Causes pore formation in the fungal cell wall
• Too toxic for parenteral use (Liposomal formulation is under development)
• Uses include candidial infection - oropharyngeal thrush, vaginal candidiasis & intertriginous infection.

411
6 Self Assessment & Review Phar macology
Pharmacology
Topical Azoles: Clotrimazole/Miconazole
• Oral Clotrimazole Troches are pleasant in taste than nystatin for treating oropharynglal thrush.
• Creams (of both) are used for dermatophytes.
• Pessaries available for vaginal infection.
• Shampoo and topical forms are used in seborrhic dermatitis and and pityriasis versicolor.
ANTIVIRAL DRUGS
ANTI HERPES and ANTI CMV AGENTS
A. Acyclovir :
- It is a prodrug, Needs triphosphorylation for activation
USES :• Primary and recurrent genital herpes
• IV DOC for HSV encephalitis and Neonatal HSV infection HSV-1&2, VZV
For VZV, higher doses are necessary
ADR : • Well tolerated - nausea, diarrhea, headache, neurotoxicity or renal failure

B. Ganciclovir :
• Needs triphosphorylation for activation
• Inhibits viral DNA polymerase
• Highly active against CMV
• Acyclovir resistant HSV are cross resistant to this.
• Oral bioavailability is 6-9%. Can be used i.v., orally or via intraocular implant.
USES : • DOC for CMV retinitis in AIDS patients. Intravitreal injection is most effective- Dual therapy (foscarnet +
ganciclovir) is superior to either of the individual agent
• Also used in CMV colitis, esophagitis and prevention of CMV infection in organ transplantation patients
ADR : • Most common is bone marrow suppression esp. neutropenia (20-40% paients).
• CNS toxicity including seizures.
C. Foscarnet :
- Inhibits viral DNA polymerase, RNA polymerase and HIV-reverse transcriptase without requiring activation by
phosphorylation.
P/K – I.V. use only (poor g.i. absorption and intolerance)
_ This drug is deposited in the BONE
_ Excretion occurs in the urine
USES : • Approved for CMV retinitis and acyclovir resistant HSV infection.
• Combination of ganciclovir + foscarnet is superior to individual agents
ADRs : • Renal insufficiency
• Hypo or Hypercalcemia, Hypo or Hyperphosphatemia
• Penile ulceration
• Headache, hallucinations, seizures.
FOMIVIRSEN
• Selectively active against CMV
• Injected intravitreally for treatment of CMV retinitis in AIDS patients
ADRs : • Iritis/Uveitis/Intraocular hypertension

OTHER ANTIVIRAL DRUGS


Anti Influenza Drugs :
A. Amantadine and rimantadine are effective in preventing influenza-A infection. They reduce the severity and duration of
influenza, when administered within 48 hrs of onset.

412
Antimicr obial & Antineoplastic Dr
Antimicrobial ugs
Drugs 6
ADRS : • Gastro-intestinal intolerance.
• CNS - Nervousness, light-headedness, difficulty in concentrating.
B. Zanamivir and oseltamivir
They inhibit neuraminidase, which is an essential glycoprotein for viral replication and release.
USES : • Acute and uncomplicated influenza. They are active against both influenza A and B
• Zanamivir is given intranasally. Oseltamivir is given orally. This is a prodrug that gets activated in the liver.
C. Ribavirini Guanosine analog that is phosphorylated intracellularly in the host cell. It appears to interfere with GTP synthesis.
It inhibits replication of wide range of DNA and RNA viruses including influenza A and B, parainfluenza, RSV, paramyxo,
HCV and HIV-1
ADR : Dose dependent hemolytic anemia. Depression and suicidal tendencies, teratogenicity and mutagenicity.
USES : Nebulized ribavirin for RSV bronchiolitis, influenza A and B infection, Lassa fever and other hemorrhagic fevers.

ANTIRETROVIRAL DRUGS
A. NUCLEOSIDE REVERSE TRANSCRIPTASE INHIBITORS (NRTIs)
Zidovudine (AZT) : Orally well absorbed and distributed well into various tissues. Eliminated primarily in the urine as
glucuronidated product. AZT has been shown to decrease disease progression. In HIV infected mothers, treatment
recommended between 14-35 weeks of gestation followed by infusion during labor and the new born given AZT syrup from
birth to 6 weeks has been shown to decrease vertical transmission.
Lamivudine is shown to enhance susceptibility in AZT resistant strain.
ADR : Myelosuppression (commonest) leading to anemia and neutropenia. Others include headaches, insomnia,
hyperpigmentation of nails and myopathy.

Didanosine: Antiretroviral activity is potentiated by Hydroxyurea (given orally). Food reduces absorption. Eliminated by
urine.
D/I : Co-administration of ganciclovir increases AUC of didanosine and decreases AUC of ganciclovir.
ADRs : Dose dependent pancreatitis,/ neuropathy, cardiomyopathy, retinal damage, optic neuritis, lactic acidosis, hepatitis.

Lamivudine : Acts synergistically with zidovudine and stavudine in both AZT-sensitive and AZT- resistant HIV strains.
Currently, approved for chronic Hepatitis B infection. It is the safest NRTI.
D/I : AUC increased with TMP-SMZ for PCP.

Zalcitabine : Acts synergistically with other Anti HIV drugs NOT to be used as monotherapy.
ADR : Peripheral neuropathy accentuated with agents that cause neuropathy i.e. INH, STAVUDINIE, DIDANOSINE. Nephrotoxic
drugs (Aminoglycosides/Ampho-B) increase it concentration and risk of neuropathy.
Stavudine : Highly effective, excreted by kidney.
Major dose related toxicity is peripheral sensory neuropathy
D/I : Zidovudine reduces phosphorylation and hence effectiveness, they should not be combined together.

Abacavir : Most effective NRTI


Metabolized by alcohol dehydrogenase and glucuronosyl transferase
ADRs : Hypersensitivity reaction occurs in 1st 6 weeks may include multiple organ systems.

B. NON-NUCLEOSIDE REVERSE TRANSCRIPTASE INHIBITORS (NNRTIs)


— Nevirapine : High oral bioavailability. No interaction with food, metabolized by CYP 3A4
USES : Used in combination with other agents as combination therapy.
Recently, it is shown to decrease vertical transmission from mother to fetus followed by treatment of the new born

413
6 Self Assessment & Review Phar macology
Pharmacology
ADRs : Severe life threatening skin rash including, Stevens Johnson’s syndrome and toxic epidermal necrolysis (TEN)
Fulminant hepatitis (LFT monitoring strongly recommended)
It induces CYP 3A4 - causing self-metabolism and decreases levels of indinavir, saquinavir, ritonavir, OCs.
— Efavirenz: - Given OD - tl/2 40-55 hrs
Bioavailability is increased with fatty foods - Metabolised by CYP 3A4 and 2B6
CSF levels 2 times higher than free plasma concentration
ADRs :
• CNS related including dizziness, drowsiness, insomnia, amnesia, confusion
• Skin rash
• Auto induction of its own metabolism
• Drug interactions occur with agents’ inducing/inhibiting some enzyme
• High rates of teratogenicity seen in monkeys, so avoid pregnancy.

Others : Delavirdine, Atevirdine, Loviride, Enrivirine, Caprivirine

C. PROTEASE INHIBITORS (Pi’s)


During later phases of HIV growth cycle, Gag and Gag-pol gene products are translated into polyproteins and then,
become immature budding particles. Protease is responsible for clearing these precursor molecules to produce final
structural proteins of the mature virion core. Protease is essential for the production of mature infectious virions during
HIV replication.
Protease inhibitors prevent the clearing of precursor molecules for a synthesis of mature infectious particles.

COMMON ADRs OF ALL Pi’s (Like steroids except osteoporosis)


1. Syndrome of altered body fat distribution - Buffalo hump, Truncal obesity with peripheral and facial atrophy
2. Insulin resistance
3. Hyperlipidemia
4. All Pi’s have been associated with increased spontaneous bleeding in patients of Hempophilia A or B

(i) SAQUINAVIR :
Adverse events are mild diarrhea, nausea, abdominal discomfort
Saquinavir undergoes extensive first-pass metabolism by CYP-3A4 in the gut.
Saquinavir levels are increased by Ritonavir, nelfinavir, and delavirdine, ketoconazole, clarithromyin and grape fruit juice.
Ritonavir + Saquinavir combination is preferred because FPM of saquinavir is decreased by ritonavir Saquinavir levels
are decreased by rifampicin, efavirenz, nevirapine
• Saquinavir should not be combined with cisapride, triazolam, midazolam and ergot derivatives, as CYP 3A4 inhibition
by saquinavir will increase their blood levels causing toxicity

(ii) RITONAVIR :
The most common adverse events are GI disturbances, (N, V, Abdominal pain), Paresthesia (Circumoral and peripheral),
altered taste, elevated serum aminotransferase levels and hypertriglyceridemia.
Ritonavir inhibits CYP 3A4 and CYP 2D6
Ritonavir is an essential component of HAART as it enhances the concentration of other anti-HIV drugs

(iii) INDINAVIR :
ADRs :Hyperbilirubinemia and Nephrolithiasis; water intake should be increased at least 1500 ml/day.
Indinavir is a substrate for CYP 3A4 and an inhibitor of it too.

414
Antimicr obial & Antineoplastic Dr
Antimicrobial ugs
Drugs 6
(iv) AMPRENAVIR : Fatty food decrease absorption.
Studies have shown the increased efficacy with 2 NRTIs - It is on inhibitor of CYP 3A4

Newer Antiretroviral agents


Include HIV Entry Inhibitor or Fusion Inhibitor (e.g. ENFURVITIDE) and HIV Nucleotide reverse transcriptase inhibitor
TENOFOVIR.
Integrase inhibitors—Reltigravir

Safest
NRTI Lamivudine
PI Ritonavir
NNRTI Efavirenz

1. ANTIMYCOBACTERIAL AGENTS
Antimycobacterial infections are difficult to cure as -
They are slowly growing organisms, as antibiotic activity ATT depends upon how rapidly they are dividing
They can be dormant enough to resist ATT
Drug penetration is difficult into the lipid-rich membrane of mycobacteria.
During infections, a substantial number remain intracellular in macrophage
Mycobacteria are notorious to develop resistance of any single drug, requiring combination therapy
INH :
• Structurally- similar to pyridoxine
• Most active of all ATTs
• Bactericidal both in-vivo and in-vltro
• It penetrates phagocytic cells, thus active against both intracellular and extracellular organism
MOA :
• INH is a prodrug activated by mycobacterial KatG (mycobacterial catalase peroxides)
• INH inhibits mycolic acid synthesis by forming covalent bond with two carrier proteins AcpM and KasA
• High level of resistance occurs due to mutations or deletion of KatG
P/K :
• Given orally readily absorbed Metabolism by acetylation in liver by N-acetyltransferase
• Hepatotoxicity is an idiosyncratic reaction and is seen only in rapid acetylators. Excretion is in urine
• ↑ Hepatotoxicity if cecetyl concentration > 2.5 = Fast
• ↑ Neurotoxicity if acetylcen concertration < 2.5 = Slow
• No dose adjustment required in renal failure, but in severe hepatic failure dose to be reduced by 50-70%
USES :
• For treatment, always given in combination
• As a single agent INH is recommended in
– Individuals at risk of developing infection after being treated such as children
– Close contacts of a case
– Recent converters (test + ve within 2 years)
– Immunocompromised individuals (HIV infected/AIDS)
ADR
• Drug induced SLE
• Hepatitis (most common and major effect) fatal in 1% of patients -discontinue therapy. Risk of hepatitis occurs with
age, rare before 20 yrs. Risk of hepatitis is more in alcoholics, pregnancy and postpartum period

415
6 Self Assessment & Review Phar macology
Pharmacology
• Peripheral neuropathy 10-20% (seen above 5 mg/kg/day) more occurs in slow acetylators. INH promotes excretion of
pyridoxine, reversed by pyridoxine 10 mg/day.
• CNS toxicity includes memory loss, psychosis and seizures (pyridoxine may respond to this)
• Pyridoxine deficiency anemia
2. RIFAMPICN
MOA :
• Binds to beta-subunit of bacterial DNA dependent RNA polymerase, thus inhibiting RNA Synthesis
• Bactericidal for mycobacteria
• It readily penetrates into most tissues and phagocytosed bacteria and even those are sequestered.
• It is excreted in the bile and undergoes enterohepatic circulation
• CNS penetration is poor, increased in meningitis
USES:
• For mycobacterial infections, it is used along with INA, PZA and ethambutol to prevent drug resistance
• Alternative to INH prophylaxis in TB
• Leprosy, prevention of meningococcal, H. influenza type-B infection
• With a second agent to eradicate staphylococcal carriage
• Serious infections such as osteomyelitis and prosthetic valve endocarditis, along with penicillins/ aminoglycosides
• With ceftriaxone or vancomycin for meningitis caused by penicillin resistant pneumococci
ADRs:
• Harmless orange discoloration to all body fluids
• Thrombocytopenia, nephritis, cholestatic jaundice, hepatitis
• Light chain proteinuria
• Flu like syndrome
• Induction of CYP-450

3. ETHAMBUTOL
• Bacteriostatic
• Inhibits bacterial enzyme ARABINOSYL TRANSFERASE which is responsible for polymerization of arabinoglycan,
an essential component of mycobacterial cell wall. Disruption of arabinoglycan synthesis alters cell barrier and
enhances activity of lipopophilic drugs such as Rifampicin and Ofloxacin.
P/K:
• Well absorbed from gut
• - 20% excreted in feces and 50% in urine. In renal failure, dose reduction required.
• Meningeal inflammation enhances CSF concentration
ADR :
• Retrobulbar neuritis causing loss of visual acuity
• Red-green color blindness
• Require periodic testing of vision
C/l : Relatively contraindicated in children < 6 yers age where checking visual acuity and color vision difficult.

4. PYRAZINAMIDE
(Bactericidal to actively dividing organism) Action is pH dependent i.e. at neutral pH it is ineffective and at 5.5 it inhibits
mycobacteria. It is taken up by macrophages and inhibits those phagocytosed bacteria.
MOA :
• It is converted to pyrazinoic acid by bacterial enzyme pyrazinamidase (Inhibit mycolic acid).

416
Antimicr obial & Antineoplastic Dr
Antimicrobial ugs
Drugs 6
USES :
• PZA is used as front line agent in 6 months regimen as sterilizing agent acting on intracellular organisms that cause
relapse.
• PZA + ofloxacin (or ciprofloxacin) prophylaxis for contacts of MDR TB.
ADRs :
• Most common - Hyperuricemia (dose reduction not required)
• Most serious – Hepatitis

5. STREPTOMYCIN
• Most TB bacilli are inhibited by streptomycin
• Non tuberculosis mycobacteria other than MAC and M.Kansasii are resistant.
• It is mainly active against EXTRACELLULAR ORGANISMS
• It meningitis it achieves adequate concentrations.
USES :
• Now much restricted: used only in life threatening infections such as meningitis, disseminated disease and TB
resistant to other drugs
ADR :
• Ototoxic and Nephrotoxic. Increased in elderly.
• Dose adjustment in renal failure
• Toxicity can be reduced by limiting total duration to no more than 6 months.

ALTERNATIVE (SECOND-LINE) AGENTS


Alternative therapy is considered:
i) In case of drug-resistance to 1st line agents
ii) Failure to conventional therapy
iii) When expert guidance is available to deal with toxic effects

1. Ethionamide :
• Chemically related to INH, inhibits synthesis of mycolic acid.
• Available as oral formulation, metabolized in liver
• CSF concentration is equal to plasma
2. Capreomycin :
• Protein synthesis inhibitor
• Given IM in treating MDR - TB (including streptomycin, amikacin resistant)
• Ototoxic, nephrotoxic (Tinnitus, deafness, vestibular disturbance)
• Injections cause local pain and sterile abscess.
3. Cycloserine :
• Inhibitor of cell wall synthesis
• Used against MDR - TB, cleared renal
ADRs : Peripheral neuropathy and CNS dysfunction including depression and psychosis. Pyridoxine 150 mg per day
reduces these symptoms
4. PAS :
• Folate synthesis antagonist
• Structurally similar to PABA and sulfa drugs.
• Cleared in the kidney - can cause crystalluria

417
6 Self Assessment & Review Phar macology
Pharmacology
ADR : • Gastro intestinal - Nausea, gastritis, diarrhea, peptic ulcer and food reduces G.I irritation
• Hypersensitivity reactions - joint pain, hepatitis, granulocytopenia - need temporary or permanent withdrawal.
5. Kanamycin/ Amikacin
Kanamycin is useful in streptomycin resistance (too toxic and better options (capreomycin, neomycin) available, kana is
absolute.
• Amikacin is useful in MDR-TB
• Also active against atypical mycobacteria
• No cross-resistance between streptomycin and amikacin
• Indicated for streptomycin resistant MDR - TB, along with at least 1-2 agents active against MDR-TB
6. Fluoroquinolenes (Ciprofloxacin, Levofloxacin)
• Active against typical and atypical mycobacteria
• Ofloxacin was used in the past, Levofloxacin is superior (This is the L-isomer of ofloxacin)
• Levofloxacin is more active against typical mycobacteria and ciprofloxacin is more active against atypical ones
• Indicated in MDR-TB
• Prophylactic use in MDR-TB with PZA Rifabutin: Cross resistance seen with Rifampicin
• It is a less potent inducer of CYP-450 enzymes than Rifampicin
• Preferred in TB in AIDS patients where protease inhibitor and NNRTI (like efavirenz) where rifampicin is a potent
inducer of metabolism
USES : • Prevention and Treatment of disseminated atypical mycobacterial infection in AIDS Patients
7. Rifapentine :
• MOA like that of Rifampicin
• Cross resistance between rifampicin, rifabutin
• Potent inducer of Cyp-450
• Its metabolite desacetyl rifapentine is also active
• No advantage over rifampicin
8. Rifabutin
• It is less potent inducer of CYR-490
• Preferred in TB in AIDS patients where proteas inhibitor NNRTI are used.

DRUGS USED IN LEPROSY


1. Dapsone
MOA :
• Inhibition of Folate synthesis
• Resistance develops to dapsone when used alone combination with rifampicin and clofazimine is necessary
P/K :
• Dapsone is well absorbed from the gut and distributed throughout the body, t1/2 is 1-2 Days
• The sulfones deposited in skin, muscles, liver and kidney
• Acetylated in liver and excreted in kidney
• DAPSONE RESISTANT LEPROSY IS SEEN IN FAST ACETYLATORS
ADR :
• Hemolysis in G-6PD deficiency
• Methemoglobinemia
• ENL is suppressed by corticosteroids and/TNF-alfa.

418
Antimicr obial & Antineoplastic Dr
Antimicrobial ugs
Drugs 6
2. Clofazimine
• It is stored in RE system and then slowly released
• Used in combination with dapsone
ADR : Skin discoloration ranging from reddish brown to black

ANTIMALARIALS - CHLOROOUINE (CHQ)


• Used for both prophylaxis and treatment of drug sensitive malaria
• Loading dose is required as it has a very large V.D. (100-1000 L/Kg)
• Completely absorbed from the gut
• Initial t l/2 is 3-5 days and, terminal half life is 1-2 months
Actions : • CHQ is a schizonticide and gametocide against all except P.falciparum
• CHQ is not active against liver stage parasites
MOA : • Prevents polymerization of Heme to Hemozoin free heme is toxic
USES : • Treatment Drug of choice for Non-falciparum and sensitive malaria CHQ does not eliminate dormant liver
forms of P. vivax and P. ovale - ADD PRIMAQUNE
Prophylaxis :
• Chemoprophylaxis agent of choice in malarious regions without P.falciparum
Treatment of Amebic liver abscess :
• High concentration reaches liver and can kill amoeba.
• To be used in amebic liver abscess in combination with metronidazole, when initial therapy with metronidazole fails
• CHQ is not useful in extra hepatic forms of amebiasis
ADRS : • Hemolysis, in G-6PD deficient patients
• Agranulocytosis, seizures, exfoliative dermatitis
• Alopecia and bleaching of hair
• IV dose can cause hypotension and cardio-respiratory arrest
C/I : • Psoriasis, porphyria (absolute)
• Myopathy, retinal and visual field abnormalities (relative)
• It is considered safe in pregnancy and in children.

Quinine and Quinidine


• 1st line therapy for falciparum malaria
• Resistance though not seen, but is increasing
P/K : • Orally absorbed, can be given I.V.
• Quinidine is D-Quinine. Equally effective
• T½ 18 hrs (Quinidine has a t l/2 of 10-12 h)
• Metabolised in liver and excreted in urine

Antimalarial Action
• Rapidly acting, highly effective blood schizonticidal for all 4 species of Malaria (only RBC stage)
• Gametocidal against P. vivax, P. ovale but NOT P. falciparum
• Not effective against liver stage of any malarial species.
USES :
• DOC for severe falciparum malaria, given i.v. (monitor cardiac function as it is cardio depressant). Oral therapy of Quinine
4- Doxycycline reduces the duration of treatment
• Nocturnal leg cramps
• Babesiosis - Quinine is first line therapy for the treatment of human babesiosis (Babesia microti) along with clindamycin.

419
6 Self Assessment & Review Phar macology
Pharmacology
ADR :
• Cinchonism, (if severe, stop therapy)
• Hypersensitivity reaction
• Hemolysis, Bone marrow depression
• Hypoglycemia due to increased release of insulin
• Uterine contractions, especially in 3rd trimester (despite this Quinine remains the DOC for severe falciparum malaria
during pregnancy)
• Hypotension, if given rapid i.v.
• Q-T prolongation.
C/I :
• Cinchonism, hemolysis, hypersensitivity
• Heart block
D/I :
• Aluminum containing antacids retard absorption
• Mefloquine enhances heart block

Mefloquine
• Chemoprophylactic agent of choice in malaria endemic zones with CHQ resistant strains
P/K : Only given orally, T-max is 18 hrs, eliminated slowly given as weekly chemoprophylaxis.
Effects : Strong blood schizonticidal, No effect on hepatic stages
USES :
• As a prophylactic agent in most strains of P. falciparum, where CHQ resistance is common
• Used for treating most forms of Falciparum malaria
• Not used for treating severe or complicated malaria where quinine is superior.
ADR :
• Most serious are neuropsychiatric, depression confusion. Acute psychosis, seizures
• Cardiac conduction defects - Arrhythmias
C/I :
• H/O epilepsy, psychiatric disorder, Arrhythmias and heart block
• Should not be administered with quinine/quinidine

PRIMAQUINE :
DOC for eradication of dormant liver forms of P. vivax and P. ovale (hypnozoites)
• Radical cure following standard therapy with CHQ
• Terminal prophylaxis of vivax and ovale malaria after standard chemoprophylaxis
• Gametocidal action : making non-infective to mosquitoes
• Pneumocystis carinii infection : clindamycin + primaquine
ADR :
• Serious ADRs include leuopenia/leukocytosis/agranulocytosis
• Cardiac conduction defects
• Hemolysis
C/I : Avoid in pts with H/O agranulocytosis or methemoglobinemia

ANTI NEOPLASTIC DRUGS


• Cancer cured by chemotherapy includes testicular, diffuse large cell lymphoma, hodgkin’s, choriocarcinoma as well as
certain childhood lymphomas such as ALL, Burkitt’s, Wilm’s, and embryonal rhabdomyosarcoma.

420
Antimicr obial & Antineoplastic Dr
Antimicrobial ugs
Drugs 6
• Cancers refractory to treatment - colon and lung cancer.
• Besides chemotherapy, and Hormone therapy, newer strategies that screens promising includes.
Gene therapy and biological response modifier therapy (BRM) is. IFNs and 1L-2
The Cell Cycle
• G0 - Resting i.e. no further division resistant to all drugs
• G1 - 40% of time, synthesis of cellular components needed for DNA synthesis
• S - 39% of time, synthesis of DNA
• G2 -19% of time. Synthesis of cellular components for mitosis
• M - 2% of time; Mitosis
Cell cycle specific agents (CCS)
• Antimetabolites (cytarabine) 5-FU, 6.MP, MTX, 6-TG)
• Bleomycin
• Podophyllotoxins -Etoposide, Teniposide
• Plant alkaloids - VCR, VBL, VDS, and VRL
• Taxanes
Cell Cycle Non-specific (CCNS)
• Alkylating agent e.g. Busulfan, CPM, mechlorethamine, melfalan, thiotepa.
• Antibiotics -ACT-D, Daunorubicin, Doxorubicin, Plicamycin, Mitoxantrone.
• Cisplatin/Carboplatin/Oxaliplatin
• Nitrosoureas - BCNU, CCNU, M-CCNU
Dosing Principles
• Drugs: more effective in combination (may be synergistic)
• More effective if drugs do not share common mechanisms of resistance.
• More beneficial if drugs do not overlap in major toxicities.
• Drugs should be in administered near their maximum individual doses
• Drugs should be administered as frequently as possible - to maximize dose intensity (dose per unit time} limiting tumor
regrowth.
• Desirable; maximum cell kill with each treatment cycle, using the highest those possible, repeating doses as frequently
as tolerable
Resistance to Chemotherapy
• Primary, when NOT exposed to the drug before
• Secondary, i.e. initially sensitive, later resistant most common is expression of MDR-1 gene for a cell surface glycoprotein
involved in efflux of drug MDR can be experimentally blocked by CCB drugs like verapamil

1. ALKYLATING AGENTS:
Cyclophosphamide, Mechlorethamine, melfalan and chlorambucil, thiotepa, nitrosoureas
MOA : Cell death primarily occurs through ALKYLATION OF DNA, within the nucleus
Besides this, they also react with - SH, Amino, -OH, -COOH, and - PO4 groups
Besides alkylation, CARBAMOYLATION of Lysine residues of proteins through formation of isocyanates.
Major site of alkylation in the DNA is the N7 position of Guanine.
Effects : Dose related toxicity particularly in the rapidly growing tissues like, bone marrow, gonads GI tract, hair follicles
etc.
• Nausea and vomiting. (30-60 min after IV dose)
• Tissue necrosis by mechlorethamine, nitrosoureas
Cyclophosphamide is to be activated in liver to 4-OH phosphamide + aldophosphamide

Phosphoramide mustard (Active)
+

421
6 Self Assessment & Review Phar macology
Pharmacology
Acrolein - (Toxicity)
• Most important (and serious) toxicity of alkylating agents is Bone marrow suppression
– WBC’s first to be affected (Short life) in 10-12 days, recovery in 3-6 weeks
– Megakaryocytes and platelets are affected next
– RBCs are least affected (long life)
After hematologic recovery, these agents can be given again
• Other toxicities include ovarian or Testicular failure and development of acute leukemia’s (still rarer)
• Busulfan has high specificity for granulocyte series hence green for CML, DOC is imatinib mesylate
• Repeated hematologic monitoring is necessary

A. NITROSOUREAS :
• All require biotransformation, by non-enzymatic decomposition to derivatives having alkylating and carbamoylating
activities.
• All are highly lipid soluble and hence penetrate BBB. Hence, useful in treating CNS tumors.
• Naturally occurring sugar containing nitro source is streptozocin useful in treating insulin secreting islet cell carcinoma
of pancreas (Insulinomas).
• Related Drug, probably acting through Alkylation (Procarbazine, Dacarbazine, Cisplatin, Carboplatin, Oxaliplatin,
Altretamine).

PROCARBAZINE :
Useful in Hodgkin’s disease
• It is teratogenic, mutagenic and leukemogenic
• Exact MOA not clear. Inhibits DNA, RNA and protein synthesis
• Bone marrow suppression, nausea and vomiting are predictable.
• Hemolytic anemia Disulfiram like reaction. CARCINOGENCITY
• Most common secondary cancers d/t procarbazine > Dacarbazine

CISPLATIN, CARBOPLATIN and OXALIPLATIN


• Exact MOA not clear (Probably by alkylation)
• Binds to N-7 of guanine of DNA
• Nausea and vomiting
• Renal dysfunction, acoustic nerve dysfunction occurs, can be minimized by saline infusion or mannitol or diuretics
Uses :
• Genitourinary cancers (Testicular, ovarian, bladder)
• With vinblastine and bleomycin, curative for Non-seminomatous testicular cancer.
I.V. Therapy is associated with more nausea and vomiting (most common)
Vomiting, two medullary centers
• Vomiting center
– stimulated by CTZ output
– stimulated by cerebral input (visual/ olfactory) and
• Chemoreceptor trigger zone (CTZ)
• Directly stimulated by toxins or drugs,
• Neurotransmitters (e.g. dopamine) stimulate the vomiting center
Not all chemotherapeutic drugs are equally emetogenic:
Most severe : (Platinum compounds)
e.g. Cisplatin, Carboplatin, Oxaliplatin Severe:

422
Antimicr obial & Antineoplastic Dr
Antimicrobial ugs
Drugs 6
Severe
• dacarbazine (DTIC) and procarbazine
• nitrosoureas
• mechlorethamine
Relatively mild
• antimetabolite
• methotrexate
• Flurouracil (5-FU)
TT
1. 5HT3 antagonist
• Ondansetron, pilanosetron
2. D2 receptor blockes
• Domperidone, metoclopramide
Hematopoietic dysfunction may be ameliorated by:
• Discontinuation of treatment
• Administration of RBC/Platelets/Packed cells
• Antibiotics to control infections
• Growth factors: G-CSF, GM-CSF, and erythropoietin etc.

2. ANTIMETABOLITES
• Folic acid antagonist binds to DHFR leading to interference in synthesis of reduced form of it that accepts one-carbon unit
• Lack of this co-factor interrupts synthesis of thymidylate/purines/aminoacids, serine and methionine thus inhibiting
synthesis of DNA, RNA and proteins
Toxic_effects_: Seen on bone marrow, skin and g.i. mucosa. Leucovorin antagonizes MTX. Used to rescue normal cells in
accidental overdose.
USES : • Curative for Choriocarcinoma
Others : • Rheumatoid arthritis
• Intractable bronchial asthma
• Psoriasis
• With PG-F2a as abortifacient

B. PURINE ANALOG - 6-MP/6-TG


• Metabolised to active form by HGPRT that inhibits purine nucleotide interconversion and inhibition of DNA and RNA
synthesis
• 6-MP is converted to inactive compounds by Xanthine oxidase
• Allopurinol (Xanthine oxidase inhibitor) increases toxicity of 6-MP.This does not occur with 6-TG (it is first Inactivated
by deamination before x-o activity to occur)
Cladribine : DOC for Hairy cell leukemia.

C. PYRIMIDINE ANALOGS (5-FU. capecitabine, cytarabine)


5-FU : Transformed into 5-FdUMP(5-Fluoro deoxyuridine monophosphate), which form a covalent bond with thymidylate
synthase, resulting in inhibition of DNA synthesis.
Another metabolite of 5-FU is FUTP that gets incorporated into RNA and inhibits its function
USES : Metastatic breast cancer

423
6 Self Assessment & Review Phar macology
Pharmacology
CYTOSINE ARABINOSIDE :
Converted to active metabolite monophosphate and triphosphate, that .competitively inhibits DNA and RNA synthesis (S-
PHASE, SPECIFIC INHIBITOR) given slow infusion.
USES : AML
ADR : Nausea/myelosuppression/stomatitis/alopecia

Gemcjtabine : Converted to di- and tri-phosphates (active) by nucleoside kinases. They inhibits DNA and RNA synthesis
USES : Non-small cell lung cancer and pancreatic cancer

3. PLANT ALKALOIDS :
Vincristin/vinblastin/vindesin/vinorelbin/vinzolidin
MOA : Polymerization of microtubules, which are the cytoskeleton of the mitotic spindle. The drugs bind to the microtubular
protein tubulin. This results in MITOTIC ARREST IN METAPHASE, DISSLUTION OF MITOTIC SPINDLE and interference
with chromosome segregation.
USES : Systemic Hodgkin’s disease and other lymphomas (Vinblastin)
Toxicity : Marrow suppression/Alopecia/Nausea
(Desacetyl vinblastine is vindesine)
VINCRISTINE : Uses acute leukemias in children, ADR neurotoxicity
VINORELBINE :Uses: Advanced non-small cell lung cancer /colorectal
ADR : Granulocytopenia

POPOPHYLLQTOXINS:
Etoposide/Teniposide
Semisynthetic derivatives of Podophyllotoxin
MOA: Block late S and G2 phase of cell cycle by blocking Topoisomerase-II through strand breakage that damage DNA
USES: Etoposide - Monocytic leukemia, testicular cancer, oat cell cancer.
Teniposide : various lymphomas
ADRs: Hematopoietic and Lymphopoietic toxicity

CAMPTOTHECINS :
Topotecan/Irinotecan
Are natural products: Inhibit topoisomerase I
USES: Metastatic ovarian cancer, including cisplatin resistant neoplasm.
ADR: Profound bone marrow depression, neutropenia, thrombocytopenia and anemia are dose limiting ADRS.
• Irinotecan is a prodrug : converted to a topoisomerase I inhibitor
• It is indicated in colon and rectal cancer resistant to 5-FU ADR: Myelosuppression (most serious, dose limiting)
Diarrhea-severe

TAXANES: Paclitaxel/Docetaxel
MOA: Spindle poison during mitosis through depolymerization.
USES: Advanced ovarian and breast cancer
ADR: Neutropenia/Thrombocytopenia peripheral neuropathy
Bone marrow suppression (Docetaxel)

ANTIBIOTICS
A. Anthracyclines (Doxorubicin/Daunorubicin)
MOA :
• High affinity binding to DNA with subsequent block of DNA/RNA synthesis and DNA strand breaking by action on
Topoisomerase II

424
Antimicr obial & Antineoplastic Dr
Antimicrobial ugs
Drugs 6
• Binding to biological membranes that affect ion-transport
• Generation of semiquinone free radicals and oxygen free radicals (oxygen radicals cause cardiotoxicity)
USES :
• Many cancers of breast, endometrium, ovary, testes, thyroid, lung, sarcomas like, neuroblastoma Ewing’s,
osteosarcoma, rhabdomyosarcoma. Hematological - Hodgkin’s, Non-Hodgkin’s and multiple myeloma
• Synergy occurs when combined with cyclophosphamide, cisplatin and nitrosoureas.
• Aunorubicin and newer idarubicin are now dedicated for acute leukemias.
ADRS :
• Bone marrow depression, but is of short duration and rapid recovery occurs
• Cardiac toxicity (Doxo/dauno)- Conduction defects - Arrhythmias
• Severe alopecia (totalis to universalis)
• Radiation recall - inflammation at previous radiation sites.

B. ACTINOMYCIN-D
• Binds to DNA and inhibits DNA dependent RNA synthesis
• It causes thrombophlebitis (Flush the vein with NS)
USES :
• With VCR for Wilm’s tumor
• With MTX for gestational choriocarcinoma (curative)
ADR:
• Bone marrow toxicity
• Immunosuppression also occurs. LIVE virus vaccines should NOT be administered during therapy
• Radiation Recall: Inflammation occurs at sites of previous radiation

C. PLICAMYCIN:
DNA dependent RNA synthesis anticancer effect
Decreases Plasma calcium levels by acting on osteoclasts. (Useful in hypercalcemia)
USES: - Testicular cancer resistant to other drugs.

D. MITOMYCIN
It is a prodrug, converted to an alkylating agent that cross links DNA
- Hypoxic tumor cells are more sensitive

USES: - With - Bleomycin and VCR to treat Sq. cell cancer of Cervix With Doxo and 5-FU for adenocarcinomas of stomach/
pancreas/lungs
Instillation in bladder for small papillomas
ADR; - Myelosuppression

E. BLEOMYCIN:
MOA: Binds to DNA and inhibits its replication

USES; With vinblastin and cisplatin for the treatment of testicular cancer (curative)
• Also for sq. cell cancers of Head and Neck, Cervix, skin, penis, and rectum.
• Intracavitatory treatment for malignant effusions on cancer ovary/breast.

425
6 Self Assessment & Review Phar macology
Pharmacology

ADR: -Lethal anaphylactoid reactions


• Anorexia and blistering keratosis of palm and sole
• Pulmonary fibrosis

L- Asparaginase:
Inhibits conversion of aspargine to aspartic, acid-causing inhibition of protein
Synthesis. Normal cells can synthesize aspargine and hence are less affected

Hydroxyurea: Inhibits DNA synthesis


USES: Melanoma/CML
ADR
Marrow suppression
Megaloblastosis unresponsive to vitamin B12 and folic acid

MITOXANTRONE:
Binds DNA, cause stand breakage and inhibit DNA and RNA synthesis

USES: Refractory acute leukemia

ADR: All usual + cardiotoxicity in form of arrhythmias

MITOTANE:
- ADRENOLYTIC DRUG, DDT congener

USES: Adrenal Cancer

426
Antimicr obial & Antineoplastic Dr
Antimicrobial ugs
Drugs 6
ALL INDIA 9. Treatment of resistant falciparum malaria in child
is : [AI 08]
a. Doxycycline
1. Which of the following is least Nephrotoxic ?
b. Artemether
a. Streptomycin [AI 08]
c. Chloroquine
b. Gentamicin
d. Clindamycin
c. Polymixin B
10. Which of the following is not an Anti-fungal ?
d. Doxycline
a. Clofazamine [AI 08]
2. Protein synthesis inhibiting drug is : [AI 08]
b. Ciclopirox
a. Cefotetan
c. Fluconazole
b. Ampicillin / Amoxicillin
d. Undecyclenic acid
c. Penicillin
11. Which of the following is the mode of action of
d. Doxycycline
paclitaxel ? [AI 08]
3. Penicillin given by oral route is : [AI 08]
a. Inhibit folic acid
a. Penicillin V (Phenoxymethyl penicillin)
b. Polymerization of tubules / assembly of tubular
b. Penicillin G (Benzyl penicillin) dimers
c. Procaine Penicillin - G
c. Topoisomerase two inhibitor
d. Benzathine Penicillin - G
d. Protein synthesis
4. Antipseudomonal penicillin is : [AI 08] 12. Which is not a common side effect of Tacrolimus?
a. Piperacillin
a. Hirsuitism [AI 08]
b. Amoxycillin
b. Neurotoxicity
c. Ampicillin c. Nephrotoxicity
d. None of the above
d. DM
5. Which of the following is least true of Methicillin
13. Peptide which is used as anti tumour agent is :
resistant SA ? [AI 08]
a. Bleomycin [AI 08]
a. Due to alterations in PBP
b. Doxorubicin / Dacarbazine
b. Plasmid mediated
c. Valinomycin
c. Dependant on beta lactamase
d. Aspartame
d. More at 37 degree celcius
14. Leucovorin rescue is given with : [AI 08]
6. Which of the following is not true regarding
a. Methotrexate therapy
tetracycline ? [AI 08]
b. 6 MP
a. Not teratogenic
c. Thiotepa
b. Tooth discoloration
d. Cytosine Arabinoside
c. Superinfection
15. Which one of the following anticancer drugs
d. Pseudomembranous colitis
causes hypercoagulability syndrome : [AI 07]
7. All of the following durgs are commonly used for
a. Carbomustine
diphtheria chemoprophylaxis except : [AI 08]
b. 5 FU
a. Tetracycline
c. L-asparaginase
b. Erythromycin
d. Melphalan
c. Penicillin
16. The following drug is not useful for MRSA :
d. Rifampicin
a. Cefaclor [Al 07; AIIMS Nov. 06]
8. Drug commonly used against Typhoid are all
except : [AI 08] b. Cotrimoxazole
a. Amikacin c. Ciprofloxacin
b. Ciprofloxacin d. Vancomycin
c. Ceftriaxone
d. Azithromycin

Answer 1. d. Doxycline 2. d. Doxycycline 3. a. Penicillin ... 4. a. Piperacillin 5. d. More at ...


6. a. Not terato ... 7. a. Tetracycline 8. a. Amikacin 9. a. Doxycycline 10. a. Clofazamine
11. b. Polymerization ... 12. a. Hirsuitism 13. a. Bleomycin 14. a. Methotrexate ... 15. c. L-asparaginase
16. a. Cefaclor

427
6 Self Assessment & Review Phar macology
Pharmacology
17. Filgrastim is used in treatment of : [Al 07] a. Cephalexin [AI 06, 04; 98;
a. Anemia b. Cloxacillin AIIMS 04, May’ 03, 96]
b. Neutropenia c. Piperacillin
c. Malaria d. Dicloxacillin
d. Filarial 26. All of the following antibacterial agents acts by
18. The following drug has maximum propensity to inhibiting cell wall synthesis, except : [AI 06; 95]
cause peripheral neuropathy : [AI 07] a. Carbapenems
a. Didanosine b. Monobactams
b. Zidovudine c. Cephamycins
c. Stavudine d. Nitrofurantoin
d. Lamivudine 27. One of the following regarding mycophenolate
19. Which of the following is not an alkylating agent? mofetil is incorrect : [AI 06]
a. Cyclophosphamide [Al 07] a. It is a prodrug
b. 5-FU b. It is a selective uncompetitive and reversible in-
c. Busulfan hibitor of inosine monophosphate dehydroge-
d. Chlorambucil nase
20. The following drug acts by hypomethylation : c. It also inhibits calcineurin
a. Gemcitabine [Al 07] d. Selectively inhibits lymphocyte proliferation
b. Homoharringtonine 28. All of the following are therapeutic uses of Peni-
cillin G, except : [AI 06]
c. Decitabine
a. Bacterial meningitis
d. Cytosine arabinoside
b. Rickettsial infection
21. Methotrexate used in high doses in : [Al 07]
c. Syphilis
a. Osteosarcoma
d. Anthrax
b. Retinoblastoma
29. All of the following are topically used sulphona-
c. Rhabdomyosarcoma
mides except : [AI 05]
d. Ewing’s sarcoma
a. Sulphacetamide
22. Nevirapine belongs to the following group :
b. Sulphadiazine
a. NNRTI [Al 07; AI 06; AIIMS May; 05]
c. Silver sulphadiazine
b. Protease inhibitor
d. Mafenide
c. Fusion inhibitor
30. Which one of the following is best associated with
d. Nucleoside reverse transcriptase inhibitor Lumefantrine ? [AI 05]
23. All are true about immunosuppressant except ? a. Antimycobacterial
a. Tacrolimus inhibits calcineurin pathway b. Antifungal
b. Steroids binds to cytosolic receptors and heat c. Antimalarial
shock proteins [Al 07]
d. Antiamoebic
c. Mycophenolate inhibit purine synthesis via GMP
31. Which one of the following drugs is Topo-
dehydrogenase
isomerase I inhibitor ? [AI 05]
d. Sirolimus will block kinase in the IL 2 receptor
a. Doxorubicin
pathway
b. Irinotecan
24. All of the following drugs are protease inhibitors
except : [AI 06] c. Etoposide
a. Nelfinavir d. Vincristine
b. Saquinavir 32. Nevirapine is a : [AI 05]
c. Abacavir a. Protease inhibitor
d. Ritonavir b. Nucleoside reverse transcriptase inhibitor
25. Which one of the following drugs is an c. Non-nucleoside reverse transcriptase inhibitor
antipseudomonal penicillin? d. Fusion inhibitor

Answer 17. b. Neutropenia 18. c. Stavudine 19. b. 5-FU 20. c. Decitabine 21. a. Osteosarcoma
22. a. NNRTI 23. c. Mycopheno ... 24. c. Abacavir 25. c. Piperacillin 26. d. Nitrofurantoin
27. c. It also ... 28. b. Rickettsial ... 29. b. Sulphadiazine 30. c. Antimalarial 31. b. Irinotecan
32. c. Non-nucleos ..

428
Antimicr obial & Antineoplastic Dr
Antimicrobial ugs
Drugs 6
33. One of the following is not penicillinase suscep- d. Ethambutol
tible : [AI 05] 39. Which is most active chemotherapeutic agent in
a. Amoxicillin the treatment of Leiomyosarcoma ? [AI 04]
b. Penicillin G a. Adriamycin
c. Piperacillin b. Doxorubicin
d. Cloxacillin c. Methotrexate
34. All the following anticancer agents cause bone d. Cisplatin
marrow depression except : [AI 04] 40. Patients suffering from multidrug resistant tuber-
a. Chlorambucil culosis can be treated with all the following drugs
b. Daunorubicin except : [AI 04]
c. Doxorubicin a. Tobramycin
d. Flutamide b. Amikacin
35. A post operative patient developed septicemia and c. Ciprofloxacin
was empirically started on combination chemo- d. Clarithromycin
therapy by a new resident doctor. However, when 41. All of the following are hormonal agents used
the patient did not respond even after 10 days of against cancer except : [AI 04, 03]
antibiotics treatment, the review of the charts was a. Letrazole
done. It was found that the resident doctor had b. Exemestane
started the combination of antibiotics which was
c. Taxol
mutually antagonistic in action. Which one of the
following is the most likely combination that was d. Tamoxifen
given ? [AI 04] 42. All of the following factors increase the risk of
a. Vancomycin and Amikacin amino glycoside renal toxicity except : [AI 03]
b. Cephalexin and Gentamicin a. Elderly person
c. Ampicillin and Chloramphenicol b. Hypokalemia
d. Ciprofloxacin and Piperacillin c. Simultaneous use with penicillin
36. Which of the following statements is not true re- d. Aminoglycoside administration in recent past
garding sulfonamides : [AI 04] 43. All of the following drugs act on cell membrane,
a. Sulfasalazine is absorbed well from GIT except : [AI 03]
b. Crystalluria can occur with sulfonamide admin- a. Nystatin
istration b. Griseofulvin
c. Sulfonamide administration to newborn may c. Amphotericin B
cause kernicterus d. Polymyxin B
d. Sulfonamides are of value in treatment of infec- 44. Bacitracin acts on : [AI 03]
tions due to nocardia species a. Cell Wall
37. Which of the following fluoroquinolones does not b. Cell Membrane
require dose adjustment in a patient with creati- c. Nucleic acid
nine clearance of <50mg/min ? [AI 04]
d. Ribosome
a. Ciprofloxacin
45. The most effective drug against M. leprae is:
b. Trovafloxacin
a. Dapsone [AI 03; AIIMS May 02]
c. Lomefloxacin
b. Rifampicin
d. Sparfloxacin
c. Clofazimine
38. A 30 year old pregnant woman develops tubercu-
losis. Which of the following antitubercular drugs d. Ethionamide
should not be used : [AI 04; PGI May 05] 46. Gemcitabine is effective in : [AI 02]
a. INH a. Head and neck cancers
b. Rifampicin b. Pancreatic cancer
c. Streptomycin c. Small - cell lung cancer
d. Soft tissue sarcoma

Answer . 33. d. Cloxacillin 34. d. Flutamide 35. c. Ampicillin ... 36. a. Sulfasalazine ...37. b. Trovafloxacin
38. c. Streptomycin 39. a and b 40. a. Tobramycin 41. c. Taxol 42. c. Simultaneous ...
43. b. Griseofulvin 44. a. Cell Wall 45. b. Rifampicin 46. b. Pancreatic ...

429
6 Self Assessment & Review Phar macology
Pharmacology
47. All the following drugs cause renal failure except: 55. A patient has hepatic encephalopathy. The drug
a. Cephaloridine [AI 02] used for gut sterilization in this patient is :
b. Amphotericin B a. Neomycin [AI 00]
c. Cefoperazone b. Netilmycin
d. Gentamicin c. Bleomycin
48. All of the following statements about Metho-trexate d. None of the above
are true except : [AI 01] 56. All of the following are examples of bactericidal
a. Folinic acid enhances the action of methotrex- drugs except : [AI 99]
ate a. INH
b. Methotrexate inhibit dihydrofolate reductase b. Rifampicin
c. Non proliferative cells are resistant to methotr- c. Ethambutol
exate d. Pyrazinamide
d. Methotrexate is used in treatment of psoriasis 57. All of the following are drugs used for ATT except:
49. Mesna is given with cyclophosphamide to : a. Kanamycin [AI 99]
a. Increase absorption [AI 01; AIIMS May 03, b. Cycloserine
b. Decrease excretion Nov. 03] c. 5-Flucytosine
c. Ameliorate haemorrhagic cystitis d. Ofloxacin
d. Decrease metabolism 58. Mechanism of action of erythromycin is interfer-
50. Mechanism of action of tetracycline is : ence with : [AI 99]
a. Binds to A site and inhibit attachment of t-RNA a. Transcription
b. Inhibits peptidyl transferase [AI 01; b. Translation
c. Causes misreading of mRNA PGI Dec. 05] c. Translocation
d. Causes termination of peptide chain elongation. d. Single transduction
51. A 35 year old patient has Ca lung, with a past his- 59. Which of the following drugs acts on “motilin” re-
tory of lung disease. Which drug should not be ceptors : [AI 99]
given : [AI 00] a. Erythromycin
a. Bleomycin b. Tetracycline
b. Vinblastine c. Norfloxacin
c. Mithramycin d. Chloramphenicol
d. Adriamycin 60. Cyclosporine acts by inhibiting the proliferation
52. Which anti HIV drug dose not cause peripheral of: [AI 99]
neuropathy : [AI 00] a. IL 1
a. Lamivudine b. IL 2
b. Stavudine c. IL 3
c. Didanosine d. Macrophages
d. Zalcitabine 61. Difference between action of DEC and lvermectin
53. Drug of choice for malaria during pregnancy is : in a case of scrotal filariasis is : [AI 99]
a. Chloroquine [AI 00] a. DEC acts more effectively on microfilariae than
b. Quinine lvermectin
c. Primaquine b. DEC acts only on microfilariae and lvermectin
d. Mepacrine acts only on adults
54. The treatment of contacts of meningococcal men- c. DEC acts on both microfilariae and adults while
ingitis is by : [AI 00] lvermectin acts on adults only
a. Rifampicin d. DEC acts on adults and lvermectin on microfi-
lariae
b. Erythromycin
c. Penicillin
d. Cephalosporins
Answer 47.c. Cefoperazone 48. a. Folinic ... 49. c. Ameliorate ... 50. a. Binds to ... 51. a. Bleomycin
52. a. Lamivudine 53. a. Chloroquine 54. a. Rifampicin 55. a. Neomycin 56. c. Ethambutol
57. c. 5-Flucytosine 58. c. Translocation 59. a. Erythromycin 60. b. IL 2 61. d. DEC acts ...

430
Antimicr obial & Antineoplastic Dr
Antimicrobial ugs
Drugs 6
62. Side-effects of Cisplatin include all of the follow- 70. Antimicrobial drug effective in pre-erythrocytic
ing except : [AI 99] phase in liver : [AI 95]
a. Nausea and vomiting a. Proguanil
b. Nephrotoxicity b. Chloroquine
c. Blindness c. Tetracycline
d. Ototoxicity d. Quinine
63. SLE like syndrome is most commonly associated
with administration of : [AI 99; PGI May 05] AIIMS
a. Rifampicin
b. Procainamide 71. Cross-resistance of isoniazid is not seen with :
c. Digitalis a. Rifampicin [AIIMS May 08]
d. Phenytoin b. Ethionamide
64. Albendazole may be used for treatment of all of c. Cycloserine
the following conditions except : [AI 98] d. Ethambutol
a. Enterobius 72. Which of the following antibiotics acts by inhibit-
b. Ascariasis ing cell wall synthesis ? [AIIMS May 08]
c. Ankylostoma a. Cefepime
d. Schitosomiasis b. Aminogycosides
65. Oral contraceptive failure may be seen with : c. Erythromycin
a. Rifampicin [AI 97] d. Doxycycline
b. Cimetidine 73. Which drug would treat both dermatophysis and
c. Propranolol candidal infection ? [AIIMS May 08]
d. Ethambutol a. Ketoconazole
66. Most common side effect of 5-fluorouracil is: b. Griesofulfin
a. G.I. toxicity [AI 97] c. Nystatin
b. Bone marrow depression d. Tolnafet
c. Cardiotoxicity 74. SIADH is caused by all except : [AIIMS May 08]
d. Neurotoxicity a. Vincristine
67. Which of following is not hepatotoxic drugs : b. Vinblastine
a. Ethambutol [AI 96] c. Actinomycin D
b. Rifampicin d. Cylophophamide
c. INH 75. Imatinib is used in treatment of : [AIIMS May 08]
d. Cycloserine a. Chronic myelomonocytic leaukemia
68. False about fluoroquinolones : [AI 95] b. MDS
a. Low toxicity to host cell c. ALL
b. DNA gyrase inhibitor d. GIST
c. Effective against pseudomonas 76. Sustained neutropenia is seen with :
d. Rapidly develop resistance a. Vinblastin [AIIMS May 08]
69. Long term are of chloroquine does not lead to : b. Cisplatin
a. Lichenoid eruptions [AI 95] c. Carmustin
b. Visual deterioration d. Carmustin
c. T wave changes in ECG 77. NOT used in scabies ? [AIIMS May 08]
d. Weight gain a. BHC
b. Permethrin
c. Ciclopirox oleamine
d. Crotamiton

Answer 62. c. Blindness 63. b. Procaina ... 64. d. Schitos ... 65. a. Rifampicin 66. a. G.I. toxicity
67. d. Cycloserine 68. d. Rapidly ... 69. d. Weight gain 70. a. Proguanil 71. b. Ethionamide
72. a. Cefepime 73. a. Ketoconazole 74. c. Actinomycin D 75. d. GIST 76. c. Carmustin
77. c. Ciclopirox ...

431
6 Self Assessment & Review Phar macology
Pharmacology
78. True about aminoglycoside is all except : b. Amikacin
a. Are bacteriostatic [AIIMS May 08] c. Rifampicin
b. Distributed only extracellularly d. Vanomycin
c. Excreted unchanged in urine 87. Which anti -TB drug can cause transient memory
79. Drug not used in H. pylori : [AIIMS May 08] loss ? [AlIMS Nov. 06]
a. Metronidazole a. Ethionamide
b. Omeprezole b. INH
c. Mosapride c. Ethambutol
d. Amoxicillin d. Pyrazinamide
80. Which drug can be used against keratomycosis 88. Which drug acts by inhibiting cell wall ?
of eye ? [AIIMS May 08] a. Cefepime [AlIMS Nov. 06]
a. Clofazamine b. Tetracycline
b. Oxytetracycline c. Erythromycin
c. Amoxicillin d. Ciprofloxain
d. Silver sulfadiazine (SSD) 89. Which of the following is an aromatase inhibitor?
81. True about protease inhibitors are all except : a. Tamoxifen [AlIMS May 06]
a. Acts as a substrate for P-glycoprotein (P-gp) and b. Letrozole
action is mediated by MDR-1 gene c. Danazol
b. Hepatic oxidative [AIIMS Nov. 07] d. Taxane
c. All protease inhibitors interfere with metabolism 90. Which of the following drugs is associated with
by drug interactions untoward side effect of renal tubular damage ?
d. Saquinavir causes maximum inhibition of a. Cisplatin [AlIMS May 06]
CYP3A4 b. Steptozotocin
82. Imatinib mode of action : [AIIMS May 07, 06] c. Methysergide
a. Competetive inhibitor of bcr abl gene product d. Cyclophosphamide
b. P glycoprotein inhibitor 91. Which of the following chemotherapeutic agents
c. P glycoprotein stimulator is associated with secondary leukemia ?
d. Competitively antagonizes the ATP binding site a. Vinblastine [AlIMS May 06]
83. Which of the following is an antimetabolite : b. Etoposide
a. Methotrexate [AIIMS May 07] c. Cisplatin
b. Vinblastine d. Bleomycin
c. Cisplatin 92. Tetracycline inhibits protein synthesis by :
d. Etoposide a. Inhibiting initiation and causing misreading of
84. FK 506 is : [AIIMS May 07] mRNA [AlIMS Nov. 05]
a. Macrolide antibiotic b. Binding to 30 S subunit and inhibits binding of
b. Imunoglobulin antibody aminoacyl tRNA
c. Opiod antagonists c. Inhibiting peptidyl transferase activity
d. Non depolarizing blocker d. Inhibiting translocation
85. Aplasia of the bone marrow not seen with : 93. The new agent pemetrexed, useful in breast can-
a. Methicillin [AIIMS May 07] cer, belongs to which of the following category
drugs : [AlIMS Nov. 05]
b. Chloramphenicol
a. Antitumor antibiotic
c. Methyl hydantoin
b. Alkylating agent
d. Chlorpromazine
c. Hormonal agent
86. If the creatinine clearance is 10ml/hr, duration of
action of which of the following drugs will be pro- d. Antimetabolite
longed ? [AlIMS Nov. 06]
a. Amphotericin -B
Answer 78. a. Are ... 79. c. Mosapride 80. d. Silver ... 81. d. Saquinavir ... 82. a. Competetive ...
83. a. Methotrexate 84. a. Macrolide ... 85. a. Methicillin 86. b. Amikacin 87. b. INH
88. a. Cefepime 89. b. Letrozole 90. a. Cisplatin 91. c. Cisplatin 92. b. Binding ...
93. d. Antimeta ...

432
Antimicr obial & Antineoplastic Dr
Antimicrobial ugs
Drugs 6
94. All of the following drugs may cause hyperkale- 101. Infliximab is directed against : [AlIMS May 05]
mia except : [AlIMS Nov. 05; May’ 04] a. Tumor necrosis factor -α (TNF - α)
a. Cyclosporine b. Interleukin - 1(Il-1)
b. Amphotericin c. Interleukin -12(IL-12)
c. Heparin d. Intercellular adhesion molecule(ICAM)
d. NSAIDs 102. Chemotherapeutic drugs can cause :
95. Which of the following chemotherapeutic drugs a. Only necrosis [AlIMS May 05]
has selective action on hypoxic tumor cells : b. Only apoptosis
a. Mitomycin C [AlIMS Nov. 05] c. Both Necrosis and apoptosis
b. Cisplatin d. Anoikis
c. Doxorubicin 103. All of the following drugs are contraindicated in
d. 5 flurouracil patients with G-6-PD deficiency, except :
96. The blood culture from a patient of febrile neutro- a. Co-trimoxazole [AlIMS May 04]
penia has grown Pseudomonas aeruginosa, It was b. Furazolidone
found to be a producer of extended spectrum beta
c. Nalidixic acid
lactamase enzyme. The best choice of antimicro-
bial therapy should be : d. Ceftriaxone
a. Ceftazidine + Amikacin [AlIMS Nov. 05] 104. Which of the following antimalarial is a slow act-
ing schizonticide : [AlIMS May 04]
b. Aztreonam + Amikacin
a. Artemether
c. Cefpirome + Amikacin
b. Mefloquine
d. Imipenem + Amikacin
c. Pyrimethamine
97. The chemotherapeutic agent, most commonly ad-
ministered by continuous infusion is : d. Quinine
a. Ara - c [AlIMS Nov. 05] 105. Thalidomide can be used in : [AlIMS May 04]
b. F - FU a. Myocardial infraction
c. Cisplatin b. Erythema nodosum leprosum
d. Etoposide c. Wernicke’s encephalopathy
98. Nephrotoxicity is a side effect of one of the follow- d. Epilepsy
ing immuno-suppressives : [AlIMS Nov. 05] 106. Which of the following is a fourth generation
a. Sirolimus Cephalosporin : [AlIMS May 04]
b. Tacrolimus a. Ceftriaxone
c. Mycophenolate mofetil b. Cefaclor
d. Azathioprine c. Cefepime
99. The most common side-effect of chemotherapy d. Cefuroxime
adminsitration is : [AlIMS May 05] 107. Which of the following drug causes Pseudo tu-
a. Nausea mour cerebri : [AlIMS May 04]
b. Alopecia a. Sparfloxacin
c. Myelosuppression b. Tetracylcline
d. Renal distinction. c. Gentamicin
100. Resistance to zidovudine develops due to : d. Clofazimine
a. Mutations at reverse transcriptase[AlIMS May 05] 108. Which of the following is caused by Amphotericin
B: [AlIMS May 04]
b. Increased efflux of the drug from inside the cell
a. Hypokalemia
c. Increased metabolism of the drug
b. Hyperkalemia
d. Decreased zidovudine 5 triphosphate formation
c. Hypermagnesemia
d. Hyponatremia

Answer 94. b. Amphotericin 95. a. Mitomycin c 96. d. Imipenem ... 97. a. Ara - c 98. b. Tacrolimus
99. None 100. a. Mutations ... 101. a. Tumor ... 102. c. Both ... 103. d. Ceftriaxone
104. c. Pyrimethamine 105. c. Wernicke’... 106. c. Cefepime 107. b. Tetracylcline 108. a. Hypokalemia

433
6 Self Assessment & Review Phar macology
Pharmacology
109. A patient with cancer received extreme degree of a nucloeotide derivative following the organ trans-
radiation toxicity. Further history revealed that the plant. The nucleotide derivative of therapeutic im-
dose adjustment of a particular drug was missed portance in this organ transplant is :
during the course of radiotherapy. Which of the a. Azathioprine [AlIMS Nov. 03]
following drugs required a dose adjustment in that b. 5-Fluorouracil
patient during radiotherapy in order to prevent
c. Cytarabine
radiation toxicity : [AlIMS May 04]
d. Allopurinol
a. Vincristine
116. Sterile haemorrhagic cystitis is caused by :
b. Dactinomycin
a. Busulfan [AlIMS Nov. 03]
c. Cyclophosphamide
b. Ketoprofen
d. 6- Mercaptopurine
c. Methicillin
110. Following drugs are immunosuppressive drug
except : [AlIMS May 04] d. Cylclophosphamide
a. Cyclosporine 117. A 58 year old woman has been diagnosed with
locally advanced breast cancer and has been rec-
b. Cefaclor
ommended for chemotherapy. She has five years
c. Azathioprine history of myocardial infarction and congestive
d. Steroids heart failure. Which antinoeplastic drug should be
111. Nevirapine is : [AlIMS May 04] best avoided : [AlIMS Nov. 03]
a. Protease inhibitor a. Anthracycline
b. Nucleoside reverse transcriptase inhibitor b. Alkylating agent
c. Non-nucleoside reverse transcriptase inhibitor c. Platinum compound
d. Fusion inhibitor d. Bisphosphonates
112. In a patient with Listeria meningitis who is allergic 118. Excessive intake [hypervitaminosis] of which of
to penicillin, the antimicrobial of choice is : the following vitamin is associated with increased
a. Vancomycin [AlIMS Nov. 04] risk of congenital malformations :
b. Gentamicin a. Vitamin-A [AlIMS Nov. 03]
c. Trimethoprim -sulphamethoxazole b. Biotin
d. Ceftriaxone c. Folic acid
113. Which one of the following statements is false d. Vitamin-K
regarding vincristine : [AlIMS Nov. 04] 119. Peripheral neuropathy may occur with the use of
a. It is an alkaloid all the following except : [AlIMS Nov. 03]
b. Its use is associated with neurotoxicity a. Vincristine
c. It does not cause alopecia b. Cisplatin
d. It is a useful drug for induction of remission in c. L- Asparaginase
acute lymphoblastic leukaemia d. Procarbazine
114. All of the following are true about therapy for tu- 120. The antimicrobial agent which inhibits the ergos-
berculosis, except : [AlIMS Nov. 04] terol biosynthesis is : [AlIMS Nov. 03]
a. ‘Flu like syndrome’ is usually seen in people a. Ciprofloxacin
taking rifampicin on daily basis. b. Amphoterecin B
b. Ethambutol accumulates in renal failure. c. 5-Flurocytosine
c. Hyperuricemia is a recognized side effect of d. Griseofulvin
pyrazinamide 121. Pyronaridine is : [AlIMS Nov. 03]
d. Red-green color impairment is an early sign of a. Antimalarial
ethambutol induced optic neuritis
b. Anti-HIV
115. A 50 year old male suffering from renal failure,
c. Antifungal
underwent kidney transplant. He was prescribed
d. Antibacterial

Answer 109. b. Dactinomycin 110. b. Cefaclor 111. c. Non-nucleo ... 112. c. Trimethoprim ...113. c. It does ...
114. a. ‘Flu like ... 115. a. Azathioprine 116. d. Cylclophos ... 117. a. Anthracycline 118. None
119. c. L- Asparaginase120. b. Amphotereci.. 121. a. Antimalarial

434
Antimicr obial & Antineoplastic Dr
Antimicrobial ugs
Drugs 6
122. An HIV positive patient is on anti retroviral therapy 129. The drug which does not cause myopathy is :
with zidovudine, lamivudine and indinavir. He is a. Chloroquine [AIIMS May 02]
proven to be suffereing from genito-urinary tuber- b. Betamethasone
culosis. Which one of the following drugs should
c. Chloramphenicol
not be given to this patient :
d. Zidovudine
a. Isoniazid [AlIMS May 03; Nov. 01]
130. Following drugs cause pancreatitis except :
b. Rifampicin
a. Captopril [AIIMS May 02]
c. Pyrazinamide
b. Furosemide
d. Ethambutol
c. Valproic acid
123. A middle aged old man, with chornic renal failure
is diagnosed to have sputum-positive pulmonary d. Didanosine
tuberculosis. His creatinine clearance is 25ml/min. 131. Which of the following will be optium treatment in
All of the following durgs need modification in a patient of chronic hepatitis B whose serum AST
doses except : [AlIMS May 03, Nov. 01, levels are raised : [AIIMS Nov. 01]
a. Isoniazide May 95] a. Lamivudine
b. Streptomycin b. Lamivudine + Interferous
c. Rifampicin c. Immunoglobulins
d. Ethambutol d. Interferons
124. Which of the following is the treatment of choice 132. Which of the following is least likely to cause in-
for cryptococcal meningitis : [AIIMS Nov. 02] terstitial nephritis on chronic use :
a. Fluconazole a. Methicillin [AIIMS Nov. 00]
b. Latraconazole b. Cephalothin
c. Fluocytosine c. Heparin
d. Amphotericine B d. Ampicillin
125. Pulmonary fibrosis is the most commmon com- 133. Which fo the following inhibits transcription:
plication after treatment with : [AIIMS Nov. 02] a. Cyclosporin [AIIMS June 00]
a. 6-mercaptopurine b. Rapamycin
b. Vincristine c. 5 - Flurouracil
c. Bleomycin d. Cyclophosphamide
d. Adriamycin 134. Regarding adverse reaction, all are correct, ex-
126. Drug induced myopathy can be caused by all of cept : [AIIMS Nov. 99]
the following except : [AIIMS Nov. 02] a. Methysergide – Retroperitoneal fibrosis
a. Atrovastaine b. Rifampicin – Optic neuritis
b. D-penicillamine c. Saraslasin – AT2 receptor antagonist
c. Ciprofloxacin d. Ethambutol – Retreobulbar neuritis
d. Chloroquine 135. Peripheral neuropathy is seen due to prolonged
127. The drug of choice for thoracic actinomycosis is: use of all, except : [AIIMS Nov. 99]
a. Amphoterecin-B [AIIMS Nov. 02] a. Zalcitabine
b. Penicillin b. Didanosine
c. Cotrimoxazole c. Stamivudine
d. Itraconazole d. Lamivudine
128. The antibiotic that inhibits protein synthesis by 136. All are true regarding chloroquine, except :
premature chain termination and which structur- a. Acts only on exo erythrocytic cycle [AIIMS Nov.
ally resembles an aminoacyl t-RNA is : b. Acts on DNA and RNA of parasite 99; Dec. 95]
a. Tetracycline [AIIMS Nov. 02] c. Causes pigmentation of nail and mucosa
b. Chloramphenicol d. Infected RBC has more drug
c. Puromycin
d. Erythromycin
Answer 122. b. Rifampicin 123. c. Rifampicin 124. d. Amphotericine B125. c. Bleomycin 126. c. Ciprofloxacin
127. b. Penicillin 128. c. Puromycin 129. c. Chloram ... 130. a. Captopril 131. a. Lamivudine
132. c. Heparin 133. a. Cyclosporin 134. b. Rifampicin ... 135. d. Lamivudine 136. a. Acts only on ...

435
6 Self Assessment & Review Phar macology
Pharmacology
137. Azidothymidine [Zidovudine]. commonest side ef- c. Penicillin
fects is : [AIIMS June 98] d. Metronidazole
a. Hyperuricemia 146. Clindamycin acts by inhibiting : [AIIMS Sep. 96]
b. Pancreatitis a. Cell wall synthesis
c. Peripheral neuropahty b. Protein synthesis
d. Macrocytic anaemia c. Cell membrane synthesis
138. Pentamidine therapy causes all except : d. Glucose utillization
a. Upper lobe infiltrates [AIIMS June 98] 147. Cyclosporin acts on : [AIIMS Sep. 96]
b. Lung Cavitation a. CD8 Cells
c. Extrathoracic manifestation b. CD4 Cells
d. Pneumothorax c. B - Lymphocytes
139. Interferon is used in treatment of all except : d. T - Lympocytes
a. Chronic myeloid leukemia [AIIMS June 98] 148. Treatment of whooping cough is :
b. Polymyositis a. Rifampicin [AIIMS Dec. 95]
c. Hairy cell leukemia b. Tetracycline
d. Hepatitis C c. Erythromycin
140. All are features of ethambutol toxicity, except : d. Ampicillin
a. Retrobulbar neuritis [AIIMS June 97]
b. Colour vision defects PGI
c. Hyperuricemia
d. Hypercalcemia 149. Tamoxifen is : [PGI Dec. 07]
141. Fastest acting drug in leprosy is : [AIIMS Feb. 97] a. Used as adjuvant therapy in receptor positive
a. Rifampicin breast cancer
b. Dapsone b. For chemoprophylaxis against breast cancer
c. Clofazimine c. Used in renal cell carcinoma
d. Ethionamide d. Used in HCC
142. Drug causing icthyosis and hyperpigmentation, 150. Which of the following are Alkylating agents :
when used in leprosy is : [AIIMS Feb. 97]
a. Cyclophosphamide [PGI Dec. 07]
a. Rifampicin
b. Ifosfamide
b. Dapsone
c. Clofazimine c. Paclitaxel
d. Ethionamide d. 5 - FU
143. Which drug is NOT effective in pseudomonas in- e. Actinomycin - D
fection : [AIIMS Feb. 97] 151. Mucositis is caused by : [PGI Dec. 06]
a. Cefaclor a. 5-Fu
b. Ceftazidime b. Methotrexate
c. Cefotaxime c. Paclitaxel
d. Carbenicillin d. Cisplatin
144. Halofantrine is used for : [AIIMS Feb. 97] e. Etoposide
a. Falciparum malaria 152. Drugs causing predominantly sensory neuropa-
b. Visceral leishmaniasis thy : [PGI Dec. 06]
c. Leprosy a. Cisplatin
d. Amoebiasis b. Pyridoxin excess
145. Drug of choice of pneumocystis carinii : c. GB syndrome
a. Cotrimoxazole [AIIMS Feb. 97] d. Suramine
b. Errythromycin

Answer 137. d. Macrocytic ... 138. b. Lung Cavitation 139. b. Polymyositis 140. d. Hyperc ... 141. a. Rifampicin
142. c. Clofazimine 143. a. Cefaclor 144. a. Falciparum ... 145. a. Cotrimoxazole 146. b. Protein ...
147. b. CD Cells 148. c. Erythromycin 149. a and b 150. a and b 151. a, b, c and e
4
152. a and b

436
Antimicr obial & Antineoplastic Dr
Antimicrobial ugs
Drugs 6
153. Drugs not given in Renal failure : [PGI Dec. 06] b. Lithium
a. Amphotericin B c. Doxycycline
b. NSAIDs d. Cycloserine
c. Mikacin e. Clindamycin
d. Ethambutol 162. Antibiotic resistance by enzyme inactivation seen
154. Drugs associated with deafness true are : in : [PGI Dec. 04]
[PGI Dec. 06] a. Penicillin
a. Streptomycin mostly vestibulotoxic b. Fluoroquinolones
b. Salicylates cause reversible deafness c. Tetracycline
c. Frusemide cause irreversible deafness d. Aminoglycoside
d. Cisplatin cause reversible deafness 163. Treatment for penicillinase producing organism:
155. Dextran is a good plasma expanders, but it has a. Ampicillin [PGI Dec. 04]
disadvantage of : [PGI Dec. 06] b. Cloxacillin
a. Blood group matching c. Methicillin
b. Causes thrombocytopenia d. Tetracycline
c. ↓ Microcirculation formation e. Cephaloridine
d. Promote roleaux 164. Drug causing deafness : [PGI Dec. 04]
156. Complications of Zidovudine : [PGI June 06] a. Cisplatin
a. Nausea & Vomiting b. Deferroxamine
b. Anemia c. Mustine hydrochloride
c. Steatosis d. Adriamycin
157. Drugs used in MRSA : [PGI June 06] 165. Anticancer drugs having highest emetogenic po-
a. Quinupristine / dalfopristine tential : [PGI Dec. 04]
b. Linezolid a. Cisplatin
c. Teicoplanin b. Vincristine
d. Penicillin c. Methotrexate
e. Piperacilin (Tazobectum) d. Busulfan
158. Complications of cyclosporine are : 166. Drugs given in the continuation phase of category
a. Hypertension [PGI June 06] II : [PGI Dec. 04]
b. Pulmonary fibrosis a. Isoniazid
c. Hirsutism b. Streptomycin
159. True about alkylating agent is/are : c. Ethambutol
a. Dose limiting mucositis [PGI Dec. 05] d. Pyrazinamide
b. Dose limiting myelosuppression e. Rifampicin
c. Cause DNA damage 167. Drugs inhibiting IL-2 : [PGI June 04]
d. Secondary carcinoma are common a. Cycloserine
e. Acts selectively on S phase b. Cyclosporine
160. Drug synergism is seen in : [PGI May 05] c. OKT-3
a. Flucytosine and amphotericin B for cryptococcal d. Tacrolimus
b. Trimethoprim and sulfamethoxazole for UTI 168. Antimitotic drugs of plant source is / are :
c. Pencillin and aminoglycoside a. Vincristine [PGI June 04]
d. Chlortetracycline + penicillin b. Isotretinoin
161. Drug dose to be decreased if patient has creati- c. Bleomycine
nine clearance <10 : [PGI May 05] d. Methotrexate
a. Amikacin

Answer 153. All are ... 154. a and b 155. a and b 156. All are ... 157. a and b
158. a and c 159. c. Cause ... 160. a, b ad c 161. a, b and d 162. d. Amino ...
163. b and c 164. a, b and c 165. a. Cisplatin 166. a, c and e 167. b and d
168. a and b

437
6 Self Assessment & Review Phar macology
Pharmacology
169. Broad spectrum anthelmintics are : 176. Treatment of chorioretinitis in AIDS patient :
a. Niclosemide [PGI June 04] a. Valciclovir [PGI June 03]
b. Praziquantel b. Ganciclovir
c. Albendazole c. Ribavirin
d. Mebendazole d. Amantadine
e. Pyrantel pamoate e. Cidofovir
170. ATT which is bactericidal : [PGI June 04] 177. Anticancer antibiotic : [PGI June 03]
a. INH a. Vancomycin
b. Pyrazinamide b. Actinomycin D
c. Ethambutol c. Bleomycin
d. PAS d. Mithramycin
e. Rifampicin 178. Best drug for anaerobic infections :
171. Methotrexate acts by : [PGI Dec. 03; Dec. 02] a. Metronidazole [PGI June 03]
a. Inhibition of dihydrofolate reductase b. Imipenem
b. Aldose dehydrogenase c. Aztreonam
c. Glutathione reductase d. Clotrimazole
d. Inhibition of sterol synthesis e. Vancomycin
172. Regarding Ritonavir in AIDS patient which of the 179. Warfarin induced skin necrosis is seen in :
following is /are true : [PGI Dec. 03] a. Protein C deficiency [PGI June 03]
a. Interacts with terfenadine b. Protein S deficiency
b. G. I. symptoms are seen c. Hemophillia
c. Contraindicated in renal failure d. Antithrombin III deficiency
d. It is NNRTI 180. Folic acid metabolism is inhibited by :
e. Should not be used in AIDS patient with bleed- a. Sulfonamides [PGI June 03]
ing disorder b. MTX
173. Ototoxicity of aminoglycoside is increased with c. N2O
concurrent use of which of the following drugs : d. Trimethoprim
a. Cisplatin [PGI Dec. 03] e. 5 - Flucytosine
b. Furosemide 181. Protease inhibitors are : [PGI Dec. 02]
c. Vancomycin a. Saquinavir
d. Vincristine. b. Nevirapine
e. Erythromycin c. Nelfinavir
174. Treatment of penicillinase producing Neisseria d. Abacavir
gonorrhoea is/are : [PGI Dec. 03]
e. Efavirenz
a. Amoxycillin
182. Metaphase arrest is caused by : [PGI Dec. 02]
b. Ciprofloxacin
a. Griseofulvin
c. Cefotaxime
b. Vincristine
d. Doxycycline
c. Paclitaxel
e. Azithromycin
d. Colchicine
175. Alkylating agents are : [PGI Dec. 03]
e. Etoposide
a. Vincristine
183. Treatment of choice for salmonella typhi :
b. Actinomycin - D
a. Cephalexin [PGI Dec. 01]
c. Chlorambucil
b. Gentamicin
d. 5 - FU.
c. CO-trimoxazole
e. Cyclophosphamide
d. Tetracycline
e. Ciprofloxacin
Answer 169. c and d 170. a, b and e 171. a. Inhibition ... 172. a and b 173. a, b,c and e
174. b and c 175. c and e 176. b. Ganciclovir 177. b, c and d 178. a and b
179. a. Protein ... 180. a, b and d 181. a and c 182. b, c and d 183. e. Ciprofloxacin

438
Antimicr obial & Antineoplastic Dr
Antimicrobial ugs
Drugs 6
184. Drugs which cause pericarditis is : d. Cardiomyopathy
a. Hydralazine [PGI Dec. 01] 192. Mechanism of action of Chloramphenicol :
b. Procainamide a. Binds to 50S subunit [PGI June 00]
c. Bretylium b. Prevents chain elongation
d. Methysergide c. Misreading of DNA
e. Amiodarone d. Binds to nuclear proteins
185. Which is true regarding cefepime : 193. Mechanism of Acyclovir resistance :
a. 4th generation cephalosporin [PGI Dec. 01] a. Thymidine kinase [PGI June 00]
b. Once a day dose is sufficient b. DNA - dependent RNA polymerase
c. Antipseudomonal action c. ↓ Spectrum
d. Dose should be reduced in liver d. ↓ Slide effects
e. It is a prodrug 194. Amoxycillin clavulanic acid combination :
186. Drugs effective agonist pseudomonas : a. ↑ Spectrum [PGI June 00]
a. Cefazoline [PGI Dec. 00] b. ↑ ½ life of amoxy
b. Ceftazidmie c. ↓ ½ life of both
c. Piperacillin d. ↑ Side effects
d. Cefuroxime 195. When allergic to penicillin, other drugs CI :
e. Cefotaxime a. Monobactams [PGI June 00]
187. Drugs which act by intercalating with DNA: b. Carbapenems
a. Actinomycin D [PGI Dec. 00] c. Cephalosporins
b. Tetracycline d. Semisynthetic penicillin
c. Bleomycin 196. Zidovudine given for HIV in pregnancy because :
d. Doxorubicin a. Decrease chance of vertical transmission
188. Drugs inhibiting formation of purines : b. Decrease severity of infection in mother
a. 5 - FU [PGI Dec. 00] c. Decrease severity of infection in new born
b. 6 - mercaptopurine d. Cause no benefit [PGI Dec. 99]
c. Hydroxyurea 197. Quinine given to a patient of falciparum malaria
d. Methotrexate caused sweating and palpitation, the likely cause
e. Cytosine arabinoside is : [PGI Dec. 99]
189. Drugs clinically interacting with ribosomes to in- a. Cinchonism
terfere with translation in bacteria : b. Hyperglycemia
a. Tetracycline [PGI Dec. 00] c. Hypoglycemia
b. Erythromycin d. Hypokalemia
c. Puromycin 198. Drugs contraindicated in infectious mononucleo-
d. Oligomycin sis is : [PGI Dec. 99]
190. Which antitubercular drug metabolism is under a. Ampicillin
genetic control : [PGI Dec. 00] b. Doxycycline
a. Rifampicin c. Atropine
b. INH d. Gentamicin
c. Cyclosporine 199. Interaction occurring when quinine and digoxin are
d. PZM given together : [PGI Dec. 99]
e. Kanamycin a. Quinine decreases excretion of digoxin
191. AZT has following side effects except : b. Quinine displaces digoxin from protein binding
sites
a. ↓ Platelet [PGI June 00]
c. Increases the metabolism of digoxin
b. Megaloblastic anemia
d. Digoxin causes cinchonism
c. Pancreatitis

Answer 184. a, b and d 185. a and c 186. b and c 187. a, c and d 188. b and e
189. b. Erythromycin 190. b. INH 191. c and d 192. a and b 193. a. Thymidine ...
194. a. ↑ Spectrum 195. c and d 196. a. Decrease ... 197. a and c 198. a. Ampicillin
199. a and b

439
6 Self Assessment & Review Phar macology
Pharmacology
200. Antifolate immunosuppressant is : [PGI Dec. 99] b. Reverse transcriptase inhibition
a. 5 FU c. Nucleic acid synthesis
b. Methotrexate d. Cell membrane
c. Cyclophohamide 206. Adenosine deaminase inhibitors is :
d. Cisplatin a. Pentostatin [PGI June 98]
201. Methotrexate is most useful in : [PGI June 99] b. TXA2
a. Abruptio placentae c. Cladribine
b. Ectopic preganancy d. COX
c. Plancenta accreta e. PG
d. Trophoblastic disease 207. Inhibits mitosis by forming microtubules :
202. Site of action of amphotericin is : [PGI June- 99] a. Vinca alkaloids [PGI June 98]
a. Cell membrane b. Docetaxel
b. Cytoplasm c. Etoposide
c. Nucleus d. Colchicine
d. Protein synthesis 208. Adriamycin is classified as : [PGI June 97]
203. Mechanism of action of chloramphenicol is : a. Antibiotic
a. Preventing binding of t RNA to ribosome b. Antimetabolite
b. Polypeptide elongation [PGI June 99] c. 5 - FU antagonist
c. Protein chain elongation d. Vinca alkaloids
d. Inhibiting cell wall synthesis 209. Long term use of chloroquine causes A/E.
204. Aerosolized ribavirin is used in the treatment of a. Lichenification [PGI June 97]
bronchiolitis with : [PGI June 99, June 98] b. Retinitis pigmentosa
a. RSV c. Yellow discoloration of skin.
b. H. influenza d. Corneal deposits
c. Pneumoccus 210. Hepatitis is a side effect of following except :
d. Streptococcus a. INH [PGI June 97]
205. Mechanism of action of zidovudine is : b. Rifampicin
a. Protein synthesis [PGI June 99] c. Pyrazinamide
d. Ethambutol

Answer 200. b. Methotrexate 201. d. Trophobla ... 202. a. Cell membrane 203. b. Polypeptide ...204. a. RSV
205. b. Reverse ... 206. a. Pentostatin 207. b. Docetaxel 208. a. Antibiotic 209. b and c
210. a and b

440
Antimicr obial & Antineoplastic Dr
Antimicrobial ugs
Drugs 6
ANSWERS, REFERENCES, EXPLANATIONS WITH INFORMATIVE ILLUSTR ATIONS

1. Ans. is d i.e. Doxycycline Ref. KDT 6/e, p 706, 713

Doxycycline is primarily excreted in faeces as conjugate hence nephrotoxicity is rare. All tetracyclines, except
doxycycline, accumulate and enhance renal failure.

Dose adjustment in renal failure


Safe in renal failure Contraindicated in renal failure
• Doxycycline • Erythromycin • All tetracyclines except Doxycycline
• Clindamycin • Chloramphenicol • Aminoglycosides
• Metronidazole • Ceftriaxone • Cephalosporin Ist generation
• Cefoperazone • Cefaclor • Nalidixic acid
• Rifampicin • Nafcillin • Nitrofurantoin
• Carbenicillin • Mezlocillin • Amphotericin – B

2. Ans. is d i.e. Doxycycline Ref. KDT 6/e, p 711

Doxycycline acts via inhibiting 30S ribosomal units. Other durgs which acts via inhibiting protein
synthesis are given below:
Drugs act On 30S
• Aminoglycoside Freeze chain initiation/interfere with polysome formation/misreading of mRNA
codons.
• Tetracycline Ribosome and inhibit aminoacyl t-RNA attachment to the A site so A chain fails to
grow.

On 50S
• Chloramphenicol Ribosome and interfere with peptide bond formation and transfer of peptide chain
from P site.
• Erythromycin & Ribosome and hinder translocation of elongated peptide chain from A to P site.
Clindamycin Premature chain termination site, the ribosome does not move along mRNA to
expose the next codon.

3. Ans. is a i.e. Penicillin V (Phenoxymethyl pencillin) Ref. KDT 6/e, p 699

• Penicillin G is highly active against most Gram positive, negative cocci, and against some spirochetes and
actinomycetes.
• Penicillin G on oral administration is poorly absorbed because gastric juice at pH 2 destroys the drug rapidly
and hence is administered IM.
• The excretion of penicillin G can be reduced by the concurrent use of probenecid, which blocks the active
secretion of penicillin by the renal tubule.

PhenOxymethyl penicillin, ClOxacillin, DiclOxacillin, FluclOxacillin: All these capital ‘O’ containing penicillins
can be given Orally.

441
6 Self Assessment & Review Phar macology
Pharmacology
Classification of narrow spectrum penicillin and its route of administration are given below:
Narrow spectrum penicillin
Beta lactamase sensitive Beta lactamase resistant
Acid stable Beta lactamase sensitive and acid stable Beta lactamase resistant and acid stable
– Penicillin V (Phenoxymethyl penicillin) – Cloxacillin (Oral, IM)
Oral – Dicloxacillin (Oral, IM)
– Flucloxacillin (Oral, IM)
Acid labile Beta lactamase sensitive and acid labile Beta lactamase resistant and acid labile
– Penicillin G (Benzyl penicillin) IM, IV – Methicillin (IM, IV)
– Procaine penicillin-G (IM depot) – Nafcillin (IM, IV)
– Benzathine penicillin-G (IM depot)

4. Ans. is a i.e. Piperacillin Ref. Harrison 16/e, p 894, Harrison 17/e, p 952

Antipseudomonal agents (Drugs in class are listed in order of decreasing in vitro activity).
Antipseudomonal Penicillins Antipseudomonal cephalosporins Fluoroquinolones
Piperacillin Ceftazidime Ciprofloxacin
Piperacillin/ Tazobactam Cefoperazone Levofloxacin
Mezlocillin Cefepime
Ticarcillin
Ticarcillin/ clavulanate
Carbapenems Monobactams Aminoglycosides Other agents
Imipenem/ Cilastin Aztreonam Tobramycin Polymyxin B
Meropenem Gentamicin Colistin
Amikacin

Note : For multidrug resistant P. aeruginosa infections, Colistin is being used in the shortest possible period.

5. Ans. is d i.e. More at 37 degree Celsius Ref. Harrison 17/e, p 806, 855, 879; Katzung 10/e, p 727

The production of a novel penicillin binding protein (PBP 2a or 2“) is primarily responsible for methicillin resistance.
This protein is synthesised by the mecA gene, which is part of a large mobile genetic element – a pathogenecity
or genomic island – called SCCmec.
It is hypothesised that acquisition of this genetic material resulted from horizontal transfer from a related
staphylococcal species, S. Sciuri.
Detection of methicillin in the microbiologic laboratory can be difficult if the strain expresses heterogeneous
resistance. Therefore, susceptibility studies are routinely performed at reduced temperatures (<35oC for 24
hours) with increased concentration of salt in the medium to enhance the expression of resistance.

Altered target PBPs are the basis of methicillin resistance in staphylococci and of penicillin resistance in
pneumococci and enterococci.

Resistance to penicillins and other beta-lactams is due to the four general mechanisms:
• Inactivation of antibiotic by beta-latamase
• Modification of target PBPs

442
Antimicr obial & Antineoplastic Dr
Antimicrobial ugs
Drugs 6
• Impaired penetration of drug to target PBPs
• Active efflux
Genes encoding beta-lactamases have been found in both chromosomal and extra chromosomal location and
in both gram positive and gram negative bacteria, these genes are often on mobile genetic elements.
Advance generation betal-lactam antibiotics, such as ceftriaxone and cefepime are stable in the presence of
plasmid mediated beta lactamases and are active against bacterial resistant to earlier generation betal-lactam
antibiotics.

6. Ans. is a i.e. Not teratogenic Ref. Harrison 17/e, p 859, 862 - 863

Contraindication of Tetracyclines
First trimester of pregnancy: tetracycline may cause staining of a child’s deciduous teeth. They have chelating
property : calcium-tetracycline chelate gets deposited in developing bone and teeth. The deciduous teeth are
affected if administered upto 5th month of pregnancy.
Tetracyclines are the most common antibiotics causing superinfections, as they cause marked suppression of
the resident flora. The intestinal superinfection by Candida albicans is most prominent; pseudomembranous
colitis can also rarely occur. Doxycycline and minocycline are less liable to cause diarrhea as only small
amounts reach the lower bowel in active form.

Mnemonic : Tetracycline : teratogenicity


TEtracycline is a TEratogen that causes staining of TEeth in the newborn.

Antibacterial drugs their toxicity and their recommendation are given below.
Antibacterial drug Toxicity in pregnancy Recommendation

Aminoglycosides Possible 8th nerve toxicity Caution


Chloramphenicol Gray syndrome in newborn Caution at term
Fluoroquinolones Arthropathy in immatue animals Caution
Clarithromycin Teratogenicity in animlas Contraindicated
Ertapenem Decreased weight in animal Caution
Erythromycin estolate Cholestatic hepatitis Contraindicated
Imipenem / cilastatin Toxicity in some pregnant animals Caution
Linezolid Embryonic and fetal toxicity in rats Caution
Meropenem Unknown Caution
Metronidazole None known, but carcinogenic in rats Caution
Nitrofurantoin Hemolytic anemia in newborns Caution; contraindicated at term
Quinupristin / dalfopristin Unknown Caution
Sulfonamides Hemolysis in newborn with G6PD Caution; contraindicated at term
deficiency; kernicterus in newborn
Tetracyclines / tigecycline Tooth discoloration, inhibition of bone Contraindicated
growth in fetus; hepatotoxicity
Vancomycin Unknown Caution

443
6 Self Assessment & Review Phar macology
Pharmacology
7. Ans. is a i.e. Tetracycline Ref. Harrison 17/e, p 892 - 893, O.P.Ghai 6/e, p 226 - 227

The close contacts with patients of diphtheria irrespective of their immunization status should be observed
carefully for carrier states or active lesions, cultured for C. diphtheriae and given chemoprophylaxis with:
a. Erythromycin : 40-50 mg/Kg/day in 2-4 divided doses in children (500 mg IV every 6 hours in adults).
b. Procaine penicillin G : 12500-25000 U/Kg in children (600000 units IM every 12 hours until the patient can
swallow comfortably, after which oral penicillin is given at 125-250 mg QID for a 14 day course.
• Alternative agents for patients who are allergic to penicillin or cannot take erythromycin include Rifampicin
and Clindamycin.

8. Ans. is a i.e. Amikacin Ref. Harrison 17/e, p 959

Antibiotic therapy for enteric fever in adults:


Indication Agent Dosage (route) Duration, days
Empirical treatment
Ceftriaxonea 1-2 g/d (IV) 7-14
Azithromycin 1 g/d (PO) 5
Fully susceptible
Ciprofloxacinb (1st line) 500 mg bid (PO) 5-7
or 400 mg q12h (IV)
Amoxicillin (2nd line) 1 g tid (PO) 14
or 2 g q6h (IV)
Chloramphenicol 25 mg/kg tid (PO or IV) 14-21
Trimethoprim-sulfamethoxazole 160/800 mg bid (PO) 14
Multidrug resistant
Ciprofloxacin 500 mg bid (PO) 5-7
or 400 mg q12h (IV)
Ceftriaxone 2-3 g/d (IV) 7-14
Azithromycin 1 g/d (PO) 5
Nalidixic acid resistant
Ceftriaxone 1-2 g/d (IV) 7-14
Azithromycin 1 g/d (PO) 5
High dose ciprofloxacin 750 mg bid (PO) 10-14
or 400 mg q8h (IV)

a
or another 3rd generation cephalosporin (e.g. cefotaxime, cefexime)
b
or ofloxacin

9. Ans. is a i.e. Doxycycline Ref. Nelson 18/e, p

Children more than 8 years old may take doxycycline orally 2 mg/kg daily for short periods, but increased
photosensitivity to sunlight may occur.

444
Antimicr obial & Antineoplastic Dr
Antimicrobial ugs
Drugs 6
10. Ans. is a i.e. Clofazamine Ref KDT 6/e, p 757, 766

Clofazamine is an antileprotic drug.


Ciclopirox olamine is a newer drug effective in tinea infectons, pityriasis versicolor and dermal candidiasis:
high cure rates are reported.

Classification of antifungal drugs based on their mechanism of action


• Inhibition of fungal cell wall synthesis – Caspofungin
• Bind to fungal cell membrane ergosterol – Amphotericin B, Nystatin
• Inhibition of ergosterol + Ianosterol synthesis – Terbinafine, Naftifine, Butenafine
• Inhibition of ergosterol synthesis – Miconazole, Clotrimazole, Ketoconazole, Fluconazole, Itraconazole and
Voriconazole.
• Inhibition of nucleic acid synthesis – 5-Flucytosine
• Disruption of mitotic spindle and inhibition of fungal mitosis – Griseofulvin
• Miscellaneous agents – Ciclopirox, Tolnaftate, Haloprogin, and Undecylenic acid.

11. Ans. is b i.e. Polymerization of tubules / assembly of tubular dimers


Ref. KDT 6/e, p 825, Goodman & Gilman 11/e, p 1352 - 1353

Paclitaxel is the original member of the taxane group of anticancer drug.


• It is a potent inhibitor of eukaryotic cell replication, blocking cell in the late G2-M phase of the cell cycle.
• It promotes abnormal assembly of microtubules via stabilization. Microtubules are not static organelles but
are in a state of dynamic equilibrium with their components, tubulin dimers.
Paclitaxel alters the equilibrium in favor of microtubules. So, abnormal array or ‘bundles’ of microtubules are
produced throughout the cell cycle (Spindle poison during mitosis through depolymerisation).

Therapeutic uses : • Ovarian cancer


• Advanced breast cancer
• Lung carcinoma
• Other tumours - head and neck and cervical carcinoma

12. Ans. is a i.e. Hirsutism


Ref. KDT 6/e, p 840; Dollery 3/e, p T8; Goodman & Gilman 11/e, p 1412 - 1413

Tacrolimus leads to less marked hypertension, hirsutism and gum hyperplasia as compared to cyclosporine;
however, it causes more marked diabetes, neurotoxicity, alopecia and diarrhea. The renal toxicity is the dose
limiting one.

Adverse reactions of tacrolimus:


• Allergic and anaphylactic reactions in IV use.
• Renal dysfunction is frequent and damage to the glomerulus and / or renal tubules is commonly expressed
as oliguria or increase in serum creatinine and urea.
• Tubular necrosis and renal failure may occur rarely.
• Leukocytosis is frequent complication.

445
6 Self Assessment & Review Phar macology
Pharmacology
• Hyperglycemia occurs frequently, by reversible inhibition off insulin gene transcription.
• Hypertension, angina pectoris tachycardia are seen occasionally.
• Cardiomyopthay in children receiving high dosage of tacrolimus.
• Tremor, headache and insomnia.
• Neuropathy, paraesthesia, anxiety, amnesia and encephalopathy occasionally occur.
• Hyperkalemia, hyperphosphatemia.

Tacrolimus (FK - 506)

It is a macrolide isolated from soil fungus

MOA Binds to cyclophilin FKBP - 12



Drug - FKBP complex binds to calcineurin

Inhibit calcineurin PO4– activation of T cell transcription factor

↓ IL - 2, IL - 3 & interferon - γ
Indications Liver kidney transplant given with glucocorticoids
Potency 10 -100 times more potent than cyclosporine
PK Highly bound to serum proteins, undergoes hepatic metabolism
by CYP3A4, renal excretion is very low

13. Ans. is a i.e. Bleomycin Ref. KDT 6/e, p 826, Goodman & Gilman 11/e, p 1194

Option “a” • Bleomycin is a mixture of closely related glycopeptide antibiotics having potent antitumour
activity.
• It chelates copper or iron, produces superoxide ions and intercalates between DNA
strands-causes chain scission and inhibits repair.
• It is highly effective in testicular tumour and squamous cell carcinoma of skin, oral cavity,
head and neck, genitourinary tract and esophagus; also useful in Hodgkin’s lymphoma.
Option “b” • Doxorubicin - anthracycline
• Dacarbazine - alkylating agent
Option “c” • Vancomycin is a glycopeptide antibiotic produced by streptococcus orientalis. It is not
used as an antitumour agent.
Option “d” • Aspartame is the name for an artificial, non-saccharide sweetener, aspratyl-phenylalanine-
1methyl ester; i.e., the methyl ester of the dipeptide of the amino acids aspartic acid and
phenylalanine.

14. Ans. is a i.e. Methotrexate therapy Ref. Dollery 3/e, p L18; KDT 6/e, p 823

Leucovorin is a vitamin which modulates the activity of certain antimetabolite anticancer drugs. It is
used as ‘rescue’ following methotrexate therapy and also to ‘potentiate’ the activity of 5-Fluorouracil
(5FU).
Leucovorin is also known as folinic acid or citrovorum factor. Following administration of high dose
of MTX, it is necessary to administer leucovorin as a rescue agent, to avoid excessive normal tissue
toxicity.

446
Antimicr obial & Antineoplastic Dr
Antimicrobial ugs
Drugs 6
Mechanism of action : Replenishment of the reduced folate pool, depleted as a result of inhibition of DHFRase
by MTX, was the mechanism of rescuer.
Modulation of 5-FU : The major locus of action of 5-FU is thymidylate synthase (TS) the enzyme responsible
for the formation of dTMP from dUMP. The 5-FU metabolite FdUMP is a potent inhibitor of TS forming an
irreversible ternary complex with TS and the cofactor 5, 10-methylene-THF. It has been suggested that reduced
intracellular concentrations of 5, 10-CH2-FH4 may limit the formation of the ternary complex and hence limit the
cytotoxicity of 5-FU.

15. Ans. is c i.e. L-Asparaginase Ref. Goodman & Gilman 11/e, p 1364

L-Asparaginase has minimal effects on bone marrow and GI mucosa (unlike other routine anticancer drugs).
The most serious toxicities result from its antigenicity as a foreign protein and its inhibition of protein synthesis.

Adverse events of L-Asparaginase due to inhibition of protein synthesis :


• Hyperglycemia : Due to insulin deficiency
• Clotting abnormalities : Due to deficient clotting factors (Hypercoagulability)
• Hypoalbuminemia
Note :
– L-Asparaginase is the only enzyme being used as an anticancer drug.
– It is used for the treatment of childhood ALL in combination with other drugs.
– It may also lead to CNS symptoms as Coma due to NH3 toxicity resulting from L-Asparaginase hydrolysis
and Pancreatitis.
– It blocks Methotrexate action : This occurs due to inhibition of protein synthesis by L-asp, an effect that
stops the progression of cells through the cell cycle and negates the effect of methotrexate.

16. Ans. is a i.e. Cefaclor Ref. Harrison 17/e, p 880

Harrison states “The resistance of Staph. aureus to methicillin indicates resistance to all semisynthetic
penicillinase resistant penicillins (SPRPs) as well as all cephalosporins. Many MRSA isolates are also
resistant to aminoglycosides, quinolones and macrolides”. Hence Option “a” is straight away ruled out.
Vancomycin is the treatment of choice while there are several other alternative drugs as shown below.
Very frequently asked question, so remember the full chart
Staphylococcus aureus DOC Alternatives
Methicillin resistant • Vancomycin • TMP - SMX • Minocycline
• Teichoplanin • Ciprofloxacin • Quinupristin
• Linezolid • Dalfopristin
Penicllin sensitive • Penicillin G. • Nacfcillin
• Oxacillin
• Cefazolin
• Vancomycin
Methicillin sensitive • Nafcillin • Cefazolin
• Oxacillin • Vancomycin
Vancomycin resistant (VRSA) • Linezolid
Resistance

447
6 Self Assessment & Review Phar macology
Pharmacology
17. Ans. is b i.e. Neutropenia Ref. Goodman & Gilman 11/e, p 1440

Filgrastim is a recombinant human granulocyte colony stimulating factor (G-CSF) effective in the
treatment of severe neutropenia after autologous hematopoetic stem cell transplantation and high
dose cancer chemotherapy. It is also effective in treatment of severe congenital neutropenia, cyclic neutropenia,
myelodysplasia and marrow damage and neutropenia of AIDS patient receiving Zidovudine treatment.

Filgrastim-Other important aspects


• It mainly stimulates CFU-G to increase neutrophil production. It also enhances the phagocytic and
cytotoxic functions of neutrophils.
• It shortens the period of severe neutropenia and reduces morbidity secondary to bacterial and fungal infections.
• It is also routinely used in patients undergoing peripheral blood stem cell collection for stem cell transplantation.
• It is administered by s.c. injection or i.v. infusion over at least 30 mins at doses of 1-20 micro gm/ Kg/
Day.

Remember :
Filgrastim : Recombinant human G-CSF
Sargramostim : Recombinant human GM-CSF
Oprelvekin : Recombinant human Interleukin-11(Thrombopoeitic growth factor)
Amifostine : Thiophosphate cytoprotectant (To reduce Cisplatin toxicity)

... Katzung 10/e, p 540

18. Ans. is c i.e. Stavudine Ref. Katzung 10/e, p 799 – 80, Goodman Gilman 11/e, p 1286

• For stavudine, the major dose-limiting toxicity is a peripheral sensory neuropathy.


Drugs Characteristic side effects
Stavudine Peripheral neuropathy, lipodystrophy, hyperlipidemia, rapidly progressive ascending
neuromuscular weakness (rare), pancreatitis
Zidovudine Macrocytic anemia, neutropenia, nausea, headache, insomnia, asthenia
Lamivudine Nausea, headache, fatigue
Didanosine (ddl) Pancreatitis, diarrhea, nausea, hyperuricemia, peripheral neuropathy

• The major clinical toxicity associated with didanosine is dose dependent pancreatitis.

Note : – Stavudine and Didanosine are never used together as concurrent use leads to increased
severity of peripheral neuropathy and potentially fatal pancreatitis.
– Stavudine and Zidovudine are never used together as both compete for intracellular
phosphorylation.
– Didanoside and Tenofovir are not to be used together as tenofovir inhibits the enzyme purine
nucleoside phosphorylase leading to increase in didanosine AUC by 44% to 60%.
– Cross resistance extends to all FDA-approved NNRTIs. Therefore, any patient who fails
treatment with 1 NNRTI because of a specific resistance mutation should be considered to have
failed the entire class.

448
Antimicr obial & Antineoplastic Dr
Antimicrobial ugs
Drugs 6
19. Ans. is b i.e. 5-FU Ref. Harrison 17/e, p 522-526; KDT 6/e, p 819 - 821

5-FU is a pyrimidine antagonist antimetabolite antineoplastic drug used for metastatic breast cancer.
Alkylating agents are: cyclophosphamide, mechlorethamine, Melfalan, Chloranbucil, thiopa,
nitrosoureas.
Other antimetabolites are given below:

a. Folate antagonist • Methotrexate (Mtx)


b. Purine antagonist • 6 - mercaptopurine (6-MP)
• 6 - thioguanine (6-TG)
• Azathioprine, Fludarabine
c. Pyrimidine antagonist • 5 - Fluorouracil (5-FU)
• Cytarabine

20. Ans. is c i.e. Decitabine Ref. Goodman & Gilman 11/e, p 1342; Harrison 17/e, p 682

“Decitabine inhibit DNA methyltransferase.” .... Goodman & Gilman


“Decitabine by hypomethylating promoter region leads to increased expression of certain tumour
suppression genes.” ... Harrison

Selected new agents under study for treatment of adults with AML.
Class of Drugs Examples Agent(s)
• MDR1 modulators Cyclosporine
• Demethylating agents Decitabine, 5-azacytidine, zebularine
• Histone deacetylase inhibitors Suberoylanilide hydroxamic acid (SAHA), valproic acid
• Heavy metals Arsenic trixoide, antimony
• HSP-90 antagonists 17-allylaminogeldanamycin (17-AAG) or derivatives
• BCR-ABL PDGFR/KIT inhibitors Imatinib, dasatinib, nilotinib
• Cell cycle inhibitors Flavopiridol
• Nucleoside analogues Clofarabine, troxacitabine
• Humanized antibodies Anti-CD33 (SGN33), anti-DR4, anti-DR5, anti-KiR
• Toxin-conjugated antibodies Gemtuzumab ozogamicin
• Radiolabeled antibodies Yttrium-90-labeled human M195

21. Ans. is a i.e. Osteosarcoma


Ref. KDT 6/e, p 823; Harrison 17/e, p 612; Goodman & Gilman 11/e, p 1338

The effective drugs for treating osteosarcoma are doxorubicin, ifosfamide, cisplatin and high dose
methotrexate with leucovorin rescue.
The use of Methotrexate has enlarged its scope to treat many difficult neoplasms.

449
6 Self Assessment & Review Phar macology
Pharmacology
Uses of Methotrexate (Mtx)

• Acute leukemia • High grade non-Hodgkin’s lymphoma


• Breast carcinoma • Gastric carcinoma
• Choriocarcinoma • Head and neck carcinomas
• Severe psoriasis • Rheumatoid arthritis
• Severe juvenile chronic arthritis

High dose Methotrexate with leucovorin rescue (HDM-L) is used for :


• Adjuvant therapy of osteosarcoma • CNS Lymphomas (Single agent therapy)
• Childhood ALL (Combination therapy)
Dosage of HDM-L : 7.5 g/m2 infusion for 6 hours followed by leucovorin at a dose of 15 mg/m2 every 6 hours for
7 doses.
Note : – Osteosarcoma is radioresistant. Preoperative chemotherapy followed by limb-sparing surgery
(which is accomplished in >80% of patients) followed by postoperative chemotherapy is the standard
management.
– The most important prognostic factor for long term survival is response to chemotherapy.
– Malignant fibrous histiocytoma is considered a part of the spectrum of osteosarcoma and is
managed similarly.

22. Ans. is a i.e. NNRTI Ref. KDT 6/e, p 767; Harrison 17/e, p 1192; Goodman & Gilman 11/e, p 1274, 1294

Replicative cycle of HIV-1, and example of a retrovirus, showing the sites of action of antiviral agents

450
Antimicr obial & Antineoplastic Dr
Antimicrobial ugs
Drugs 6
23. Ans. is c i.e. Mycophenolate inhibit purine synthesis via GMP dehydrogenase
Ref. Harrison 17/e, p 1778; Goodman & Gilman 11/e, p 1413; KK Sharma 1/e, p 913, 382; KDT 6/e, p 841

Maintenance Immunosuppressive Drugs


Agent Pharmacology Mechanisms Side effects

Glucocorticoids Increased bioavailability Binds cytosolic receptors & Hypertension, glucose


with hypoalbuminemia heat shock proteins. Blocks intolerance, dyslipidemia,
& liver disease; predni- transcription of IL-1, -2, -3, -6 osteoporosis
sone / prednisolone TNF-α, and IFN-γ
generally used
Cyclosporine Lipid-soluble, polypeptide, Trimolecular complex with Nephrotoxicity, hyperten-
(CsA) variable absorption, micro- cyclophilin & calcineurin → sion, dyslipidemia, glucose
emulsion more predictable block in cytokine (e.g., IL-2) intolerance, hirsutism/hyper-
production; however, stimulates plasia of gums unusual, &
TGF-β production diabetes more likely
Tarcrolimus Macrolide, well abosorbed Trimolecular complex with Similar to CsA, but hirsutism/
(FK506) FKBP-12 & calcineurin →block hyperplasia of gums un-
in cytokine (eg., IL-2) produciton; usual, & diabetes more likely
may stimulate TGF-β production
Azathioprine Mercaptopurine analogue Hepatic metabolites inhibit Marrow suppression (WBC >
purine syntheis RBC > platelets)
Mycophenolate Metabolized to mycophenolic Inhibits purine synthesis via Diarrhea/cramps; dose-
mofetil (MMF) acid inosine monophosphate related liver & marrow
dehydrogenase suppression is uncommon
Sirolimus Macrolide, poor oral bio- Complexes with FKBP-12 & Hyperlipidemia, thrombocy-
availability then blocks p70 56 kinase in topenia
the IL-2 receptor pathway for
proliferation

Mycophenolate is semisynthetic fungal antibiotics.


It is a new immunosuppressant, a prodrug of mycophenolic acid which selectively inhibits inosine monophosphate
dehydrogenase, an enzyme essential for de novo synthesis of guanosine nucleotides in the T and B cells.
It inhibits lymphocyte proliferation, antibody production and cell mediated immunity.
It also interferes with leucocyte adhesion to endothelial cells through inhibition of E-selectin and P-selectin.
Adverse effects : Vomiting, diarrhea and leucopenia.

24. Ans. is c i.e. Abacavir Ref. KDT 6/e, p 767; Harrison 17/e, p 1193; Goodman & Gilman 11/e, p 1350

All “vir” comes under protease inhibitors except abacavir and tenofovir which are NRTIs.
Nucleoside RTIs
• Zidovudine • Didanosine
• Zalcitabine • Stavudine
• Lamivudine • Abacavir
• Emtricitabine

451
6 Self Assessment & Review Phar macology
Pharmacology
25. Ans. is c i.e. Piperacillin Ref. Harrison 16/e, p 894, Harrison 17/e, p 952

Already explained, refer answer no. 4

26. Ans. is d i.e. Nitrofurantoin Ref. KDT 6/e, p 668 - 669

Nitrofurantoin blocks carbohydrate metabolism by inhibiting its acetyl CoA synthesis.


Nitrofurantoin by inhibiting intermediary metabolism in bacteria causes its action.
Mechanism of action of antibiotics

↓ ↓ ↓ ↓ ↓
On cell wall On cell membrane Protein synthesis Intermediary On DNA gyrase
inhibitors metabolism

Inhibit cell wall Promote leakage MNEMONIC Sulfonamides Fluoroquinolones


synthesis from membrane Bye AT 30s Trimethoprim
– Penicillins Polypeptide CELLS AT 50s Nitrofurantoin
– Vancomycin • Polymyxin Pyrimethamine
– Cephalosporins • Colistin Ethambutol
– Cycloserine Polyene PAS
– Bacitracin • Amphotericin-B
– Monobactam • Nystatin
– Carbapenems
– Teicoplanin
– Fasfomycin

↓ ↓
30S Inhibitors 50S inhibitors
Aminoglycosides : Chloramphenicol :
Inhibits/ freeze initiation complex Interference with peptide bond
Interfere with polysomes formation and transfer of
formation and peptide chain from P-site.
misreading of m-RNA Erythromycin telithromycin
(1st step in protein synthesis) Hinder translocation
Tetracyclines : of elongated peptide chain
Inhibit Aminoacyl t-RNA back from A site to P-site.
attached to A site Lincomycin, Linezolid, Clindamcin,
Streptogramins (Quinupristin, Dalfopristin)

Exception : Linezolid binds to 50S ribosomal unit near interface between 30S subunit and thus, inhibits the
formation of initiation complex.

27. Ans. is c i.e. It also inhibits calcineurin Ref. KDT 6/e, p 841

Mycophenolate is a new immunosuppressant, prodrug of mycophenolic acid which selectively inhibits inosine
monophosphate dehydrogenase an enzyme essential for de novo synthesis of guanosine nucleotides in the
T and B cells.

452
Antimicr obial & Antineoplastic Dr
Antimicrobial ugs
Drugs 6
It inhibits lymphocyte proliferation, antibody production and cell mediated immunity.
It also interferes with leucocyte adhesion to endothelial cells through inhibition of E-selectin and P-selectin.
Adverse effects : Vomiting, diarrhea and leucopenia.

28. Ans. is b i.e. Rickettsial infection Ref. KDT 6/e, p 696

Antibacterial spectrum of penecillin-G


PnG is a narrow spectrum antibiotic; activity is limited primarily to gram positive bacteria.
Cocci Bacillim
Group A Streptococci Bacillus anthracis
Group B Pneumococci Corynebacterium diphteriae
Staphylococcus aureus All clostridia
Neisseria gonorrhoea Listeria
Neisseria Meningitidis Spirochetes (syphilis)
Actinomyces

Actinomyces israelii is only moderately sensitive. Majority of gram negative bacilli (except a few E. coli and
Proteus), Mycobacterium tuberculosis, Rickettsiae, chlamydiae, protozoa, fungi and viruses are totally insensitive
to PnG.

29. Ans. is b i.e. Sulphadiazine Ref. KDT 6/e, p 683 - 684

Sulphadiazine is rapidly absorbed orally and rapidly excreted in urine, hence given orally.
• It is 50% plasma protein bound and 20-40% acetylated [↑ ↑ risk of crystalluria].
• It has good penetration in CSF and brain, therefore used in meningitis.
Sulfonamides
Oral Topical
– Sulfadiazine – Sulfacetamide-eye drop ointment
– Sulfamethoxazole – Mafenide -cream (used in burns)
– Sulfamoxole – Silver sulfadiazine-cream (used in burns)
– Sulfadoxine
– Sulfamethopyrazine
– Sulfasalazine

30. Ans. is c i.e. Antimalarial Ref. CMDT ‘05, p 875; Harrison 17/e, p 1270-1290

Lumefantrine has blood schizontocidal activity against a wide range of Plasmodia.

LUMEFANTRINE
• Lumefantrine is an aryl alcohol related to halofantrine, available as fixed-dose combination with artemether.
• Combined formulation of artemether and lumefantrine has been developed for treatment of falciparum malaria
in areas where P. falciparum is resistant to chloroquine and antifolates.
• Lumefantrine does not cause cardiotoxicity like halofantrine.

453
6 Self Assessment & Review Phar macology
Pharmacology

– Mode of action similar to the arylamino alcohol group of antimalarial drugs like : Quinine and
Mefloquine, Halofantrine.
– Mechanism of action it interact with heme.
– Pharmacokinetics same as Halofantrine, oral absorption is highly variable and improved when
drug taken with food.
– Half life 4.5 hours
– Recrudescence rate is low

31. Ans. is b i.e. Irinotecan Ref. KDT 6/e, p 825-826

Topotecan and irinotecan are two recently introduced semisynthetic analogue of camptothecin.
Irinotecan alters the function of the enzyme, Topoisomerae-I.
ETOPOSIDE IRINOTECAN DOXORUBICIN
• A semisynthetic derivative • A semisynthetic derivative • Anticancer antibiotics
of podophyllotoxin of camptothecin
• Alters the enzyme • Alters the enzyme • Activate topoisomerase IInd
Topoisomerase IInd function Topoisomerase Ist function
• Breaking of DNA strand not • Braking of DNA strand not • Causes breaks in DNA
affected, only resealing of affected, only resealing of strands, generates quinone
strand is prevented strand is prevented type free radicals
• Used primarily in testicular • Used in advanced colorectal • Used in acute leukemia, many
tumours, Ca lung, Hodgkins cancer, Ca lung, cervix, ovary solid tumours
and Ca bladder
• Side effects • Side effects • Side effects
– Alopecia – Diarrhea – Cardiotoxicity
– Leucopenia – Neutropenia – Marrow depression
– G.I. disturbances – Thrombocytopenia – Alopecia
– Haemorrhage – Stomatitis

Vincristine : • A mitotic inhibitor


• Metaphase arrest occurs
• Most useful in inducing remission in childhood acute leukemia, not good for maintenance.
• Adverse effects : – Peripheral neuropathy – Alopecia
– Bone marrow depression is minimal – SIADH

32. Ans. is c i.e. Non-nucleoside reverse transcriptase inhibitor


Ref. KDT 6/e, p 767; Harrison 17/e, p 1192; Goodman & Gilman 11/e, p 1350

Already explained, refer answer no. 10

33. Ans. is d i.e. Cloxacillin Ref. KDT 6/e, p 100; Harrison 16/e, p 47

Penicillinase resistant penicillins are : – Methicillin – Oxacillin – Flucloxacillin


– Cloxacillin – Naficillin

454
Antimicr obial & Antineoplastic Dr
Antimicrobial ugs
Drugs 6
Classification of Semisynthetic Penicillins

Acid resistant Penicillinase resistant- Extended spectrum- Lactamase inhibitor


penicillins pencillins
• Penicillin -V • Methicillin • Clavulanic Acid
• Oxacillin • Sulbactam
• Cloxacillin

Aminopenicillins Carboxypenicillins Ureidopenicillin Mecillinam


- Ampicillin - Carbenicillin - Piperacillin - Amdinocillin
- Bacampicillin - Carbenicillin indanyl - Mezlocillin
- Amoxicillin - Carbenicillin phenyl
- Ticarcillin

34. Ans. is d i.e. Flutamide Ref. Lippincott’s Pharmacology 2/e, p 378

Here, we can reach the answer simply by excluding other options, i.e. Chlorambucil, Daunorubicin, Doxorubicin
are well known agents causing bone marrow depression.
Flutamide
A nonsteroidal drug having specific antiandrogenic action but no other hormonal activity.
It increases LH secretion by blocking feed back inhibition.
Plasma testosterone levels increase in males which partially overcome the direct antiandrogenic action of
flutamide.
Uses : • Palliative effect may occur in advanced prostatic carcinoma.
• Has been tried in female hirsutism along with oral contraceptives.
Side Effects : • Gynecomastia • Breast tenderness
• Liver damage • Nausea and hot flushes

Comparison of Myelosuppressive potential of anticancer drugs

455
6 Self Assessment & Review Phar macology
Pharmacology
35. Ans. is c i.e. Ampicillin and Chloramphenicol Ref. KDT 6/e, p 676 - 678

• Bactericidal Drugs : Drugs acts primarily on rapidly multiplying bacteria.


• Bacteriostatic Drugs : Drugs which retards the multiplication of bacteria.

Here in question, ampicillin is cidal and chloramphenicol is a static drug. There is no beneficial role in combining
both because the multiplication of bacteria is prevented by chloramphenicol while ampicillin acts only on
rapidly multiplying bacteria. Hence, there is no role of giving ampicillin.

Note : If the organism is highly sensitive to the cidal drug-response to the combination of static and cidal drug
is equal to the static drug given alone (apparent antagonism), because cidal drug acts primarily on rapidly
multiplying bacteria, while the static drug retards multiplication e.g. penicillin + tetracycline/chloramphenicol on
pneumococci which are highly sensitive to penicillin.
• If the organism has low sensitivity to the cidal drug-synergism may be seen e.g. penicillin + sulphonamide
for actinomycosis, rifampicin + dapsone for leprosy.
Bactericidal agents Bacteriostatic agents
• Aminoglycosides • Chloramphenicol
• Bacitracin • Clindamycin
• Beta-lactam antibiotics • Ethambutol
• Isoniazid • Macrolides
• Metronidazole • Nitrofurantoin
• Polymyxins • Novobiocin
• Pyrazinamide • Oxazolidinedione
• Quinolones • Sulfonamides
• Quinupristin - dalfopristin • Tetracyclines
• Rifampicin • Trimethoprim
• Vancomycin

36. Ans. is a i.e. Sulfasalazine is absorbed well in GIT Ref. KDT 6/e, p 661, 682 - 684

Here, option ‘a’ is wrong because only 10-20% of orally administered Sulfasalazine is absorbed.
Sulfasalazine is a compound of 5-ASA with sulfapyridine linked by an azo bond. It has low solubility
and hence, poorly absorbed from the ileum.

Sulfonamides
Indications :
– Rheumatoid arthritis
– Juvenile chronic arthritis
– Ankylosing spondylitis
Side Effects :
• Nausea, vomiting and epigastric pain
• Crystalluria is dose related. It can be minimised by taking plenty of fluids and by alkalinizing the urine in
which sulfonamide and their acetylated derivatives are more soluble. (option ‘b’)
• Hypersensitivity reactions (serum sickness like syndrome, photosensitivity) is reported.
• Stevens - Johnson syndrome and exfoliative dermatitis are more common with long acting agents.
• Hepatitis

456
Antimicr obial & Antineoplastic Dr
Antimicrobial ugs
Drugs 6
• Topical use is not recommended because of risk of contact sensitization.
• Hemolysis in a dose dependent manner in individuals with G-6-PD deficiency.
• Kernicterus may be precipitated in the new born, especially if premature by displacement of bilirubin from
plasma protein binding sites and more permeable blood brain barrier. (option ‘c’)
• Sulfonamides are effective against Actinomyces, Nocardia and Toxoplasma. (option ‘d’)

37. Ans. is b i.e. Trovafloxacin Ref. Harrison 17/e, p 857; Goodman & Gilman 11/e, p 1121

• Fluoroquinolones which are excreted by non renal mechanisms do not require dose adjustment in renal
failure, they are:
– Nalidixic acid – Pefloxacin – Grepofloxacin
– Trovafloxacin – Moxifloxacin
• Fluoroquinolones which are excreted by renal mechanisms requires dose adjustment in renal failure, they
are:
– Ciprofloxacin – Cinafloxacin – Enoxacin
– Gatifloxacin – Levofloxacin – Lomefloxacin
– Norfloxacin – Ofloxacin
• Fluoroquinolones which are excreted 50% renally and 50% non renally, so dose adjustment is needed in
renal failure:
– Sparfloxacin

Antibacterial drug dose adjustments in patients with renal impairment (list in Harrison 17/e, p 857)
Antibiotic Major route Dosage adjustment with Renal impairment
of Excretion
Aminoglycoside Renal Yes
Azithromycin Biliary No
Cefazolin Renal Yes
Cefepime Renal Yes
Ceftazidime Renal Yes
Ceftriaxone Renal/biliary Modest reduction in severe renal impairment
Ciprofloxacin Renal/biliary Only in severe renal insufficiency
Clarithromycin Renal/biliary Only in severe renal insufficiency
Daptomycin Renal Yes
Erythromycin Biliary Only when given in high IV doses
Levofloxacin Renal Yes
Linezolid Metabolism No
Metronidazole Biliary No
Nafcillin Biliary No
Penicillin G Renal Yes (when given in high IV doses)
Piperacillin Renal Only with Clcr of <40 mL/min
Quinupristin/ dalfopristin Metabolism No
Ticarcillin Renal Yes
Tigecycline Biliary No
TMP-SMX Renal/biliary Only in severe renal insufficiency
Vancomycin Renal Yes

457
6 Self Assessment & Review Phar macology
Pharmacology
38. Ans. is c i.e. Streptomycin Ref. Harrison 17/e, p 859, 1032, KDT 6/e, p 909

Streptomycin, due its high risk of fetal ototoxicity is not used in pregnancy.
Pyrazinamide, Ethambutol is also unsafe in pregnancy.
Rifampicin, Rifabutin, Isoniazid is safe.

Use of First-Line Antimycobacterial agents in Patient with Renal or Hepatic disease and in Pregnant
women
Use in Indicated Circumstances
Renal Disease : Creatinine Clearance Rate

Agent Severe Hepatic Disease <60 but >30 mL/min <30 mL/min Pregnancy
Azithromycin No change No change Decrease dose No evidence of risk
Clarithromycin No change No change Decrease dose Risk cannot be ruled out
Ethambutol No change No change No change Risk cannot be ruled out
Isoniazid Avoid use or monitor carefully No change No change Risk cannot be ruled out
Pyrazinamide Avoid use or monitor carefully No change Decrease dose Risk cannot be ruled out
Rifabutin No change No change No change No evidence of risk
Rifampin Avoid use or monitor carefully No change No change Risk cannot be ruled out
Rifapentine Avoid use or monitor carefully No change No change Risk cannot be ruled out
Streptomycin No change Decrease dose Decrease dose & Definite evidence of risk
frequency

Use of antimycobacterial agents in patients with Renal or Hepatic disease


Important Points about ATT :
Renal Failure Liver Disease
• Rifampin, Isoniazid, Pyrazinamide are safe • Safe – Streptomycin and Ethambutol
• Avoid Streptomycin and Ethambutol • Do not give pyrazinamide
• Do not give Thiacetazone • Avoid – Rifampin

39. Ans. is a and b i.e. Adriamycin; and Doxorubicin Ref. Harrison 17/e, p 573, 580

Doxorubicin / Adriamycin is effective against leiomyosarcoma arising in the uterus.


A chemotherapy regimen including an anthracycline and ifosfamide with growth factor support improved overall
survival for high-risk extremity soft tissue sarcomas.

40. Ans. is a i.e. Tobramycin Ref. KDT 6/e, p 740; Harrison 17/e, p 1017-1018

• Multidrug resistant category is defined when there is resistance to INH or to INH+RMP or may be to more
number of drugs.
• For both INH + RMP resistance : PZA + ETB + Sm (or ethionamide) + Ciprofloxacin (or ofloxacin or levofloxacin)
are given for 12-18 months.
• Extensive drug resistance [XDR] strains mean resistant to FQ and one or more of the injectable drugs
amikacin, kanamycin or capreomycin. ... Harrison 17/e, p 1018
• CDR means complete drug resistance.

458
Antimicr obial & Antineoplastic Dr
Antimicrobial ugs
Drugs 6
Recommended antituberculosis Treatment Regimens
Initial Phase Continuation Phase
Indication Duration, Drugs Durations, Drugs
Months Months
New smear or culture positive 2 HRZE 4 HR
cases
New culture negative cases 2 HRZE 2 HR
Pregnancy 2 HRE 7 HR
Failure & relapse — — — —
Resistance (or intolerance) to H Throughout (6) RZE
Resistance to H + R Throughout (12-18) ZEQ + S (or another)
injectable agent)
Resistance to all first line drugs Throughout (24) 1 injectable agent +3 of
these 4; ethionamide,
cycloserine, Q, PAS
Standardized re-treatment 3 HRZES 5 HRE
(Susceptibility testing unavailable)
Drug intolerance to R Throughout (12) HZE — —
Drug intolerance to Z 2 HRE 7 HR

41. Ans. is c i.e Taxol Ref. Katzung 10/e, p 896; Harrison 17/e, p 568

Taxol : is a brand name for paclitaxel and is not a hormonal agent. It is indicated for metastatic ovarian and
breast cancer. Major toxicity of paclitaxel is reversible myelosuppression and “stocking and glove” neuropathy.

Now, let us consider the other 3 options separately.


Letrozole & • They are nonsteroidal competitive inhibitor of aromatase that has no inhibitory effect on
Anastrozole adrenal glucocorticoid or mineralocorticoid synthesis.
• They are presently approved for :
– First-line treatment of postmenopausal women with metastatic breast cancer that
is ER-positive.
– Treatment of postmenopausal women with metastatic breast cancer that is ER-
positive and has progressed while on tamoxifen therapy.
– An adjuvant therapy of postmenopausal women with hormone-positive, early-stage
breast cancer.
Exemestane • It is a steroidal hormonal agent that binds to and irreversibly inhibits aromatase.
• There appears to be a lack of cross resistance between exemestane and nonsteroidal
aromatase inhibitors.
• It is indicated for the treatment of advanced breast cancer in postmenopausal women
whose disease has progressed on tamoxifen therapy.
Tamoxifen • It is a selective estrogen receptor modulator (SERM) and is useful in estrogen receptor
expressing breast cancer.

459
6 Self Assessment & Review Phar macology
Pharmacology
Endocrine Therapies for Breast Cancer
Therapy Comments
Castration For premenopausal women
– Surgical
– LHRH agonist
Antiestrogens
– Tamoxifen Useful in pre & postmenopausal women
– Pure antiestrogens Responses in tamoxifen-resistant & aromatase inhibitor resistant patients
Surgical adrenalectomy Rarely employed second-line choice
Aromatase inhibitors Low toxicity, now first choice for metastatic disease
High-dose progestogens Common fourth-line choice after Als, tamoxifen & fulvestrant
Hypophysectomy Rarely used
Additive androgens or estrogens Plausible fourth-line therapies; potentially toxic

42. Ans. is c i.e. Simultaneous use with penicillin Ref. KDT 6/e, p 721; Harrison 17/e, p 862

Entry of aminoglycosides are increased by giving synergistically with penicillin as penicilin inhibits cell
wall and better penetration of aminoglycoside take place.
Penicillins administered with aminoglycosides do not increase the nephrotoxicity but are used synergistically.
All aminoglycosides are Ototoxic and Nephrotoxic.

Nephrotoxicity is more likely to be encountered when : • Used in the elderly patients


• Therapy continued for more than 5 days
• At higher doses
• In the setting of renal insufficiency.

Once daily administration of aminoglycosides may cause, less nephrotoxicity than more frequent
administration.

Comparative toxicity of important Aminoglycosides : ... KDT 6/e, p 721


Systemically used Ototoxicity Nephrology
Aminoglycoside vestibular cochlear
• Streptomycin ++ + +
• Gentamicin ++ + ++
• Kanamycin + ++ ++
• Tobramycin ++ + +
• Amikacin + ++ ++
• Sisomicin ++ + ++
• Netilmicin + + ++

43. Ans. is b i.e. Griseofulvin Ref. KDT 6/e, p 668 - 669; Harrison 17/e, p 852

Griseofulvin acts by interfering in mitosis (within nucleus); hence it certainly does not act on cell membrane.

460
Antimicr obial & Antineoplastic Dr
Antimicrobial ugs
Drugs 6
Mechanism of action of antibiotics:
On cell membrane : Promote leakage from membrane :

Polypeptide • Polymyxin • Colistin • Gramicidin


Polyene • Amphotericin-B • Nystatin • Daptomycin

44. Ans. is a i.e. Cell Wall Ref. KDT 6/e, p 668; Harrison 17/e, p 852

Bacitracin prevents addition of cell wall subunits by inhibiting recycling of membrane lipid carrier.
Bacitracin is a polypeptide antibiotic, act by inhibiting cell wall synthesis and active against gram
+ve organisms.

Drugs inhibiting cell wall synthesis

• Penicillins • Cephalosporins • Cycloserine • Teicoplanin


• Vancomycin • Bacitracin • Monobactum • Carbapenems

45. Ans. is b i.e. Rifampicin Ref. KDT 6/e, p 753; Harrison 17/e, p 1026

Rifampicin is the only bactericidal drug amongst the antileprosy drugs and can render Leprosy rapidly non
contagious.

Antimicrobial regimens recommended for the treatment of leprosy in adults


Form of Leprosy More Intensive Regimen WHO Recommended Regimen (1982)
Tuberculoid Dapsone (100 mg/d) for Dapsone (100 mg/d, unsuperised) plus rifampin (600 mg/
(paucibacillary) 5 years month, supervised) for 6 months
Lepromatous Rifampin (600 mg/d) Dapsone (100 mg/d) plus clofazimine (50 mg/d),
(multibacillary) for 3 years plus dapsone unsupervised; & rifampin (600 mg) plus clofazimine
(100 mg/d) indefinitely (300 mg) monthly (supervised) for 1-2 years

46. Ans. is b i.e. Pancreatic cancer Ref. Katzung 10/e, p 890 - 891; Harrison 17/e, p 588

Gemcitabine, a deoxycytidine analogue, produces improvement in the quality of life for patients with advanced
pancreatic cancer. However, duration of survival is only modestly improved.

GEMCITABINE
Mechanism of Action :
A pyrimidine antagonist
↓ Deoxycytidine kinase
Gets converted to Gemcitabine triphosphate in body

Inhibits DNA synthesis

461
6 Self Assessment & Review Phar macology
Pharmacology

Widely used in the treatment of : • Pancreatic cancer


• Non - small cell lung cancer
• Bladder cancer
Limited use in the treatment of : • Ovarian cancer
• Soft tissue sarcoma

Myelosuppression is the principal dose limiting toxicity.


The deoxycytidine analogue gemcitabine, given as a single agent (gemcitabine 1000 mg/m2 weekly for 7
weeks followed by 1 week rest, then weekly for 3 weeks every 4 weeks thereafter), has been the preferred
treatment for these patients since it was shown to yield clinical benefit and improved survival compared to 5-
fluorouracil. ... Harrison 17/e, p 588

47. Ans. is c i.e. Cefoperazone Ref. KDT 6/e, p 706

Cefoperazone is primarily excreted in bile, hence nephrotoxicity is rare.

Drugs causing Renal failurem


Drugs Reasons
COX Inhibitors, ACE-Inhibitors • Renal hypoperfusion
Amphotericin-B • Renal Artery Stenosis
Aminoglycoside, Cisplatin, Cyclosporine, Acetaminophen, Ethylene Glycol • Acute Tubular Necrosis
β-lactam, Sulfonamides, Diuretics, Captopril, NSAIDs, Allopurinol • Interstitial Nephritis

Drugs whose dose is not reduced in renal failure


• Doxycycline • Erythromycin
• Clindamycin • Chloramphenicol
• Metronidazole • Ceftriaxone
• Cefoperazone • Cefaclor
• Rifampicin • Nafcillin
• Carbenicillin • Mezlocillin

Antimicrobials contraindicated in renal failure


• All tetracyclines except Doxycycline • Aminoglycoside
• Cephalosporin Ist generations • Nalidixic acid
• Nitrofurantoin • Amphotericin – B

48. Ans. is a i.e. Folinic acid enhances the action of methotrexate


Ref KDT 6/e, p 592

• Folinic acid (Leucovorin or Citrovorum factor) is N5 Formyltetrahydrofolate. It is an active coenzyme which


does not need to be reduced by dihydrofolate reductase before it can act. In methotrexate treated patient,
DHFRase is inhibited but folinic acid bypasses this enzymatic block and is still able to act while folic acid
becomes useless.
• When leucovorin is used, the daily dose of 5 FLUOROURACIL must be reduced because Leucovorin
potentiates 5 - Fluorouracil.

462
Antimicr obial & Antineoplastic Dr
Antimicrobial ugs
Drugs 6

For more details about uses of MTX, refer answer no. 14

49. Ans. is c i.e. Ameliorate hemorrhagic cystitis Ref. KDT 6/e, p 822

Hemorrhagic cystitis caused by cyclophosphamide is blocked by systemic administration of mesna and


by irrigating the bladder by acetylcysteine.

Cyclophosphamide : is an alkylating agent used in chemotherapy. Cyclophosphamide is metabolized in liver


by enzyme of cytochrome P 450 into active metabolites.

Hemorrhagic cystitis is prevented by :


• Proper hydration after cyclophosphamide administration
• IV administration of free - SH group compounds bind and inactivate vescicotoxic acrolein.
Free SH group agents are : – Mesna (Systemic)
– Acetyl cysteine (Irrigatory)

Other side effects of cyclophosphamide : – Nausea/vomiting


– Alopecia

Ifosfamide : It is a congener of cyclophosphamide with long t½ (half life).


It is less emetogenic and there is less chance of alopecia than cyclophosphamide.
Dose limiting toxicity = heamorrhagic cystitis.
Treatment – IV. Mesna
1. Aldophosphamide ⎫
⎬ Prominent immunosuppressant properties
2. Phosphamide mustard ⎭

463
6 Self Assessment & Review Phar macology
Pharmacology
50. Ans. is a i.e. Binds to a site and inhibit attachment of t-RNA Ref. Harrison 17/e, p 852; KDT 6/e, p 711

Tetracycline binds to ribosomes and inhibit aminoacyl – t-RNA attachment to the A site then aminoacid
chain fail to group.

Mechanims of Action of and Resistance to Major Classes of Antibacterial Agents


Antibacterial Agent Major Cellular Mechanism of Action Major Mechanisms of Resistance
Target
β-Lactams (penicillin Cell wall Inhibit cell-wall cross-linking 1. Drug inactivation (β-lactamase)
& cephalosporins) 2. Insensivity of target (altered penicillin-binding
proteins)
3. Decreased permeability (altered gram –ve outer
membrane porins)
4. Active efflux
Vancomycin Cell wall Interferes with addition of new Alteration of target (substitution of terminal amino
cell wall subunits (muramyl acid of peptidoglycan subunit)
pentapeptides)
Bacitracin Cell wall Prevents addition of cell wall Not defined
subunits by inhibiting recycling
of membrane lipid carrier
Macrolides Protein synthesis Bind to 50S ribosomal subunit 1. Alteration of target (ribosomal methylation &
(erythromycin) mutation of 23S rRNA)
2. Active efflux
Lincosamides Protein synthesis Bind to 50S ribosomal subunit Alteration of target (ribosomal methylation)
(clindamycin)
Chloramphenicol Protein synthesis Bind to 50S ribosomal subunit 1. Drug inactivation (chloramphenicol acetyltrans-
ferase)
2. Active efflux
Tetracycline Protein synthesis Binds to 30S ribosomal subunit 1. Decreased intracellular drug accumulation
(active reflux)
2. Insensitivity of target
Aminoglycosides Protein synthesis Binds to 30S ribosomal subunit 1. Drug inactivation (aminoglycoside-modifying
(gentamicin) enzyme)
2. Decreased permeability through gram –ve outer
membrane
3. Active efflux
Mupirocin Protein synthesis Inhibits isoleucine tRNA synthetase Mutation of gene for target protein or acquisition of
new gene for drug insensitive target
Quinupristin / dalfo- Protein synthesis Binds to 50S ribosomal subunit 1. Alteration of target (ribosomal methylation:
pristin (Synercid) dalfopristin)
2. Active efflux (quinupristin)
3. Drug inactivation (quinupristin & dalfopristin)
Linezolid Protein synthesis Bind to 50S ribosomal subunit Alteration of target (mutation of 23S rRNA)
Sulfonamides & Cell metabolism Competitively inhibit enzymes Production of insensitive targets [dihydropteroate
trimethoprim involved in two steps of folic acid synthetase (sulfonamides) & dihydrofolate reductase
biosynthesis (trimethoprim)] that bypass metabolic block

464
Antimicr obial & Antineoplastic Dr
Antimicrobial ugs
Drugs 6
Rifampin Nucleic acid Inhibit DNA dependent RNA Insensitivity of target (mutation of polymerase gene)
synthesis polymerase
Metronidazole Nucleic acid Intracellularly generates short- Not defined
synthesis lived reactive intermediates that
damage DNA by electron transfer
system
Quinolones DNA synthesis Inhibit DNA gyrase (A subunit) & 1. Insensitivity of target (mutation of gyrase genes)
(ciprofloxacin) topoisomerase IV 2. Decreased intracellular drug accumulation
(active efflux)
Novobiocin DNA synthesis Inhibits DNA gyrase (B subunit) Not defined
Polymyxins Cell membrane Disrupt membrane permeability Not defined
(polymyxin B) by charge alteration
Gramicidin Cell membrane Forms pores Not defined
Daptomycin Cell membrane Forms channels that disrupt Not defined
membrane potential

51. Ans. is a i.e. Bleomycin Ref. Katzung 10/e, p 895

Bleomycin is an antibiotic anticancer drug, it causes pulmonary toxicity which usually presents with pneumonitis
with cough, dyspnoea, dry inspiratory crackles on physical examination and infiltrates on chest X-ray and
therefore not indicated in this patient with history of prior lung disease.
Drugs causing pleural / Pulmonary fibrosis
• Acyclovir • Amiodarone • Azathioprine
• Bleomycin • Busulphan • Carmustine
• Chlorambucil • Cyclosphosphamide • Gold
• Melphalan • Methotrexate • Methysergide
• Mitomycin C • Nitrofurantoin • Procarbazine
• Sulphonamides

52. Ans. is a i.e. Lamivudine Ref. KDT 6/e, p 767; Harrison 17/e, p 1192

Anti HIV drug which does not cause peripheral neuropathy is Lamivudine.
Side Effects of antiretroviral (HIV) drug

Pancreatitis + Peripheral neuropathy Lactic acidosis + Hepatomegaly with steatosis


Didanosine Zidovudine
Zalcitabine Zalcitabine
Stavudine

Most common side effects of Zidovudine is Macrocytic anaemia and Neutropenia.

53. Ans. is a i.e. Chloroquine Ref. KDT 6/e, p 786

Chloroquine can be used for treatment of malaria during pregnancy because no abortifacient or
teratogenic effects have been reported with it.

465
6 Self Assessment & Review Phar macology
Pharmacology
• Mefloquine can be used in pregnancy except in 1st trimester in an area which is chloroquine resistant.
• Quinine causes hemolysis in pregnant woman resulting in hemoglobinuria (black water fever) and kidney
damage.

54. Ans. is a i.e. Rifampicin Ref. Harrison 17/e, p 913

Chemoprophylaxis has been suggested for close contacts. Close contacts should receive :
• Rifampicin (DOC) – 600mg BD for two days.
Alternatively, adults can be treated by :
• Ciprofloxacin – 750 mg single dose
• Azithromycin – 500 mg single dose
• Ceftriaxone – 250 mg IM single dose (Safe in pregnancy)
• Ofloxacin – 400 mg single dose

Close contacts are those individuals who have had contact with oropharyngeal secretions of index case either
by kissing, sharing toys, beverage or cigarettes.
Chemoprophylaxis for various diseases
Disease Chemoprophylaxis
Meningitis Sulphadiazine for 4 days only if the strain is shown to be non resistant, for
household and close community contacts; Immunization should be initiated in
all cases (against serogroup A and C)
According to Harrison 16/e, 2476/854, Rifampicin 600mg bd for 2 days or/
Azithromycin
500 mg single dose
For Pregnant – Ciprofloxacin 500mg single dose.
Ceftriaxone 200 mg IM one dose.
Cholera Tetracycline or Furazolidone for household contacts.
Single dose Doxycycline is also preferred
Bacterial Conjunctivitis Erythromycin ophthalmic ointment (no effect on viral conjunctivitis)
Diphtheria Erythromycin (and first dose of vaccine)
Influenza Amantadine (effective only for type A)
Plague Tetracycline for contacts of Pneumonic plague.
Leprosy Dapsone.

55. Ans. is a i.e. Neomycin Ref. KDT 6/e, p 725

Normally NH3 is produced by colonic bacteria. This is absorbed and converted to urea by liver. In severe
hepatic failure, detoxification of NH3 dose not occur, blood NH3 level rise and produce encephalopathy. Neomycin,
by suppressing intestinal flora, diminishes NH3 production and lowers its blood level; clinical improvement is
seen in 2-3 days.

Uses of Neomycin : • Hepatic coma


• Topically for infected wound, ulcers, burns external ear infections,
conjunctivitis.
Other drugs : • Metronidazole, lactulose, rarely flumazenil.

466
Antimicr obial & Antineoplastic Dr
Antimicrobial ugs
Drugs 6
56. Ans. is c i.e. Ethambutol Ref. KDT 6/e, p 739

All first line antitubercular drugs are bactericidal except Ethambutol which is static.
Bacteriostatic ATT
• Ethambutol • PAS
• Ethionamide • Cycloserine

57. Ans. is c i.e. 5-flucytosine Ref. KDT 6/e, p 739 - 740

5-flucytosine is an anticancer drug and not an antitubercular drug.


ATT drugs are :
First Line Drugs Second Line Drugs
↓ Old Drugs Newer Drugs
• Isoniazid (H) • Thiacetazone • Ciprofloxacin
• Rifampin (R) • Paraaminosalicylic acid (PAS) • Ofloxacin
• Pyrazinamide (Z) • Ethionamide • Clarithromycin
• Ethambutol (E) • Cycloserine • Azithromycin
• Streptomycin (S) • Kanamycin • Rifabutin
• Amikacin • Sparfloxacin
• Capreomycin

58. Ans. is c i.e. Translocation Ref. KDT 6/e, p 71

• Erythromycin & binds to 50S ribosome and hinder translocation of elongated of peptide chain from
Clindamycin A to P site, the ribosome does not move along mRNA to expose the next codon.
• Aminoglycoside binds to 30S, freeze initiation and causes misreading of mRNA code.
• Tetracycline binds to 30S ribosome and inhibit aminoacyl t RNA attachment to the A site.
• Chloramphenicol binds to 50S ribosome and interfere with peptide bond formation & transfer of peptide
chain from P site.

59. Ans. is a i.e. Erythromycin Ref. KDT 6/e, p 727

Erythromycin stimulates motilin receptors in the G.I.T. thereby induces gastric contractions, hastens
gastric emptying and promotes intestinal motility.
Mechanism of action of tetracycline and chloramphenicol is explained above question.
Norfloxacin is a fluoroquinolone that acts by inhibiting the enzyme, bacterial DNA gyrase.

60. Ans. is b i.e. IL2 Ref. KDT 6/e, p 837; K K Sharma 1/e, p 913

Cyclosporine is an immunosuppressant which inhibits T lymphocyte proliferation and IL2 cytokine


β production.
production and also stimulates TGF-β

467
6 Self Assessment & Review Phar macology
Pharmacology
CYCLOSPORINE :
Mechanism of Action :

Uses : DOC in organ or tissue transplantation

Adverse effects : • Peripheral neuropathy • Nephrotoxicity


• Hyperglycemia • Hyperlipidemia
• Hirsutism • Gingival overgrowth
• Cholelithiasis

Also see comparison between other immunosuppressant, refer answer no. 23

61. Ans. is d i.e. DEC acts on adults and ivermectin on microfilariae Ref. KDT 6/e, p 812 - 813

Diethylcarbamazine citrate (DEC) Ivermectin


• Acts by sensitizing the microfilaria, so that • GABA agonistic action lead to tonic paralysis
they become susceptible to phagocytosis • No lethal action on adult worms
• Causes rapid disappearance of microfilaria of • Invermectin active against microfilarias in scrotal
W. bancrofti, B malayi from peripheral blood filariasis.
but not the microfilaria in hydrocele • DOC for onchocerciasis.
• Kills the adult worm of W. bancrofti, B. malayi.

468
Antimicr obial & Antineoplastic Dr
Antimicrobial ugs
Drugs 6
62. Ans. is c i.e. Blindness Ref. KDT 6/e, p 827

Cisplatin is a platinum containing antineoplastic drug that is hydrolysed intracellularally to produce a highly
reactive moiety which causes cross links of DNA at site N7 of guanine residues.

Side effects :
• High emetic potential • High renal toxicity : this is most important dose dependent toxicity
• Neuropathy • Tinnitus and deafness - ototoxicity
• Hyperuricemia

Uses : It is very effective in metastatic testicular and ovarian carcinoma, also in other solid tumors.

63. Ans. is b i.e. Procainamide Ref. Harrison 17/e, p 2083

Among the most frequent are the antiarrhythmics procainamide, disopyramide, and propafenone; the
antihypertensive hydralazine; several agniotensin-converring enzyme inhibitors and beta blockers; the antithyroid
prophylthiouracil; the antipsychotics chlorpromazine and lithium; the anticonvulsants carbamazepine and
phenytoin; the antibiotics isoniazid, minocycline, and macrodantin; the antirheumatic sulfasalazine; the diuretic
hydrochlorothiazide; the antihyperlipidemics lovastatin and simvastatin; and interferons and TNF inhibitor.
Both Procainamide and Phenytoin causes SLE like syndrome. But in question it is asked most commonly. So,
the answer is Procainamide.

Please remember the drug causing SLE like syndrome (Mnemonic - Plz Check BOM In SHIP).

Plz Procainamide In Interferon α


Check Chlorpropamide S Sulphonamide
B Barbiturates H Hydralazine
O OCP I INH
M Methyldopa P Phenytoin

64. Ans. is d i.e. Schistosomiasis Ref. KDT 6/e, p 809; Harrison 17/e, p 1334

Albendazole is used in all condition given in options except Schitosomiasis.

All Flukes DOC is praziquantel except Faciola hepatica (liver flukes) for which DOC is Triclabendazole/
Bithional.
Drug of Choice Diseases / Worm
• Mebendazole / Albendazole Ascariasis lumbricoides roundworm
Trichuris trichura whipworm
Trichinella
• Mebendazole / Albendazole Enterobius (Pinworm)
Ankylostoma (Hookworm)
• Ivermectin / Albendazole Strongyloides stercoralis
• Metronidazole Dracunculosis

469
6 Self Assessment & Review Phar macology
Pharmacology
The 3 broad spectrum antihelminthic drugs : Albendazole, Mebendazole, and Thiabendazole.
• Niclosamide is highly effective against Taenia, Diphyllobothrium latum, Hymenolepis nana & Thread worms.
• Niclosamide is highly effective against Schistosomes.
• Pyrantel pamoate is highly effective against Ascaris, Enterobius and Ankylostoma.
• Pyrantel pamoate in safe in pregnancy (Ascariasis).

Drug of choices of All Parasites

Trematodes Cestodes
All flukes Taenia solium - Pork Tapeworm
• Blood flukes - Schistosomias Taenia saginata - Beef Tapeworm
• Liver flukes - Fasciola hepatica Hymenolepis nana - Dwarf Tapeworm
• Oriental flukes - Liver flukes Echinococcus gr.- Dog Tapeworm
DOC of all flukes is Praziquantel DOC of all Cestodes is Praziquantel
Except Fasciola hepatica Except Echinococcus granulosus
DOC is Triclabendazole / Bithional DOC is Albendazole
DOC of neurocysticercosis - Albendazole

Nematodes

• Ascaris • Enterobius (Pinworm) • Strongyloides • Wucheria bancrofti


• Trichuris trichura • Ankylostoma duodenale • Cutaneous larva migrans • Brugia malayi
• Trichinella (Hookworm) • Onchocerca volvulus • Loa loa
Mebendazole / Albendazole Mebendazole/Albendazole Ivermectin Diethyl carbamazine

• Chaga’s disease : Nifurtimox / Benzridazole


• Sleeping sickness : Suramine / Pentamidine

65. Ans. is a i.e. Rifampicin Ref. KDT 6/e, p 741

Rifampicin causes suppression of intestinal flora leading to decrease intrahepatic circulation of OCP. Hence
causes OCP failure.
Drugs causing OCP failure : • Phenytoin • Carbamazepine • Griseofulvin
• Rifampicin • Ampicillin • Tetracycline
• Phenobarbitone • Primadone

66. Ans. is a i.e. G.I. toxicity Ref. KDT 6/e, p 824; Katzung 10/e, p 888

5-FU causes myelosuppression and mucositis. Mucositis explains gastrointestinal toxicity and because GIT
has the largest surface area, it should be the best option.

5-FU (fluorouracil)
• It is pyrimidine antagonist.

470
Antimicr obial & Antineoplastic Dr
Antimicrobial ugs
Drugs 6
• 5-FU is converted in the body to the corresponding nucleotides 5-fluro-2 deoxyuridine
monophosphate, which inhibits thymidylate synthesis, i.e. selective failure of DNA synthesis.

• It is particularly used for many solid tumors : – breast – colon – stomach


– esophagus – liver – urinary bladder
• It causes the Hand Foot Syndrome.

Chemotherapeutic agent Delayed toxicity


Capecitabine Diarrhea, hand & foot syndrome, myelosuppression, nausea & vomiting
Cladribine Myelosuppression, nausea & vomting, & immunosuppression
Cytarabine Nausea & vomiting, bone marrow depression with leukopenia & thrombo-
cytopenia, & are cerebellar ataxia
Fludarabine Myelosuppression, immunosuppression, fever, myalgias, & arthralgias
5-Fluorouracil Nausea, mucositis, diarrhea, bone marrow depression, & neurotoxicity
Gemcitabine Nausea, vomiting, diarrhea, myelosuppression
6-Mercaptopurine Myelosuppression, immunosuppression, & hepatotoxicity
Methotrexate Mucositis, diarrhea, bone marrow depression with leukopenia & thromboctyo-
penia
Methotrexate Mucositis, diarrhea, bone marrow depression with leukopenia & thrombo-
cytopenia
Pemetrexed Myelosuppression, skin rash, mucositis, diarrhea, & fatigue
6-Thioguanine Myelosuppression, immunosuppression, & hepatotoxicty

67. Ans. is d i.e. Cycloserine Ref. KDT 6/e, p 744; Katzung 10/e, p 774

In tuberculosis with liver diseases the drugs which is safe is Streptomycin and Ethambutol, do not give
Pyrazinamide and avoid Rifampin.
Cycloserine : Only 1/3rd is metabolised in liver and rest all excreted in kidney unchanged.
Ethambutol : It is (50%) excreted by kidney.
Apparently both are safe in liver disease but cycloserine is better answer here.

Important points about ATT :


Pregnancy • Do not give Streptomycin – permanent deafness in baby
Renal Failure • Rifampin, Isoniazid, Pyrazinamide are safe.
• Avoid Streptomycin and Ethambutol
• Do not give Thiacetazone

68. Ans. is d i.e. Rapidly develop resistance Ref. KDT 6/e, p 688

Fluoroquinolone resistant mutants are not easily selected hence resistance to Fluoroquinolone has been slow
to develop.
Option ‘b’ • FQs inhibit the enzyme bacterial DNA gyrase which nicks double standard DNA, introduces
negative supercoil and then reseal the nicked ends.
• DNA gyrase : – Subunit A - carries out nicking of DNA, Fluoroquinolone binds to ‘A’ subunit.
– Subunit B - introduces –ve supercoils.

471
6 Self Assessment & Review Phar macology
Pharmacology
Option ‘a’ • In place of DNA gyrase/topoisomerase IV, the mammalian cells possess an enzyme
topoisomerase II which has very low affinity for fluoroquinolone, hence the low toxicity to host
cell.
Option ‘c’ • Ciprofloxacin and levofloxacin have antipseudomonal activity.

69. Ans. is d i.e. Weight gain Ref. KDT 6/e, p 786; Katzung 10/e, p 849

Side effects of Chloroquine are : Nausea, vomiting, anorexia, epigastric pain which may lead to loss of
appetite and weight loss, NOT weight gain.

CHLOROQUINE
Adverse effect : Uses (Mnemonic RED LIP) :
• Hypotension, arrhythmias (T wave changes) R – Rh. arthritis
• High doses – Retinal damage and corneal deposits E – Extraintestinal amoebiasis
• Photoallergy and greying of hairs D – DLE
• Lichenoid eruption L – Lepra reaction
• Convulsions I – Infectious mononucleosis
P – Photogenic reaction

70. Ans. is a i.e. Proguanil Ref. KDT 6/e, p 782

Proguanil, pyrimethamine and primaquine are used in pre-erythrocytic phase liver.

Comparative properties of antimalarial drugs.

Pre-Erythro- Erythroctyic Phase Exo Gametes


cytic stage Erythrocytic
Drug Fal. Viv. Activity Onset Viv. Fal. Viv.
Chloroquine – – + Fast – – +
Mefloquine – – – Intermediate – – –
Quinine – – + Intermediate – – +
Proguanil + + + Slow – * *
Pyrimethamine – – + Slow – * *
Primaquine + + + – + + +
Sulfonamides – – + Slow – – –
Tetracyclines + – + Slow – – –
Artemisinin – – + Faster – + +
Lumefantrine – – + Intermediate – – –

* Do not kill gametes but inhibit their development in mosquito.


Fal. - P.falciparum; Viv - P.vivax

472
Antimicr obial & Antineoplastic Dr
Antimicrobial ugs
Drugs 6
71. Ans. is b i.e. Ethionamide Ref. Katzung 10/e, p 772

Mechanisms of Isoniazid (INH) resistance :


• High level resistance - deletions in katG gene (encodes catalase needed for INH bioactivation) and
often not cross-resistant to ethionamide.
• Low level resistance - deletions in inhA gene (encodes acyl carrier protein, the “target”).

72. Ans. is a i.e. Cefepime Ref. KDT 6/e, p 668

Bacitracin is a polypeptide antibiotic, acts by inhibiting cell wall synthesis and active against gram
+ve organisms.

Drugs inhibiting cell wall synthesis

• Penicillins • Cephalosporins • Cycloserine • Teicoplanin


• Vancomycin • Bacitracin • Monobactum • Carbapenems

73. Ans. is a i.e. Ketoconazole Ref. KDT p 763

Ketoconazole is used for treatment of most serious mycoses. It is useful in indolent non-CNS
blastomycosis and in indolent coccidioidomycosis. It is also effective against oropharyngeal candidiasis
even in patients of AIDS. For this indication, it is given orally.
Ketoconazole is ineffective in the treatment of cryptococcosis, mucormycosis and aspergillosis.

Main adverse effects :


• It markedly inhibits the synthesis of testosterone and estradiol which may lead to gynaecomastia and
irregular menstrual cycles respectively.
• Increase liver function tests and rare hepatotoxicity.

Pharmacokinetics :
• Effective orally
• Absorption of ketoconazole is decreased by antacids
• Absorption of itraconazole is increased by food
• Only fluconazole penetrates into the CSF and can be used in meningeal infection. Fluconazole is eliminated
in the urine, largely in unchanged form.
• Ketoconazole and itraconazole are metabolized by liver enzymes.
• Inhibition of hepatic P450s.

Drug interactions :
• Ketoconazole inhibits mammalian cytochrome P450 (CYP3A4).
• This leads to several drug interactions :
– H2 receptor blockers, proton pump inhibitors and antacids decrease ketoconazole absorption by decreasing
gastric acidity.
– Ketoconazole increases the serum concentrations of cisapride, terfenadine, astemizole and quinidine
leading to fatal ventricular fibrillation and arrhythmias.
– Rifampicin and phenytoin accelerate ketoconazole metabolism and reduce its efficacy.

473
6 Self Assessment & Review Phar macology
Pharmacology
– Ketoconazole raises the plasma concentrations of warfarin, cyclosporine, tacrolimus and HMG-CoA
reductase inhibitors (except pravastatin and fluvastatin).

74. Ans. is c i.e. Actinomycin D Ref. Harrison 17/e, p 2222

Drugs causing SIADH are :

• Vasopressin or DDAVP • Phenothiazines • Chlorpropamide


• Tricyclic antidepressants • Oxytocin, high dose • Monoamine oxidase inhibitors
• Vincristine • Serotonin reuptake inhibitors • Carbamazepine
• Nicotine

75. Ans. is d i.e. GIST Ref. Harrison 17/e, p 502; Katzung 10/e, p 897

Imatinib is indicated for treatment of chronic myelogenous leukemia (CML), a pluripotent hematopoietic stem
cell disorder characterized by the t(9:22) Philadelphia chromosomal translocation and GIST (gastrointestinal
stromal tumors).
Molecular target is Bcr-Abl, c-Abl, c-Kit, PDGFR-α/β.
Mechanism: by blocking ATP binding to tyrosine kinase active site.
Dasatinib and Nilotinib are 2 drugs developed for most imatinib resistant mutant; effective in imatinib resistant
CML.

FDA-Approved Molecularly Targeted Agents for the Treatment of Cancer


Drug Molecular Target Mechanism of Action
All-trans retinoic acid PML-RARα oncogene Inhibits transcriptional repression by the
(ATRA) PML-RARα
Imatinib Bcr-Abl, c-Abl, c-Kit, PDGFR-α/β Blocks ATP binding to tyrosine kinase active
site.
Sunitinib c-Kit, VEGFR-2, PDFGR-β, Flt-3 Inhibits activated c-Kit & PDGFR in GIST;
inhibits VEGFR in RCC
Sorafinib RAF, VEGFR-2, PDGFR-α/β Flt-3, c-Kit Target VEGFR pathways in
RCC. Possible activity against BRAF in
melanoma, colon cancer, & other
Erlotinib EGFR Competitive inhibitor of the ATP binding site
of the EGFR.
Gefitinb EGFR Inhibitor of EGFR tyrosine kinase.
Bortezomib Proteasome Inhibits proteolytic degradation of multiple
cellular proteins

Clinical uses : Chronic myelogenous leukaemia (CML), GIT stromal cell tumour.

Adverse effects : Nausea, vomiting, oedema, muscle cramps, elevation of transaminases, neutropenia,
thrombocytopenia.

474
Antimicr obial & Antineoplastic Dr
Antimicrobial ugs
Drugs 6
76. Ans. is c i.e. Carmustine Ref. Harrison 17/e, p 379

Causes of Neutropenia
Decreased production of neutrophils:
Drug induced – Alkylating agents (carmustine or BCNU, nitrogen mustard, busulfan, chlorambucil,
cyclophosphamide); antimetabolites (methotrexate, 6-mercaptopurine, 5-flucytosine); noncytotoxic agents
[antibiotics (chloramphenicol, penicillin, sulfonamides), phenothiazines, tranquilizers (meprobamate),
anticonvulsants (carbamazepine), antipsychotics (clozapine), certain diuretics, anti-inflammatory agents,
antithyroid drugs.

77. Ans. is c i.e. Ciclopirox olamine Ref. KDT 6/e, p 757, 862 - 864; Harrison 16/e, p 2748 - 2749

Ciclopirox olamine is a miscellaneous antifugal drug.


Other durgs given to treat scabies and discrib below:

Drugs used for Scabies are : • Permethrin • Sulphur • Lindane (BHC)


• Dicophane (DDT) • Benzyl benzoate • Ivermectin
• Crotamiton

• Permethrin is toxic to Pediculus humanus, Pthirus pubis, and Sarcoptes scabiei. Less than 2% of an
applied dose is absorbed percutaneously. Residual drug persists up to 10 days following application.
Permethrin 1% cream to affected areas of pediculosis for 10 minutes and then rinsed off with warm water.
For the treatment of scabies, a single application of 5% cream (Elimite) is applied to and then washed off.
• Lindane (Hexachlorocyclohexane) the current recommendation is for a single application to the entire
body from the neck down, left on for 8-12 hours, and then washed off.
• Crotamiton is a scabicide with some antipruritic properties. Crotamiton (Eurax) is available as a cream or
lotion. Suggested guidelines for scabies treatment call for two applications to the entire body from the chin
down at 24 hours intervals, with a cleansing bath 48 hours after the last application.
• Sulphur formulation is 5% precipitated sulfur in petrolatum.
• Malathion is an organophosphate cholinesterase inhibitor that is hydrolyzed by plasma carboxylesterases
much faster in humans than in insects, thereby providing a therapeutic advantage in treating pediculosis.

78. Ans. is a i.e. Are bacteriostatic Ref. KDT 6/e, p 668 – 669, 719

The tetracyclines are primarily bactericidal antibiotics, acts by inhibiting protein synthesis and active
against aerobic gram negative bacilli.
They are 30S Inhibitors.
Aminoglycosides :
• Inhibits/ freeze initiation complex.
• Interfere with polysomes formation and misreading of m-RNA (1st step in protein synthesis).

79. Ans. is c i.e. Mosapride Ref. KDT 6/e, p 637 - 638; Goodman & Gilman’s 11/e, p 979 - 980

Drug used for H. pylori are :


TRIPLE THERAPY for 14 Days (BD)
PCM : PPI + Clarithromycin 500 mg + Metronidazole 500 mg
PCA : PPI + Clarithromycin 500 mg + Amoxicillin 1 gm

475
6 Self Assessment & Review Phar macology
Pharmacology
QUADRUPLE THERAPY for 14 Days
PMBT : PPI (BD) + Metronidazole 500 mg (TDS) + Bismuth subsalicylate 525 mg
(QID) + Tetracycline 500 mg (QID)
OR
H2 receptor antagonist (BD) + Metronidazole 250 mg (QID) + Bismuth
subsalicylate 525 mg (QID) + Tetracycline 500 mg (QID)

80. Ans. is d i.e. Silver sulfadiazine (SSD) Ref. KDT 6/e, p 684

Silver sulfadiazine is used topically as 1% cream, it is active against a large number of bacteria and fungi.
It slowly releases silver ions which is responsible for the antimicrobial action. It is considered to be the most
effective drug for preventing infection of burnt surfaces and chronic ulcers.

81. Ans. is d i.e. Saquinavir causes maximum inhibition of CYP3A4


Ref. Goodman & Gilman 11/e, p 1297 - 1301

Protease inhibitors - Important facts:


• These drugs are peptide like chemicals that competitively inhibit the action of the virus aspartyl protease.
• They prevent proteolytic cleavage of HIV gag and pol polyproteins that include essential structural (p17, p24,
p9 and p7) and enzymatic (reverse transcriptase, protease, and integrase) components of the virus. This
prevents the metamorphosis of HIV virus particles into their mature infectious form.
• The clearance of all approved HIV protease inhibitors is mainly through hepatic oxidative metabolism.
• All, but one of these drugs are substrates predominately for CYP3A4. Nelfinavir’s major metabolite, M8, is
formed by CYP2C19.
• The elimination half-life range from 1.8 to 10 hours and most of these drugs can be dosed once or twice daily.
• Most HIV protease inhibitors are substrates for the P-glycoprotein drug transporter (P-gp), which is an efflux
pump encoded by the mdr1 gene.
• P-gp in capillary endothelial cells of the blood-brain barrier limits the penetration of these drugs into the
brain.
• The important toxicity common to all the drugs is potential for metabolic drug interactions. Most of these
drugs inhibit CYP3A4 with ritonavir being by far the most potent.
• Ritonavir, nelfinavir, and amprenavir are also moderate inducers of hepatic enzymes including CYP3A4 and
glucuronosyl S-transferase.
• The common practice is to combine HIV protease inhibitors with a low dose of ritonavir to take advantage of
that drug’s remarkable capacity to inhibit CYP3A4 metabolism. Lopinavir is available only in a coformulation
with ritonavir that is designed to take advantage of this beneficial pharmacokinetic drug interaction.
• The speed with which HIV develops resistance to protease inhibitors is intermediate between that of nucleoside
analogs and NNRTIs.
• Resistance to one HIV protease inhibitor occurs often, with the patient retaining sensitivity to other drugs.
• Saquinavir causes minimum inhibition of CYP3A4.

82. Ans. is a i.e. Competetive inhibitor of bcr-abl gene product


Ref. Goodman & Gillman 11/e, p 1367 - 1368; Harrison 17/e, p 502

476
Antimicr obial & Antineoplastic Dr
Antimicrobial ugs
Drugs 6
Imatinib
Mechanism of action : Imatinib has specific action on tyrosine residues only. Imatinib specifically inhibits the
proliferation of myeloid cell lines that express Bcr-Abl fusion protein associated with CML. It also inhibits the
proliferation of GIT stromal tumour cells dependent on c-kit protooncogene which increases tyrosine kinase
activity.
Drug Molecular Target Mechanism of Action

Imatinib Bcr-Abl, c-Abl, c-Kit, PDGFR-α/β Flt-3 Blocks ATP binding to tyrosine kinase active

Clinical use : Chronic myelogenous leukaemia (CML), GIT stromal cell tumour.
Adverse effects : Nausea, vomiting, oedema, muscle cramps, elevation of transaminases, neutropenia,
thrombocytopenia

83. Ans. is a i.e. Methotrexate


Ref. Goodman & Gillman 11/e, p 1316 - 1318; CMDT ‘05 p 1621; Katzung 10/e, p 887

Anti Metabolites

Folate antagonist • Methotrexate (Mtx)


Purine antagonist • 6 - mercaptopurine (6-MP)
• 6 - thioguanine (6-TG)
• Azathioprine, Fludarabine
Pyrimidine antagonist • 5 - Fluorouracil (5-FU)
• Cytarabine

84. Ans. is a i.e. Macrolide antibiotic Ref. Goodman & Gilman 11/e, p 1412 - 1413

Tacrolimus (FK - 506)

• It is a macrolide isolated from soil fungus


MOA • Binds to cyclophilin FKBP - 12

Drug - FKBP complex binds to calcineurin

Inhibit calcinurin PO4– activation of T cell transcription factor

↓ IL - 2, IL - 3 & interform - γ
Indications Liver kidney transplant given with glucocorticoids
Potency 10 -100 times more potent than cyclosporine
Pk Highly bound to serum proteins. Undergoes hepatic metabolism by
CYP3A4 Renal excretion is very low
Adverse effect More toxic than cyclosporine
Nephrotoxicity and Neurotoxicity tremors, seizures, hallucinations.
Anaphylactoid reactions to injection vehicle have been reported.

477
6 Self Assessment & Review Phar macology
Pharmacology
85. Ans. is a i.e. Methicillin Ref. Harrison 16/e, p 618 table 94.3

Drugs causes Severe myelosuppression include : • Carmustine • Cytarabine


• Daunorubicin • Doxorubicin
• Paclitaxel • Alkylating agent
• Antimetabolites

Almost all chemotherapeutic agents, except asparaginase, bleomycin, vincristine, gefitinib and some hormones
and hormone antagonists, cause myelosupression.

Comparison of Myelosuppressive potential of anticancer drugs

86. Ans. is b i.e. Amikacin Ref. Washington, p 656

• Decrease in renal functions leads to a decrease in renal clearance of drugs and leads to an increase in their
t½. Hence t½ of several drugs is markedly prolonged. The reduction in their dose and close monitoring is
essential to avoid their accumulation and toxicity (e.g., drugs like digoxin, lithium, gentamicin, streptomycin,
atenolol, sotalol, imipramine, phenobarbital).
• Adjustment of the maintanance dose involves reducing each dose given, or lengthening the time between
doses. The most convenient and useful guide for the degree of renal impairment is creatinine clearance.
Adjusted dosing interval (hr) ..... Washington, p 656

Medication < 10 ml / min


• Amikacin > 24 hours
• Vancomycin 48 – 96 hours
• Amphotericin B 24 – 36 hours
• Rifampicin Normal

87. Ans. is b i.e. INH Ref. Goodman & Gilman 11/e, p 1207

• A number of mental abnormalities may appear during the use of drug INH, including euphoria, transient
impairment of memory, separation of ideas and reality, loss of self-control, and florid psychoses.

478
Antimicr obial & Antineoplastic Dr
Antimicrobial ugs
Drugs 6
• The incidence of adverse reactions to isoniazid was estimated to be 5.4%.
• The most prominent of these reactions were rash (2%), fever (1.2%), jaundice (0.6%), and peripheral neuritis
(0.2%).

88. Ans. is a i.e. Cefepime Ref. KDT 6/e, p 668

Cefepime, a cephalosporin acts by inhibiting bacterial cell wall.

Mechanism of Action of Antibiotics


Inhibit cell wall synthesis
– Penicillins – Vancomycin
– Cephalosporins – Cycloserine
– Bacitracin – Monobactum
– Carbapenems – Teicoplanin

89. Ans. is b i.e. Letrozole


Ref. Katzung 10/e, p 896; Goodman & Gilman 11/e, p 1385

Aromatase inhibitors are of two types :


• Type I aromatase inhibitor – They cause irreversible inhibition of aromatase e.g. Exemestane.
• Type II aromatase inhibitor – They cause reversible inhibition of aromatase e.g. Anastrazole, Letrozole.

Letrozole It is a nonsteroidal competitive inhibitor of aromatase that is significantly more


potent than aminoglutethimide and acts in the same way as anastrozole. It is also
indicated for first-line treatment of postmenopausal women with hormone receptor-
positive metastatic breast cancer and for second-line treatment of postmenopausal
women with advanced breast cancer after progression on tamoxifen therapy.
Exemestane It is a steroid hormonal suicide inhibitor that binds to and irreversibly inactivates
aromatase.
Anastrozole It is a selective nonsteroidal inhibitor of aromatase that has no inhibitory effect on
adrenal glucocorticoid or mineralocorticoid synthesis.
Aminoglutethimide It is a nonsteroidal inhibitor of corticosteroid synthesis at the first step involving the
conversion of cholesterol to pregnenolone.

90. Ans. is a i.e. Cisplatin Ref. KDT 6/e, p 707

Drugs causing Renal failure :


Drugs Reasons
Aminoglycoside, Cisplatin, Cyclosporine, Acetaminophen, Ethylene Glycol • Acute Tubular Necrosis
COX Inhibitors, ACE-Inhibitors • Renal hypoperfusion
Amphotericin-B • Renal Artery Stenosis
β-lactam, Sulfonamides, Diuretics, Captopril, NSAIDs, Allopurinol • Interstitial Nephritis

479
6 Self Assessment & Review Phar macology
Pharmacology
91. Ans. is c i.e. Cisplatin Ref. Goodman & Gilman 11/e, p 1333 - 1334; Katzung 10/e, p 886

Cisplatin is mutagenic, teratogenic, and carcinogenic. The use of cisplatin - or carboplatin - based chemotherapy
for women with ovarian cancer is associated with a fourfold increased risk of developing secondary leukemia.
Cisplatin has been associated with the development of AML, usually 4 years or more after treatment.

92. Ans. is b i.e. Binding to 30 S subunit and inhibits binding of aminoacyl tRNA
Ref. Lippincott 2/e, p 396; KDT 6/e, p 711

Tetracycline binds to 30s ribosome and inhibit aminoacyl t RNA attachment to the A site.

Mechanism of Action of Antibiotics through inhibting protein synthesis.


• Aminoglycoside binds to 30s, freeze initiation and causes misreading of mRNA code.
• Chloramphenicol binds to 50s ribosome and interfere with peptide bond formation & transfer of peptide
chain from P site.
• Erythromycin & binds to 50s ribosome and hinder translocation of elongated of peptide chain from
Clindamycin A to P site, the ribosome does not move along mRNA to expose the next codon.

93. Ans. is d i.e. Antimetabolite Ref. Harrison 17/e, p 526; Katzung 10/e, p 887

• Pemetrexed is a novel folate-directed antimetabolite. It is “multitargeted” in thatit inhibits the activity of


several enzymes, including thymidylate synthetase, dihydrofolate reductase, and glycinamide ribonucleotide
formyltransferase, thereby affecting the synthesis of both purine and pyrimidine nucleic acid precursors.
• To avoid significant toxicity to the normal tissues, patients receiving pemetrexed should also receive low-
dose folate and vitamin B12 supplementation.
• Pemetrexed has notable activity against certain lung cancers and, in combination with cisplatin, also against
mesotheliomas.
Pemetrexed is new member of antifolate antimetabolite with activity in the S phase of the cycle also.
Used in : • Breast carcinoma • Non small cell lung cancer
• Colon Ca • Pancreatic cancer
Adverse effects : • Myelosuppression • Skin rashes
• Mucositis • Diarrhea
• Fatigue • Neutropenia
• Anemia • Thrombocytopenia

It inhibit 3 enzyms involved in purine and pyrimidine synthesis :


• Thymidylate synthase
• Dihydrofolate reductase
• Glycinamide ribonucleotide formyltransferase.

94. Ans. is b i.e. Amphotericin Ref. Harrison 17/e, p 283; KDT 6/e, p 758 - 759

Although Amphotericin B causes nephrotoxicity, but it does not cause hyperkalemia.

480
Antimicr obial & Antineoplastic Dr
Antimicrobial ugs
Drugs 6
Drug causing
Hyperkalemia • ACE inhibitors • Heparin • Cyclosporine
• K+ sparing diuretic • Pentamidine • NSAIDs
Hypokalemia • Insulin • Tetracycline • Theophyline
• Corticosteroids • Amphotericin B • Penicillin derivatives

Adverse Effects of Amphotericin B

Long Term Toxicity Acute Reaction



Nephrotoxicity Anemia CNS toxicity • On IV
• is does related : • Due to bone • intrathecal injection : – Fever
– Azotemia marrow – Headache – Aches and
– Reduced GFR depression – Vomiting pain
– Renal tubular acidosis • Slowly progressive – Nerve palsies – Nausea
– Hypokalemia anemia hypochromic – Seizures – Vomiting
– Hypomagnesemia normocytic – Arachnoiditis – Dyspnoea
– Inability to concentrate – Chills
urine • Due to release
of cytokines

95. Ans. is a i.e. Mitomycin C Ref. Goodman & Gilman 10/e, p 1431; Harrison 17/e, p 525; KDT 5/e, p 776

Mitomycin C is a potent radiosensitizer and acts preferentially on hypoxic cells.


Mitomycin C :
• It is antibiotic anticancer drug.
• Mechanism of action - It is transformed intracellularly to a form which acts as an alkylating agent and kills
cells in G1-M phase by inhibiting topoisomerase II.
• Indications - Cancer of stomach, cervix, colon, rectum, bladder etc.
• Contraindications - Bone marrow depression.
• Adverse drug reactions - Bone marrow depression with thrombocytopenia, anemia, gastro intestinal tract
toxicity, hemolytic - uremic syndrome.

96. Ans. is d i.e. Imipenem + Amikacin


Ref. KDT 6/e, p 708 - 709; K K Sharma 1/e, p 752; Harrison 17/e, p 954

Strains of P. aeruginosa may develop resistance with carbapenems, so, simultaneously use of
aminoglycoside of any type is recommended.
Imipenem-cilastatin 0.5 g i.v. 6 hourly (max 4 g/day) has proved effective in a wide range of serious hospital
acquired infections including those in neutropenic, cancer and AIDS patients.

Imipenem : It is an extremely potent and very broad spectrum β-lactam antibiotic whose range of activity
includes gram positive cocci, enterobacteriaceae, Pseudomonas and Cl. difficile. It is resistant
to most β-lactamases and inhibits penicillinase producing staphylococci.

481
6 Self Assessment & Review Phar macology
Pharmacology
97. Ans. is a i.e. Ara-C Ref. Harrison 17/e, p 681, 682; Katzung 10/e, p 635, 890

Cytarabine is usually administered as continuous IV infusion.


Typical schedules for administration of ara -C employ bolus doses every 12 hours for 5-7 days or a continuous
infusion for 7 days. ..... Goodman & Gilman 11/e, p 1345
Cytarabine is cell cycle specific and acts primarly during S phase. Its main use is to induce remission in acute
leukemia in children, also in adults. Other uses are Hodgkin’s disease and non Hodgkin’s disease. It is given
1.5-3 mg/lg i.v. BD for 5-10 days also by continuous iv infusion.
In view of cytarabine’s S phase specificity, the drug is highly schedule-dependent and must be given either by
continuous infusion or every 8 - 12 hours for 5 - 7 days.

98. Ans. is b i.e. Tacrolimus


Ref. Goodman & Gilman 11/e, p 1413; Katzung 10/e, p 917

Tacrolimus (FK - 506) Sirolimus/ Rapamicin/mTOR inhibitors


• It is a macrolide isolated from Sirolimus is a macrocyclic lactone
soil fungus produced by streptomyces lygroscopicus
MOA • Binds to cyclophilin FKBP - 12 It inhibits T-lymphocytes activation by
↓ inhibiting IL-2 and other T-cell growth
Drug - FKBP complex binds to factor receptors. Sirolimus immunophillin
calcineurin FKBP-12 does not affect calcineurin
↓ activity. It bind to inhibits a protein kinase,
Inhibit calcineurin PO4– - activation designated mammalian target
of T cell transcription factor

↓ IL - 2, IL - 3 & interferon - γ
Indications Liver kidney transplant given with
glucocorticoids
Potency 10 -100 times more potent than
cyclosporine
Pk Highly bound to serum proteins Extensively metabolized by CYP3A4 uses
undergoes hepatic metabolism by indicated for prophylaxis of organ transplant
CYP3A4 Renal excretion is rejection in combination with a calcineurin
very low inhibitors and glucocorticoids
Adverse effect More toxic than cyclosporine Profound myelosupression, hepatotoxicity,
Nephrotoxicity and Neurotoxicity diarrhoea, hypertriglyceridemia, headache,
tremors, seizures, hallucinations, thrombocytopenia hypo or hyperkalemia,
hyperglycemia, hypertension, GI fever and GI effects. Delayed wound healing
complication

99. Ans. is a i.e. Nausea Ref. Harrison, 17/e, p 681, 682

The most common side effect of chemotherapy is nausea with or without vomiting.

482
Antimicr obial & Antineoplastic Dr
Antimicrobial ugs
Drugs 6
Complications of Chemotherapy :

Acute Late

1. Nausea/vomiting 1. Cardiovascular dysfunction


2. Myelosuppression • Cardiomyopathy
3. Alopecia • CHF
4. Gonadal dysfunction 2. Pulmonary dysfunction
– Pulmonary fibrosis (bleomycin)
3. Renal/bladder dysfunction.
• ↓ renal function • Fanconi like syndrome.
• Cystitis • Bladder cancer
4. Nervous system dysfunction.
• Peripheral neuropathy • Hearing loss
5. Eye -cataract
6. Infertility
7. Raynaud’s phenomena
8. Oral complications - Stomatitis, oral ulcer.
9. Secondary malignancies - It is major cause of death for
those who are cured of cancer.
Features Causative drug
1. Pulmonary fibrosis B – Bleomycin
B – Busulphan
C – Cyclophosphamide
M – Melphalan/Methotrexate/Methysergide
A – Amiodarone/Acyclovir
N – Nitrofurantoin
2. No Pulmonary fibrosis – Doxorubicin, Methyldopa
3. Vomiting/ Nausea – Cisplatin (maximum)
– Mithramycin
– Actinomycin D
– Dacarbazine
– Cyclophosphamide
4. Maximum immunosuppression – Methotrexate
5. Minimum Bone marrow toxicity – Cisplatin
6. Maximum Alopecia – Cyclophosphamide – Dactinomycin
7. Skin pigmentation A – Arsenic - (Rain drop pigmentation)
B – Busulphan
C – Clofazamine/ Chlorpromazine/Cyclophosphamide
D – Dapsone

100. Ans. is a i.e. Mutations at reverse transcriptase


Ref. Goodman & Gilman 11/e, p 1283; Harrison 17/e, p 1194, 1196; KDT 5/e, p 770

• Zidovudine resistance is associated with mutations at reverse transcriptase codons 41, 215 and
219.
• Resistant to AZT occurs by point mutations which alter reverse transcriptase enzyme.
• Mutations accumulate gradually and resistance develops in 1/3 patients after 1 year (Harrison →6
Months) of zidovudine monotherapy.

483
6 Self Assessment & Review Phar macology
Pharmacology

α (TNF - α
101. Ans. is a i.e. Tumour necrosis factor -α α) Ref. CMDT ‘05, p 1617, 780; Katzung 10/e, p 589

TNF - α BLOCKING DRUGS • ADALIMUMAB • INFLIXIMAB • ETANERCEPT


INFLIXIMAB

Mechanism of action : – It is a chimeric (25% mouse, 75% human) monoclonal antibody that binds
with high affinity to soluble and possibly membrane-bound TNF-α.
– It complexes with soluble TNF-α and prevents its interaction with p55 and
p75 cell surface receptors. This results in down-regulation of macrophage
and T cell function.
Indications : – Rheumatoid arthritis (MC indications) – Ulcerative colitis
– Psoriasis – Psoriatic arthritis
– Juvenile chronic arthritis – Wegener’s granulomatosis
– Giant cell arteritis – Sarcoidosis
Adverse Effects : – Upper respiratory tract infections – Nausea
– Sinusitis – Headache – Rash, Cough

As a potent macrophage inhibitor, infliximab can be associated with activation of latent tuberculosis,
and screening for latent tuberculosis is recommended prior to starting this therapy.
• These are upcoming drugs, so a lot of questions are expected to be asked from them in forthcoming
PGMEEs.
Antibody Antagonist Receptor Use Type
Cytokines & Cytokine Inhibitors to Inhibits Immune Responses and Inflammation
Infliximab TNFα – Rheumatoid arthritis Chimeric
Crohn’s colitis
Adalimumab TNFα – Rheumatoid arthritis Recombinant
Eternacept (Recombinant TNFα – Juvenile R.A. Humanized
combination of TNF – Ig)
Anakinra IL-1 α & β – Rheumatoid arthritis Recombinant
(Competitive Inhibitor)
Monoclonal Antibodies or Toxins Against T or B Cells
Omalizumab IgE – Allergic Reaction Humanized
Trastuzumab (Herceptin) HER-2/neu – Breast Cancer
Cetuximab EGFR – Metastatic Colorectal Ca Humanized
Bevacizumab VEGF – Metastatic Colorectal Ca
(First Antiangiogenic Agent)
Muromab (OKT3) CD - 3 (T-cell) – Acute graft rejection
Rituximab CD - 20 (anti B cell) – Lymphoma, Vasculitis Humanized
Alemtuzumab (H) CD - 52 – CLL Humanized
Daclizumab CD - 25 – Ulcerative colitis Humanized

484
Antimicr obial & Antineoplastic Dr
Antimicrobial ugs
Drugs 6
102. Ans. is c i.e. Both Necrosis and apoptosis Ref. Harrison 16/e, p 469

Cellular mechanism of Cell death

The majority of chemotherapeutic drugs can be divided in to alkylating agents, antimetabolites, anthracyclines,
plant alkaloids, topoisomerase inhibitors, and other antitumour agents. All of these drugs affect cell division or
DNA synthesis and function in some way.

485
6 Self Assessment & Review Phar macology
Pharmacology
Some newer agents don’t directly interfere with DNA. These include monoclonal antibodies and the new tyrosine
kinase inhibitors e.g. imatinib mesylate (Gleevec or Glivec), which directly targets a molecular abnormality in
certain types of cancer (chronic myelogenous leukemia, gastrointestinal stromal tumors).
In addition, some drugs may be used which modulate tumor cell behaviour without directly attacking those
cells. Hormone treatments fall into this category of adjuvant therapies.
Major categories.
Alkylating agents
Alkylating antineoplastic agent : Alkylating agents are so named because of their ability to add alkyl groups
to many electronegative groups under conditions present in cells. Cisplatin and carboplatin, as well as oxaliplatin
are alkylating agents.
Other agents are mechlorethamine, cyclophosphamide, chlorambucil. They work by chemically modifying a
cell’s DNA.

Anti-metabolites : Anti-metabolites masquerade as purine ((azathioprine, mercaptopurine)) or pyrimidine -


which become the building blocks of DNA. They prevent these substances becoming incorporated in to DNA
during the “S” phase (of the cell cycle), stopping normal development and division. They also affect RNA
synthesis. Due to their efficiency, these drugs are the most widely used cytostatics.

Plant alkaloids and terpenoids : These alkaloids are derived from plants and block cell division by preventing
microtubule function. Microtubules are vital for cell division and without them it can not occur. The main examples
are vinca alkaloids and taxanes.

Vinca alkaloids : Vinca alkaloids bind to specific sites on tubulin, inhibiting the assembly of tubulin into
microtubules (M phase of the cell cycle). They are derived from the Madagascar periwinkle, Catharanthus
roseus (formerly known as Vinca rosea). The vinca alkaloids include:
• Vincristine • Vinblastine
• Vinorelbine • Vindesine
Podophyllotoxin : Podophyllotoxin is a plant-derived compound used to produce two other cytostatic drugs,
etoposide and teniposide. They prevent the cell from entering the G1 phase (the start of DNA replication) and
the replication of DNA (the S phase). The exact mechanism of its action is not yet known.
Taxanes : The prototype taxane is the natural product paclitaxel, originally known as Taxol and first derived
from the bark of the Pacific Yew tree. Docetaxel is a semi-synthetic analogue of paclitaxel. Taxanes enhance
stability of microtubules, preventing the separation of chromosomes during anaphase.
Topoisomerase inhibitors
Topoisomerases are essential enzymes that maintain the topology of DNA. Inhibition of type I or type II
topoisomerases interferes with both transcription and replication of DNA by upsetting proper DNA supercoiling.
• Some type I topoisomerase inhibitors include camptothecins: irinotecan and topotecan.
• Examples of type II inhibitors include amsacrine, etoposide, etoposide phosphate, and teniposide. These
are semisynthetic derivatives of epipodophyllotoxins, alkaloids naturally occurring in the root of American
Mayapple (Podophyllum peltatum).
Antitumour antibiotics

Antineoplastic
The most important immunosuppressant from this group is dactinomycin, which is used in kidney transplantations.
Monoclonal antibodies : Monoclonal antibodies work by targeting tumour specific antigens, thus enhancing
the host’s immune response to tumour cells to which the agent attaches it self. Examples are trastuzumab
(Herceptin), cetuximab, and rituximab (Rituxan or Mabthera). Bevacizumab (Avastin) is a monoclonal antibody
that does not directly attack tumor cells but instead blocks the formation of new tumor vessels.

486
Antimicr obial & Antineoplastic Dr
Antimicrobial ugs
Drugs 6
Hormonal therapy : Several malignancies respond to hormonal therapy. Strictly speaking, this is not
chemotherapy. Cancer arising from certain tissues, including the mammary and prostate glands, may be inhibited
or stimulated by appropriate changes in hormone balance.
• Steroids (often dexamethasone) can inhibit tumour growth or the associated edema (tissue swelling), and
may cause regression of lymph node malignancies. Dexamethasone is also an antiemetic, so it may be
used with cytotoxic chemotherapy even if it has no direct effect on the cancer.
• Prostate cancer is often sensitive to finasteride, an agent that blocks the peripheral conversion of testosterone
to dihydrotestosterone.
• Breast cancer cells often highly express the estrogen and/or progesterone receptor. Inhibiting the production
(with aromatase inhibitors) or action (with tamoxifen) of these hormones can often be used as an adjunct to
therapy.
• Gonadotropin-releasing hormone agonists (GnRH), such as goserelin possess a paradoxical negative
feedback effect followed by inhibition of the release of FSH (follicle-stimulating hormone) and LH (luteinizing
hormone), when given continuously.

103. Ans. is d i.e. Ceftriaxone Ref. Harrison 17/e, p 657

Drugs that carry risk of clinical hemolysis in persons with G6PD deficiency.
Definite risk Possible risk Doubtful risk
Antimalarials Primaquine Chloroquine Quinine
Dapsone/chlorproguanil
Sulphonamides/sulphones Sulphametoxazole Sulfasalazine Sulfisoxazole
Others Sulfadimidine Sulfadiazine
Dapsone
Antibacterial/antibiotics Cotrimoxazole Ciprofloxacin Chloramphenicol
Nalidixic acid Norfloxacin p-Aminosalicylic acid
Nitrofurantoin
Niridazole
Antipyretic/analgesics Acentanilide Acetylsalicylicacid Acetylsalicylic acid <3 g/d
Phenazopyridine high dose (3 g/d) Acetaminophen
(Pyridium) Phenacetin
Other Naphthalene Vitamin K analogues Doxorubicin
Methylene blue Ascorbic acid >1g Probenecid
Rasburicase

104. Ans. is c i.e. Pyrimethamine Ref. KDT 6/e, p 757; Rang & Dale 5/e, p 675

Erythrocytic schizontocides

Fast Acting Medium Acting Slow Acting

Artemisin (Fastest), Halofantrine, Mefloquine, Quinine Pyrimethamine, Proguanil,


Chloroquine, Mepacrine Sulfonamide, Tetracycline

For more details, refer answer no. 70

487
6 Self Assessment & Review Phar macology
Pharmacology
85. Ans. is b i.e. Erythema nodosum leprosum Ref. Harrison 17/e, p 1026; Katzung 10/e, p 918 - 919

• Thalidomide inhibits angiogenesis and has anti-inflammatory and immunomodulatory effects. It inhibits TNFα
reduces phagocytosis by neutrophils, increases production of IL-10, alters adhesion molecule expression,
and enhances cell-mediated immunity via interactions with T cells. The complex actions of thalidomide
continue to be studied as its clinical use evolves.
• Clinical trials in other disease as well as in solid tumors like colon cancer, renal cell carcinoma, melanoma
and prostate cancer, with variable results to date.

Thalidomide is used in severe ENL (Type 2 Lepra reaction).


Other uses of Thalidomide :

• Ankylosing Spondylitis → Harrison 16/e, p 1995


• Behcet’s syndrome → Harrison 17/e, p 2132
• Crohn’s disease → Harrison 16/e, p 1787
• Multiple myeloma → Harrison 17/e, p 675
• Systemic sclerosis → Harrison 16/e, p 1987
• Inflammatory bowel disease → Harrison 17/e, p 1897
• Amylodosis → Harrison 17/e, p 2148

Treatment of ENL or Type 2 Lepra Reaction

Severity Treatment

a. If ENL is mild • Antipyretics only


(no fever, no organ involvement,
occasional crops of only a few skin papules)
b. If ENL is severe • Brief courses (1-2 wks) of glucocorticoids
(Fever, organ involvement, many (initially 40-60mg/d)
skin lesions) • If despite 2 courses of glucocorticoids ENL is
recurring and persisting
↓ then
THALIDOMIDE (100-300mg nightly)
↓ After ENL is controlled
THALIDOMIDE (50-200mg nightly)
• CLOFAZAMINE in high doses (300mg nightly)
has some efficacy against ENL

106. Ans. is c i.e. Cefepime Ref. KDT 6/e, p 707

Only cefepime and cefpirome are 4th generation cephalosporins.

Cephalosporins are classified as :


GENERATION PARENTERAL ORAL REMARKS
FIRST • Cephalothin • Cephalexin • High activity against Gram+ve bacteria
• Cefazolin • Cephradine • Weak activity against Gram-ve bacteria
• Cefadroxil

488
Antimicr obial & Antineoplastic Dr
Antimicrobial ugs
Drugs 6
SECOND • Cefuroxime • Cefaclor • High activity against Gram+ve bacteria
• High activity against Gram-ve bacteria
• Cefoxitin • Cefuroxime- • Some members active against anaerobes
• Cefotetan axetil
THIRD • Cefotaxime • Cefixime • Highly augmented activity against Gram –ve
enterobacteriaceae
• Ceftizoxime • Cefpodoxime- • Weak activity against Gram+ve bacteria.
proxetil • All are highly resistant to β -lactamases from
Gram –ve bacteria
• Ceftriaxone • Cefdinir • Some active against Pseudomonas e.g.,
Ceftazidime, Cefoperazone
• Ceftazidime • Ceftibuten
• Cefoperazone
FOURTH • Cefepime • Spectrum similar to 3rd generation except :
• Cefpirome a. It is more resistant to chromosomal β -
lactamases(e.g. produced by enterobacter)
and some extended spectrum β
lactamases that destroy 3rd generation
cephalosporins.
b. It has good activity against Ps. aeruginosa,
Enterobacteriaceae, S. aureus,
S. pneumoniae, Hemophilus, Neisseria,
Penicillin resistant strains of Streptococci.

Adverse Effects :

Cephalosporins are more toxic than penicillins.


• Pain : – Most severe with cephalothin i.m.
– Thrombophlebitis of vein can occur in i.v. injection.
• Diarrhoea : – More common with
a. Oral cephradine
b. Parenteral cefoperazone (it is significantly excreted in bile)
• Hypersensitivity reactions – Similar to penicillins
– Rashes are most frequent
– 10% patient allergic to pencillins show cross reactivity with cephalo-
sporins.
– Skin test for sensitivity are unreliable.
• Nephrotoxicity – Cephaloridine – high grade (so, withdrawn from market)
– Cephalothin – low grade
• Bleeding –
Occurs with cephalosporins having methylthiotetrazole substitution at
position 3. (e.g. - Cefoperazone - Ceftriaxone
- Cefotetan - Cefamondole
Bleeding occurs due to hypoprothrombinemia by warfarin mechanism.

More common in patients with cancer, renal failure or intrabdominal infections.

– In contrast - Moxalactam ⎫
- Carbenicillin ⎬ are penicillins
- Ticarcillin ⎭

489
6 Self Assessment & Review Phar macology
Pharmacology


α − Carboxyl substitution at position 7

Alter surface platelet receptors

Prolong bleeding time

• Neutropenia and Thrombocytopenia : Mostly occurs with Ceftazidime


• Disulfiram like reaction with Alcohol : Occurs with Cefoperazone

107. Ans. is b i.e. Tetracycline Ref. Harrison 17/e, p 47, 105, 188

Drugs causing Pseudotumour cerebri OR Benign intracranial hypertension.

Menomnics : ASHOKA-QT A Amiodarone


S Steroids (Withdrawl)
H Hypervitaminosis - A
O Oral Contraceptives
K Ketamine
A Alcohol
Q Quinolones
T Tetracyclines
(Alphabet ‘A’ for Amiodarone and alcohol not Aminoglycosides as may be provided in option).
Pseudotumor Cerebri

Clinical Features • Headache


• Diplopia and other visual disturbance due to papilledema
• Abducens nerve dysfunction
Investigation • CT and MRI show no abnormalities, normal ventricles
• CSF is normal
• Lumbar puncture reveals increased intracranial pressure.
Treatment • Acetazolamide reduces formation of CSF & the treatment is usually started with this
drug.
• Sometimes oral corticosteroids are necessary.
• Obese patients should be advised to lose weight.
• Repeated lumber puncture to lower CSF.
• If these treatment fail, surgical placement of lumboperitoneal or other shunts.

Untreated Pseudotumor cerebri leads to secondary optic atrophy and permanent visual loss, so
treatment is monitored by checking visual acuity or fundoscopic appearance.

108. Ans. is a i.e. Hypokalemia Ref. KDT 6/e, p 759

Drugs causing Hypokalemia :

• Insulin • Tetracycline • Theophyline


• Carticosteroids • Amphotericin B • Penicillin derivatives

490
Antimicr obial & Antineoplastic Dr
Antimicrobial ugs
Drugs 6
109. Ans. is b i.e. Dactinomycin Ref. KDT 6/e, p 775

Here, patient has developed toxicity of radiotherapy due to taking a radiosensitiser, therefore dose of
drug should be decreased during radiotherapy.
Radiosensitisers
• Anticancers – Actinomycin D / Dactinomycin – Bleomycin – Cisplatin
– Doxorubicin – Fludarabine – Gemcitabine
– Hydroxyurea – Paclitaxel – Mitomycin C
– Topotecan – Vinorelbine
• Nitroimidazoles – Metro / Misonidazole

Please recheck this :


Anticancer drugs Toxicity Protective agents
Anthracycline Cardiotoxicity Dexrazoxane
Cisplatin Nephrotoxicity Amifostine
Cisplatin 8th nerve damage Glutathione
Emesis Ondonsetron
Cardiotoxicity Dexrazoxane
Cyclophosphamide Hemmorrhagic cystitis Mesna
Methotrexate Bone marrow depression Folinic acid

Radio-mimetic : • Busulphan • Cisplatin • Nitrosourea


Radio-protective : • Amifostine • IL - 1 • GM-CSF

110. Ans. is b i.e. Cefaclor Ref. KDT 6/e, p 838; Katzung 9/e, p 949

Cefaclor is cephalosporin, it will not suppress immune system.


Immunosuppressive drugs :

Specific T-cell inhibitors ( = Calcineurin inhibitors) Cytotoxic drugs (=anti proliferative drugs)
a. Cyclosporine a. Azathioprine
b. Tacrolimus (= FK 506) b. Chlorambucil
c. Sirolimus (= Rapamycin) c. Cyclophosphamide
d. Methotrexate
e. Mycophenolate mofetil (MMF)
f. Leflunomide

Glucocorticoids Antibodies
a. Prednisolone and others a. Muromonab CD3 (=OKT3)
b. Antithymocyte globulin
c. Rho(D) immunoglobulin

491
6 Self Assessment & Review Phar macology
Pharmacology

Uses of Immunosuppressants :
• Organ transplantation (Liver, Kidney, Heart)
• Autoimmune disease (ITP, AC GN, SLE, RA, WG.........)
• Isoimmune disease (Hemolytic disease of newborn)
• To suppress graft vs. host disease in bone marrow transplants.

For more details, refer answer no. 23

111. Ans. is c i.e. Non-nucleoside reverse transcriptase inhibitors Ref. KDT 6/e, p 767

Non-nucleoside revese transcriptase inhibitors

Drugs Side Effects

• Nevirapine Skin rash, abnormal liver function test


• Delaviridine Skin rash, abnormal LFT
• Efavirenz Rash dysphoria, elevated LFT

For more details, refer answer no. 22

112. Ans. is c i.e. Trimethoprim - sulphamethoxazole Ref. Harrison 17/e, p 897; 16/e, p 854, 2476

• Clinical data indicate that ampicillin is the drug of choice. Adults should receive IV ampicillin at
high doses (2 g every 4 h), and most experts recommend the addition of gentamicin for synergy
(1.0- 1.7 mg/kg every 8 h). TMP-SMX, given IV, is the best alternative for the penicillin allergic patient
(15-20 mg of TMP / kg per day in divided dsoes every 6-8 h). The dosages recommended cover CNS
infection and bacteremia for duration); dosages must be reduced for patients with renal insufficiency.
• Case reports document success with vancomycin, tetracycline, and erythromycin, although there are also
reports of clinical failure with all three agents.
• Cephalosporins are not effective and should not be used. Neonates should receive ampicillin and gentamicin
at doses based on weight.

Antimicrobial therapy of CNS bacterial infections

Pathogens Antibiotic preferred Resistant case

Listeria monocytogenes Ampicillin ± Gentamycin Penicillin Resistant / Allergic-Cotrimoxazole


Neisseria meningitides Penicillin G or ampicillin Penicillin resistant ceftriaxone or cefotaxime
Streptococci pneumonia Penicillin G Penicillin intermediate
Cetriaxone or cefotaxime
Penicillin
Ceftriaxone or cefotaxime + Vancomycin
Staphylococcal sp. Nafcillin Methicillin - Vancomycin
Gram -ve bacilli Ceftriaxone or cefotoxime
(except Pseudomonas)
Pseudomonas Ceftazidime
Haemophilus influenzae Ceftriaxone or cefotaxime

492
Antimicr obial & Antineoplastic Dr
Antimicrobial ugs
Drugs 6
Streptococcal agalactiae Penicillin G / ampicillin
Bacteroides fragilis Metronidazole
Fusobacterium sp. Metronidazole

113. Ans. is c i.e. It does not cause alopeica


Ref. KDT 6/e, p 825; Goodman & Gilman 11/e, p 1351 - 1352; Katzung 10/e, p 891

Vincristine
• Vinca alkaloid of the Madagascar periwinkle plant, vinca rosae. Option ‘a’
• Cell cycle specific anticancer drug. It binds to microtubular protein tubulin, prevents its polymerization and
assembly of microtubules, cause disruption of mitotic spindles and interfere with cytoskeletal function.
• Mitotic phase arrest at metaphase.
• Functions as a mitotic spindle poison leading to arrest of cells in the M phase of the cell cycle.
• Dissolution of mitotic spindle.
• Interference with chromosome segregation.
• Both normal and malignant cell exposed to vinca alkaloid undergo apoptosis.
• P- glycoprotein (membrane efflux pump) production on malignant cell leads to resistance to vinca alkaloid.
• Verapamil reverse this resistance.

Side effect Use

• Mostly neurological, • ALL in children


motor neuropathy • Hodgkin’s lymphoma
• Alopecia Option ‘c’ • Non Hodgkin’s lymphoma
• Myelosuppression • Pediatric solid tumor :
• Severe constipation – Wilm’s tumor
• SIADH – Neuroblastoma
– Rhabdomyosarcoma
– Ewing’s sarcoma

114. Ans. is a i.e. ‘Flu like syndrome’ is usually seen in people taking rifampicin on daily basis
Ref. KDT 6/e, p 740; Katzuntg 10/e, p 774

If administered less often then twice weekly rifampin causes a flu-like syndrome characterised by fever
chills, myalgias, anemia and thrombocytopenia.
Flu like syndrome seen in people taking rifampicin on alternate day regimen, not on daily basis.

Important features of different antitubercular drugs

Drug Mechanisms of Action and Resistance Adverse effects

Isoniazid Inhibit mycolic acid synthesis Hepatitis, Peripheral neuritis, Hemolysis


High level resistance – deletion in Cat K in G6PD deficiency, SLE in slow
gene (codes for catalase) acetylators
Rifampin Inhibits DNA dependent RNA polymerase Proteinuria, Hepatitis, Flu-like syndrome,
Resistance due to mutation in repo B gene Red orange urine, Thrombocytopenia

493
6 Self Assessment & Review Phar macology
Pharmacology

Ethambutol Inhibits synthesis of arabinogalactan Dose dependent retrobulbar neuritis,


(cell wall component) ↓visual acuity and Red-green blindness
Pyrazinamide Action similar to that of INH Polyarthralgia, myalgia, Hepatitis & Rash
Resistance due to mutation in the gene Hyperuricemia, Phototoxicity,
which encodes for enzyme generation in the ↑ Porphyrin synthesis
the active metabolite.
Streptomycin Protein synthesis inhibitor Deafness, Vestibular dysfunction and
Nephrotoxicity

115. Ans. is a i.e. Azathioprine Ref. KDT 6/e, p 840; Katzung 10/e, p 918 - 923

• Drugs used in renal (organ) transplant are immunosuppressive drugs and out of all drugs in the
options, only nucleotide having immunosuppressive property is azathioprine.
• Other immunosuppressive drugs used in organ transplantation are :

– Glucocorticoids – Tacrolimus
– Immunophilin ligand - Sirolimus – Mycophenolate mofetil
– Cyclosporine – Thalidamide

Cytotoxic agents : Azathioprine, cyclophosphamide, leflunomide, hydroxychloroquine, others vincristin,


methotrexate, cytrabine.
Immunosuppressive antibodies : Monoclonal antibodies (MABS).

Maintenance Immunosuppressive Drugs


AGENT PHARMACOLOGY MECHANISMS SIDE EFFECTS
Glucocorticoids Increased bioavailability Binds cytosolic receptors – Hypertension
with hypoalbuminemia and and heat shock proteins. – Glucose intolerance
liver disease, prednisone/ Blocks transcription of IL- – Osteoporosis
prednisolone generally used. 1, 2,3, 6, TNF -α and IFN-γ – Dyslipidaemia
– Osteoporosis
Cyclosporine Lipid soluble polypeptide, Trimolecular complex with – Nehprotoxicity
variable absorption, cyclophilin and calcineurin – Hypertension
microemulsion more ↓ – Hirsuitism
predicatable block in cytokine (IL-2) – Dyslipidemia
production; however – Glucose intolerence
stimulates TGF-β – Gum hyperplasia
production – Tremors
Tacrolimus Macrodide antibiotic, Trimolecular complex with – Similar to cyclosproine
well absorbed FKBP-12 and Calcineurin but less likely to cause
↓ hirsuitism,
Block in cytokine (IL-2) gumhyperplasia
production; may – More likely to
stimulate TGF-β precipitate diabetes
production
Azathioprine Mercaptopurine Hepatic metabolities inhibit – Marrow suppression
analogue purine synthesis (WBC > RBC >
Platelets)

494
Antimicr obial & Antineoplastic Dr
Antimicrobial ugs
Drugs 6
Mycophenolate Metabolised to Inhibits purine synthesis – Diarrhoea/Cramps
mofetil mycophenolic acid via IMP dehydrogenase – Liver and marrow
(MMF) suppression is
Sirolimus Macrolide, poor oral Complexes with FKBP-12 – Hyperlipidema
bioavailablility and then blocks p 7056 – Thrombocytopenia
kinease in the IL-2 receptor
pathway for porlfieration

116. Ans. is d i.e. Cyclosphosphamide Ref. KDT 6/e, p 822; Katzung 10/e, p 585

Haemorrhagic cystitis caused by cyclophosphamide is blocked by systemically administration of mesna


and by irrigating the bladder with acetylcyteine.
Cyclophosphamide causes significant dose related infertility in both men and women as well as bone marrow
suppression. Other side effects are alopesia, hemorrhagic cystitis and rarely bladder cancer.

117. Ans. is a i.e. Anthracyline Ref. KDT 6/e, p 826; Katzung 10/e, p 893

• Anthracyclines are anticancer antibiotics and include : – Doxorubicin


– Daunorubicin
• The major toxicity of anthracyclines is potentially irreversible dose related cardiotoxicity.

Since, the patient is already having a history of MI and CHF, so this group of drugs is absolutely contra
indicated.

Anthracylines :
• They cause breaks in DNA strands and function by : – Inhibiting Topoisomerase II, and
– Generating quinone type free radicals
– Max - activity is exerted at → S - Phase
– Toxicity exhibited at → G-II - Phase

• Cardiotoxicity is increased when given together with Trastuzumab (Herceptin), the anti-HER2/neu antibody.
• The other main dose limiting toxicity of all anthracyline is myelosuppression, with neutropenia more common
than thrombocytopenia.
• Other adverse effects are alopecia, stomatitis and mucositis.

Uses : Doxorubicin :

• Ca breast; Ca endometrium; Ca ovary; Ca testicle; Ca thyroid; Ca lung; Ca bladder


• Soft tissue sarcoma; Hodgkin’s lymphoma; Non-Hodgkin’s lymphoma
• Childhood tumours : – Neuroblastoma – Ewing’s sarcoma
– Osteosarcoma – Rhabdomyosarcoma

Daunorubicin :
• Acute leukemias

495
6 Self Assessment & Review Phar macology
Pharmacology
118. Ans. is a i.e. Vitamin - A Ref. KDT 6/e, p 854; Goodman & Gilman 11/e, p 1779

• Synthetic Vit A (Isotretinoin) - if exposed to foetus leads to birth defects - craniofacial, heart and CNS
abnormalities (Accutane embryopathy).
• Congenital abnormalities can occur in infants whose mothers have consumed about 7.5 to 12 mg of retinol
daily during the first trimester of pregnancy.
• Moreover, women who have been treated with synthetic retinoids that accumulate in fat should
practice contraception after discounting therapy until the drug has been eliminated from the body.

119. Ans. is c i.e. L- Asparaginase Ref. KDT 6/e, p 825, 827 - 828

Drugs causing Peripheral Neuropathy

Antitubercular Antibiotic Anticancer Anti hypertensive Antimalaria Others


Ethambutol Nalidixic acid Cisplatin Glutathimide Chloroquine Phenytoin
Isoniazid Nitrofurantoin Vincristine Hydralazine Amiadarone
Streptomycin Chloramphenicol Procarbazine Tricyclic
Ethionamide Metronidazole antidepressant

120. Ans. is b i.e. Amphoterecin B Ref. KDT 6/e, p 758

Mechanism of Action
Amphotericin B binds to the ergosterol, a fungal cell membrane sterol and alters the permeability of the fungal
cell by froming pores (channels) in the fungal cell membrane through which K+, Na+, Mg2+, H+ and other
macromolecules leak out leading to cell death.

Amphotericin B :
– It is a polyene antifungal derived from Streptomyces nodosus.

Mechanism of action :
It has high affinity for ergosterol present in fungal cell membrane.

Combine with it

Get inserted into the membrane

Several molecules together, orient themselves in such a way, so as to form a ‘micropore’.

Cell death
– It is one of the most toxic systemically used antibiotic although it is the least toxic polyene.
– It is not absorbed, so not used orally except for intestinal candidiasis. It is given i.v.

121. Ans. is a i.e. Antimalarial Ref. Rang & Dale 5/e, p 682

“Several new drugs are currently under test for antimalarial activity with positive results in animals and
in prelimnary trials in humans.”

496
Antimicr obial & Antineoplastic Dr
Antimicrobial ugs
Drugs 6
• One of these, Pyronardine, has been used in China for almost 10 years.
• It is active against P. falciparum and P. vivax and is also active in chloroquine resistant P. falciparum.
• It is effective orally and has low toxicity.

122. Ans. is b i.e. Rifampicin Ref. Harrison 17/e, p 1033 - 1132; Katzung 10/e, p 777, 802

• In place of rifampicin and rifapentine, we use rifabutin because it is a less potent inducer inHIV infected
patient who are receiving concurrent antiritrovial therapy with a protease inhibitor or non nucleosive reverse
transciptase inhibitor - drug that also are cytochrome P450 substrate.
• The most important of the pharmacokinetic complications result from the metabolism of the NNRTI and PI
agents by the CYP450 enzyme system, or inhibitors of CYP3A4 as well as substrates, drug-drug interactions
may have marked clinical ramifications.
• In the treatment of tuberculosis, the use of rifampin, a standard antimycobacterial agent but also one of the
most potent 3A4 inducers, may either decrease efficacy (eg. atazanavir, lopinavir) or increase toxicity (eg.
saquinavir) or concurrent antiretroviral agents, owing to alteration of serum levels.

Drugs Interacting Drug Mechanism / Effect

Indinavir, Saquinavir, Rifampin Induction of metabolism-marked


Ritonavir decrease in protease inhibitor drug levels
So, concomitant use should be avoided
Zalcitabine, Stavudine, Drugs that cause peripheral Potential for increased risk of peripheral
Didanosine Neuropathy : Neuropathy
INH, Zalcitabine, Didanosine,
Stavudine
Zidovudine Drugs that cause bone marrow Increased bone marrow suppression
suppression-TMP/SMX, ganciclovir
Ritonavir Delavirdine Inhibition of metabolism -70% ↑ ↑ in
ritonavir levels

There is also a huge list of drug interactions which you need not remember. The above mentioned
are the important ones.

123. Ans. is c i.e. Rifampicin Ref. Harrison 15/e, p 1018 (table 168-1); Katzung 10/e, p 773

Important Points about ATT :

Liver Disease • Safe – Streptomycin and Ethambutol


• Do not give pyrazinamide
• Avoid – Rifampin
Renal Failure • Rifampin, Isoniazid, Pyrazinamide are safe.
• Avoid Streptomycin and Ethambutol
• Do not give Thiacetazone
Pregnancy • Do not give Streptomycin – permanent deafness in baby

497
6 Self Assessment & Review Phar macology
Pharmacology
This question has been repeated several times. Its advised to go through tables in CMDT and in
Harrison mentioned above in reference.

• Most nephrotoxic aminoglycoside → Gentamicin


• Least nephrotoxic aminoglycoside → Tobramycin
• Only tetracycline safe in renal failure → Doxycycline
• Least nephrotoxic antitubercular → Rifampicin

124. Ans. is d i.e. Amphotericin B Ref. Harrison 17/e, p 1243, 1253; Katzung 10/e, p 782 - 783

• Amphotericin B has broad spectrum fungicidal action.


• Used in life-threatening mycotic infections.
• Induction therapy is especially important for immunosuppressed patients and those with severe fungal
pneumonia, cryptococcal meningitis with altered mental status, or sepsis syndrome due to fungal infection.

Treatment of Cryptococcosis :

Type of Disease First Choice Treatment Alternative Treatment

• Disease in – Amphotericin B → 2 weeks – Amphotericin B → 2 weeks


AIDS Patient (0.7-1mg/kg daily) and until the (0.7-1mg/kg/d) then
clinical condition is stable, then
– Fluconazole → 8 weeks – Itraconazole → 8 weeks
(400mg/d) then (400mg/d) then
– Fluconazole → for lifelong – Itraconazole → for lifelong
(200mg/d) (200mg/d)
• Disease in Non
AIDS Patient
1. Meningitis – Amphotericin B → 10 Weeks – Switch to Fluconazole
(0.6-0.7mg/kg/d) (400mg/d) when patients
(Drug of choice) condition has improved
– Flucytosine may be added ↓
but dose should be kept < 100 μg/ml 6-12 months
2. Pulmonary – Immunosuppressed →Same
Disease patients treatment as
for meningitis
– Normal people → Fluconazole – Itraconazole
→ 6 - 12 months (400mg/d) (400mg/d) → 6-12 months

So, Amphotericin B is the treatment of choice in all forms of cryptococcal infections.

125. Ans. is c i.e. Bleomycin Ref. Harrison 17/e, p 92

• This question has been repeated many times before and the two most common drugs causing pulmonary
fibrosis are : 2Bs – Bleomycin
– Busulphan

498
Antimicr obial & Antineoplastic Dr
Antimicrobial ugs
Drugs 6
Drugs causing Pulmonary Fibrosis :

• Acyclovir • Amiodarone • Bleomycin


• Busulphan • Carmustine • Cyclophosphamide
• Melphalan • Methotrexate • Methysergide
• Nitrofurantoin • Sulphonamides

Blemoycin :
• It is a mixture of glycopeptide antibiotics.
• It chelates Copper or Iron

Produces superoxide ions

Intercalates between DNA strands

Cause chain scission and inhibits repair

• Highly effective in : • Testicular tumour • Hodgkin’s lymphoma


• SCC of skin, head and neck, GIT, esophagus

126. Ans. is c i.e. Ciprofloxacin Ref. Harrison 16/e, p 2539

Drugs causing Myopathy : C Chloroquine, Clofibrate, Cimetidine


H HMG CoA reductase inhibitors
I Interferons
L L-asparaginase
G Glucocorticoid
O OCS
Z Zidovudine
A Amphotericin-B

Statin induced myopathy is increased by :

• Erythromycin • Nicotinic acid


• Gemfibrozil • Cyclosporine
• Ketoconazole

• Ciprfloxacin does not cause myopathy.


• Its side effects are : – Gastrointestinal intolerance, but diarrhea is infrequent.
– CNS : Dizziness, headache, anxiety, insomnia, impairment of concentration.
– Skin/ hypersensitivity : Photosensitivity and swelling of lips.
– Tendonitis and tendon rupture.

499
6 Self Assessment & Review Phar macology
Pharmacology
127. Ans. is b i.e. Penicillin Ref. Harrison 17/e, p 998

Penicillin is the treatment of choice of all types of Actinomycosis infection.

Category Agent

• Drugs for which there is successful – Penicillin (Drug of choice)


clinical experience – Amoxicillin ⎫
– Erythromycin ⏐
– Tetracycline ⎬ Other drugs effective
– Doxycycline ⏐ in Actinomycosis
– Minocycline ⏐
– Clindamycin ⎭

Note : Actinomycosis requires prolonged treatment with high doses of antimicrobial agents due to drugs’ poor
penetration of the thick-walled masses common in this condition and/or sulphur granules.

128. Ans. is c i.e. Puromycin Ref. KDT 6/e, p 711; Ganong 19/e, p 25

Puromycin-amino acid complex substitutes for tRNA-aminoacid complex and prevent addition of further aminoacids
to polypeptides.
Puromycin is the drug which inhibit protein synthesis by premature chain termination and which
structurally resembles an aminoacyl t-RNA.

129. Ans. is c i.e. Chloramphenicol Ref. Harrison 16/e, p 2539

Already explained, refer answer no. 126

130. Ans. is a i.e. Captopril Ref. Harrison 17/e, p 2007

Drugs causing Acute pancreatitis


Menomonic : Very Much GLAD Organ Pancreas TDS
V Valproic acid O OCP
M Methyldopa P Pentamidine
G Glucocorticoids T Tetracycline
L L- asparaginase D Didanosine
A Azathioprine S Sulfonamides
D Diuretics

131. Ans. is a i.e. Lamivudine Ref. Harrison 17/e, p 1957 - 1960

New antiviral drugs being developed for the treatment of chronic hepatitis B
• Telbivudine • Emtricitabine (FTC)
• Tenoflovir • Clevudine (L-MFAU)

500
Antimicr obial & Antineoplastic Dr
Antimicrobial ugs
Drugs 6
Although the combination of lamivudine and PEG IFN suppresses HBV DNA more profoundly during therapy
than does monotherapy with either drug alone (and is much less likely to be associated with lamivudine
resistance), this combination used for a year is no better than a year of PEG IFN in achieving sustained
responses.
Both lamivudine and interferon are effective in the treatment of chronic hepatitis B. But according to Harrison
“Combined use of interferon and lamivudine does not offer any advantage over the use of either drug
alone.”
So, the best answer is lamivudine.

Acute hepatitis Treatment of choice


• B • No treatment - 90% of cases recovery occurs early.
• C • Interferon

Combination therapy with interferon and lamivudine is the treatment of choice in chronic hepatitis ‘C’.

132. Ans. is c i.e. Heparin Ref. Harrison 17/e, p 1753

Drugs causing Interstitial nephritis


• Cephalosporin • Penicillins esp - Methicillin
• Allopurinol • Ciprofloxacin
• Phenindione • Sulphonamides
• Thiazides • NSAIDs
• Rifampicin

133. Ans. is a i.e. Cyclosporine Ref. KDT 6/e, p 838; Katzung 10/e, p 917

Cyclosporine is a fat soluble peptide antibiotic that appears to act at an early stage in the antigen receptor -
induced differentiation of T - cells and blocks their activation.
Cyclosporine binds to cyclophilin, a member of a class of intercellular proteins called immunophillins. Cyclosporine
and cyclophilin forms a complex that inhibits a cytoplasmic phosphate, calcineurin that is necessary for the
activation of a T cell - specific transcription factor.
Other important points : Is the most effective drug for prevention and treatment of graft rejection.
– It is routinely used in renal, hepatic, cardiac, bone narrow and other transplantations.
– Major side effect of cyclosporine is nephrotoxicity.

134. Ans. is b i.e. Rifampicin – Optic neuritis Ref. KDT 6/e, p 741

Methysergide
Adverse effects : • Nausea, abdominal pain, diarrhea, nervousness and other CNS effects
• Prolonged use has caused abdominal, pulmonary and endocardial fibrosis
• Retroperitoneal fibrosis

Rifampicin
Adverse effects : • Hepatitis, a major adverse effect.
• Respiratory syndrome

501
6 Self Assessment & Review Phar macology
Pharmacology

• Purpura, haemolysis, shock and renal failure


• Cutaneous syndrome – flushing, pruritis + rash, redness and watering of eyes.
• Flu syndrome : with chills, fever, headache, malaise and bone pain.
• Abdominal syndrome : Nausea, vomiting, abdominal cramp with or without diarrhoea.
Urine and secretions may become orange - red but this is harmless.

Ethambutol (tuberculostatic)
Loss of visual acuity/colour vision, field defects due to optic neuritis is the most important dose and duration of
therapy dependent toxicity.
Because young children may be unable to report early visual impairment, it should not be below 6 yrs. of age.
• Hyperuricemia due to interference with urate excretion.
Saraslasin : It is an antagonist of angiotensin - 2 receptor (AT2).

135. Ans. is d i.e. Lamivudine Ref. KDT 6/e, p 767

Lamivudine is an anti HIV drug which does not cause peripheral neuropathy.
Side effects of antiretroviral (HIV) drugs :

Pancreatitis + Peripheral neuropathy Lactic acidosis + Hepatomegaly with Steatosis

Didanosine Zidovudine
Zalcitabine Zalcitabine
Stavudine

Most common side effect of zidovudine is macrocytic anaemia and neutropenia.

136. Ans. is a i.e. Acts only on exo erythrocytic cycle Ref. KDT 6/e, p 781 - 782; Rang & Dale 5/e, p 675

Chloroquine is rapidly acting erythrocytic schizontocide and active against all species of plasmodium.
Mechanism of action of Chloroquine :
Chloroquine is concentrated inside infected erythrocytes (higher conc. in infected RBC)

Prevents polymerization of heme into hemazoin

Heme damages plasmodium membrane

137. Ans. is d i.e. Macrocytic anaemia Ref. KDT 6/e, p 767

Remember : Most common side effect of zidovudine is Macrocytic anaemia and neutropenia.

Nucleoside reverse trancriptase inhibitors :


Drugs Side Effects
• Zidovudine Anemia, neutropenia, nausea, malaise, headache, insomnia, myopathy, lactic acidosis,
hepatomegaly with steatosis.
• Didanosine Pancreatitis, peripheral neuropathy, dry mouth, hepatitis.

502
Antimicr obial & Antineoplastic Dr
Antimicrobial ugs
Drugs 6
• Zalcitabine Peripheral neuropathy, pancreatitis, lactic acidosis, hepatomegaly, steatosis, oral ulcer
(aphthous).
• Stavudine Peripheral neuropathy, pancreatitis, hepatitis.
• Lamivudine Rash, peripheral neuropathy.
• Abacavir Rash, fever, hypersensitivity (can be fatal), nausea, vomiting, malaise loss of appetite.

138. Ans. is b i.e. Lung cavitation


Ref. CMDT ‘05, p 1302; KDT 6/e, p 805 - 806; Katzung 10/e, p 860 - 862; Goodman & Gilman 11/e, p 1111

Aerosolized pentamidine may also increase the incidence of pyohemothorax in patient with a history
of pneumocystis carinii infection.

Pentamidine is a highly toxic drug :


• Rapid IV administration : – Hypotension – Tachycardia
– Dizziness – Dyspnea
• I.M. administration : – Pain at the injection site, sterile abscesses may develop.
• Pancreatic toxicity is common. Hypoglycemia due to inappropriate insulin release may be followed by
hyperglycemia.
• Reversible renal insufficiency is also common.
Other side effects :

• Rash • Metallic taste • Fever


• Gastrointestinal symptoms • Acute pancreatitis • Hypocalcemia
• Abnormal liver function tests • Thrombocytopenia • Hallucinations
• Cardiac arrhythmias

139. Ans. is b i.e. Polymyositis Ref. KDT 6/e, p 779

Uses of Interferon - α :
• Chronic hepatitis B and C
• AIDS Related Kaposi’s sarcoma
• Hairy cell leukemia
• Condyloma acuminata caused by Papilloma virus
• Herpes simplex; herpes zoster and CMV infection in immunocompromised patient
• Rhinoviral cold
• Chronic myelogenous leukemia and multiple myeloma
• Melanoma
• Metastatic RCC
• Multiple myeloma

Other Important Points :


• In most of above conditions, we use INF-α
• INF β i.e. interferon β used in – Multiple sclerosis, Hairy cell leukemia, Condyloma accuminata
• INF α i.e. interferon α used in– Chronic granulomatous disease

503
6 Self Assessment & Review Phar macology
Pharmacology
• They are low molecular weight glycoprotein produced by host in response to viral infection.
• They are host specific.

• They are of 3 types : Interferon Source

• α alpha – B Lymphocytes, macrophages


• β beta – Fibroblasts, epithelial cells
• γ gamma – T lymphocytes

140. Ans. is d i.e. Hypercalcemia Ref. KDT 6/e, p 740 - 741

Adverse effect of Antitubercular drugs


Drug Adverse effects
Isoniazid Hepatitis, Peripheral neuritis, Hemolysis in G6PD deficiency, SLE in slow acetylators
Rifampin Proteinuria, Hepatitis, Flu-like syndrome, Red orange urine, Thrombocytopenia
Ethambutol Dose dependent retrobulbar neuritis, ↓visual acuity and Red-green blindness
Pyrazinamide Polyarthralgia, myalgia, Hepatitis & Rash Hyperuricemia, Phototoxicity,
↑ Porphyrin synthesis
Streptomycin Deafness, Vestibular dysfunction and Nephrotoxicity

141. Ans. is a i.e. Rifampicin Ref. KDT 6/e, p 753

Rifampicin is the only bactericidal drug amongst the antileprosy drugs that can render leprosy rapidly non
contagious.
For more details, kindly see answer no. 45

142. Ans. is c i.e. Clofazimine Ref. KDT 6/e, p 752 - 753

Adverse effects of Clofazimine


• Reddish black discoloration of skin
• Dryness of skin and itching
• Acne form eruptions and phototoxicity
• Conjunctival pigmentation
• Discoloration of hair and body secretions

143. Ans. is a i.e. Cefaclor Ref. Harrison 17/e, p 952

Cephalosporins used for pseudomonas infection are Ceftazidime, Cefoperazone, and Cefepime.

144. Ans. is a i.e. Falciparum malaria Ref. KDT 6/e, p 782; Rung & Dale p 675

• Halofantrine is a phenanthrene methanol derivative. It acts on blood schizonticide.

504
Antimicr obial & Antineoplastic Dr
Antimicrobial ugs
Drugs 6
• It is effective against Plasmodium falciparum resistant to chloroquine and sulfa pyrimethamine, as well as
against plasmodium vivax.
Erythrocytic schizontocides
Fast Acting Medium Acting Slow Acting
Artemisin (Fastest), Halofantrine, Mefloquine, Quinine Pyrimethamine, Proguanil,
Chloroquine, Mepacrine

145. Ans. is a i.e. Cotrimoxazole Ref. KDT 6/e, p 762; Harrison 16/e, p 1195

Pneumocystis carinii causes severe pneumonia in neutropenic AIDS patients. Cotrimoxazole is the drug of
choice in prophylaxis as well as treatment.

146. Ans. is b i.e. Protein synthesis Ref. KDT 6/e, p 668

Clindamycin inhibits protein synthesis by acting on 50S subunit of bacterial ribosomes.

For more details, refer answer no. 26

147. Ans. is b i.e. CD4 Cells Ref. KDT 6/e, p 837

Cyclosporine is an immunosuppressant which inhibits T lymphocyte proliferation and IL2 cytokine


production.

CYCLOSPORINE :
Mechanism of Action
Binds to cyclophilin

Inhibits calcineurin (cytoplasmic phosphates)

↓ activation of T cell transcription factor

↓ IL-2, IL-3 and interferon-γ

Uses
DOC for organ or tissue transplantation

Adverse effects
• Peripheral neuropathy • Nephrotoxicity
• Hyperglycemia • Hyperlipidemia
• Hirsutism • Gingival overgrowth
• Cholelithiasis

505
6 Self Assessment & Review Phar macology
Pharmacology
148. Ans. is c i.e. Erytrhromycin Ref. KDT 6/e, p 729; Harrison 17/e, p 861

Erythromycin

First choice drug Second choice drug

• Whooping cough • Chlamydia trachomatis


• Atypical pneumonia caused by • Campylobacter enteritis
Mycoplasma pneumoniae • Legionnaire’s pneumoniae
• Chancroid • Penicillin resistant staphylococcal infections

Organisms Drug of Choice

• Corynebacterium diptheriae – A macrolide (Erythromycin)


• Clostridium tetani – Penicillin G, vancomycin
• E. coli – Ciprofloxacin or levofloxacin, cephalosporin
• H. influenzae – Trimethoprin sulfamethoxazole; Amoxicillin, clavulanate, cefuroxime
• Salmonella typhi – Ciprofloxacin / levofloxacin, Ceftriaxone
• Campylobacter jejuni – Ciprofloxacin
• Legionella – Azithromycin, Fluoroquinolone

149. Ans. is a and b i.e. Used as adjuvant therapy in receptor positive breast cancer; and For
chemoprophylaxis against breast cancer Ref. Goodman & Gilman 11/e, p 1557; KDT 6/e, p 304

Therapeutic uses of Tamoxifen are :


• Tamoxifen is highly efficacious in the treatment of breast cancer. It is used alone for palliation of advanced
breast cancer in women with ER-positive tumors, and it is now indicated as the hormonal treatment of
choice for both early and advanced breast cancer in women of all ages.
• Tamoxifen reduces the risk of developing contralateral breast cancer and is approved for primary prevention
of breast cancer in women at high risk.
• Prophylactic treatment should be limited to 5 years.

150. Ans. is a and b i.e. Cyclophosphamide; and Ifosfamide Ref. KDT 6/e, p 819 - 820

ANTI CANCER DRUGS


Alkylating agents
a. Nitrogen mustards • Cyclophosphamide
• Ifosfamide
• Chlorambucil
• Melphalan
b. Ethylenimine • Thiotepa
c. Alkyl sulfonate • Busulphan
d. Nitrosoureas • Carmustine
• Lomustine
e. Triazine • Dacarbazine

506
Antimicr obial & Antineoplastic Dr
Antimicrobial ugs
Drugs 6
151. Ans. is a, b, c and e i.e. 5-Fu; Methotrexate; Paclitaxel; and Etoposide
Ref. KDT 6/e, p 820, Harrison 17/e, p 521

Drugs that frequently causes Mucositis


• Bleomycin • Actinomycin-D
• Daunorubicin • Fluorouracil
• Methotrexate • Paclitaxel
• Mithramycin • Etoposide (High dose)
• Hydroxyurea • Cytosine arabinoside
• Topotecan • Doxorubicin

152. Ans. is a and b i.e. Cisplatin; and Pyridoxine excess


Ref. KDT 6/e, p 752, 827 - 832; Harrison 16/e, p 473

Option ‘b’ Cisplatin, an antineoplastic drug, causes pure sensory “neuropathy”.


Option ‘a’ Dapsone, a dermatologic agent, used for leprosy causes dose related pure motor neuropathy.
Option ‘c’ Arsenic, used as a herbicide and insecticide, causes both sensory and motor neuropathy.
Option ‘d’ Lead, a heavy metal, causes selective motor neuropathy with prominent wrist drop.
Option ‘e’ Hypothyrodism, causes pure sensory neuropathy. Carpal tunnel and other entrapment
syndromes are common in this disorder.

Conditions causing Pure Motor Neuropathy : • Hypoglycemia • Dapsone


• Suramin • Inorganic lead

Drugs causing Peripheral Neuropathy


Antitubercular Antibiotic Anticancer Antihypertensive Antimalaria Others
Isoniazid Nalidixic acid Cisplatin Glutethimide Chloroquine Phenytoin
Streptomycin Nitrofurantoin Vincristine Hydralazine Amiodarone
Ethionamide Chloramphenicol Procarbazine Tricyclic
Ethambutol Metronidazole antidepressant

153. Ans. is a, b, c and d i.e. Amphotericin B; NSAIDs; Amikacin; and Ethambutol


Ref. KDT 6/e, p 704 - 706

Drugs whose dose is not reduced in renal failure


Doxycycline Erythromycin
Metronidazole Chloramphenicol
Cefoperazone Ceftriaxone
Rifampicin Cefaclor
Carbenicillin Nafcillin
Mezlocillin

507
6 Self Assessment & Review Phar macology
Pharmacology

Antimicrobials contraindicated in renal failure


• All tetracyclines except Doxycycline • Aminoglycoside
• Cephalosporin Ist generations • Nalidixic acid
• Nitrofurantoin • Amphotericin – B

154. Ans. is a and b i.e. Streptomycin mostly vestibulotoxic; and Salicylates cause reversible deafness
Ref. Harrison 15/e, p 436; KDT 6/e, p 702

Drugs causing Deafness

• Aminoglycosides • Desferrioxamine • Aspirin (reversible) • Erythromycin (reversible)


• Cidofovir • Ethacrynic acid • Chloroquine • Frusemide
• Quinine • Interferon • Cisplatin • Mustine hydrochloride

Drugs causing Vestibular disorder :


1. Aminoglycosides
2. Mustine
3. Quinine
• Adriamycin causes cardiotoxicity, marrow depression, alopecia, stomatitis, vomiting and local tissue
damage. It has no role in ear damage.

155. Ans. is a and b i.e. Blood group matching; Causes thrombocytopenia Ref. KDT 6/e, p 622

Dextran has nearly all the properties of an ideal plasma expander except :
• It may interfere with blood grouping and cross matching.
• Though the dextran used clinically is not antigenic, its structure is similar to other antigenic polysaccharides.
Some polysaccharide reacting antibodies, if present in the patient in sufficient quantities, may cross react
with dextran and trigger anaphylactic reaction. Urticaria, itching, bronchospasm, fall in BP occur occasionally;
anaphylactic shock is rare.
• It can interfere with coagulation and platelet function and thus prolong bleeding time; should not be used in
hypofibrinogenemia, thrombocytopenia or in presence of bleeding.

156. Ans. is a, b and c i.e. Nausea & Vomiting; Anemia; and Steatosis Ref. KDT 6/e, p 771

Remember : All the points of antiretroviral drugs used in HIV are Important
Group A : Nucleoside Reverse Transcriptase Inhibitor

Drugs Side Effects


• Zidovudine Anemia, neutropenia, nausea, malaise, headache, insomnia, myopathy, lactic acidosis,
hepatomegaly with steatosis.
• Didanosine Pancreatitis, peripheral neuropathy, dry mouth, hepatitis.
• Zalcitabine Peripheral neuropathy, pancreatitis, lactic acidosis, hepatomegaly, steatosis, oral ulcer
(aphthous).
• Stavudine Peripheral neuropathy, pancreatitis, hepatitis.

508
Antimicr obial & Antineoplastic Dr
Antimicrobial ugs
Drugs 6
• Lamivudine Rash, peripheral neuropathy.
• Abacavir Rash, fever, hypersensitivity (can be fatal), Nausea, vomiting, malaise, loss of appetite.

Group B : Non-nucleoside nucleoside reverse transcriptase


Drugs Side effects
• Nevirapine Skin rash, abnormal liver function test (LFT)
• Delavirdine Skin rash, abnormal LFT
• Efavirenz Skin rash dysphoria, abnormal LFT

Group C : Nucleotide reverse transcriptase inhibitors


Drugs Side Effects
• Tenofovir Gastrointestinal distress

Group D : Protease Inhibitors


Drugs Side Effects
• Saquinavir Diarrhea, nausea, headache, hyperglycemia, fat redistribution, lipid abnormalities
• Ritonavir Nausea, abdominal pain, hyperglycemia, fat redistribution
• Indinavir Nephrolithiasis, Indirect hyperbilirubinemia, hyperglycemia
• Nelfinavir Diarrhea, hyperglycemia, fat redistribution
• Amprenavir N/V, rash, oral paraesthesia, hyperglycemia
• Lopinavir Diarrhea, hyperglycemia, fat redistribution
Group E : Fusion inhibitor
Drug Side effects

Enfuvirtide Hypersensitivity reactions, ↑ ed rate of bacterial pneumonia

157. Ans. is a and b i.e. Quinupristine / dalfopristine; and Linezolid Ref. Harrison 17/e, p 880

Vancomycin is the treatment of choice while there are several other alternative drugs as shown below. Very
frequently asked question, so remember full chart.
For more details, refer answer no. 16

158. Ans. is a and c i.e. Hypertension; and Hirsutism Ref. KDT 6/e, p 839

Toxicity of Cyclosporine are : • Nephrotoxic


• Neurotoxic
• Hepatotoxic

It can cause hypertension, hyperuricemia and an increase in LDL. It can also cause tremors, hirsutism
and gingival hyperplasia (increase in TGF-β causes growth of extracellular matrix resulting in interstitial
fibrosis).

509
6 Self Assessment & Review Phar macology
Pharmacology
159. Ans. is c i.e. Cause DNA damage Ref. Harrison 16/e, p 469 - 470; KDT 6/e, p 821

ALKYLATING AGENTS :
• Alkylating agents are electrophile molecule that covalently modify bases in DNA.
• Major site of alkylation in DNA is N7 portion of guanine.
• They are cell (dividing and resting) cycle non specific and are active in all phase of cell cycle.

Common toxicities to all alkylating agents are : – Myelosuppression – Gonadal dysfunction


– Alopecia – Mucositis
– Secondary neoplasm (particularly leukemia)

160. Ans. is a, b and c i.e. Flucytosine and amphotericin B for cryptococcal; Trimethoprim; &
sulphamethoxazole for UTI; and Pencillin and aminoglycoside Ref. KDT 6/e, p 677

Drug Synergism : When the action of one drug is facilitated or increased by the other, they are said to be
synergistic. In a synergistic pair, both the drugs can have action in the same direction or given alone one may
be inactive but still enhance the action of the other when given together.

Now, considering each option separately.


Option ‘a’ Flucytosine has supra-additive action with AMB in the case of fungi sensitive to both, e.g.
Cryptococcoses, Coccidioidomycosis. (AMB increases the penetration of 5-FC into the
fungus).
Option ‘b’ Sulfamethoxazole was selected for combining with trimethoprim because both have nearly
the same t½ (∼ 10hr). Optimal synergy in case of most organisms is exhibited at a
concentration ratio of sulfamethoxazole 20 : trimethoprim 1, the MIC of each component may
be reduced by 3-6 times.
Option ‘c’ Penicillin/ampicillin + streptomycin/gentamicin is combined for use in enterococcal subacute
bacterial endocarditis. Aminoglycoside get penetrated when penicillin inhibits bacterial wall.
Option ‘d’ This is the wrong answer. No synergism is reported for chlortetracycline + penicillin.

Other important antimicrobial combinations :


• Carbenicillin/ticarcillin + gentamicin for pseudomonas infection, especially in neutropenic patients.
• Ceftazidime + ciprofloxacin for pseudomonas infected orthopaedic prosthesis.
• Rifampicin + INH in tubercular infection.

161. Ans. is a, b and d i.e. Amikacin; Lithium; and Cycloserine Ref. Harrison 17/e, p 857

Antimicrobial Contraindicated in Renal Failure :


• All tetra except Doxycycline • Aminoglycoside
• Cephalosporins first generation • Nalidixic acid
• Nitrofurantoin • Amphotericin – B

510
Antimicr obial & Antineoplastic Dr
Antimicrobial ugs
Drugs 6
162. Ans. is d i.e. Aminoglycosides Ref. KDT 6/e, p 688, 696, 720

Resistance to different antimicrobials is acquired in various ways. So, lets analyse each option
separately :
Option ‘d’ Mechanisms of resistance to Aminoglycosides :
• Acquisition of cell membranes bound inactivating enzymes which phosphorylate/adenylate
or acetylate the antibiotic
• Mutations decreasing the affinity of ribosomal proteins that normally bind the
aminoglycosides
• Decreased efficiency of aminoglycoside transporting mechanism
Option ‘a’ The primary mechanism of acquired resistance to Penicillins is through the production of
penicillinase which destroys the β - lactam ring.
Option ‘b’ Resistance to fluoroquinolones is due to chromosomal mutation producing a DNA gyrase or
topoisomerase IV with reduction of affinity for fluoroquinolones.
Option ‘c’ Resistance to tetracyclines is by 2 ways :
a. Tetracycline concentrating mechanism becomes less efficient or bacteria acquire capacity
to pump it out
b. Plasmid mediated synthesis of protection protein, which protects the ribosomal binding
site from tetracyclines.

163. Ans. is b and c i.e. Cloxacillin; and Methicillin Ref. KDT 5/e, p 658; Harrison 16/e, p 47 (Table)

Penicillinase resistant penicillins are :

– Methicillin – Oxacillin – Cloxacillin – Nafcillin

For more details about Classification of Semisynthetic Penicillins, kindly see answer no. 16

164. Ans. is a, b and c i.e. Cisplatin; Desferrioxamine; and Mustine hydrochloride


Ref. Harrison 17/e, p 521, 522; KDT 5/e, p 776 - 777

Already explained, refer answer no. 173

165. Ans. is a i.e. Cisplatin Ref. KDT 6/e, p 821

• The emetogenic potential of cytotoxic drugs is as follows.

High Moderate Mild

↓ ↓ ↓ ↓
• Cisplatin • Carboplatin • Bleomycin • Hydroxyurea
• Mustine • Cytarabine • Chlorambucil • Vincristine
• Cyclophosphamide • Procarbazine • Busulfan • Methotrexate
• Actinomycin D • Vinblastine • Fluorouracil • Etoposide
• Dacarbazine • Doxorubicin • 6 - mercaptopurine • L - asparaginase
• Mithramycin • Daunorubicin • 6 - thioguanine

Nausea and vomiting occurs due to direct stimulation of CTZ by the drug as well as generation of emetic
impulses/mediators from the upper gastrointestinal tract and other areas.

511
6 Self Assessment & Review Phar macology
Pharmacology
166. Ans. is a, c and e i.e. Isoniazid; Ethambutol; and Rifampicin Ref. KDT 6/e, p 747

First Line Antituberculosis Drugs : Isoniazid + rifampicin + pyrazinamide + ethambutol (or streptomycin) for
2 months followed by a course of isoniazid + rifampicin for the next 4 months.
The treatment regimens followed in India under the Revised National Tuberculosis Control Programme (RNTCP
1997).
TB category Initial phase Continuation phase Total duration
I 2H3 R3 Z3 E3 4H3 R3 6
II 2H3 R3 Z3 E3 S3 + 1H3 R3 Z3 E3 5H3 R3 E3 8
III 2H3 R3 Z3 4H3 R3 6

• The numeral before a phase is the duration of that phase in months.


• The numeral in subscript (e.g. H3 R3) is number of doses of that drug per week. If there is no subscript
numeral, then the drug is given daily.

167. Ans. is b and d i.e. Cyclosporine; and Tacrolimus Ref. KDT 6/e, p 837

Cyclosporine and Tacrolimus both are immunosuppressant which inhibits T lymphocyte proliferation
IL2 cytokine production.

MECHANISM OF ACTION :
Cyclosporine Tacrolimus (FK - 506)
Binds to cyclophilin • It is a macrolide isolated from soil fungus
↓ • Binds to cyclophilin FKBP - 12
Inhibits calcineurin (cytoplasmic phosphates) ↓
↓ Drug - FKBP complex binds to calcineurin
↓ activation of T cell transcription factor ↓
↓ Inhibit calcinurin PO4– activation of T cell transcription factor
↓ IL-2, IL-3 and interferon-γ ↓
↓ IL - 2, IL - 3 & interform - γ

168. Ans. is a and b i.e. Vincristine; and Isotretinoin Ref. Goodman & Gilman 11/e, p 1351

Let’s consider each option separately.


• Option ‘a’ – Vincristine is the extract of Periwinkle plant (Catharanthus roseus). The extract of
this plant yields four products. Vincristine, Vinblastine, Vinleurosine and Vinrosidine.
• Option ‘b’ – Isotretinoin (13 – cis retinoic acid) used in skin cancer has a plant source.
• Option ‘c’ – Bleomycins are an important group of DNA cleaving antibiotics produced as
fermentation products of Streptococcus verticillus.

169. Ans. is c and d i.e. Albendazole; and Mebendazole Ref. KDT 6/e, p 809 - 814

• The 3 broad spectrum antihelminthic drugs : – Albendazole – Mebendazole – Thiabendazole

512
Antimicr obial & Antineoplastic Dr
Antimicrobial ugs
Drugs 6
• Niclosamide is highly effective against Taenia, Diphyllobothrium latum, Hymenolepis nana and Thread worms.
• Niclosamide is highly effective against Schistosomes.
• Pyrantel pamoate is highly effective against Ascaris, Enterobius and Ankylostoma.

Drug of choices of All Parasites are given in answer no. 64

170. Ans. is a, b and e i.e. INH; Pyrazinamide; and Rifampicin Ref. KDT 6/e, p 639 - 640

All first line Antitubercular drugs are bactericidal except Ethambutol which is static.

Bacteriostatic ATT : • Ethambutol • PAS • Ethionamide • Cycloserine

171. Ans. is a i.e. Inhibition of dihydrofolate reductase Ref KDT 6/e, p 823

Methotrexate is folate antagonist, it inhibits dihydrofolate reductase (DHFRase).


• Methotrexate is an antifolate immunosuppressant :
– It inhibits dihydrofolate reductase (DHFrase)- blocking the conversion of dihydrofolic and (DHFA) to
tetrahydrofolic acid (THFA) which is an essential coenzyme required for one carbon transfer reactions in
de novo purine synthesis and amino acid interconversions.
– It has cell cycle specific action - kills cells in S phase; primarily inhibits DNA synthesis.
– It exerts major toxicity on bone marrow - low doses given repeatedly cause megaloblastic anaemia, but
high doses produce pancytopenia.
The toxicity of methotrexate is antagonised by ...
• Folinic acid (Leucovorin or Citrovorum factor) is N5 Formyltetrahydrofolate. It is an active coenzyme which
does not need to be reduced by dihydrofolate reductase before it can act. In methotrexate treated patient,
DHFRase is inhibited but folinic acid bypasses this enzymatic block and is still able to act while folic acid
becomes useless.
• When leucovorin is used, the daily dose of 5 FLUOROURACIL must be reduced because Leucovorin
potentiates 5-Fluorouracil.

513
6 Self Assessment & Review Phar macology
Pharmacology
172. Ans. is a and b i.e. Interacts with terfenadine; and G.I. symptoms are seen
Ref. Goodman & Gilman 11/e, p 1369, 1370 Katzung 10/e, p 810

• Ritonavir is an important antiretroviral drug and lets look at each option separately.

Option ‘a’ – Ritonavir is metabolised primarily in the liver by cytochrome P450 isoforms, especially
CYP3A4 and less so by CYP2D6
– It potentially inhibits CYP3A4, markedly increasing plasma levels of terfenadine which is
also metabolised by this cytochrome leading to serious ventricular arrhythmia (Torsades
de pointes)
Option ‘b’ – Side effects are :
• GI symptoms are most frequent
• Paraesthesia
• Hepatic toxicity
• ↑ serum cholesterol
• Insulin resistance
Option ‘c’ – Ritonavir and its metabolites are eliminated from the body predominantly in the faeces
(86%) with minor urinary elimination (11%), so not contraindicated in renal failure.
Option ‘d’ – It is a protease inhibitor and not NNRTI.
Option ‘e’ – No relevance of this option.

173. Ans. is a, b, c and e i.e. Cisplatin, Furosemide, Vancomycin; and Erythromycin


Ref. Harrison 15/e, p 436; KDT 6/e, p 821, 832

• Aminoglycosides are ototoxic and should not be used along with the following ototoxic drugs :
OTOTOXIC DRUGS
Aminoglycoside Cytotoxic drugs Analgesics High ceiling Antimalarials
antibiotics diuretics
• Streptomycin • Cisplatin • Salicylates • Furosemide • Quinine
• Gentamicin • Nitrogen mustard • Indomethacin • Ethacrynic acid • Chloroquine
• Kanamycin • Phenylbutazone
• Amikacin

174. Ans. is b and c i.e. Ciprofloxacin; and Cefotaxime Ref. Harrison 16/e, p 861

Drugs used for Penicillinase producing N. Gonorrhoea (PPNG)

First line drugs Second line drugs

Ceftriaxone Spectinomycin
Cefexime Ceftizoxime
Ciprofloxacin Cefotaxime
Ofloxacin Cefotetan
Levofloxacin Cefoxitin

If chlamydiae infection is not ruled out : Add either : Azithromycin (single dose)
or Doxycycline

514
Antimicr obial & Antineoplastic Dr
Antimicrobial ugs
Drugs 6
175. Ans. is c and e i.e. Chlorambucil; and Cyclophosphamide Ref. KDT 6/e, p 819 - 820

ANTI CANCER DRUGS


Alkylating agents
a. Nitrogen mustards • Cyclophosphamide
• Ifosfamide
• Chlorambucil
• Melphalan
b. Ethylenimine • Thiotepa
c. Alkyl sulfonate • Busulphan
d. Nitrosoureas • Carmustine
• Lomustine
e. Triazine • Dacarbazine

176. Ans. is b i.e. Ganciclovir Ref. Harrison 17/e, p 844, 847

• Chorioretinitis due to toxoplasmosis can be seen alone or, more commonly in association with CNS
toxoplasmosis.
• Most common abnormal findings on fundoscopic examination are cotton wool spots.
• Therapy for CMV retinitis consist of intravenous ganciclovir or foscarnet with cidofovir, foscarnet have been
shown to be slightly more effective than ganciclovir.
• Systemic toxicity of ganciclovir is high (bone marrow depression, rash, fever, vomiting, neuropsychiatric
disturbances) and uses is restricted to severe CMV infections in immunocompromised (AIDS, transplant
recipient) patients.
• Toxicity of foscarnet is high: damages kidney, produces a renal diabetes like condition, acute renal failure
can also occur.

177. Ans. is b, c and d i.e. Actinomycin; Bleomycin; and Mithramycin Ref. KDT 6/e, p 820

Anticancer antibiotic drugs


• Actinomycin D (Dactinomycin) • Doxorubicin
• Daunorubicin • Mitoxantrone
• Mitomycin C • Mithramycin (= Plicamycin)
• Bleomycin

178. Ans. is a and b i.e. Metronidazole; and Imipenem Ref. Harrison 17/e, p 1005

• Metronidazole is selectively toxic to anaerobic microorganisms (Inactive against mixed anaerobes and aerobic
infections).
• It is generally used in combination with gentamicin or cephalosporins (many are mixed infections).

515
6 Self Assessment & Review Phar macology
Pharmacology
Antimicrobial therapy for infection involving commonly encountered anaerobic gram –ve rods.
Category I Category II Category III Category IV
(<2% resistance) (<15% resistance) (Variable resistance) (Resistance)
Imipenem Cefoxitin Penicillin Aminoglycoside
Metronidazole Clindamycin Cephalosporins Quinolones
Meropenemm Tetracycline Monobactam
Ampicillin/Sulbactam Vancomycin
Ticarcillin/Clavulinic acid Erythromycin
Piperacillin/Tazobactam
Chloramphenicol

179. Ans. is a i.e. Protein C deficiency Ref. CMDT ‘05, p 513

The syndrome of Warfarin induced skin necrosis may occur in patients with undiagnosed protein C deficiency.
Protein C is vitamin K dependent and has a shorter half life than the coagulation proteins.
Warfarin, by creating a vitamin K-dependent state, will transiently deplete protein C before it leads to
anticoagulation. During the period of hypercoagulability due to unopposed protein C depletion, thrombosis of
skin vessels may lead to infarction and necrosis.
The syndrome can be prevented by the use of heparin for 5-7 days until Warfarin induces coagulation.

180. Ans. is a, b and d i.e. Sulfonamides; Mtx; and Trimethoprim


Ref. Harrison 16/e, p 601; CMDT ‘05, p 471

Inhibitors of dihydrofolate reductase Inhibitors of folate absorption


• Methotrexate • Phenytoin
• Pyrimethamine • Sulfasalazine
• Triamterene • Co-trimoxazole
• Pentamidine • Barbiturates
• Trimethoprim • Ethanol

181. Ans. is a and c i.e. Saquinavir; and Nelfinavir Ref. KDT 6/e, p 767; Goodman & Gilman 11/e, p 1276

All VIR are protease inhibitor except Abacavir which is NRTI and Raltigravir which is integrase inhibitor.

Protease inhibitors are : • Saquinavir • Ritonavir


• Indinavir • Nelfinavir
• Amprenavir • Lopinavir

182. Ans. is b, c and d i.e. Vincristine; Paclitaxel; and Colchicine Ref. KDT 6/e, p 830 - 831

Drugs causing cell cycle specific inhibition are given below :


M phase • Vincristine, vinblastine, paclitaxel, colchicine
G1 phase • Vinblastine
S phase • Mtx, cytarabine, 6-TG, 6-MP, hydroxyurea, mitomycin C, doxorubicin, daunorubicin.
G2 phase • Daunorubicin, bleomycin, paclitaxel.

516
Antimicr obial & Antineoplastic Dr
Antimicrobial ugs
Drugs 6
183. Ans. is e i.e. Ciprofloxacin Ref. Harrison 17/e, p 958

Antibiotic therapy for enteric fever in adults


Indication Agent Dosage (route) Duration, days
Empirical treatment
Ceftriaxone 1-2 g/d (IV) 7-14
Azithromycin 1 g/d (PO) 5
Fully susceptible
Ciproflxacin (1st line) 500 mg bid (PO) 5-7
or 400 mg q12h (IV)
Amoxicillin (2nd line) 1 g tid (PO) 14
or 2 g q6h (IV)
Chloramphenicol 25 mg/kg tid (PO or IV) 14-21
Trimethoprim-sulfamethoxazole 160/800 mg bid (PO) 14
Multidrug resistant
Ciprofloxacin 50 mg bid (PO) 5-7
or 400 mg q12h (IV)
Ceftriaxone 2-3 g/d (IV) 7-14
Azithromycin 1 g/d (PO) 5
Nalidixic acid resistant
Ceftriaxone 1-2 g/d (IV) 7-14
Azithromycin 1 g/d (PO) 5
High dose ciprofloxacin 750 mg bid (PO) 10-14
or 400 mg q8h (IV)

NARST – Nalidixic acid Resistant S. typhi


• For the last decade, Quinolones remained the first choice drugs in typhoid fever. Lately, quinolones resistance
(NARST) is being frequently reported.
• Quinolones are the only available oral antibiotics for the the treatment of multidrug resistant S. typhi infections.
The greatest experience has been for ciprofloxacin (500mg orally BD for 10 days).
• Option “a” and “b” – Incorrect – First and second generation cephalosporins as well as aminoglycosides
are ineffective in treating clinical infections.
• Chloramphenicol remained the standard treatment of enteric fever until emergence of resistance in 1970s,
so no longer used now.
• In 1989, MDR S. typhi emerged and they are resistant to :
– Chloramphenicol – Ampicillin – Trimethoprim
– Streptomycin – Sulphonamides – Tetracycline

184. Ans. is a, b, and d i.e. Hydralazine, Procainamide and Methysergide Ref. Harrison 17/e, p 1489

Pericarditis due to the following drugs is related to hypersensitivity or autoimmunity.


Drugs causing pericarditis
• Procainamide • Hydralazine
• Isoniazid • Minoxidil
• Anticoagulants • Methysergide
• Phenytoin

517
6 Self Assessment & Review Phar macology
Pharmacology
185. Ans. is a and c i.e. 4th Generation cephalosporin; and Antipseudomonal action
Ref. Harrison 17/e, p 952

Option “a” is correct –


4th generation cephalosporins are Cefepime and Cefpirome.

Important points about Cefepime :


a. It is a fourth generation cephalosporin.
b. It is more resistant to hydrolysis by chromosomal β lactamases (e.g. those produced by enterobacter) and
some extended spectrum β lactamases that inactivate many 3rd generation cepahalosporins.
c. Good activity against :
– Pseudomonas aeuroginosa – Enterobacteriacae
– Staphylococcus aureus – S. pneumoniae
– Hemophilus – Neisseria

d. It is 100% cleared by the kidneys and dosage is to be reduced in renal failure.


e. Plasma half life is 2 hours.
f. It is not a prodrug.

186. Ans. is b and c i.e. Ceftazidmine; and Piperacillin Ref. Harrison 17/e, p 952

Antipseudomonal Agents (Drugs in class are listed in order of decreasing in vitro activity)
Antipseudomonal Penicillins Antipseudomonal cephalosporins Fluoroquinolones
Piperacillin Ceftazidime Ciprofloxacin
Piperacillin/Tazobactam Cefoperazone Levofloxacin
Mezlocillin Cefepime
Ticarcillin
Ticarcillin/Tazobactam

187. Ans. is a, c and d i.e. Actinomycin D; Bleomycin; and Doxorubicin Ref. KDT 6/e, p 826

All the antibiotic anticancer drugs acts by intercalating between DNA stands and interfering with its
template function.
Antibiotic anticancer drugs
• Actinomycin -D (Dactinomycin) • Doxorubicin
• Daunorubicin • Bleomycin
• Mitoxantrone • Mitomycin C
• Mithramycin (Plicamycin)

518
Antimicr obial & Antineoplastic Dr
Antimicrobial ugs
Drugs 6
188. Ans. is b and d i.e. 6 - Mercaptopurine; and Methotrexate Ref. KDT 6/e, p 820 - 824

First, go through this classification :


Anti metabolites
a. Folate antagonist • Methotrexate
b. Purine antagonist • 6 - Mercaptopurine
• 6 - Thioguanine
• Azathioprine
c. Pyrimidine antagonist • 5 - Fluorouracil
• Cytarabine (Cytosine arabinoside)

Methotrexate is a folate antagonist but it is also said to inhibit formation of purines because

Methotrexate inhibits the enzyme dihydrofolate reductase

Purine synthesis is inhibited because tetrahydrofolate is used in purine synthesis

189. Ans. is b i.e. Erythromycin Ref. KDT 6/e, p 727

Erythromycin : Ribosome and hinder translocation of elongated of peptide chain from A to P. Premature
chain termination.

For more details, refer answer no. 2

190. Ans is b i.e. INH Ref. Katzung 10/e, p 60; KDT 6/e, p 740

Drugs whose metabolism is under genetic control are given below.

Drugs Clinical consequences

• Codeine Reduced analgesia


• Debresoquin Orthostatic hypotension
• Ethanol Facial flushing, CVS symptoms
• Hydralazine Lupus like syndrome
• INH Peripheral neuropathy
• Mephenytoin Overdose toxicity
• Mercaptopurines Myelosuppression
• Nicotine Lesser addiction
• Nortriptyline Toxicity
• Omeprazole Increased efficacy
• Succinylcholine Prolonged apnoea
• S-warfarin Bleeding
• Tolbutamide Cardiotoxicity

519
6 Self Assessment & Review Phar macology
Pharmacology
191. Ans. is c and d i.e. Pancreatitis; and Cardiomyopathy Ref. KDT 6/e, p 767

All the antiretroviral drugs used in HIV are V.Important from PGMEE point of view.

Nucleoside reverse transcriptase inhibitors


Drugs Side Effects
• Zidovudine (AZT) Anemia, neutropenia, nausea, malaise, headache, insomnia, myopathy, lactic
acidosis, hepatomegaly with steatosis.
• Didanosine Pancreatitis, peripheral neuropathy, dry mouth, hepatitis.
• Zalcitabine Peripheral neuropathy, pancreatitis, lactic acidosis, hepatomegaly, steatosis, oral
ulcer (aphthous).
• Stavudine Peripheral neuropathy, pancreatitis, hepatitis.
• Lamivudine Rash, peripheral neuropathy.
• Abacavir Rash, fever, hypersensitivity (can be fatal), Nausea, vomiting, malaise loss of appetite.

192. Ans. is a and b i.e. Binds to 50S subunit; and Prevents chain elongation Ref. KDT 6/e, p 716

Chloramphenicol is a protein synthesis inhibitor acting on the 50S ribosome to interfere with peptide bond
formation.
For more details about Mechanism of action of other antibiotics, kindly see answer no. 50

193. Ans. is a i.e. Thymidine Kinase Ref. KDT 6/e, p 768; Goodman & Gilman 11/e, 1247

Mechanism of Acyclovir resistance :


• Absence or partial production of viral thymidine kinase (mainly in Herpes simplex virus).
• Altered thymidine kinase substrate specificity i.e. phosphorylation of thymidine but not acyclovir (mainly in
Varicella zoster virus).
• Altered viral DNA polymerase.

For clear understanding of resistance, we should first look at the mechanism of action of Acyclovir.

– Acyclovir is preferentially taken up by virus infected cells.


– It has lower toxicity on host cells, several hundred lower chemotherapeutic index.

Also remember :
• Acyclovir is active against herpes group of virus in the following order :
H. simplex I > H. simplex II > Varicella zoster virus = Epstein - barr virus CMV is practically not affected.
• It is primarily excreted unchanged in urine, both by glomerular filtration and tubular secretion; so renal
impairment necessitates dose reduction.

Adverse effects are generally mild and well tolerated


• Topical – Stinging and burning sensation
• Oral – Headache, nausea, malaise
• IV – Rashes, sweating, emesis, fall in B.P. Dose dependent decrease in GFR
Tremors, lethargy, hallucinations, convulsions

520
Antimicr obial & Antineoplastic Dr
Antimicrobial ugs
Drugs 6

194. Ans. is a i.e. ↑ Spectrum Ref. KDT 6/e, p 702 - 703

• The Addition of : β - lactamase inhibitors such as (Clavulinic acid, Sulbactam and Tazobactam) with
certain extended spectrum penicillins such as (Ampicillin, Amoxicillin and Piperacillin).
Extends the spectrum of these penicillins to include many organisms that are resistant by virtue of β -
lactamase production.
• The combination has no effect on half lives or the adverse effects.
Addition of Clavulinic acid with Amoxicillin
Restablishes the activity of amoxicillin against Does not establish the activity of amoxicillin against
• β-lactamase producing S. aureus • Serratia
(but not MRSA that have altered PBPs) • Indole positive proteus
• H. influenzae • Citrobacter
• N. gonorrhoeae • Enterobacter
• E.Coli • Pseudomonas
• All Proteus species • Acinetobacter
• Klebsiella Because these produce chromosomal β-
• Salmonella, Shigella lactamases that are not inhibited by these
• Anaerobes (including Bacteroides species) compounds.

Also remember about Clavulinic acid :


– Obtained from Streptomyces clavuligerus.
– It has a β-lactam ring but no antibacterial activity of its own.
– It inhibits (class II to class V) of β lactamases but not (class I cephalosporinase) produced by both Gram +ve
and Gram-ve bacteria.
– It is a progressive inhibitor i.e. binding of β lactamase is reversible initially but becomes covalent later –
inhibition increases with time.

521
6 Self Assessment & Review Phar macology
Pharmacology
– It is also called suicide inhibitor because it gets inactivated after binding to the enzymes.
– Both clavulinic acid and amoxicillin have : - Similar t½ of 1 hour.
- Similar tissue distribution.

195. Ans. is c and d i.e. Cephalosporin; and Semisynthetic penicillin Ref. KDT 6/e, p 697 - 698

• All forms of natural and semisynthetic penicllin can cause allergy but it occurs more commonly on parenteral
administration, being highest with procaine penicillin.
• There is partial cross-sensitivity between different types of penicillins e.g. Cephalosporins.
• Monobactams (Aztreonam) lack cross-sensitivity with other beta-lactam antibiotics and can be used in
patients allergic to penicillin and cephalosporin.

196. Ans. is a i.e. Decreases chance of vertical transmission


Ref. KDT 6/e, p 777; Harrison 16/e, p 1082, 17/e p 1190, 1200

• Zidovudine is given for HIV in pregnancy because it decreases the chance of vertical transmission. There is
no relationship between Zidovudine use in pregnancy and severity of infection.
• The effect of treatment on HIV in pregnancy is given below.

Treatment modality Vertical transmission (in %)

1. No treatment 25 - 35%
2. Zidovudine < 5%
(from 2nd trimester through delivery
& to the infant for 6 wks after birth)
3. HAART regimen ≤ 1%
(from 2nd trimester through delivery
& to the infant for 6 wks after birth)
4. HAART regimen
(from 2nd trimester through delivery 0%
& to the infant for 6 wks after birth)
Plus
Caesarian section delivery

HAART = Highly affective antiretroviral combination therapy.

197. Ans. is a and c i.e. Cinchonism; and Hypoglycemia


Ref. Goodman & Gilman 10/e, p 1088 - 1089; KDT 6/e, p 789

• Quinine causes hypoglycemia due to release of insulin from pancreas. Sweating, palpitation and
tachycardia are common symptoms of hypoglycemia.
• Quinine is associated with a triad of dose related toxicities when it is given at full therapeutic or
excessive doses.
These are : • Cinchonism
• Hypoglycemia
• Hypotension

522
Antimicr obial & Antineoplastic Dr
Antimicrobial ugs
Drugs 6
• Cinchonism is characterised by tinnitus, high-tone deafness, visual disturbances, headache, dysphoria,
nausea, vomiting, vertigo, mental confusion, diarrhoea and flushing. It disappears completely after the drug
is stopped.
• Hypoglycemia can occur even at therapeutic doses due to stimulation of insulin release from pancreatic
beta cells. Despite treatment with glucose infusions, this complication can be serious and life threatening,
especially in prolonged severe infection and pregnant women, who have increased sensitivity to insulin.
• Hemolysis can occasionally result, especially in pregnant women and in patients of falciparum malaria,
resulting in hemoglobinuria (black water fever) and kidney damage. This can also result in Hyperkalemia.

198. Ans. is a i.e. Ampicillin Ref. Harrison 16/e, p 1046

“A morbilliform or papular rash, usually on the arms to trunk, develops in 5% of cases. Most patients
treated with ampicillin develops a macular rash. This rash is not predictive of future adverse reactions to
penicillin”.

So, ampicillin is said to be contraindicated in patients of infectious mononucleosis. There is no link of antibiotics
mentioned in other options with the disease.

199. Ans. is a and b i.e. Quinidine decreases excretion of digoxin; and Quinidine displaces digoxin from
protein binding sites Ref. KDT 6/e, p 511 - 512

• Quinidine increased plasma concentration of digoxin by :


a. Displacing it from tissue binding sites.
b. Decreasing its renal and biliary clearance (inhibits P - glycoprotein).
• Quinine has similar action on heart as quinidine, but less marked.
• Quinine is the d - isomer of Quinine.
• Other important drug interactions of Quinidine are as follows :
a. Diuretics, by inducing hypokalemia increase the incidence of torsades de pointes and VF due to Quinidine.
b. Vasodilators given to patients receiving quinidine will produce synergistic fall in BP.
c. Quinidine inhibits CYP2D6 : prolongs t½ of propafenone and reduces conversion of codeine to morphine.
d. Synergistic cardiac depression with potassium salts, verapamil and β blockers.
• Cinchonism is caused by Quinine.

200. Ans. is b i.e. Methotrexate Ref. KDT 6/e, p 823

Methotrexate is folate antagonist, it inhibits dihydrofolate reductase (DHFRase).


• Methotrexate is an antifolate immunosuppressant :
– It inhibits dihydrofolate reductase (DHFrase)- blocking the conversion of dihydrofolic and (DHFA) to
tetrahydrofolic acid (THFA) which is an essential coenzyme required for one carbon transfer reactions in de
novo purine synthesis and amino acid interconversions.
– It has cell cycle specific action - kills cells in S phase; primarily inhibits DNA synthesis.
– It exerts major toxicity on bone marrow - low doses given repeatedly cause megaloblastic anaemia, but
high doses produce pancytopenia.

523
6 Self Assessment & Review Phar macology
Pharmacology
201. Ans. is d i.e. Trophoblastic disease Ref. Goodman & Gilman 11/e, p 1335; KDT 6/e, p 823, 832

• Methotrexate is most useful in choriocarcinoma and related trophoblastic tumours of women.


• Cure is achieved in approx 75% of advanced cases treated sequentially with methotrexate and dactinomycin,
and in over 90% when early diagnosis is made.
• Methotrexate, either locally or systemically can be used in ectopic pregnancy when adnexal mass is <3
cm. Local application (transvaginal) is simple and successful in 90% of cases (Shaw).
Other uses of Methotrexate are :
• Acute lymphoblastic leukemia in children.
• Treatment and prophylaxis of meningeal leukemia or lymphoma and meningeal carcinomatosis.
• Osteosarcoma, Burkitt’s lymphoma, Carcinomas of breast, head and neck, ovary and bladder.
• Mycosis fungoides.
• Rheumatoid arthritis.
• Severe, disabling psoriasis.
• As immunosuppressant.

202. Ans. is a i.e. Cell membrane Ref. KDT 6/e, p 758, 668

• Amphotericin B acts on cell membrane. It has high affinity for fungal cell membrane ERGOSTEROL. After
combining with it, a micropore is formed, through which leakage of cell contents occur.
• The following drugs act by inhibiting the cell wall and the cell membrane :
On cell wall On cell membrane
• Penicillins Polypeptide
• Vancomycin • Polymyxin
• Cephalosporins • Colistin
• Cycloserine Polyene
• Bacitracin • Amphotericin-B
• Nystatin

203. Ans. is b i.e. Polypeptide chain elongation Ref. KDT 6/e, p 668

Protein Synthesis Inhibitors

Mnemonic : Bye AT 30 CELLS AT 50


30S Inhibitors 50S inhibitors
Aminoglycosides : Chloramphenicol :
Inhibits/ freeze initiation complex Interference with peptide bond
Interfere with polysomes formation and transfer of
formation and peptide chain from P-site.
misreading of m-RNA Erythromycin telithromycin
(1st step in protein synthesis) Hinder translocation
Tetracyclines : of elongated peptide chain
Inhibit Aminoacyl t-RNA back from A site to P-site.
attached to A site Lincomycin, Clindamcin, Streptogramins

524
Antimicr obial & Antineoplastic Dr
Antimicrobial ugs
Drugs 6
Exception : Linezolid binds to 50s ribosomal unit near interface between 30s subunit and thus inhibits the
formation of initiation complex.

204. Ans. is a i.e. RSV Ref. KDT 6/e, p 778

• Nebulized ribavirin has been used for respiratory syncytial virus (RSV) bronchiolitis in infants and children,
particularly those with congenital heart disease, prematurity or other high risk conditions.
• Ribavirin is a purine nucleoside analogue and has broad spectrum antiviral activity, including that against
influenza A and B, respiratory syncitial virus and many other DNA and double stranded RNA viruses.
• Prominent toxic effects are anemia, hemolysis, CNS and GIT symptoms. It is also teratogenic.

205. Ans. is b i.e. Reverse transcriptase inhibition Ref. KDT 6/e, p 770

• Zidovudine is a thymidine analogue and the prototype NRTI.


• After phosphorylation in the host cell—zidovudine triphosphate selectively inhibits viral reverse transcriptase
(RNA dependent DNA polymerase) in preference to cellular DNA polymerase. The formation of new double
stranded viral DNA is blocked.
• It is also incorporated into the growing viral DNA to cause chain termination.

• It prevents infection of new cells by HIV but has no effects on virus directed DNA that has already
integrated into the host chromosome.

206. Ans. is a i.e. Pentostatin Ref. Goodman & Gilman 11/e, p 1346

ADA inhibitors
Pentostatin • It is a transition state analogue of intermediate stage in the adinosine deaminase (ADA)
reaction and is a potent inhibitors of this enzyme.
• Inhibition of ADA leads to accumulation of intracellular adenosine and deoxyadenosine
nucleotides, which block DNA synthesis.
• It is extremely effective in producing complete remission in hairy cell leukemia and is
effective even in patients who are refractory to interferone-alfa.
Cladribine • It is an adenosine deaminase-resistant purine analogue.
• It produces DNA strand breaks and NAD and ATP depletion to as well as apoptosis in
some cell lines
• It is drug of choice in hairy cell leukemia because of its high effectiveness and low toxicity
(complete response in 80% of patients)

525
6 Self Assessment & Review Phar macology
Pharmacology
207. Ans. is b i.e. Docetaxel Ref. KDT 6/e, p 819 - 820

Looking at each option separately.


Vinca alkaloids – These are mitotic inhibitors.
– They bind to microtubular protein – ‘tubulin’, prevent its polymerization, cause
disruption of mitotic spindle and interfere with cystoskeletal function.
– The chromosomes fail to move apart during mitosis : metaphase arrest occurs.
Docetaxel – It enhances polymerization of tubulin : a mechanism opposite to that of vinca
akaloids.
– The microtubules are stablized and their depolymerization is prevented.
– Abnormal arrays or ‘bundles’ of microtubules are produced throughout the cell
cycle.
Etoposide – It is not a mitotic inhibitor but inhibits DNA topoisomerase II function
– It arrests cells in the G2 phase in cause DNA breaks the subsequent resealing of
the cleaved DNA strand is prevented.
Colchicine – It causes metaphase arrest by binding to microtubules of mitotic spindle.

208. Ans. is a i.e. Antibiotics Ref. KDT 6/e, p 819 - 820

Adriamycin is an Anticancer antibiotics


Anticancer antibiotics
• Actinomycin D (Dactinomycin) • Doxorubicin
• Daunorubicin • Mitoxantrone
• Mitomycin C • Mithramycin
• Bleomycin (= Plicamycin)

209. Ans. is b and c i.e. Retinitis pigmentosa; and Yellow discoloration of skin Ref. KDT 6/e, p 786

All other options other then Retinitis pigmentosa are side effects of chloroquine.

Other side effects of chloroquine are as follows :


I. Long term use II. Prolong used of High dose III. Nausea
• Loss of hearing graying of hear • Loss of vision • Vomiting
• Rushes • Retinal damage • Anorexia
• Photoallergy • Corneal deposit • Epigastric pain
• Mental disturbance • Headache
• Myopathy

On parenteral administration causes : • Hypotension


• Cardiac depression
• Arrhythmias
• CNS toxicity including convulsions.

526
Antimicr obial & Antineoplastic Dr
Antimicrobial ugs
Drugs 6
210. Ans. is a and b i.e. INH; and Rifampicin Ref. Harrison 15/e, p 433

Drug causing Hepatitis are :


Mnemonics = Jaundice Ne 6 Children On Table MARE

N Nitrofurantoin O OCP
C6 Chlorpropamide, T Tamoxifen
Chlorpromazine, M Methimazole
Calcium channel blockers A Anabolic steroids
Cyclosporine R Rifampicin
Carbamazepine E Erythromycin
Clavulinic acid

527
6 Self Assessment & Review Phar macology
Pharmacology

CHAPTER REVIEW
• This section includes questions of V arious Other PGMEES from 1990 – 2008.
Various
• Questions are ar ranged in increasing order of page sequence of KDT 6 Edition. This is
arranged
done to mak e refer
make ral system more easy and uncomplicated to save the precious time
referral
of PGMEE Aspirant.

1. Anthraquinoes causes : (Manipal 07) [Ref. KDT 6/e, p 840]


a. Adenoma 6. A post-operative patient develop septicemia and
b. Lung Ca was empiracally started on combination
chemotherapy by a new resident doctor. However,
c. Bladder Ca
when the patient did not respond even after 10
d. Ca oesophagus
days of antibiotics treatment, the review of the
[Ref. KDT 6/e, p] charts was done. It was found that the resident
2. All are aminoglycoside antibiotics except : doctor had started the combination of antibiotics
a. Linomycin (Manipal 07) which was mutally antagonistic in action. Which
b. Netilmycin one of the following is the most likely combination
c. Farmycetin that was given ? (Manipal 06)
d. Sisomycin a. Vancomycin and amikacin
[Ref. KDT 6/e, p 719] b. Cephelexin and gentamicin
3. All the drugs ar used in treatment of enteric fever c. Ampicillin and chloramphenicol
except : (Manipal 07) d. Ciprofloxacin and piperacillin
a. Vancomycin [Ref. KDT 6/e, p 677]
b. Cefoperazone 7. The most important target of action of
c. Refampicin chlorambucil is : (COMED 06)
d. Azithromycin a. Myeloid tissue
[Ref. KDT 6/e, p] b. Lymphoid tissue
4. All of the following anti-cancer agents causes bone c. Neural tissue
marrow depression except : (Manipal 06) d. Skin
a. Chlorambucil [Ref. KDT 6/e, p 822]
b. Daunorubicin 8. Which of the following drug is contraindicated in
c. Doxorubicin pregnancy ? (APPG 06)
d. Flutamide a. Chloroquine
[Ref. KDT 6/e, p] b. Primaquine
5. Which of the following statements is not true about c. Quinine
tarcolimus ? (Manipal 06) d. Amidoquine [Ref. KDT 6/e, p 791]
a. It is macrolide antibiotic 9. Chloroquine resistant malaria is treated by all
b. It is indicated for the porphylaxis of organ except : (Manipal 06)
transplant rejection a. Quinine
c. Glucose intolerance is a well recognized side b. Proguanil
effect c. Mefloquin
d. It can be safely administered with any d. Halofantrin
nephrotoxic [Ref. Goodman & Gilman 10/e, p 1098]

Answer 1. a. Adenoma 2. a. Linomycin 3. a. Vancomycin 4. d. Flutamide 5. None


6. c. Ampicillin ... 7. a. Myeloid ... 8. a. Chloroquine 9. b. Proguanil

528
Antimicr obial & Antineoplastic Dr
Antimicrobial ugs
Drugs 6
10. All of the following is true about “Imiquimod” 17. All are known side effects of INH except :
except : (APPG 06) a. Hepatitis (NIMHANS 06)
a. Direct antiviral activity b. Rash
b. Indirect antiviral c. Peripheral neuritis
c. Antitumor activity d. Flu syndrome
d. It release cytokines [Ref. KDT 6/e, p 741]
[Ref. Katzung 9/e, p 825; Goodman Gillman 11/ 18. Which of the following Antitubercular Drugs
e, p 1338] inhibits mycotic acid synthesis : (Orissa 05)
11. Which one of the statements is false regarding a. Rifampicin
adefovir dipivoxil ? (Karn 06) b. Pyrazinamide
a. Acyclic nucleotide analogue c. Ethambutol
b. Well tolerated orally d. Cycloserine
c. Used in chronic hepatitis C infection [Ref. KDT 6/e, p 742]
d. Used in anti-retroviral therapy 19. Which one of the following anti tuberculosis drugs
[Ref. Harrison 16/e, p 1848; Goodman Gillman does not have bactericidal action :
10/e, p 1339] a. Rifampicin (UPSC 05)
12. Treatment of choice of cryptococcus neoformans: b. Para-aminosalicylic acid
a. Amphoterecin B (HP 06) c. Pyrazinamide
b. INH d. Isoniazid
c. Ketoconazole [Ref. KDT 6/e, p 743]
d. Metronidazole 20. All of the following are complication of INH except:
[Ref. Harrison 16/e, p 1184] a. Psychosis (SGPGI 05)
13. Which one of the following drugs is useful in b. Optic neuritis
treating Crohn’s disease ? (Karn 06) c. Hepatitis
a. Infliximab d. Peripheral neuropathy
b. Azathioprine [Ref. KDT 6/e, p 741]
c. Tacrolimus 21. The drug that is not effective against
d. Cyclosporine streptococcus : (APPGE 05)
[Ref. Harrison 16/e, p 1786–1787] a. Ciprofloxacin
14. All the following anti-retroviral drugs produce b. Azithromycin
dyslipidemia except : (Karn 06) c. Chloramphenicol
a. Atazanavir d. Vancomycin
b. Saquinavir [Ref. KDT 6/e, p 732]
c. Amprinavir 22. All of the following drugs are indicated in the
d. Nelfmavir treatment of MRSA except : (SGPGI 05)
[Ref. Katzung 9/e, p 817–818] a. Ciprofloxacin
15. Treatment of mycobacteria avium complex is all b. TMP-SMZ
except : (APPG 06) c. Vancomycin
a. Ciprofloxacin d. Oxacillin
b. Clarithromycin [Ref. KDT 6/e, p 700, 732 - 733]
c. Rifabutin 23. Patients on amphotericin ‘β β’ therapy may develop
d. Pyrizinamide [Ref. KDT 6/e, p 749] signs and symptoms of deficiency of one of the
16. Antimicrobial of choice in pregnancy : cations. The cations is : (Karn. 05)
a. Clarithromycin (Manipal 06) a. Sodium
b. Streptomycin b. Potassium
c. Doxycycline c. Magnesium
d. Chloramphenicol d. Calcium
[Ref. KDT 6/e, p 730] [Ref. KDT 6/e, p 759]
Answer 10. c. Antitumor ... 11. c. Used in ... 12. a. Amphoterecin B 13. a. Infliximab 14. a. Atazanavir
15. d. Pyrizinamide 16. b. Streptomycin 17. d. Flu syndrome 18. b. Pyrazinamide 19. b. Para-...
20. a. Psychosis 21. d. Vancomycin 22. a. Ciprofloxacin 23. b. Potassium

529
6 Self Assessment & Review Phar macology
Pharmacology
24. Amphotericin should be administered in : b. Retroviral protease inhibitor
a. Glucose infusion (COMEDK 05) c. Nucleoside reverse transcriptase inhibitor
b. Mannitol infusion d. None of the above
c. Saline infusion [Ref. KDT 6/e, p 767]
d. Ringer lactate infusion 32. Flouroquinolones act by : (SGPGI 05)
[Ref. KDT 6/e, p 758] a. Binding to DNA gyrase and preventing DNA
25. Which one of the following antifungal agnets is synthesis
used only intravenously : (UPSC 05) b. Inhibiting protein synthesis
a. Fluconazole c. Inhibiting cell wall synthesis
b. Amphotericin d. Causing misreading of RNA
c. Amphotericin [Ref. KDT 6/e, p 688]
d. Ketozonazole 33. 4th generation cephalosporin is : (Manipal 05)
[Ref. KDT 6/e, p 758] a. Cefoxitin
26. How long does the protection from passive b. Cefixime
immunization using an immune globulin c. Cefepime
preparation typically last : (MAHA 05) d. Cefdinir
a. 1 to 3 days Ref. KDT 6/e, p 704]
b. 1 to 3 weeks 34. Clindamycin is useful in : (Manipal 05)
c. 1 to 3 months a. Toxoplasmosis
d. 1 tp 3 years b. Pneumocystis carinii
[Ref. Park 18/e, p 95] c. B. fragilis
27. 4th Generation Cephalosporin is : (Orissa 05) d. All of the above
a. Cefepime [Ref. KDT 6/e, p 731-732]
b. Cefuroxime 35. Which of the following pharmacologic agents is
c. Cefoperazone most effective in causing activation of phagocytes
d. Cefactor and production of oxygen metabolites that are
toxic microorganisms : (MAHA 05)
[Ref. KDT 6/e, p 704]
a. Dexamethasone
28. Drug which can be used safely in renal failure :
b. Interferon-alpha-2b
a. Crystalline penicilline (Manipal 05)
c. Interferon-beta-la
b. Aminoglycoside
d. Interferon-gamma-lb
c. Cephalothin
[Ref. KDT 5/e, p 59, 202; Goodman & Gilman, p
d. Methicillin
1478]
[Ref. KDT 6/e, p 673]
36. Drug safe in pregnancy with HIV infection is :
29. Mechanism of action of clavulanicacid :
a. Zidovudine (Orissa 05)
a. Binds DNA gyrase (Manipal 05)
b. Indinavir
b. Binds nucleic acid
c. Lamuvidine
c. Binds to bacterial cell wall
d. All of the above
d. Binds to β-lactam ring
[Ref. KDT 6/e, p 776 - 777]
[Ref. KDT 6/e, p 702]
37. Which one of the following statements is false
30. Rapid anti-malarial schizontocide : (Manipal 05) regarding Vincristine : (JIPMER 05)
a. Primaquine a. It is an alkaloid
b. Artemisinin b. It use is associated with neurotoxicity
c. Chloroquine c. It does not cause alopecia
d. Quinine d. It is useful drug for induction of remission in acute
[Ref. KDT 6/e, p 792] lymphoblastic leukaemia
31. Neverapine is : (Manipal 05) [Ref. KDT 6/e, p 825]
a. Non-nucleoside transcriptase inhibitor
Answer 24. c. Saline infusion 25. b. Amphotericin 26. b. 1 to 3 weeks 27. a. Cefepime 28. a. Crystalline ...
29. d. Binds to ... 30. b. Artemisinin 31. a. Non-nucle ... 32. a. Binding to ... 33. c. Cefepime
34. d. All of the ... 35. d. Interferon... 36. a. Zidovudine 37. c. It does ...

530
Antimicr obial & Antineoplastic Dr
Antimicrobial ugs
Drugs 6
38. Drug which is antifungal and anticancer : Ivermectin is used to treat it. It acts by following
a. Ketoconazole (AMU 05) neurotransmitter : (UPPGMEE 04)
b. Flucytosine a. Acetylcholine
c. 5-Flurouracil b. Adrenaline
d. Cytosine arabinoside c. Noradrenaline
[Ref. KDT 6/e, p 823 - 824] d. GABA
39. Actinomycin D acts by : (SGPGI 05) [Ref. KDT 6/e, p 813]
a. Inhibiting cell wall synthesis 46. Antineoplastic drug not acting at cell level :
b. Inhibiting protein synthesis a. Cisplatin (Jipmer 04)
c. Antagonising c-AMP b. Vincristine
d. Damaging DNA linkage in the cell c. Hydroxyurea
[Ref. KDT 6/e, p 826] d. Busulfan
40. Which is not used as a antiherpes drug : [Ref. KDT 6/e, p 819 - 820]
a. Acyclovir (AMU 05) 47. The antiretroviral drug which is also effective in
b. Nelfinavir chronic active hepatitis-B infection is : (Karn 04)
c. Ganciclovir a. Zidovudine
d. Foscarnet b. Nelfinavir
[Ref. KDT 6/e, p 767] c. Efacirenz
41. All of the following antimicrobial agents are d. Lamivudine
bactericidal except : (SGPGI 05) [Ref. KDT 6/e, p 771 - 772]
a. Cephalexin 48. Chloroquine high dose for long period can be give
b. Roxithromycin rise to : (APPGE 04)
c. Isoniazid a. Retinal damage
d. Rifampicin b. Corneal deposits
Ref. KDT 6/e, p 669, 740 for ‘c’] c. Aplastic anemia
42. Which of the following is most active against both d. All
Dormant and nondormant bacilli : (AMU 05) [Ref. KDT 6/e, p 786]
a. Pyrazinamide 49. Streptomycin acts by utilizing differential action
b. Rifampicin in prokaryote and eukaryoate of : (Orissa 04)
c. Streptomycin a. Translation
d. INH b. Transcription
[Ref. KDT 6/e, p 740] c. Transmethylation
43. Which of the following will be optimum treatment d. DNA interference
in a patient of chronic hepatitis B whose serum [Ref. KDT 6/e, p 711; Lippincott 2/e, p 39]
AST levels are arised : (MAHA 05) 50. Nucleoside analogues used in the treatment of HIV
a. Lamivudine infection include : (Karnataka 04)
b. Lamivudine + interferons a. Indinavir
c. Immunoglobulins b. Nevirapine
d. Interferons c. Didanosine
[Ref. KDT 6/e, p 772] d. Efavirenz
44. Drug of choice for Tropical Pulmonary Eosinophilia [Ref. KDT 6/e, p 767]
is : (Karnat 05) 51. True about of antifungal drug is all of the following
a. Diethylcarbamazipine (DEC) except : (Karnataka 04)
b. Corticosteroids a. Ketoconazole inhibits ergosterol synthesis
c. Ivermectin b. Ketoconazole causes adrenal insufficiency
d. None of the above c. Nystatin is used in Systemic disease
[Ref. KDT 6/e, p 813] d. Fluoconazole is used in candidiasis
45. Crusted scabies is caused by sarcoptis scabei [Ref. KDT 6/e, p 760]
Answer 38. c and d 39. d. Damaging ... 40. b. Nelfinavir 41. b. Roxithromycin 42. d. INH
43. a. Lamivudine 44. c. Ivermectin 45. d. GABA 46. None 47. d. Lamivudine
48. a. Retinal damage 49. a. Translation 50. c. Didanosine 51. c. Nystatin is ...

531
6 Self Assessment & Review Phar macology
Pharmacology
52. Vincristine arrests cell cycle at : (Jipmer 04) 59. The antibiotic known to cause “Red Man”
a. Metaphase syndrome is : (Karnataka 04)
b. Anaphase a. Clindamycin
c. Telophase b. Cefadroxil
d. Prophase c. Vancomycin
[Ref. KDT 6/e, p 825] d. Azithromycin
53. Immunosupressant : (Manipal 04) [Ref. KDT 6/e, p 732]
a. Steroids 60. Macrolide antibiotic which causes minimal
b. Cyclosporine hepatic cytochrome 450 involvement is :
c. Tacrolimus a. Erythromycin (Kerala 03)
d. All b. Roxithromycin
[Ref. KDT 6/e, p 837] c. Azithromycin
54. The anti tubercular drug safe in liver disease is : d. None
a. INH (UPSC 04) [Ref. KDT 6/e, p 731]
b. Rifampicin 61. Hemorrhagic cystitis is seen following the
c. Ethambutol administration of : (Manipal 03)
d. Pyrazinamide a. Adriamycin
[Ref. KDT 6/e, p 742] b. Cyclophosphamide
55. Which one of the following anbtibiotics is not c. Bleomycin
effective against pseudomonas aeruginosa : d. Busulphan
a. Cefatazidime (UPSC 04) [Ref. KDT 6/e, p 822]
b. Piperacillin 62. Cardiac toxicity is a side effect of : (Manipal 03)
c. Vanomycin a. Bleomycin
d. Aztreonam b. Adriamycin
[Ref. KDT 6/e, p 732; Harrison 15/e, p 967] c. Methotrexate
56. Drug that blocks “translation” : (Manipal 04) d. Daunorubicin
a. Quinolones [Ref. KDT 6/e, p 826]
b. Penicilins 63. Ototoxity is not a side effect of : (Manipal 03)
c. Tetracyclines a. Ampicillin
d. Cephalosporins b. Streptomycin
[Ref. KDT 6/e, p 668] c. Kanamycin
57. Chloroquine toxicity manifests as all except : d. Gentamycin
a. Myopathy (APPGE 04) [Ref. KDT 6/e, p 721]
b. Retinopathy 64. Drug which is NOT used for TB and leprosy is :
c. GI bleed a. Ofloxacin (Kerala 03)
d. Hypertension b. Rifampicin
[Ref. KDT 6/e, p 786, 792] c. Ethionamide
58. The following drugs are useful in the treatment of d. None
isoniazid poisoning : (Karnataka 04) [Ref. KDT 6/e, p 739 - 740, 751]
a. Pyridoxine 65. The drug that does’ not cause cholestatic jaundice
b. Diazepam is : (Manipal 03)
c. Bicarbonate a. INH
d. All of the above b. Chlorpromazine
[Ref. KDT 6/e, p 740 - 741, 801] c. O.C. pills
d. Erythromycin
[Ref. KDT 6/e, p 728]

Answer 52. a. Metaphase 53. d. All 54. c. Ethambutol 55. c. Vanomycin 56. c. Tetracyclines
57. d. Hypertension 58. d. All of the above 59. c. Vancomycin 60. c. Azithromycin 61. b. Cyclophos ...
62. d. Daunorubicin 63. a. Ampicillin 64. d. None 65. d. Erythromycin

532
Antimicr obial & Antineoplastic Dr
Antimicrobial ugs
Drugs 6
66. All the following regarding erythromycin are true 73. A 35 year old pilot has been detected to have
excepts : (Manipal 03) pulmonary tuberculosis. Which one of the
a. Its mainly excreted in bile following should not be prescribed for him :
b. It binds irreversitbly to 50S ribsomomal subunits a. Isoniazid (UPSC 02)
c. It is bactericidial b. Ethambutol
d. It is contraindicvated in patient with hepatic c. Pyrazinamide
dysfunction d. Streptomycin
[Ref. KDT 6/e, p 727- 728] [Ref. KDT 6/e, p 742]
67. Which of the following drug absorption is 74. Most common adverse effect of zidovudine is :
increased in Achlorhydria : (Jharkand 03) a. Hemolytic anemia (Manipal 02)
a. Ketoconazole b. Myopathy
b. Penicillin c. Bone marrow depression
c. Chloramphenicol d. Hepatotoxicity
d. Ciprofloxacin [Ref. KDT 6/e, p 771]
[Ref. KDT 6/e, p 762, 827] 75. The most important factor which limits the use of
68. All these drugs are potentially contraindicated in the combination of isoniazid, rifampicin and
children except : (Manipal 03) pyrazinamide in tuberculosis patient is :
a. Ciprofloxacin a. Ototoxicity (UPSC 02)
b. chloramphenicol b. Nephrotoxicity
c. Tetracyclines c. Bone marrow suppression
d. Azithromycin d. Hepatotoxicity
[Ref. KDT 6/e, p 730] [Ref. KDT 6/e, p 749]
69. Benzathine pencilin is the drug of choice in : 76. Mechanism of action of INH does not include :
a. Primary syphills (Manipal 02) a. Kills intracellular bacilli (Manipal 02)
b. Acute gonorhea b. Kills dormant bacilli
c. Bacterial endocarditis c. Most dangerous complication is peripheral
d. Meningococcal inferction neuritis
[Ref. KDT 6/e, p 698] d. It penetrates all body tissues
70. Antimalarial of choice in a pregnant woman (first [Ref. KDT 6/e, p 740]
trimester) who is Chloroquine resistant is : 77. MOA of clavulanic acid : (Manipal 02)
a. Quinine (JIPMER 02) a. b-lactamase inhibitor
b. Proguanil b. Non progressive inhibitor
c. Dapsone & pyrimethamine c. Extended spectrum penicillin
d. Mefloquine d. None of the above
[Ref. KDT 6/e, p 788; Laurence p 242; Dutta Obs. [Ref. KDT 6/e, p 702]
313 for ‘a’] 78. A feature of vancomycin : (Manipal 01)
71. Mebendazole is not the drug of choice for : a. It is glycopeptidases
a. instestinal amebiasis (Manipal 02) b. Rifampin
b. Asscariasis c. Ciprofloxacin
c. Ancylostona duodenale d. Ceftriaxone
d. Trichurea trichuris [Ref. KDT 6/e, p]
[Ref. KDT 6/e, p 809] 79. DOC for meningococcal meningitis is:
72. Chamydial infection can be tyreated by : a. Penicillin (Manipal 01)
a. Tetracycline (Manipal 02) b. Rifampin
b. Chloramphenicol c. Ciprofloxacin
c. Amoxycilin d. Ceftriaxone
d. Penicilin [Ref. KDT 6/e, p]
[Ref. KDT 6/e, p 715]
Answer 66. b. It binds ... 67. a. Ketoconazole 68. d.Azithromycin 69. a. Primary syphills 70. a. Quinine
71. a. instestinal ... 72. a. Tetracycline 73. b. Ethambutol 74. c. Bone ... 75. d. Hepatotoxicity
76. b. Kills ... 77. b. Non ... 78. a. It is ... 79. a. Penicillin

533
6 Self Assessment & Review Phar macology
Pharmacology
80. Ascariasis is not treated by : (Manipal 01) 87. Which of the following sulphonamides is least
a. Pyrantel pamoate absorbed : (Up 00)
b. Piperazine a. Sulphamethoxazole
c. Albendazole b. Sulfadiazine
d. DEC c. Sulfasaline
[Ref. KDT 6/e, p 702] d. Succinyl sulfathiazole
81. Mechanism of action of azathioprine is : [Ref. KDT 6/e, p 713]
a. Folate antagonis (Up 01) 88. All are true about fluoroquinolones except :
b. Purine antagonist a. Bactericidial (Up 00)
c. Pyrimidien antagonist b. Teratogenic
d. Vinca-alkaloids c. β-lactm-ring
[Ref. KDT 6/e, p 740] d. Photosensitivity
82. Biliary sludge is produced by : (Up 01) [Ref. KDT 6/e, p 688-689]
a. Ciprofloxacin 89. Which of the following acts on the pseudomonas
b. Ceftriazone aeruginaosa : (Up 00)
c. Carbenicillin a. Ceftazidime
d. Piperacillin b. Cefazoline
[Ref. KDT 6/e, p 1148] c. Cephalothin
83. Which is a protease inhibitor : (NIMHANS 01) d. Cephradine
a. Azothymidine [Ref. KDT 6/e, p 706]
b. Zidovidine 90. Cyclosporine A-act on : (Up 00)
c. Squanavir a. CD4+T-lymphocyte
d. Acyclovir b. CD3+L-Lymphocyte
[Ref. KDT 6/e, p 767 - 772] c. CD8+T-Lymphocyte
84. ALL of the following are used to treat leprosy d. CD5+lymphocyte
except : (J & K 0I) [Ref. KDT 6/e, p 837-839]
a. Ofloxacin 91. Side effect of amphotericin -B are all except :
b. Dapsone a. Hypokalemia (Up 00)
c. Clofazimine b. azotemia
d. Pyrimenthamine [Ref. KDT 6/e, p 751] c. Decreased-GFR
85. Which of the following antiviral drugs cannot be d. Neuropathy
given orally : (Kerala 00) [Ref. KDT 6/e, p 759]
a. Zidovudine 92. A patient treated with chloroquine 1.5gm for 3 day
b. Zalcitabine in P. vivex infection, after 4 months PBS shows
c. Acyclovir P.vivex Treatment is : (Up 00)
d. lodoxuridine a. Mefloquine
e. Amantadine b. Quinine
[Ref. KDT 6/e, p 770 for ‘a’, p 768 for ‘c’; Harrison c. Artimether
15/e, p 1098] d. Pyrimethamine + sulfonamides
86. Which of the following tetracycliens is metabolites [Ref. KDT 6/e, p 790]
through liver : (Up 00) 93. Which of the following quinolones acts on
a. Chloretetracycline anaerobes : (Up 00)
b. Demeclocycline a. palloxacin
c. Minocycline b. Ofloxacin
d. Tetracycline c. Lomefloxacin
[Ref. KDT 6/e, p] d. Sparifloxacin
[Ref. KDT 6/e, p 692]

Answer 80. d. DEC 81. b. Purine ... 82. b. Ceftriazone 83. c. Squanavir 84. d. Pyrimenth ...
85. d. lodoxuridine 86. d. Tetracycline 87. d. Succinyl ... 88. b. Teratogenic 89. a. Ceftazidime
90. a. CD4+T-... 91. d. Neuropathy 92. d. Pyrimethamine + ...93. d. Sparifloxacin

534
Antimicr obial & Antineoplastic Dr
Antimicrobial ugs
Drugs 6
94. Mechanism of action of chloramphenicol is : c. Hepatotoxicity
a. Binds to 50s units and inhibit protein synthesis d. Hypertension
b. Binds to 30s units and inhibit protein synthesis [Ref. KDT 6/e, p]
c. Inhibit bacterial cell wall synthesis (Up 00) 102. Amphotericin-B site of action is : (UP 99)
d. Inhibit bacterial DNA gyrase a. Cell wall
[Ref. KDT 6/e, p 716] b. Nucleus
95. DOC for cisplating induced vomiting : c. Cytoplasm
a. Ondasetron (Manipal 00) d. Mitochondria
b. Metoclopramide [Ref. KDT 6/e, p]
c. Domepridone 103. Anti-AIDS drug benefits by : (UP 99)
d. Prochlorperazine a. 3 viral replication
[Ref. KDT 6/e, p 702] b. 3 viral transmission
96. Drug of choice for Kala-azar : (MP 00) c. 3 viral lysis
a. Diethyl carbazine d. All of the above
b. Pentamidine [Ref. KDT 6/e, p]
c. Albendazole 104. All drugs are active against pseudomonas except:
d. Metronidazole a. Amikacin (UP 99)
[Ref. Harrison 16/e, p 1235 table (196.2)] b. Pipercillin
c Polymyxins-B
97. Drug used in chloroquine resistant malaria : d. Cephalosporins
a. Qunine (UP 99) [Ref. KDT 6/e, p 952]
b. Mefloquine 105. Oral penicillin is : (UP 99)
c. Pyrimethamine a. Penicillin G
d. Chloroquine b. Carbenicillin
[Ref. KDT 6/e, p 787] c. Mezlocillin
98. Cortisone causes A/E : (UP 99) d. Phenomythy penicillin
a. Na+ retention [Ref. KDT 6/e, p 699]
b. K+ excretion 106. Topically acting drug is : (UP 99)
c. Lipolysis a. Streptomycin
d. Glycogenolysis b. Neomycin
[Ref. KDT 6/e, p 277-279] c. Amikacin
99. Long term side effect of chloroquine is : (UP 99) d. Sisomicin
a. Hypotension [Ref. KDT 6/e, p 725]
b. Irreversible comeal deposit 107. Which is example of “dye” : (UP 98)
c. Retinopathy a. Acdepsone
d. Arrythmias b. Rifampicin
[Ref. KDT 6/e, p 786] c. clofazimine
100. Aminoglycosides S/E are A/E : (UP 99) d. Becampicilline
a. Hepatotoxic [Ref. KDT 6/e, p 752]
b. Ototoxic 108. Maximum enterohepatic circulation is seen in :
c. Nephrotoxic a. Vancomycin (UP 98)
d. Neuromuscular blockade b. Ciprofloxacin
[Ref. KDT 6/e, p ] c. Ampicilline
101. Most serious S/e of cyclosporine : (UP 99) d. Erythromycin
a. Bone marrow suppression [Ref. KDT 6/e, p 700]
b. Renal failure

Answer 94. a. Binds to 50s ... 95. a. Ondasetron 96. b. Pentamidine 97. b. Mefloquine 98. d. Glycogenolysis
99. c. Retinopathy 100. a. Hepatotoxic 101. b. Renal ... 102. a. Cell wall 103. a. 3 viral ...
104. c Polymyxins-B 105. a. Penicillin G 106. b. Neomycin 107. c. Clofazimine 108. c. Ampicilline

535
6 Self Assessment & Review Phar macology
Pharmacology
109. Adverse effect of cyclopsporine is : (UP 98) a. Cisplatin
a. Hirsutism b. Methotrexate
b. Macrocytic-anemia c. Bleomycin
c. Alopecia d. Cyclophosphamide
d. Stomatitis [Ref. KDT 6/e, p 834]
[Ref. KDT 6/e, p 839] 117. Synergism seen in A/E : (UP 96)
110. Which of the following drug acts by cell wall a. Penicilline + streptomycine
synthesis : (UP 98) b. Amphotericine + 5flucytosine
a. Penicilline c. Penicilline + chloretetracycline
b. Erythromycin d. Trimethoprim + Sulphemethaxole
c. Streptomycin [Ref. KDT 6/e, p 682]
d. Rifampicin 118. Mechanism of action of penicllline is : (UP 96)
[Ref. KDT 6/e, p 668] a. Cell wall synthesis
111. Gentamicin is obtained from : (UP 98) b. Inhibit protein synthesis
a. Streptomyces griseus c. Interefere with DNA function
b. Micromonospora israyli d. Leakage from cell membrane
c. Micromonospora purpurea [Ref. KDT 6/e, p 672]
d. Streptomyces fragilis 119. Which drug coloured all body fluide is : (UP 96)
[Ref. KDT 6/e, p 723] a. R-cin
112. Imipenem combined with cilastain is more useful b. Phenoplphthalein
because : (UP 98) c. Ethambutol
a. It inhibits the metabolism of imipenem d. Streptomycin
b. It increases more bioavailability of the drug [Ref. KDT 6/e, p 754]
c. It increases the synthesisi of the imipenem 120. Pulmonary fibrosis caused by : (UP 96)
d. It causes reversible inhibitor of dihydropeptidase a. Busulfan
[Ref. KDT 6/e, p 708] b. Thiacetazone
113. Which of the following acts by inhibiting the reverse c. Ethionamide
transcriptase : (UP 98) d. Propranolol
a. Zidovudins [Ref. KDT 6/e, p 829]
b. Amantadine 121. Gresiofulvin is used in A/E : (UP 96)
c. Acyclovir a. T. capitis
d. Trifluridine b. T. unguum
[Ref. KDT 6/e, p 770] c. Versicolar
114. Following drug are inhibitors of dihydrofolate d. T. pedis
reductase except : (UP 97)
[Ref. KDT 6/e, p 774]
a. Methotrexate
122. Streptomycin sulphate is NOT absorbed orally
b. Pyrimethamine because : (UP 95)
c. Pentamidine a. Greatly ionised at wider pH
d. Cytasine arabinose b. Degraded by Gastric pH
[Ref. Harrison 14/e, p 657] c. Insoluble in water
115. Erythromycin used in following disease except : d. Degraded by intestine enzyme
a. Epiglottis (UP 97) [Ref. KDT 6/e, p 722]
b. Campylobactor enteritis 123. All are used in kala-azar except : (UP 95)
c. Whooping cough a. Allopurinol
d. Legionnaire’s pneumonia b. Amphotericin - B
[Ref. KDT 6/e, p 741] c. Pentamidine
116. Most emetic chemothereputic agent is : (UP 96) d. Cycloserine [Ref. KDT 6/e, p 805]

Answer 109. a. Hirsutism 110. a. Penicilline 111. c. Micromon ... 112. d. It causes ... 113. a. Zidovudins
114. d. Cytasine ... 115. a. Epiglottis 116. a. Cisplatin 117. c. Penicilline + ... 118. a. Cell wall ...
119. a. R-cin 120. a. Busulfan 121. c. Versicolar 122. a. Greatly ... 123. d. Cycloserine

536
Antimicr obial & Antineoplastic Dr
Antimicrobial ugs
Drugs 6
124. Tetracycline is drug of choice in : (UP 95) c. Erythrocytic stage
a. Giardiases d. Gametocidal stage
b. Trichomoniasis [Ref. KDT 6/e, p 785]
c. Leishmaniasis 132. Procaine in procaine penicillin : (Kerala 95)
d. Balantidium coli a. Relieves pain
[Ref. KDT 6/e, p 715] b. Prolongs duration of action
125. III generation cephalosporin are more advantage c. Increases absorption
over II generation is due to : (UP 95) d. Adjuvant
a. More active against Gram -ve enterobacilli [Ref. KDT 6/e, p 357]
b. More active against gram +ve enterobacilli 133. To treat methotrexate toxicity .......... is used :
c. Less active against Gram +ve enterococci a. Folic acid (Jipmer 95)
d. Less active against gram +ve enterobacilli b. Folinic acid
[Ref. KDT 6/e, p 705] c. Riboflavin
126. Metronidizole used for : (UP 95) d. Cyanocobalamin
a. Gram +ve cocci [Ref. KDT 6/e, p 592; Harrison 16/e, p 476-477]
b. Gram -ve cocci 134. WhidrATT is used alone in prophylaxis : (UP 94)
c. Aerobic bacilli a. INH
d. Anaerobic bacilli b. R-cm
[Ref. KDT 6/e, p 748] c. Ethambutol
127. Treatment of choice for amoebic liver abscess : d. Pvnzinamide
a. Drainage + irraigation (UP 95) [Ref. KDT 6/e, p 463]
b. Drainage + metronidazole 135. Cyclophosphamide is NOT used in treatment :
c. Observation a. Clioriocarcinoma (UP 94)
d. Metronidazole + Tinidazole b. Breast carcinoma
[Ref. KDT 6/e, p 598] c. Testicular carcinoma
128. Example of antimetabolite is : (UP 95) d. Renal cell carcinoma
a. Cyclophosphamide [Ref. KDT 6/e, p 841]
b. Amethopterin 136. Which is NOT a aniino glycoside : (UP 94)
c. Chlorumbucil a. Framycitin
d. Busulpan b. Oleandromycin
[Ref. KDT 6/e, p 820] c. Neomycin
129. Penicillinase resistant drug is : (UP 95) d. Amikacin
a. Methicilline [Ref. KDT 6/e, p 719]
b. Cloxacilline 137. Which of the following does NOT inhibit cell wall
c. Ampicillin synthesis : (UP 94)
d. Amoxycilline a. Erythromycm
[Ref. KDT 6/e, p 699] b. β Lactam
130. Which drug is used with dapsone in multibacillary c. Vancomycin
leprosy is : (UP 95) d. Bacitracin
a. INH [Ref. KDT 6/e, p 668]
b. Ethambutol 138. Which is example of supra additive effect is :
c. Clofazemine a. Sulphamethoxazole + Trimethoprim (UP 94)
d. Pyrizenamide b. Paracetamol + Ibuprofen
[Ref. KDT 6/e, p 755] c. Halothane +N2O
131. Chloroquine is aginst with : (UP 95) d. Doxycyclme+cloxacillme
a. Pre erythrocytic stage [Ref. KDT 6/e, p 56]
b. Exoerythrocytic stage

Answer 124. d. Balantidium ... 125. a. More active ... 126. d. Anaerobic ... 127. b. Drainage + ... 128. b. Amethopterin
129. a and b 130. c. Clofazemine 131. c. Erythrocytic ... 132. b. Prolongs ... 133. b. Folinic ...
134. a. INH 135. a. Clioriocarc ... 136. b. Oleandromycin 137. a. Erythromycm 138. a. Sulphametho ...

537
6 Self Assessment & Review Phar macology
Pharmacology
139. Tetracycline of choice in CRF patinet is : (UP 94) 146. Which is not true about mebendazole :
a. Oxytetracycline a. Safe in pregnancy (AIIMS Dec. 94)
b. Chlortetracycline b. Broad spectrum antihelmintic
c. Doxycyclines c. Relatively low systemic bioavailability
d. Methacycline d. Active against both larva and adult worm
[Ref. KDT 6/e, p 713] [Ref. KDT 6/e, p 808-810]
140. Drug of choice for cyst passers : (UP 94) 147. Administration of which may lead to nero
a. Diloxanide furoate psychiatric symptom : (AIIMS Dec. 94)
b. Tinidazole a. Cycloserim
c. Emetine b. Ethionamide
d. Chloroquine c. Rifampicin
[Ref. KDT 6/e, p 80] d. Cephalosporin
141. One of the following drug is not effective against [Ref. KDT 6/e, p 757]
anaerobes : (AIIMS Dec. 94) 148. Which is not a third generation cephalosporin :
a. Penicilline a. Ceftriaxone (AIIMS Dec. 94)
b. Chloremphenicol b. Cefotaxime
c. Gentamicin-Penentration in cell is O2 c. Ceftizoxime
d. Clindamycin d. Cefuroxime 2nd gen oral
[Ref. KDT 6/e, p 731] [Ref. KDT 6/e, p 704]
142. Which of the following drug act against pseudo 149. All are true about contrimoxazole, except :
monas : (AIIMS Dec. 94) a. Resistance is delayed due to drugs in
a. Piperacilline combination (AIIMS May 94)
b. Methicilline b. Trimethoprim : Sulfamethoxazole = 1:5
c. Nafacilline c. Similar pharmacokinetic for both components
d. Cloxacilline d. Trimethoprim increases absorption of
[Ref. KDT 6/e, p 727] sulfamethoxazole
143. Which of the following is not a prodrug : [Ref. KDT 6/e, p 691]
a. L-dopa (AIIMS Dec. 94) 150. Which drug is NOT ototoxic : (UP 93)
b. Phenobarbitone a. Kanamycin
c. Sulphasalazine b. Neoniycin
d. Becampicilline c. Vincristinc
[Ref. KDT 6/e, p 21] d. Cisplatin
144. Which of the following drug is not used to treat [Ref. KDT 6/e, p]
leishmaniasis : (AIIMS Dec. 94) 151. Pyrimidine antagonist is : (UP 93)
a. Cyclosporin a. Mercaptopurine
b. Ketoconazole b. Busulfan
c. Pentamidine c. 5 FU
d. Amphotericin B d. Cyclophosphainidc
[Ref. KDT 6/e, p 812] [Ref. KDT 6/e, p 824]
145. Which of the following is not indicated in digitalis 152. Which of the following drugs is least effective as
poisoning : (AIIMS Dec. 94) an anti-inflammatory agent : (UP 93)
a. Potassium a. Plienylbutazone
b. Haemodialysis b. Acetaminophen
c. Phenytoin c. Aspirin
d. Lidocaine d. Indomethacin
[Ref. KDT 6/e, p 740] [Ref. KDT 6/e, p 187]

Answer 139. c. Doxycyclines 140. a. Diloxanide ... 141. c. Gentamicin-... 142. a. Piperacilline 143. b. Phenobarbitone
144. a. Cyclosporin 145. b. Haemodialysis 146. a. Safe in ... 147. a. Cycloserim 148. d. Cefuroxime ...
149. d. Trimethoprim... 150. c. Vincristinc 151. c. 5 FU 152. b. Acetaminophen

538
Antimicr obial & Antineoplastic Dr
Antimicrobial ugs
Drugs 6
153. Cyproterone acetate is : (UP 93) d. Melphalan
a. Antiprogestins [Ref. KDT 6/e, p]
b. Anti androgen 162. Melphalan is used in : (AIIMS May 93)
c. Antiestrogen a. Multiple myeloma
d. Any of the above b. Wilm’s tumour
[Ref. KDT 6/e, p 304] c. Neuroblastoma
154. Haemorrhagic. cystitis caused by : (UP 93) d. Retinoblastoma
a. Cyclophosphanude [Ref. KDT 6/e p 829]
b. Chlommbucil 163. Cyclosporine is active against : (AIIMS May 93)
c. Cytarbine a. CD4 lymphocyte
d. Carboplatin b. CD8 lymphocyte
[Ref. KDT 6/e, p] c. CD14 lymphocyte
155. Drug used in treatment of carcinoma thyroid is : d. B-lymphocyte
a. Doxyrubicin (AIIMS Nov. 93) [Ref. KDT 6/e, p 839]
b. 5 fluro uracil 164. Which of the following does not cause bone
c. Methotrexate marrow suppression : (AIIMS May 93)
d. Vinblastine a. Daunorobicin
[Ref. KDT 6/e, p] b. Cisplatinum less bone ……
156. Which of the following is cardiotoxic : c. Cyclophosphamide
a. Methotrexate (AIIMS Nov. 93) d. Vincristine 543 NH
b. Cyclophosphamide [Ref. KDT 6/e, p 668]
c. Adriamycin 165. Find the drug whose toxicity decrease healing
d. Vincristine power : (AIIMS Dec. 92)
[Ref. KDT 6/e, p 833] a. Thiotepa
157. Ciprofloxacin acts on : (AIIMS Nov. 93) b. Cyclophosphamide
a. DNA histone protein c. 5-FU
b. DNA gyrase d. Nitrogen mustard
c. Cy AMP [Ref. KDT 6/e, p 828]
d. m RNA polymerase 166. Metabolism of theophylline is decreased by
[Ref. KDT 6/e, p] a. Ampicillin (AIIMS Dec. 92)
158. Photosensitivity is a side effect of : b. Phenytoin
a. Ofloxacin (AIIMS Nov. 93) c. Rifampicin
b. Ciprofloxacin d. Ciprofloxacin
c. Norfloxacin [Ref. KDT 6/e, p 232]
d. Pefloxacin 167. Primaquin is not a : (AIIMS June 92)
[Ref. KDT 6/e, p] a. Primary tissue schizonticide
160. Neurocysticercosis is treated by all drug, except : b. Secondary tissue schizonticide
a. Albendazole (AIIMS May 93) c. Sporonticide
b. Niclosamide d. Gametocide
c. Flubendazole [Ref. KDT 6/e, p 801]
d. Praziquantal 168. DNA-dependent RNA polymerase of the bacteria
[Ref. KDT 6/e, p 817] a. INH inhibit mycolic acid synthe (AIIMS June 92)
161. Haemorrhagic cystitis is caused by : b. Streptomycin 305 ribosome
a. Cyclophosphamide (AIIMS May 93) c. Ciprofloxacin DNA
b. Bhusulphan d. Rifampicin
c. Prednisolone [Ref. KDT 6/e, p 741]

Answer 153. b. Anti ... 154. a. Cyclophos ... 155. a. Doxyrubicin 156. b. Cyclophos ... 157. b. DNA gyrase
158. d. Pefloxacin 160. b. Niclosamide 161. a. Cyclophos ... 162. a. Multiple ... 163. a. CD4 lymphocyte
164. b. Cisplatinum ... 165. d. Nitrogen ... 166. d. Ciprofloxacin 167. c. Sporonticide 168. d. Rifampicin

539
6 Self Assessment & Review Phar macology
Pharmacology
169. Antacids decreases the biovailability of all the 176. Nucleic acid synthesis is inhibited by :
following drugs, except : (AIIMS June 92) a. Ampicilin (Kerala 94)
a. Ofloxacin b. Sulpha drugs
b. Ketoconazole c. Griseofulvin
c. Oxytetracyclin d. Cephalosporins
d. Erythromycin [Ref. KDT 6/e, p 668, 683]
[Ref. KDT 6/e, p 638] 177. Sulpha drugs act by : (Kerala 94)
170. Which of the following is drug of choice for a. Covalent modification
intestinal amo-ebiasis : [June 1991] b. Non competitive inhibition
a. Choroquine ext c. Allosteric modification
b. Emetine highly toxic d. Competitive inhibition
c. Metronidazole [Ref. KDT 6/e, p 683]
d. Diloxanide furoate luminal amabi 178. Sulphamethoxazole is combined with :
[Ref. KDT 6/e, p 805] a. Trimethoprim (Kerala 94)
171. Mebendazole is used in all, except : (June 1991) b. Clotrimazole
a. Strongyloides stercoralis c. Cotrimoxazole
b. Hook worm d. Nitrofurantoin
c. Round worm [Ref. KDT 6/e, p 685]
d. Trichuris trichura 179. Ciprofloxacillin acts by inhibiting :
[Ref. KDT 6/e, p 819] a. Protein synthetsis (JIPMER 92, 93)
172. All of the following causes peripheral neuritis b. DNA gyrase
except : (June 1991) c. Cell wall synthesis
a. Vincristine therapy d. mRNA synthesis
b. INH therapy [Ref. KDT 6/e, p 668, 688]
c. Folic acid deficiency 180. DNA gyrase inhibitor : (JIPMER 99)
d. Chronic alcoholism a. Tetracyclines
e. dapsone b. Quinolones
[Ref. Harrison 17/e, p 2658] c. Monobactums
173. Most potent cardiac stimulant of following is : (6. d. Erythromycin
a. Epinephrine (June 1991) [Ref. KDT 6/e, p 668, 687 - 688]
b. Adrenaline 181. Nalidixic acid is used in : (Kerala 94)
c. Noradrenaline a. UTI
d. Salbutamol b. Bacillary dysentry
[Ref. KDT 6/e, p] c. Enteric fever
174. First generation cephalosporins are not effective d. Malaria
in ...... infection : (AI 90)
[Ref. KDT 6/e, p 687]
a. Pseudomonas
182. CNS side effects of ciprofloxacin are increased
b. Proteus by simultaneous intake of : (JIPMER 98)
c. Streptococci a. Salicylates
d. Staphylococci b. Ceftriaxone
[Ref. KDT 6/e, p 673; Harrison 16/e, p 883 for ‘b’] c. Nifedipine
175. Safest antibiotic in Renal failure : (PGI 93) d. Erythromycjn
a. Cloxacillin [Ref. KDT 6/e, p 689]
b. Streptomycin
c. Gentamicin
d. Cephaloridine
[Ref. KDT 6/e, p 673 - 674]

Answer 169. d. Erythromycin 170. c. Metronidazole 171. a. Strongyloides ... 172. c. Folic ... 173. b. Adrenaline
174. a. Pseudomonas 175. a. Cloxacillin 176. b. Sulpha ... 177. d. Competitive ... 178. a. Trimethoprim
179. d. mRNA ... 180. b. Quinolones 181. a. UTI 182. a. Salicylates

540
Antimicr obial & Antineoplastic Dr
Antimicrobial ugs
Drugs 6
183. Fluoroquinolones in children are contraindicated 190. Acid Labile penicillin is : (Jipmer 91)
as it causes : (CUPGEE 01) a. Cloxacillin
a. Arthropathy b. Flucloxacillin
b. Agranulocytosis c. Methicillin
c. Anemia d. Phenoxy Methyl penicillin
d. Thrombocytopenia e. None of the above
[Ref. KDT 6/e, p 691] [Ref. KDT 6/e, p 699]
184. Increased photosensitivity is seen with : 191. Drug of choice of MRSA infection : (Kerala 01)
a. Ciprofloxacin (Kerala 03) a. Cloxacillin
b. Sparfloxacin b. Amoxicillin
c. Floxacin c. Teicoplanin
d. Pefloxacin d. Carbenicillin
[Ref. KDT 6/e, p 692] [Ref. KDT 6/e, p 700; Harrison 16/e, p 821]
185. The penicillin with longest duration of action is : 192. Which is not useful in fungal infections :
a. Pottassium phenethicillin (DNB 91) a. Nystatin (Kerala 94)
b. Benethamine penicillin b. Clotrimozole
c. Procaine penicillin c. Gentian violet
d. Benzathine penicillin d. Ampicillin
[Ref. KDT 6/e, p 697] [Ref. KDT 6/e, p 700]
186. The ½ life of Benzyl Penicillin is : (CUPGEE 95) 193. Enterohepatic circulation is seen in : (TN 97)
a. 30 minutes a. Ampicillin
b. 60 minutes b. Benzylpenicillin
c. 90 minutes c. Norfloxacin
d. 24 hours d. Streptomycin
[Ref. KDT 6/e, p 696 - 697] [Ref. KDT 6/e, p 700]
187. Of the patients who have previously been given 194. Orally as well as parenterally effective penicillin
penicillin without incident, what percentage will is : (TN 90)
have an allergic reaction when given penicillin : a. Phenoxymethyl penicillin
a. Fewer that 1 % (MAHA 05) b. Amoxycillin
b. 1 to 5% c. Cloxacillin
c. 5 to 10% d. Carbenicillin
d. 10 to 15% [Ref. KDT 6/e, p 700 - 701]
[Ref. KDT 6/e, p 697] 195. Ampicillin is used in all except : (TN 91)
188. Most serious adverse effect of penicillin is : a. Urinary tract infection
a. Skin rashes (Kerala 94) b. Lower respiratory infection
b. Jarish hexheimer reaction c. Typhoid
c. Anaphylaxis d. Infectious mononucleosis
d. Convulsion [Ref. KDT 6/e, p 701, 756]
[Ref. KDT 6/e, p 697 - 698; Harrison 16 /e, p 877] 196. The Combination of amoxycillin+clavulanic acid is
189. Drug of choice for chemo prophylaxis against used because : (Delhi 96)
Rheumatic fever is : (Karn 05) a. It broadens the spectrum of amoxycillin
a. Penicillin V b. It decreases the side effect of amoxycillin
b. Erythromycin c. It decreases the toxicity of clavulanic acid
c. Benzathine penicillin d. It increases the oral bioavalability of amoxycillin
d. Procaine penicillin [Ref. KDT 6/e, p 702 - 703]
[Ref. KDT 6/e, p 697 - 698]

Answer 183. a. Arthropathy 184. b. Sparfloxacin 185. d. Benzathine ... 186. a. 30 minutes 187. a. Fewer that 1 %
188. c. Anaphylaxis 189. b. Erythromycin 190. c. Methicillin 191. c. Teicoplanin 192. d. Ampicillin
193. a. Ampicillin 194. b and c 195. d. Infectious ... 196. a. It broadens ...

541
6 Self Assessment & Review Phar macology
Pharmacology
197. Beta lactam ring is present in all except : [Ref. KDT 6/e, p 722]
a. Vancomycin (Jipmer 91) 204. False about streptomycin : (TN 98)
b. Imipenem a. Has poor bioavailability
c. Cephalexin b. Has high urine clearance
d. Sulbactam c. Extensively bound to plasma proteins
[Ref. KDT 6/e, p 694 - 709] d. Has poor enterohepatic circulation
198. Drugs used in neutropenic patients should [Ref. KDT 6/e, p 722]
be : (Kerala 94) 205. Which of the following aminoglycosides is most
a. Bactericidal resistant to bacterial inactivating enzymes?
b. Bacteriostatic a. Gentamycin (Karn 95)
c. Highly potent b. Amikacin
d. Be given by IV route c. Tobramycin
[Ref. KDT 6/e, p 724; Harrison 16/e, p 796] d. Kanamycin
199. Chloramphenicol inhibits protein biosynthesis by: [Ref. KDT 6/e, p 724]
a. Releasing the incomplete peptide material from 206. Aminoglycoside which is not ototoxic :
ribosomes (ICS 98) a. Tobramycin (Jipmer 91)
b. Inhibiting the binding of mRNA to ribosomes b. Amikacin
c. Inhibiting DNA polymerase c. Netilmycin
d. Inhibiting union of 60s and 40s lobes of d. Gentamycin
ribosomes to form full ribosome [Ref. KDT 6/e, p 719, 724; Katzung 5/e, 789–
[Ref. KDT 6/e, p 716] 790]
200. Streptomycin was discovered by : 207. Which of the following is not a Aminoglycoside :
a. Robert Koch (PGI 86; Kerala 90) a. Cycloserine (JIPMER 84; AMC 90)
b. Alexander Flemmings b. Framycetin
c. SA Wasksman c. Amikacin
d. Louis Pasteur d. Paramomysin
[Ref. KDT 6/e, p 719] [Ref. KDT 6/e, p 725]
201. Streptomycin single dose is more effective than 208. Which of the following aminoglycosides is not
multiple small doses because it is : (Jipmer 04) available for parenteral use ? (TN 95)
a. Concentration dependent lytic effect a. Sisomycin
b. Longer half life b. Amikacin
c. Not dependent on renal clearance c. Framycetin
d. Tubular secretion and reabsorption is more d. Gentamycin
[Ref. KDT 6/e, p 720] [Ref. KDT 6/e, p 719, 725]
202. Gentamycin, the amino glycoside antibiotic 209. Which of the following drug acts on “motilin”
produces nerve deafness by causing : (J & K 05) receptors : (SGPGI 04)
a. Demyelination of 8th cranial nerve fibres a. Erythromycin
b. Thickening of ear drum b. Tetracycline
c. Destruction of auditory ossicles c. Norfloxacin
d. Obstruction of mechanosensitive channels in the d. Chloramphenicol
stereocillia of hair cells [Ref. KDT 6/e, p 727]
[Ref. KDT 6/e, p 721] 210. Which antibiotic causes Torsade-de-pointes ?
203. Which one of the following ant T.B. drugs is a. Erythromycin (Burdwan 00)
contraindicated in pregnancy : (UPSC 05) b. Griseofulvin
a. Rifampicin c. Ciproflaxin
b. Isoniazid d. Amoxicilin
c. Streptomycin [Ref. KDT 6/e, p 645, 727 - 728]
d. Pyrazinamide
Answer 197. a. Vancomycin 198. a. Bactericidal 199. a. Releasing ... 200. c. SA Wasksman 201. a. Concentra ...
202. d. Obstruction ... 203. c. Streptomycin 204. c. Extensively ... 205. b. Amikacin 206. c. Netilmycin
207. a. Cycloserine 208. c. Framycetin 209. a. Erythromycin 210. a. Erythromycin

542
Antimicr obial & Antineoplastic Dr
Antimicrobial ugs
Drugs 6
211. Erythromycin is the drug of choice in the treatment 218. The least toxic antitubercular drug : (Kerala 90)
of childhood infection due to the following a. Thiacetazone
organisms except : (UP 97) b. Cycloserine
a. Pertussis c. Ethambutol
b. Mycoplasma pneumoniae d. Rifampicin
c. Pneumocystis carini [Ref. KDT 6/e, p 742]
d. Chlamydia trachomatis 219. Neuropsychatric manifestations is seen with the
[Ref. KDT 6/e, p 728; Harrison 15/e, p 1184 for following anti TB drug : (Kerala 97)
‘c’] a. Cycloserine
212. In a CRF patient does is reduced for all except : b. Ethamabutol
a. Polymyxin-B (MP 98) c. INH
b. Vancomycin d. Rifampicin
c. Amoxycillin [Ref. KDT 6/e, p 744]
d. Gentamycin 220. In tuberculosis, multidrug treatment is used for
[Ref. KDT 6/e, p 734; CMDT 2002 p 931] the following reason : (JIPMER 90)
213. Which aminoglycoside can’t be used as Anti TB a. Decrease drug resistance
drug : (JIPMER 98) b. Action at different level
a. Gentamycin c. Reduced complication
b. Amikacin d. All of the above
c. Tobramycin [Ref. KDT 6/e, p 746 - 747]
d. Streptomycin 221. The anti-tubercular drug which should not be used
[Ref. KDT 5l/e, p 739 - 740] in pregnancy is : (Delhi 96)
214. INH is metabolied in the body by : (Jipmer 93) a. Rifampicin
a. Acetylation b. Ethambutol
b. Conjugation c. Pyrazinamide
c. Hydroxylation d. Streptomycin
d. Decarboxylation [Ref. KDT 6/e, p 722, 748]
[Ref. KDT 6/e, p 740] 222. Short course chemotherapy is not used in any of
215. Flu like syndrome is a side effect of : the following except : (Karnat 96)
a. PAS (Jipmer 93; CUPGEE 96) a. HIV associated T.B.
b. Rifampicin b. Bone and joint T.B.
c. Pyrazinamide c. Central nervous system T.B.
d. Streptomycin d. T.B. lymphadenitis
[Ref. KDT 6/e, p 741] [Ref. KDT 6/e, p 747 - 748]
216. Resistance to drugs in Tuberculosis develops by: 223. Steroids are indicated in : (Karnat 96)
a. Transduction (Delhi 92) a. Miliary T.B.
b. Conjugution b. T.B. Meningitis
c. Transformation c. T.B. Spine
d. Mutation d. Endobronchial T.B.
[Ref. KDT 6/e, p 741] [Ref. KDT 6/e, p 749]
217. Which drug increases uric acid level : (UP 96) 224. In the treatment of tuberculosis, corticosteroid
a. Pyrazinamide therapy is indicated in all of the following except:
b. Rifampicin a. Progressive primary pulmonary tuberculosis
c. INH b. Miliary tuberculosis (UPSC 98)
d. Ethambutol c. Tubercular pericardial effusion
[Ref. KDT 6/e, p 742] d. Tubercular meningitis
[Ref. KDT 6/e, p 749]

Answer 211. c. Pneumocys ... 212. a. Polymyxin-B 213. a and c 214. a. Acetylation 215. b. Rifampicin
216. d. Mutation 217. a and b 218. c. Ethambutol 219. a. Cycloserine 220. d. All of the ...
221. d. Streptomycin 222. b, c and d 223. a and b 224. a. Progressive ...

543
6 Self Assessment & Review Phar macology
Pharmacology
225. Antifungal agent having affinity for keratin : 232. Which of the following antimalarials is relatively
a. Gresieofulvin (TN 99) safe in pregnancy : (Karn. 94)
b. Ketoconazole a. Primaquine
c. Nystatin b. Co-trimoxazole
d. Clotrimazole c. Chloroquine
[Ref. KDT 6/e, p 760] d. Pyrimethamine
226. The antiviral drug 5-iodo-2 deoxy uridine (IDU) [Ref. KDT 6/e, p 786]
inhibits the synthesis of : (Karn. 98) 233. Which of the following drug can be used without
a. DNA reducing the dosage in renal failure : (AI 91)
b. RNA a. Cefaclor
c. DNA and RNA b. Rifampicin
d. None of the above c. Erythromycin
[Ref. KDT 6/e, p 767] d. Doxycycline
227. Idoxuridine : (Karn 99) [Ref. KDT 6/e, p 673, 728, 720, 786; Harrison
a. Is an analogue of thymidine 16/e, p 795, 946]
b. Is too toxic for routine systemic use 234. Drug of choice for cerebral malaria is : (PGI 93)
c. Inhibits the replication of somerna-containing a. Metkalfin
viruses by causing the synthesis of fraudulent b. Quinine
visral orotein c. Chloroquine
d. Is useful as topical treatment for herpes simplex d. Primaquin
infection of the skin and cornea [Ref. KDT 6/e, p 788]
[Ref KDT 6/e, p 767 - 768] 235. Which is not cysticidal for entamoeba : (PGl 93)
228. Primary mechanism of action of Zidovudin is : a. Paramomycin
a. Proliferation of B & T cells (Karn. 94) b. Chloroquine
b. Inhibition of transcriptase c. Tetracycline
c. Activation of macrophages d. Diloxanide [KDT 6/e, p 738, 793]
d. Stimulation of lymphokines 236. Flu-like syndrome may be caused by the intake of:
[Ref. KDT 6/e, p 767, 770] a. Rifampicin (CUPGEE 96)
229. Interferon 3 to 5 million units thrice a week for b. Aspirin
three to six months is useful in the treatment of : c. INH
a. Wilson’s disease (ICS 98) d. Ranitidine
b. 1- antitrypsin deficiency [Ref. KDT 6/e, p 741, 801]
c. Chronic hepatitits B 237. Neurocysticercosis all can be given except :
d. Chronic autoimmune hepatitis a. Albendazole (JIPMER 98)
[Ref. KDT 6/e, p 779] b. Prazequantel
230. Chloroquine in malaria acts on : (Kerala 95) c. Niclosemide
a. Erythrocytic cycle d. Ivermectin
b. Exo erythrocytic cycle [Ref. KDT 6/e, p 809 t.(61.1); Harrison 16/e, p
c. Liver 1273, 1274]
d. Schizonts 238. Pyrantel pamoate is effective in both :
[Ref. KDT 6/e, p 785] a. Amoebiasis and trichuriasis (Kerala 97)
231. Chloroquine cause exacerbation of : (AI 91) b. Enterobius and ascariasis
a. Malaria c. Amoebiasis and strogyloides
b. Psorisasis d. Taenia solium and ascariasis
c. SLE [Ref. KDT 6/e, p 809, 811]
d. Photosensitivity
[Ref. KDT 6/e, p 786]

Answer 225. a. Gresieofulvin 226. a. DNA 227. a and b 228. b. Inhibition ... 229. c. Chronic ...
230. a. Erythrocytic ... 231. b. Psorisasis 232. c. Chloroquine 233. c and d 234. b. Quinine
235. c. Tetracycline 236. a. Rifampicin 237. c and d 238. b. Enterobius ...

544
Antimicr obial & Antineoplastic Dr
Antimicrobial ugs
Drugs 6
239. Which drug is immunomodulator : (DNB 01) 246. Drug which arrests mitosis in metaphase :
a. Levamisole a. Busulfan (AI 93)
b. Methotrexate b. 5-Fu
c. Ifosfamide c. Vinca Alkaloids
d. Procarbazine d. Methotrexate
[Ref. KDT 6/e, p 812] [Ref. KDT 6/e, p 824]
240. Which is not an antiviral drug : (Keala 94) 247. An antimetabolite which undergoes
a. Vidarbine biotransformation to form an inhibitor of DNA
b. Acyclovir polymerase is: (JIPMER 81; AIIMS 86, 90)
c. Zidovidin a. Vinablastine
d. Mitomycin b. Cytosine arabinoside
[Ref. KDT 6/e, p 820] c. Methotrexate
241. Which of the following is not an alkylating d. Chlorambucil
agent : (CUPGEE 96) [Ref. KDT 6/e, p 825]
a. Cytarabine 248. Griseofulvin is not useful in one of the following:
b. Nitrosoureas a. Tinea capitis (Kerala 90)
c. Thiotepa b. Tinea cruris
d. Melphalan c. Tinea versicolor
[Ref. KDT 6/e, p 820] d. Tinea pedis
242. The antifungal antibiotic used intravenously for [Ref. KDT 6/e, p 760 - 761]
systemic mycosis is : (TNPSC 00) 249. Which is wrong about griseofulvin : (Al 93)
a. Griseofulvin a. Antabuse like action
b. Nystatin b. Fat increases absoiption
c. Amphotericin c. Half life-48 hours
d. Hamvein d. Increased Phenobarbitone absoiption
[Ref. KDT 6/e, p 757 - 760, 821] [Ref. KDT 6/e, p 760 - 761]
243. MAO of amphotericin B : (St. Johns 02) 250. The drug used to treat epidermophyton infection
a. Acts on DNA is : (TN 95)
b. Acts on RNA a. Nystatin
c. Acts on sterols in membrane b. Griseofulvin
d. Alters protein synthesis c. Amphotericin B
[Ref. KDT 6/e, p 758 - 759, 820 - 821] d. Salicylic acid
244. The folic acid antagonist which has antineoplastic [Ref. KDT 6/e, p 760 - 761]
activity and extensively used as a chemothera 251. Which of the following antifungal drugs given orally
peutic agent is : (Delhi 96) is not useful in systemic fungal infections :
a. Pentamidine a. Griseofulvin (Kerala 00)
b. Methotrexate b. Flucytosine
c. Trimethoprim c. Fluconazole
d. Pyrimethamine d. Ketoconazole
[Ref. KDT 6/e, p 823] e. Irrconazole
245. Mitosis is inhibited by : (Kerala 91) [Ref. KDT 6/e, p 760 - 761]
a. 5-Fu 252. Maximum endocrinal side effects are seen with :
b. Methotrexate a. Ketoconazole (JIPMER 98)
c. Vincristine b. Fluconazole
d. Cyclophosphamide c. Miconazole
[Ref. KDT 6/e, p 825] d. Nystatin
[Ref. KDT 6/e, p 762 - 763, 827]

Answer 239. a. Levamisole 240. d. Mitomycin 241. a. Cytarabine 242. c. Amphotericin 243. c. Acts on ...
244. b. Methotrexate 245. c. Vincristine 246. b. 5-Fu 247. b. Cytosine ... 248. c. Tinea ...
249. c. Half life... 250. b. Griseofulvin 251. a. Griseofulvin 252. a. Ketoconazole

545
6 Self Assessment & Review Phar macology
Pharmacology
253. Which of the following anticancer antibiotics is c. Merozoites
cardiotoxic : (Karn. 94) d. Trophozoites
a. Mitomycin [Ref. KDT 6/e, p 784, 843]
b. Adriamycin 261. Treatment for Chloroquine resistant plasmodium
c. Bleomycin falciparum infection is : (Kerala 95)
d. Mithramycin a. Pyrimethamine
[Ref. KDT 6/e, p 826] b. Primaquine
254. Anticancer drug with maximum emetic effect is : c. Fancidar
a. Busulphan (Bihar 00) d. All
b. Chlorambucil [Ref. KDT 6/e, p 789, 850; Park 18/e, p 208–209]
c. Thiotepa 262. Chloroquine is used in all except : (TN 91)
d. Cisplatinum a. Intestinal amoebiasis
[Ref. KDT 6/e, p 827] b. Hepatic amoebiasis
255. All are true about Ketoconazole except : c. SLE
a. Antacids reduce absoiption (MAHE 98) d. Malaria
b. Extensively metabolized [Ref. KDT 6/e, p 781 - 787, 852]
c. Highly protein bound 263. Which one of the following is an immunostimu-
d. Rifampicin increases its activity lant : (Kerala 94; NIMHANS 88)
[Ref. KDT 6/e, p 76762 - 763, 826 - 827] a. Miconazole
256. Antifungal which is used orally only is : b. Levamisole
a. Fluconazole (CUPGEE 99) c. Ketonazole
b. Clotrimazole d. Albendazole [Ref. KDT 6/e, p 812, 874]
c. Itraconazole 264. Volume of distribution (Vd) for chloro quine is :
d. Econazole a. 5 -8L (Karnataka 01)
[Ref. KDT 6/e, p 827] b. 9-15L
257. The enzyme which is used as an anticancer drug: c. 100-650L
a. L-asparginase (TN 98) d. Above 1300L
b. Cytosine arabinoside [Ref. Goodman Gillama 10/e, p 1079]
c. Methotrexate 265. Chloroquine in large doses causes : (AP 97)
d. Vincristine a. Irreversible retinopathy
[Ref. KDT 6/e, p 820, 827] b. Cataract
258. Antiviral drugs are : (PGI 90) c. Corneal deposits
a. Ketoconazole d. Glaucoma
b. Amantidine [Ref. Goodman & Gilman 10/e, p 1080]
c. Acyclovir 266. The cells most commonly affected by silver
d. Flucytosine sulfadiazine cream are : (JIPMER 80, 81; TN 90)
[Ref. KDT 6/e, p 757, 767, 832] a. Lymphocytes
259. Which is chemotheraputic agent that must be b. Neutrophils
included in treatment of ovarian carcinoma : c. Mast cells
a. Methotrexate (JIPMER 95) d. Eosinophils
b. Cyclophosphamide [Ref. Goodman & Gilman 10/e, p 1175]
c. Flourouracil 267. Which is true of Bacitracin : (Jipmer 9l)
d. Pro carbazine a. Poly peptide
[Ref. KDT 6/e, p 832; Harrison 16/e, p 555, 556] b. Systemic antifungal
260. Malarial relapse is due to : (JIPMER 95) c. Derived from bacteria
a. Sporozoites d. Effective against gram negative organism
b. Hypnozoites [Ref. Goodman Gillman 11/e, p 1265]

Answer 253. b. Adriamycin 254. d. Cisplatinum 255. d. Rifampicin ... 256. c. Itraconazole 257. a. L-asparginase
258. b and c 259. b. Cyclophosph ... 260. b. Hypnozoites 261. b and c 262. a. Intestinal ...
263. b. Levamisole 264. c. 100-650L 265. a. Irreversible ... 266. b. Neutrophils 267. a an c

546
Antimicr obial & Antineoplastic Dr
Antimicrobial ugs
Drugs 6
268. The following are the toxicity of cyclosporin except: c. Ciprofloxacin
a. Nephrotoxicity (AIIMS 80; Kerala 90) d. Cefazoline
b. Hypokalemia [Ref. Harrison 16/e, p 900]
c. Tremor 273. Drug of choice for plague is : (CUPGEE 95)
d. Hirsutism a. Tetracycline
[Ref. Goodman & Gilman 11/e, p 1469] b. Chloramphenicol
269. Drug of choice in Gonorrhoea is : (Jimper 92) c. Streptomycin
a. Norfloxacillin d. Any of the above
b. Doxycycline [Ref. Harrison 16/e, p 925]
c. Sulfonamide 274. Drug of choice for cholera prophylaxis :
d. Penicillin a. Chloramphenicol (Kerala 96)
[Ref. Harrison 16/e, p 861] b. Procaine penicillin
270. Drug of choice for penicillinase producing c. Doxycycline
gonocococcus : (CUPGEE 95) d. Erythromycin
a. Benzathine Penicillin G e. Septran
b. 3rd generation cephalosporin [Ref. Harrison 15/e, p 984; Park 18/e, p 182]
c. Quinolines 275. Drug of choice for P.Carnii : (Assam 95)
d. None a. Cotrimoxazole
[Ref Harrison 16/e, p 861] b. Erythromycin
271. Drug of choice in pseudomonas aeroginosa is all c. Gentamycin
except : (Rajasthan 97) d. Pentamidine
a. Carbenicillin [Ref. Harrison 16/e, p 1195]
b. Azlocillin 276. Erythromycin is the drug of choice for all except:
c. Meziocillin a. Diptheria (AI 91)
d. Moxalactum b. Whooping cough
[Ref. Harrison 16/e, p 894 table (136.3)] c. Pneumocystitis carinii
272. The drug of choice for chloramphenicol resistant d. Chlamydia trachomatis
typhoid fever is : (AI 91; Jipmer 93) [Ref. Harrison 16/e, p 835 for ‘a’, p 877 for ‘b’, p
a. Cefaclor 1195 for ‘c’]
b. Cefotaxine

Answer 268. b. Hypokalemia 269. d. Penicillin 270. b and c 271. d. Moxalactum 272. c. Ciprofloxacin
273. c. Streptomycin 274. c. Doxycycline 275. a. Cotrimoxazole 276. c and d

547
NOTES
7 Self Assessment & Review Phar macology
Pharmacology

550
Miscellaneous 7
ALL INDIA AIIMS

1. Drug coloring Urine are all except : [AI 08] 8. Hypercalcemia is caused by all except :
a. Rifampicin a. Phenytoin [AIIMS Nov. 07]
b. Quinine b. Lithium
c. Nitrofurantoin c. Vitamin D intoxication
d. None of the above d. Thiazide
2. Which of the following drugs causes constipation? 9. Dose of all is reduced in renal failure, except :
a. Propranolol [AI 06] a. INH [AIIMS June 98]
b. Verapamil b. Rifabutin
c. Nitroglycerin c. Ethambutol
d. Catopril d. Kanamycin
3. BAL is useful in treating poisoning due to all ex- 10. Dose of which drug is not altered in CRF :
cept? [AI 04] a. Rifampicin [AIIMS June 97]
a. Lead b. Gentamicin
b. Organic Mercury c. Tetracycline
c. Cadmium d. Cephalosporins
d. Arsenic 11. Gynaecomastia is a side effect of all, except :
4. The most appropriate drug used for chelation a. Digitalis [AIIMS June 97]
therapy in beta thalassemia major is : [AI 03]
b. Ketoconazole
a. Oral desferrioxamine
c. Rifampicin
b. Oral deferiprone
d. Spironolactone
c. Intramuscular EDTA
12. Hyperuricemia is a side effect of :
d. Oral Succimer
a. INH [AIIMS June 97]
5. Which of the following is correctly matched :
b. Rifampicin
a. Dimercaprol : Iron [AI 02]
c. Streptomycin
b. Calcium di-sodium EDTA : Arsenic
d. Pyrazinamide
c. Penicillamine : Copper
13. Pneumopathy is a side effect of all except :
d. Desferrioxamine : Lead
a. Alpha methyldopa [AIIMS June 97]
6. The primary mechanism of action of fluoride on
topical application is : [AI 02] b. Busulphan

a. Conversion of hydroxyapatite to fluoroapatite by c. Melphalan


replacing the OH ions d. Nitrofurantoin
b. Inhibition of plaque bacteria 14. All are true regarding prazosin, except :
c. Form a reservoir in saliva a. First dose effect [AIIMS Feb. 97]
d. Improvement in tooth morphology b. Impaired gluconse tolerance
7. Drug containing two sulfhydryl groups in a mol- c. Selective alpha-1 blocker
ecule : [AI 01] d. Is an antihypertesive drug
a. BAL
b. EDTA
c. Pencillamine
d. None of the above

Answer 1. a. Rifmpacin 2. b. Verapamil 3. c. Cadmium 4. b. Oral defe ..... 5. c. Penicillamine


6. a. Conversion ... 7. a. BAL 8. a. Phenytoin 9. b. Rifabutin 10. a. Rifampicin
11. c. Rifampicin 12. d. Pyrazinamide 13. a. Alpha ... 14. b. Impaired ...

551
7 Self Assessment & Review Phar macology
Pharmacology

PGI d. Ranitidine
e. Methyldopa
15. Not correctly matched : [PGI Dec. 07] 22. Hyperkalemia is associated with : [PGI June 04]
a. Phenytoin - hypertrichosis a. ACE inhibitors
b. Fenoldapam - hypertensive emergency b. Chlorthliadone
c. Fosinopril - produgs c. Amphotericin-B
d. Glulisine - long acting insulin d. Amiodarone
16. Drugs causing gynaecomastia : e. Rifampicin
a. Ketoconazole [PGI Dec. 07] 23. Drugs avoided in seizure are : [PGI June 04]
b. Spironolactone a. Ciprofloxacin
c. Phenytoin b. Cycloserine
d. Warfarin c. Glucocorticoids
e. Prazosin d. Ketoconazole
17. Drugs causing hypokalemia : [PGI Dec. 06] 24. Drugs causing pulmonary fibrosis : [PGI June 04]
a. Amphotericin B a. Bleomycin
b. Insulin b. Mitoxanthrone
c. Cyclosporine c. Byspirone
d. Carbenoxolone d. Nitrofurantin
e. NSAIDs 25. Bone marrow depressive drugs in the treatment
of AIDS patient are : [PGI June 03]
18. Hyperkalemia is caused by : [PGI Dec. 06]
a. Didanosine
a. Amphotincin B
b. Zalcitabine
b. ACE - Inhibitors
c. Dapsone
c. Cyclosporine
d. Cortimoxazole
d. GM -CSF
e. Gancyclovir
e. Succinylcholine
26. Drugs causing nephrotoxicity : [PGI Dec. 02]
19. Drugs which is given I.V. : [PGI June 05]
a. Gentamycin
a. Heparin
b. Cloxacillin
b. Pantoprazole
c. Phenacetin
c. Ranitidine
d. Erythromycin
d. Sumatriptan
27. Drugs causing cholestatic jaundice :
e. Neomycin
a. Estrogen [PGI Dec. 02]
20. Drugs causing osteoporosis : [PGI June 05]
b. Cyclosporine
a. Vit. K
c. INH
b. Lithium
d. Phenothiazine
c. Dilantin
e. Etheambutol
d. Heparin
28. Pleural fibrosis caused by : [PGI June 02]
e. Etidronate
a. Phenytoin
21. Drugs causing SLE are : [PGI June 05]
b. Methysergide
a. Isoniazid
c. Amiodarone
b. Hydralazine
d. Ergotamine
c. Procanamide
e. Ranitidine

Answer 15. d. Glulisine ... 16. a, b and c 17. a, b and d 18. b, c and e 16. a, b and c
20. b and d 21. a, b, c and d 22. a. ACE inhibitors 23. a, b and c 24. a and d
25. c, d and e 26. a and c 27. a, b and d 28. b and c

552
Miscellaneous 7
29. Motor Neuropathy caused by : [PGI June 02] b. Minocycline
a. Dapsone c. Sulfonamides
b. Cisplatin d. Gold
c. Arsenic e. Rifampicin
d. Lead 33. FK - 506 is used in : [PGI June 01]
e. Hypothyrodism a. Oragan transplant
30. Urine is colored by : [PGI June 02] b. Bronchial asthma
a. Thiamine c. Diabetic diarrhoea
b. Rifampicin d. Chemosterapecutic agen
c. Mepacrine 34. Free radical scavengers are : [PGI June 01]
d. INH a. Vit-C
e. Riboflavin b. Vit-E
31. Which drug is implicited in the causation of neph- c. Vit-A
rotic syndrome : [PGI Dec. 01] d. Glutathione
a. Gold e. Iron
b. Amphoterecin B 35. Drugs causing contipation are : [PGI June 01]
c. Rifampicin a. Verapamil
d. Ibuprofen b. Quinidine
e. Captopril c. MAO-Inhibitor
32. Skin pigmentation occurs with which of the fol- d. Tricylic antidepressants
lowing drugs : [PGI Dec. 01]
e. Ferrous sulphate
a. Clofazimine

Answer 29. a, c and d 30. b and e 31. a, d and e


32. a and b 33. a. Organ ... 34. a, b, c and d
35. a, d and e

553
7 Self Assessment & Review Phar macology
Pharmacology

ANSWERS, REFERENCES, EXPLANATIONS WITH INFORMATIVE ILLUSTR ATIONS

1. Ans. is a i.e. Rifampicin Ref. Goodman & Gilman 11/e, p 1208; KDT 5/e, p 700

Patient on rifampicin should be warned abut highly colored (red, orange or pink) urine and pink tears
while on rifampicin.
Nitrofurantoin deepens the normal yellow colour of urine.
Rifampicin is distributed throughout the body and is present in effective concentrations in many organs and
body fluids, including the CSF. This is best exemplified by the fact that the drug may impart an orange-red color
to urine, feces, saliva, sputum, tears and sweat, the patients must be warned.

Substances causes colouring of urine :


Substances Urine colour
Rifampicin Orange
β-caroteen Orange
Vit. B supplement Orange
Pirydium Orange
Methylene blue Green urine or blue urine
Phenophthilene Red colour

2. Ans. is b i.e. Verapamil Ref. KDT 6/e, p 530; Harrision 15/e, p 432 (table 71.2)

With verapamil, nausea, constipation and bradycardia are more common than other CCBs (Calcium channel
blockers), while flushing, headache and ankle edema are less common.
• This question has been directly picked up from Harrison’s 15/e, p 432 table. Since, these tables have now
been deleted from newer edition of Harrison; I will recommend you to go through these tables.
• Here is a quick glance on the drugs.
Drug Causing

Constipation Diarrhoea
• Aluminium hydroxide • Antibiotics (broad spectrum)
• Barium sulphate • Clindamycin
• Calcium carbonate • Cocaine
• Ferrous sulphate • Colchicine
• Ganglion blockers • Digitalis
• Ion exchange resins : • Guanethedine
– Cholestyramine • Lactose exipients
– Colestipol • Lincomycin
• Opiates • Magnesium in antacids
• Phenothiazines • Methyldopa
• Tricyclic antidepressants • Misoprostol
• Verapamil • Oral contraceptives
• Purgatives
• Reserpine
• Ticlopidine

554
Miscellaneous 7
Comments
• Verapamils is phenyl alkylamine, hydrophilic papaverine congener
• Nifedepine is a dihydropyridine
• Diltiazem is a hydrophilic benzothiazepine

About the dihydropyridines

• Felodipine once daily administration


• Amlodipine small diurnal fluctuation
• Nitrendipine long acting vasoselective
• Lacidipine once daily vasoselective
• Nimodipine penetrates blood-brain barrier

3. Ans. is c i.e. Cadmium Ref. KDT 6/e, p 865 - 868

Chelating Route of Used for Contraindicated Specific Points


Agent Administration (Not Used)
Dimercaprol IV All heavy Metals Perfectly contrain- As adjuvant to EDTA
(BAL) Specific for Hg dicated in=Iron and in lead poisoning
Eg -, Au, Bi, Sb Cadmium poisoning Penicillamine in
Cu poison
Desforrioxamine IV / IM Iron chelator Transfusion Siderosis
Deferipone Oral and Iron Fe load in liver
injectable Cirrhosis
Doxrazoane Iron Doxorubcin induce
cardiomyopathy
Calcium IV Specific for lead Not used in Hg
disodium Poisoning Also in, poisoning
edetate Cd, Mn, Cu, Fe
EDTA Poisoning
Penicillamine Oral Specific for Cu/Hg Adjuvant to EDTA
Poisoning Wilson disease now
II Doc to zinc
Trientine Oral Cu poisoning Willson disease
disease
Zinc acetate Oral Specific for
Willson disease

4. Ans. is b i.e. Oral deferiprone Ref. KDT 6/e, p 867 - 868

Route Uses Specific Points

Desforrioxamine IV / IM Iron chelator Transfusion Siderosis


Deferipone Oral and injectable Iron Fe load in liver, Cirrhosis

555
7 Self Assessment & Review Phar macology
Pharmacology
5. Ans. is c i.e. Penicillamine : Copper Ref. KDT 6/e, p 867

Route Uses Specific Points


Penicillamine Oral Specific for Cu/Hg Adjuvant to EDTA
Poisoning Wilson disease now
II Doc to zinc

6. Ans. is a i.e. Conversion of hydroxyapatite to fluoroapatite by replacing the OH ions


Ref. Essentials of Preventive and Community Dentisry Dr. Soben Peter 1/e, p 220

• When hydroxypetite is exposed to low fluoride concentration (about 1ppm) a layer of fluoroapetite forms on
the hydroxyapetite crystals. The thin layer governs the rate of enamel dissolution and reduces rate of caries
progression.
• During caries attack, the enamel is exposed to a acidic pH of about 5.5 or lower, thereby leading to its
dissolution.
• The presence of fluoride reduces the solubility of enamel by promoting the precipitation of hydroxyapetite
and phosphate mineral.
Ca10 (PO) 6 (OH)2 + 2F → Ca10 (PO) 6F2 + 20H (OH ions aer replace)
(hydroxy-apetite) (fluoro-apetite)

Agents proved to be most active in inhibiting dental caries are :


• Neutral sodium fluoride (NaF).
• Acidulated phosphate fluoride (APF).
• Stannuous fluoride (SnF2).

7. Ans. is a i.e. BAL Ref. KDT 6/e, p 865 - 866

Dimercaprol (British Antil Lewisite; BAL)


• BAL (Dimercapral) contains two SH groups and it bind those metals which produce their toxicity by interacting
with sulfhydryl containing enzymes in the body, i.e. As, Hg, Au, Bi, Ni, Sb, Cu.
• The complex of 2 molecules of dimercaprol with one metal ion is more stable than 1:1 complex.
• The dimercaprol metal complex spontaneously disociates releasign the metal at a slow rate; also dimercaprol
is partly oxidized in the body : further emphasizing the necessity to have excess dimercaprol available.

8. Ans. is a i.e. Phenytoin Ref. KDT 6/e, p 405; Goodman & Gilman 11/e, p 510

Phenytoin causes hypocalcemia and osteomalacia by desensisitizing target tissues to vit. D and interferes
with calcium metabolism. Rest of the drugs causes hypercalcemia.

9. Ans. is b i.e Rifabutin Ref. Harrison 16/e, p 947

• Out of all the antimycobacterial drugs, Rifabutin and Rifampicin are the safest drugs which can be used in
CRF.

556
Miscellaneous 7
• Ethambutol is also safe, but if both Rifabutin and Ethambutol are given in options, Rifampicin is choosen.
• Rifabutin is a safe drug in CRF and dose need not be decreased. In contrast, the other 3 drugs mentioned
are nephrotoxic in one or the other way and the dose has to be decreased in presence of renal failure.

10. Ans. is a i.e. Rifampicin Ref. Harrison 16/e, p 647

Use of antituberclar agents in patients with Renal or Hepatic disease and in Pregnant women are given below.
Pregnancy • Do not give Streptomycin – Permanent deafness in baby

Renal Failure • Rifampicin, Isoniazid, Pyrazinamide are safe.


• Avoid Streptomycin and Ethambutol
• Do not give Thiacetazone
Liver Disease • Safe – Streptomycin and Ethambutol
• Do not give Pyrazinamide
• Avoid – Rifampin

11. Ans. is c i.e. Rifampicin Ref. KDT 5/e, p 433

Gynaecomastia is a side effect of the following drugs :

• Ca channel antagonist • Isoniazid • Clomiphene


• Ketoconazole • Digitalis • Methyldopa
• Estrogen • Phenytoin • Ethianomide
• Reserpine • Griseofulvin • Spironolacton
• Testosteron

12. Ans. is d i.e. Pyrazinamide Ref. Harrison 15/e, p 432

Sulfinpyrazone is a uricosuric drug, so it cannot cause hyperuricemia (GOUT).

Drug causing hyperuricemia :


Mnemonics : LEAD Poisoning Causes Stones and Nephropathy -
L Levodopa
E Ethacrynic acid
A Alcohol
D Diuretics (Chlorthalidone)
P Pyrazinamide
C Cyclosporine (Cytotoxin)
S Salicylates (low dos;pe Aspirin)
N Nicotinic acid

13. Ans. is a i.e. Alpha methyldopa Ref. Harrison 15/e, p 427

• This question has beeen repeated many times before and the 2 most common drugs causing pulmonary
fibrosis are 2Bs : – Bleomycin and Busulphan.

557
7 Self Assessment & Review Phar macology
Pharmacology

Drugs causing Pulmonary Fibrosis :


• Acyclovir • Amiodarone
• Bleomycin • Busulphan
• Carmustine • Cyclophosphamide
• Melphalan • Methotrexate
• Methysergide • Nitrofurantoin
• Sulphonamides

14. Ans. is b i.e. Impaired gluconse tolerance Ref. KDT 6/e, p 134

Prazosin is the first of highly selective α1 blocker. It blocks sympathetically mediated vasoconstriction
and produces fall in BP which is attended by only mild tachycardia.
• Prazosin dilates arterioles more than veins.
– Postural hypotension occurs, specially in the beginning – dizziness and fainting as ‘First dose effect’.
This can be minimized by starting with a low dose and taking it at bed time.
• ACE inhibitors induce hypotension but postural hypotension is not a problem.
• Nifedipine also induce hypotension. Postural hypotension may occur but is not as prominent as with prazosin.
• Atenolol is a selective β1 antagonist and does not dilate vessels.

15. Ans. is d i.e. Glulisine - long acting insulin Ref. Goodman Gilman 11/e, p 249, 803, 1626

Option “a” • Phenytoin causes hypertrichosis.


• Hirsutism (in females) and acne may appear due to increased secretion of androgens.
Option “b” • Fenoldapam is a immediate acting drug used in malignant hypertension. ... GG - 249
Option “c” • Fosinopril is an ACE inhibitor, which is prodrug. ... GG - 803
• ALL ACE inhibitors are produgs except : C – Captopril
L – Lisinopril
Option “d” • Glulisine is an immediate acting insulin analogue. ... GG - 1626

16. Ans. is a, b and c i.e. Ketoconazole; Spironolactone; and Phenytoin Ref. KDT 5/e, p 433

Mnemonic : DISCOS – • Digoxin


• Isoniazid
• Spironolactone
• Cimetidine
• Oestrogens

Gynaecomastia is a side effect of the following drugs :


• Ca channel antagonist • Isoniazid • Clomiphene
• Ketoconazole • Digitalis • Methyldopa
• Estrogen • Phenytoin • Ethianomide
• Reserpine • Griseofulvin • Spironolacton
• Testosteron

558
Miscellaneous 7
17. Ans. is a, b and d i.e. Amphotericin B; Insulin; and Carbenoxolone
Ref. Harrison 15/e, p 425; 16/e, p 261

Drugs causing Hypokalemia

• Alkali induced alkalosis • Insulin • Amphoterecin B


• Laxatives • Carbenoxolone • Osmotic diuretics
• Corticosteroide • Tetracyclines • Diuretics
• Theophylline • Gentamicin • Vit. B12

18. Ans. is b, c and e i.e. ACE - Inhibitors; Cyclosporine; and Succinylcholine


Ref. Harrison 15/e, p 425; 16/e, p 261

Drugs causing Hyperkalemia

• ACE inhibitors • NSAIDs • Amiloride


• Pentamidine • Cyclosporine • Pot. salts of drugs
• Cytotoxics • Spironolactone • Digitalis overdose
• Succinylcholine • Heparin • Triamterene
• Lithium • Trimethoprim

19. Ans. is a, b and c i.e. Heparin; Pantoprazole; and Rantidine Ref. KDT 6/e, p 598, 633, 630, 171, 725

• Heparin is a large ionized molecule, therefore not absorbed orally. Injected I.V. it acts instantaneously but
after subcutaneous injection, anti coagulant effect develops after 60min. It does not cross blood brain barrier
or placenta. It is the anticoagulant of choice during pregnancy.
• Pantoprazole is a newer H+ K+ ATPase inhibitor. It is the only proton pump inhibitor available for I/V
administration.
• Ranitidine is H2 blocker which can be given orally, I/M or I/V.
• Sumatriptan can be used by oral, nasal spray or subcutaneous route.
• Neomycin is highly toxic drug, can be used orally or topically.

20. Ans. is b and d i.e. Lithium; and Heparin Ref. Harrison 16/e, p 2271 (table 333-3)

Drugs associated with increased risk of generalised osteoporosis in adults are :


Hormones Metals Cytotoxic drugs Others
• Glucocorticoids • Aluminium • Cyclosporine • Alcohol
• Thyroxine • Lithium • Tacrolimus • Anticonvulsant drugs
• GnRH agonist
• Heparin

559
7 Self Assessment & Review Phar macology
Pharmacology
21. Ans. is a, b, c and d i.e. Quinidine; Isoniazid; Hydralazine; Interferon Alpha
Ref. Harrison 15/e, p 433

Both Procainamide and Phenytoin causes SLE like syndrome. But in question it is asked most commonly. So,
the answer is Procainamide.

Please remember the drug causing SLE like syndrome (Mnemonic - Plz Check BOM In SHIP).
Plz Procainamide In Interferon α
Check Chlorpropamide S Sulphonamide
B Barbiturates H Hydralazine
O OCP I INH
M Methyldopa P Phenytoin

Features of drug induced lupus which differentiates it from SLE :


• Sex ratio is nearly equal (less female predilection).
• Antinuclear antibodies (ANA) are positive.
• Nephritis and CNS features are not generally seen.
• Hypocomplementemia and antibodies to ds DNA are absent.
• Clinical features and most lab abnormalities revert to normal, when offerding drug is withdrawn.

22. Ans. is a i.e. ACE inhibitors Ref. Harrison 15/e, p 425; 16/e, p 261

• ACE inhibitors block the conversion of angiotension I to angiotensin II. This leads to impaired aldosterone
release, finally resulting in K+ conservation and hyperkalemia.
For more details, refer answer no. ..............................................

23. Ans. is a, b and c i.e. Ciprofloxacin, Cycloserine; and Glucocorticoids Ref. Harrisons 16/e, p 2362

Drugs and other substances that can cause seizures


• Antimicrobials /Antivirals • Psychotropics
– β - lacam antibiotics – Antidepressants
– Quinolones – Antipsychotics
– Cycloserine – Lithium
– Acyclovir • Theophylline
– Isoniazid • Glucocorticoids
– Ganciclovir • Flumazenil
• Anaesthetics and analgesics • Sedative - hypnotic withdrawl
– Meperidine – Alcohol
– Tramadol – Barbiturates
– LA – Benzodiazepines
• Immunomodulatory drugs • Drugs of abuse
– Cyclosporine – Amphetamine
– OKT - 3 – Cocaine
– Tacrolimus (FK - 506) – Phencyclidine
– Interferons – Methylphenidate

560
Miscellaneous 7
24. Ans. is a and d i.e. Bleomycin; and Nitrofurantin Ref. Harrison 15/e, p 427

Since, it is an old question repeated time and again, I will advise you to go through the tables in Harrison 15e.

Drugs causing Pleural / Pulmonary Fibrosis

• Acyclovir • Amiodarone • Azathioprine


• Bleomycin • Busulphan • Carmustin
• Chlorambucil • Cyclosphosphamide • Gold
• Melphalan • Methotrexate • Methysergide
• Mitomycin C • Nitrofurantoin • Procarbazine
• Sulphonamides

25. Ans. is c, d and e i.e. Dapsone; Cotimoxazole; and Ganciclovir Ref. Harrion 16/e, p 1114

Causes of bone marrow suppression in patients with in HIV infection.


Diseases Drugs

↓ ↓ ↓
– HIV infection – Zidovudine – 5 - Flucytosin
– Mycobacterial infection – Dapsone – Ganciclovir
– Fungal infection – Cotirmoxazole – Interferon - α
– B19 Parvovirus infection – Pyrimethamine – Foscarnet
– Lymphoma – Trimetrexate

26. Ans. is a and c i.e. Gentamicin; and Phenacetin Ref. Harrison 15/e, p 433

Drugs causing nephropathy (Acute tublar necrosis).

• Aminoglycosides • Methoxyflurane
• Amphoterecin B • Polymyxin
• Cephaloridine • Phenacetin
• Colistin • Radioiodinated contrast media
• Cyclosporine • Sulfonamides
• IV immune globulin • Tetracyclines

27. Ans. is a, b and d i.e. Estrogen; Cyclosporine; and Phenothiazine Ref. Harrison 15/e, p 433

Drugs causing Cholestatic Jaundice :


Mnemonics : Jaundice Ne 6 Children On Table MARE

• Nitrofurantoin • Oral contraceptive pills


• Chlorpropamide • Tamoxifen
• Cyclosporine • Methimazole
• Chlorpromazine • Anabolic steroids
• Carbamazapine • Rifampicin
• Clavulinic acid • Erythromycin estolate
• Calcium channel blocker

561
7 Self Assessment & Review Phar macology
Pharmacology
28. Ans. is b and c i.e. Methysergide; and Amiodarone Ref. Harrison 15/e, p 427

Already explained, refer answer no. 24

29. Ans. is a, c and d i.e. Dapsone; Arsenic; and Lead

Option ‘a’ Dapsone, a dermatologic agent, used for leprosy causes dose related pure motor neuropathy.
Option ‘b’ Cisplatin, an antineoplastic drug, causes pure sensory neuropathy.
Option ‘c’ Arsenic, used as a herbicide and insecticide, causes both sensory and motor neuropathy.
Option ‘d’ Lead, a heavy metal, causes selective motor neuropathy with prominent wrist drop.
Option ‘e’ Hypothyrodism, causes pure sensory neuropathy. Carpal tunnel and other entrapment
syndromes are common in this disorder.

Conditions causing Pure Motor Neuropathy : • Hypoglycaemia • Dapsone


• Suramin • Inorganic lead

30. Ans. is b and e i.e. Rifampicin; and Riboflavin

Option ‘a’ Thiamine (vit. B1) is a colourless compound and it does not colour the urine at all.
Option ‘b’ Rifampicin, a bactericidal antitubercular drug and also most efficient bactericidal drug in
leprosy → causes discolouration of urine and other body secretions which become orange
red.
Option ‘c’ Mepacrine, an antimalarial drug. It is also active against Giardia and tapeworms → It is a
yellow powder, long term use discoloures skin and eye but not urine.
Option ‘d’ INH metabolities are excreted in urine, but do not discolour.
Option ‘e’ Riboflavin (vit. B2), is a yellow coloured flavone compound → Larger doses are excreted
unchanged in urine, giving urine a yellow colour.

31. Ans. is a, d and e i.e. Gold; Ibuprofen; and Captopril Ref. Harrison 16/e, p 1687 (table 264 - 5)

Drugs causing Neprotic syndrome :


• Gold • Penicilamine • Captopril
• NSAIDs • Probenecid • Trimethadione
• Chlormethiazole • Mercury

Option ‘b’ : Amphoterecin B cause : – Azotemia


– ↓ GFR
– Acidosis
– Hypkalemia
– Inability ot concentrateurine

Option ‘c’ : Rifampicin is safely used in renal failure.

562
Miscellaneous 7
32. Ans. is a and b i.e. Clofazimine; and Minocycline Ref. CMDT ‘01, p 193 - 194

Pigmentrary changes in skin caused by drugs :


Appearance Distribution and comments Causative drugs
• Flat hyperpigmented – Forehead and cheeks – Oral contraceptive Pills
areas (chloasma, mealasma)
– The most common pigmentary
disorder asso. with drug ingestion
• Blue gray discolouration – Light exposed areas – Chlopromazine and
related phenothiazines
• Brown or blue gray – Generalised – Heavy metals
disclouration (Ag, Au, Bi, As)
• Blue black disclouration – Minocycline,
on shins Chloroquine
• Yellow disclouration – Generalised – Quinacrine
• Blue black of nails – Chloroquin
and palate and
dipigmentation of hair
• State gray colour – Photo exposed areas – Amiodarone
• Brown discolouration In darkly pigmented – Zidovudine
of nails patients – Hydroxyureas
• Reddish black – Sun exposed areas – Clofazimine

33. Ans. is a i.e. Organ transplant Ref. KDT 6/e, p 840

Tacrolimus (FK - 506)


• It is a newer immunosuppressant.
• It is particularly valuable in liver transplantation because its absorption is not dependent on bile.
• Because of more potent action it is also suitable for suppressing acute rejection.
• It is chemically different from cyclosporine but having the same mechanism of action and is ∼100 times
more potent. It binds to a different cytoplasmic protein labelled “FKBP” but the subsequent steps are the
same, i.e. inhibition of helper T cells via calcineurin.
• As compare to cyclosporine it causes :
– Less - Hypertension, hirsutism and gum hyperplasia.
– More - Precipitate diabetes, neurotoxicity, alopecia and diarrhoea.

34. Ans. is a, b, c, and d i.e. Vit-C; Vit-E; Vit-A; and Glutathione Ref. Harper 25/e, p 649, 767

Option ‘e’ – Iron is not a free radical scavenger but itself is involved in the generation of free radicals by
catalysing 2 reactions : • Fenton reaction
• Haber-weiss reaction.

563
7 Self Assessment & Review Phar macology
Pharmacology
The major free radicals along with the free radical scavengers are given below :
Features Reactive species Antioxidant
• 1
O–2 Singlet oxygen – Vitamin A, β – carotene, Vitamin E
• O2– Superoxide free radical – Superoxide dismutase, Vitamin E, β-carotene
• OH– Hydroxyl free radical

• RO Alkoxyl free radical

• ROO Peroxyl free radical – Vitamin E, Vitamin C
• H2O2 Hydrogen peroxide – Catalase, Glutathione Peroxidase
• LOOH Lipid peroxides – Glutathione peroxidase

35. Ans. is a, d and e i.e. Verapamil; Tricylic antidepressants; and Ferrous sulphate
Ref. Harrison 15/e, p 432 (table 71.2)

• This question has been directly picked up from Harrison’s 15/e, p 432 table. Since, these tables have now
been deleted from newer edition of Harrison; I will recommend you to go through these tables.
• Here is a quick glance on the drugs.
Drug Causing

Constipation Diarrhoea
• Aluminium hydroxide • Antibiotics (broad spectrum)
• Barium sulphate • Clindamycin
• Calcium carbonate • Cocaine
• Ferrous sulphate • Colchicine
• Ganglion blockers • Digitalis
• Ion exchange resins : • Guanethedine
– Cholestyramine • Lactose exipients
– Colestipol • Lincomycin
• Opiates • Magnesium in antacids
• Phenothiazines • Methyldopa
• Tricyclic antidepressants • Misoprostol
• Verapamil • Oral contraceptives
• Purgatives
• Reserpine
• Ticlopidine

564
Miscellaneous 7
CHAPTER REVIEW
• This section includes questions of V arious Other PGMEES from 1990 – 2008.
Various
• Questions are ar ranged in increasing order of page sequence of KDT 6 Edition. This is
arranged
done to mak e refer
make ral system more easy and uncomplicated to save the precious time
referral
of PGMEE Aspirant.

1. Drug monitoring (serem levels) is done for : 6. Which of the following antitussive is contra-
a. Salbutamol (DNB 01) indicateed in patients with suspected glucoma :
b. Theophylline a. Noscapine (DNB 92)
c. Corticosteroids b. Codeine
d. lepratropine c. Benzonatate
[Ref. KDT 6/e, p 34 - 35] d. Carbetapentane
2. Drug levels should be monitored in the treatment [Ref. KDT 6/e, p 214 - 215]
of : (Jipmer 91) 7. Budesonide is : (J & K 05)
a. Diabetes a. A new bronchodilator
b. Epilepsy b. Used in AIDS
c. Status Asthmaticus c. A new antiepileptic
d. Hypertension d. New antihypertensive
[Ref. KDT 6/e, p 34 - 35] [Ref. KDT 6/e, p 217]
3. All of the following reduces bronchial secretions 8. Which of these can be given intra nasally ?
except : (CUPGEE 99) a. Histrelin (CUPGEE 02)
a. Alcohol b. Naferelin
b. Atropine c. Goserelin
c. Phenothiazines d. Synthetic progestogens
d. Imipramine [Ref. KDT 6/e, p 238 - 239; Goodman & Gilman
[Ref. KDT 6/e, p 106] 10/e, p 1552]
4. Mechanism of action of sildenafil is : (UPSC 05) 9. Troglitazone is the drug used in the tratment of :
a. A-adrenoceptor antagonist a. Petit mal epilepsy (JIPMER 00)
b. Dopamine agonist b. Type 2 diabetes mellitus
c. Phosphodiesterase Type 5 inhibitor c. Hyperlipidaemia
d. Direct action on vascular smooth muscles d. Sea sickness
[Ref. KDT 6/e, p 295 - 296] [Ref. KDT 6/e, p 239 - 270]
5. Hemodialysis is not useful in propranolol poisoing 10. Tizanidine is used in the treatment of :
beacuse of : (JIPMER 91) a. Hypertension (JIPMER 02)
a. Intra erythrocytic storage b. Spasticity
b. High protein binding c. Glaucnoma
c. Rapid metabolism d. Benign prostatic hyperplasia
d. Extensive distribution [Ref. KDT 6/e, p 349; Goodman & Gilman 10/e, p
[Ref. KDT 6/e, p 20 - 21, 138] 235]

Answer 1. b. Theophylline 2. b and c 3. a. Alcohol 4. c. Phosphodi ... 5. b. High protein ...


6. d. Carbetapentane 7. None 8. b. Naferelin 9. b. Type 2 ... 10. b. Spasticity

565
7 Self Assessment & Review Phar macology
Pharmacology
11. Chloropropamide Intracts with : (AI 91) 18. Group of drugs likely to induce hyperglycemia
a. Diazepam and hypercalcemia : (Orissa 98)
b. Alcohol a. Beta-adrenoreceptor blocker
c. Lorazepam b. Thiazides
d. Chlordia zepoxide c. MAO inhibitors
[Ref. KDT 6/e, p 380, 383] d. Tricyclic antidepressants
12. Which of the following substances does not bind [Ref. KDT 6/e, p 568]
to activated charcoal : (J & K 05) 19. Drug acting as a free radical scavenger is :
a. Methanol a. Finasteride (MP 00)
b. Carbamazepine b. Lamotrigine
c. Dapsone c. Cyclophosphamide
d. Salicylates d. Gugulip
[Ref. KDT 6/e, p 386] [Ref. KDT 6/e, p 618]
13. Acts on NMDA receptors : (Nimhans 01) 20. Breast feeding is not contra indicated when the
a. Opiod mother taking the following medications excepts:
b. Benzodiazepine a. Metronidazole (JIPMER 92)
c. Non-benzodiazepine b. Cimetidine
d. NSAIDS c. Theophylline
[Ref. KDT 4/e, p 469] d. Chloramphenicol
14. Slow channel syndrome’ is due to : (JIPMER 98) [Ref. KDT 6/e, p 617 - 619; Katzung 8/e, p 1033]
a. Accumulation of Na+ ions in channels 21. Cytoprotective drug : (Calcutta 00)
b. Accumulation of K+ ions in channels a. Misopristol
c. Accumulation of mg++ ions in channels b. Ranitidine
d. Accumulation of Ca++ ions in channels c. Omeprazole
[Ref. KDT 6/e, p 529] d. Ketoconazole
15. Maximum tachycardia is seen with : [Ref. KDT 6/e, p 633]
a. Nifedipine (Rohtak 98) 22. Which of the following drugs is not effective on
b. Verapamil the treatment of motion sickness : (AMC 91)
c. Propanolol a. Metoclopramide
d. Amlodipine b. Cyclizines
e. Atenolol c. Meclozine
[Ref. KDT 6/e, p 545] d. Hyoscine
16. First dose hypotension is caused by : (MAHE 05) [Ref. KDT 6/e, p 644]
a. Prazosin 23. The following drug is not secreted in bile :
b. Clonidine a. Penicillin (Kerala 97)
c. Sodium nitroprusside b. Phenophthalein
d. Propranolol c. Novobiocin
[Ref. KDT 6/e, p 545] d. Erythromycin
17. Diazoxideisa : (TN 91) [Ref. KDT 6/e, p 896]
a. Myocardial depressant 24. The drug causing curare like effect is : (AI 91)
b. Centrally acting agent a. Chloromphenicol
c. Antihypertensive acting directly on blood vessels b. Clindamycin
d. Adrenergic blocking agent c. Tetracycline
[Ref. KDT 6/e, p 548] d. Streptomycin
[Ref. KDT 6/e, p 721]

Answer 11. b. Alcohol 12. a. Methanol 13. c. Non- ... 14. d. Accumulation... 15. a. Nifedipine
16. a. Prazosin 17. c. Antihyperten... 18. b. Thiazides 19. d. Gugulip 20. None
21. a. Misopristol 22. a. Metoclopramide 23. a. Penicillin 24. d. Streptomycin

566
Miscellaneous 7
25. Drug most concentrated in collagen : (PGI 93) 27. Antineoplastic activity is seen with : (JIPMER 99)
a. Sulfa a. Retinoids
b. Penicillin b. Saturated fatty acids
c. Chloroquine c. Fish proteins
d. Mepacrine d. Polyunsaturated fatty acids
[Ref. KDT 6/e, p 788] [Ref. KDT 6/e, p 870 - 871; Goodman & Gilman
26. Drug of choice of hepatolenticuiar degeneration: 10/e, p 1777]
a. Calcium disodium edetate (DNB 92) 28. Which of the following chelating agents is derived
b. Penicillamine from a micro- organism ? (Kerala 00)
c. Desferoxamine a. Dimercaprol
d. Dimercaprol b. EDTA
[Ref. KDT 6/e, p 867] c. Desferoxamine
d. Trientine
e. None of the above
[Ref. KDT 4/e, p 867 - 868]

Answer 25. d. Mepacrine 26. b. Penicillamine


27. a. Retinoids 28. c. Desferoxamine

567
NOTES
Questions fr om Bihar
from Bihar,, Jharkhand, K ar
Kar natak
arnatak a, Kolk
nataka, ata MP
Kolkata MP,, UP Entrance Examinations (2000-2006)

General Pharmacology B. Facilitative diffusion


C. Sodium symport
1. Which among the following is not an anti-
D. Active transport
psychotic:
A. Risperidone (Ref. KDT/6th ed. /425) 8. Loading dose of a drug is given :
B. Haloperidol A. To achieve high concentration in short time
C. Fluoxetine B. Rapid onset of action (Ref. KDT/6th ed. /34)
D. Clozapine C. Less complication
D. All
2. Alkalinization of urine is done for :
A. Weak acidic drugs (Ref. KDT/6th ed. /30) 9. Which drug is contraindicated in pregnancy :
B. Weak basic drugs A. Captopril (Ref. KDT/6th ed. /553)
C. Strong acidic drugs B. Pyrazinamide
D. Strong basic drugs C. Nifedipine
D. Ondansetron
3. About 1st order kinetics, true is :
(Ref. KDT/6th ed. /31) 10. Plasma drug monitoring is done for :
A. Clearance remains constant (Ref. KD\T/6th ed. /35)
B. Fixed amount of the drug eliminated A. Drug with high safety margin
C. Half life increases with dose B. Drug with low safety margin
D. Decreased clearance with increasing dose C. Drug with high safety margin
D. None
4. Cytochrome P450 which is most commonly
involved in drug metabolism is : 11. Drug that crosses placenta is :
A. CYP 2D6 (Ref. KDT/6th ed. /28, 24) A. Isoniazid (Ref. KDT/6th ed. /743)
B. CYP 3A4 B. Rifampicin
C. CYP 2C19 C. Pyrazinamide
D. CYP 2D3 D. All of the above

5. Neural tube defect occurs due to : 12. Enzyme inhibitor is :


A. Valproate (Ref. KDT/6th ed. /407) A. Ketoconazole (Ref. KDT/6th ed. /27)
B. Phenytoin B. Rifampicin
C. Diazoxide C. Tolbutamide
D. None D. Phenobarbitone

6. Drug response curve is useful in : 13. Clearance (Ref. KDT/6th ed.HPIM, 31, 421)
A. Therapeutic index (Ref. KDT/6th ed. /54) A. Refers to the efficacy of elimination of a drug
B. Efficacy of drug by an organ or whole body
C. Potency of drug B. Cannot be greater than blood flow to an organ
D. All of the above C. Determines the steady state of drug
concentration
7. Saturation kinetics is seen in all except : D. All of the above
A. Simple diffusion

Answer 1. C. Fluoxetinez 2. A. Weak acid drugs 3. A. Clearance ... 4. B. CYPA3 5. A. Valporate


6. D. All of the ... 7. D. Active transport 8. A. To achieve ... 9. A. Captopril 10. B. Drug with low...
11. D. All 12. A. Ketokonazole 13. D. All the above

569
Self Assessment & Review Pharmacology
14. In drug metabolism, hepatic cytochrome P-450 20. Various methods used to delay drug absorption
system is responsible for: are all except:
A. Phase I reactions (hydrolysis, oxidation, (Ref. KDT/6th ed. /16)
reduction, etc.) only A. Sustained release preparation
B. Phase II reactions (conjugation, synthesis, B. Oily suspension for intramuscular injection
etc.) only (Ref. KDT/6th ed. /24 ) C. Incorporating adrenaline with the drug for
C. Both phase I and phase II reactions subcutaneous injection
D. Converting hydrophilic metabolites to lipophilic D. Use water soluble salt form of the drug
metabolites
21. One of the potent microsomal enzyme inducer
15. One of the drugs listed below is contraindicated drug is: (Ref. KDT/6th ed. /741)
in the treatment of pregnancy-induced A. Captopril
hypertension: B. Erythromycin
A. Enalapril (Ref. KDT/6th ed. /205) C. Rifampicin
B. Nifedepine D. Cimetidine
C. Magnesium sulphate
D. Hydralazine 22. The most potent drug to reduce plasma
cholesterol level is:
16. Zero-order kinetics is seen with all of the A. Plant sterols (Ref. KDT/6th ed. /614, 615)
following except: B. Fibrates
(Ref. Clinical Pharmacology, Laurence, C. Anion exchange resins
Bennett, Brown. 8th edition (85) D. Statins
A. Phenytoin
B. Salicylates in high doses 23. Which of the following is a synthetic reaction:
C. Methotrexate A. Reduction (Ref. KDT/6th ed. /25)
D. Ethanol B. Acetylation
C. Oxidation
17. Time for peak plasma concentration (Tmax) D. Hydrolysis
indicates: (Ref. KDT/6th ed. /15, 16)
A. The rate of elimination 24. Idiosyncrasy is: (Ref. KDT/6th ed. /81)
B. The onset of effect A. A genetically determined abnormal reaction
C. The rate of absorption to drugs
D. The intensity of effect B. A characteristic toxic effect at therapeutic
doses
18. Volume of distribution for chloroquine (Vd) is: C. An altered physiological state produced by
A. 5-8 L (Ref.KDT/6th ed. / 786) repeated drug use
B. 9-15 L D. An immunologically mediated reaction
C. 100-650 L 25. The following statements are true, except:
D. Above 1300 L (Ref. KDT/6th ed. /13)
19. One of the potential microsomal inhibitor drug A. Acidic drugs are largely ionized at acidic pH
is: (Ref. KDT/6th ed. /407) B. Basic drugs are excreted faster if urine is
A. Phenobarbitone acidified
B. Griseofulvin C. Lipid soluble non-electrolytes readily cross
C. Sodium valproate biological membranes
D. Phenytoin D. Unionized form of drug is readily absorbed

14. A. Phase I .... 15. A. Enalapril 16. C. Methotrexate 17. C. The rate of... 18. D. Above 1300 L
Answer
19. C. Sodium ..... 20. D. Use water.. 21. C. Rifampicin 22. D. Statins 23. B. Acetylation
24. A. A genetically... 25. A. Acidic ...

570
Questions fr om Bihar
from Bihar,, Jharkhand, K ar
Kar natak
arnatak a, Kolk
nataka, ata MP
Kolkata MP,, UP Entrance Examinations (2000-2006)

26. The following statements about microsomal enzymes 33. Drug administered sublingually due to increased
are true, except: (Ref. KDT/6th ed. /26, 27, 28) hepatic first pass metabolism :
A. Cytochrome P450 is a microsomal enzyme A. Stanozolol (KDT/6th ed. /270,276,279,292)
B. Drugs can induce microsomal enzyme B. Tamoxifen
C. Located in mitochondria of hepatic cells C. Diethyl stilbestrol
D. Non-microsomal enzymes are not inducible D. Natural oestrogen
by drugs 34. All are deposited in the eye except :
27. Which of following is true regarding recent A. Gold (Ref. KDT/6th ed. /805)
guidelines of WTO in relation to international B. Chloroquine
drug trade: C. Amiodarone
A. Import of generic drugs by developing countries D. Antimony
B. Supply of drugs for diseases like HIV, malaria
35. Which of the following is a prodrug:
at low price to developing countries A. Captopril (Ref. KDT/6th ed. /24)
C. Patent of drugs to be given to multinational B. MethyIdopa
companies only C. Prazosin
D. All of the above D. Propranolol
28. Neural tube defect occurs with:
36. Drug which should be given only intrave-nously:
A. Folic acid (Nelson/17th ed./1984)
A. Aminophylline (Ref. KDT/6th ed. /221)
B. Valproate (Ref.KDT/6th ed. /85)
B. Heparin
C. Lithium
C. Lignocaine
D. Amiodarone
D. Adrenaline
29. Hepatic microsomal enzyme inducer:
37. Pharmacokinetics refers to
A. Rifampicin (Ref.KDT/6th ed. /27)
A. Absorption, distribution, metabolism and
B. Cimetidine
excretion of drug (Ref. KDT 6/th ed. 3-4)
C. Sulfonamides
B. Physiological and biochemical effects of drug
D. Ciprofloxacin
and their mechanism of action
30. Hepatic microsomal enzyme inhibitor: C. Application of pharmacological information
A. Rifampicin (Ref.KDT/6th ed. /27) together with knowledge of the disease for its
B. Cimetidine prevention, mitigation or cure
C. Phenobarbitone D. Preparing suitable forms for administration of
D. Ethyl alcohol drugs in man
31. Which of the following is given by sublingual 38. Partial agonists have (Ref.KDT/6th ed. / 37)
route to avoid hepatic first pass metabolism :
A. Affinity and submaximal intrinsic activity
A. Natural estrogen (Ref. KDT/6th ed./270)
B. Affinity but intrinsic activity between 0 to –1
B. Stanozolol
C. Affinity but no intrinsic activity
C. Tamoxifen
D. Stilbestrol D. Affinity and maximal intrinsic activity

32. One of the following is a prodrug : 39. Drug causing hemolysis in G-6PD deficiency
A. Alpha methyldopa (Ref.KDT/6th ed. /24) A. Estrogen (Ref.KDT/6th ed. /791 )
B. Captopril B. Penicillin
C. Propranolol C. Primaquine
D. Prazosin D. Mefloquine
26. C. Located in .... 27. B. Supply of .. 28. B. Valproate 29. A. Rifampicin 30. B. Cimetidine
Answer
31. B. Stanozolol 32. A. Alpha .... 33. A. Stanozolol 34. D. Antimony 35. B. Methyldopa
36. B. Heparin 37. A. Absorption ... 38. A. Affinity and... 39. C. Primaquine

571
Self Assessment & Review Pharmacology
40. Tachyphylaxis is (Ref.KDT/6th ed. / 68) 47. Dose of drugs are usually adjusted according to
A. Requirement of higher dose of drug to produce A. Body surface area (Ref. KDT/6th ed. / 61)
a given response B. Body weight
B. Rapid development of tolerance C. Sex
C. Tolerance of micro-organisms to inhibitory D. Age
action of antimicrobials 48. Zero order kinetics is shown by all except
D. Development of tolerance to pharmacologically A. Ethyl alcohol (Ref. KDT/6th ed. / 823)
related drug B. Phenytoin
41. All of the following drugs cross placenta except C. Methotrexate
A. Lithium (Ref.KDT/6th ed. /598) D. High dose salicylates
B. Morphine 49. True regarding efficacy and potency of drug is
C. Heparin (Ref. KDT/6th ed. / 53, 54; Katzung 7/e,p 266,28)
D. Warfarin A. Potency is more important than efficacy
42. All of the following drugs cause hemolysis in B. ED60 is a measure of efficacy of drug
G-6-PD deficient persons except C. Drugs having similar action have different
A. Cotrimoxazole (Ref. Already explained) efficacy
B. Rifampicin D. Height of dose response curve demotes
C. Furazolidone efficacy of drug
D. Primaquine 50. Removal of acidic drugs from the body is done
43. All are prodrugs except (Ref. KDT/6th ed. / 24) by using
A. Dipivefrine A. Citric acid (Ref. KDT/6th ed. / 264, 387)
B. Indomethacin B. Hydrochloric acid
C. Enalapril C. Ammonium chloride
D. Levodopa D. Sodium bicarbonate

44. All of the following drugs are metabolized by 51. Therapeutic drug monitoring must be done for
acetylation except A. Lignocaine (Ref. KDT/6th ed. / 435)
A. INH (Ref. KDT/6th ed. / 428; Goodman and B. Erythromycin
C. Penicillin
B. Hydralazine Gilman’s 10/e,p 14)
D. Lithium
C. Chlorpromazine
D. Procainamide 52. Undesirable but unavoidable pharma-
codynamic effects of a drug is
45. All are given by intranasal route except
A. Intolerance (Ref. KDT/6th ed. / 79)
A. Nicotine(Ref. KDT 6/e, p 458; Satoskar 16/e)
B. Side effect
B. Ether
C. Idiosyncracy
C. Posterior pituitary extract
D. Toxic effect
D. Pethidine
53. Which of the following is an enzyme inducer
46. Acidic drugs are absorbed in
A. Penicillin (Ref. KDT/6th ed. / 27)
A. Colon (Ref. KDT/6th ed. / 13)
B. Phenobarbitone
B. Ileum
C. Digoxin
C. Duodenum
D. Paracetamol
D. Stomach

40. B. Rapid .... 41. C. Heparin 42. B. Rifampicin 43. B. Indomethacin 44. C. Chlopromazine
Answer
45. D. Pethidine 46. D. Stomach 47. B. Body weight ... 48. C. Methotrexate ... 49. D. Height of ...
50. D. Sodium ... 51. D. Lithium 52. B. Side effect 53. B. Phenobarbitone

572
Questions fr om Bihar
from Bihar,, Jharkhand, K ar
Kar natak
arnatak a, Kolk
nataka, ata MP
Kolkata MP,, UP Entrance Examinations (2000-2006)

54. Grey baby syndrome is caused by 60. Which one of the following drugs does not
(Ref. KDT/6th ed. / 717) undergo hepatic first pass effect
A. Chloramphenicol A. Propranolol (Ref. KDT/6th ed. / 255, 256)
B. Cephalosporins B. Lidocaine
C. Tetracyclines C. Insulin
D. Salicylates D. Morphine
55. Interdose interval depends upon 61. Which one of the following drugs has narrow
(Ref. Goodman & Gilman 10/e,p 22) therapeutic range?(Ref. KDT/6th ed. / 404, 405;
A. Binding to plasma protein Chaudhuri’s Medical Pharmacology 2/e,p 43)
B. Elimination half life A. Phenytoin
C. Hepatic clearance B. Prazosin
D. Volume of distribution C. Piroxicam
D. Propranolol
56. Drugs interact with their receptor sites by
forming (Ref. Katzung 8/e,p 2; Craig 6/e,p 6) 62. Which one of the following drugs does not
A. Ionic bonds have an active metabolite
B. Covalent bond A. Lisinopril (Ref. KDT/6th ed. / 485)
C. Coordinate covalent bonds B. Allopurinol
D. Wan der wall forces with basic drugs only C. Propranolol
D. Diazepam
57. Drug to be avoided in renal disease
A. Doxycycline (Ref. KDT/6th ed. / 720, 721) Pharmacology of ANS, PNS & Autacoids
B. Nitroprusside
1. Which among the following is not an anti-
C. Ceftriaxone
psychotic drug : (Ref. KDT/6th ed. /425)
D. Gentamicin A. Risperidone
58. The time required to reach steady state after a B. Haloperidol
dosage regimen depends on C. Fluoxetine
(Ref. Goodman and Gilman 10/e, D. Clozapine
p 23; Katzung8/e,p 46)
2. Rofecoxib was banned due to :
A. Dose of drug
A. Ischemic heart disease
B. Dosage interval B. Renal complication (Ref. KDT/6th ed. /197)
C. Half life of drug
C. Liver adenoma
D. Route of administration
D. Gastric ulcer
59. Healthy normal human volunteers participate 3. Long acting non-depolarizing muscle relaxant
in which phase of clinical trial
is :
A. Phase IV (Ref. Craig 6/e,p 90)
A. Succinylcholine (Ref. KDT/6th ed. /339)
B. Phase III B. Mivacurium
C. Phase II
C. Pancuronium
D. Phase I D. Phenylephrine

54. A. Chloramphenicol 55. B. Elimination .. 56. A. Ionic bonds 57. D. Gentamycin 58. C. Half life ....
Answer
59. D. Phase I 60. C. Insulin 61. A. Phenytoin 62. A. Lisinopril 1. C. Fluoxetine
2. A. Ischemic ... 3. C. Pancuronium

573
Self Assessment & Review Pharmacology
4. Histamine blocker in stomach acts through develop (Ref. KDT/6th ed. /139)
(in regulation of gastric secretion) A. Asthma
(Ref. KDT/6th ed. /627) B. CHF
A. Decreasing cAMP in stomach C. Bradycardia
B. Increasing cAMP in stomach D. All of the above
C. Increasing IP3 in stomach
12. Beta blocker having both alpha and beta
D. Decreasing IP3 in stomach
blocking property : (Ref. KDT/6th ed. /137)
5. Betaxolol is a : (Ref. KDT/6th ed. /145) A. Carvedilol
A. α Blocker B. Sotalol
B. β Blocker C. Nadolol
C. Calcium channel blocker D. Pindolol
D. None
13. Epinephrine acts by : (Ref. KDT/6th ed. /118)
6. Which among the following is contraindicated A. Adenyl cyclase
in myasthenia patient B. Phosohodiesterase
A. Aminoglycosides(Ref. KDT/6th ed. /721-722) C. Phospholipase
B. Sulphonamides D. None of the above
C. Penicillin
14. Which of the following is not given in myas-
D. All
thenia gravis (Ref. KDT/6th ed. /734)
7. Patient has a history of GIT bleeding. Which A. Clofibrate
drug is given for pain control ? B. PolymyxinB
A. Nimesulide (Ref. KDT/6th ed. /197) C. Penicillin
B. Ibuprofen D. All of the above
C. Rofecoxib
15. Which one of the following drugs is not a
D. Pentozocine
uterine relaxant?
8. Vasomotor reversal of Dale is due to : A. Phenobarbitone
A. Alpha blocker (Ref. KDT/6th ed. /123) B. Phenytoin (Ref. KDT/6th ed. /126, 131, 330)
B. Beta blocker C. Salbutamol
C. Alpha and beta blocker D. Terbutaline
D. None
16. Which one of the following drugs is not a long
9. Beta blocker for stopping premature labor : acting neuromuscular blocking agent?
A. Carvedilol (Ref. KDT/6th ed. /127) A. Doxacurium (Ref. KDT/6th ed. /339, )
B. Terbutaline B. Mivacurium
C. Pindolol C. Pancuronium
D. Nadolol D. Pipecuronium
10. Not an H2 receptor antagonist is : 17. Tamsulosin, á competitive a, adrenoceptor
A. Ranitidine (Ref. KDT/6th ed. /628) antagonist has affinity for which of the follow-
B. Cimetidine ing receptors?
C. Omeprazole A. a1A (Ref. KDT/6th ed. /132, 147)
D. Famotidine B. a1D
C. None of the above
11. Old patient taking beta blockers is prone to
D. Both a and b
4. D. Fecrasing IP3... 5. B. β Blocker 6. A. Aminoglycosides 7. C. Rofecoxib 8. A. Alfa blocker
Answer
9. A. Carvediol 10. C. OMPERAZOLE 11. D. All 12. A. Carvedilol 13. A. Adenyl cyclase
14. B. Polymixin B 15. B. Phenytoin 16. B.Mivacurium 17. A. a
1A

574
Questions fr om Bihar
from Bihar,, Jharkhand, K ar
Kar natak
arnatak a, Kolk
nataka, ata MP
Kolkata MP,, UP Entrance Examinations (2000-2006)

18. Which one of the following is not a 5-HT D. All of the above
receptor antagonist?
24. Most common drug to be used for cycloplegia
A. Ketanserin(Ref. KDT/6th ed. /166, 167, 190)
in a child of 6 years is: (Ref. PARS-p- 112, 113)
B. Methysergide
A. Homatropine
C. Tropisetron
B. Tropicamide
D. Lanreotide
C. Atropine
19. Ideal drug employed in the preoperative D. Scopolamine
preparation for surgical excision of
25. Full vagolytic dose of atropine is__micro-
pheochromocytoma is:
grams/kg:
A. Atenolol (Ref. HPIM-p-135)
A. 10 (Ref. LEE-756)
B. Phenoxybenzamine
B. 20
C. Reserpine
C. 30
D. Clonidine
D. 40
20. Short elimination half-life (8-10 min) of esmolol
26. In oral poisoning with carbamate insecticides,
(beta-1 adrenergic blocker) is due to:
_________may be hazardous:
A. Rapid redistribution (Ref. KDT/6th ed. /141)
A. Pralidoxime (Ref. KDT/6th ed. /105)
B. Rapid elimination by kidney
B. Atropine
C. Hydrolysis by blood esterases
C. Magnesium sulphate purgative
D. Rapid protein binding
D. Gastric lavage with activated charcoal
21. Which of the following is a non-catecholamine
27. Dobutamine increases:
sympathomimetic drug? (Ref. KATZ-p-126)
A. Heart rate(Ref. HPIM-p-126, KDT/6th ed. 126)
A. Ephedrine
B. Cardiac output
B. Dopamine
C. Blood pressure
C. Isoproterenol
D. Plasma volume
D. Dobutamine
28. Lower dose of dopamine in cardiogenic shock
22. The rate-limiting enzyme in the biosynthesis
will increase: (Ref. HPIM-p-126)
of dopamine is: (Ref. KDT/6th ed. / 116)
A. Cardiac output
A. Tryptophan hydroxylase
B. Urine output
B. Aromatic amino acid decarboxylase
C. Heart rate
C. Tyrosine hydroxylase
D. Blood pressure
D. Dopamine-b-hydroxylase
29. Fexofenadine is the metabolic product of:
23. Adverse effect of beta-blockers may include:
(Ref. KATZ-p- 270) 8th ed., KDT 6/e, p 156-158
A. Congestive heart failure (Ref. HPIM-p-139)
A. Loratadine
B. Blunting of sympathetic response to oral
B. Astemizole
hypoglycemia drugs (OHA)
C. Cetirizine
C. Bronchial asthma
D. Terfenadine

18. D. Lanreotide 19. B. Phenoxyben.... 20. C. Hydrolysis .... 21. A. Ephedrine 22. C. Tyrosine ....
Answer
23. D. All of the above 24. C. Atropine 25. D. 40 26. A. Pralidoxime 27. B. Cardiac output
28. B. Urine output 29. D. Terfenadine

575
Self Assessment & Review Pharmacology
30. The following statements about physostigmine 37. The following statements about beta
are true except: (Ref. KDT/6th ed. /101) adrenoceptor blockers are true, except:
A. Directly acts on cholinoceptor A. The altered plasma lipid profile carries less
B. Crosses blood-brain barrier risk of coronary disease
C. Specific antidote for belladonna poisoning B. Prolong insulin induced hypoglycemia
D. Can be administered intravenously C. Cause bradycardia (Ref. KDT- 142, 143)
D. Exacerbate variant angina
31. Non-sedative antihistamine is:
A. Cyclizine (Ref. KDT/6th ed. /156) 38. All of the following statements about H1
B. Astemizole antihistaminics are true except:
C. Meclizine (Ref. KDT/6th ed. /155, 157)
D. Pyrilamine A. Tolerance may develop to some of its side
effects
32. Wide angle glaucoma can be treated with:
B. Local application can cause contact dermatitis
A. Esmolol (Ref. KDT/6th ed. /144)
C. Combination with an H2 antagonist may
B. Labetalol
control some cases of chronic urticaria
C. Timolol
D. Many of them exhibit cholinergic effect
D. Atenolol
39. Which of the following cannot be given in a
33. Misoprostol is used for all except:
lactating mother with rheumatoid arthritis:
(Ref. KDT/6th ed. /330,331; HPIM-p-1657)
A. Naproxen (Goodman-Gillman/10th ed./710)
A. Termination of pregnancy
B. Mefenamic acid
B. Peptic ulcer C. Diclofenac sodium
C. Induction of labour
D. Flurbiprofen
D. Delaying the delivery
40. Use of prostaglandin in pregnancy causes all
34. The drug of choice in cardiogenic shock is: Except:
A. Norepinephrine (Ref. KDT/6th ed. /126)
B. Epinephrine (Obstetrics-DCD / 6th ed./604) (Ref.KDT/6th ed. /
C. Dopamine 176)
D. Isoproterenol A. Vomiting
B. Bronchodilation
35. Which of the following antihypertensive is an
C. Uterine rupture
alpha-adrenoreceptor blocker
D. Fever
A. Clonidine (Ref. KDT/6th ed. /540)
B. Alpha-methyldopa 41. NSAIDs inhibit all except:
C. Atenolol A. Prostaglandin (Ref.KDT/6th ed. /185)
D. Prazosin B. Prostacycline
C. Thromboxane
36. The sub-type of cardiac muscarinic receptor D. Leukotrienes
is predominantly: (Ref. KDT- 95)
42. Selective B1 blocker with B2 agonist is:
A. M1 A. Propranolol (Ref.KDT/6th ed. /141)
B. M3 B. Labetolol
C. M2 C. Atenolol
D. M1 and M3 D. Celiprolol

30. A. Directly ..... 31. B. Astemizole 32. C. Timolol 33. D. Delaying .... 34. C. Dopamine
Answer
35. D. Prazosine 36. C. M2 37. A. The altered .... 38. D. Many of ..... 39. C. Diclofenac ....
40. B. Bronchodilation 41. D. Leukotrienes 42. D. Celiprolol

576
Questions fr om Bihar
from Bihar,, Jharkhand, K ar
Kar natak
arnatak a, Kolk
nataka, ata MP
Kolkata MP,, UP Entrance Examinations (2000-2006)

43. Which NSAID should not be used with 51. Corticosteroids cause all except:
methotrexate: A. Hyperglycemia (Ref. KDT/6th ed. /277-278)
A. Diclofenac (T.B.ofTherapeutics-Herfindaland B. Reduce bronchiolar dilatation effect of
B. Paracetamol Gourley/7th ed./663) catecholamines
C. Indomethacin (Ref.KDT/6th ed. /190) C. Free fatty acid
D. Aspirin D. Negative calcium balance
44. Partial opioid agonist is: 52. Ethyl alcohol is used in methyl alcohol poison-
A. Morphine (Ref.KDT/6th ed. /464) ing because: (Ref. KDT/6th ed. /387)
B. Methadone A. It acts as an antidote
C. Buprenorphine B. It competes with methyl alcohol for the same
D. Naloxone metabolic pathway
45. Morphine is a: (Ref.KDT/6th ed. /463) C. Renal excretion of methyl alcohol
A. Mixed agonist D Destroys methyl alcohol
B. K receptor
53. Which one of the following is the shortest acting:
C. 8 receptor
D. u receptor A. Renin (Ref. KDT/6th ed. /125)
B. Noradrenaline
46. Labetalol is a: (Ref.KDT/6th ed. /143) C. Aldosterone
A. Alpha antagonist D. Isoprenaline
B. Beta antagonist
C. Alpha and Beta agonist 54. Which of the following does not act
D. Alpha and Beta antagonist intracellularly: (KDT/6th ed./112,KDT/6th ed./228
47. Beta blocker causes: (Ref.KDT/6th ed. /139) for option A; KDT/6th ed./274 for option B; KDT/
A. Bronchodilation 6th ed./268 for option C)
B. Bradycardia A. T3
C. Cerebellar ataxia B. Estrogen
D. Muscle atrophy C. Steroids
D. Dopamine
48. Acetylcholine in not used therapeutically as it is:
A. Not easily available in the market 55. Histamine is not released by:
B. Costly (Ref. KDT/6th ed. /98) A Apomorphine (Ref. KDT/6th ed. /223)
C. Orally inactive B Dextran
D. Rapidly hydrolysed in the body C Sodium cromoglycate
D Polymyxin B
49. Parasympatholytics used in bronchial asthma
cause: (KDT/6th ed./97, 222) 56. Propranolol is indicated in all except
A. Dilatation of both larger and smaller bronchi A. Hypertension (Ref. KDT/6th ed. / 142-143)
B. Dilatation of larger bronchi only B. Headache
C. Dilatation of smaller bronchi only C. Asthma
D. Has no effect on bronchiolar dilatation D. Essential tremor
50. Dissociative anaesthesia is seen with: 57. All are true about ipratropium bromide except
A. Ketamine (Ref. KDT/6th ed. /376) A. Increased IOP (Ref. KDT/6th ed. / 94,97)
B. Propofol B. Urinary retention
C. Etomidate C. Bronchoconstriction
D. Thiopentone D. Relaxes bronchial muscles
43. D. Aspirin 44. C. Buprenorphine 45. A. Mixed agonist 46. D. Alpha and .... 47. B. Bradycardia
Answer
48. D. Rapidly ........ 49. B. Dilatation .... 50. A. Ketamine 51. B. Reduces ..... 52. B. It competes ....
53. C. 54. D. Dopamine 55. C. Sodium .... 56. C. Asthma 57. C. Bronchocon....

577
Self Assessment & Review Pharmacology
58. Non-catecholamine sympathomimetic is C. Sulfinpyrazone
A. Amphetamine (Ref. KDT/6th ed. / 103) D. Frusemide
B. Adrenaline
65. Myasthenia gravis is diagnosed using
C. Nor-epinephrine
A. Edrophonium (Ref. KDT/6th ed. / 104)
D. Dopamine
B. Neostigmine
59. Drug which is mydriatic but not a cycloplegic, C. Carbachol
is D. Physostigmine
A. Tropicamide (Ref. KDT/6th ed. /123)
66. d-TC is a (Ref. Already explained)
B. Homatropine
A. Ganglionic blocker
C. Atropine
B. Competitive long acting blocker
D. Phenylephrine
C. Competitive short acting blocker
60. d-Tubocurarine is (Ref. KDT/6th ed. / 339) D. Depolarising blocker
A. Centrally acting agent
67. Drug of choice for acute attack of migraine
B. Directly acting agent
A. Sumatriptan (Ref. KDT/6th ed. /-170, 171)
C. Competitive blocker
B. Caffeine
D. Non competitive neuromuscular blocker
C. Methysergide
61. All cross blood brain barrier except D. Propranolol
A. Atropine (Ref. KDT/6th ed. / 115) 68. All are non-sedative antihistaminics except
B. Hexamethonium A. Certizine (Ref. KDT/6th ed. / 156)
C. Morphine B. Astemizole
D. Physostigmine C. Terfenadine
62. Atropine is obtained from (Ref. KDT/6th ed. /; D. Triprolidine
A. Dhatura alba Reddy 21/e,p 620,622) 69. All of the following drugs increase histamine
B. Claviceps purpura release except (Ref. KDT/6th ed. / 166)
C. Pappavar somniferous A. Morphine
D. Cannabis sativus B. Cyproheptadine
63. Neostigmine does not cross blood brain barrier C. Polymyxin B
because it is a (Ref. KDT/6th ed. /102) D. D-Tubocurarine
A. Acetyl ester 70. Thromboxane A2 is inhibited by
B. Dipyridine derivative A. Dazoxiben (Ref. KDT/6th ed. / 672)
C. Quaternary ammonium compound B. Sulphinpyrazone
D. Tertiary ammonium compound C. Aspirin
64. All except one cause hyperuricemia and are D. Dipyridamole
contraindicated in gout 71. Adrenaline is used in all except
(Ref. KDT/6th ed. / 202, 203; Harrison 17th ed. / A. Bronchial asthma(Ref. KDT/6th ed. / 129-130)
432) B. Along with LA infiltration
A. Ethacrynic acid C. Myocardial infarction
B. Chlorthiazide D. Penicillin anaphylaxis

58. A. Amphetamine 59. D. Phenylephrine 60. C. Competitive ... 61. B. Hexamethonium 62. A. Datura alba
Answer
63. C. Quaternary .. 64. C. Sulfinpyrazone 65. A. Endrophonium 66. B. Competitive ..... 67. A. Sumatriptan
68. D. 1 69. B. Cyproheptadine 70. A. Dazoxiben 71. C. Myocardial ..

578
Questions fr om Bihar
from Bihar,, Jharkhand, K ar
Kar natak
arnatak a, Kolk
nataka, ata MP
Kolkata MP,, UP Entrance Examinations (2000-2006)

72. Aspirin is useful in all except A. Cycloplegia is required


A. Myalgia (Ref. KDT/6th ed. / 190-191) B. Mydriasis is required without cycloplegia
B. Deep seated abdominal pain C. Mydriasis and cycloplegia both are required
C. Pre-eclampsia D. Mydriasis and cycloplegia both are not required
D. Deep vein thrombosis
79. Which one of the following is aspirin
73. Selective â1-blockers are all except A. Acetyl salicylic acid (Ref. KDT 6/e, p187)
A. Atenolol (Ref. KDT/6th ed. / 136) B. P-aminosalicylic acid
B. Esmolol C. Para-aminobenzoic acid
C. Celiprolol D. Methyl salicylate
D. Bisoprolol Pharmacology of Central Nervous System
74. All are true about COX-2 except 1. Which among the following is not an
A. Are constitutionally expressed on cell surface antipsychotic:
(Ref. KDT/6th ed. / 197) A. Risperidone (Ref. KDT/6th ed. /425)
B. Induced at the site of inflammation B. Haloperidol
C. Are utilized in generation of eicosanoids with C. Fluoxetine
a ring structure D. Clozapine
D. Activation of COX-2 leads to ulceroprotective
effect on gastric mucosa 2. Site of action of opioid receptor is :
A. Area postrema (Ref. KDT/6th ed. /420)
75. Local anaesthetics (Ref. KDT/6th ed. / 352-353)
B. Dorsal horn
A. Blocks the release of neurotransmitter
C. Injury site
B. Inhibits the efflux of sodium from neurons
D. Brain
C. Increases the release of inhibitory
neurotransmitters 3. Drug not acting on neuromuscular junction :
D. Blocks the influx of sodium into cell A. Baclofen (Ref. KDT/6th ed. /428)
B. Carisoprodol
76. For myasthenia gravis, DOC is
C. Haloperidol
(Ref. KDT/6th ed. / 102, Harrison 17th ed. / 2619;
D. None
17/e,p 2621)
A. Neostigmine 4. True about parkinsonism :
B. D-tubocurarine (Ref. KDT/6th ed. /421)
C. Gallamine A. Amantadine is cholinergic
D. Hexamethonium B. Vit. B6 enhances the action of L-Dopa
C. COMT prolongs the action of L-Dopa
77. Treatment of atropine toxicity is
D. None of the above
(Ref. KDT/6th ed. / 113; CMDT, 2004,p 1660)
A. Flumazenil 5. All are TNF-á antagonist used in rheumatoid
B. Naloxone arthritis except :
C. 2-pralidoxime (Ref. KDT/6th ed. /205) (Bihar 2005)
D. Physostigmine A. Ifosfamide
78. Atropine is substituted by phenylepherine to B. Infliximab
facilitate fundus examination when C. Etanercept
(Ref. KDT/6th ed. / 112, 113) D. Adalimumab

72. B. Deep seated ... 73. C. Celiprolol 74. A. Are Constitu..... 75. D. Blocks the .... 76. A. Neostigmine
Answer
77. D. Physostigmine 78. B. Mydriasis is ..... 79. A. Acetyl ....... 1. C. Fluoxetine 2. B. Dorsal horn
3. C. Haloperidol 4. D. None... 5. A. Ifosfamide

579
Self Assessment & Review Pharmacology
6. False about mechanism of action of anticonvulsants B. Beta blocker
A. Ethosuximide – due to K+ channel opening C. Verapamil
B. Phenytoin – Na+ channel inactivation D. Amitryptyline
prolongation (Ref. KDT/6th ed. /404) 14. All are selective serotinin reuptake inhibitors
C. Diazepam – Increase GABA release except
D. Gabapentin -Increase GABA release A. Fluoxetine (Ref. KDT/6th ed. /439)
B. Fluvoxamine
7. Which of eh following opioid receptor mediates
C. Paroxetine
dysphoria : (Ref. KDT/6th ed. /462)
D. Amoxapine
A. Lambda
B. Kappa 15. Folate deficiency occurs due to :
C. Mu A. Phenytoin (Ref. KDT/6th ed. /405)
D. All B. Carbamazepine
C. Phenobarbitone
8. The side effect of phenytoin when its level is
D. All of the above
above therapeutic level is :
A. Ataxia (Ref. KDT/6th ed. /405) 16. Common side effect of chlorpromazine are all except
B. Gum hypertrophy A. Osteoporosis (Ref. KDT/6th ed. /431 - 432)
C. Osteoclastoma B. Parkinson’s diseases
D. Hirsutism C. Skin rashes
D. Amenorrhea
9. Drug of choice in absence seizure :
A. Valproate (Ref.KDT6thed. /408) 17. Drug of choice in myoclonic seizures :
B. Phenytoin A. Valproate (Ref. KDT/6th ed. /408)
C. Diazepam B. Epsolin
D. Ethosuximide C. Carbamazepine
D. Diazepam
10. Selective serotonin reuptake inhibitor is :
A. Desipramine (Ref. KDT/6th ed. /439) 18. Galactorrhea is caused by :
B. Amitriptyline A. Phenothiazines (Ref. KDT/6th ed. /431)
C. Dothepine B. Bromocriptine
D. Fluoxetine C. Pyridoxine
D. None of the above
11. All are atypical antipsychotic drugs except :
A. Clozapine (Ref. KDT/6th ed. /425) 19. Which one of the following drugs is used in
B. Risperidone absence seizures? (Ref. KDT/6th ed. /408, 392)
C. Olanzapine A. Phenobarbitone
D. Loxapine B. Phenytoin
C. Valproate
12. Ondansetron is : (Ref. KDT/6th ed. /646)
D. Carbamazapine
A. 5HT3 antagonist
B. 5HT1 antagonist 20. The side effects of lithium used in psychiatry
C. 5HT2 antagonist include all except: (Ref. HPIM-p-435, 436)
D. 5HT3 agonist A. Nausea, vomiting
B. Tremors
13. Drug of choice for acute attack of migraine :
C. Hyperthyroidism
A. Sumatriptan (Ref. KDT/6 th ed. /170)
D. Hypercalcemia
6. A. Ethosuximide ... 7. B. Kappa 8. A. Ataxia 9. A. Valporate 10. D. Fluoxetine
Answer
11. A. 12. A. 5HT3 antagonist 13. A. Sumatriptan 14. D. Amoxapine 15. A. Phenytoin
16. A. Osteoporosis 17. A. Valporate 18. A. Phenothiazines 19. C. Valproate 20. C. Hyperthy...

580
Questions fr om Bihar
from Bihar,, Jharkhand, K ar
Kar natak
arnatak a, Kolk
nataka, ata MP
Kolkata MP,, UP Entrance Examinations (2000-2006)

21. Which of the following is a specific antagonist 28. Fluoxetine is used for the treatment of:
to diazepam? (Ref. HPIM-p-394) A. Vomiting (Ref. KDT/6th ed. / 446)
A. Ephedrine B. Insomnia
B. Lorazepam C. Myoclonic state
C. Flumazenil D. Endogenous depression
D. Aminophylline
29. Malignant hyperthermia is treated with:
22. Drug of choice for the treatment of negative A. Chlorpromazine (Ref. KDT/6th ed. / 347)
symptoms of schizophrenia is: B. Dantrolene
A. Chlorpromazine (Ref. KDT/6th ed. / 429) C. Clorazepate
B. Haloperidol D. Primidone
C. Clozapine
30. Carbamazepine is useful in the following
D. Doxepin
conditions, except: (Ref. KDT /6th ed. / 406)
23. Analgesic 100 times more potent than A. Grand mal epilepsy
morphine is: B. Petit mal epilepsy
A. Pethidine (Ref. KDT/6th ed. / 459) C. Psychomotor epilepsy
B. Fentanyl D. Trigeminal neuralgia
C. Methadone
31. None of the following analgesics is available
D. Naloxone
in sublingual formulation except:
24. Ondansetron is a potent: (Ref. HPIM-p-641) A. Piroxicam (Ref. KDT/6th ed. / 465, 466)
A. Antiemetic B. Pentazocine
B. Anxiolytic C. Buprenorphine
C. Analgesic D. Ketorolac
D. Hypokalemia
32. Treatment of choice in drug induced
25. Hydroxytryptophan is a precursor of: parkinsonism:
A. LSD (Ref. KDT/6th ed. /162) A. Benzhexol (Ref. KDT/6th ed. /421)
B. Epinephrine B. Bromocriptine
C. Norepinephrine C. Levodopa
D. Serotonin D. Amantadine
26. Reserpine causes the serum level of serotonin 33. All are used in the prophylaxis of migraine
in the brain to: (Ref. KDT/6th ed. /549) except:
A. Rise A. Propranolol (Goodman-Gillman/10th ed./281)
B. Fall B. Sodium valproate (KDT/6th ed./166)
C. Has no effect C. Clonidine (CMDT/2006/946)
D. Disappear D. Sumatriptan (Davidson/19th ed. /1119)
(Ref.KDT/6th ed. /172)
27. The following statements are true about
dopamine except: (Ref. KDT/6th ed. /126) 34. All the following drugs are used in chronic pain
A. Useful in parkinsonism syndrome except:
B. No effect on diastolic blood pressure A. Phenytoin (Goodman-Gillman/10th ed./281)
C. Little effect on diastolic blood pressure B. Carbamazepine
D. Increases urine outflow in congestive heart C. Valproate
failure D. Clonazepam

21. C. Flumazenil 22. C. Clozapine 23. B. Fentanyl 24. A. Antiemetic 25. D. Seotonin
Answer
26. B. Fall 27. A. Useful in ... 28. D. Endogenous... 29. B. Dantrolene 30. B. Petit mal .....
31. C. Buprenorphine 32. A. Benzhexol 33. D. Sumatriptan 34. C. Valproate

581
Self Assessment & Review Pharmacology
35. Which of the following benzodiazepine is most B. Peripheral polyneuritis 318, 349)
conveniently used through IM route: C. Poliomyelitis
A. Diazepam (Ref.KDT/6th ed. /450) D. Spasmodic dysmenorrhea
B. Lorazepam
43. Fluoxetine can be administered following the
C. Nitrazepam
use of a MAO inhibitor after a period of:
D. Oxazepam
A. 3 days
36. Clozapine is used in: (Ref.KDT/6th ed. /429) B. 14 days
A. Catatonic schizophrenia C. 21 days
B. Resistant schizophrenia D. 30 days
C. Manic depressive psychosis
44. Drug of choice in trigeminal neuralgia:
D. All of the above
A. Carbamazepine (Ref. KDT/6th ed. /406)
37. Hit and run drug is: (Ref.KDT/6th ed. /549) B. Valproate
A. Reserpine C. Misoprostol
B. Prazosin D. Sucralfate
C. Methyldopa
45. Emergent hallucination is seen in:
D. Propranolol
A. Chloroform (Ref. KDT/6th ed. /376)
38. Which of the following drug influence the B. Ether
splanchnic circulation: (Ref.KDT/6th ed. /126) C. Halothane
A. Adrenaline D. Ketamine
B. Noradrenaline
46. Malignant neuroleptic syndrome is seen in :
C. Dopamine
A. Amoxapine (Ref. KDT/6th ed. /445)
D. Isoprenaline
B. Doxepine
39. Doxapram is: (Ref.KDT/6th ed. /470) C. Imipramine
A. Analeptic D. Selegiline
B. Analgesic
47. Morphine acts on: (Ref. KDT/6th ed. /455)
C. Sedative
A. CTZ
D. Anticonvulsant
B. Vomiting centre
40. Extrapyramidal syndrome is least in: C. Pons
A. Chlorpromazine (Ref.KDT/6th ed. /430) D. Cerebellum
B. Haloperidol
48. Which one is serotonin selective agent:
C. Olanzapine
A. Fluoxetine (Ref. KDT/6th ed. /439)
D. Fluphenazine
B. Nortriptyline
41. Clozapine has been withdrawn in the treatment C. Amitriptyline
of schizophrenia due to: (KDT/6th ed./429) D. Bupropion
A. Tardive dyskinesia
49. Which one is MAO-B selective agent:
B. Agranulocytosis
A. Selegiline (Ref. KDT/6th ed. /415)
C. Parkinsonism
B. Clorgiline
D. Bradycardia
C. Moclobemide
42. Baclofen is not used in : D. Cyproterone
A. Cerebral palsy (Ref. KDT/6th ed. /317,

35. B. Lorazepam 36. B. Resistant .... 37. A. Reserpine 38. C. Dopamine 39. A. Analeptic
Answer
40. C. Olanzepine 41. B. Agranulocytosis 42. A. Cerebral palsy 43. B. 14 days 44. A. Carbamazepine
45. A. Ketamine 46. A. Amoxapine 47. A. CTZ 48. A. Fluoxetine 49. A. Selegiline

582
Questions fr om Bihar
from Bihar,, Jharkhand, K ar
Kar natak
arnatak a, Kolk
nataka, ata MP
Kolkata MP,, UP Entrance Examinations (2000-2006)

50. GABA transaminase inhibitor is: 57. Flumazenil is (Ref. KDT/6th ed. / 394)
A. Phenobarbitone (Ref. KDT/6th ed. /407) A. Benzodiazepine agonist antagonist
B. Valproate B. Benzodiazepine agonist
C. Carbamazepine C. Benzodiazepine antagonist
D. Tricyclic antidepressant D. Inverse agonist of benzodiazepine
51. Non-selective MAO inhibitor is 58. Pentylenetetrazol induced seizures can be
A. Moclobemide (Ref.KDT/6th ed. / 439-440) prevented by all except
B. Clorgiline A. Clonazepam (Ref. KDT/6th ed. / 405, 406)
C. Phenelzine B. Valproate
D. Selegeline C. Ethosuximide
D. Carbamazepine
52. Psychosis is induced by the use of
(Ref.KDT/6th ed. / 286, Harrison 17th ed. / 436) 59. Flushing with alcohol is seen in all except
A. Steroids A. Chlorpropamide(Ref. Goodman and Gilman’s
B. Atropine B. Amoxicillin 10/e,p 44; KDT6/e,p 701)
C. Morphine C. Metronidazole
D. Penicillin D. Furazolidone
53. Prostaglandin with maximum ulceroprotective 60. Allopurinol potentiates the action of
effect A. Ampicillin (Ref. KDT/6th ed. / 208)
A. PGF2á (Ref. KDT/6th ed. /-178) B. Probenecid
B. PGI2 C. Corticosteroids
C. PGE2 D. 6-mercaptopurine
D. PGE1 61. Drug, which is absolutely contraindicated in
54. Drug automation is seen in poisoning with acute intermittent porphyria
A. Dhatura (Ref. KDT/6th ed. / 391) A. Digoxin (Ref. Harrison 17th ed. / ed. /
B. Phenytoin 432,2264)
C. Barbiturates B. Pencillin
D. Benzodiazepines C. Glucococorticoids
D. Barbiturates
55. Granisetron is a (Ref. KDT/6th ed. / 647)
62. Which drug causes hypertensive crisis in a
A. 5HT3 blocker
patient on MAO inhibitor therapy
B. GABA agonist
(Ref. KDT/6th ed. / 440)
C. 5HT3 agonist
A. Norepinephrine
D. GABA antagonist
B. Guanethidine
56. Hemodialysis is helpful in poisoning with all C. Tyramine
except D. Phenobarbitone
(Ref. Harrison 17th ed. / 2601; 16/e,p 2586)
63. All are side effects of amitriptyline except
A. Methanol
A. Dry mouth (Ref. KDT/6th ed. / 444)
B. Barbiturates
B. Weight loss
C. Salicylates
C. Constipation
D. Benzodiazepines
D. Fine tremors

50. B. Valproate 51. C. Phenelzine 52. A. Steroids 53. C. PGE 54. C. Barbiturates
Answer .....
55. A. 6HT blocker 57. C. Benzodiaze.. 58. D. Carbamazepine 59. B. Amoxicillin 60. D. 6-mercaptopu...
3
61. D. Barbiturates 62. C. Tyramine 63. B. Weight loss

583
Self Assessment & Review Pharmacology
64. Which of the following coenzyme abolishes 70. The drug antabuse acts by
the therapeutic effect of levodopa (Ref. KDT/6th ed. / 386; Martindale The Complete
A. Carbidopa (Ref. KDT/6th ed. / 418) Drug Ref. 32/e,p 1673)
B. Thiamine A. Blocking oxidation of alcohol
C. Pyridoxine B. Increasing pulmonary excretion of ethanol
D. Benserazide C. Enhancing urinary excretion of ethanol
D. Decreases ethanol absorption
65. Minimal extrapyramidal side effects are seen
with Pharmacology of Cardiovascular &
(Ref. KDT/6th ed. / 429; Ahuja 4/e,p 166) Renal System
A. Thioridazine
1. For a pregnant lady, which antihypertensive
B. Haloperidol
is not given : (Ref. KDT/6th ed. /553)
C. Clozapine
A. ACE inhibitor
D. Prochlorperazine
B. Calcium channel blocker
66. Which is not metabolized in liver C. â Blocker
A. Diazepam (Ref. KDT/6th ed. / 450) D. None
B. Oxazepam
2. All are vasodilator drugs except :
C. Nitrazepam
A. Methyl dopa (Ref. KDT/6th ed. /504)
D. Lorazepam
B. Nitroprusside
67. Depression is not a side effect of C. Hydralazine
(Ref. Harrison 17th ed. / 436;KDT/6th ed. / D. None of the above
429; Satoskar 18/e,p 939)
3. Which of the following is not an ionotropic
A. Propranolol
drug: (Ref. KDT/6th ed. /549)
B. Oral contraceptives
A. Amrinone
C. L-dopa
B. Nitroprusside
D. Flupenthixol
C. Amiodarone
68. Drug of choice for myoclonic seizures is D. Phenylephrine
(Ref. Harrison 17th ed. / 2363; KDT/6th ed. / 408)
4. A patient on cisplatin, which of the following
A. Phenytoin
diuretics would be preferred :
B. Valproic acid
A. Mannitol (Ref. KDT/6th ed. /572)
C. Phenobarbitone
B. Acetazolamide
D. Carbamazepine
C. Thiazide
69. Which one of the following drugs is used to D. Frusemide
treat status epilepticus (Ref. KDT/6th ed. / 409)
5. Pethidine as compared to morphine :
A. Sodium valproate
A. Dose not suppress cough
B. Diazepam
B. Aggravates the cough(Ref. KDT/6th ed. /458)
C. Primidone
C. Suppress cough
D. Carbamazepine
D. None

64. C. Pyridoxine 65. C. Clozapine 66. B. Oxazepam 67. D. Flupenthixol 68. B. Valproic acid
Answer
69. B. Diazepam 70. A. Blocking .... 1. A. ACE inhibitor 2. A. Methyl dopa 3. B. Nitroprusside
4. A. Mannitol 5. A. Dose not ...

584
Questions fr om Bihar
from Bihar,, Jharkhand, K ar
Kar natak
arnatak a, Kolk
nataka, ata MP
Kolkata MP,, UP Entrance Examinations (2000-2006)

6. False about digoxin is : D. Tumor necrosis factor


A. Oral dose more than parenteral dose
14. ACE receptor antagonist is :
B. Onset of action after 6 hours and t1/2 is 48
A. Losartan (Ref. KDT/6th ed. /483)
hours (Ref. KDT/6th ed. /494 (d), 497 (b))
B. Enalapril
C. Preterm child need more than term child
C. Ramipril
D. Increases the heart rate
D. Captopril
7. All are antiplatelet drugs except :
15. Indication of ACE inhibitor in diabetes mellitus is :
A. Clopidogrel (Ref. KDT/6th ed. /609)
A. Type I nephropathy (Ref. KDT/6th ed. /487)
B. Abciximab
B. Type II nephropathy
C. Ticlopidine
C. Both of the above
D. Aprotinine
D. None of the above
8. All are dihydropyridines except :
16. Cough and angioedema in ACE inhibitor is due to :
A. Nifedipine (Ref. KDT/6th ed. /522)
A. Bradykinin (Ref. KDT 5/e, p 483)
B. Nimodipine
B. Renin
C. Verapamil
C. Angiotensin-II
D. Felodipine
D. All of the above
9. Class III antiarrhythmic drug is :
17. Which of the following decreases A-V conduction :
A. Amiodarone (Ref. KDT/6th ed. /511)
A. Esmolol (Ref. KDT/6th ed. /494)
B. Phenytoin
B. Digitalis
C. Propafenone
C. Lidocaine
D. Pindolol
D. Verapamil
10. Triamterine causes : (Ref. KDT/6th ed. /571)
18. Supraventricular arrhythmia can be treated with:
A. Hypokalemia
A. Adenosine (Ref. OHCM-p-112, 518)
B. Muscle cramps
B. Flecainide
C. Decrease in urea level
C. Tocainide
D. Better glucose tolerance
D. Disopyramide
11. About quinidine, which is correct :
19. The major clinical use of nimodipine is in:
A. High doses causes increase in blood pressure
A. Hypertension (Ref. KDT /6th ed. /532)
B. It induces vagus stimulation(Ref. KDT/6th ed.
B. Angina pectoris
C. It decreases automaticity in heart /571)
C. Subarachnoid hemorrhage
D. It has antianginal property
D. Raynaud’s phenomenon
12. Treatment of choice in digoxin toxicity :
20. Loop diuretics such as furosemide act by:
A. Digoxin antibody (Ref. KDT/6th ed. /499)
A. Decreasing tubular reabsorption of Na+ and
B. Lidocaine
increased GFR (Ref. Davidson’s-p-563)
C. Phenytoin
B. Decreasing H secretion with resultant increase
+
D. Dialysis
in Na+ and K+ excretion
13. Enalapril increases : (Ref. KDT/6th ed. /483) C. Inhibiting Na+ -K + -2Cl – cotransporter in the
A. Bradykinin medullary thick ascending limb
B. Interferon D. Inhibiting Na+ -K+ exchange in the collecting
C. Platelet activating factor duct

6. D. Increases the ... 7. D. Aprotinine 8. C. Verpamil 9. A. Amidarone 10. B. Muscle cramps


Answer
11. C. It decreased .... 12. A. Digoxin antibody 13. A. Bradickynin 14. B. Enlapril 15. C. Both of...
16. A. Bradykinin 17. B. Digitalis 18. A. Adenosine 19. C. Subarachnoid ...20. C. Inhibiting ......

585
Self Assessment & Review Pharmacology
21. The following drugs cause methemo- D. Verapamil
globinemia: (Ref. HPIM-p-524, 452) 28. Which of the following is a cerebroselective
A. Aniline
calcium channel blocker?
B. Dapsone
A. Cefalexin (Ref. KDT/6th ed. /172)
C. Nitrates B. Cefuroxime
D. All of the above
C. Flunarizine
22. Sudden withdrawal of which of the following D. Diltiazem
drug could result in serious adverse 29. Hyperkalemia may occur if potassium-sparing
cardiovascular changes in a patient taking the
diuretics are given along with:
drug over a long time: (Ref. KDT/6th ed. /546,
A. Captopril (Ref. KDT/6th ed. /204, 484)
HPIM-446) B. Atenolol
A. Phenelzine (MAO inhibitor)
C. Prazosin
B. Enalapril (ACE inhibitor)
D. Clonidine
C. Clonidine (a2 agonist)
D. Fluoxetine (serotonin reuptake inhibitor) 30. The following drugs are useful in diabetes
insipidus except: (Ref. HPIM-p-2057)
23. Side effects of thiazides may include:
A. Desmopressin
A. Hypokalemia (Ref. KDT/6th ed. /566, 567) B. Lithium
B. Hyperuricemia
C. Chlorpropamide
C. Hyperglycemia D. Hydrochlorthiazide
D. All of the above
31. “Slow” ventricular tachycardia requires:
24. ACE inhibitors are contraindicated in all of the A. Intravenous verapamil (Ref. HPIM-p-304)
following except: (Ref. HPIM-p-486-487, KDT) B. Intravenous digoxin
A. Pregnancy C. Oral quinidine
B. Diabetes D. Nitrendepine
C. Bilateral renal artery stenosis
D. Renal failure 32. Widely used antiarrhythmic calcium channel
antagonist:
25. Furosemide is useful in: (p-566, 567) A. Amlodipine (Ref. KDT/6th ed. / 517)
A. Hypertension B. Nifedipine
B. Refractory edema C. Nitrendepine
C. Hypocalcemia D. Verapamil
D. Hypokalemia
33. Nocturnal leg cramps can be relieved by:
26. Cisapride is useful in: (Ref. HPIM-p-240, 645) A. Quinine (Ref. KDT/6th ed. /348)
A. Gastrointestinal reflux disease B. Chloroquine
B. Gastrointestinal spasm C. Sulfadoxine
C. Carcinoid syndrome D. Pyrimethamine
D. Gastrointestinal hypermotility
34. Postural hypotension is a common side effect of:
27. Choice of calcium channel blocker in A. Clonidine (Ref. KDT/6th ed. /549-551)
subarachnoid hemorrhage is: B. Guanethidine
A. Nifedipine (Ref. KDT/6th ed. /532) C. Beta blockers
B. Nimodipine D. Angiotensin
C. Diltiazem
21. D. All of the ... 22. C. Clonidine ... 23. D. All of the ... 24. B. Diabetes 25. D. Hypokalemia
Answer
26. A. Gastrointestinal ...27. B. Nimodipine 28. A. Cefalexin 29. A. Captopril 30. B. Lithium
31. B. 32. D. Verapamil 33. A. Quinine 34. B. Guanethidine

586
Questions fr om Bihar
from Bihar,, Jharkhand, K ar
Kar natak
arnatak a, Kolk
nataka, ata MP
Kolkata MP,, UP Entrance Examinations (2000-2006)

35. Which of the following statements about 42. Diuretics commonly used in chemotherapy:
dopamine is correct: (Ref. KDT/6th ed. /126) A. Spironolactone (KDT/6th ed./536)
A. Used in congestive cardiac failure B. Lasix
B. Its plasma half life is 6 hours C. Mannitol
C. Therapeutically useful in Parkinsonism D. Thiazide
D. Act only on dopaminergic receptors
43. Role of colchicine in acute gout is:
36. Which of the following diuretics promote (Ref.KDT/6th ed. /206)
calcium reabsorption: (Ref. KDT/6th ed. / 567) A. Inhibits xanthine oxidase
A. Chlorothiazide B. Prevents chemotaxis and inflammation
B. Furosemide C. Increases uric acid excretion
C. Acetazolamide D. Stimulates granulocyte migration
D. Mithramycin
44. Cardiotoxicity is caused by:
37. All of the following antihypertensives decrease (Ref.KDT/6th ed. /206)
plasma renin activity except: A. Terfenadine with metronidazole
A. Clonidine (Ref. KDT/6th ed. / 564) B. Cyclosporine with verapamil
B. Methyldopa C. Fluconazole with rifampicin
C. Atenolol D. Erythromycin with beta blocker
D. Chlorothiazide
45. K+ sparing diuretic: (KDT /6th ed.561)
38. Frusemide causes all except: A. Thiazides
A. Hyperglycemia (Ref. KDT/6th ed./ 562) B. Ethacrynic acid
B. Hypomagnesemia C. Acetazolamide
C. Hypokalemia D. Amiloride
D. Acidosis
46. Drug of choice in a case of LVH with hyper-
39. Which is least nephrotoxic: tension:
A. Ciprofloxacin (Ref. KDT/6th ed. /691) A. ACE inhibitor (Ref. KDT/6th ed. /551)
B. Ofloxacin B. Beta blocker
C. Lomefloxacin C. Ca++ channel blocker
D. Pefloxacin D. Diuretics
40. Renin secretion is increased by all except: 47. All of the following is true regarding Losartan
A. a-adrenergic drug (Ganong/21st ed//462) except:
B. Vasopressin A. Cough (Ref. KDT/6th ed. /488)
C. Prostaglandin B. No angioedema
D. Catecholamine C. No hypotension in high renin hypertension
41. Palpitation is seen in all except: D. No fetal teratogenicity

(CMDT/2006/278,309,371,1139) 48. All are false regarding minoxidil except


A. Thyrotoxicosis A. K+ channel opener (Ref. KDT/6th ed. /-548)
B. Pheochromocytoma B. K+ channel blocker
C. Hypoglycemia (Davidson/19th ed./414) C. Predominant venodilator
D. A-V block (Harrison/17th ed. /81) D. Causes alopecia

35. A. Used in ..... 36. A. Chlorothiazide 37. D. Chlorothiazide 38. D. Acidosis 39. D. Pefloxacin
Answer
40. B. Vasopressin 41. D. A-V block 42. C. Mannitol 43. B. Prevents .... 44. .....................
+
45. D. Amiloride 46. A. ACE inhibitor 47. A.Cough 48. A. K channel ....

587
Self Assessment & Review Pharmacology
49. Digitalis acts by inhibiting 56. Ouabain acts through
A. c-AMP (Ref. KDT/6th ed. / 496) (Ref. KDT/6th ed. / 493, KDT/6th ed. / 496)
B. K+-H+ ATPase A. Na+-K+ ATPase
C. Na+-H+ ATPase B. Ca++ channels
D. Na+-K+ ATPase C. Adenyl cyclase
D. H+ K+ ATPase
50. All are adverse effects of captopril except
A. Hypotension (Ref. KDT/6th ed. / 484) 57. Drug not to be given in ischemic heart disease
B. Hypokalemia A. Atenolol (Ref. Harrison 17th ed. / 1396)
C. Cough B. Streptokinase
D. Dysguesia C. ACE inhibitor
D. Isoproteronol
51. Which of the following is the drug of choice
for paroxysmal supraventricular tachycardia 58. All are indications for calcium channel blockers
A. Digoxin (Ref. KDT/6th ed. / 518) except
B. Amiodarone A. Angina pectoris (Ref. KDT/6th ed. / 543)
C. Verapamil B. Hypertension
D. Adenosine C. Arrhythmias
D. Congestive heart failure
52. Drug not useful in acute myocardial infarction
A. Antithrombin (Ref. KDT/6th ed. /-491) 59. Drug useful in cardiogenic shock is
B. Thrombolytics (Ref. KDT/6th ed. / 126; Harrison 17th ed. /
C. Plasminogen activator inhibitor 226,1396; 16/e,p 1616)
D. Antiplatelets A. Dopamine
B. Digoxin
53. Verapamil is used in (Ref. KDT/6th ed. / 517)
C. Atropine
A. Digitalis toxicity
D. Phenylepherine
B. Sick sinus syndrome
C. Ventricular tachycardia 60. Nitrates decreases myocardial oxygen consumption
D. Paroxysmal supraventricular tachycardia by all of the following mecha-nisms except
54. Which of the following is wrongly matched (Ref. Goodman and Gilman’s 10/e,p 847;KDT/6th
combination of antiarrhythmic drugs and their ed. / 523)
class A. Increasing the left ventricular end-diastolic
A. Lignocaine-IA (Ref. KDT/6th ed. / 511) pressure
B. Mexilitine-IB B. By dilatation of the capacitance vessels
C. Amiodarone-III C. By decreasing the size of heart
D. Verapamil-IV D. Reduction of cardiac work

55. Mechanism of action of Na-nitroprusside is Pharmacology of Respiratory, Endocrinal &


(Ref. Goodman and Gilman’s 10/e,p; KDT/6th Gastrointestinal System
ed. / 523, 524) 1. Which among is not SERMs
A. c-AMP A. Flutamide (Ref. KDT/6th ed. /303)
B. K+ channels B. Ormeloxifen
C. Ca++ channels C. Tamoxifen
D. Guanylate cyclase D. Raloxifene

+ +
49. D. Na -K AT ... 50. B. Hypokalemia 51. D. Adenosine 52. C. Plasminogen ... 53. D. Paroxysmal ...
Answer + +
54. A. Lignocaine-IA 55. D. Guanylate .... 56. A. Na -K AT ... 57. D. Isoproteronol 58. D. Congestive ....
59. A. Dopamine 60. A. Increasing ...... 1. A. Flutamide

588
Questions fr om Bihar
from Bihar,, Jharkhand, K ar
Kar natak
arnatak a, Kolk
nataka, ata MP
Kolkata MP,, UP Entrance Examinations (2000-2006)

2. Which among the following is not an androgen C. Psychosis


receptor blocker D. Growth retardation
A. Finasteride (Ref. KDT/6th ed. /293, 294)
10. Granulomatous hepatitis is due to :
B. Cyproterone
A. Methyl dopa
C. Flutamide
B. Valproic acid
D. None of the above
C. Amiodarone
3. Which is a mast cell stabilizer: D. Danazol
A. Nedocromil (Ref. KDT/6th ed. /223)
11. Heparin does not cause :(Ref. KDT/6th ed. /597)
B. Budesonide
A. Osteoporosis
C. Zileuton
B. Factor V inhibition
D. None of the above
C. Thrombocytopenia
4. Which of following is a cough suppressant D. Prolong APTT
A. Pholocodeine (Ref. KDT/6th ed. /213)
12. Which is fastest calcium lowering agent :
B. Acetylcystine
A. IV calcitonin (Ref. KDT/6th ed. /335)
C. Guargum
B. Plicamycin
D. None of the above
C. Etidronate
5. All are protease inhibitors except : D. Zolendronate
A. Ritonavir (Ref. KDT/6th ed. /768)
13. In which of the following disease is cortico-
B. Amprenavir
steroids indicated? (Ref. KDT-6–p 284)
C. Tenofovir
A. Osteoporosis
D. Nelfinavir
B. Peptic ulcer
6. All cause hypothyroidism except : C. Collagen disease
A. Rifampicin (Ref. KDT/6th ed. /249) D. Tuberculosis
B. Captopril
14. Following are the adverse effects of estrogens
C. Lithium
except : (Ref. KDT-6/e p-300)
D. Amiodarone
A. Suppression of libido
7. Insulin having longest duration of action : B. Fusion of epiphyses
A. Isophane insulin (Ref. KDT/6th ed. /259) C. Hot flushes
B. PZI D. Gynaecomastia in males
C. Insulin zinc suspension
15. The antithyroid drug preferred for use during
D. Plane insulin
pregnancy and lactation is: (p-25)
8. Which of the following is not a starting criteria A. Propylthiouracil
for sulfonylurea therapy : B. Methimazole
A. Total pancreatectomy C. Carbimazole
B. NIDDM (Ref. KDT/6th ed. /266) D. Radioactive iodine
C. Diabetes after 60 years
16. Heparin therapy is monitored by:
D. None of the above
A. Bleeding time (Ref. KDT/6th ed. /598)
9. Side effects of steroids are all except : B. Coagulation time
A. Hypoglycemia (Ref. KDT/6th ed. /280) C. Prothrombin time
B. Hypertension D. Partial thromboplastin time

2. A. Fiansteride 3. A. Nedocromoil 4. D. None 5. C. Tenofovir 6. B. Captopril


Answer
7. B. PZI 8. A. Total ...... 9. A. Hypoglecmia 10. A. Methyldopa 11. B. Factor V .....
12. D. Zolendronate 13. C. Collagen disease 14. C. Hot flushes 15. A. Propylthiouracil 16. D. Partial throm......

589
Self Assessment & Review Pharmacology
17. Iodophores are a mixture of the following: 24. Choice of drug by inhalation in acute asthma
A. Iodine and alcohol (Ref. KDT/6th ed. /859, not responding to salbutamol is:
B. Iodine and aldehyde 860) A. Ipratropium bromide
C. Iodine and surface active agents B. Atropine sulfate (Ref. KDT/6th ed. /222)
D. Iodine and phenols C. Isopropamide
D. Hyoscine methyl bromide
18. People with high risk of developing breast
cancer can be prevented by prophylactic 25. Epsilon aminocaproic acid is used to reduce
administration of: (Ref. KATZ-p-304) bleeding due to: (Ref. KDT/6th ed. /611)
A. Tamoxifen A. Heparin
B. Aminoglutethimide B. Warfarin
C. Diethylstilbestrol C. Thrombocytopenia
D. Flutamide D. Hyperplasminaemia

19. Long acting insulin preparations are frequently 26. Long-term complications of steroid therapy
administered by: (Ref. KDT/6th ed. /260) include all except: (Ref. KDT/6th ed. /286)
A. Oral route A. Growth retardation
B. Intramuscular route B. Cataract
C. Intradermal route C. Potbelly
D. Subcutaneous route D. Stress fracture

20. All of the following drugs are used in the 27. Anabolic steroids may produce the following
management of obesity except: side effects except: (Ref. KDT/6th ed. /286)
A. Metformin (Ref. HPIM-485) A. Precocious puberty in children
B. Sibutramine B. Cholestatic jaundice
C. Cyproheptadine C. Delayed closure of epiphysis in children
D. Fenfluramine D. Acne in males and females
21. Albendazole is effective against all except: 28. Mebeverine hydrochloride is useful in:
A. Ascariasis (Ref. HPIM-p-810) A. Ulcerative colitis
B. Neurocysticercosis B. Irritable bowel syndrome
C. Echinococcus granulosus C. Peptic ulcers (Ref. DSON-p-471, 17th edn.)
D. Schistosomiasis D. Crohn’s disease
22. Advantage of salmeterol over sulbutamol is 29. Antidote for heparin overdose is:
its: A. Vitamin K (KDT/6th ed. /600)
A. Shorter duration of action (Ref. HPIM-p-218) B. Protamine sulphate
B. More potency C. Dextran sulphate
C. Longer duration of action D. Vitamin C
D. Lesser cardiac effects
30. Which one of the following drugs is effective in
23. Low molecular weight heparin inhibits clotting the treatment of benign prostatic hyperplasia?
factor: A. Amlodipine (Ref. KDT/6th ed. /136, 294)
A. IIa (Ref. KDT/6th ed. /599) B. Finasteride
B. IXa C. Clonidine
C. Xa D. Glycopyrrolate
D. XIa
17. C. Iodine and .... 18. A. Tamoxifen 19. D. Subcutaneous ... 20. C. Cyproheptadine 21. D. Schistosomiasis
Answer
22. C. Longer ....... 23. C. Xa 24. A. Ipratropium .... 25. D. Hyperplasmin....26. C. Potbelly
27. C. Delayed ...... 28. B. Irritable ...... 29. B. Protamine....... 30. B. Finasteride

590
Questions fr om Bihar
from Bihar,, Jharkhand, K ar
Kar natak
arnatak a, Kolk
nataka, ata MP
Kolkata MP,, UP Entrance Examinations (2000-2006)

31. The drug producing hyopolipidemic effect by 38. All of the following are true regarding chlor-
inhibiting HMG-CoA reductase enzyme is: propamide except:
A. Nicotinic acid(Ref. KDT/6th ed. /614, HPIM) A. It is short acting (Ref.KDT/6th ed. /267)
B. Gemfibrozil B. It can cause hypoglycaemia in elderly
C. Guggulipid C. Causes weight gain
D. ............... D. Associated with alcoholic flush

32. Which of the following statements about oral 39. Steroid causes: (KDT/6th ed./244)
contraceptives (OCPs) is true? A. Increased TSH
A. Estrogens act by making cervical mucus thick B. Increased FSH
B. Venous thrombosis is due to progestins C. Prevent de-iodination
C. Mestranol is a progestin used in combination D. All of the above
pills (Ref. KDT/6th ed. / 315, 316) 40. All the drugs cause diarrhea except:
D. Nausea and vomiting are common side effects A. Diuretics
33. Which of the following anticholinergic is B. B-blocker
claimed to act selectively on bronchial C. ACE inhibitor
muscle? D. Calcium channel blocker
A. Isopropamide (Ref. KDT- 112) 41. Which of the following hypolipidemic agent
B. Ipratropium has antioxidant property: (KDT/6th ed./581)
C. Benztropine A. Clofibrate
D. Pirenzapine B. Probucol
C. Gemfibrozil
34. The following are likely adverse reactions of
D. Lovastatin
oral contraceptives except:
A. Thromboembolism 42. Sulfonylurea acts by all of the mechanisms except:
B. Weight gain (Ref. KDT/6th ed. /315, 316) A. Increase secretion of insulin
C. Carbohydrate intolerance B. Increase synthesis of insulin
D. Lactorrhea C. Increase insulin receptor in the peripheral tissue
D. Reverse the down regulation of insulin
35. Anticoagulant of choice in pregnancy:
receptors (Ref.KDT/6th ed. /266)
A. Heparin (Ref.KDT/6th ed. / 598)
B. Warfarin 43. As a component of ORS, function of glucose is:
C. Discumarol A. Helps in sodium absorption
D. Dimercaprol B. Adds taste (Ref.KDT/6th ed. /658)
C. Increase shelf-life of ORS
36. Which one has least mineralocorticoid activity:
D. Arrests further diarrhea
A. Cortisol (Ref. KDT/6th ed. /282)
B. Prednisolone 44. All of the following are true except:
C. Fludrocortisone A. Systemic steroids are to be given in acute
D. Methylprednisolone severe asthma (Ref.KDT/6th ed. /224-225)
B. IV steroid gives early relief in acute severe
37. Not true about the composition of ORS:
asthma
A. NaCl-3.6 gm (Ref. KDT/6th ed. /659) C. Inhalational steroids are to be given in
B. KCI-1.6 gm persistent attack of asthma
C. Bicarbonate – 2 gm D. Inappropriate administration of steroid causes
D. Glucose – 2 gm adrenal suppression
31. 32. D. Nausea and .... 33. B. Ipratropium 34. D. Lactorhoea 35. A. Heparin
Answer
36. D. Methylpredniso....37. C. Bicarbonate-2 gm 38. A. It is short .... 39. C. Prevent ....... 40. D. Calcium .....
41. B. Probucol 42. B. Increase ........ 43. A. Helps in ..... 44. B. IV steroid ....

591
Self Assessment & Review Pharmacology
45. Steroid causes increase in blood glucose level D. HDL and VLDL
by: (KDT/6th ed./277)
A. Decreasing the rate of gluconeogenesis 52. In insulin resistance, daily insulin requirement
B. Inhibiting hormone senstitive lipase is more than: (Ref. KDT/6th ed. /265)
C. Decreasing uptake of glucose by peripheral A. 100 units
tissues B. 200 units
D. Reducing utilization of glucose in the cells C. 300 units
D. 400 units
46. Protamine is a selective antagonist of:
A. Diazepam (Ref.KDT/6th ed. /600) 53. Cytoprotective action of which drug is used to
B. Heparin treat peptic ulcer: (Ref. KDT/6th ed. /634)
C. Morphine A. Ranitidine
D. Warfarin B. Famotidine
47. The drug acting by increasing insulin receptor C. Misoprostol
sensitivity: D. Sucralfate
A. Phenformin (Ref.KDT/6th ed. /269-270)
B. Rosiglitazone 54. T ½ of insulin is: (Ref. KDT/6th ed. /259)
C. Acarbose A. 1-2 minutes
D. Glimepiride B. 6-9 minutes
C. 20 minutes
48. GnRH agonist used as nasal spray in the
D. 46 minutes
treatment of endometriosis:
A. Leuprolide (Ref. KDT/6th ed./239) 55. 5-alpha reductase inhibitor is:
B. Gonadorelin A. Finasteride (Ref. KDT/6th ed. /294)
C. Nafarelin B. Fluoxetine
D. Histrelin C. Flutamide
49. Partial estrogenic agonist used for palliative D. Cyproterone
therapy of postmenopausal breast carcinoma
56. Lispro insulin is given: (Ref. KDT/6th ed. /261)
is:
A. Tamoxifen (Ref. KDT/6th ed. /304) A. Within ½ hour
B. Clomiphene B. Postprandial glucose
C. Tibolone C. After preprandial glucose
D. Natural estrogen D. All of the above
50. LMW heparin is different from HMV heparin in 57. OCP protects against:
all except: (Obstetrics-DCD/6th ed./684)
A. It can be used at home in a case of DVT as an A. Osteosclerosis
injection (Ref. KDT/6th ed./562,599) B. Pathologic fracture
B. Less chance of osteoporosis with equal C. Osteochondritis
efficacy
D. Rickets
C. More protein bound
D. Less chance of thrombocytopenia 58. Which of the following does not decrease
bronchial secretion: (Ref. KDT/6th ed. /108)
51. Nicotinic acid acts by all except:
A. Atropine
A. HDL degradation (Ref. KDT/6th ed. /580,618)
B. Alcohol
B. Decreased cholesterol synthesis
C. Imipramine
C. Degrades fibrinogen
D. Phenothiazine

45. ................... 46. B. Heparin 47. B. Rosiglitazone 48. C. Nafarelin 49. A. Tamoxifen
Answer
50. C. More protein ... 51. A. HDL degradation 52. B. 200 units 53. C. Misoprostol 54. B. 6-9 minutes
55. A. Finasteride 56. A. Within ½ hours 57. B. Pathologic..... 58. B. Alcohol

592
Questions fr om Bihar
from Bihar,, Jharkhand, K ar
Kar natak
arnatak a, Kolk
nataka, ata MP
Kolkata MP,, UP Entrance Examinations (2000-2006)

59. Which is false regarding ketotifen on factor IIa


A. Bronchodilator (Ref. KDT/6th ed. / 224) B. Inactivation of factor VIIa
B. Is orally active C. Activating antithrombin III
C. Can be used in prophylaxis of asthma D. Converting prothrombin into thrombin
D. Acts by inhibiting mediator release from mast
66. Theophylline causes all except
cells
(Ref. KDT/6th ed. / 220,221; Satosker 16/e,p 176)
60. One of the following drug is used in the A. Diuresis
treatment of peptic ulcer B. Tinnitus
A. Tropicamide (Ref. KDT/6th ed. /-628) C. Arrhythmias
B. Pirenzipine D. Anaphylaxis
C. Atropine
67. Tamoxifen is given in
D. Ipratropium
(Ref. KDT/6th ed. / 304; CMDT 2003/p 1366)
61. Drug causing primary rapidly progressive A. Carcinoma ovary
pulmonary fibrosis B. Carinoma prostate
(Ref. Craig,p 106; Nelson 16/e,p 1292, CMDT- C. Estrogen receptor positive breast-carcinoma
2004,p 1636) D. Seminoma
A. Paraquat 68. Corticosteroids causes all except
B. Organophosphorus (Ref. KDT/6th ed. / 283, 284, 285)
C. Kerosine A. Psychosis
D. Amiodarone B. Muscular hypertrophy
62. All are inhalational steroids except C. Peptic ulceration
A. Dexamethasone (Ref. KDT 6/e, p282) D. Suppression of pituitary-adrenal axis
B. Budesonide 69. Thromboembolism is due to which component
C. Trimcinolone acetonide of oral pills (Ref. KDT/6th ed. / 300)
D. Beclomethasone A. Iron
B. FSH
63. OCP failure is seen with the use of
C. Estrogen
A. INH (Ref. KDT 6/p-317)
D. Progesterone
B. Pyrazinamide
C. Rifampicin 70. Which among the following is a spermat-
D. Amphotericin B ogenesis inhibitor (Ref. KDT/6th ed. / 318)
64. Which of the following belongs to steroid A. Gossypol
receptor super family B. Gestodene
(Ref. KDT/6th ed. / 330, 331, Harpar 26/e,p 684) C. Gelusil
A. Insulin D. Gepirone
B. Gastrin 71. Antidiabetic effect of sulfonylureas is by
C. Vit. D3 A. Increasing insulin secretion
D. Encephalin B. Tissue sensitivity to insulin
C. Tissue sensitivity to glycogen
65. Heparin acts by (Ref. KDT/6th ed. / 597)
D. Glucose production
A. Selective inhibition of factor Xa with little effect

59. A. Bronchodilator 60. B. Pirenzipine 61. A. Paraquate 62. A. Dexamethas... 63. C. Rifampicin
Answer
64. C. Vit. D 65. C. Activating... 66. B. Anaphylaxis 67. C. Estrogen... 68. B. Muscular...
3
69. C. Estrogen 70. A. Gossypol 71. A. Increasing...

593
Self Assessment & Review Pharmacology
72. Which among the following has both C. Terfenadine
estrogenic and antiestrogenic property D. Amphotericin B
A. Clofibrate (Ref. KDT/6th ed. / 303, 304)
3. Which among the following is not an ototoxic
B. Clonidine
drug
C. Clomiphene
A. Vincristine (Ref. KDT 6th ed. /)
D. Chlorpromazine
B. Aminoglycosides
73. Drug given for metoclopramide induced C. Vinblastine
dystonic reaction (Ref. KDT/6th ed. / 157, 158) D. Penicillin
A. Chlorpromazine
4. All are alkylating agents except :
B. Prochloperazine
A. Cyclophosphamide
C. Promethazine
B. Lomustine (Ref. KDT/6th ed. /819-820)
D. Pheniramine C. Busulphan
74. Ergometrine is commonly used for D. Zalcitabine
A. Toxemia of pregnancy(Ref. KDT/6th ed. / 322) 5. All are luminal amoebicide except :
B. Dysmenorrhea A. Emetine (Ref. KDT/6th ed. /798)
C. PPH B. Diloxanide furoate
D. To shorten 2nd stage of labour C. Iodoquinol
75. In Addison’s disease, the drug to be given is D. Tetracycline
(Ref. Ref. KDT/6th ed. / 286, Harrison 17th ed. / 6.Linezolid is best for :
2099; 16/e,p 2143) A. MRSA (Ref. KDT/6th ed. /733)
A. Prednisolone B. VRSA
B. Hydrocortisone C. K. pneumoniae
C. Betamethasone D. VISA
D. DOCA
7. Which protease inhibitor has bostering effect?
76. Apparent volume of distribution of drug A. Amprenavir (Ref. KDT/6th ed. /561)
exceeds total body fluid volume, if a drug is B. Tenofovir
(Ref. KDT/6th ed. / 18, 19) C. Nelfinavir
A. Poorly soluble in plasma D. Ritonavir
B. Poorly bound to plasma protein
C. Slowly eliminated from body 8. Membranous glomerulonephropathy is caused
D. Sequestered in body tissues by:
A. Phenytoin
Antimicrobial & Antineoplastic Drugs B. Ethosuximide
1. For methicillin resistant staphylococci, the drug C. Captopril
of choice is: (Ref. KDT/6th ed. /700) D. None
A. Vancomycin 9. Single dose aminoglycoside is more
B. Metronidazole preferable than 8 hourly dose because :
C. Piperacillin
(Ref. KDT/6th ed. /720)
D. None of the above
A. MIC
2. In dermatophytes, which is not indicated: B. Increase perfusion of renal cortex
A. Fluconazole (Ref. KDT/6th ed. /758) C. Post antibiotic effect
B. Terbinafine D. None

72. C. Clomiphene 73. C. Promethazine 74. C. PPH 75. B.Hydrocortisone 76. D. Sequestred...
Answer
1. C. Pipracillin 2. D. Amphotericine B 3. D. Penicillin 4. D. Zalcitabine 5. A. Emetine
6. A. MRSA 7. D. Ritonavir 8. C. Captopril 9. C. Post antibiotic effect

594
Questions fr om Bihar
from Bihar,, Jharkhand, K ar
Kar natak
arnatak a, Kolk
nataka, ata MP
Kolkata MP,, UP Entrance Examinations (2000-2006)

10. Amikacin is used in all except : C. Piperacillin


A. Brucella (Ref. KDT/6th ed. /724) D. None
B. Serratia
18. All have antipseudomonal action except :
C. Enterococci
A. Piperacillin (Ref. KDT/6th ed. /732)
D. Staphylococci
B. Ticarcillin
11. Ampicillin is not given in EB virus infection C. Ceftazidime
due to : D. Vancomycin
A. Due to increased toxicity
19. Macrocytic anaemia is noted with all except :
B. Skin rash (Ref. KDT/6th ed. /701)
A. Phenytoin (Ref. KDT/6th ed. /689)
C. Blindness
B. Methotrexate
D. Convulsion
C. Pyrimethamine
12. Axonal sensory polyneuropathy is caused by : D. Ciprofloxacin
A. INH (Ref. KDT/6th ed. /741)
20. Most of the resistance of antituberculous drugs
B. Amiodarone
is due to:
C. Cisplatin
A. Mutation (Ref. KDT/6th ed. /740-742)
D. Lead
B. Conjugation
13. Ivermectin is used in the treatment of : C. Transduction
A. Filariasis (Ref. KDT/6th ed. /813) D. Transference
B. Ascariasis
21. Drug that is not used in renal failure:
C. Teniasis
A. Ethambutol (Ref. KDT/6th ed. /743)
D. Hookworm infestation
B. Rifampicin
14. The antibiotic to be given in pregnant women is : C. Isoniazid
A. Ciprofloxacin (Ref. KDT/6th ed. /909) D. Streptomycin
B. Cefuroxime
22. Treatment of choice of coccidiodomycosis is :
C. Metronidazole
A. Amphotericin B (Ref. KDT/6th ed. /758)
D. Chloramphenicol
B. Fluconazole
15. All are true about nitrates except : C. Flucytosine
A. Contraindicated in HOCM D. Griseofulvin
B. Indicated in severe pericarditis
23. Which antituberculous drug is not given in
C. Contraindicated in glaucoma
pregnancy:
D. Indicated in angina
A. Pyrazinamide (Ref. KDT/6th ed. /748, 909)
16. Treatment of drug resistant malaria is : B. Isoniazid
A. Quinine (Ref. KDT/6th ed. /783) C. Ethambutol
B. Pyrimethamine D. None of the above
C. Mefloquine
24. Drug amphotericin B is used in the treatment
D. Chloroquine
of :
17. Which among the following is not a beta A. Sleeping sickness (Ref. KDT/6th ed. /759)
lactamase inhibitor : B. Kala azar
A. Sulbactum (Ref. KDT/6th ed. /702) C. Malaria
B. Clavulanic acid D. Filaria

10. D. Staphalococci 11. B. Skin rash 12. A. INH... 13. A. Filariasis 14. B. Cefuroxime
Answer
15. B. Indicated in... 16. A. Quinine 17. C. Piperacillin 18. A. Vancomycin 19. D. Ciprofloxacine
20. A. Mutation 21. D. Streptomycin 22. A. Amphotericine 23. A. Pyrizinamide 24. B. Kala azar

595
Self Assessment & Review Pharmacology
25. Drug having no effect on leptospirosis : 32. Drugs used in the treatment of pneumocystis
A. Penicillin (Ref. KDT/6th ed. /699, 715, 729) carinii pneumonia include: (Ref. HPIM-1182)
B. Erythromycin A. Trimethoprim-sulfamethoxazole
C. Streptomycin B. Dapsone
D. Tetracycline C. Clindamycin
D. All of the above
26. Toxicity of ciprofloxacin is :
A. Phototoxicity (Ref. KDT/6th ed. /689) 33. Drugs useful in the treatment of chronic
B. Neurotoxicity hepatitis C include: (Ref. HPIM-1748)
C. Rash A. Glucocorticoids
D. Cartilage necrosis B. Interferon alpha
C. Hydroxyurea
27. Which one of the statements is false regarding
D. None of the above
adefovir dipivoxil? (Ref. KDT/6th ed. /778)
A. Acyclic 34. The antiretroviral drug which is also effective
B. Well tolerated in chronic active hepatitis-B infection is:
C. Used in chronic hepatitis C infection A. Zidovudine (Ref. Davidsons-p-771-772)
D. Used in anti-retroviral therapy B. Nelfinavir
C. Efavirenz
28. Patient on amphotericin ‘B’ therapy may
D. Lamivudine
develop signs and symptoms of deficiency of
one of the cations. This cation is: 35. The antibiotic known to cause Red Man’s
A. Sodium (Ref. Davidson’s-p-757, 788, 759) syndrome is:
B. Potassium A. Clindamycin (Ref. Davidsons-p-732)
C. Magnesium B. Cefadroxil
D. Calcium C. Vancomycin
D. Azithromycin
29. Drug of choice for chemoprophylaxis against
rheumatic fever is: (Ref. Davidson’s-453) 36. The following drugs are useful in the treatment
A. Penicillin V of isoniazid poisoning: (Ref. Davidsons-p-174)
B. Erythromycin A. Pyridoxine
C. Benzathine penicillin B. Diazepam
D. Procaine penicillin C. Bicarbonate
D. All of the above
30. Most important side effect of ethambutol is:
A. Hepatotoxicity (Ref. Davidson’s-p- 742) 37. Drug of choice for hookworm infestation is:
B. Renal toxicity A. Piperazine citrate (Ref. HPIM-p-809)
C. Peripheral neuropathy B. Bephenium hydroxyl naphthoate
D. Retro bulbar neuritis C. Mebendazole
D. Albendazole
31. Nucleoside analogues used in the treatment
38. Drug of choice for ascariasis is:
of HIV infection include:(Ref. KDT/6th ed. /771)
A. Piperazine citrate (Ref. HPIM-p-809)
A. Indinavir
B. Bephenium hydroxyl naphthoate
B. Nevirapine
C. Mebendazole
C. Didanosine
D. Albendazole
D. Efavirenz

25. C. Streptomycin 26. D. Cartilage... 27. C. Used in.... 28. B. Potassium 29. C. Benzathene...
Answer
30. D. Retro bulbar... 31. C. Didanosine 32. D. All of the.. 33. B. Interferon... 34. D. Lamivudine
35. C. Vancomycin 36. D. All of the... 37. C. Mebendazole 38. D. Albendazole

596
Questions fr om Bihar
from Bihar,, Jharkhand, K ar
Kar natak
arnatak a, Kolk
nataka, ata MP
Kolkata MP,, UP Entrance Examinations (2000-2006)

39. Which one of the following is a fourth 46. The antiviral drug 5-iodo 2-deoxy uridine (IUDR)
generation cephalosporin: inhibits the synthesis of:(Ref. KDT/6th ed. /767)
A. Carbamazepine (Ref. KDT/6th ed. /704) A. DNA
B. Cefpirome B. RNA
C. Cephalexin C. DNA and RNA
D. Cefuroxime D. None of the above
40. The drug of choice for malignant hyperthermia is: 47. Drug undergoing enterohepatic circulation is:
A. Carbamazepine (Ref. KDT-p 347)
B. Dantrolene
A. Sulfonamide (Ref. KDT-p 728)
C. Diazepam
B. Penicillin
D. Clonazepam
C. Erythromycin
41. Proliferation independent agents include all D. Streptomycin-
of the following except:
48. Nocturnal leg cramps can be relieved by:
A. Vincristine (Ref. KDT/6th ed. /825,
B. Carmustine Goodman and Gilman) A. Quinine (Ref. KDT/6th ed. /348)
C. Melphalan B. Chloroquine
D. Cyclophosphamide C. Sulfadoxine
D. Pyrimethamine
42. Drugs of choice for the treatment of
neurocysticercosis are: 49. Pyazinamide forms part of multidrug regimen
(Ref. KDT/6th ed. /810, 814) in TB because: (Ref. KDT-742, HPIM)
A. Hydroquinone and metronidazole A. Cheap
B. Metronidazole and pyrantel pamoate B. Less toxic
C. Albendazole and praziquantel C. No resistance
D. Praziquantel and chloroquine D. Kills intracellular organisms
43. The drug of choice for bacterial vaginosis is: 50. The cytotoxic agent induced emesis can be
A. Doxycycline (Ref. HPIM-p 799) controlled by the following drugs except:
B. Metronidazole A. Nibilone (Ref. HPIM-p- 641;KDT)
C. Penicillin B. Glucocorticoids
D. Fluconazole C. Hyoscine
D. Ondansetron
44. Which of the following is a second-generation
cephalosporin? (Ref. KDT/6th ed. /704) 51. Clavulanic acid is combined with amoxicillin
A. Cefalexin in order to: (Ref. KDT/6th ed. /702)
B. Cefuroxime A. Reduce the risk of allergic reaction
C. Ceftazidime B. Prolong the half life of amoxicillin
D. Ceftriaxone C. Reduce the side effect of amoxicillin
D. Increase antimicrobial spectrum of amoxicillin
45. Metronidazole is useful in the following
conditions except: 52. Red discoloration of urine is seen with:
A. Giardiaisis (Ref. KDT/6th ed. /799, 800) A. Chlorpromazine (Ref. KDT/6th ed. /754)
B. Guinea worm infection B. Dantrolene
C. Trichomoniasis C. Clorazepate
D. Tapeworm infestation D. Primidone

39. D. Cefpirome 40. B. Dantrolene 41. A. Vincristine 42. C. Albendazole... 43. B. Metronidazole
Answer
44. B. Cefuroxime 45. D. Tapeworm... 46. A. DNA 47. C. Erythromycin 48. A. Quinine
49. D. Kills intracellular 50. C. Hyoscine 51. D. Increase... 52. B. Dantrolene

597
Self Assessment & Review Pharmacology
53. Which of the following aminoglycosides is (Ref.KDT/6th ed. /786) (Park/18th ed./209)
most resistant to bacterial inactivating (Goodman-Gillman/10th ed. /1079)
enzymes? A. It is rapidly absorbed
A. Gentamicin (Ref. KDT/6th ed. /724) B. It is rapidly metabolized
B. Amikacin C. It has increased tissue binding
C. Tobramycin D. It is rapidly eliminated
D. Kanamycin 61. Saquinavir acts as an anti-HIV agent by acting as:
(CMDT/2004/1283,1284) (Harrison/17th ed.1902)
54. 5-fluorouracil is a: (Ref. KDT/6th ed. / 824, 831)
A. Reverse transcriptase inhibitor
A. Purine antagonist
B. Inhibits CD4+ cell proliferation
B. Folate antagonist
C. Inhibits binding of virus particle to cell
C. Pyrimidine antagonist D. Protease inhibitor (Ref.KDT/6th ed. /767)
D. Alkylating agent
62. True about extended spectrum penicillin:
55. Methotrexate is: (Ref. KDT/6th ed. /823) (Ref.KDT/6th ed. /702)
A. Folic acid antagonist A. Broad-spectrum action against anaerobes
B. Folinic acid antagonist B. Acts against Pseudomonas
C. Formamino-glutamic acid antagonist C. Works only in alkaline medium
D. None of the above D. Acts synergistically with clavulanic acid and
sulbactam
56. Which of the following antimalarials is
relatively safe in pregnancy: 63. Which of the following aminoglycoside is not
A. Primaquine (Ref. KDT/6th ed. /785, 786) ototoxic:
B. Co-trimoxazole A. Amikacin (Ref. KDT/6th ed./679)
C. Chloroquine B. Neomycin
D. Pyrimethamine C. Kanamycin
D. Streptomycin
57. Primary mechanism of action of Zidovudine is:
(Ref. KDT/6th ed. /770) 64. All are bacteriostatic except:
A. Proliferation of B and T cells A. PAS (Ref.KDT/6th ed. /669)
B. Inhibition of reverse transcriptase B. Thiacetazone
C. Activation of macrophages C. Ethambutol
D. Stimulation of lymphokines D. Pyrazinamide
58. Which of the following anticancer antibiotics 65. Which of the following is contraindicated in
is cardiotoxic: (Ref. KDT/6th ed. /833) pregnancy: (Obstetrics-DCD/6th ed./660)
A. Mitomycin A. Erythromycin (Ref.KDT/6th ed. /714)
B. Adriamycin B. Tetracycline (O.P.Ghai/6th ed./169)
C. Bleomycin C. Cephalexin
D. Mithramycin D. Penicillin
59. All causes myelosuppression except: 66. Which of the following occurs in children on
A. Docetaxel (Ref. KDT/6th ed. / 826) administration of fluoroquinolones:
B. Vincristine
A. Arthropathy (Ref.KDT/6th ed. /689)
C. Methotrexate
B. Gray baby syndrome
D. Irinotecan
C. Enamel hypoplasia
60. Chloroquine given in 600 mg as loading dose for: D. Hemolytic anaemia

53. A. Amikacin 54. D. Alkylating agent 55. A. Folic acid... 56. C. Chloroquine 57. B. Inhibition of...
Answer
58. B. Adriamycin 59. B. Vincristine 60. C. It has increased..61. D. Protease ... 62. D. Acts....
63. ...................... 64. D. Pyrazinamide 65. B. Tetracycline 66. A. Arthropathy

598
Questions fr om Bihar
from Bihar,, Jharkhand, K ar
Kar natak
arnatak a, Kolk
nataka, ata MP
Kolkata MP,, UP Entrance Examinations (2000-2006)

67. Ampicillin causes increase in rash of: 74. Which of the following drug causes skin
A. Erythema toxicam (Ref.KDT/6th ed. /700) pigmentation
B. Infectious mononucleosis (Ref. Harrison 17th ed. / 323,433; 16/e,p 303)
C. Fifth disease A. Actinomycin-D
D. German measles B. Cyclophosphamide
C. Methotrexate
68. Glucose-6- phosphate dehydrogenase
D. Busulphan
deficiency related hemolysis is caused by:
A. Acetaminophen (Harrison/17th ed/685) 75. Drug of choice for CMV retinitis
B. Chloramphenicol (Ref. Harrison 17th ed. / 1895; 16/e,p 1051;
C. Ampicillin A. FoscarnetGoodman and Gilman 10/e, p 1157)
D. Diazepam B. Ganciclovir
C. Acyclovir
69. Clofazimine therapy causes:(KDT /6th ed./752) D. Zidovudine
A. Hemolytic anaemia
76. Following when given with oral contraceptives
B. Pigmentation of skin result in their failure(Ref.KDT/6th ed. / 316/317)
C. Agranulocytosis A. Rifampin
D. Methemoglobinemia B. Doxycycline
70. Immunomodulator leading to increase T- cell C. Ciprofloxacin
D. Streptomycin
mediated immunity: (KDT/6th ed./812)
A. Levamisole 77. Pulmonary fibrosis is a side effect of all except
B. Interferon
C. Chloroquine (Ref. KDT 6/P-826;Harrison 17th ed. / 434)
D. Interleukins A. Bleomycin
B. Methotrexate
71. Fluoroquinolones are contraindicated in C. Daunorubicin
children due to: (Ref. KDT/6th ed. /689) D. Cyclophosphamide
A. Nephropathy
B. Hepatotoxicity 78. Major toxicity of rifampicin in children is
C. Arthropathy A. Neurotoxicity (Ref.KDT/6th ed. / 700)
D. Cardiotoxicity B. Dermatotoxicity
C. Nephrotoxicity
72. Oral systemic antifungal agent are all except:
D. Hepatotoxicity
A. Fluconazole (Ref. KDT/6th ed. /762) 79. Ist line antitubercular drugs are all except
B. Miconazole A. INH (Ref. KDT/6th ed. /-739)
C. Itraconazole B. Rifampicin
D. Amphotericin B
C. Cycloserine
73. Which antitubercular drug is contraindicated: D. Ethambutol

A. Rifampicin in renal failure 80. Drug which acts by inhibiting protein synthesis
(Ref. KDT/6th ed. /748) A. Cephalosporin (Ref. KDT/6th ed. /-727)
B. Ethambutol in hepatotoxicity B. Macrolide
C. Pyrazinamide in mild renal failure C. Bacitracin
D. Streptomycin in pregnancy D. Polymyxin

67. B. Infectious... 68. B. Chloramphenicol 69. B. Pigmentation... 70. A. Levamisole 71. C. Arthropathy
Answer
72. B. Miconazole 73. D. Streptomycin... 74. D. Busulphan 75. B. Ganciclovir 76. A. Rifampin
77. C. Daunorubicin 78. D. Hepatotoxicity 79. C. Cycloserine 80. B. Macrolide

599
Self Assessment & Review Pharmacology
81. Drug causing nephritic syndrome D. Trichuris trichura
A. Penicillamine (Ref. Harrison 17th ed. /-224) 88. Drug not useful in cholroquine resistant malaria
B. Carbenicillin A. Quinine (Ref. KDT/6th ed. /-791)
C. Penicillin B. Mefoquine
D. Polymyxin-B C. Artesunate
82. Which of the following do not interact with D. Primaquine
pyridoxine 89. Drug useful for Cisplatin induced vomiting
A. Cycloserine (Ref. KDT/6th ed. /-741) A. Octreotide (Ref. KDT/6th ed. / 827, 828)
B. Rifampicin B. Domperidone
C. Isoniazid C. Ondansetron
D. Levodopa D. Metoclopramide

83. Which of the following antitubercular drug 90. Synergistic action is seen with all except
colors saliva, urine and body secretions A. Rifampicin + Isoniazid(Ref. KDT/6th ed. / 702)
A. Lithium (Ref. KDT/6th ed. / 742) B. Penicillin + Gentamicin
C. Penicillin + Streptomycin
B. Warfarin
D. Carbenicillin + Kanamycin
C. Cycloserine
D. Rifampicin 91. Drug, which attaches to motilin receptors
(Ref. KDT/6th ed. / 727, 669, 663)
84. Following are all indications of 5-FU except A. Erythromycin
(Ref. KDT/6th ed. / -824, Harrison 17/e, B. Norfloxacin
A. Carcinoma liver p 724; 16/e,p 652) C. Doxycyline
B. Carcinoma stomach D. Penicillin
C. Carcinoma breast
D. Mycosis fungoides 92. Lupus erythematous is most commonly caused by
(Ref. Harrison 17th ed. / 1926; 16/e,p 1967, KDT/
85. Least nephrotoxic among the following 6th ed. / 512)
A. Amikacin (Ref. KDT/6th ed. /-722) A. Hydralazine
B. Gentamicin B. Methyldopa
C. Kanamycin C. Isoniazid
D. Streptomycin D. Procainamide
86. All are effective against pseudomonas 93. All inhibit cell wall synthesis except
aeruginosa except A. Bacitracin (Ref. KDT/6th ed. / 711)
(Ref. Harrison 17th ed. / 967; 16/e,p 894)
B. Penicillin
A. Ticarcillin
C. Erythromycin
B. Azithromycin
D. Cephalosporins
C. Imipenem
D. Cefepime 94. Drugs which has both intra-and extra-intestinal
87. Mebendazole is effective against the following antiamoebic actions
infestations except(Ref. KDT/6th ed. / 809,810) A. Chloroquine (Ref. KDT/6th ed. / 798, 799)
A. Strongyloides stercoralis B. Metronidazole
B. Roundworm C. Diloxanote furoate
C. Hookworm D. None of the above

81. A. Penicillamine 82. B. Rifampicin 83. D. Rifampicin 84. D. Mycosis 85. D. Streptomycin
Answer
86. B. Azithromycin 87. .................... 88. D. Primaquine 89. C. Ondansetron 90. D. Carbenicillin...
91. A. Erythromycin 92. D. Procainamide 93. C. Erythromycin 94. B. Metronidazole

600
Questions fr om Bihar
from Bihar,, Jharkhand, K ar
Kar natak
arnatak a, Kolk
nataka, ata MP
Kolkata MP,, UP Entrance Examinations (2000-2006)

95. DOC for kala azar is 102. Drug of choice for methicillin resistant Staph.
(Ref. Goodman and Gilman’s 10/e,p 1062-1068) aureus is (Ref. Goodman & Gilman 10/e,p 732
A. Albendazole (table-43-1)
B. Metronidazole A. Nystatin
C. Diethyl carbazine B. Ciprofloxacin
D. Pentamidine C. Carbenecillin
D. Vancomycin
96. In Ethambutol toxicity, defect is seen in
A. Red vision (Ref. Harrison 17th ed. / 103. A folic acid antagonist useful as an
B. Blue vision 1020; 16/e,p 949) antineoplastic and imunosupressant agent
C. Green vision A. Methotrexate (Ref. KDT/6th ed. / 823)
D. Yellow vision B. Cyclosporine
C. Azathioprine
97. Probenecid interacts with (Ref. KDT/6th ed. /
D. Doxorubicin
700)
A. Erythromycin 104. Topically used antifungal agent is
B. Ampicillin A. Clotrimazole (Ref. KDT/6th ed. / 762)
C. Vancomycin B. Griseofulvin
D. Streptomycin C. Amphotericin B
D. Ketoconazole
98. Hemorrhagic cystitis is caused by
A. Methotrexate (Ref. KDT/6th ed. / 822; 105. Vancomycin acts by (Ref. Already Explained)
B. Cisplatin Katzung 8/e,p 609) A. Inhibition of cell wall synthesis
C. Cyclophosphamide B. Inhibition of protein synthesis
D. Adriamycin C. Leakage from cell membrane
D. Inhibition of DNA gyrase
99. Tissue schizontocide which prevents relapse
of vivax malaria (Ref. KDT/6th ed. / 791, 780) 106. Drug causing megaloblastic anemia is
A. Primaquine A. Pyrimethamine
B. Chloroquine B. Methyldopa (Ref. KDT/6th ed. / 746;
C. Quinine C. INH Harrison 17th ed. / 676; 16/e,p 602)
D. Pyrimethamine D. Chloramphenicol

100. Carbenicillin (Ref. KDT/6th ed. / 702) 107. Drug of choice for chloroquine resistant
A. Is administered orally falciparum malaria is
B. Is effective in pseudomonas infection A. Primaquine (Ref. KDT/6th ed. / 788,
C. Has no effect in proteus infection B. Chloroquine Already Explained)
D. Is a macrolide antibiotic C. Quinine
D. Pyrimethamine
101. Beta lactamase inhibitor is (Ref. KDT/6th ed. /
699) 108. INH can be used in presence of
A. Clavulanic acid A. Psychosis (Ref. KDT/6th ed. / 740)
B. Carbenicillin B. Jaundice
C. Idoxuridine C. CRF
D. Piperacillin D. Epilepsy

Answer 95. D. Pentamidine 96. C. Green vision 97. B. Ampicillin 98. C. Cyclophospham...99. A. Primaquine
100. C. Is effective... 101. A. Calvulanic... 102. D. Vancomycin 103. A. Methotrexate 104. A. Clotrimazole
105. A. Inhibition of... 106. A.Pyrimethamine 107. C. Quinine 108. A. Psychosis

601
Self Assessment & Review Pharmacology

109. Which of the following drugs can cause renal C. Bleomycin


toxicity D. None
A. Nortriptyline (Ref. KDT/6th ed. / 720, 721; 4. Scleromatous skin changes are seen in all
B. Aminglycoside Harrison 17th ed. / 436) except:
C. Lignocaine A. Adriamycin (Ref. KDT /6/e, p 720)
D. Rifampicin B. Bleomycin
C. Steroid
110. Which one of the following alkaloids is used
D. Busulphan
as anti-cancer agent (Ref. KDT/6th ed. / 820)
A. Vincristine 5. Cancer can be prevented by ingestion of:
B. Atropine A. Beta carotene (Ref. KDT/6th ed./880)
C. Ephedrine B. Vitamin C and E
D. Papaverine C. Selenium
D. All the above
111. Diethyl carbamazine is usually effective in
(Ref. KDT/6th ed. / 813, Harrison 17th ed. / 1196; 6. All the following drugs tend to produce
Goodman and Gilman 10/e,p 1062-68) myotonia except:
A. Kala-azar A. Chloroquine (Ref. HPIM-2538)
B. Taenia saginata B. Clofibrate
C. Filariasis C. Cyclosporine
D. Neurocysticercosis D. Propranolol
112. Which of the following immunosuppressive 7. An example of live attenuated vaccine is:
agents acts selectively by inhibiting helper T-cells A. Triple antigen (DPT) (Ref. KDT/6th ed. /880)
A. Methotrexate (Ref. KDT/6th ed. / 786) B. B.C.G.
B. Fluorouracil 5-FU C. Salk vaccine
C. Cyclosporine D. ATS
D. Doxorubicin
8. Pulmonary fibrosis is an important
Miscelleneous complication of:
A. Melphalan (Ref. KDT/6th ed. / HPIM-p-822)
1. Macrocytic anemia is noted with all except :
B. Busulfan
A. Phenytoin (Ref. KDT/6th ed. /689)
C. Hydroxyurea
B. Methotrexate
D. Cyclophosphamide
C. Pyrimethamine
D. Ciprofloxacin 9. In breast feeding, all are contraindicated
except: (Ref.KDT/6th ed. /909-910)
2. Pulmonary fibrosis is noted with all except :
A. Chloramphenicol
A. Busulfan
B. Phenytoin
(Ref. KDT 6/e, p 661;Harrison 16/e, 601)
C. Cyclophosphamide
B. Bleomycin
C. Nitrofurantion D. Propylthiouracil
D. Bumetenide 10. Most important toxicity of cisplatin:
3. Pulmonary fibrosis occurs most commonly by: A. Nephrotoxicity (Ref. KDT/6th ed. /777, 828)
A. Clindamycin (Ref. KDT/6th ed. /516) B. Ototoxicity
C. Pulmonary fibrosis
B. Amiodarone
D. Neurotoxicity
109. B. Aminglycoside 110. A. Vincristine 111. C. Filariasis 112. C. Cyclosporine 1. D. Ciprofloxacine
Answer
2. D. Bumetenide 3. B. Amidarone 4. C. Steroid 5. D. All the... 6. A. Chloroquine
7. B. B.C.G. 8. Busulfan 9. D. Propylthiouracil 10. A. Nephrotoxicity

602
Questions fr om Bihar
from Bihar,, Jharkhand, K ar
Kar natak
arnatak a, Kolk
nataka, ata MP
Kolkata MP,, UP Entrance Examinations (2000-2006)

11. Most common cause of lung fibrosis is:


A. Busulfan (Ref. KDT/6th ed. /826-827)
B. Bleomycin
C. Methotrexate
D. Phenytoin
12. All are used in the management of
hypercalcemia except:
(Ref. Harrison 17th ed. / 2220; 16/e,p 2261)
A. D-penicillamine
B. Mithramycin
C. Biphosphonates
D. Steroids
13. Constipation is seen as adverse effect of
following Ca++ channel blocker
(Ref. KDT/6th ed. /-530)
A. Amplodipine
B. Felodipine
C. Verapamil
D. Nifedipine
14. All are dietary fibres except
(Ref. KDT/6th ed. / 873, 874; Goodman
A. Cellulose & Gilman’s 10/e,p1044)
B. Riboflavin
C. Pectin
D. Hemicellulose
15. Which of the following is correctly matched
pair of heavy metals and their chelating agent
A. Arsenic-desferroxamine
B. Iron-BAL (Ref. KDT/6th ed. / 867)
C. Mercury-calcium disodium edentate
D. Copper-D-penicillamine

11. B. Bleomycin 12. A. D-penicillamine 13. C. Verapamil 14. B. Riboflavin 15. D. Copper-D-......
Answer

603

Das könnte Ihnen auch gefallen